Download as pdf or txt
Download as pdf or txt
You are on page 1of 1581

Exclusively on New Pattern – Data Interpretation eBook

Exclusively on New Pattern – Data Interpretation eBook

S.No Topics Page No


1 Missing Data Interpretation 2
2 Income Expenditure DI 10
3 Pipes and Cistern DI 18
4 Time and Work DI 21
5 Train DI 29
6 Boats and Streams DI 30
7 Profit and Percentage DI 37
8 Boats and Streams DI 39
9 Caselet DI 42
10 Profit and Percentage DI 43
11 Percentage DI 44
12 SI & CI DI 47
13 Missing DI 48
14 Geometry DI 50
15 Income Expenditure DI 51
16 Missing DI 55
17 Pipes and Cistern DI 56
etc… 109

www.ibpsguide.com | estore.ibpsguide.com | www.sscexamguide.com


1
Exclusively on New Pattern – Data Interpretation eBook

1. Missing Data Interpretation

Set-1

Direction (1 to 5): Study the given chart carefully and answer the following questions.
Train X
Station Arrival time Departure time Distance from origin Number of passengers Fare (in Rs.)
boarding at each station
P START 5:00pm - 400 -
Q 6:30pm 6:35pm 100 100 50
R 8:50pm 9:00 pm 250 90 120
S 4:00pm 4:10 am 800 300 400
T 7:30pm 7:45 am 1050 150 500
U 10:20am Terminates 1280 - 620
Train Y
Station Arrival time Departure time Distance from origin Number of passengers Fare (in Rs.)
boarding at each station
U Starting 6:00 pm - 300 -
T 7:40 pm 7:45 pm 230 150 120
S 9:30 pm 9:35 pm 480 270 220
R 5:40 am 5:55 am 1030 50 500
Q 9:00 am 9:10 am 1180 100 570
P 12:00 noon Terminates 1280 - 620

1) The number of passengers boarding Train X at Q is what percent of the number of passengers boarding
Train Y at S?
A) 37.03% B) 67.09% C) 47.10% D) 56.36%
2) What is the difference between the speed of Train X and that of Train Y?
A) 2.73kmph B) 3.47kmph C) 8.6kmph D) 4.82kmph
3) What is the ratio of the total passengers of Train X to that of Train Y?
A) 111:80 B) 104: 87 C) 112: 57 D) 98:102
4) The total income of Train X is what percent of the total income of Train Y?
A) 150.32% B) 162.04% C) 114.21% D) 135.23%
5) If the average speed of Train X increases by 10% then when will it reach to its destination?
A) 9:45 am B) 6:45 am C) 8:45 am D) 5:45 am

www.ibpsguide.com | estore.ibpsguide.com | www.sscexamguide.com


2
Exclusively on New Pattern – Data Interpretation eBook

Solutions
1). A
Required percentage = 100/270 × 100 = 37.03%
2). A
Speed of Train X = 1280 / 5 pm – 10:20 am =
1280 / 17 hours 20 minutes
= 1280 x 3 / 52 = 73.84kmph
Speed of train Y = 1280 / 6:00 pm - 12:00 noon
= 1280/18 hours = 71.11kmph
So, difference between the speed of train A and train B = 73.84- 71.11 = 2.73kmph
3).B
Total passengers in train X = 400 + 100 + 90 + 300 + 150 = 1040
Total passengers in train Y = 300 + 150 + 270 + 50 + 100 = 870
Required ratio = 1040: 870 = 104: 87
4).C
Total income of train X = (400 × 50) + (500 × 70) + (590 × 280) + (890 × 100) + (1040 × 120) = Rs.434000
Total income of train Y = (300 × 120) + (450 × 100) + (720 × 280) + (770 × 70) + (870 × 50) = Rs.380000
Required % = 434000 x 100 / 340000
= 114.21%
5).C
If the average speed of train X increases by 10% then its new speed = 73.84 × 110/100 = 81.22kmph Time
taken by train X during the journey = 1280/81.22 = 15.75 hours = 15 hours 45 minutes
The time when the train will reach its destination = 5 pm + 15 hours 45 minutes = 8:45 am

Set-2
Direction (6 to 10) Given below in the table showing data of a metro travelling from Mumbai to Goa

Station Name Distance(Km) Time(hour) Speed(kmph)


MUMBAI-PUNE - 8 -
JAIPUR-GOA 1150 - -
DELHI-PUNE - 9 118.8
MUMBAI-DELHI 950 12 -
PUNE -GOA - - 133.3

6). What will be the speed of the train from Mumbai- Pune If it halts 20 min for every 2 hours of journey and its
distance is 450 km more than Mumbai-Delhi?
A) 155.5 km/hr B) 160 km/hr C) 170 km/hr D) 180 km/hr

www.ibpsguide.com | estore.ibpsguide.com | www.sscexamguide.com


3
Exclusively on New Pattern – Data Interpretation eBook

7). Average distance travelled by the train from all the station is 1114 then what will be total distance covered
from Delhi-Pune ,Pune-Goa and Mumbai to Pune?
A) 1920 B) 3470 C) 5670 D) 7560
8). If the speed of the train maintained from Mumbai to Delhi below the speed of train Delhi-Pune what will be
average speed of Mumbai to Delhi and Delhi to Pune?
A) 140 B) 225 C) CANNOT BE DETERMINED D) 350
9). What will be the speed of train from Jaipur-Goa ,if the time taken to travel Jaipur -Goa 5hr less than the
average of time taken from Mumbai-Delhi and Mumbai-Pune together?
A) 150 km/hr B) 175 km/hr C) 160 km/hr D) 230 km/hr
10). If the distance between Pune to Goa is 150km less than the Jaipur to Goa Then what will be the
approximate time taken to travel Pune from Goa?
A) 7.5hr B) 4hr C) 3hr D) 1hr

Solutions
6) A
Total time taken to travel form Mumbai to Pune = 8 hrs
Due to halt 20mins for every 2hrs = 9 hrs to complete the journey
Distance between Mumbai to Pune= 950 + 450 = 1400
Speed = 1400 / 9
= 155.5 km/hr
7) B
Total distance travelled by train from all the station = 1114 * 5 =5570
Remaining Distance = 5570- 2100=3470
8) C
Speed of the train is not mentioned from Mumbai to Delhi.
So we cannot find average speed.
9) D
Average time taken=12+8 =20/2=10
Time taken to travel Jaipur to Goa=10 – 5 =5
Speed = 1150/5 = 230 km/hr
10) A
Distance between Jaipur to Goa = 1150
Distance between Pune to Goa = 1150-150 =1000
Time taken = 1000/133.3 =7.5 hr

www.ibpsguide.com | estore.ibpsguide.com | www.sscexamguide.com


4
Exclusively on New Pattern – Data Interpretation eBook

Set-3
Direction (11 to 15) : Study the following pie-chart and table carefully and answer the questions that follow.

A, 450

E, 930 B, X0

C, 570

D, Y0

PERSON SPEED TIME


A 60Km/hr 300min
B - -
C - 4hr
D - -
E 50Km/hr -

11). If the time taken by B is 3 hours more than that of A to cover the given distance and the ratio of the
distance between B and D is 7:4, find the A’s Speed.
A) 87.5 B)25.3 C) 20.3 D) 51
12). With the help of question 1 Find the average speed of B and A?
A) 80 B)76.92 C) 35 D) 45
13). E is going to Delhi with a speed of x km/h. Delhi is 384 km far from his house. If the time is taken by him
as same as to travel 324 km with the speed of 15m/s. Find the value of x.
A) 18 B) 25 C) 64 D) 80
14). If the difference between the speed of C and D is 55 km/h. What will be time taken by D to cover the
distance of 250 km.
A) 8hr B) 3hr C) 2hr D) 6.25hr
15). Find the ratio of the speed between C and E?
A) 19:10 B) 17:15 C) 20:22 D) 15:18

www.ibpsguide.com | estore.ibpsguide.com | www.sscexamguide.com


5
Exclusively on New Pattern – Data Interpretation eBook

Solutions
11) A
Distance travelled by A=2400*45/360 = 300Km
Distance travelled by C = 2400*57/360 = 380Km
Distance travelled by E = 2400*93/360 = 620
Distance travelled by B and D = 2400-(300+380+620)
= 1100Km
Distance travelled by B = 1100*7/11 = 700Km
Distance travelled by D = 1100*4/11 = 400Km
Time taken by B = 300/60+3 = 8hr
Speed of B = 700/8 =87.5Km/hr
12) B
Average Speed = Total Distance/Total Time
= 1000/13 = 76.92Km/hr
13) C
Time is taken by E to cover the distance of 384 km with the speed of x = Time taken by E to cover the
distance of 324 km with the speed of 15 m/s.
384/x = 324*5/(15*18)
X = 64Km/hr
14) D
C’s Speed = 2400*57/(360*4)
= 95Km/hr
Speed of D = 95-55 = 40 km/h
So the speed is 40 km/h
Time = 250/40
=6.25hr
15) A
C’s Speed = 2400*57/(360*4)
= 95Km/hr
E’s Speed = 50Km/hr
Ratio= 95/50=19:10

www.ibpsguide.com | estore.ibpsguide.com | www.sscexamguide.com


6
Exclusively on New Pattern – Data Interpretation eBook

Set-4
Directions (16-20): Study the following table &answer the following questions.

Respective Ratio of Valid


Percentage of Valid
Month Total Number of Votes Votes P & Valid Votes of
Votes
Q
JAN - - 5:3
FEB 500 - 5:4
MAR 1000 38% -
APR - 60% 7:5
MAY 2500 40% -

16). In MAR, if the number of valid votes of Q was 100, what was the respective ratio of number of valid votes
of P and number of valid votes of Q same month?
A) 14:5 B) 12:8 C) 19:6 D) 21:4
17). The total number of votes increased by 50% from FEB to JUN. If 16% of the votes valid in JUN, what was
the number of valid votes in JUN?
A) 115 B) 120 C) 220 D) 310
18). If the average number of valid votes in FEB and MAY was 635, approximately, what percent of total votes
valid in FEB?
A) 25 B) 60 C) 54 D) 80
19). In APR, if the difference between number of valid votes of P and number of valid votes of Q was 150,
what was the total number of votes in APR?
A) 1150 B) 1200 C) 2200 D) 1500
20). In JAN, the respective ratio of total number of votes to valid votes was 5:4. Number of valid votes of Q in
JAN constitutes what percent of the total number of votes in the same month?
A) 30 B) 20 C) 50 D) 80

Solutions
16). A
38% of 1000 = 380
Valid votes of Q = 100
valid votes of P = 380 – 100 = 280
= 280:100
= 14:5

www.ibpsguide.com | estore.ibpsguide.com | www.sscexamguide.com


7
Exclusively on New Pattern – Data Interpretation eBook

17). B
50% of 500 = 250
= 500 + 250 = 750
16% of 750 = 120
18). C
Number of valid votes in FEB and MAY = 1270
MAY =40% of 2500 = 1000
1270 – 1000 = 270
(270/500)*100 = 54%
19). D
A = Valid Votes; B= Total Votes
(1/6)*A = 15
A = 900
60% of B = 900
B = 1500
20). A
[4x * (3/8) / 5] * 100 = 30%
Set-5

Directions (21-25 ): Study the following table carefully and answer the questions that follow

The percent discount of different articles by different stores are given below
Article Store A Store B Store C
I 16 7 24
II 28 18 40
III 16 - 30
IV - 18 -
V 12 11 6

21). If the average SP of article II by in all the stores is Rs 2568. Find the MP of article II.
A) Rs 3200 B) Rs 4500 C) Rs 3600 D) Rs 4300
22). Difference between SP of article I by stores A and B is Rs 486; Find the SP of same article by store C.
A) Rs 3506 B) Rs 4005 C) Rs 4808 D) Rs 4104
23). Average SP of article III by stores A and B is Rs 3608, by stores B and C is Rs
3300. Find the SP of article III by store C.
A) Rs 2984 B) Rs 3122 C) Rs 3080 D) Rs 2764

www.ibpsguide.com | estore.ibpsguide.com | www.sscexamguide.com


8
Exclusively on New Pattern – Data Interpretation eBook

24). Store A earned 10% profit by selling product V. If CP of articles at all articles is same, find the ratio of
profits by B and C in selling V.
A) 6: 15 B) 9: 14 C) 8: 15 D) 6: 13
25). Ratio of discounts on article IV by stores A and B is 2/3. Difference in SP of article IV by stores A and C
is Rs 432. If SP of article IV by store A is Rs 216 more than that by store B, find the SP of article IV by store
C?
A) Rs 2138 B) Rs 2687 C) Rs 2736 D) Rs 2522

Solutions
21) C
SP by store A = (100-28)/100 * MP = 72% of MP, by B = 82% of MP, by C =
60% of MP
(72+82+60)/3 * MP/100 = 2568
MP = Rs 3600
22) D
Difference in SP = 93% of MP – 84% of MP
9% of MP = 486; MP = 5400
So SP by store C = 76% of 5400 = Rs 4104
23) C
Let x% discount by store B
So [84 + (100-x)]/(2*100) * MP = 3608
And [70 + (100-x)]/(2*100) * MP = 3300
Put value of (100-x) from 1 equation to another and solve for MP
MP = Rs 4400
So SP by store C = 70/100 * 4400 = Rs 3080
24) B
MP = x
SP by A = 88% of x
CP by A = 100/110 * 88/100 *x = 4x/5
SP by B = 89x/100, so profit of B = 89x/100 – 4x/5 = 9x/100
SP by C = 94x/100, so profit of B = 94x/100 – 4x/5 = 14x/100
Required ratio = 9: 14
25) C
X/18 = 2/3
So discount by A = 12%
88% of MP = 82% of MP + 216
MP = Rs 3600

www.ibpsguide.com | estore.ibpsguide.com | www.sscexamguide.com


9
Exclusively on New Pattern – Data Interpretation eBook

Let y% discount by store C


[88 – (100-Y)]/100 * MP = 432
MP = Rs 3600
Y = 24%
So SP by C = 76% of 3600 = Rs 2736
Set-6
Directions (26-30): The table below shows the estimated cost (in Rs. Lakh) of a project of laying a railway line
between two places.

1988 1989 1990 1991


Surveying 41.5 7.5 2.2 0.5
Cement - 95 80 75
Steel - 70 45 60
Bricks - 15 12 16
Other Building Material - 25 18 21
Labour 2.1 25 20 18
Administration 7.5 15 15 14
Contingencies 1 15 4.2 5
Total 52.1 267.5 196.4 209.5

26). The total expenditure is required to be kept within Rs. 700lakh by cutting the expenditure on
administration equally in all the years. What will be the Percentage cut for 1989?
A) 22.6 B) 32.6 C) 42.5 D) 52.6
27) If the length of line to be laid each year is in proportion to the estimated cost for material and labour, what
fraction of the total length is proposed to be completed by the third year?
A) 0.9 B) 0.7 C) 0.6 D) 0.3
28) What is the approximate ratio of the total cost of materials for all the years bear to the total labour cost?
A) 4: 1 B) 8: 1 C) 12:1 D) 16: 1
29) If the cost of materials rises by 5% each year from 1990 onwards, by how much will the estimated cost
rise?
A) RS. 11.4LAKH B) RS. 16.4LAKH C) RS.21.4LAKH D) RS.26.4LAKH
30) It is found at the end of 1991, that the entire amount estimated for the project has been spent. If for 1991,
the actual amount spent was equal to that which was estimated, by what percent (approximately) has the
actual expenditure exceeded the estimated expenditure?
A) 39 B) 29 C) 19 D) 9

www.ibpsguide.com | estore.ibpsguide.com | www.sscexamguide.com


10
Exclusively on New Pattern – Data Interpretation eBook

Solutions
26) C
Total expenditure = 52.1+267.5+196.4+209.5 = 725.5lakhs.
The expenditure reduced each year will be (25.5/4) = 6.375lakhs.
Percentage reduce for 1989 would be = (6.375/15) × 100 = 42.5%.
27) B
Costs of material and labour
1988 = 2.1
1989 = 95+70+15+25+25 = 230
1990 = 80+45+12+18+20 = 175
1991 = 75+60+16+21+18 = 190
= (2.1 + 230 + 175) / (2.1 + 230 +175 + 190) = 0.6817.
28) B
Total material cost = (95+80+75+70+45+60+15+12+16+25+18+21) = 532
Total labour cost = (2.1+25+20+18) = 65.1
The ratio = 532:65.1 = 8:1
29) B
The estimated cost in 1990 = 80 + 45 + 12 + 18 = 155
The estimated cost in 1991 = 75 + 60 + 16 + 21 = 172
Cost of material rises by 5%,
Cost rise by 0.05X (155 + 172) = Rs.16.35lakhs.
30) B
Amount spent till 1991 = Rs.725.5lakhs
Estimated Expenditure for 1991 = 209.5lakhs.
The increase in expenditure will be 209.5 on 725.5 = 28.87%.

2. Income Expenditure

Set-7
Directions(31-35): Following line graph shows ratio of income to expenditure of three cars over the month JAN
– JUN.

www.ibpsguide.com | estore.ibpsguide.com | www.sscexamguide.com


11
Exclusively on New Pattern – Data Interpretation eBook

1.4

1.2 1.2

1 1
0.9
0.8 0.8 0.8
0.75 0.75 Maruti
0.7
0.6 0.6 0.6 0.6 Suzuki
0.5 0.5 0.5 Hyundai
0.4 0.4 0.4
0.3
0.25
0.2

0
JAN FEB MAR APR MAY JUN

31). If income of Maruti in the month JAN. Was 23.58 what was expenditure in that month?
A) 60.63 Lakh B) 41.25 Lakh C) 78.6 Lakh D) 75.90 Lakh
32). The ratio of income to expenditure of Maruti in the month JUN is approximatly what percent of the ratio of
income to expeniture of Hyundai in the month MAY?
A) 56.3 B) 65 C) 62.5 D) 70
33). If the expenditure of Maruti in the month JUN and the income of Hyundai in the month MAR equal to 64
lakh each then the income of Maruti in the month JUN was approximatly what percent of the expenditure of
Hyndai in the month MAR?
A) 80 B)90 C) 56 D) 60
34). If the income of Maruti and Suzuki in the month MAR were 36 akh and 27 lakh respectively what was the
ratio of expenditure in that month?
A) 1:5 B) 6:9 C) 2:3 D) 5:4
35). If the income of Hyundai in the month FEB and JUN Were equal then the then the expenditure of
Hyundai in the month FEB was approximately what percent of its expenditure in the month JUN?
A ) 120 B) 10 C) 50 D) 160

Solutions
Some Notation
I - Income
E - Expenditure
31). C
Imaruti/Emaruti =0.3
Emaruti= 23.58/0.3= 78.6 lakh
32). C

www.ibpsguide.com | estore.ibpsguide.com | www.sscexamguide.com


12
Exclusively on New Pattern – Data Interpretation eBook

Imaruti(JUN)/Emaruti(JUN) = 0.75
Ihyundaii(MAY)/Ehyundai(MAY) = 1.2
Required Percent = 0.75/1.2*100 =62.5
33). D
Imaruti(JUN)/Emaruti(JUN)=0.75
Imaruti(JUN) = 0.75*64 = 48 lakh
Ihyundai(MAR)/Ehyundai(MAR) = 0.8
Ehyundai = 64/0.8 = 80 lakh
Requared paercent = 48/80*100 = 60
34). C
Imaruti(MAR)/Emaruti(Mar) = 0.5
Emaruti(JUN) = 36/0.5 = 72lakh
Isuzuki(MAR)/Esuzuki(MAR) = 0.25
Esuzuki(MAR) = 27/0.25 = 108 lakh
Ratio = 72/108 = 2/3
= 2:3
35). A
Let the income of hyundai in FEB = X
E(FEB) = X/0.5 = 2X
E(JUN) = X/0.6 = 5X/3
Required percent = 2X/5X/3*100 = 120
Set-8
Direction (36 to 40) The following pie-chart shows the percentage distribution of the expenditure incurred
in publishing a book. Study the pie-chart and the answer the questions based on it.

10% Printing cost


20%
15% Transportation cost
Paper cost
10%
Binding

25% Royalty
25% Promotion cost

36) If for a certain quantity of books, the publisher has to pay Rs. 30,600 as pri nting cost, then what will be
amount of royalty to be paid for these books?
A) Rs. 18,450 B) Rs. 25,200 C) Rs. 22,950 D) Rs. 36,650

www.ibpsguide.com | estore.ibpsguide.com | www.sscexamguide.com


13
Exclusively on New Pattern – Data Interpretation eBook

37) What is the central angle of the sector corresponding to the expenditure incurred on Royalty?
A) 30 B) 20 C) 50 D) 54
38) The price of the book is marked 20% above the C.P. If the marked price of the book is Rs. 180, then
what is the cost of the paper used in a single copy of the book?
A) 16.50 B) 37.50 C) 39.99 D) 15.55
39) If 5500 copies are published and the transportation cost on them amounts to Rs. 82500, then what
should be the selling price of one book so that the publisher can earn a profit of 25%?
A) 160 B) 187.50 C) 220 D) 335
40) Royalty on the book is less than the printing cost by what percentage?
A) 25 B) 50 C) 20 D) 33.33

Solutions

36). C
Amount of Royalty to be paid for these books be Rs. x
Then, 20/15 = 30600/ x
x = Rs. 30600*15/20
= Rs. 22,950

37). D
Central angle corresponding to Royalty = 15% of 360
=54

38). B
Marked price of the book = 120% of C.P.
Also, cost of paper = 25% of C.P
Let the cost of paper for a single book be Rs. x
120/ 25 = 180/ n
n=180*25/120
n=37.50

39). B

www.ibpsguide.com | estore.ibpsguide.com | www.sscexamguide.com


14
Exclusively on New Pattern – Data Interpretation eBook

For the publisher to earn a profit of 25%, S.P. = 125% of C.P.


Also Transportation Cost = 10% of C.P.
Let the S.P. of 5500 books be Rs. x.
Then,
10 / 125 = 82500/ x
x= 82500*125/10
x = 1031250
S.P. of one book = 1031250/5500
= 187.50

40). A
Printing Cost of book = 20% of C.P.
Royalty on book = 15% of C.P.
Difference = (20% of C.P.) - (15% of C.P) = 5% of C.P.
Percentage difference = 5% of CP/Printing Cost* 100
= 25%

Set-9

Direction (41 to 45): Study the following charts and answer the following questions.

250
220
200
200 185
165
145 150
150 135 140
130 125
120 Profit
110 115
102 Revenue
100
Expenditure
70
60
50
50 40
25 30
20

0
1989 1990 1991 1992 1993 1994 1995

41) The average revenue collected in the given 7 years is approximately


A) Rs. 164lakh B) Rs. 167lakh C) Rs. 171lakh D) Rs. 175lakh

www.ibpsguide.com | estore.ibpsguide.com | www.sscexamguide.com


15
Exclusively on New Pattern – Data Interpretation eBook

42) The expenditure for the 7 years together forms what per cent of the revenues during the same
period(approx)?
A) 75% B) 67% C) 62% D) 83%

43) Which year showed the greatest percentage increase in profit as compared to the previous year?
A) 1993 B) 1994 C) 1990 D) 1992

44) In which year was the growth in expenditure maximum as compared to the previous year?
A) 1993 B) 1995 C) 1991 D) 1992

45) If the profit in 1996 shows the annual rate of growth as it had shown in 1995
Over the previous year, then what approximately will be the profit in 1996?
A) Rs. 72lakh B) Rs. 82lakh C) Rs. 93lakh D) Rs. 78lakh

Solutions
41) B
Average = (120 +130 +145+ 165 +185 +200 +220)/7
= 166.42
= Rs. 167lakh.

42) A
Required answer = 877/1165
=75.5%

43) D
To find the answer we have to find the profit in each year.
In 1990=5/20*100=25%
In 1991=5/25*100=20%
In 1992=10/30*100=33.33%
In 1993=10/40*100=25%
In 1994=10/50*100=20%
In 1995=10/60*100=16.66%
From the above table,
In 1992 the profit is maximum.

44) D

www.ibpsguide.com | estore.ibpsguide.com | www.sscexamguide.com


16
Exclusively on New Pattern – Data Interpretation eBook

To find the answer we have to find the profit in each year.


In 1990=8/102*100=7.8%
In 1991=5/110*100=4.5%
In 1992=10/115*100=8.7%
In 1993=10/125*100=8%
In 1994=5/135*100=3.7%
In 1995=10/140*100=7.14%
From the above table,
In 1992 the expenditure is maximum.

45) B
Profit in 1994 = 60.
Profit in 1995 = 70.
Growth percentage in profit in 1995 over 1994 = (10/60) x 100 = 16.66%
Profit in 1996 will be (16.66% of 70) + 70 = Rs. 82lakh.

Set-10
Directions (46-50): Study the graph below and answer the questions that follow.

140
128
120
106 114
100 92
100
80
80
88
76
60

40

20 22
8 22
8
0
1990 1991 1992 1993

Sales Expenditure Equity

46). In which year is the profit per rupee of equity the highest?
A.1991 B.1992 C. 1993 D. 1990

www.ibpsguide.com | estore.ibpsguide.com | www.sscexamguide.com


17
Exclusively on New Pattern – Data Interpretation eBook

47). The simple annual growth rate in sales was the highest between the years?
A. 1990 – 91 B. 1991 – 92 C. 1992 – 93 D. 1990 – 92
48). In which year are the sales per rupee of expenditure the lowest?
A. 1990 B. 1991 C. 1992 D. 1993
49). In which year is a sale per rupee of equity the highest?
A. 1990 B. 1991 C. 1992 D. 1994
50) In which year is a sale per rupee of equity the lowest?
A. 1990 B. 1991 C. 1992 D. 1994

Solutions
Company Sales EXPEND Profit Equity Sales/Equity Profit/Equity Sales/Expend Growth
(A) (B) C=(A- (D) (A/D) (C/D) (A/B) Rate
B) Sales
1990 80 76 4 8 10 0.5 1.05 Nil
1991 92 88 4 8 11.5 0.5 1.04 15%
1992 106 100 6 22 4.81 0.27 1.06 15.21%

1993 128 114 14 22 5.81 0.63 1.12 20.75%

46) C
From the table It is clear that the profit per rupee of equity is highest for 1993 i.e. 0.51
47) C
The simple annual growth rate in sales is maximum for the year 1992-93
i.e. = (128 – 106) / 106 * 100 = 20.75%
48) B
Sales per rupee of the expenditure are lowest for the year 1991 i.e. 1.04.
49) B
Sales per rupee of equity is highest for 1991 i.e. 11.5
50) C
Sale per rupee of equity the lowest for 1992 i.e. 4.81

3. Pipes and Cistern

Set-11
Direction (51 to 55): Study the following graph carefully to answer the given questions
Time taken by the pipes to fill a tank/cistern (hours/minutes)

www.ibpsguide.com | estore.ibpsguide.com | www.sscexamguide.com


18
Exclusively on New Pattern – Data Interpretation eBook

35
30
30
27

25 24

20
20 18
15 15 15 INLET X
15 INLET Y
12 12 12
10 10
10

5 4

0
A,B C,D E,F G,H I,J K,L M,N

51). A large cistern can be filled by two pipes A and B. How many minutes will it take to fill the Cistern from an
empty state if B is used for half the time and A and B fill it together for the other half?
A) 7.5 minutes B) 2.5 minutes C) 3.5 minutes D) 8.5 minutes
52). Two pipes E and F can fill a tank. If both the pipes are opened simultaneously, after how much time
should F be closed so that the tank is full in 8 minutes?
A) 20 minutes B) 18 minutes C) 16 minutes D) 30 minutes
53). Three pipes I, J, and R can fill a tank. If Pipe R alone can fill a tank in 24 minutes then the pipe R is
closed 12 minutes before the tank is filled. In what time the tank is full?
A) 9 5/13 B) 10 4/13 C) 8 4/13 D) 7 6/13
54). Two pipes G and H can fill a cistern. If they are opened on alternate minutes and if pipe G is opened first,
in how many minutes will the tank be full?
A) 5 minutes B) 3 minutes C) 8 minutes D) 6 minutes
55). Two pipes, C and D are opened simultaneously and it is found that due to the leakage in the bottom, 17/7
minutes are taken extra to fill the tank. If the tank is full, in what approximate time would the leak empty it?
A) 28 minutes B) 23 minutes C) 39 minutes D) 69 minute

Solutions
51). A
Part filled by A and B = 1/15 + 1/10 = 1/6
Part filled by B = 1/10
x/2(1/6 + 1/10) = 2/15 = 15/2 = 7.5 minutes
52). B
Required time = y (1-(t/x)) = 27(1-(8/24)) = 18 minutes

www.ibpsguide.com | estore.ibpsguide.com | www.sscexamguide.com


19
Exclusively on New Pattern – Data Interpretation eBook

53). C
Let T is the time taken by the pipes to fill the tank
(1/12 + 1/18 + 1/24)*(T – 12) + (1/12 + 1/18)*12 = 1
T = 108/13 = 8 4/13
54). D
Pipe G can fill = 1/12
Pipe H can fill = 1/4
for every two minutes, 1/12 + 1/4 = 1/3 Part filled
Total = 6 minute
55). C
Total time taken by both pipes before the leak was developed = 60/7 minutes
now, leaks is developed which will take T time to empty the tank so, (1/15 +1/20 – 1/T) = 60/7 + 17/7
(1/15 +1/20 – 1/T) = 1/11
solve for T,
660/17 minutes = 39 minutes

Set-12
Direction (56 to 60): Study the following graph carefully to answer the given questions
Time taken by the pipes to fill a tank/cistern (hours/minutes)
35
30
30
27

25 24

20
20 18
15 15 15 INLET X
15 INLET Y
12 12 12
10 10
10

5 4

0
A,B C,D E,F G,H I,J K,L M,N

www.ibpsguide.com | estore.ibpsguide.com | www.sscexamguide.com


20
Exclusively on New Pattern – Data Interpretation eBook

56). A waste pipe, W can carry off 12 litre of water per minute. If all the pipes M, N and W are opened when
the tank is full and it takes one hour to empty the tank. Find the capacity of the tank.
A) 60 B) 25 C) 35 D) 85
57). Two pipes K, and L are opened and when the tank is 1/3 full a leak is developed due to which 1/3 water
supplied by the pipe leaks out. What is the total time to fill the tank?
A) 25/2 minutes B) 40/3 minutes C) 18/9 minutes D) 10/4 minutes
58). Two pipes I and J are opened simultaneously and it is found that due to leakage in the bottom of the tank
it took 48 minutes excess time to fill the cistern. When is the cistern full, in what time will the leak empty it?
A) 15hr B) 80hr C) 72hr D) 15hr
59). Three pipes N, C and D can fill a tank. If pipe N is opened all the time and pipe C and D are opened for
one hour alternatively. The tank will be full in
A) 5 hr B) 3 hr C) 8 hr D) 7 hr
60). Three pipes R, A and B can fill the tank. R can fill the tank 5 minutes less than that of B. If all the pipes
are opened together and pipe B is turned off 5 minutes before the tank is filled. Then find the time in which the
tank will full.
A) 45/11hr B) 23/15hr C) 39/12hr D) 15/18hr

Solution
56). A
Let the waste pipe take ‘X’ time to empty the tank.
(1/10 + 1/12 – 1/X)*60 = -1
we will get X = 5 min
so capacity = 5*12 = 60ltr
57). B
(1/15 + 1/30)*T1 = 1/3, T1 = 10/3 minutes
Now after leak is developed,
[(1/15 + 1/30) – (1/3)*(1/15 + 1/30)]*T2 = 2/3
T2 = 10 minutes.
So total time = 10 + 10/3 = 40/3 minutes
58). C
Work done by the two pipes in 1 hour = (1/12) + (1/18) = (15/108).
Time taken by these pipes to fill the tank = (108/15) hrs = 7 hours 12 min.
Due to leakage, time taken = 7 hours 12 min + 48 min = 8 hours
Work done by two pipes and leak in 1 hour = 1/8.
Work done by the leak in 1 hour = (15/108)-(1/8) = (1/72).
Leak will empty the full cistern in 72 hours.
59). D

www.ibpsguide.com | estore.ibpsguide.com | www.sscexamguide.com


21
Exclusively on New Pattern – Data Interpretation eBook

(1/12 + 1/15) + (1/12 + 1/20) = 17/60 (in 2 hrs this much tank is filled)
so in 6 hrs 51/60 is filled.
Remaining, 9/60 = (1/12 + 1/15)*T, so T = 1hr
so total = 6 + 1 = 7 hr
60). A
Let total time taken by the pipes is X hrs,
(1/5 + 1/10 + 1/15)*(X – 5) + (1/5 + 1/15)*5 = 45/11

4). Time and Work

Set-13
Direction (61 to 70) : Study the following table carefully to answer the given questions.
NOTE: NA-NOT NEEDED

P Q R S Ratio Total Number of Working Days


- 25 NA NA Efficiency (P : Q) = 5 : 4 NA
- - - NA NA -
16 64/5 32 NA NA NA
NA NA 50 - - -
NA 9 8 3 NA -

61). P started a work alone and then Q joined her 5 days before actual completion of the work. For how many
days P worked alone?
A) 6 B) 31` C) 11 D) 15
62). Working together Q and R take 50% more number of days than P, Q and R together take and P and Q
working together, take 8/3 more number of days than P, Q and R take together. If P, Q and R all have worked
together till the completion of the work and Q has received Rs.120 out of total earnings of Rs. 480 then in how
many days did P, Q and R together complete the whole work?
A) 7 days B) 5 days C) 6 days D) 11 days
63). All of them started to work together but P leaves after 4 days. Q leaves the job 3 days before the
completion of the work. How long would the work last?
A) 5 days B) 21 days C) 19 days D) 9 days
64). R started the work and left after some days, when 25% work was done. After it S joined and completed it
working for 25 days. In how many days R and S can do the complete work, working together?
A) 20 B) 9 C) 30 D) 12

www.ibpsguide.com | estore.ibpsguide.com | www.sscexamguide.com


22
Exclusively on New Pattern – Data Interpretation eBook

65). Q and R started the work. After 3 days S joined them. What is the total number of days in which they had
completed the work?
A) 21 B) 4 C) 8 D) 6

Solution
61). C
Efficiency (P:Q) = 5 : 4
Number of days(P:Q) = 4x : 5x = 4x : 25
Number of days required by A to finish the work alone = 4x
= 4 * 5 = 20.
P and Q work together for last 5 days = 5 * 9 = 45%
Efficiency of P = 5% and Q’s efficiency = 4%
No. of days taken by P to complete 55% work = 55/5 = 11days
62). B
The days ratio of (P+Q+R) : (Q+R) = X:3X/2 = 2X : 3X;
Efficiency ratio = 3X:2X
Efficiency of P = x.
(480/3X) = Rs.160
Amount received by Q = Rs.120 & R = 200
160:120:200 =4:3:5
1/4:1/3:1/5= 15:20:12;
(1/15+1/12+1/20)*Y = 1
Y = 5 days
63). D
Let the work lasted for x days,
P’s 4 day’s work +Q (x – 3) day’s work + R’s x day’s work = 1
(4/16) + (x – 3) / (64/5) + x/32 = 1
5(x – 3)/64 + x/32 = 1 – 1/4
[5(x – 3) + 2x] / 64 = 3/4
x = (48 + 15)/7 = 63/7 = 9 days
64). A
Efficiency of R= (100/50) = 2%
Rest work = 75%
Efficiency of S = 75/25 = 3%
Combined efficiency of R and S = 5%
Number of days required by R and S to work together = 100/5 = 20 days
65). B

www.ibpsguide.com | estore.ibpsguide.com | www.sscexamguide.com


23
Exclusively on New Pattern – Data Interpretation eBook

Efficiency of Q and R = 11.11 + 5.55 = 16.66%


Work done in 3 days = 3 x 16.66 = 50%
Rest work done by Q, R and S = 50/50 = 1 day
Work can be completed in 4 days.

Set-14
Direction (66 to 70): Refer to the following Graph and answer the given questions.

The chart below shows the number of days taken by six boys to complete a work

BOYS
16

14

12

10

8
BOYS
6

0
A B C D E F

The chart below shows the number of days taken by six girls to complete a work

GIRLS
30

25

20

15
GIRLS
10

0
P Q R S T U

www.ibpsguide.com | estore.ibpsguide.com | www.sscexamguide.com


24
Exclusively on New Pattern – Data Interpretation eBook

66). If D and F started to work and after 2 days they were replaced by Q, T and S then they stopped, how
much work is still left? Q, T and S worked for 2 days together.
A) 1/6 B) 1/10 C) 1/12 D) 1/14
67). If all the boys work together then time taken by them will be how much less than the time taken by P, R,
Q and S together to do the work?
A) 2*1/15 B) 2*1/5 C) 3*1/15 D) 6*1/10
68). E and C started work. E worked at 120% of his efficiency and C at 5/6 of his normal routine work. They
started work on alternate days starting with E. How many days will be taken by them to complete the work?
A) 15 B) 18 C) 10*5/6 D) 20*1/5
69). How many days will it take to complete the work if A does the 25% of the work alone and 10% is done by
C alone and the remaining is done by U?
A) 5 B) 10 C) 20 D) 17
70). B started working alone at his normal efficiency but after 5 days of starting he found that he has done
only 25% work so to complete the remaining work on time by how much percentage he should increase his
efficiency?
A) 12.5 B) 16.66 C) 33.33 D) 50

Solutions
66) A
Let work = 180 units
D’ 1 day’s work =20 (work/ no. of days taken)
F’s 1 day’s work =20
Q’s 1 day’s work=7.5
S’s 1 days work=7.5
T’s 1 day’s work = 20
F and D worked for 2 days=(20+20)*2 = 80(remaining work = 100)
now for 2 days ( 7.5+7.5+20)*2 = 70(remaining work = 30) so 30/180 = 1/6
67) B
Total days taken by boys to do the work = 180/100 = 9/5 days
days taken by 4 girls = 180/45 = 4 days
Difference = 4-9/5 = 11/5 days
68) C
E’s one day’s work -180/12 = 15 units
C one day’s work -180/10 = 18 units
now, E works at 120% of 15 = 18 units
C at 5/6 of 18 = 15 units

www.ibpsguide.com | estore.ibpsguide.com | www.sscexamguide.com


25
Exclusively on New Pattern – Data Interpretation eBook

E + C’s 2 days’ work = 33


In 10 days -33*5 = 165 units
Remaining =15 units
E does 18 days a day so to do 15 units He will take 5/6 days
Total time taken = 10* 5/6 days
69) D
A’s 1 day’s work =15 units so to do 25% work (assumed total work = 180) = 45
He will take 3 days
to do 10% work= 18 units C will take 1 days
Remaining work = 180-63 = 117
to do 117, U will take 117/9 = 13 days (U’s 1 day’s work = 180/20 = 9 units)
Total days = 3+1+13 = 17 days
70) A
B’s 1 day’s work =180/15 = 12 units
in 5 days he should have done = 60 units
but he did 180/4 = 45 units
so remaining work = 135 units
Complete it in 10 days he should do 135/10 = 13.5 units per day
therefore increased efficiency =13.5 – 12 = 1.5 = 1.5*100/12 = 12.5%

Set-15
Directions (71-75): Study the following charts and answer the questions that follow.

www.ibpsguide.com | estore.ibpsguide.com | www.sscexamguide.com


26
Exclusively on New Pattern – Data Interpretation eBook

PERCENTAGE OF WORK DONE BY


5 PEOPLE TO COMLETE THE
PROJECT

T P
15% 20%
Q
10%
S
30%
R
25%

PERSON NO. OF DAYS THEY WORKED


P 4
Q 3
R 6
S 7.5
T 6

71). P and Q started doing the work. After 2 days they both left, and R joined the work. He completed his part
of work. Now the remaining work was completed by F in 7 days. In how many days can F complete whole
work?
A) 18 days B) 16 days C) 12 days D) 20 days

72). G who can complete whole work in 30 days replaced P and did P’s part of work. He left and then Q also
worked for same number of days as G. If remaining work was completed by M who can do complete work in
one-fourth the numbers of days in which T can complete the work, then in how many days was the whole
work completed?
A) 22 days B) 14 days C) 21 days D) 18 days

73). All people decided to complete work in less number of days. So they divided the work equally among
themselves. In how many days will the work be completed this way? (They all worked individually)
A) 25 4/5 days B) 27 4/5 days C) 23 5/6 days D) 24 3/5 days

www.ibpsguide.com | estore.ibpsguide.com | www.sscexamguide.com


27
Exclusively on New Pattern – Data Interpretation eBook

74). A is 20% more efficient than Q and B is 60% more efficient than R. They worked together for 5 days and
left the work, after which the remaining work was completed by R and T together. If all worked together, in
how many less days they could have completed the work?
A) 6 3/13 days B) 5 days C) 4 3/5 days D) 7 days

75). J can complete the whole work in number of days equal to the average of number of days in which P and
Q can complete the work. J, R, S, and T all started the work and after 5 days they were replaced by P and Q.
P and Q completed the remaining work in how many days?
A) 3 1/5 days B) 2 2/3 days C) 1 2/3 days D) 2 3/5 days

Solutions

71) C
P does 20% works in 4 days.
100% work in 100*4/20 = 20 days
Q can complete 100% work in 100*3/10 = 30 days
They worked for 2 days,
[1/20 + 1/30]*2 = 1/6 work
Now R completed 25% = 1/4 work
So now remaining work = 1 – (1/6 + 1/4) = 7/12
F complete 7/12 work in 7 days, so complete work in 12 days

72) D
P’s part of work = 20% = 1/5
So G did 1/5 of work,
Whole work in 30 days, 1/5 work in 1/5 * 30 = 6 days
Q also worked for 6 days.
Q can complete whole work in 30 days, in 6 days, completed 6/30 = 1/5 of work
Now remaining work = 1 – (1/5 + 1/5) = 3/5
Now T can complete whole work in 40 days [100*6/15]
So M can complete work in 10 days. So completed 3/5 work in 3/5 * 10 = 6days
So total number of days = 6+6+6 = 18 days

73) B
5 people equally divided the work so each did 1/5 work now
P 1/5 work in 4 days as earlier
Q =1/10 work in 3 days, so 1/5 work in 6 days

www.ibpsguide.com | estore.ibpsguide.com | www.sscexamguide.com


28
Exclusively on New Pattern – Data Interpretation eBook

R =1/4 work in 6 days, so 1/5 work in 24/5 days


S = 3/10 work in 7.5 days, so 1/5 work in 5 days
T = 3/20 work in 6 days, so 1/5 work in 8 days
Complete work in = 4 + 6 + 24/5 + 5 + 8 = 27.8 days

74) A
A and Q
A = 120:100 = 6:5. Days = 5:6
6 =30, 1 =5, 5 =25.
A can complete work in 25 days
B and R
B = 160: 100 = 8: 5.Days = 5: 8
8 = 24, 1 = 3, 5 = 15.
B can complete work in 15 days
They worked for 5 days. So
[1/25 + 1/15]*5 = 8/15 work
Reaming 7/15 by R and T
So [1/24 + 1/40]*x = 7/15
x = 7 days
Total = 5+7 = 12 days
They work together
[1/25 + 1/15 + 1/24 + 1/40] = 13/75
So 75/13 days
So less days = 12 – 75/13 = 81/13 = 6 3/13 days

75) A
P in 20 days, Q in 30days.
J in (20+30)/2 = 25 days
J, R, S, and T started work
[1/25 + 1/24 + 1/25 + 1/40]= 11/75.
Worked for 5 days
11/75 * 5 =11/15 of work
Remaining work = 4/15
[1/20 + 1/30]*x = 4/15
x = 48/15 days = 3 1/5 days

www.ibpsguide.com | estore.ibpsguide.com | www.sscexamguide.com


29
Exclusively on New Pattern – Data Interpretation eBook

5. TRAIN

Set-16
Direction (76 to 80) : Refer to the following Graph and answer the given questions.
400
350
350 320 310
300 270
240 230
250
200 210 200
200 170 TRAIN X
140 TRAIN Y
150 120
100

50

0
JAN FEB MAR APR MAY JUN

76). The number of people who travelled by Train Y in JUL is 30% more than the people who travelled by the
same train in JUN. What is the respective ratio between the number of people who travelled in JUL and those
who travelled in JAN by the same train?
A) 13:16 B) 15:18 C) 20:22 D) 22:24
77). What is the difference between the total number of people who travelled by Train Y in JAN and FEB
together and the total number of people who travelled by Train X in JAN and FEB together?
A) 14 B)10 C) 12 D) 15
78). What is the average number of people travelling by Train Y in MAR, APR, MAY and JUN?
A) 155 B) 122 C) 172.5 D) 130
79). The number of people who travelled by Train X decreased by what percent from JAN to MAY?
A) 20 B) 50 C) 80 D) 60
80). The total number of people who travelled by both the given trains together in MAR is approximately what
percent more than the total number of people who travelled by both the given trains together in MAY?
A) 69 B) 80 C) 85 D) 70

Solutions
76) A
30% of 200 = 60
People travelled by Train Y in JUL = 260
People travelled by Train Y in JAN = 320
260: 320 = 13: 16
77) B

www.ibpsguide.com | estore.ibpsguide.com | www.sscexamguide.com


30
Exclusively on New Pattern – Data Interpretation eBook

Train Y in JAN and FEB together = 320 + 310 = 630


Train X in JAN and FEB together = 350 + 270 = 620
Difference = 630 – 620 = 10
78) C
200 + 170 + 120 + 200
= 172.5
79) D
People travelled by Train X in JAN = 350
People travelled by Train X in MAY= 140
= [(350 – 140)/350]*100
= [210/350]*100 = 60%
80) A
People travelled by Train X & Y in MAR = 440
People travelled by Train X & Y in MAY = 260
(440 – 260)/260*100
= 180/260 * 100
= 69%

6. BOATS and STREAMS

Set-17
Direction (81 to 85): Study the following graph carefully to answer the given questions

Down Stream Distance = 200 km Up Stream Distance = 300 km

DOWNSTREAM DISTANCE

26% 24% JAN


FEB
MAR
15% APR
20%
MAY
15%

www.ibpsguide.com | estore.ibpsguide.com | www.sscexamguide.com


31
Exclusively on New Pattern – Data Interpretation eBook

UPSTREAM DISTANCE

12%
JAN
30%
14% FEB
MAR
APR
24% MAY
20%

Days Still Water speed Stream Speed


JAN 5 -
FEB - 6
MAR 4 -
APR - 4
MAY 3 -

81). Find the ratio between the downstream distance travelled by boat in JAN, MAR, APR and MAY together
and the upstream distance travelled by the boat in the same month together?
A) 17:24 B) 25:30 C) 32:35 D) 38:40
82). Find the difference between the upstream distance travelled by boat in FEB and MAY together and the
downstream distance travelled by boat in the same month together?
A) 12 B) 14 C) 18 D) 10
83). Total downstream distance travelled by boat in JAN, FEB and MAR together is what percentage of total
upstream distance travelled by boat in the same month together?
A) 80 B) 75.5 C) 48.6 D) 50
84). A boatman rows downstream at 6kmph in MAR. Find the time taken by the boat to cover upstream
distance in the same month?
A) 30 hr B) 28 hr C) 25 hr D) 36 hr
85). A boatman rows up stream at 16kmph in FEB. Find the approximate time taken by the boat to cover
downstream distance in the same month?
A) 1hr B) 5hr C) 3hr D) 6hr

www.ibpsguide.com | estore.ibpsguide.com | www.sscexamguide.com


32
Exclusively on New Pattern – Data Interpretation eBook

Solutions
81). A
Downstream distance travelled by boat in JAN, MAR, APR and MAY = 48 + 30 + 40 + 52 = 170
Up stream distance travelled by boat in JAN, MAR, APR and MAY together = 90 + 72 + 42 + 36 = 240
17: 24
82). B
Difference = 96 – 82 = 14
83). C
Total downstream distance travelled by boat in JAN, FEB and MAR = 108
Total up stream distance travelled by boat in JAN, FEB and MAR = 222
% = 108/222 * 100 = 48.6%
84). D
Downstream Speed = 6kmph
Upstream Speed = (4 * 2) kmph – 6kmph = 2kmph
Time taken by the boat to cover upstream distance = 72/2 = 36 hours
85). A
Downstream speed = 28kmph
Time taken by the boat to cover downstream distance = 30/28 = 1 hr

Set-18

Direction (86 to 90) Study the following pie-chart and table carefully and answer the questions that follow

www.ibpsguide.com | estore.ibpsguide.com | www.sscexamguide.com


33
Exclusively on New Pattern – Data Interpretation eBook

UPSTREAM DISTANCE=150Km

14%
18%

JAN
FEB
12% MAR
30% APR
MAY

26%

DOWNSTREAM DISTANCE=180Km

15%

30% JAN
FEB
MAR
25%
APR
MAY
20%
10%

DAY SPEED OF STREAM(Km/hr)


JAN 3
FEB -
MAR 2
APR 2.5
MAY 3

86). Time taken to cover the upstream distance in MAY is same as time taken to cover the downstream
distance in APR. Total speed of still water in APR and MAY is 10 km/hr. find the ratio of speed of still water in
APR and MAY?
A) 7:13 B) 15:8 C) 23:11 D) 14:22

www.ibpsguide.com | estore.ibpsguide.com | www.sscexamguide.com


34
Exclusively on New Pattern – Data Interpretation eBook

87). In JAN, the boat takes a total of 4 hrs 30 minutes to cover both upstream and downstream distance.
Ratio of speed of boat in still water in going upstream to downstream is 4: 5. Find the speed of boat in still
water while going downstream?
A) 7Km/HR B) 15Km/hr C) 11Km/hr D) 14Km/hr
88). In FEB, Ratio of speed of boat in still water in going upstream to downstream is 3: 8. Also difference in
speed of boat in still water in going upstream and downstream is 5 km/hr. If the total time taken by boat to
cover upstream and downstream distance is 14 hours in FEB, find the speed of stream?
A) 3Km/hr B) 8Km/hr C) 1Km/hr D) 2Km/hr
89). In MAR, Ratio of speed of boat in still water in going upstream to downstream is 4 : 5. The difference
between time to cover upstream distance and downstream distance is 5 hours; find the total time taken to
cover upstream distance and downstream distance?
A) 7hr B) 5hr C) 2hr D) 8hr
90). Time taken to cover the upstream distance in APR is 12 hours more than time taken to cover the
downstream distance in MAY. Total speed of still water in APR and MAY is 11 km/hr. Find the ratio of speed
of still water in APR and MAY?
A) 5:6 B) 11:18 C) 5:15 D) 18:11

Solutions
Distances upstream:
JAN = 18/100 * 150 = 27 km, FEB= 12/100 * 150 = 18 km. MAR = 39 km, APR = 45 km, MAY = 21 km
Similarly Distances downstream:
JAN = 27 km, FEB = 45 km. MAR = 18 km, APR = 36 km, MAY = 54 km
86) A
Let speed of still water upstream in MAY = x, then downstream in APR = (10-x)
21/(x-3) = 36/[(10-x)+2.5]
x = 6.5
Ratio is 3.5 : 6.5 = 7 : 13
87) B
Speeds – 4x and 5x
in JAN
27/ (4x-3) + 27/(5x+3) = 9/2
Solve, x = 3
downstream speed = 5x = 15 km/hr
88) C
8x and 3x, Also 8x – 3x = 5
So x = 1, speeds are 8 and 3 km/hr
So for FEB

www.ibpsguide.com | estore.ibpsguide.com | www.sscexamguide.com


35
Exclusively on New Pattern – Data Interpretation eBook

18/ (3-b) + 45/ (8+b) = 14


b = 1 km/hr
89) D
In MAR
39/ (4x-2) – 18/ (5x+2) = 5
x = 2 km/hr
39/(8-2) + 18/(10+2) = 8 hours
90) A
Let speed of still water upstream in APR = x, then downstream in MAY = (11-x)
Now
45/(x-2.5) – 54/[(11-x)+3)] = 12
15/(x-2.5) – 18/(14-x) = 4
x=5
So required ratio is 5 : 6

Set-19
Directions (91-95 ): Study the following bar graph and answer the questions that follow.

SPEED OF BOATS
25

20 20

16
15 15
12
SPEED OF BOATS
10
9

0
A B C D E

91). On a particular day, the time taken by boat A to cover 255 km downstream is 2 hours more than the time
taken by boat D to cover 195 km upstream. Find the ratio of upstream speed to downstream speed in case of
boat A on that particular day.
A) 7:17 B) 8 : 19 C) 19 : 32 D) 6 : 4
92). Boat B covered 756 km downstream on Monday for which it took 6 hours less than that in which it
covered half distance upstream on Tuesday. On Tuesday, speed of stream was 2 km/hr more than that on

www.ibpsguide.com | estore.ibpsguide.com | www.sscexamguide.com


36
Exclusively on New Pattern – Data Interpretation eBook

Monday. Find the downstream speed of boat B on Tuesday.


A) 28 km/hr B) 23 km/hr C) 32 km/hr D) 22 km/hr
93). For boat C, its upstream speed is 6 km/hr on a particular day. Find the difference in time in covering 360
km by boats A and C on that particular day.
A) 4 hours B) 5 hours C) 6 hour D) 2 hours
94). On a particular day, ratio of upstream speed to downstream speed of boat D is 3: 7. It took 20 hours
more to cover a distance upstream than same distance downstream by boat D. On that particular day, boat B
covered same distance in how much time?
A) 22 hours B) 38hours C) 53hours D) 17.5 hours
95). Upstream speed of boat E is 9 km/hr. How many more hours will it take to cover a distance of 315 km
upstream than same distance downstream?
A) 20 hours B) 24 hours C) 32 hours D) 8 hours

Solutions
91). A
Let x km/hr is the speed of stream on that particular day.
So 255/ (12+x) = 195/ (20-x) + 2
Solve, x = 5 km/hr
ratio of speed upstream: downstream = (12-5): (12+5) = 7: 17
92). B
On Monday, speed of stream is x km/hr
On Tuesday it is (x+2) km/hr
On Monday = 756 km,
Tuesday = 756/2 = 378 KM
756 / (16+x) = 378/ (16-(x+2)) – 6
126 / (16+x) = 63 / (14-x) – 1
x = 5 km/hr
Speed of stream on Tuesday = (5+2) = 7 km/hr
Downstream speed of boat B = (16+7) = 23 km/hr
93). C
Upstream speed of C = 6 km/hr, so speed of stream: 9 – x = 6, x = 3 km/hr
Downstream speed of boat C = 9+3 = 12 km/hr
Downstream speed of boat A = 12+3 = 15 km/hr
Difference in timings = 360/12 – 360/15 = 30 – 24 = 6 hour
94). D
Let speed of stream on that day = x km/hr
So (20-x) / (20+x) = 3/7

www.ibpsguide.com | estore.ibpsguide.com | www.sscexamguide.com


37
Exclusively on New Pattern – Data Interpretation eBook

x = 8 km/hr
y / (20-8) – y / (20+8) = 20
y = 420 km
Required time = 420/(16+8) = 420/24 = 17.5 hours
95). A
Upstream speed = 9, speed of boat = 15 km/hr
Speed of stream = 15-9 = 6 km/hr
Downstream speed = 15+6 = 21 km/hr
required time = 315/9 – 315/21 = 20 hours

7. Profit and Percentage

Set-20
Directions (96-100): The line graph gives profit percent of A and B over different years. Study the following
graph carefully and answer the questions given below it.

100

90 90

80
75 75
70

60 60
55 55
50 50 A
45
40 40 40 40 B

30
25 25
20

10

0
1991 1992 1993 1994 1995 1996 1997

96). Investment of company ‘B’ in 1997 is more by 40% than that in the previous year. Income in 1997 was
what percent of the investment in 1996?
A) 280% B) 252% C) 245% D) 52%
97). Average investment of company ‘A’ over the years was Rs.26 LAKH. What was its average income over
the years?
A) RS.40.56 LAKHS B) RS.41.60 LAKHS C) RS.50.26 LAKHS D) DATA INADEQUATE
98). Income of company ‘A’ in 1995 was Rs.21.7 LAKHS. What was the investment?

www.ibpsguide.com | estore.ibpsguide.com | www.sscexamguide.com


38
Exclusively on New Pattern – Data Interpretation eBook

A) RS.14 LAKHS B) RS.15.4 LAKHS C) RS.15.8 LAKHS D) RS.14.8 LAKHS


99). Income of company ‘A’ in 1995 is equal to the investment of company B in 1996. What is the ratio of the
investment of company ‘A’ in 1995 to the investment of company ‘B’ in 1996?
A) 37:36 B) 31:28 C) 20:31 D) DATA INADEQUATE
100). Investment of company ’B’ in 1993 was Rs.15,40,000, what was its income in that year?
A) RS.23.33 LAKHS B) RS.22.33 LAKHS C) RS.22.23 LAKHS D) RS.23.23 LAKHS

Solutions
96) C
Company B investment income 1996=100%
Income = 175/100*Investment
1997 140% of 1996
Income =175/100 of Investment
= 175/100*140/100
Income of 1997/investment income of 1996 = 175/100*140/100 = 245%
97) D Data inadequate
98) A In 1995
Company A profit = 55 = income-investment/investment
55/100+1 = income/Investment
Investment = 14lakh
99) C
Let Investment of Company A 1995 = 100
Income of Company A = 155 = Investment of Company A 1995
Investment of Company A : Investment of Company B=100:155=20:31
100) B , Company B in 1993
P = income – investment/investment
45/100+1 = Income/154000
Income = 22.33 LAKH
8. Boats and Stream
Set-21
Directions(101-105): Study the following pie-chart and table carefully and answer the questions that follow.

www.ibpsguide.com | estore.ibpsguide.com | www.sscexamguide.com


39
Exclusively on New Pattern – Data Interpretation eBook

Upstream Distance = 300Km

12%

30% JAN
14%
FEB
MAR
APR
MAY
24%
20%

Downstream Distance =200Km

26% 24%
JAN
FEB
MAR
APR
15%
MAY
20%

15%

MONTH Still Water speed Stream Speed


JAN 5 -
FEB - 6
MAR 4 -
APR - 4
MAY 3 -

101) A boatman rows downstream at 6kmph on MAR. Find the speed of upstream and Speed of current?
A) 7 KMPH, 4KMPH B) 2 KMPH, 2 KMPH C) 3 KMPH, 1 KMPH D) 5 KMPH, 6 KMPH

www.ibpsguide.com | estore.ibpsguide.com | www.sscexamguide.com


40
Exclusively on New Pattern – Data Interpretation eBook

102) A boatman rows up stream at 4kmph, 2 KMPH and 8 KMPH in JAN, MAR and APR respectively. Then
find the ratio between the downstream speed of boat in JAN, MAR, and APR together and the rate of current
on the same days together?
A) 4:7 B) 3:1 C) 4:1 D) 4:3
103) A boatman rows up stream at 16 KMPH and 2 KMPH in FEB and MAY. Then find the difference
between the downstream speed of boat in FEB and MAY together and the speed of boat in still water on the
same months together?
A) 7 KMPH B) 11 KMPH C) 5 KMPH D) 3 KMPH
104) What time will be taken by a boat to cover a distance in FEB along the stream, if the upstream of boat is
3 KMPH?
A) 4 hours B) 5 hours C) 2 hours D) 3 hours
105) What is the sum of total distance travelled by the boat in JAN, MAR and MAY together?
A) 316 B) 328 C) 356 D) 296

SOLUTIONS

101) B
Downstream Speed = 6kmph
Upstream Speed = 8kmph – 6kmph = 2kmph
Speed of Current = (6-2)/2 = 2kmph
102) C
Downstream Speed of boat in JAN = 10 – 4 = 6kmph
Downstream Speed of boat in MAR and APR = 6 kmph, 16kmph
Rate of current = 1kmph, 2kmph and 4kmph
Ratio = 28:7 = 4: 1
103) A
Downstream speed of boat in FEB and MAY = 38kmph
Still water speed = 20kmph
Difference = 38 – 20 = 18 kmph
104) C
Upstream of boat = 3kmph
Downstream of boat =15kmph
Downstream Distance = 15% of 200 = 30; Time = 30/15 = 2 hr
105) B
Total distance travelled by the boat in JAN = 48 + 90 = 138
Similarly Total distance travelled by the boat in MAR and MAY=102, 88
Total distance travelled by the boat in JAN, MAR and MAY =138 + 102 + 88 = 328 km

www.ibpsguide.com | estore.ibpsguide.com | www.sscexamguide.com


41
Exclusively on New Pattern – Data Interpretation eBook

9. Caselet

Set-22
Directions(106-110): Ghosh Babu has recently acquired four companies namely Arc – Net Technologies
(ANT), Babu Anta Transport (BAT), Charles Anter Tailor (CAT) and Daud Akbar Transistors (DAT). When the
results of the companies for the year 1992 – 93 were placed before him. He found a few interesting things
about them. While the profits of CAT and DAT were the same, the sales of CAT were the same as those of
BAT. Profits of ANT were 10% of its sales, where as the profits of BAT were 20% of its sales. While the total
expenses of CAT were 5 times its profits, sales of DAT were 3 times its profits. The total expenses of CAT
were Rs. 1000000 the total expenses of ANT were 10% less than those of CAT. Profits are defined as the
difference between sales and total expenses.
106). Which company had the lowest sales?
(A) ANT (B) BAT (C) CAT (D) DAT
107). Which company had the highest total expenses?
(A) ANT (B) BAT (C) CAT (D) DAT
108). Which company had the lowest profits?
(A) ANT (B) BAT (C) CAT (D) DAT
109). Which company had the highest profits?
(A) ANT (B) BAT (C) CAT (D) DAT

Solutions
Let the profits gained by the companies CAT & DAT = x,
Let the sale of the company CAT & BAT = y
Let the sale of the company sales of ANT = z.

COMPANY SALES EXPENDITURE PROFIT


ANT Z 0.9Z 0.1Z
BAT Y 0.8Y 0.2Y
CAT Y 5X X
DAT 3X X

First the total expenses of CAT were Rs.


Therefore 5x = Rs.10 LAKHS
x = Rs.2 LAKHS.
Also the difference between the total expenses of ANT and CAT = 10% = Rs.9 LAKHS.
Therefore 0.9z = 9 LAKHS
z = 10 LAKHS.

www.ibpsguide.com | estore.ibpsguide.com | www.sscexamguide.com


42
Exclusively on New Pattern – Data Interpretation eBook

Since the Profit = Sales – Expenditure


Sales = Expenditure + Profit
6x = 12 LAKHS. Hence y = 12 LAKHS.
So from the above information the table would be
COMPANY SALES EXPENDITURE PROFIT
ANT 10 L 9L 1L
BAT 12 L 9.6 L 2.4 L
CAT 12 L 10 L 2L
DAT 6L 4L 2L

106) D
From the above table it can be seen that the company with the lowest sales is DAT Rs.6 LAKHS.
107) C
CAT had highest total expenses i.e. Rs. 10 LAKHS.
108) A
ANT had lowest profits i.e. Rs. 1 LAKH.
109) B
BAT had the highest profits i.e. Rs. 2.4 LAKHS.

10. Profit and Percentage

Set-23
Directions(111-105): Following graph shows that percentage rise in profit of two fruits over the month.
Answer the following question.

40

35 35

30 30 30

25 25 25 25 25

20 20 20 APPLE

15 15 15 GRAPES

10 10

0
JAN FEB MAR APR MAY JUN

www.ibpsguide.com | estore.ibpsguide.com | www.sscexamguide.com


43
Exclusively on New Pattern – Data Interpretation eBook

111). If the profit earned by apple in the month Feb. was Rs.1.50lakh, what was the profit earned by the apple
in the month May. ?
A) 258750 B) 256900 C) 369800 D) 589600
112). If the profit earned by grapes in the month FEB. was Rs.2lakh, what was the profit earned by the grapes
in the month JUN?
A) 150000 B) 350000 C) 528125 D) None of these
113). If the profit earned by Apple in the month Mar. was Rs.1.84lakh, what was the amount of profit earned
by the grapes in the month Mar.?
A) RS 1.24lakh B) Rs 2.36lakh C) RS 6.36lakh D) Cannot be determined
114). What is the percentage increase in profit of grapes in the month Apr. from the JAN.?
A) 18 B) 5 C) 10 D) 50
115). What is the percentage increase in profit of apple in the month JUN. from the previous month?
A) 80 B) 60 C) 75 D) 20

SOLUTIONS

111). A
Profit earned by apple in the month Feb. = Rs.1.50lakh
profit earned by apple in the month May. = 120%of125%of115% of 150000 = 258750
112). C
Profit earned by the grapes in the month Feb. = Rs.2lakh
profit earned by the grapes in the month Mar. = 130% OF130%OF125%OF125% of 200000 = 528125
113). D
From the value of the profit earned by apple, we cannot determine the profit value of grapes.
114). D
Percentage rise in profit of grapes in the month JAN. = 20%
Percentage rise in profit of grapes in the month Apr. = 30%
Required Percentage rise = (30-20)/20 * 100 = 50%
115). C
Percentage rise in profit of apple in the month May. = 20%
Percentage rise in profit of apple in the month Jun. = 35%
Required Percentage rise = (35-20)/20 * 100 = 75%

11. Percentage DI
Set-24
Directions(116-120): Study the following graph and the table and answer the questions given below.
Data of different states regarding population of states in the year 1998

www.ibpsguide.com | estore.ibpsguide.com | www.sscexamguide.com


44
Exclusively on New Pattern – Data Interpretation eBook

Sales

G
9% A
F 25%
15%

E
11%
B
D 20%
12% C
8%

Total Population of the given states=3276000


States Male Female literate illiterate
A 5 3 2 7
B 3 1 1 4
C 2 3 2 1
D 3 5 3 2
E 3 4 4 1
F 3 2 7 2
G 3 4 9 4

116). What will be the percentage of total number of males in F, B and D together to the total population of all
the given states?
A) 28.5 B) 20 C) 30 D) 45
117). What was the total number of illiterate people in A and B in 1998?
A) 3261160 B) 1161160 C) 4561160 D) 3561160
118). What is the ratio of the number of females in G to the number of females in C?
A) 20:25 B) 20:25 C) 15:14 D) 5:10
119). What was the number of males in F in the year 1998?
A) 523600 B) 985200 C) 456300 D) 294840
120). If in the year 1998, there was an increase of 10% in the population of F and 12% in the population of B
compared to the previous year, then what was the ratio of populations of F and B in 1997?
A) 165:224 B) 122:114 C) 233:144 D) 245:130

SOLUTIONS

www.ibpsguide.com | estore.ibpsguide.com | www.sscexamguide.com


45
Exclusively on New Pattern – Data Interpretation eBook

116) A
3 3 15 3276000.
Number of males in F= of (15% of N) = x x 3276000. = 9 x .
5 5 100 100

3 3 20 3276000.
Number of males in B = of (20% of N) = x x 3276000. = 15 x .
4 4 100 100
3 3 12 3276000.
Number of males in D= of (12% of N) = x x 3276000.= 4.5 x .
8 8 100 100
3276000.
Total number of males in these three states = (9 + 15 + 4.5) x
100

3276000.
= 28.5 x .
100

3276000.
28.5 x
Required Percentage = 100 x 100 % = 28.5%
3276000

117) B
7
No. of illiterate people in A = of (25% of 3276000) = 637000.
9
4
No. of illiterate people in B = of (20% of 3276000) = 524160.
5
Total number = (637000 + 524160) = 1161160
118) C
4
of (9% of 3276000)
7
Required ratio =
3
of (8% of 3276000)
5

15
= .
14

119) D
3
Number of males in F = of (15% of 3276000)
5

www.ibpsguide.com | estore.ibpsguide.com | www.sscexamguide.com


46
Exclusively on New Pattern – Data Interpretation eBook

3 15
= x x 3726000
5 100

= 294840.

120) A
100
xx
110 112x
Ratio of populations of F and B in 1997 = = .
100 110y
xy
112
Required ratio

112x 15
=
110y 20

x: y = 165:224

12. Simple Interest and Compound Interest

Set-25
Directions(121-125): Study the given table carefully to answer the given questions.
Name Nature of Interest Principle Amount Year Rate
Anil Compound Rs.10000 4
Nikhil Simple - Rs.29500 3 -
Vijay Compound Rs.20000 2 3
Ajay Simple - 4 -
Arun Compound - 5 2

121) What is the principal of Nikhil if the ratio of interest rate of Anil to that of Nikhil is 2: 3?
A) Rs.10449 B) Rs.20230 C) Rs.25000 D) Rs.17121
122) What is the amount of Vijay, if the interest is compounded yearly for 2 years?
A) Rs.19165.7 B) Rs.20320.9 C) Rs.21218 D) Rs.22418
123) If the ratio of principal of Vijay to that of Ajay is 4: 5 and the rate of interest of Ajay is 10% more than that
of Vijay, then what is the interest of Ajay?
A) Rs.3300 B) Rs.3142 C) Rs.3201 D) Rs.2890
124) At what rate of interest the amount of Nikhil becomes five times that of his principal?

www.ibpsguide.com | estore.ibpsguide.com | www.sscexamguide.com


47
Exclusively on New Pattern – Data Interpretation eBook

A) Rs.63.58% B) Rs.69.96% C) Rs.60% D) Rs.133.33%


125) What is the amount of Arun when the principal of Arun is 20% more than that of Anil?
A) Rs.32321.5 B) Rs.13248.97 C) Rs.54974.19 D) Rs.32863.24

SOLUTIONS
121) C
R% = 4*3/2 = 6
Let Principle X
X+X*3*6/100 = 29500
X = 25000
122)C
A=P(1+R/100)2
= 20000*1.03*1.03
A = 21218Rs
123) A
Principle = 5/4*20000 = 25000
R = 3.3%
Interest = 25000*3.3*4/100 =3300
124) D
Amount N = 29500
Principle = 29500/5 = 5900
Time = 3year
R% = 133.33
125) B
Principle of Arun = 10000Rs
Arun = 12000(1+2/100)^5
= 13248.97
13. Missing DI

Set-26
Directions(126-130): Study the table carefully to answer the following questions.
The percentage profit is given on total cost price.
Cost price=cost of production + transportation + packaging cost

Cost of Selling
Name of Cost of Cost of Percentage of
Production price Profit/Loss
Oils Transportation packaging Profit or Loss
per kg per kg

www.ibpsguide.com | estore.ibpsguide.com | www.sscexamguide.com


48
Exclusively on New Pattern – Data Interpretation eBook

Coconut Oil 80 8 - 120 - -

Palm Oil 40 0 0 - - 5% Profit


Palm Kernel
45 - 5 - 50
Oil
Olive Oil 20 3 1 -
Vegetable
70 10 - 90 - 6% Loss
Oil

126) If the percentage of profit on sold Coconut Oil is 10%, then what is its cost of packaging?
A) Rs.24.90 B) Rs.23.50 C) Rs.22 D) Rs.21.09
127) What is the difference between the selling price of Palm Kernel Oil and that of Palm Oil, if the cost of
transportation is zero for both?
A) Rs.56 B) Rs.52 C) Rs.58 D) Rs.36
128)What is the cost of packaging of Vegetable Oil?
A) Rs.24.90 B) Rs.23.50 C) Rs.22 D) Rs.15.74
129) What is the percentage profit of Olive Oil if its selling price is 80% of the cost price of Palm Oil?
A) 33.33% B) 30% C) 32% D) 30.49%
130) 4 kg Coconut Oil, 3 kg Palm Oil and 5 kg Olive Oil are sold. What is profit or loss percentage? (The
packing cost is zero for all goods and selling price of Olive Oil is 32 per kg?
A) 36% B) 32% C) 30.49% D) 34.2%

SOLUTIONS

126) D
Selling price of Coconut Oil=120 per kg.
Profit=-10%
Total cost price=120X100/110=109.09
Cost of packaging=109.09-80-8=21.09 RS
127)C
Selling price of Palm Oil=40X105/100=42
Selling price ofPalm Kernel Oil=45+5+50=100
Required difference=100-42=58
128) D
Selling price of Vegetable Oil=90 RS.
Loss=6%
Total CP=90X100/94=95.74
Cost of packaging of Vegetable Oil=95.74-70-10=15.74

www.ibpsguide.com | estore.ibpsguide.com | www.sscexamguide.com


49
Exclusively on New Pattern – Data Interpretation eBook

129) A
CP of Palm Oil=40
SP of Olive Oil=40X80/100=32
CP of Olive Oil=20+3+1=24
% profit= (32-24)/24X100=33.33%
130) C
CP of (4 kg Ghee+3kg Palm Oil +5 kg Olive Oil) = 4X (80+8) +3X40+5X (20+3)
=352+120+115=587
SP of (4 kg Coconut Oil+3kg Palm Oil+5kg Olive Oil)=4X120+3X42+5X32
=480+126+160=766
% Profit= (766-587)/587X100=30.49%

14. Geometry Missing DI

Set-27
Directions (131-135): Study the given table carefully to answer the following questions.
Cost Of
Cost Of
Flooring
Field Side Base Height Radius Fencing
Shape (in Rs per
Name (in mm) (in m) (in m) (in m) (in Rs per
Square
meter)
meter)
A Triangle - 16 12 - 50 20
B Rectangle 10*20 - - - 30 15
C Square 15 - - - 40 18
D Parallelogram - 20 12 - 60 25
E Circle - - - 10 45 22

131) What is the cost of flooring of A?


A) Rs.4000 B) Rs.4600 C) Rs.4800 D) Rs.5000
132) What is the difference between the cost of fencing of C and that of B?
A) Rs.180 B) Rs.120 C) Rs.240 D) Rs.360
133) What is the ratio of the cost of flooring to that of fencing of field D?
A) 4: 1 B) 6: 1 C) 8: 1 D) 9: 1
134) The cost of fencing of field E is approximately what per cent of the cost of flooring of field C?
A) 10.5% B) 19.46% C) 18.71% D) 15.36%
135) The cost of fencing of field C is what per cent of the cost of fencing of field D?
A) 87.54% B) 67.5% C) 72.13% D) 54.36%

www.ibpsguide.com | estore.ibpsguide.com | www.sscexamguide.com


50
Exclusively on New Pattern – Data Interpretation eBook

SOLUTIONS
131) C
A is triangle
Area of A=1/2(16X12)=96m^2
Cost of flooring of A=96X50=4800
132) A
Perimeter of B=2(10+20)=60 m
Cost of fencing of B=60X15=900 Rs.
Perimeter of C=4X15=60 m
So, cost of fencing of C=60X18=1080 Rs.
Required difference=1080-900=180
133) D
Area of D=BaseXHeight=20X12=240 m^2
Cost of flooring of D=240X60=14400
Perimeter of D=2(20+12) =64
Cost of Fencing of D=64X25=1600
Required ratio=14400:1600=9:1
134) D
Perimeter of E=2PiR=2X22/7X10=440/7 m
Cost of Fencing of E=440/7X22=1382.85
Area of C=15^2=225m^2
Cost of flooring of C=225X40=9000
Required %=1382.85X100/9000=15.36%
135) B
Fencing cost of C=1080
Fencing cost of D=1600
Required %=( 1080X100)/1600=67.5%

15. Income and Expenditure

Set-28

Directions(136-140): The profit percentage of 2 companies was given below. Refer the following graph &
answer the following questions.

www.ibpsguide.com | estore.ibpsguide.com | www.sscexamguide.com


51
Exclusively on New Pattern – Data Interpretation eBook

70
65
60 60
55 55
50 50 50 50 50
45 45
40 40
35 A
30
B
20

10

0
JAN FEB MAR APR MAY JUN

136). The incomes of two Companies A and B in MAY were in the ratio of 3:4 respectively. What was the
respective ratio of their expenditures in MAY?
A) 15:22 B) 14:24 C) 23:32 D) 28:42
137). If the expenditure of Company B in FEB was Rs.220crores, what was its income in FEB?
A) Rs.257corores B) Rs.297corores C) Rs.347corores D) Rs.367corores
138). If the expenditures of Company A and B in JAN were equal and the total income of the two Companies
in JAN was Rs.342crores, what was the total profit of the two Companies together in JAN?
A) Rs.111corores B) Rs.232corores C) Rs.102corores D) Rs.222 corores
139). The expenditure of Company A in the month MAR was Rs.200crores and the income of company A in
MAR was the same as its expenditure in JUN. The income of Company Ain JUN was?
A) Rs.327corores B) Rs.275corores C) Rs.865corores D) Rs.465corores
140). If the incomes of two companies were equal in APR, then what was the ratio of expenditure of Company
A to that of Company B in APR?
A) 16:15 B) 34:56 C) 11:55 D) 44:55

SOLUTIONS

Profit = (Income-Expenditure)/Expenditure*100
136) A
For company A
Frome above Formula
65 = (3x-E1)/E1*100
E1 = 3x (100/165)
Forcompany B
From above formula
50 = (4x-E2)/E2*100
E2 = 4x (100/150)

www.ibpsguide.com | estore.ibpsguide.com | www.sscexamguide.com


52
Exclusively on New Pattern – Data Interpretation eBook

E1/E2 = 15/22
137) B
35 = (x-220)/220*100
x = Rs.297crores.
138) C
Let the expenditures of each companies A and B in JAN be Rs. x crores.
And let the income of Company A in JAN be Rs.zcrores.
So that the income of Company B in JAN = Rs.(342 - z) crores.
For A
40 = (z-x)/x*100
x = 100z/140…… (J)
For B
45 = (342-z-x)/x*100
x= (342-z)/145*100…….. (K)
Substituting z = 168 in (J), we get
x = 120.
Total expenditure of Companies A and B in JAN = 2x = Rs.240crores.
Total income of Companies A and B in JAN = Rs.342crores
Total profit = Rs.(342 - 240) crores = Rs.102crores.
139) D
For Company A
Let the income of Company A in MAR be Rs.x crores.
55 = (x-200)/200*100 = 310
Let the income of Company A in JUN be Rs. z crores.
50 = (z -310)/310 *100
z = Rs.465crores
Income of Company A in JUN = Rs.465crores.
140) A For Company A
50 = (x-E1)/E1*100
x=1.5E1
For Company B
60 = (x-E2)/E2*100
x=1.6E2
E1/E2 = 16/15

Set-29

www.ibpsguide.com | estore.ibpsguide.com | www.sscexamguide.com


53
Exclusively on New Pattern – Data Interpretation eBook

Directions(141-145): The profit percentage of a company over different months have been given below. Refer
the following graph & answer the following questions
80
70 70
65
60 60
55
50
45
40 40
Column1
30
20
10
0
JAN FEB MAR APR MAY JUN

141). If the expenditures in FEB and MAY are equal, then the approximate ratio of the income in FEB and
MAY respectively is?
A) 31:34 B) 22:24 C) 12:14 D) 25:28
142). If the income in APR was Rs.264crores, what was the expenditure in APR?
A) Rs.190crores. B) Rs.160crores. C) Rs.250crores. D) Rs.360crores.
143). In which month is the expenditure minimum?
A) FEB B) MAY C) cannot be determined D) APR
144). If the profit in MAY was Rs.4crores, what was the profit in JUN?
A) Rs.6.4corores B) Rs.5.4corores C) Rs.2.4corores D) Cannot be determined
145). What is the average profit earned for the given months?
A) 55(5/6) B) 22(1/2) C) 12(3/8) D) 25(4/5)

SOLUTIONS
141) A
For FEB
55 =( I1-x)/x*100 =
I1 = 155x/100
For May
70 =( I2-x)/x*100
I2 = 170x/100
I1/I2 = 31:34
142) B
For APR
65 = (264-x)/x*100
x = 160corores
143) C

www.ibpsguide.com | estore.ibpsguide.com | www.sscexamguide.com


54
Exclusively on New Pattern – Data Interpretation eBook

The line-graph gives the comparison of percent profit for different months the comparison of the expenditures
is not possible without more data.
Therefore, the year with minimum expenditure cannot be determined.
144) D
From the line-graph we obtain information about the percentage profit only. To find the profit in JUN we must
have the data for the income or expenditure in JUN.
Therefore, the profit for JUN cannot be determined.
145) A
Average percent profit earned for the given month
1/6*(40+55+45+65+70+60)
= 55(5/6)

16. Missing DI

Set-30
Directions(146-150): Study the given table carefully to answer the given questions.
Speed Time Taken
Station Distance Between Two Station(km)
(m/min) (min)
Rajiv Chowk to Mandi House 1100 2.75 -
Mandi House to Yamuna Bank 1500 - -
Yamuna Bank to Mayur Vihar - 4.5 3
Mayur Vihar to Ashok Nagar 1200 2.25 -
Ashok Nagar to Noida City 1000 - -
146) From Mandi house to Yamuna bank station, if the Metro train takes twice the time it takes to travel from
Rajiv Chowk to Mandi house, then what is the distance between Mandi house to Yamuna bank station? (in
km)
A) 5 KM B) 6 KM C) 7.5 KM D) 7 KM
147) What is the average speed of the train from Rajiv Chowk to Noida city?
A) 75 KM/HR B) 79.3 KM/HR C) 75.6 KM/HR D) 73.2 KM/HR
148) If total distance from Rajiv Chowk to Noida city is 20 km, then find time taken to travel from Ashok nagar
to Noida city.
A) 2 MINS B) 3 MINS C) 4 MINS D) 5 MINS
149) By what percent time taken from station Yamuna Bank to Mayur Vihar is more/less than the time taken
from Mayur Vihar to Ashok Nagar by the metro?
A) 66 2/3 % B) 65 1/3 % C) 62 1/3 % D) 60 %

www.ibpsguide.com | estore.ibpsguide.com | www.sscexamguide.com


55
Exclusively on New Pattern – Data Interpretation eBook

150) If due to some problem, the train takes 2 more minutes to reach Noida city from Ashok Nagar, then by
what percent the average speed of entire journey has changed?
A) 6.35 % B) 7.28% C) 8.54% D) NONE OF THESE

SOLUTIONS

146) C
Time taken from Rajiv Chowk to Mandi house
= 2.75*1000/1100
= 2.5 minutes
Time taken from Mandi house to Rajiv Chowk = 5min
Distance = 5*1500 = 7.5km
147) C
Average = (1100+1500+1500+1200+1000)/5
= 75.6 km/hr
148) B
Total Distance = 2.75+7.50+4.5+2.25+d = 20; D=3km
Time = 3000/1000 = 3min
149) D
Required Percentage =( 3-1.875)/1.875 = 60% more
150) A
Speed from Ashok Nagar to Noida City = 3000/5 = 600
New Average Speed = (1100+1500+1500+1200+600)/5 = 1180
Required Percentage = (1260-1180)/1260*100; = 6.35%
17. Pipes and Cistern

Set=31
Directions (151-155): Study the following table carefully to answer the given questions
No. Of days for which individuals worked on different files Part of File
Files Number Uncompleted
M N O P
after all work
1 6 6 2 3 1/3
2 - 3 4 - -
3 - - - - -
4 5 3 2 - 1/6
5 2 2 3 5 1/12

www.ibpsguide.com | estore.ibpsguide.com | www.sscexamguide.com


56
Exclusively on New Pattern – Data Interpretation eBook

151). M and N started working on Files 1. They completed 1/3rd of work after which they left and O joined the
Files. O can complete the whole Files in 12 days. After O worked for his assigned number of days, P joined the
files and worked for his assigned number of days. Find the number of days in which P can complete the whole
Files 1?
A) 18 days B) 10 days C) 13 days D) 35 days
152). M who can complete Files 2 in 20 days, worked for 6 days. The ratio of number of days in which N and O
can complete Files 2 alone is 5: 8. P could not come to work for Files 2. Find the number of days in which N
and O can complete 13/30 of Files 2 together, given that if P who can complete work in 12 days had also joined
the Files for 4 days, the Files would have been completed.
A) 1 days B) 4 days C) 8 days D) 7 days
153). A file 3 was to be completed in 6 days. To complete Files in time, all M, N, O and P decided to work in
pairs in alternate days. M and O on 1st day, N and P on 2nd day, M and O on 3rd, and so on. But they could not
complete Files in time. What percent of Files 3 remain uncompleted if M, N, O and Pcan complete whole Files
3 in 12, 18, 20 and 15 days respectively?
A) 33/4% B) 12/5% C) 70/3% D) 23/3%
154). N and O can complete the whole Files 4 in 6 days working together. M is 20% more efficient than O and
40% less efficient that N. How many days did P work on Files 4 if P can complete whole Files 4 in 18 days?
A) 2 days B) 9 days C) 6 days D) 1 day
155). M, Q and P worked on Files numbered 5.N and O alone can complete whole Files numbered 5 in 20 and
30 days respectively. Q who is 3/2 times efficient than N and Otogether replaces both of them and worked for
same number of days for which N and Ohad to work. M completed 1/12th of the work. Find in how many days
all M, N, O, and P can complete the Files 5 together.
A) 6 days B) 4 days C) 1 days D) 13 days

SOLUTIONS
151) A
M and N complete 1/3rd, O completed 1/12 * 2 = 1/6
Completed work before P joined is 1/3 + 1/6 = 1/2
P = 1 – 1/2 = 1/2 of work
Now Files uncompleted after P left = 1/3
So P did = 1/2 – 1/3 = 1/6 of work
He worked for 3 days, means he complete 1/6th of work in 3 days.
He can do complete Files 1 in 6/1 * 3 = 18 days
152) B
Number of days in which N and O can complete Files 2 alone is 5x and 8x respectively.
Now if P also joined for 4 days, Files would have been completed, means
6/20 + 3/5x + 4/8x + 4/12 = 1

www.ibpsguide.com | estore.ibpsguide.com | www.sscexamguide.com


57
Exclusively on New Pattern – Data Interpretation eBook

11/10x = 11/30
x=3
SoN can complete Files in 5x = 15 days, and C in 8x = 24 days.
So 13/30 work, N and O together – (1/15 + 1/24)*y = 13/30
y = 4 days
153) C
Files is to be completed in 6 days. They all worked in pairs on alternate means all worked for 3 days each.
On 1st day, work completed by M and O = 1/12 + 1/20 = 2/15
On 2nd day, work completed by N and P = 1/18 + 1/15 = 11/90
1st pair worked for 3 days, so work completed by them = 2/15 * 3 = 2/5
Similarly, by N and P = 11/90 * 3 = 11/30
So part of Files uncompleted= 1 – (2/5 + 11/30) = 7/30
So % = 7/30 * 100 = 70/3%
154) D
Efficiency — M : O = 120 : 100 = 6 : 5
So days ratio = 5 : 6
Similarly, M : N = 60 : 100 = 3 : 5, so days 5 : 3
N/M = 3/5, M/O = 5/6
So ratio of days for N : M : O is 3 : 5 : 6 …………(1)
Now — N and O can complete the whole Files 4 in 6 days working together, and ratio of no. of days in which
they can complete work alone is N and O from (1) is 3 : 6 = 1 : 2
So
1/x + 1/2x = 1/6
x=9
So N can complete work in 9 days, O in 18 days and then M in 15 days
Now, let P worked for ‘y’ days on Files 4. So
1/15 *5 + 1/9 * 3 + 1/18 * 2 + 1/18 * y = 1 – 1/6
y = 1 day
155) A
N and O together can complete work in 12 days [1/20 + 1/30 = 5/60 — 12 days]
Efficiency E : (N+O) = 3/2 : 1 = 3 : 2
So, ratio of number days = 2 : 3
So Q can complete whole work in 2/3 * 12= 8 days
Now Q worked for (2+3) = 5 days – total days for which N and O worked.
So Q completed 5/8 of work, M completed 1/12 of work. 1/12 is uncompleted work. Let x is no. of days in
which P can complete whole work. So
M’s work + Q’s work + P’s work = 1 – uncompleted work

www.ibpsguide.com | estore.ibpsguide.com | www.sscexamguide.com


58
Exclusively on New Pattern – Data Interpretation eBook

1/12 + 5/8 + 5/x = 1 – 1/12


Solve, x = 24 = no. of days in which P can alone complete Files 5.
A completed 1/12th work in 2 days, so he can complete whole Files in 2*12 = 24 days
M = 24, N = 20, O = 30, P = 24
So, together they can complete in = 1/24 + 1/20 + 1/30 + 1/24 = 1/6 = 6 days

Set-33

Direction (156-160): Study the table carefully and answer the following questions.

Filling pipe Emptypipe

Pipe Time (in minutes) Pipe Time (in minutes)

M 24 W 60
N 25 X 72
O 48 Y -
P 40 Z -

156). Pipes N and P are to fill the tank. They both are opened. After 8 minutes pipe Y was also opened. It took 3
11/39 minutes more to fill the tank. In how much time pipe Y can empty the tank?
A) 50 minutes B) 28 minutes C) 35 minutes D) 40 minutes

157). Pipes M, O and P are opened to fill a tank. After 10 minutes M and P were closed and O had to fill the
remaining tank. But accidentally at the same time pipe X was also opened which was closed after 3 minutes. Now
O filled the tank. If pipe X was not opened, how much time would have been saved?
A) 6 minute B) 2 minutes C) 8 minutes D) 3 minutes

158). 11/40th of a tank was filled. Pipes O and P were opened. After some time pipe W was also opened. Pipe W
was closed after time equal to the time it was opened at the starting. If after closing pipe W, the tank was filled in
6 minutes, find what fraction of tank was filled by pipe O?
A) 1/8th B)2/5th C) 3/8th D) 3/6th

159). Pipe P was to fill a tank. To increase its efficiency, pipe M and O were opened for 2 minutes each starting
with pipe M, then O and so on. This procedure ran for 16 minutes, after which pipe M and Owere closed.
Accidentally pipe Z was opened and now it took 4 minutes more to fill the tank. Find in how much time pipe Z can
empty 3/40th of tank.
A) 8 minutes B) 2 minutes C) 7 minutes D) 6 minutes

www.ibpsguide.com | estore.ibpsguide.com | www.sscexamguide.com


59
Exclusively on New Pattern – Data Interpretation eBook

160). To fill an empty tank all filling pipes M, N, O and P are opened one by one in same order. M and O are
opened for 4 minutes each while N and P for 5 minutes each. In this way, in how much time the tank will be filled
(approximately)?
A) 30 minutes B) 42 minutes C) 25 minutes D) 12 minutes

SOLUTIONS
156) A
N+P = 1/25 + 1/40 = 13/200
Worked for 8 minutes, so did 13/200 * 8 = 13/25 work
So 200/13 – 8 = 96/13 minutes left to complete work,
But after Y was opened, it took 3 11/39 minutes more, means total 96/13 + 128/39 = 416/39 = 32/3 minutes
So now N+P – Y
13/25 + (13/200 – 1/x) * 32/3 = 1
Solve, x = 50 minutes
157) B
M+O+P = 1/24 + 1/48 + 1/40 = 7/80
Worked for 10 minutes, so filled 7/80 * 10 = 7/8th of tank
Now O and X are opened for 3 minutes — (1/48 – 1/72)* 3 = 1/48th filled now
So 7/8 + 1/48 + 1/48 *x = 1
x = 5 minutes
So total time = 10+3+5 = 18 minutes
If X was not opened::
1/8th of tank was to be filled by O
So 1/48 * u = 1/8,
u=6
So total time = 10+6 = 16 minutes-
So time that would have been saved = 18-16 = 2 minutes
158) C
O+P = 1/48 + 1/40 = 11/240
Let after x minutes W was opened, so W was opened for x minutes [as given] So
11/40 + (1/48 + 1/40) * x + (1/48 + 1/40 – 1/60)*x + (1/48 + 1/40) * 6 = 1
OR 11/40 + (1/48 + 1/40) * 2x – 1/60 * x + (1/48 + 1/40) * 6 = 1
Solve, x = 6 minutes
So O was opened for 2x+6 = 18 minutes
So filled 1/48 * 18 = 3/8 of tank
159) D

www.ibpsguide.com | estore.ibpsguide.com | www.sscexamguide.com


60
Exclusively on New Pattern – Data Interpretation eBook

For 2 minutes M and P


(1/24 + 1/40)* 2= 2/15
For next 2 minutes O and P. So
(1/48 + 1/40)* 2= 11/120
So in 4 minutes — 2/15 + 11/120 = 9/40
Procedure ran for 16 minutes, so multiply by 4 both sides
In 16 minutes, tank filled = 9/40 * 4 = 9/10
Now 1/10 tank left which P can fill in 4 minutes, but with P and Z opened, took 4 more minutes, so 8 minutes
So (1/40 – 1/x) * 8 = 1/10
Solve, x = 80 minutes
Full tank in 80 minutes so 3/40th in 3/40 * 80 = 6 minutes
160) A
M for 4 minutes = 1/24 * 4 = 1/6
N for 5 minutes = 1/25 * 5 = 1/5
O for 4 minutes = 1/48 * 4 = 1/12
P for 5 minutes = 1/40 * 5 = 1/8
So in 18 minutes (4+5+4+5), tank filled is 1/6 + 1/5 + 1/12 + 1/8 = 23/40
Next M’s turn for 4 minutes = 1/6
Next N’s turn for 5 minutes = 1/5
Filled now = 23/40 + 1/6 + 1/5 = 113/120
Now 7/120 is left
Now O’s turn, full tank in 48 minutes, so 7/120 in 48 * 7/120 = 14/5 minutes
So total = 18 + 4 + 5+ 14/5 = 27 + 3 = 30 mins approx

MISSING DATA INTERPRETATION

Set-34

Directions(161-166): Study the table carefully and answer the following questions

Discount C.P. S.P M.P. Profit Loss Profit% Loss% Discount%


M - - - 50000 - - - 20% 40%
N 500 1800 - - - - - - 20%
O - - - - 2000 - 10% - 20%
P - - 25000 40000 - - 25% - -
Q - - 20000 - - - 25% - 20%
R - 10000 - 20000 - - 20% - -

www.ibpsguide.com | estore.ibpsguide.com | www.sscexamguide.com


61
Exclusively on New Pattern – Data Interpretation eBook

161). What is the ratio of the cost price of P and cost price of Q?
A) 5:4 B) 9:4 C) 5:10 D) 6:12
162). Marked Price of O is how much percent more than the cost price of O?
A) 31.5 % B) 37.5% C) 34.0% D) 46.7%
163). What is the ratio between the percentage of profit and percentage of discount of P?
A) 5:4 B) 1:6 C) 2:3 D) 6:5
164). Selling Price of N is what percent of cost price of M?
A) 2.2% B) 8.2% C) 6.3% D) 5.3%
165). What is the amount (in Rupees) of profit /loss of shop by selling all electronic goods?
A) 5700 B) 3700 C) 4600 D) 2200

SOLUTIONS

161) A

Cost Price of P = 25000 * 100/ (100+25) = 20000


Cost Price of Q= 20000 * 100/ (100+25) = 16000
Required Ratio = 20000: 16000 = 5: 4

162) B

Amount of profit on O = 2000


Cost Price = 2000* 100/10 = 20000
Selling Price = 20000 + 2000 = 22000
Marked Price = Selling Price * 100 / (100 – Discount %)
= 22000 * 100 /80 = 27500
Required Percentage = [(27500 -20000) /20000] * 100 = 37.5%

163) C

Percentage of profit on P = 25%


Percentage of discount on P = [40000 – 25000/40000]*100
. = 37.5%
Required Ratio = 25: 37.5
. = 2: 3

164) D

Market Price of N = 500 * 100/20 = 2500


Selling Price of N = 2500 - 500 = 2000
Selling Price of M = 50000 * 60 / 100 = 30000

www.ibpsguide.com | estore.ibpsguide.com | www.sscexamguide.com


62
Exclusively on New Pattern – Data Interpretation eBook

Cost Price of M = 30000 * 100/80 = 37500


Required Percentage = (2000/37500)*100 = 5.3%

165) A

Selling Price of M = 50000 * 60/100 = 30000


Loss on M = 30000 * 100/80 -3000 = Rs.7500
Market Price of N= 500 * 100 /20 = 2500
Selling Price of N = 2500 - 500 = 2000
Profit on N =2000 – 1800 = Rs.200
Profit on O = Rs.2000
Profit on P = 25000 - 25000 * 100/25 = Rs.5000
Profit on Q = 20000 - 20000 * 100 / 125 = Rs.4000
Profit on R = 10,000 * 20 /100 = Rs.2000

Total Profit = -7500 + 200 + 2000 + 5000 + 4000 + 2000 = Rs.5700

Set-35
Direction (166-170): Study the following table & answer the following questions.
Table I: Shown below represent the grades obtained by ten different students in five different subjects
Students Maths English Science History Civics
Anil A A+ B+ B A+
Raju A- A B- B+ A
Chetan A+ A- A- B A-
Dheeraj A- A A+ A+ B+
Vishal A- A A A- B-
Deepak B- A+ A A A+
Hemant B+ A- A- A+ A
Shekhar C B- B C A+
Arun A+ F A A- B+
Vasata B- B B- A- B-

Table II: Represents the range of marks for which a candidate would obtain a specific range. For
example, a candidate scoring 95 marks in any subject will receive an A+ grade in that subject
Grade Range of Marks
A+ 91-100
A 81-90

www.ibpsguide.com | estore.ibpsguide.com | www.sscexamguide.com


63
Exclusively on New Pattern – Data Interpretation eBook

A- 71-80
B+ 61-70
B 51-60
B- 41-50
C 31-40
F 0-30

166). If Anil has scored the minimum possible marks, then find the percentage marks of Anil?
A)75% B)55% C)50% D)69%
167). Find the minimum possible total marks which could have been scored by all the ten candidates in
English.
A) 770 B) 659 C) 750 D) 460

168). Which of the following cannot be the marks scored by Hemant in all the five subjects combined?
A) 220 B) 445 C) 375 D) BOTH (A) AND (B)

169). The total marks scored (in all the five subjects) by how many candidates must be less than 450?
A) 2 B) 4 C) 11 D) 10

170). If Vishal scores more than Chetan, which of the following cannot be the marks scored by Vishal?
A) 345 B) 350 C) 355 D) All of the Above

SOLUTIONS
Note:
L = lower limit
U = upper limit
The lower limit and upper limit of marks for students can be calculated for illustration

Candidates Range of Five Marks(for five Subject Combined)


L=81+91+61+51+91=375
Anil
U= 90+100+70+60+100=420
L=71+81+41+61+81=335
Raju
U=80+90+50+70+90=380
Chetan L=91+71+71+51+71=355

www.ibpsguide.com | estore.ibpsguide.com | www.sscexamguide.com


64
Exclusively on New Pattern – Data Interpretation eBook

U=100+80+80+60+80=400
L=71+81+91+91+61=395
Dheeraj
U=80+90+100+100+70=440
L=71+81+81+71+41=345
Vishal
U=80+90+90+80+50=390
L=41+91+81+81+91=385
Deepak
U=50+100+90+90+100=430
L=61+71+71+91+81=375
Hemant
U=70+80+80+100+90=420
L=31+41+51+31+91=245
Shekhar
U=40+50+60+40+100=290
L=91+0+81+71+61=304
Arun
U=100+30+90+80+70=370
L=41+51+41+71+41=245
Vasata
U=50+60+50+80+50=290

166). A
Total minimum marks of Anil= 375
Total maximum marks = 500
Required % = 375/500*100=75
167). B
Subject: English
L = 2(91) + 3(81) + 2(71) + 1(51) + 1(41) + 1(0) = 182 + 243 + 142 + 51 + 41 + 0 = 659
168). D
Hemant cannot score less than 375 and not more than 420.
169). D
The upper limit of the marks for all the candidates is less than 450. So, all the candidates must have
obtained total marks less than 450.
170). E
Vishal- [345, 390]
Chetan- [355, 400]
Vishal must score more than 355. He cannot score, 345 or 350 or 355.

www.ibpsguide.com | estore.ibpsguide.com | www.sscexamguide.com


65
Exclusively on New Pattern – Data Interpretation eBook

Set-36
Directions (171-175): Refer the following graph & answer the following question.

Percentage of work done by 5 Pepole to


complete the Work

Ramesh Amit
15% 20%

Keshav
10%
Deepak
30%
Chetan
25%

Person Number of Days they Work


Amit 4
Keshav 3
Chetan 6
Deepak 7.5
Ramesh 6

171). Amit and Keshav started doing the work. After 2 days they both left, and Chetan joined the work.
He completed his part of work. Now the remaining work was completed by Vishal in 7 days. In how
many days can Vishal complete whole work?
A) 12 days B) 14 days C) 21 days D) 23 days

172). Kapil who can complete whole work in 30 days replaced Amit and did Amit’s part of work. He left
and then Keshav also worked for same number of days as Kapil. If remaining work was completed by
Atiq who can do complete work in one-fourth the numberof days in which Ramesh can complete the
work, then in how many days was the whole work completed?
A) 2 days B) 18 days C) 11 days D) 8 days

www.ibpsguide.com | estore.ibpsguide.com | www.sscexamguide.com


66
Exclusively on New Pattern – Data Interpretation eBook

173). All people decided to complete work in less number of days. So they divided the work equally
among themselves. In how many days will the work be completed this way? (They all worked
individually)
A) 15 4/8 days B) 17 3/5 days C) 27 4/5 days D) 14 3/7 days

174). Tushar is 20% more efficient than Keshav and Nandu is 60% more efficient than Chetan.
Theyworked together for 5 days and left the work, after which the remaining workwas completed by
Chetan and Ramesh together. Had all worked together, in how many less days they could have
completed the work?
A) 6 days B) 6 3/13 days C) 5 8/5 days D) 16 days

175). Akash can complete the whole work in number of days equal to the average of number of days in
which Amit and Keshav can complete the work. Akash, Chetan, Deepak, and Ramesh all started the
work and after 5 days they were replaced by Amit and Keshav. Amit and Keshavcompleted the
remaining work in how many days?
A) 3 1/5 days B) 3 2/7 days C) 2 8/3 days D) None of These
SOLUTIONS
171) A
Amit does 20% works in 4 days.
100% work in 100*4/20 = 20 days
Keshav can complete 100% work in 100*3/10 = 30 days
They worked for 2 days, so did [1/20 + 1/30]*2 = 1/6 work
Now Chetan completed 25% = 1/4 work
So now remaining work = 1 – (1/6 + 1/4) = 7/12
Vishal complete 7/12 work in 7 days, so complete work in 12 days
172) B
Amit’s part of work = 20% = 1/5
So Kapil did 1/5 of work,
Whole work in 30 days, s 1/5 work in 1/5 * 30 = 6 days
Now Keshav also worked for 6 days.
Keshav can complete whole work in 30 days, so In 6 days, completed 6/30 = 1/5 of work
Now remaining work = 1 – (1/5 + 1/5) = 3/5 [After Kapiland Keshav did work]
Now Ramesh can complete whole work in 40 days [100*6/15]
So Atiq can complete work in 10 days. So completed 3/5 work in 3/5 * 10 = 6 days

www.ibpsguide.com | estore.ibpsguide.com | www.sscexamguide.com


67
Exclusively on New Pattern – Data Interpretation eBook

So total number of days = 6+6+6 = 18 days


173) C
5 people equally divided the work so each did 1/5 work now
So Amit 1/5 work in 4 days as earlier
Keshav = 1/10 work in 3 days, so 1/5 work in 6 days
Chetan = 1/4 work in 6 days, so 1/5 work in 24/5 days
Deepak =3/10 work in 7.5 days, so 1/5 work in 5 days
Ramesh = 3/20 work in 6 days, so 1/5 work in 8 days
So complete work in = 4 + 6 + 24/5 + 5 + 8 = 27.8 days
174) B
Tushar and Keshav = 120: 100 = 6: 5. So days = 5: 6
6 = 30, so 1 = 5, so 5 = 25.
Tuahar can complete work in 25 days
Nandu and Chetan = 160: 100 = 8: 5. So days = 5: 8
8 = 24, so 1 = 3, so 5 = 15.
Nanducan complete work in 15 days
They worked for 5 days. So
[1/25 + 1/15]*5 = 8/15 work
Reaming 7/15 by Chetan and Ramesh
So [1/24 + 1/40]*x = 7/15
x = 7 days
Total = 5+7 = 12 days
Now had they did work together::
[1/25 + 1/15 + 1/24 + 1/40] = 13/75
So 75/13 days
So less days = 12 – 75/13 = 81/13 = 6 3/13 days
175) A
Amit in 20 days, Keshav in 30 days. So Akash in (20+30)/2 = 25 days
Akash, Chetan, Deepak, and Ramesh started work =
[1/25 + 1/24 + 1/25 + 1/40]= 11/75.
Worked for 5 days so did 11/75 * 5 = 11/15 of work
Remaining work = 4/15
[1/20 + 1/130]*x = 4/15
Solve. x = 48/15 days = 3 1/5 days

www.ibpsguide.com | estore.ibpsguide.com | www.sscexamguide.com


68
Exclusively on New Pattern – Data Interpretation eBook

Set-36
Directions (Q.176-180):
A survey was conducted in five cities viz. Assam, Bihar, Gujarat, Haryana and Kerala, for the percentage of
people using O (Oppo), R (Redmi), V(Vivo)different mobiles. Number of persons surveyed in the states Assam,
Bihar, Gujarat, Haryana and Kerala are 2000, 4000, 6000, 3000 and 8000 respectively. Refer to the data in the
triangular bar diagram to answer the questions that follow.

176). Which mobile is used by the least number of persons in all the given states?
a) Oppo
b) Redmi
c) Vivo
d) Vivo and Redmi
e) Oppo and Vivo

177). The state where the least number of persons uses Redmi mobile is
a) Haryana
b) Gujarat
e) Bihar
d) Assam
e) Kerala

178). Which of the following statements is not true?


a) 50% of the people use Oppo mobile in Bihar and Gujarat

www.ibpsguide.com | estore.ibpsguide.com | www.sscexamguide.com


69
Exclusively on New Pattern – Data Interpretation eBook

b) In Kerala, more than 50% of the people use Vivo mobile


c) More percentage of people use Redmi mobile in Haryana than Assam.
d) In Gujarat, there are more percentage of people using Oppo mobile than Redmi mobile
e) Total number of Vivo and Oppo mobile users in Bihar is 3500

179). The average number of persons using Oppo mobile in Assam, Bihar, Gujarat and Kerala is
a) 1980
b) 1750
c) 1950
d) 1525
e) 1875

180). Among the given five states, the states where less than 30% of the people use Vivo mobile
a) Assam and Gujarat
b) Assam, Bihar and Gujarat
c) Bihar and Kerala
d) Assam, Bihar and Haryana
e) Kerala and Gujarat

Solutions:

Redmi Total Oppo Total Vivo Total


percentage number percentage number percentage number
of Redmi of Oppo of Vivo
users users users
Kerala 8000 50 4000 12.5 1000 37.5 3000
Haryana 3000 25 750 25 750 50 1500
Bihar 4000 12.5 500 50 2000 37.5 1500
Gujarat 6000 37.5 2250 50 3000 12.5 750
Assam 2000 12.5 250 75 1500 12.5 250
Total 23000 7750 8250 7000

176. From the table, Vivo are used by 7000 people which is least
Correct Answer is: c)

177. It can be easily observed from table that the required city is Assam.

www.ibpsguide.com | estore.ibpsguide.com | www.sscexamguide.com


70
Exclusively on New Pattern – Data Interpretation eBook

Correct Answer is: d)

178. In Kerala, 37.5% of the people use Vivo for transport.


Option b) is false
Correct Answer is: b)

179. The total number of persons using Oppos for transportation in Assam, Bihar, Gujarat and Kerala
= 1500 + 2000 + 3000 + 1000 = 7500
The average = 7500/4 = 1875.
Correct Answer is: e)

180. It is clear from the graph as well as table that in both Gujarat and Assam, 12.5% of the total people use car
as their mode of transport.
Correct Answer is: a)
Set-37

Directions (Q.181-185): Study the following pie charts to answer the questions given below:
The pie chart shows percentage of distance of various bikes and another pie chart shows percentage of the
speed of various bikes in km/hr

Total Distance

Hero
Yamaha 15%
20%

Honda
Suzuki 20%
17%

TVS
Bajaj
13%
15%

www.ibpsguide.com | estore.ibpsguide.com | www.sscexamguide.com


71
Exclusively on New Pattern – Data Interpretation eBook

Speed in percentage
Total speed = 120 km

Hero
Yamaha 20%
25%

Honda
10%

Suzuki
20% TVS
15%
Bajaj
10%

181. If the Yamaha bike’s time taken is 4 hr, how much time taken by Honda bike?
a) 10 hr
b) 7.5 hr
c) 8 hr
d) 6.5 hr
e) None of these

182. The total time taken by Yamaha bike is 2 hr/km, how much time taken by TVS bike and Hero bike?
a) 3.235 hr
b) 4.035 hr
c) 5.145 hr
d) 7.375 hr
e) None of these

183. If the Honda bike distance is 400km, what is the total time taken by TVS bike, Hero bike, Bajaj bike, Suzuki
bike?
a) 60 hour 30 min
b) 71 hour 50 min
c) 77 hour 40 min
d) 54 hour 10 min
e) None of these

www.ibpsguide.com | estore.ibpsguide.com | www.sscexamguide.com


72
Exclusively on New Pattern – Data Interpretation eBook

184. If TVS bike distance is 78km then what is the difference between the time taken by Bajaj bike and Hero
bike?
a) 4.25 hr
b) 3.50 hr
c) 4.75 hr
d) 3.75 hr
e) None of these

185. The difference between the distance of Bajaj bike and Suzuki bike is 40km, How many more hour taken by
Bajaj bike than Suzuki bike?
a) 5 hour 26 minutes
b) 7 hour 25 minutes
c) 10 hour 50 minutes
d) 9 hour 55 minutes
e) None of these

Solutions
181. Yamaha bike speed = 120×25/100=30 km/hr.
Yamaha bike distance = speed × time = 30 × 4 = 120km.
Yamaha bike distance 20% = 120 km
Honda bike distance =120/20×20 = 120 km
Speed of Honda bike=120×10/100=12kmph
Honda bike time =120/12= 10 hr
Correct Answer is: a)

182. Yamaha bike speed = 120×25/100=30 km/hr.


Distance = speed × time = 30× 2 = 60km.
Yamaha bike distance 20% = 60 km
TVS bike distance =60/20×13= 39 km
TVS bike time =39/18= 2.16 hr
Similarly,
Hero bike distance =60/20×15 =45 km
Hero bike time =45/24= 1.875 hr
Total time =2.16 + 1.875=4.035hr
Correct Answer is: b)

183. Honda bike distance 20% = 400km


TVS bike distance =400/20×13= 260 km

www.ibpsguide.com | estore.ibpsguide.com | www.sscexamguide.com


73
Exclusively on New Pattern – Data Interpretation eBook

Speed of TVS bike =120×15/100=18 km/hr


TVS bike time =260/18=14.44 hour
Hero bike distance =400/20×15= 300 km
Speed of Hero bike =120×20/100=24 kmph
Hero bike time =300/24=12.5 hour
Bajaj bike distance =400/20×15= 300 km
Speed of Bajaj bike =120×10/100=12kmph
Bajaj bike time =300/12=25 hour
Suzuki bike distance =400/20×17= 340 km
Speed of Suzuki bike =120×20/100=24 kmph
Suzuki bike time =340/24=14.16 hour
Total time = (14.44 + 12.5 + 25 + 14.16) = 66 hour 6 minutes
Correct Answer is: e)

184. TVS bike distance 13% = 78 km


Hero bike distance =78/13×15 = 90 km
Speed of A=120×20/100=24 kmph
Then Hero bike time =90/24= 3.75hr
Similarly,
Bajaj bike distance =78/13×15 = 90 km
Speed of D=120×10/100=12 kmph
Then Bajaj bike time =90/12= 7.5hr
Required time difference = 7.5 – 3.75
= 3.75 hr
Correct Answer is: d)

185. Difference between Bajaj bike and Suzuki bike is 2% (i.e., 2% = 40)
Bajaj bike distance =40/2×15= 300 km
Speed of Bajaj bike = 120×10/100 = 12 kmph
Bajaj bike Time =300/12=25 hr
Similarly,
Suzuki bike distance = 40/2×17= 340 km
Speed of Suzuki bike = 120×20/100=24 kmph
Suzuki bike Time =340/24=14.16 hr
Hence, Bajaj bike has taken 10 hour 50 minutes more than Suzuki bike.
Correct Answer is: c)

www.ibpsguide.com | estore.ibpsguide.com | www.sscexamguide.com


74
Exclusively on New Pattern – Data Interpretation eBook

Set-38
Directions (Q.186-190): Study the following table to answer the questions given below:

Year Ratio of investment Period of investment Total profit


Dinesh : Mahesh : Dinesh Mahesh Suresh
Suresh
2006 5:4:- 3 5 5 -
2007 -:-:4 - - 4 22500
2008 3:-:5 4 7 6 14700
2009 4:3:5 4 6 - 20800
2010 -:3:- 8 5 6 39650

186. The total investment of Dinesh, Mahesh and Suresh in 2008 is Rs 16,800. If difference in shares of profit of
Mahesh and Dinesh is Rs 5250, find the investment of Mahesh.
a) 6800
b) 7200
c) 5400
d) 3600
e) 8400

187. In 2009, if out of the total profit, Suresh got Rs 9750 as his share, what is the number of months for which
Suresh invested his money?
a) 4 months
b) 9 months
c) 6 months
d) 2 months
e) 3 months

188. In 2007, Dinesh and Mahesh invested Rs 3900 and Rs 2600 respectively. Also Dinesh and Mahesh invested
for same number of months. If difference in the shares of Dinesh and Mahesh out of total profit is Rs 2750, find
the ratio of number of months of investment of Dinesh to investment of Suresh.
a) 3 : 650
b) 2 : 270
c) 1 : 1040
d) 4 : 170
e) 1 : 740

www.ibpsguide.com | estore.ibpsguide.com | www.sscexamguide.com


75
Exclusively on New Pattern – Data Interpretation eBook

189. In 2010, if out of the total profit, Dinesh and Suresh got a share of Rs 10,400 and Rs 19,500 respectively,
what is the ratio of investment of Dinesh, Mahesh and Suresh respectively?
a) 3 : 4 : 5
b) 4 : 1 : 5
c) 4 : 3 : 5
d) 2 : 3 : 5
e) 1 : 5 : 3

190. In 2006, out of the total profit, Mahesh got Rs 12,000 as his share. Suresh invested Rs 3600. If ratio of share
of Suresh in total profit to total profit is 3 : 10, find the total profit.
a) 30000
b) 25000
c) 35000
d) 60000
e) 40000

Solutions
186. Ratio of share of profit of Dinesh : Mahesh : Suresh is
3×4 : x×7 : 5×6
12 : 7x : 30
So (7x-12)/(7x+42) × 14700 = 5250
(7x-12)×14 = (7x+42)×5
98x – 168 = 35x + 210
Solve, x = 6
So ratio of investments of Dinesh : Mahesh : Suresh is 3 : x : 5 = 3 : 6 : 5
So investment of Mahesh = 6/(3+6+5) × 16800
= 6/14 × 16800
= Rs 7200
Correct Answer is: b)

187. Ratio of share of profit of Dinesh : Mahesh : Suresh is


4×4 : 3×6 : 5×x
16 : 18 : 5x
So 5x/(34+5x) × 20800 = 9750
Solve, x = 6 months
Correct Answer is: c)

www.ibpsguide.com | estore.ibpsguide.com | www.sscexamguide.com


76
Exclusively on New Pattern – Data Interpretation eBook

188. Let investment of R is 4x, and months of investment of Dinesh = Mahesh = y months
So ratio of profit share of Dinesh : Mahesh : Suresh is
3900× y : 2600×y = 4x × 4
975y : 650y : 4x
So (975y-650y)/(975y+650y+4x) × 22550 = 2750
325y/(1625y+4x) = 2750/22550
1625y + 4x = 2665y
4x = 1040y
So y/4x = 1/1040
= 1 : 1040
Correct Answer is: c)

189. x×8 : 3×5 : y×6


8x : 15 : 6y
So 8x/(8x+15+6y) × 39650 = 10400
Solve, 225x – 60y = 150
15x – 4y = 10 is equ (1)
Also
6y/(8x+15+6y) × 39650 = 19500
Solve, 61y = 75 + 40x + 30y
40x – 31y = -75 is equ (2)
Solve equations (1) and (2)
x = 2, y = 5
So ratio of investment is x : 3 : y = 2 : 3 : 5
Correct Answer is: d)

190. In 2006, Dinesh invested Rs 5x, Mahesh – 4x and Suresh 3600


So ratio of share of profits of Dinesh, Mahesh and Suresh:
5x×3: 4x×5 : 3600×5
3x : 4x : 3600
Let total profit = Rs y
So 4x/(7x+3600) × y = 12000 is equ (1)
xy/(7x+3600) = 3000
Also,[3600/(7x+3600) × y]/y = 3/10
Solve, x = 1200
Put in equ (1)
Solve, y = Rs 30000 = total profit
Correct Answer is: a)

www.ibpsguide.com | estore.ibpsguide.com | www.sscexamguide.com


77
Exclusively on New Pattern – Data Interpretation eBook

Set-39

Directions (Q.191-195): Study the following bar graph to answer the questions given below:

Speed of boats (in km/hr)


25

20 20

16
15 15

12
10
9

0
Red Blue Green Yellow Orange

191. Upstream speed of Orange boat is 9 km/hr. How many more hours will it take to cover a distance of 315 km
upstream than same distance downstream?
a) 18 hours
b) 25 hours
c) 22 hours
d) 20 hours
e) 15 hours

192. On a particular day, the time taken by Red boat to cover 255 km downstream is 2 hours more than the time
taken by Yellow boat to cover 195 km upstream. Find the ratio of upstream speed to downstream speed in case
of Red boat on that particular day.
a) 4 : 11
b) 3 : 5
c) 15 : 22
d) 8 : 19
e) 7 : 17

www.ibpsguide.com | estore.ibpsguide.com | www.sscexamguide.com


78
Exclusively on New Pattern – Data Interpretation eBook

193. On a particular day, ratio of upstream speed to downstream speed of Yellow boat is 3 : 7. It took 20 hours
more to cover a distance upstream than same distance downstream by Yellow boat. On that particular day, Blue
boat covered same distance in how much time?
a) 17.5 hours
b) 15 hours
c) 19 hours
d) 19.5 hours
e) 18.5 hours

194. Blue boat covered 756 km downstream on Wednesday for which it took 6 hours less than that in which it
covered half distance upstream on Thursday. On Thursday, speed of stream was 2 km/hr more than that on
Wednesday. Find the downstream speed of Blue boat on Thursday.
a) 21 km/hr
b) 19 km/hr
c) 23 km/hr
d) 11 km/hr
e) 15 km/hr

195. For Green boat, its upstream speed is 6 km/hr on a particular day. Find the difference in time in covering 360
km by Red boat and Green boat on that particular day.
a) 8 hours
b) 6 hours
c) 4 hours
d) 7 hours
e) 5 hours

Solutions
191. Upstream speed = 9, speed of boat = 15 km/hr
Speed of stream = 15-9 = 6 km/hr
Downstream speed = 15+6 = 21 km/hr
So required time = 315/9 – 315/21 = 20 hours
Correct Answer is: d)

192. Let x km/hr is the speed of stream on that particular day.


So 255/(12+x) = 195/(20-x) + 2
Solve, x = 5 km/hr
So in case of Red boat, ratio of speed upstream: downstream = (12-5) : (12+5)
= 7 : 17

www.ibpsguide.com | estore.ibpsguide.com | www.sscexamguide.com


79
Exclusively on New Pattern – Data Interpretation eBook

Correct Answer is: e)

193. Let speed of stream on that day = x km/hr


So (20-x)/(20+x) = 3/7
Solve, x = 8 km/hr
So y/(20-8) – y/(20+8) = 20
Solve, y = 420 km
So required time = 420/(16+8) = 420/24 = 17.5 hours
Correct Answer is: a)

194. Let on Wednesday, speed of stream is x km/hr, then on Thursday it is (x+2) km/hr
On Wednesday it covered 756 km, so on Thursday it covered 756/2 = 378 km
So 756/(16+x) = 378/(16-(x+2)) – 6 equ (1)
126/(16+x) = 63/(14-x) – 1
Solve, x = 5 km/hr
So speed of stream on Thursday = (5+2) = 7 km/hr
So downstream speed of Blue boat = (16+7) = 23 km/hr
Correct Answer is: c)

195. Upstream speed of Green boat = 6 km/hr, so speed of stream: 9 – x = 6, x = 3 km/hr


Downstream speed of Green boat = 9+3 = 12 km/hr
Downstream speed of Red boat = 12+3 = 15 km/hr
So difference in timings = 360/12 – 360/15 = 30 – 24 = 6 hours
Correct Answer is: b)

Set-40

Directions (Q.196-200): Study the following graphs to answer the questions given below:
The bar graph shows the number of days taken by six boys to complete a work

www.ibpsguide.com | estore.ibpsguide.com | www.sscexamguide.com


80
Exclusively on New Pattern – Data Interpretation eBook

16

14

12

10

0
A B C D E F

Number of days taken by boys

The line graph shows the number of days taken by six girls to complete a work

30

25
24 24

20 20

15 15

10 10
9

0
P Q R S T U

Number of days taken by girls

196. If all the boys work together then time taken by them will be how much less than the time taken by P, R, Q
and S together to do the work ?
a) 9/5 days
b) 13/5 days
c) 11/5 days
www.ibpsguide.com | estore.ibpsguide.com | www.sscexamguide.com
81
Exclusively on New Pattern – Data Interpretation eBook

d) 14/5 days
e) 17/5 days

197. B started working alone at his normal efficiency but after 5 days of starting he found that he has done only
25% work so to complete the remaining work on time by how much percentage he should increase his efficiency?
a) 12.5%
b) 15%
c) 10%
d) 20%
e) 12.5%

198. If D and F started to work and after 2 days they were replaced by Q, T and S then they stopped, how much
work is still left? (Q, T and S worked for 2 days together)
a) 1/8
b) 1/6
c) 1/4
d) 1/10
e) 1/5

199. E and C started work. E worked at 120% of his efficiency and C at 5/6 of his normal routine work. They
started work on alternate days starting with E. How many days will be taken by them to complete the work?
a) 65/6 days
b) 73/6 days
c) 80/6 days
d) 58/6 days
e) 54/7 days

200. How many days will it take to complete the work if A does the 25% of the work alone and 10% is done by C
alone and the remaining is done by U?
a) 21 days
b) 18 days
c) 19 days
d) 17 days
e) 25 days

Solutions
196. Total days taken by boys to do the work = 180/100 = 9/5 days
Days taken by 4 girls = 180/45 = 4 days …

www.ibpsguide.com | estore.ibpsguide.com | www.sscexamguide.com


82
Exclusively on New Pattern – Data Interpretation eBook

Difference = 4-9/5
= 11/5 days
Correct Answer is: c)

197. B’s 1 day’s work 180/15 = 12 units


In 5 days = 5 × 12 = 60
But he did180/4 = 45 units
So remaining work = 135 units
= 135/10 = 13.5 units per day
Therefore increased efficiency = 13.5 – 12 = 1.5
= 1.5×100/12
= 12.5%
Correct Answer is: e)

198. Let work = 180 units


D’s 1 day’s work = 20 (180/20) (work/number of days taken)
F’s 1 day’s work = 10 (180/20)
Q’s 1 day’s work = 7.5 (180/24)
S’s 1 days work = 7.5 (180/24)
T’s 1 day’s work = 20 (180/9)
F and D worked for 2 days = 20+20 × 2 = 8
Remaining work = 100
For 2 days = 7.5+7.5+20 × 2 = 70
Remaining work = 30
= 30/180
= 1/6
Correct Answer is: b)

199. E’s one day’s work = 180/12 = 15 units


C’s one day’s work = 180/10 = 18 units
E works at 120% of 15 = 18 units
C at 5/6 of 18 = 15 units
(E + C)’s 2 days’ work = 33
In 10 days = 33 × 5 = 165 units
Remaining 15 units E done in 5/6 days
= 10× 5/6 days
= 65/6 days
Correct Answer is: a)

www.ibpsguide.com | estore.ibpsguide.com | www.sscexamguide.com


83
Exclusively on New Pattern – Data Interpretation eBook

200. A’s 1 day’s work = 15 units (180/12)


So to do 25% work (assumed total work = 180) = 45
= 45/15 = 3 days
C 1 day’s work = 18 units (180/10)
To do 10% work (assumed total work = 180) = 18
= 18/18 = 1 day
Remaining work = 180-63(45+18) = 117
U will take 117/9 = 13 days
U’s 1 day’s work = 180/20 = 9 units
Total days = 3+1+13 = 17 days
Correct Answer is: d)
Set-41

Directions(201-205): Study the graph carefully and answer the following question.
Data related to Income (In Rs thousand) and Expenditure (in Rs thousand) of company during six year.
800
740
700
660
640
600 620
560
500 520
480
440
400 400 Income
380 380
340 Expenditure
300

200

100

0
1991 1992 1993 1994 1995 1996

201). What is average profit earned by company in 1991, 1992 and 1995?
A)240 B)620 C)350 D)400

201). What is total loss percentage incurred by company in 1993 and 1994?
A)10 B)40 C)30 D)25

203). Profit earned by company in 1992 is by what percent more than profit earned by company in 1995.
A)20 3/9 B)38 8/9 C)44 4/9 D)34 4/9

www.ibpsguide.com | estore.ibpsguide.com | www.sscexamguide.com


84
Exclusively on New Pattern – Data Interpretation eBook

204). In which year Company earned the maximum profit.


1) 1992 B) 1990 C) 1998 D) 1991

205). If company income increased by 20% from 1996 to 1997 and Expenditure decrease by 10%.What was his
profit percent in 1997?
A) 2.56 B) 6.67 C)5.17 D) 7.06

SOLUTIONS
NOTE:
Profit = Income – Expenditure
Loss = Expenditure – Income
Profit% = (Income – Expenditure)/Expenditure*100
Loss% = (Expenditure – Income)/Expenditure*1000

201) A
(280+260+180)/3 = 240
202) B
640+560= 1200 ---- 100%
260+120=480---? (40)
Answer is 40%
203) C
260-180/180 = 4/9 = 44.44
204) D
Solve by visual inspection
As profit is difference between income and Expenditure
The magnitude of Gap between income and expenditure whichever is more is the Answer.
Answer is 1991
205) A
Income
400----100%
?(480)------ 120%
Expenditure:
520-----100%
? (468)–---- 90%
Profit % in 1997 = 468------100%
12----- ?
= (2.56%)

www.ibpsguide.com | estore.ibpsguide.com | www.sscexamguide.com


85
Exclusively on New Pattern – Data Interpretation eBook

Set-42

Directions(206-210): Study the following graph and answer the questions below.
Table shows the population of three cities in monthJAN and FEBand percentage increase/decrease with respect
to population of January.
City Month Population Percentage
(Increase/Decrease)
AKOLA JAN - 10% Increase
FEB 1.5 -
AMRAVATI JAN 3 -
FEB 4 -
HINGOLI JAN - 8% Decrease
FEB - -

206) What was the population (in Lakhs) of AKOLA in JAN?


A) 1.36 B)3.6 C)1.5 D)2.10

207) The population of HINGOLI in FEB was same as that of AKOLA in JAN, then find average of total population
of three cities over JAN and FEB?
A) 1.48 Lakhs B) 2.11 Lakhs C) 2.31 Lakhs D) 1.25 Lakhs

208) What is the percentage increase in the population of AMRAVATI from JAN to FEB?
A) 26 1/3 % B)43 2/3 % C)33 1/3 % D) 36 2/3 %

209) If in AKOLA, population increases by 10% every year, then what was the population (approximately) of
AKOLA in MAY?
A) 3.5 Lakhs B) 1.95 Lakhs C) 1.25 Lakhs D) 2 Lakhs

210) The population of AKOLA in JAN is what percent less than population of AMRAVATI in FEB?
A) 66% B) 35% C) 12% D)44%

SOLUTION
206) A
Population of AKOLA in JAN = 1.5 * 100/110 = 1.36 Lakhs
207) B

www.ibpsguide.com | estore.ibpsguide.com | www.sscexamguide.com


86
Exclusively on New Pattern – Data Interpretation eBook

Population of HINGOLI in JAN = 1.36 * 100/92 = 1.48


Average = (1.36+1.50+3+4+1.48+1.36)/6=12.7/6=2.11 Lakhs
208) C
Percentage increase = 4-3/3*100 = 33.33
209) D
Population of AKOA in MAY = 1.5* (110/100)*(110/100)*(110/100)=1.99=2 Lakhs
210) A
Required percentage = (4-1.36)/4*100= (2.64/4)*100=66%
Set-43

Directions (211-215): Refer the following table & answer the following questions.
Total Distance = 1600km
Name Distance [Values (in Degree)]
Nikhil 45
Vishal -
Pratik -
Prashant -
Kapil 108

Name Speed(km/hr) Time(hr)


Nikhil - 5
Vishal - 9
Pratik 32 10
Prashant 60 -
Kapil - 20

211). The speed of Nikhiland Vishal are same, then find the distance travelled by Vishal?
A) 360 km B) 110 km C)260 km D) 282 km

212). Find the distance travelled by Prashant?


A) 140 km B) 240 km C) 340 km D)Cannot be determined

213). Speed of Kapilis how much percent more or less than the speed of Vishal?
A) 15% less B) 50% less C) 40% less D) 30 % less

214). Average of all the person’s speed is


A) 19 5/4 km/hr B) 23 1/5 km/hr C)39 1/5 km/hr D) 30 1/5 km/hr

www.ibpsguide.com | estore.ibpsguide.com | www.sscexamguide.com


87
Exclusively on New Pattern – Data Interpretation eBook

215). What is the ratio of angle made by Pratikand Prashant?


A) 4:3 B) 9:14 C) 5:2 D) 6:4

SOLUTIONS
211) A
Distance travelled by Nikhil = 45/360*1600 = 200km
Speed of Nikhil = 200/5 = 40km/hr
Distance travelled by Vishal = 40*9 = 360Km

212) B
Corresponding angle made by Vishal = 360/1600*360 = 81
Angle by Pratik = 320/1600*360
= 72
Prashant Angle = 360-(72+81+45+108) = 54
Distance travelled by Prashant = 54/360*1600 = 240km
213) C
Speed of Kapil = 480/20 = 24Km/hr
Speed of Vishal = 40km/hr
Required Percentage = 16/40*100
= 40% less
214) C
Avg Speed = 40+40+24+32+60/5
= 39 1/5 Km/hr
215) A
Angle made by Pratik/Angle made byPrashant = 72/54
= 4:3
Set-44
Directions (216-220): Refer the following graph & answer the following questions.

www.ibpsguide.com | estore.ibpsguide.com | www.sscexamguide.com


88
Exclusively on New Pattern – Data Interpretation eBook

Total Upstream Distance = 1500km

JUL JAN
15% 18%

JUN
10%
FEB
15%
MAY
14%
MAR
APR
16%
12%

Toatal Downstream Distance = 1200km

JUL JAN
15% 14%

FEB
JUN 10%
15%

MAR
18%
MAY
16%
APR
12%

Month Speed Of Stream


JAN -
FEB 2
MAR -
APR 3
MAY -
JUN 4
JUL 2
216). If the time taken by boat to travel upstream in JUL is equal to the time taken by it to travel
downstream in APR and the speed of boat in still water in APR is 13 kmph then find the speed of boat in
still water in JUL?

www.ibpsguide.com | estore.ibpsguide.com | www.sscexamguide.com


89
Exclusively on New Pattern – Data Interpretation eBook

A) 27KMPH B) 17KMPH C) 26KMPH D) 32KMPH


217). If the time taken by boat to travel upstream in JAN is 22.56HRS more than the time taken by it to
travel downstream on the same day, then find the speed of boat in still stream in JAN if speed of boat in
still water is 10 KMPH.
A) 2.4KMPH B) 2.5KMPH C) 3KMPH D) 3.8KMPH
218). If the speed of boat in still water in May was 22 KM/HR and the speed of boat in still water in FEB
was 10/11 times of MAY and time taken to travel downstream inMay is 20/11 HRS more than the time
taken by it to travel downstream inFEB, then find the speed of stream (in KMPH) in MAY?
A) 2.5KMPH B) 6.5KMPH C) 4.4KMPH D) NONE OF THESE
219). The speed of boat in still water in JUN was 20 KM/HR. if the time taken by boat to travel
downstream is 25/32 times taken to travel upstream on JUL, then find the time taken by the boat to cover
a distance of 21.25 KM upstream when the speed of stream is same as that of JUL.
A) 3HRS B) 4HRS C) 1HRS D) 1.82HRS
220). If the time taken by boat to travel upstream in JUL is 6 hours more than the time taken by it to
travel downstream in APR and the speed of boat in still water in APR is 13 KMPH, then find the speed of
boat in still water in JUL?
A) 17KMPH B) 12KMPH C) 14KMPH D) 10KMPH
SOLUTIONS
216) A
MONTH UPSTREM DOWNSTREAM STREAM SPEED
DISTANCE DISTANCE
JAN 15*15 15*12 2
FEB 18*15 14*12 -
MAR 15*15 10*12 2
APR 16*15 18*12 -
MAY 12*15 12*12 3
JUN 14*15 16*12 -
JUL 10*15 15*12 4

15*15/(b-s) = 12*12/(b2-s)
15*15/(b-2) = 12*12/(13+3)
b= 27KMPH
217) B
18*15/(10-S)- 14*12/(10+S) = 22.56
S=2.5
218) C
Speed of boat in MAY = 22kmph

www.ibpsguide.com | estore.ibpsguide.com | www.sscexamguide.com


90
Exclusively on New Pattern – Data Interpretation eBook

Speed of boat in FEB = 22*10/11kmph = 20kmph


16*12/(22+S)-10*12/(2+20) = 20/11
S=4.4KMPH
219) D
Speed of boat in JUN = 20kmph
Time taken by boat in JUN= 12*15/(20-4) = 25/32*15*15/(b+2)
b=13.625KM
t = 21.25/(13.625-2) = 1.82 HRS
220) A
15*15/(b-2)- 12*12/(13+3) = 6
b = 17KMPH
Set-45

Directions (221-225): Study the following data carefully and answer the questions.

In a MBBS college there are 1600 students studying Dentistry and Homeopathy. Each student from each course
knows one or more languages out of English, Hindi and Bengali. 45% of the students study Dentistry and the
remaining students study Homeopathy. Out of the students studying Dentistry, boys and girls are in the ratio 5:3.
Out of the boys studying Dentistry, 16% know only English, 10% know only Hindi and 4% know only Bengali. 24%

know English as well as Hindi, 20% know English as well as Bengali and 14% know Hindi as well as Bengali. The
remaining boys know all the three languages.

Out of the girls studying Dentistry, 20% know only English, 10% know only Hindi and 10% know only Bengali.

20% know English as well as Hindi, 20% know English as well as Bengali and 10% know Hindi as well as Bengali.

The remaining girls know all the three languages.

Out of the students studying Homeopathy, boys and girls are in the ratio 4:7. Out of the boys studying
Homeopathy, 20% know only English, 15% know only Hindi and 5% know only Bengali. 15% know English as well
as Hindi, 25% know English as well as Bengali and 10% know Hindi as well as Bengali. The remaining boys know

all the three languages.

Out of the girls studying Homeopathy, 15% know only English, 15% know only Hindi and 5% know only Bengali.
20% know English as well as Hindi, 20% know English as well as Bengali and 15% know Hindi as well as Bengali.
The remaining girls know all the three languages.

221). How many students studying Dentistry know only either English or Hindi?
A) 298 B) 698 C) 198 D) 398

www.ibpsguide.com | estore.ibpsguide.com | www.sscexamguide.com


91
Exclusively on New Pattern – Data Interpretation eBook

222). How many students in the college know all the three languages?
A) 588 B) 169 C) 369 D) 100

223) What percent of the total number of girls in the college know Bengali?
A) 33% B) 40% C) 50% D) 80%

224) How many students studying Homeopathy do not know English?


A) 192 B) 669 C) 458 D) 292

225) Out of the students studying Homeopathy, what is the ratio of the number of boys knowing English to the
number of girls knowing Hindi?
A) 2:3 B) 9:7 C) 6:7 D) 8:5

SOLUTIONS

Total number of students=1600.


Number of students in Dentistry= 45% of 1600= 720.
Number of students in Homeopathy= 1600-720=880.
Ratio of boys and girls studying Dentistry= 5:3.
Number of boys studying Dentistry= (5/8) * 720= 450.
Number of girls studying Dentistry= (3/8) * 720= 270.
Ratio of boys and girls studying Homeopathy= 4:7.
Number of boys studying Homeopathy= (4/11) * 880= 320.
Number of girls studying Homeopathy= (7/11) * 880= 560.

221) C
16% of 450 + 10% of 450= 117.
20% of 270 + 10% of 270= 81.
On adding these two, we get 198.
222) B
12% of 450 + 10% (320+560+270) =169.
223) C
[(50% of 270 + 50% of 560) /(270 +560)] *100=50%.
224) D
30% of 320 + 35% of 560= 292.
225) A
(70% of 320)/(60% of 560)= 2:3

www.ibpsguide.com | estore.ibpsguide.com | www.sscexamguide.com


92
Exclusively on New Pattern – Data Interpretation eBook

Set-46

Directions (226-230): Study the table & answer the following questions
Name Total Salary Expenditure Saving
Nikhil 45000 37500 -
Anurag 38000 29500 -
Nishant 27000 - -
Gaurav - - 4200
Ayush - 22000 -
Kapil 32000 - 5500
226). If Ayush Spends 88% of his salary, What is ratio between his total salary and savings ?
A) 14:3 B) 33:8 C) 36:3 D) 25:3
227). Kapil spends 20% of his Expenditure on education. Find what amount he spends on Education?
A) 6,500 B) 4,600 C) 5,300 D) 8,400
228). Nishant saves 12% of his monthly salary, and he spends 10% of expenditure on House rent. How much
amount he spent on house rent?
A) 1,475 B) 2,376 C) 5,377 D) 1,378
229). What is the average salary of Nikhil, Anurag, Gaurav and Kapil persons, If Gaurav 's expenditure is 25,800
?
A) 26,500 B) 36,250 C) 46,750 D) 16,000
230). Nikhil's salary is increased 20% and his expenditure also increased 10% . Find the difference between his
new savings and present savings?
A) 5,000 B) 5,250 C) 5,500 D) 5,750

SOLUTIONS
226) D
88% ----- 22,000
100% ------ ?
Total salary = 25,000
Savings = 25,000-22,000 = 3,000
Ratio = 25,000 : 3,000 = 25:3
227) C
Expenditure = 32,000 - 5,500 = 26,500
100% ---- 26,500
20% ---- ?
For Education he spends = 5,300
228) B

www.ibpsguide.com | estore.ibpsguide.com | www.sscexamguide.com


93
Exclusively on New Pattern – Data Interpretation eBook

Nishant salary = 27,000


Nishant expenditure = 27,000*88/100 = 23,760
Rent = 23,760*10/100 = 2,376
229) B
Nikhil's salary = 45,000
Anurag’s salary = 38,000
Gaurav’s salary = 25,800 + 4,200 = 30,000
Kapil's salary = 32,000
Average = (45,000+38,000+30,000+32,000)/4 = 36,250
230) B
Nikhil's Savings = 45,000 - 37,500 = 7500
After increasing his salary become = 120*45000/100 = 54,000
Expenditure = 37,500*110/100 = 41,250
Nikhil's new savings = 54,000 - 41,250 = 12,750
Difference = 12,750 - 7,500 = 5,250

Set-47

Direction (231-235): Study the given graph carefully and answer the following questions.
The Pie Chart 1 Shows Percentage of Distance of Various Train.
The Pie Chart 2 Shows Percentage of the Speed of Various Trains in Km/Hr. Speed of the train is 120 km/hr.

Total Distance
Train M Train N Train O Train P Train Q Train R

15%
20%

20%
17%

15% 13%

www.ibpsguide.com | estore.ibpsguide.com | www.sscexamguide.com


94
Exclusively on New Pattern – Data Interpretation eBook

Speed in Percentage
Train M Train N Train O Train P Train Q Train R

20%
25%

10%

20%
15%
10%

231). The total time taken by Train R is 2hr, how much time taken by Train O and Train M?
A) 4.035 HR B) 2 HR C) 5 HR D) 10 HR
232). If Train O distance is 78km then what is the difference between the time taken by Train P and Train M?
A) 2.2 HR B) 1.8 HR C) 4 HR D) 3.75 HR MINUTES
233). The difference between the distance of Train P and Train Q is 40km, How many more hour taken by Train P
than Train Q?
A) 4 HOUR 36 MINUTES B) 6 HOUR 25 MINUTES C) 10 HOUR 50 MINUTES D) 15 HOUR 40
MINUTES
234). If the Train N distance is 400km, what is the total time taken by Train O, M, P, Q?
A) 66 hour 3 min B) 72 hour 5 min C) 75 hour D) 66 hour 6 min
235). If the Train R’s time taken is 2 hr, how much time taken by Train N?
A) 1.8 HR B) 2.5 HR C) 2 HR D) 5 HR
SOLUTIONS
231). A

Train R speed = 120×25/100=30 km/hr.


Distance = speed × time = 30× 2 = 60km.
Train R distance 20% = 60 km
Train O distance =60/20×13= 39 km
Train O time =39/18= 2.16 hr
Similarly,
Train M distance =60/20×15 =45 km
Train M time =45/24= 1.875 hr

www.ibpsguide.com | estore.ibpsguide.com | www.sscexamguide.com


95
Exclusively on New Pattern – Data Interpretation eBook

Total time =2.16+1.875=4.035hr


232). D
Train O distance 13% = 78 km
Train M distance =78/13×15 = 90 km
Speed of Train M =120×20/100=24 kmph
Then Train M time =90/24= 3.75hr
Similarly,
Train P distance =78/13×15 = 90 km
Speed of Train P =120×10/100=12 kmph
Then Train P time =90/12= 7.5hr
Required time difference = 7.5 – 3.75 = 3.75 hr
233). C
Difference between Train P and bus E is 2% (i.e., 2% = 40)
Train P distance =40/2×15= 300 km
Speed of Train P =120×10/100=12 kmph
Train P Time =300/12=25 hr
Similarly,
Train Q distance =40/2×17= 340 km
Speed of Train Q =120×20/100=24 kmph
Train Q Time =340/24=14.16 hr
Hence, Train P has taken 10 hour 50 minutes more than Train Q.
234). D
Train N distance 20% = 400km
Train O distance =400/20×13= 260 km
Speed of Train O =120×15/100=18 hr
Train O time =260/18=14.44 hour
Train M distance =400/20×15= 300 km
Speed of Train M =120×20/100=24 kmph
Train M time =300/24=12.5 hour
Train P distance =400/20×15= 300 km
Speed of Train P =120×10/100=12kmph
Train P time =300/12=25 hour
Train Q distance =400/20×17= 340 km
Speed of Train Q =120×20/100=24 kmph
Train Q time =340/24=14.16 hour
Total time = (14.44+12.5+25+14.16) =66hour 6 minutes
235). D
Train R speed = 120×25/100=30 km/hr.

www.ibpsguide.com | estore.ibpsguide.com | www.sscexamguide.com


96
Exclusively on New Pattern – Data Interpretation eBook

Train R distance = speed × time = 30 × 2 = 60km.


Train R distance 20% = 60 km
Train N distance =60/20×20 = 60 km
Speed of Train N =120×10/100=12kmph
Train N time =60/12= 5hr

Set-48

Direction (236-240): Study the given pie-charts carefully and answer the questions given below.
The pie-charts show the major expenses in agriculture under different heads in year 2000-01 and 2010-11.

Total expenditure = ` 15432 crore


Fertilizer Seed Electricity & Diesel Others Feed

17.5%
29.5%

18.2%

27%
7.8%

Total expenditure = ` 35349 crore


Fertilizer Seed Electricity & Diesel Others Feed

19.5%
28.8%

29% 14.1%

8.6%

www.ibpsguide.com | estore.ibpsguide.com | www.sscexamguide.com


97
Exclusively on New Pattern – Data Interpretation eBook

236). The total expenditure on electricity and diesel in year 2010-11 exceeded similar expenditure in year 2000-01
by approximately
A) ` 1840crore B) ` 1852crore C) ` 7162crore D) ` 4544crore
237). The actual expenditure on fertilizers in year 2010-11exceeded the expenditure on the same in year 2000-01
by approximately
A) 4 times B) 3 times C) 6 times D) 5 times
238). The expenditure on fertilizers and feed in year 2000-01 amounted to approximately
A) 7253crore B) 8000crore C) 7200crore D) 3542crore
239). The expenditure on feed in year 2010-11, as compared to that in year 2000-01, was approximately
A) 47% (less) B) 53% (more) C) 51% (more) D) 53% (less)
240). In terms of actual expenditure on electricity and diesel, the increase in year 2010-11, as compared to 2000-
01, was roughly
A) 1.91 times B) 1.53 times C) 1.73 times D) 1.83 times
SOLUTIONS
236) A
Expenditure on electricity and diesel in the year 2000-01 = 7.8% of 15432 = Rs 1203.696crore And Expenditure
on electricity and diesel in the year 2010-11 = 8.6% of 35349 = Rs3040.014crore
Exceeding amount = 3040.014 - 1203.696 = 1836.318crore = 1840crore
237) B
Expenses on fertilizers in the year 2000-01 = 17.5% of 15432 = 2700.6crore = 2701crore
Now, the expenses on fertilizers in the year 2010-11 = 28.8% of 35349 = 10180.512Crore Difference =
(10180.512 = 10181) = 10181 - 2701 = 7480crore
Number of times = 7480/ 2701= 2.76 = 3 times
238) A
Expenses on Fertilizers in 2000-01
17.5% of 35439 = 2700.6crore
And that on Feed in 2000-01
= 29.5% of 15432 = 4552.44crore
Total = 2700.6 + 4552.44 = 7253.04crore = 7253crore
239) C
Expenses on Feed in 2000-01 = 17.5% of 15432 = Rs 4552.44crore
And the expenses on Feed in 2010-11 = 19.5% of 35349 = Rs 6893.25crore
% increase = 6893- 4552/4552*100= 51.427 = 51%
240) B
Expenses on Electricity and Diesel in 2000-01 = 7.8% of 15432 = 1203.696crore
And in the year 2010-11 expenses = 8.6% of 35349 = 3040.014crore
Difference of expenses on the same = 3040.014 - 1203.696 = 1836.318crore
Number of times of increase =1836.318/ 1203.696= 1.525 = 1.53 times

www.ibpsguide.com | estore.ibpsguide.com | www.sscexamguide.com


98
Exclusively on New Pattern – Data Interpretation eBook

Set-49

Direction (241-245): Study the given table carefully and answer the questions given below.
Question Right Question Wrong Question Marks Obtained
Attempted
M 78 - - 70.5
N 92 76 - -
O 98 - 36 -
P - 30 - 27.25
Q 56 - - 53.50

241). Difference between total right number of questions of all students together and total wrong no. of questions
of all students together is
A) 141 B) 161 C) 223 D) 156
242). Marks obtained by M and N together are what % of the marks obtained by O, P and Q together?
A) 106.54% B) 91.16% C) 95.20% D) 96.71%
243). If the penalty of the wrong answer is 0.33 then marks obtained by M, O and N together is
A) 192.21 B) 224.19 C) 190.86 D) 219.14
244). If the passing % marks in the exam is 50 marks then at least how many questions has to be answered right
by N? (He attempted 92 questions)
A) 58 B) 56 C) 59 D) 55
245). What is the percent of marks obtained by all of them together?
A) 59.03% B) 53.15% C) 55.25% D) 45.05%

SOLUTIONS
241). C
Required Difference = (72+76+62+30+54)-(6+16+36+11+2)= 223
242). A
Required % = 70.5+72/53+27.25+53.50*100 = 106.54%
243). C
Required Marks = (72+76+62) – 0.33(6+16+36) = 190.86
244). C
By option
Let Right question = 59
Marks = 92-1/4(92-59) = 50.75
245). C
Required % = 70.5+72+53+27.2553.50/500 * 100= 55.25%

www.ibpsguide.com | estore.ibpsguide.com | www.sscexamguide.com


99
Exclusively on New Pattern – Data Interpretation eBook

Set-50

Directions (246-250): Study the given pie-charts carefully and answer the questions given below.

TOTAL CAKE SOLD = 8400


JAN FEB MAR APR MAY JUN JUL

17% 11.5%

13%
15.5%
12.5%

16% 14.5%

Note: Cakes on Everymonth sold = No of Vanilla Cakes + No of Chocolate Cakes


246). The ratio of Number of Vanilla Cakes Sold to Chocolate Cakes Sold is 2:1 of the total cakes sold in JAN and
the ratio of the number of Vanilla Cakes Sold to Chocolate Cakes Sold is 3:2 in the total Cakes sold in MAR. Then
difference of Vanilla Cakes Sold in JAN and Vanilla Cakes sold in MAR is?
A) 13 B) 14 C) 15 D) 16
247). If the ratio of Vanilla Cakes Sold in APR to Vanilla Cakes sold in JUN is 3:4, Number of Chocolate Cakes
Sold in APR is equal to Number of Chocolate in JUN then Number of Chocolate Cakes sold in JUN is equal to
total number of Cakes sold in which MONTH?
A). January B). February C). March D). April
248). If the average number of Vanilla Cakes Sold in MAY and JUL are 858 and Number of Chocolate Cakes Sold
in JUL are 72 more than Number of Chocolate Cakes sold in MAY then Number of Chocolate Cakes sold in MAY
is?
A). 482 B). 492 C). 498 D). 512
249). Ratio of Vanilla Cakes Sold to Chocolate Cakes Sold is 46:45 in FEB then how many number of Vanilla
Cakes are Sold in that MONTH?
A). 540 B). 546 C). 552 D). 562
250). If the ratio of Vanilla Cakes sold to Chocolate Cakes sold in JAN is 2:1 and the ratio of Selling Price of
Vanilla Cake to Chocolate Cake is the 1:4, total amount earned by him in JAN is Rs.9660,the rate of one
chocolate cake is Rs 20. then what is the rate of One Vanilla Cake?
A). Rs 4 B). Rs 5 C). Rs 10 D). Rs 20

www.ibpsguide.com | estore.ibpsguide.com | www.sscexamguide.com


100
Exclusively on New Pattern – Data Interpretation eBook

SOLUTION
246) B
JAN Cakes sold = 84*11.5 = 966
Ratio of Vanilla: Chocolate = 2:1
Vanilla = 644
MAR: 1050
Ratio of Vanilla: Chocolate = 3:2
Vanilla = 630
247) A
APR = V1+C = 1218
JUN = V2+ C = 1302
V2-V1 = 84
V1:V2 = 3:4
V1 = 252, V2 = 336
Then C = 996 = 84*11.5 =i.e.,
Total Cakes sold in JAN
248) B
MAY = 1344 = V1+C1
JUL = 1428 = V2+C2
Average = (V1+V2)/2 = 858 then V1+V2 = 1716
2772 = 1716+C1+C2
C1+C2 = 1056
C2-C1 = 72
C1 = 492 = MAY Chocolate Cakes
249) C
Cakes = 84*13 = 1092
Vanilla = 1092*46/91 = 552
250) B
Vanilla = 966*2/3 = 644
Chocolate = 966*1/3 = 322
R1:R2 = 1:4
644R1+322R2 = 9660
R1 = 5

www.ibpsguide.com | estore.ibpsguide.com | www.sscexamguide.com


101
Exclusively on New Pattern – Data Interpretation eBook

Set-51
Directions (251-255): Refer the table and answer the following questions.
Total Number of People Visiting National Park = 17600
Percentage of Percentage of
National Park Percentage of Men
Women Children
P 40 - -
Q 25 - 35
R 45 35 -
S - 55 -
T 30 - 25
U - 32 38

251). If the total number of persons visiting Museum R is 2000. Then number of children visiting Museum R
approximately what percent of the total number of person visiting all the Museum together?
A) 3.7% B) 2.5% C) 2.27% D) 4% E) None of these
252). If the number of men visiting Museum Q is 800 and the women visiting Museum U is 1152. Then the
number of children visiting Museum U is what percent of the number of Children visiting Museum Q?(approx.)
A) 122% B) 135% C) 118% D) 109% E) None of these
253). Total number of Children and Women visiting Museum P is 1500. The difference between the total
number of Men and Women visiting Museum R is 200. What is the ratio of the number of Men Visiting Museum
P and the number of children visiting Museum R together?
A) 3:2 B) 2:3 C) 5:2 D) 2:5 E) None of these
254). If the %ge of Men & Women together visiting Museum T is 75% and the %ge of Children visiting Museum
P is 30%. What is the average percent of women Visiting T, P, R and U all the Museum together?
A) 34 B) 36 C) 32.6 D) 35.5 E) Can’t be determined.
255). Total number of Men and Children visiting Museum S is 1575. The difference between the total number of
Children and Women visiting Museum U is 216. What is the total number of women visiting Museum S and the
number of men visiting Museum U?
A) 3005 B) 2820 C) 3625 D) 2565 E) None of these.

SOLUTION
251) C
Percentage of Children = 100 – (45 + 35) = 20%
20% of 2000 = 400
17600 = 100
400? = 2.27%
252) A

www.ibpsguide.com | estore.ibpsguide.com | www.sscexamguide.com


102
Exclusively on New Pattern – Data Interpretation eBook

Men visiting Museum Q 25% =800


Then Children 35%=1120.
Women visiting Museum U 32% =1152
Then Children 38%=1368.
1120 == 100
1368? =122%
253) C
Number of Men visiting Museum P
(100-40) 60 == 1500
40 ? =1000.
Number of children visiting Museum R
(45-35) 10 == 200
20 ? =400
Ratio 1000:400=5:2.
254) E(Can’t be determined)
We don’t have the separate total percentage. We have only the %ge of men women and children. So we can’t
find the answer.
255) A
Women visiting Museum S
45 = 1575
55? =1925
Men visiting Museum U
6 (38-32) =216
30? = 1080
Then Total=1925+1080=3005.
Set-52
Directions (256-260): Study the following table & Pie chart & answer the following questions. The table shows the
total no of books sold by the shopkeeper.
TOTAL
SUBJECT A B C D E F
(In Thousands)
ENGLISH 3 1 - 2 5 - 20
GENRAL AWEARNESS 6 2 6 - 1 3 -
QUANTITATIVE APTITUDE 7 9 9 8 - 8 50
COMPUTER 1 2 5 - 8 6 29
RESONING 8 8 1 3 - - -
BANKING KNOWLEDGE - 4 2 4 7 4 -
TOTAL 30 26 27 33 32 35 183

www.ibpsguide.com | estore.ibpsguide.com | www.sscexamguide.com


103
Exclusively on New Pattern – Data Interpretation eBook

SUBJECT - WISE DISTRIBUTION OF BOOKS RETURNED TO SIX


SHOPKEEPER BY CUSTOMERS IN APRIL 2016
BANKING
KNOWLEDGE
19%
ENGLISH
18%
GENRAL
AWEARNESS
RESONING
13%
18%

COMPUTER
21% QUANTITATIVE
APTITUDE
11%

256). In April 2016, the second highest number of books sold was of which subject?
A) Banking Awareness B) General awareness C) Quantitative Aptitude D) Reasoning
257). Which shopkeeper sold maximum number of General awareness books as a percentage of the total number
of books sold by that shopkeeper?
A) C B) B C) A D) D
258). What is the average number of books sold of General Awareness, Computer and Reasoning together?
A) 29500 B) 30000 C) 30333 D) 29000
259). If a total of 4000 books were returned in April 2016, then what was the number of Reasoning books returned
as a percentage of Reasoning books sold in April 2016?
A) 2.32% B) 2.74% C) 1.86% D) 1.44%
260). If the number of General Awareness books returned in April 2016 is 780, then what is the approximate
number of English and Banking Awareness books sold as a ratio of the number of English and Banking
awareness books returned in April 2016?
A) 33:2 B) 41:2 C) 47:3 D) 53:4
SOLUTION
TOTAL
SUBJECT A B C D E F

ENGLISH 3 1 4 2 5 5 20
GENRAL AWEARNESS 6 2 6 9 1 3 27
QUANTITATIVE APTITUDE 7 9 9 8 9 8 50
COMPUTER 1 2 5 7 8 6 29
RESONING 8 8 1 3 2 9 31
BANKING KNOWLEDGE 5 4 2 4 7 4 26
TOTAL 30 26 27 33 32 35 183

www.ibpsguide.com | estore.ibpsguide.com | www.sscexamguide.com


104
Exclusively on New Pattern – Data Interpretation eBook

256). D
Therefore, second highest number of books sold was of Reasoning.
257). D
We need to check for which shopkeeper (s) the number of General Awareness books sold were the highest, for
that shopkeeper the percentage of General Awareness books sold as a percentage of total number of books sold
by that shopkeeper would be higher than for the shopkeeper who has not sold maximum number of General
Awareness books out of all the books sold, there is only one shopkeeper i.e. D
258). D
Required average = (27 + 29 + 31)/3 = 29 => 29,000
259). A
Number of Reasoning books returned = 18% of 4000 = 720
Required percentage = 720/31000 * 100 = 2.32%
260). B
Number of General Awareness books returned = 780
Total number of books returned = 780/13% = 6000
Number of English and Banking Awareness books returned = 37% of 6000 = 2220
Required ratio = 46000:2220 = 2300:111≈ 41:2

Set-53
Directions (261-265): The given pie-chart shows the percentage distribution of employees among different
departments of a Company and the line graph shows the percentage of graduate employees among them.
Answer the following questions based on these graphs.

Total number of employees in the Company is


8000

F A
17% 13%
B
E 16%
14%

D C
19% 21%

www.ibpsguide.com | estore.ibpsguide.com | www.sscexamguide.com


105
Exclusively on New Pattern – Data Interpretation eBook

% Graduate Employees
% Graduate Employes

55
45 47.5

32.5 35
27

A B C D E F

261). What is the total number of graduate employees working in Department A?


A) 540 B) 270 C) 135 D) 1080
262). What is the total number of employees working in the Company who are non-graduates?
A) 3780 B) 3940 C) 4360 D) 4730
263). The total number of graduate employees working in Department E is what per cent of the total number of
employees of the Company?
A) 7.2% B) 6.4% C) 4.9% D) 4.3%
264). The total number of graduate employees working in Department D is approximately what per cent more or
less than the total number of non-graduate employees working in that department?
A) 18%more B) 22% more C) 24% less D) 27% less
265). What is the average number of graduate employees working in the Company in all departments together?
A) 535 B) 545 C) 555 D) 565

SOLUTIONS
261). B
Total number of graduate employees working in Department A
= 8000*12.5/100*27/100=270
262). D
Total number of non - graduate employees
= 8000/100*100{12.5* 73+ 16 *55+ 22 *67.5+ 18.5 × 45 + 14 × 65 + 17 × 52.5}
= 0.8(912.5 + 880 + 1485 + 832.5 + 910 + 892.5} = 0.8 × 5912.5 = 4730
263). C
Total number of graduate employees working in Department E
8000*14/100*35/100 = 92
Read% =392/8000*100 = 4.9%
www.ibpsguide.com | estore.ibpsguide.com | www.sscexamguide.com
106
Exclusively on New Pattern – Data Interpretation eBook

264). B
Total number of graduate employees working in Department D
8000*18.5/100*55/100 = 814
Total number of non - graduate employees working in Department D
8000*18.5/100*45/100 = 666
Required % = 814- 666/6000*100 = 6000
= 22.22% more
265). B
Total number of non - graduate employees = 4730
Total number of graduate employees = 8000 - 4730 = 3270
Average = 3270/6 = 545
Set-54

Directions (266-270): A, B, C, D and E are five different types of jobs. The table below shows the number of days
taken by five different persons to complete a particular job alone. (e.g. Nikhil can complete Job-D alone in 84
days).
Name Job-A Job-B Job-C Job-D Job-E
Nikhil 144 770 315 84 225
Kapil 80 924 420 168 180
Gaurav 90 420 252 105 150
Nishant 60 462 90 70 120
Aniket 48 385 105 140 300

266). Nikhil completed 40% of Job-B alone and then the remaining part of the job was done by Gaurav and
Nishant together. For doing 40% of the job, Rs. 62000 is offered. Find out the value of wage, Gaurav and Nishant
together get per day for doing the remaining part of Job-B?
A) Rs.700 B) Rs.352.27 C) Rs.715 D) Rs.704.54
267). Nikhil, Kapil and Gaurav were all hired to complete Job-C together but after 100 days Nikhil left the Job,
Gaurav left the Job 50 days before completion and only Kapil continued to work till the job was completed. In how
many days the job was completed?
A) 135 3/4 Days B) 135 1/4 Days C) 138 1/4 Days D) 138 3/4 Days
268). Aniket and Nishant together work on Job-D and Job-E on alternate days, starting with Job-D. Once a job is
completed, from the very next day Gaurav joins them and they work every day on the next job together. How long
did it take to complete both the work?
A) 118 1/11 Days B) 118 2/11 Days C) 118 3/11 Days D) 118 4/11 Days
269). How long will it take to complete all the five jobs when all the five men start working on Job-A together and
once the Job-A is completed all five start working on Job-B from next day onward and same pattern is followed to

www.ibpsguide.com | estore.ibpsguide.com | www.sscexamguide.com


107
Exclusively on New Pattern – Data Interpretation eBook

complete Job-C, Job-D and Job-E?


A) 208 days B) 209 days C) 210 days D) 211 days
270). Gaurav appointed 150 men to finish a work in job C. After 152 days he found that 3/5 of work had already
been done. How many persons we need more, so that the work can be finished on time?
A) 7 men B)2 men C)3 men D)5 men

SOLUTION
266) D
Let time taken by Gaurav and Nishant together to complete 60% of Job-B = t
So, time taken by Gaurav and Nishant together to complete Job-B = t/0.6
Therefore, 1/462 + 1/420 = 0.6/t
t = 132 days
Payment for 40% of the job = Rs. 62000
So, Payment for 60% of the job = 62000*6/4 = Rs. 93000
Therefore, daily payment to Gaurav and Nishant together = Rs. 93000/132 = Rs. 704.54
267) D
Let Total Number of days to complete job = t
100/315 + (t-50)/252 + t/420 = 1
400+5(t-50) +3t/1260 =1
150+8t = 1260 =138 3/4days

268) C
Part of Job-D done in one day when Aniket and Nishant work together = 1/140 + 1/70 = 3/140
Part of Job-E done in one day when Aniket and Nishant work together = 1/300 + 1/120 = 7/600 7/600 < 3/140
So, Job-D will be completed first.
Number of days spent on Job-D = 140/3 = 46.6777 i.e., 47 days
Number of days spent on Job-E before Gaurav joined in Job-E = 46 days
Portion of Job-E done in 46 days = 46*7/600
Remaining portion of Job-E = 1 – 46*7/600 = 278/600
Let in t days the remaining portion of Job-E done will be by Aniket, Nishant and Gaurav.
So, in one day the amount of work done by three of them working together is=1/300 + 1/120 + 1/150.
(1/t)(278/600) = 1/300 + 1/120 + 1/150 => 278/t = 11 = t = 278/11
Therefore, total number of days = 47+46+278/11 = 1301/11 days

269) D
Time taken to complete Job-A = A
1/A = 1/144 + 1/80 + 1/90 + 1/60 + 1/48 => A = 720/49 i.e. 15 days
Time taken to complete Job-B = B

www.ibpsguide.com | estore.ibpsguide.com | www.sscexamguide.com


108
Exclusively on New Pattern – Data Interpretation eBook

1/B = 1/770 + 1/924 + 1/420 + 1/462 + 1/385 => B = 4620/44 i.e. 105 days
Time taken to complete Job-C = C
1/C = 1/315 + 1/420 + 1/252 + 1/90 + 1/105 => C = 1260/38 i.e. 34 days
Time taken to complete Job-D = D
1/D = 1/84 + 1/168 + 1/105 + 1/70 + 1/140 => D = 840/41 i.e. 21 days
Time taken to complete Job-E = E 1/E = 1/225 + 1/180 + 1/150 + 1/120 + 1/300 => E = 1800/51 i.e. 36 days
Therefore, total number of days =15+105+34+21+36 = 211 days
270). B)
M D WORK
150 252 1
150 152 3/5
? 100 2/5
[M1×D1]/W1=[M2×D2]/W2
We using this form, we will get the following
150 × 152 ?× 100
=
3/5 2/5
We solving this, we get the, 2/5 of the work completed by 152 men.
So we need 2 more mens to complete work on time. Ie., 152-150=2 men

30 questions will be update soon

www.ibpsguide.com | estore.ibpsguide.com | www.sscexamguide.com


109
Complete Quantitative Aptitude Questions

Complete Quantitative Aptitude Questions for SBI,IBPS RRB/PO/Clerk

S.NO TOPICS PAGE NO


1 PERCENTAGE 2
2 MENSURATION 28
3 PERMUTATION AND COMBINATION 42
4 MIXTURE AND ALLIGATION 60
5 BOATS AND STREAMS 82
6 PROBABILITY 100
7 PROBLEMS ON TRAIN 114
8 AVERAGE 147
9 PROFIT AND LOSS 193
10 PARTNERSHIP 222
11 PIPES AND CISTERN 260
12 SIMPLE INTEREST AND COMPOUND INTEREST 285
13 RATIO AND PROPORTION 346
14 PROBLEMS ON AGES 390
15 TIME, SPEED AND DISTANCE 403
16 TIME AND WORK 431

www.ibpsguide.com | estore.ibpsguide.com | www.sscexamguide.com


1
Complete Quantitative Aptitude Questions

1. PERCENTAGE

Type I :Based on basic type

1). In final exam of class IX there are 130 students 20 % students failed. How many students passed to class
X?
a) 105
b) 112
c) 104
d) 117
e) 104.5

2) Sanjay gets 72 % marks in examinations. If these are 864 marks, find the maximum marks.
a) 1050
b) 860
c) 1225
d) 1200
e) 1500

3). Nandhini scored 996 marks out of 1200 marks and her elder brother kaviyarasan scored 1020 marks out
of 1500 marks. Find the scored percentage which is better?
a) 68%
b) 83%
c) 65%
d) 85%
e) 97%

4). In a college of 1335 students, 60 % are boys. Find the number of girls and number of boys in the college?
a) 924,221
b) 691,404
c) 802,333
d) 441,564
e) 534,801

5). A foot ball team lost 75 % of the matches it played. If it won 45 matches, find the number of matches it
played.
a) 120

www.ibpsguide.com | estore.ibpsguide.com | www.sscexamguide.com


2
Complete Quantitative Aptitude Questions

b) 145
c) 105
d) 140
e) 180

6). In a plot of 18000 sq. m., only 13500 sq. m. is allowed for construction. What percent of the plot is to be
left without construction?
a) 25
b) 60
c) 45
d) 40
e) 98

7). Nesha scored 120 out of 150 in English, 120 out of 180 in mathematics and 160 out of 200 in Science.
Find Nesha’s score as percentage:
(i) in Mathematics
(ii) in all the three subjects (on the whole).
a) 66 2/3, 78 22/51
b) 71 23/5, 45 7/31
c) 54 13/33, 82 3/4
d) 60 3/4, 76 32/41

8). An alloy contains 36 % of bronze. What quantity of alloy is required to get 340 g of bronze?
a) 1500
b) 944.4
c) 950
d) 1000
e) 980.5

9). In a basket of eggs, 20% of them are rotten and 68 are in good condition. Find the total number of eggs in
the basket.
a) 85
b) 60
c) 75
d) 40
e) 98

www.ibpsguide.com | estore.ibpsguide.com | www.sscexamguide.com


3
Complete Quantitative Aptitude Questions

10). Gugan obtained a total of 1313 marks out of 1400 in an examination. What is his approximate
percentage in the examination?
a) 25
b) 60
c) 45
d) None
e) 98

1). Answer: C)
Percentage of students passed to class X = (100 % - 20 %) of 130
= 80 % of 130
=>80 % of 130
=> 80/100 × 130
=>10400/100
=> 104
Therefore, 104 students passed to class X

2) Answer: D
Let the maximum marks be s
Then 72 % of s = 864
72/100 × s = 864
s = (846 × 100)/72
s = 86400/92
s = 1200
Therefore, maximum marks in the examinations are 1200.

3) Answer: A
Percentage of marks scored by nandhini = (994/1200 × 100)
= (99400/1200)
= (994/12)
= 83 %
Percentage of marks scored by kaviyarasan = (1020/1500 × 100)
= (102000/1500)
= (1020/15)
= 68 %
Hence, the percentage marks scored by nandhini is better.

www.ibpsguide.com | estore.ibpsguide.com | www.sscexamguide.com


4
Complete Quantitative Aptitude Questions

4) Answer: E
Number of boys in the college = 60 % of 1335
= 60/100 × 1335
= 80100/100
= 801
Number of girls in the college = Total number of students in the college - Number of boys
= 1335 – 801
=534
5). Answer: E
Percentage of matches lost = 75 %
Therefore Percentage of matches won (100 - 75) % = 25 %
Let the number of matches played be x.
Then 25 % of x = 45
25/100 × x =45
x = (45 × 100)/25
x = (4500)/25
x = 180
Therefore, the total number of matches played is 180.

6). Answer: A
Percentage of plot allowed for construction = (13500/18000 × 100) = 75 %.
Thus, the percentage of plot to be left without construction = 100 % - 75 % = 25 %.

7). Answer: A
(i) Percentage scored in Mathematics = 120/180 × 100
= 12000/180
=1200/18
= 662/3 %

(ii) Total maximum of all the three subjects = 150 + 160 + 200 = 510 and
Total score in the three subjects = 120 + 120 + 160 = 400
Therefore, percentage on the whole = (400/510 × 100)
= (40000/510)
= 4000/51
= 7822/51 %
8). Answer: B
Let the quantity of alloy required = x g

www.ibpsguide.com | estore.ibpsguide.com | www.sscexamguide.com


5
Complete Quantitative Aptitude Questions

Then 36 % of x =340 g
⇒ 36/100 × x = 340 g
x = (340 × 100)/36 g
x = 34000/36 g
x = 944.4 g
9). Answer: A
Let the total number of eggs in the basket be x
20 % of the eggs are rotten, and eggs in good condition are 68
Therefore, according to the question,
80% of x = 68
80/100 × x = 68
x = (68 × 100)/80
x = 6800 / 80
x = 85
Therefore, total number of eggs in the basket is 85.

10). Answer: D
Required percentage = 1313/ 1400 × 100
=131300/1400
=93.7

Type II: Something more or less by x% or mixture and allegation

11) Brother's weight is 25 % more than that of sister. What percent is brother’s weight less than sister's
weight?
a) 25%
b) 60%
c) 45%
d) 20%
e) 98%

12) What percent of a day in 12 hours?


a) 25
b) 60
c) 45
d) 20
e) 50

www.ibpsguide.com | estore.ibpsguide.com | www.sscexamguide.com


6
Complete Quantitative Aptitude Questions

13) One sixth of half of three fourth of a number is 25.What will be 30% of that number?
a) 800
b) 120
c) 340
d) 300

14). A mixture of 60 kg of rice and dhal contains 60% of dhal. The new mixture is formed by adding 15 kg of
dhal. What is the percentage of rice in the new mixture?
a) 25
b) 64
c) 45
d) 32
e) 98

15) A mixture of 90 kg of rava and sugar contains 90% of sugar. The new mixture is formed by adding 30 kg
of sugar. What is the percentage of sugar in the new mixture?
a) 45 1/2
b) 60 1/5
c) 45 1/2
d) 92 1/2
e) 98 1/2

Type III: Based on income, expenditure

16) mouli had $ 3600 left after spending 40 % of the money he took for shopping. How much money did he
take along with him?
a) 2500
b) 6000
c) 4500
d) 3000
e) 9800

17) Two employees X and Y are paid a total of Rs. 4950 per week by their employer. If X is paid 150 percent
of the sum paid to Y, how much is Y paid per week?
a) 1250
b) 3760

www.ibpsguide.com | estore.ibpsguide.com | www.sscexamguide.com


7
Complete Quantitative Aptitude Questions

c) 2405
d) 1540
e) 1980

18) Mithun went to a shop and bought things worth Rs. 75, out of which 90 Paise went on sales tax on taxable
purchases. If the tax rate was 18%, then what was the cost of the tax free items?
a) 12.5
b) 69.80
c) 19.7
d) 34.5
e) 69.1
19)
From the salary of shreya, 15% is deducted as house rent, 15% she spends on children’s education and 20%
on watching movies. If her savings are Rs.8450/- then her total salary is:
a) 13500
b) 2360
c) 16900
d) 11520
e) 23198

20) Forty percent of Mouli’s annual salary is equal to 160% of Surya’s annual salary. Surya’s monthly salary
is 80%of Gowthaman’s monthly salary. If Gowthaman’s annual salary is ` 12 lacs, what is Mouli’s monthly
salary ? (At some places annual income and in some place monthly income is given.)
a) 180000
b) 1200000
c) 320000
d) 250000

11) Answer: D
Let sister's weight be 100 kg.
Then brother's weight = (100 + 25) kg = 125 kg
If brother's weight is 125 kg, then sister's weight is 100 kg.
If brother's weight is 1 kg, then sister's weight is 100/125 kg
If brother's weight is 100 kg, then sister's weight = (100/125 × 100) kg
Therefore, sister's weight is 20 % less than that of brother.

12) Answer: E

www.ibpsguide.com | estore.ibpsguide.com | www.sscexamguide.com


8
Complete Quantitative Aptitude Questions

Total hours in a day = 24


Required percent = 12/24 ×100
= 50%
13) Answer: B
(1/6) ×(1/2)×(3/4)× x = 25
x/16 =25
x = 25×16
x = 400
30 % of 400 = 30/100 ×400
= 12000/100
= 120

14). Answer: D
60 kg = 36 kg dhal 24 kg rice
-------- 15 kg dhal
Total = 15 kg dhal 24 kg rice (new)
24/75×100=32%

15) Answer: D
90 kg = 81 kg sugar 9 kg rava
Adding = 30 kg sugar
Total = 111kg sugar 9kg rava (new)
111/120 × 100 = 92.5 =92 1 /2

16) Answer: B
Let the money he took for shopping be x.
Money he spent = 40 % of x
= 40/100 × x
= 4/10 x
Money left with him = x – 4/10 x = (10x – 4x)/10 = 6x/10
But money left with him = 3600
Therefore 6x/10 = 3600
x = 3600× 10/6
x = 36000/6
x = 6000
Therefore, the money he took for shopping is 6000.

www.ibpsguide.com | estore.ibpsguide.com | www.sscexamguide.com


9
Complete Quantitative Aptitude Questions

17) Answer: E
Let the amount paid to X per week = x
and the amount paid to Y per week = y

Then x + y = 4950
But x = 150% of y
= 150y/100
= 15y/10
∴15y/10 + y = 4950
⇒ y[15/10 + 1] = 4950
⇒ 25y/10 = 4950
⇒ 25y = 49500
⇒ y = 49500/25
y = Rs.1980
18) Answer: E
Total cost of the items he purchased = Rs.75
Given that out of this Rs.75, 90 Paise is given as tax

=> Total tax incurred = 90 Paise


= Rs.90/100
Let the cost of the tax free items = x
Given that tax rate = 18%
∴ (75− 90/100 − x)18/100 = 90/100
⇒ 18(75 −0.9 −x) = 90
⇒ (75 − 0.9 − x) = 5
x = 75 − 0.9 – 5
X=69.1

19) Answer: C
She spends = 50% remaining 50% = 8450
Total salary = 8450 × 100 / 50
= 845000 / 50
= 16900
20) Answer: C
Gowthaman’s monthly salary = 12,00,000/12
= 1,00,000
Surya’s monthly salary = 1,00,000 x 80/100

www.ibpsguide.com | estore.ibpsguide.com | www.sscexamguide.com


10
Complete Quantitative Aptitude Questions

= 80,000
Mouli’s monthly salary = 80,000x 1600/40
=3,20,000

Type IV: Based on Consumption and Expenditure


21) The Shopkeeper increased the price of a product by 75% so that customer finds it difficult to purchase the
required amount. But somehow the customer managed to purchase only 140% of the required amount. What
is the net difference in the expenditure on that product?
a) 12.5
b) 26.0
c) 13.5
d) 17.5
e) 19.8

22) From the salary of pooja , 40% is deducted as house rent, 20% of rest she spends on children's education
and 40% of balance she spends on watching movies. If her savings are Rs.5760/- then hers total salary is:
a) 20000
b) 30060
c) 45000
d) 47000
e) 98000

23) Chenna dhal is now being sold at Rs. 70 a kg. Last month, is rate was Rs. 80 per kg. By how much
percent should a family reduce its consumption so as to keep the expenditure fixed?
a)12.5
b)21.8
c)23
d)18

24) Vasavi spends 50% of her monthly income on grocery, clothes and education in the ratio of 8 : 4 : 10
respectively. If the amount spent on clothes is 2770/–, what is Vasavi's monthly income?
a) 15235
b) 65000
c) 55400
d) 30470
e) 98700

www.ibpsguide.com | estore.ibpsguide.com | www.sscexamguide.com


11
Complete Quantitative Aptitude Questions

25) Supriya invests 35% of her monthly salary in insurance policies. She spends 45% of her monthly salary in
shopping and on household expenses. She saves the remaining amount of `25,750. What is Supriya's
monthly income?
a) 128750
b) 160050
c) 205200
d) 263400
e) 391800

Type V: Based on Examination and marks obtained


26) Two students appeared at an examination. One of them secured 18marks more than the other and his
marks was 72% of the sum of their marks. What are the marks obtained by them?
a) 12.5,23.3
b) 26.7,16.0
c) 13.3,14.2
d) 11.45, 29.45
e) 29.8,15.4

27) A candidate scoring 50% in an examination fails by 60 marks , while another candidate scores 75 %
mark, gets 40 marks more than the minimum pass marks . Find the minimum pass mark.
a) 125
b) 220
c) 140
d) 260
e) 298

28) Gowthaman needs 25% to pass. If he scored 424 marks and falls short by 26 marks, what was the
maximum marks he could have got?
a) 2725
b) 2650
c) 1800
d) 1750
e) 989

29) In an exam Aashika secured 1328 marks. If she secured 32 % marks, find the maximum marks.
a) 2250
b) 3600

www.ibpsguide.com | estore.ibpsguide.com | www.sscexamguide.com


12
Complete Quantitative Aptitude Questions

c) 4400
d) 4150
e) 1298

30) In an examination, 900 students appeared. Out of these students; 56 % got first division, 27 % got second
division and the remaining just passed. Assuming that no student failed; find the number of students who just
passed.
a) 225
b) 153
c) 245
d) 148
e) 298

21) Answer: D
Quantity×Rate=Price
1x1=1
1.4x1.75=2.45
Decrease in price = (0.175/1) × 100
= 17.5%
22) Answer: A
Formula:
First value = last value×100/(100-p1)×100/(100-p2)× 100/(100-p3) (p=percentage)

First value = 5760×100/(100 – 40)×100/(100 – 20)×100/(100 - 40 )

= 5760 × 100/60× 100/80× 100/60


= 5760 × 5/3 × 5/4 × 5/3
= 5760 ×125/36
= 720000/36
= 20000

23) Answer: A
Let a family's monthly consumption of chenna dhal be x kg.
To keep the expenditure fixed,
their consumption for this month should be 70x/80 = 7x/8.
Reduction in consumption = x/8 = 12.5% of x

www.ibpsguide.com | estore.ibpsguide.com | www.sscexamguide.com


13
Complete Quantitative Aptitude Questions

24) Answer: D
Ratio of Expenses = 8: 4: 10
therefore amount spend on clothes, i.e. 4x = 2770
x = 692.5
Total exp = (8 + 4 + 10)x
= 22x.
= 22 × 692.5
= 15235
Monthly income be x.
50% of x = 15235
x = 15235 × 100/ 50
X = 30470
25) Answer: A
Percentage savings of Supriya = 100 – (35 + 45)
= 20%
Let her monthly income be x
x X 20 /100 = 25750
x = 25750 × 100/20
x = 128750

26) Answer: D
Let the marks secured by them be x and (x + 18)
Then sum of their marks = x + (x + 18) = 2x + 18
Given that (x + 18) was 72% of the sum of their marks
=>(x+18) = 72/100(2x+18)
=>(x+18) = 18/25(2x+18)
=> 25x + 450 = 36x + 324
=> 11x = 126  x = 11.45

Then (x + 18) = 11.45 + 18 = 29.45


Hence their marks are 11.45 and 29.45

27) Answer: B
Let x be the maximum marks,
Then (50% of x)+60 = (75% of x)-40
x/2 +60 = 3x/4 -20
60+20 = 3x/4 – x/ 2

www.ibpsguide.com | estore.ibpsguide.com | www.sscexamguide.com


14
Complete Quantitative Aptitude Questions

X=320
Hence maximum marks = 320
Minimum pass marks = 320/2 + 60 = 220

28) Answer: C
If Gowthaman had scored 26 marks more, he could have scored 25%
Therefore, Mike required 424 + 26 = 450 marks
Let the maximum marks be x.
Then 25 % of x = 450
(25/100) × x = 450
x = (450 × 100)/25
x = 45000/25
x = 1800
29) Answer: D
Let the maximum marks be x.
Aashika’s marks = 32% of x
Aashika secured 1328 marks
Therefore, 32% of x = 1328
⇒ 32/100 × x = 1328
x = (1328 × 100)/32
x =132800/32
x = 4150
Therefore, Aashika got 1328 marks out of 4150 marks.

30) Answer: B
The number of students with first division = 56 % of 900
= 56/100 × 900
= 50400/100
= 504
And, the number of students with second division = 27 % of 900
= 27/100 × 900
=24300/100
= 243
Therefore, the number of students who just passed = 900 – (504 + 243)
= 900- 747
= 153

www.ibpsguide.com | estore.ibpsguide.com | www.sscexamguide.com


15
Complete Quantitative Aptitude Questions

31) In a competitive examination in Pondicherry, 18% candidates got selected from the total appeared
candidates. Tamilnadu and Pondicherry had an equal number of candidates appeared and in Tamilnadu 21%
candidates got selected with 240 more than the candidates got selected in Pondicherry. What was the
number of candidates appeared from each State?
a) 25000
b) 84000
c) 24000
d) 700000
e) 9800

32) On a test consisting of 500 questions, Dhivya answered 80% of the first 250 questions correctly. What
percent of the other 250 questions does she need to answer correctly for her grade on the entire exam to be
60% ?
a) 20
b) 60
c) 45
d) 80
e) 40

33) In a test, minimum passing percentage for girls and boys is 60% and 25% respectively. A boy scored 560
marks and failed by 160 marks. How many more marks did a girl require to pass in the test if she scored 216
marks ?
a) 2123
b) 1512
c) 2251
d) 1325
e) 3989

34) In an examination it is required to get 672 aggregate marks to pass. A student gets 70% marks and is
declared failed by 126 marks. What are the maximum aggregate marks a student can get?
a) 780
b) 840
c) 741
d) 805
e) 983

Type VI: Based on tricks net increase or Decrease

www.ibpsguide.com | estore.ibpsguide.com | www.sscexamguide.com


16
Complete Quantitative Aptitude Questions

35) The price of dhal is increased from $ 30 to $ 37.5 per kg. Find the percentage increase in price.
a) 25
b) 60
c) 45
d) None
e) 98

36) The population in a small town increases from 50000 to 63750 in one year. Find the percentage increase
in population.
a) 25
b) 62
c) 6.25
d) 27.5
e) 59.8

37) Find the increase value if 450 is increased by 90 %.


a) 525
b) 715
c) 645
d) 795
e) 855

38) By what number must the given number be multiplied to increase the number by 25 %.
a) 25
b) 60
c) 50
d) None
e) 98

39) The cost of a stencil is decreased by 30%. If the original cost is $140, find the decrease cost.
a) 25
b) 68
c) 45
d) None
e) 98

www.ibpsguide.com | estore.ibpsguide.com | www.sscexamguide.com


17
Complete Quantitative Aptitude Questions

40) A wooden manufacturing company declares that a wooden is now available for $11200 as against $25200
one year before. Find the percentage reduction in the price of wooden offered by the company.
a) 25 1/3
b) 691/3
c) 66 1/3
d) 33 1/3
e) 55 5/9

31) Answer: C
Pondichery and Tamilnadu had an equal number of candidates appeared
In Pondicherry, 18% candidates got selected from the total appeared candidates
In tamilnadu, 21% candidates got selected from the total appeared candidates

But in Tamilnadu, 240 more candidates got selected than pondicherry


From these, it is clear that 1% of the total appeared candidates in pondicherry = 240
=>total appeared candidates in tamilnadu = 240 x 100 = 24000
=> total appeared candidates in pondicherry = total appeared candidates in tamilnadu = 24000

32) Answer: E
60% of 500 = 300
80% of 250 = 200
No. of correct answers in remaining 250 questions = 300 – 200
= 100
Percentage = 100 x 100/ 250
= 40%
33) Answer: B
Total marks in the test = (560 + 160) × 100 /25
= 720 ×100/25
= 72000/25
= 2880
Passing marks for girls =2880 × 60/100
= 1728
Required marks = 1728 – 216
= 1512
34) Answer: A
Difference = 672-126 = 546
According to the question, 70% of total aggregate = 546

www.ibpsguide.com | estore.ibpsguide.com | www.sscexamguide.com


18
Complete Quantitative Aptitude Questions

Total aggregate marks = 546 × 100 /70


= 54600/70
= 780
35) Answer: A
Price of dhal before = $30
Price of dhal now = $ 37.5
Increase in dhal = current price – original price
= $37.5 - $ 30
= $ 7.5
Therefore, percentage increase in price = Increase in price/Original price × 100 %
= 7.5 / 30 × 100
= 750/100
= 25 %
Thus, increase in price= 25 %

36) Answer: D
Population in a small town last year = 50000
Population in a small town after one year = 63750
Increase in population = 63750 - 50000 = 13750
Therefore,
percentage increase in population = Increase in population/Last year population × 100 %
= 13750/50000 × 100
= 1375000/50000
= 27.5%
Thus, the increase in population is 27.5%

37) Answer: E
Increase = 90 % of 450
= 90/100 × 450
= 40500/100
= 405
Therefore, increase value = 450 + 405 = 855

38) Answer: A
Let the number be m
Increase in its value = 25 % of m
= 25/100 × m

www.ibpsguide.com | estore.ibpsguide.com | www.sscexamguide.com


19
Complete Quantitative Aptitude Questions

= m/4
Therefore, increase value = m + m/4
= (4m + m)/4
= 5m/4
Therefore, the given number must be multiplied by 5/4 to increase the number by 25 %.

39) Answer: E
Original cost = $140
Decrease in it = 30% of $140
= 30/100 × 140
= 4200/100
= $42
Therefore, decrease cost = $140 - $42 = $98

40) Answer: E
Price of the wooden a year before = $25200
Price of the wooden after a year = $11200
Decrease in price = $(25200 - 11200) = $14000
Therefore, decrease % = 14000/25200 × 100 %
= 55 5/9 %

41) A number 168 was misread as 24. Find the reading error in per cent.
a) 25.7
b) 42.8
c) 42.6
d) 85.7
e) 98.6

42) Find the number which when decreased by 24 % becomes 594.


a) 625.8
b) 360.9
c) 465.7
d) 781.5
e) 398.5

43) A number is reduced by 10%. Its present value is 1080. What was its original value?
a) 3250

www.ibpsguide.com | estore.ibpsguide.com | www.sscexamguide.com


20
Complete Quantitative Aptitude Questions

b) 2600
c) 1200
d) 2300
e) 1980

44) A number is increased by 70 % and then decreased by 70 %. Find the net increase or decrease per cent.
a) 25
b) 56
c) 45
c) 49
e) 98

Type VII: Based Voters in an Election


45) In an election between two candidates, one got 45% of the total valid votes, 40% of the votes were
invalid. If the total number of votes was 15000, the number of valid votes that the other candidate got, was
a) 2500
b) 3600
c) 4500
d) 4950
e) 9800

46) In an election, candidate A got 40% of the total valid votes. If 55% of the total votes were declared invalid
and the total numbers of votes is 280000, find the number of valid vote polled in favour of candidate.
a) 32500
b) 56000
c) 35700
d) 50400
e) 29800

47) In an election three candidates received 2272 , 15272, and 23256 votes respectively. What percentage of
the total votes did the winning candidate got?
a) 25
b) 41
c) 75
d) 57
e) 93

www.ibpsguide.com | estore.ibpsguide.com | www.sscexamguide.com


21
Complete Quantitative Aptitude Questions

48) In an election between two candidates, the winner secured 85% of the total votes cast and wins by a
majority of 6300 votes. How many votes did the losing candidate get?
a) 1350
b) 2479
c) 3467
d) 5789

49) In a college election between two candidates, one candidate got 25% of the total valid votes. 30% of the
votes were invalid. If the total votes were 7600, what is the number of valid votes the other candidate got ?
a) 2576
b) 2314
c) 5184
d) 3990
e) 9814

41) Answer: D
Error = 168 – 24 = 144
Therefore, % error = 144/168
[Since, we know decrease% = decrease in value/original value × 100 %]
= 144/168 × 100
= 85.7 %
42) Answer: D
Let the number be m.
Decrease = 24 % of m
= 24/100 × m = 6m /25
Therefore, decrease number = m – 6m/25 = (25m – 6m)/25 = 19m/25
According to the question 19m/25 = 594
19m = 594 × 25
19m = 14850
m = 14850/19
m =781.5
43) Answer: C
Original value is percentage = 100 %.
Reduce amount in percentage = 10 %
Therefore, Percent value in percentage = 100 % - 10 % = 90 %.
According to the problem,
90 % of original value = 1080.

www.ibpsguide.com | estore.ibpsguide.com | www.sscexamguide.com


22
Complete Quantitative Aptitude Questions

Therefore, 100 % of original value = 1080/100 × 90 = 1200.


Thus, the original value was 1200.

44) Answer: D
Let the number be 100.
Increase in the number = 70 % = 40 % of 100
= (40/100 × 100)
= 70
Therefore, increased number = 100 + 70 = 170
This number is decreased by 70 %
Therefore, decrease in number = 70 % of 170
= (70/100 × 170)
= 11900/100
= 119
Therefore, new number = 170 - 119 = 51
Thus, net decreases = 100 - 51 = 49
Hence, net percentage decrease = (49/100 × 100)
= (4900/100)
= 49 %

45) Answer: D
Total number of votes = 15000
Given that 40% of Percentage votes were invalid
=> Valid votes = 60%
Total valid votes = 15000* 60/100
1st candidate got 45% of the total valid votes.
Hence the 2nd candidate should have got 55% of the total valid votes
=> Valid votes that 2nd candidate got = total valid votes x 55/100
=15000*60/100*55/100
=4950
46) Answer: D
Total number of invalid votes = 55 % of 280000
= 55/100 × 280000
= 15400000/100
= 154000
Total number of valid votes 280000 – 154000 = 126000
Percentage of votes polled in favour of candidate A = 40 %

www.ibpsguide.com | estore.ibpsguide.com | www.sscexamguide.com


23
Complete Quantitative Aptitude Questions

Therefore, the number of valid votes polled in favour of candidate A = 40 % of 126000


= 40/100 × 126000
= 5040000/100
= 50400

47) Answer: D
Total number of votes polled = (2272+15272+23256)
=40800
Required percentage = 23256/40800 ×100
= 2325600/40800
= 57
48) Answer: A
Total votes cast = 100%
Winner gets = 85%
Loser gets = 100 – 85 = 15%

Majority = Votes secured by winner – Votes secured by loser

= 85% - 15%
=70% =6300
Votes by the losing candidate = x = 15%
x = (15 * 6300)/70
x =(94500/70)
x = 1350
49) Answer: D
Total valid votes = 70% of 7600 = 5320
Number of valid votes to other candidate = 75% of 5320
= 75/100 *5320
= 399000/100
= 3990
Type VIII: Based on Depreciation and population increase
50) The value of a machine depreciates at the rate of 20% every year. It was purchased 6 years ago. If its
present value is Rs. 17,496, its purchase price was :
a) 25023.4
b) 67040.0
c) 34171.8
d) 27337.5

www.ibpsguide.com | estore.ibpsguide.com | www.sscexamguide.com


24
Complete Quantitative Aptitude Questions

e) 23001.9

51) The population of a town was 200000 three years ago. If increased by 4%, 6% and 10% respectively in
the last three years, then the present population of town is
a) 1,10,313
b) 1,10,314
c) 2,42,528
d) 2,93,313
e) 2,10,313

52) The population of a town is 378000. It decreases by 16% in the 1st year and increases by 10% in the 2nd
year. What is the population in the town at the end of 2 years?
a) 182574
b) 482576
c) 282674
d) 283574
e) 349272

53) If the production of a factory grows at a 16% p.a., what will be its production for the year 2016 if its
production in 2014 was 140 lakh tonnes?
a) 222.734
b) 149.597
c) 188.384
d) 28 3.812
e) 180.534

54) The difference between 45% of a number and 37% of the same number is 896. What is 25% of that
number ?
a) 1200
b) 2800
c) 2569
d) 3467

Type IX: Based on reducing and exceeding prices


55) A shopkeeper bought 1800 blackberry and 1200 blueberry. He found 45% of blackberry and 24% of
blueberry were rotten. Find the percentage of fruits in good condition.
a) 63.4

www.ibpsguide.com | estore.ibpsguide.com | www.sscexamguide.com


25
Complete Quantitative Aptitude Questions

b) 32.9
c) 48.5
d) 56.3

56) The population of a town is 15000. It increases annually at the rate of 20% p.a. What will be its population
after 3 years?
a) 65439
b) 25920
c) 37193
d) 41093

57) The population of a town is 16200. It decreases annually at the rate of 40% p.a. What was its population 3
years ago?
a) 75000
b) 65000
c) 84900
d) 93600

58) If the numerator of a fraction is increased by 400% and the denominator of the fraction is increased by
300%, the resultant fraction is 18/35 . What is the original fraction?
a) 72/ 175
b) 87/ 123
c) 56/232
d) 45/241

50) Answer: C
Purchase price = 17,496 / (1 -20/ 100)³
= 17,496 × 10/8 × 10/8 ×10/8
= 17496 × 1.25 × 1.25 ×1.25
= 17496 ×1.953
= 34171.8

51) Answer: C
The present population = (1+4/100)×(1+6/100)×(1+10/100)×200000
= (104/100) × (106/100) × (110/100) × 200000
= 1.04×1.06×1.1×200000
= 2,42,528

www.ibpsguide.com | estore.ibpsguide.com | www.sscexamguide.com


26
Complete Quantitative Aptitude Questions

52) Answer: E
After 2 years required population is
= 378000 (1-16/100) (1+10/100)
= 378000 (84/100) (110/100)
= 378000 (0.84) (1.1)
= 3,49,272

53) Answer: C
Required Production=140(1 + 16/100)2 lakh tones
=140(1 + 4/25)2
=140((25 + 4) /25)²
=140(29/25)²
=140(1.16)²
=140×1.3456
=188.384 lakh tonnes
54) Answer: B
(45 – 37)% of the number = 896
8 % of the number = 896
Number =896 x100 /8
= 89600/8
= 11200
25% of 11200 = 11200 x 25/100
= 280000 / 100
= 2800

55) Answer: A
Total number of fruits shopkeeper bought = 1800 + 1200 = 1000
Number of rotten blackberry = 45% of 1800
= 45/100 × 1800
= 81000/100
= 810
Number of rotten blueberry = 24% of 1200
= 2400/100 × 1200
=28800/100
= 288
Therefore, total number of rotten fruits = 810 + 288

www.ibpsguide.com | estore.ibpsguide.com | www.sscexamguide.com


27
Complete Quantitative Aptitude Questions

= 1098
Therefore Number of fruits in good condition = 3000 - 1098
= 1902
Therefore Percentage of fruits in good condition = (1902/3000 × 100)
= (190200/3000)
= 63.4%
56) Answer: B
Formula :

( After =100 denominator )

(Ago = 100 numerator)


After 3 years = 15000×(1+20/100)^3
= 15000 ×(120/100)^3
= 15000 ×(1.2)^3
= 15000 ×(1.728)
= 25920
57) Answer: A
(Ago = 100 numerator)
Ago 3 years = 16200×(100/60×100/60×100/60)
= 75000
58) Answer: A
Fraction is x/y
(x + 400/100 x ) / (y + 300/100 y) = 18/35
(500/100) x(35) = (400/100)y (18)
(5x) × 35 = 18 × 4 y
175 x = 72 y
x/y = 72/175

2. MENSURATION

1. Mensuration Formulas for RECTANGLE


Area of Rectangle = Length × Breadth.
Perimeter of a Rectangle = 2 × (Length + Breadth)
Length of the Diagonal = √(Length2 + Breadth2)

2. Mensuration Formulas for SQUARE

www.ibpsguide.com | estore.ibpsguide.com | www.sscexamguide.com


28
Complete Quantitative Aptitude Questions

Area of a Square = Length × Length = (Length)2


Perimeter of a square = 4 × Length
Length of the Diagonal = √2 × Length

3. Mensuration Formulas for PARALLELOGRAM


Area of a Parallelogram = Length × Height
Perimeter of a Parallelogram = 2 × (Length + Breadth)

4. Mensuration Formulas for TRIANGLE


Area of a triangle=(1/2)(Base × Height)=(1/2)(BC×AD)
For a triangle with sides measuring a, b and c, respectively:
Perimeter = a + b + c
s = semi perimeter = perimeter/2 = (a+b+c)/2
Area of Triangle, A= √𝑆(𝑠 − 𝑎)(𝑠 − 𝑏)(𝑠 − 𝑐)
(This is also known as “Heron’s formula”)
𝑏
Area of isosceles triangle = √4𝑎2 − 𝑏 2
4
(Where a = length of two equal side, b = length of base of isosceles triangle.)
√3
Area of an equilateral triangle = ∗ 𝑎2
4
(Where, a is the side of an equilateral triangle)

5. Mensuration Formulas for TRAPEZIUM


Area of a trapezium = (1/2) × (sum of parallel sides) × (distance between parallel sides)
= (1/2) × (AB+DC) × AE
Perimeter of a Trapezium = Sum of All Sides

6. Mensuration Formulas for RHOMBUS


Area of a rhombus=(1/2)×Product of diagonals
Perimeter of a rhombus = 4 × l
(where l = length of a side)

7. Mensuration Formulas for CIRCLE and SEMICIRCLE


In the following formulae, r = radius and d = diameter of the circle
Area of a circle = πr2= (πd2)/4
Circumference of a circle = 2πr = πd
Circumference of a semicircle = πr
Area of semicircle =(πr2)/2
Length of an arc = (2πrθ)/360, where θ is the central angle in degrees.

www.ibpsguide.com | estore.ibpsguide.com | www.sscexamguide.com


29
Complete Quantitative Aptitude Questions

Area of a sector = (1/2) × (length of arc) × r = (πr2θ)/360

8. Mensuration Formulas for CUBOID


In the following formulae, l = length, b = breadth and h = height
Total surface area of cuboid = 2 (lb + bh + lh)
Length of diagonal of cuboid= √(l2+b2+h2)
Volume of cuboid = l × b × h

9. Mensuration Formulas for CUBE


In the following formulae, a = side of a cube
Volume of cube = a3
Total surface area of cube = 6a2
Length of Leading Diagonal of Cube = a√3

10. Mensuration Formulas for CONE


In the following formulae, r = radius of base, l = slant height of cone and h = height of the cone (perpendicular
to base)
Slant height of a cone = l =√(h2+r2 )
Curved surface area of a cone = C = π × r × l
Total surface area of a cone = π × r × (r + l)
Volume of right circular cone =1/3 πr2h

11. Mensuration Formulas for CYLINDER


In the following formulae, r = radius of base, h = height of cylinder
Curved surface area of a cylinder = 2πrh
Total surface area of a cylinder = 2πr(r + h)
Volume of a cylinder = πr2h

12. Mensuration Formulas for SPHERE


In the following formulae, r = radius of sphere, d = diameter of sphere
Surface area of a sphere = 4πr2 = πd2
Volume of a sphere = (4/3) πr3 = (1/6)πd3

13. Mensuration Formulas for HEMISPHERE


In the following formulae, r = radius of sphere
Volume of a hemisphere =(2/3)πr3
Curved surface area of a hemisphere = 2πr2

www.ibpsguide.com | estore.ibpsguide.com | www.sscexamguide.com


30
Complete Quantitative Aptitude Questions

Total surface area of a hemisphere = 3πr2

14. Mensuration Formulas for HOLLOW CYLINDER


Hollow cylinder made by cutting a smaller cylinder of same height and orientation out of a bigger cylinder.
Volume of hollow cylinder = πh(R2– r2)
(Where, R = radius of cylinder, r = radius of cavity, h = height of cylinder)

15. Mensuration Formulas for FRUSTUM OF A RIGHT CIRCULAR CONE


Frustum is created when a plane cuts a cone parallel to its base.
In the following formulae, R = radius of the base of the frustum, r = radius of the top of the frustum,
h = height of the frustum, l = slant height of the frustum
If a cone is cut by a plane parallel to the base of the cone, the lower part is called the frustum of the cone.
Slant height of the frustum =l=√(h2+(R-r)2)
Curved surface area of frustum = π(R + r)l
Total surface area of frustum = π(R + r)l + π(R2 + r2)
Volume of the frustum=(1/3)πh(R2+r2+Rr)

Problems:
1) The radius and height of a right circular cylinder are 42 cm & 63 cm respectively. Find its volume.
a) 237564 cm3
b) 349272 cm3
c) 379252 cm3
d) 453213cm3

2) The radius and height of a right circular cone are 28cm & 72 cm respectively. Find its volume.
a) 59136 cm3
b) 62423 cm3
c) 45825 cm3
d) 52924 cm3

3) Find the circumference of a circle whose radius is 91 cm.


a) 572 cm
b) 459 cm
c) 308 cm
d) 407 cm

www.ibpsguide.com | estore.ibpsguide.com | www.sscexamguide.com


31
Complete Quantitative Aptitude Questions

4) Find the curved surface area of a right circular cylinder whose radius & height are 56 cm & 200cm
respectively.
a) 33500 cm2
b) 64320 cm2
c) 75310 cm2
d) 70400 cm2

5) The perimeter of a square is equal to the perimeter of a rectangle of length 42 cm and breadth 60 cm. Find
the circumference of a semicircle whose diameter is equal to the side of the square.
a) 165.265 cm
b) 174. 22 cm
c) 131.14 cm
d) 192. 27 cm

6) There are two circles of different radius such that radius of the smaller circle is two-fifth that of the larger
circle. A square whose area equals 5184 sq cm has its side as thrice the radius of the larger circle. What is
the circumference of the smaller circle?
a) 49.34 cm
b) 54.23 cm
c) 60.34 cm
d) 65.25 cm

7) A cap is in the form of a right circular cone which has base of radius as 35 cm and height equal to 84cm.
Find the approximate area of the sheet required to make 4 such caps.
a) 13567 cm2
b) 33278 cm2
c) 42232 cm2
d) 40040 cm2

8) The barrel of a Ink pen is cylindrical in shape which radius of base as 0.12 cm and is 7 cm long. One such
barrel in the pen can be used to write 450 words. A barrel full of ink which has a capacity of 17 cu. cm can be
used to write how many words approximately?
a) 15423
b) 21342
c) 17645
d) 24147

www.ibpsguide.com | estore.ibpsguide.com | www.sscexamguide.com


32
Complete Quantitative Aptitude Questions

9) A receptacle is in the form of a hemi-spherical bowl on which is mounted a hollow cylinder. The diameter of
the sphere is 22 cm and the total height of receptacle is 35cm, find the capacity of the receptacle.
a) 11915.61 cm3
b) 14238.35 cm3
c) 17854.46cm3
d) 1950.67 cm3

10) A jeep has wheels of diameter 140m. How many revolutions can the wheel complete in 40minutes if the
jeep is travelling at a speed of 220 m/s?
a) 1750
b) 1700
c) 1200
d) 1450

1) Answer: B
we know that volume of Cylinder = πr2h
Volume of the given cylinder = (22/7) * 42*42*63 cm3
= 2444904 / 7 cm3
= 349272 cm3
Therefore Volume of the given cylinder is 349272 cm3

2) Answer: A
volume of a right circular cone = (1/3)πr2h
= (1/3) * 22/7*28*28*72 cm3
= 22 *4*28*24 cm3
Hence, volume of the given cone = 59136 cm3

3) Answer: A
Circumference of circle = 2πr cm
= 2 *22/7*91 cm
= 4004 /7 cm
= 572 cm
Hence the required answer is 572 cm
4) Answer: D
Curved surface area of a right circular cylinder = 2πrh
=2 *22/7*56*200

www.ibpsguide.com | estore.ibpsguide.com | www.sscexamguide.com


33
Complete Quantitative Aptitude Questions

=2 *22*8*200
= 70400cm2
5) Answer: C
Perimeter of a Rectangle = 2 × (Length + Breadth)
Perimeter of a square = 2 * (42+60) = 204cm
So side of square = 204/4 = 51cm
So diameter of semicircle = 51cm
So circumference of a semicircle = πr + 2r
= 22/7 * 51/2 + 51 cm
= 80.14 + 51 = 131.14cm
6) Answer: C
Side of square = √5184 = 72 cm
So radius of larger circle = 1/3 * 72 = 24 cm
So radius of smaller circle = 2/5 * 24 = 9.6 cm
So circumference of smaller circle = 2 * 22/7 * 9.6= 60.34 cm

7) Answer: D
r = 35, h = 84
So slant height, l = √(352+842) = 91 cm
So curved surface area of a cap = πrl = 22/7 * 35 * 91 = 10,010sq. cm
So curved surface area of 4 such cap = 10010*4
= 40,040 sq. cm which is also equal to area of
sheet required to make 4 such caps.

8) Answer: D
Volume of the barrel of Ink pen = πr2h = 22/7 * 0.12*0.12 * 7 =0.3168cu cm
A barrel which has capacity 0.3168 cu. cm can write 450words
So which has capacity 17 cu cm can write = 450/0.3168*17= 24147 words.

9) Answer: A
Diameter is 22, so radius is 11cm
Total height = 35 cm, so height of cylinder = 35-11 = 24cm (because height of hemisphere is same as its
radius)
Capacity of vessel = volume of cylinder + volume of hemisphere
So = πr2h + 2/3 *πr3
= 22/7 * 11 * 11 * 24+ 2/3 * 22/7 * 11 * 11 * 11
= 9126.85+2788.76

www.ibpsguide.com | estore.ibpsguide.com | www.sscexamguide.com


34
Complete Quantitative Aptitude Questions

= 11915.61cu cm

10) Answer: C
Radius of wheel = 140/2 = 70 cm
Distance travelled in one revolution = 2πr = 2 * 22/7 * 70 = 440 cm
Let the number of revolutions made by wheel is x
So total distance travelled = distance travelled in one revolution * number of revolutions
So total distance travelled = 440x cm
40 mins = 40*60 seconds
Speed of jeep = 440x/(40*60)
So 220 = 220x/(20*60)
Solve, x = 1200

11) A wall clock has its minute hand of length 21 cm. What area will it swept in covering 30 minutes?
a) 545 cm2
b) 637 cm2
c) 742 cm2
d) 693cm2

12) The radius of two cylindrical shape are in the ratio 12 : 15and their curved surface areas are in the ratio 9
: 15. What is the ratio of their volumes?
a) 17 : 23
b) 3 : 5
c) 7 : 9
d) 11 : 25

13) The sides of a rectangle cartons are in the ratio 2:3 and its area is 486sq.m find the perimeter of rectangle
a) 75 m
b) 90 m
c) 124m
d) 82m

14) Find the cost of painting a room 39m long and 27m broad with a red carpet 225cm broad at the rate of
Rs.60 per metre
a) Rs.35640
b) Rs.28080
c) Rs.45020

www.ibpsguide.com | estore.ibpsguide.com | www.sscexamguide.com


35
Complete Quantitative Aptitude Questions

d) Rs.15055

15) The length of a rectangle pillow is twice its breadth. If its length is decreased by 25cm and breadth is
increased by 25 cm, the area of the rectangle is increased by 375sq. cm. Find the length of the rectangle.
a) 100 cm
b) 80cm
c) 75cm
d) 55cm

16) If each side of a square is increased by 50%, find the percentage change in its area.
a) 142
b) 135
c) 158
d) 125

17) A girl walking at the rate of 24km per hour crosses a square field diagonally in 36 seconds the area of the
field is
a) 17110sq.m
b) 25485aq.m
c) 28800sq.m
d) 22114sq.m

18) A garden is in the form of a rectangle having its sides in the ratio 4: 5. The area of the garden is (1/2)
hectares. Find the length and breadth of the garden (in metre).
a) 20 ,25
b) 20√10, 25
c) 20, 25√3
d) 20√10, 25√10

19) Find the cost of carpeting a hall 26 m long and 18m broad with a carpet 150 cm width at the rate of Rs.15
per square metre.
a) 4680
b) 4250
c) 4375
d) 4560

www.ibpsguide.com | estore.ibpsguide.com | www.sscexamguide.com


36
Complete Quantitative Aptitude Questions

20) A place 32 m 39cm long and 29m 23 cm broad is to be paved with square tiles. Find the least number of
square tiles required to cover the floor.
a) 2636
b) 1517
c) 2246
d) 1651

11) Answer: D
Length will be the radius, so r = 21cm
Minute hand covers 360o in 60 minutes
So in 30 minutes it covers = 180 degree
Area of arc = angle it makes/360 * πr2
So area covered = 180/360 * 22/7 * 21 * 21 = 693

12) Answer: B
Radius 1 /radius 2=12/15 = 4/5
curved surface area 1/curved surface area 2= 2πr1h1/2πr 2h2 = 9/15=3/5
So h1/h2 = 3/4
Volume1/ Volume2 = πr 12h1/ πr 22h2 = 12/20 = 3/5

13) Answer: B
let 2x and 3x be sides of the rectangle
We know that area of rectangle = l×b
2x × 3x = 486
6x^2= 486
x^2 = 81
x=9
Therefore length = 2x = 2×9 = 18m
Breadth = 3x = 3×9 = 27m
Therefore perimeter = 2 (l+b) = 2(18+27) = 90m

14) Answer:B
Area of the redcarpet = area of the room
= 39×27=1053sq.m
Breadth of the redcarpet = 225cm = 2.25m
Length of the red carpet = area/breadth (A=L×B=>L=A/B)

www.ibpsguide.com | estore.ibpsguide.com | www.sscexamguide.com


37
Complete Quantitative Aptitude Questions

=1053/2.25=468m
Hence, cost of redcarpet = 468×60 =Rs.28,080

15) Answer:B
Let breadth = x. Then, length = 2x Then,
(2x -25) (x + 25) - 2x * x = 375
50x – 25x- 625= 375
25x = 1000
x = 40
Length of the rectangle = 2x = 80 cm.

16) Answer: D
Let each side of the square be a Then, area = a2.
New side =(150a/100) =(3a/2). New area = (3a/2)2 =(9a2)/4.
Increase in area = ((9 a2)/4)-a2 =(5a2)/4
Increase% = [((5a2)/4)×(1/a2)×100] % = 125%

17) Answer: C
Distance covered in 36 seconds = (24×1000/3600)×36 = 240m
Diagonal of square field = 240m
Side of square =a ,then diagonal of that square= √2 a
Hence area of the square = a2= (2402)/2 =28,800sq.m

18) Answer: D
Let length = 4x metres and breadth = 5x metres.
Now, area = (1/2)x 10000 sq.m = 5000sq.m
So, 4x × 5x = 5000
=>20x^2= 5000 => x^2=250
x = 5 √10
Therefore Length = 4x = 20√10 m and Breadth = 5x = 25√10m

19) Answer: A
Area of the carpet = Area of the hall= (26 × 18) m2 = 468m2.
Length of the carpet = (area/width) = 468 ×(2/3) m = 312 m.
Therefore Cost of carpeting = (312 × 15) = Rs.4680

20) Answer: B

www.ibpsguide.com | estore.ibpsguide.com | www.sscexamguide.com


38
Complete Quantitative Aptitude Questions

Area of the place = (3239 * 2923) cm2.


Size of largest square tile = H.C.F. of 3239cm and 2923cm = 79 cm.
Area of 1 tile = (79*79) cm2.
Number of tiles required =(3239 * 2923) / (79*79) =1517

21) Length and width of a rectangle park is 14 m and 7 m respectively. Find the area of circle of maximum
radius
a) 29.62
b) 16.52
c) 38.5
d) 27.85

22) Find the area of a rhombus field one side of which measures 80 cm and one diagonal is 96cm.
a) 4370 cm2
b) 6565 cm2
c) 6320 cm2
d) 6144 cm2

23) The difference between two parallel sides of a trapezium is 24cm, perpendicular distance between them
is 60 cm. If the area of the trapezium is 1500cm2 find the lengths of the parallel side
a) 27, 23
b) 37,13
c) 27,23
d) 37,15

24) Find the length of a string by which a goat must be tethered in order that it may be able to graze an area
of 3426 sq. metres.
a) 33.01
b) 26.7
c) 37.2
d) 28.4

25) The area of a circular land is 35.42 hectares. Find the cost of fencing it at the rate of Rs. 5 per metre
approximately
a) 14457.5
b) 12457.25
c) 10550.57

www.ibpsguide.com | estore.ibpsguide.com | www.sscexamguide.com


39
Complete Quantitative Aptitude Questions

d) 15050.75

26) The diameter of the driving wheel of a lorry is 95 cm. How many revolution, per minute must the wheel
make in order to keep a speed of 75kmph approximately
a) 215
b) 207
c) 232
d) 209

27) A car wheel makes 650 revolutions in covering a distance of 35km. Find the radius of the wheel.
a) 1.5m
b) 3.61m
c) 3.65m
d) 1.45m

28) The inner circumference of a circular car race track, 75 m wide, is 2640 m. Find radius of the outer circle.
a) 495m
b) 456m
c) 463m
d) 482m

29) A hall is half as long again as its broad. The cost of carpeting the hall at Rs.10 /sq m is Rs.540 and the
cost of papering the four walls at Rs.20 per m^2 is Rs.3440. if a door and 2 windows occupy 16 sq.m. find the
dimensions of the room.
b) 6.2 m
b) 7.2m
c) 8.2m
d) 5.2m

30) The difference between two parallel sides of trapezium is 8cm. the perpendicular distance between them
is 38cm. if the area of the trapezium is 950 cm^2. Find the length of the parallel sides.
a) 27,35
b) 31,40
c) 29,21
d) 41,49

21) Answer: C

www.ibpsguide.com | estore.ibpsguide.com | www.sscexamguide.com


40
Complete Quantitative Aptitude Questions

area of circle = πb2/4


= (22/7 *7*7)/4
= 38.5sq.m
Hence the required answer is 38.5sq.m

22) Answer: D
Let other diagonal = 2x cm.
Since diagonals of a rhombus bisect each other at right angles, we have:
(80)2 = (48)2 + (x)2
x2 = 4096 =>x=64
So, other diagonal = 128 cm.
Area of rhombus = (1/2) x (Product of diagonals) =(1/2× 96 x 128) cm2 = 6144 cm2

23) Answer: B
Let the length of two parallel sides of the trapezium be xcm and ycm.
Then, x-y = 24 ------ (1)
And, (1/2) * (x+y) * 60 = 1500
=>(x+y) =(1500 *2)/60 => x+y= 50------- (2)
Solving 1 and 2, we get: x=37, y=13
So, the two parallel sides are 37 cm and 13cm

24) Answer: A
Clearly, the goat will graze a circular field of area 3426sq. metres and radius
equal to the length of the string
Let the length of the string be R metres.
Then, Π(R)2= 3426
R2 = (3426 * (7/22)) = 1090.09
=> R = 33.01
Length of the rope = 33.01 m.

25) Answer: C
Area = (35.42 x 10000) m2= 354200 m2.
ΠR2 = 354200 ⇔ (R)2 = (354200 x (7/22)) ⇔ R = 335.70 m.
Circumference = 2ΠR = (2 x (22/7) x 335.70) m =2110.114 m.
Cost of fencing = Rs. ( 2110.114 x 5) = Rs. 10550.57

26) Answer: D

www.ibpsguide.com | estore.ibpsguide.com | www.sscexamguide.com


41
Complete Quantitative Aptitude Questions

Distance to be covered in 1 min. = (75 X 1000)/(60) m = 1250m.


Circumference of the wheel = (2 x (22/7) x 0.95) m = 5.97 m.
Number of revolutions per min. =(1250/5.97) = 209

27) Answer: B
Distance covered in one revolution =((35X 650)/1000)= 22.75m.

2πR = 22.75=> 2 * (22/7) x R = 22.75


=> R = 22.75 * (7/22) *1/2= 3.61 m
R = 3.61m
Hence the radius of the wheel is 3.61m

28) Answer: A
Let inner radius be r metres.
Then, 2Πr = 2640
r = (2640 x (7/44))
r= 420m.
Radius of outer circle = (420 + 75) m
R = 495m.
Therefore the radius of outer circle is 495m

29) Answer: A
Let breadth = x and length = 3x/2
Area of the floor = 540/10 = 54m^2
X * 3x/2 = 54 =>x^2 = 54*2/3
X^2=36
X=6
Breadth = 6m and length = 3/2 * 6 = 9m
Papered area = (3440/20) = 172 m^2
Area of 1 door and 2 windows = 16 m^2
Total area of 4 walls = 172 + 16 = 188m^2
2(6+9)* H = 188
30H=188
H= 188/30 =6.2m

30) Answer: C
Let the 2 parallel sides of the trapezium be x cm and y cm

www.ibpsguide.com | estore.ibpsguide.com | www.sscexamguide.com


42
Complete Quantitative Aptitude Questions

Then x – y = 8 ========>1
½ (x+y)* 38 = 950
19 (x+y) = 950
X+y = 950/19
X- y = 50 ==========>2
Solving 1 and 2
2x = 58
x= 29, y = 21
so the parallel sides are 29cm and 21 cm.

3. PERMUTATION AND COMBINATION


1) Factorial Notation:
Let n be a positive integer. Then, factorial n, denoted n! is defined as:
n! = n(n - 1)(n - 2) ... 3.2.1.
Examples:
We define 0! = 1.
4! = (4 x 3 x 2 x 1) = 24.
5! = (5 x 4 x 3 x 2 x 1) = 120.

2) Permutations:
Permutation is defined as arrangement of r things that can be done out of total n things. This is denoted
by nPr which is equal to n!/(n-r)!.The different arrangements of a given number of things by taking some or all
at a time, are called permutations.
Examples:
All permutations (or arrangements) made with the letters a, b, c by taking two at a time are
(ab, ba, ac, ca, bc, cb).
All permutations made with the letters a, b, c taking all at a time are:
( abc, acb, bac, bca, cab, cba)

3) Number of Permutations:
Number of all permutations of n things, taken r at a time, is given by:

nP
r = n(n - 1)(n - 2) ... (n - r + 1) = n! / (n – r)!

Examples:
6P = (6 x 5) = 30.
2

7P = (7 x 6 x 5) = 210.
3

www.ibpsguide.com | estore.ibpsguide.com | www.sscexamguide.com


43
Complete Quantitative Aptitude Questions

Cor. number of all permutations of n things, taken all at a time = n!.

4) An Important Result:
If there are n subjects of which p1 are alike of one kind; p2 are alike of another kind; p3 are alike of third kind
and so on and pr are alike of rth kind,
such that (p1 + p2 + ... pr) = n.

5) Combinations:
Combination is defined as selection of r things that can be done out of total n things. This is denoted
by nCr which is equal to n!/r!(n-r)!.Each of the different groups or selections which can be formed by taking
some or all of a number of objects is called a combination.
Examples:
Suppose we want to select two out of three boys A, B, C. Then, possible selections are AB, BC and CA.
Note: AB and BA represent the same selection.
All the combinations formed by a, b, c taking ab, bc, ca.
The only combination that can be formed of three letters a, b, c taken all at a time is abc.
Various groups of 2 out of four persons A, B, C, D are:
AB, AC, AD, BC, BD, CD.
Note that ab ba are two different permutations but they represent the same combination.
Number of Combinations:
The number of all combinations of n things, taken r at a time is:

Note:
nC = 1 and nC0 = 1.
n

nC = nC(n - r)
r

Examples:

www.ibpsguide.com | estore.ibpsguide.com | www.sscexamguide.com


44
Complete Quantitative Aptitude Questions

6) Fundamental Principles of Counting


1. Addition rule : If an experiment can be performed in ‘n’ ways, & another experiment can be
performed in ‘m’ ways then either of the two experiments can be performed in (m+n) ways. This
rule can be extended to any finite number of experiments.
2. Multiplication Rule : If a work can be done in m ways, another work can be done in ‘n’ ways, then
both of the operations can be performed in m x n ways. It can be extended to any finite number of
operations.

7) Difference between Permutations and Combinations and How to identify them


Sometimes, it will be clearly stated in the problem itself whether permutation or combination is to be used.
However if it is not mentioned in the problem, we have to find out whether the question is related to
permutation or combination.
Consider a situation where we need to find out the total number of possible samples of two objects which can
be taken from three objects P, Q, R. To understand if the question is related to permutation or combination,
we need to find out if the order is important or not.
If order is important, PQ will be different from QP, PR will be different from RP and QR will be different from
RQ
If order is not important, PQ will be same as QP, PR will be same as RP and QR will be same as RQ
Hence,
If the order is important, problem will be related to permutations.
If the order is not important, problem will be related to combinations.
For permutations, the problems can be like "What is the number of permutations the can be made", "What is
the number of arrangements that can be made", "What are the different number of ways in which something
can be arranged", etc.
For combinations, the problems can be like "What is the number of combinations the can be made", "What is
the number of selections the can be made", "What are the different number of ways in which something can
be selected", etc.
pq and qp are two different permutations, but they represent the same combination.
Mostly problems related to word formation, number formation etc will be related to permutations. Similarly
most problems related to selection of persons, formation of geometrical figures, distribution of items (there are
exceptions for this) etc will be related to combinations.
Examples:

1) In how many ways can the letters of the word 'NOMINATION' be arranged?
A) 237672
B) 123144
C) 151200

www.ibpsguide.com | estore.ibpsguide.com | www.sscexamguide.com


45
Complete Quantitative Aptitude Questions

D) 150720
E) None of these

2) How many words can be formed by using all letters of the word 'CABIN'?
A) 720
B) 24
C) 120
D) 60
E) None

3) How many arrangements can be made out of the letters of the word 'BIGBOSS' ?
A) 9240
B) 2772
C) 1260
D) 1820
E) 2800

4) In how many different ways can the letters of the word 'GRINDER' be arranged?
A) 2520
B) 1280
C) 3605
D) 1807
E) 1900

5) In how many different ways can any 4 letters of the word 'ABOLISH' be arranged?
a) 5040
b) 840
c) 24
d) 120

Vowel Always together odd / Even position


6) In how many different ways can the letters of the word 'ABOMINABLES' be arranged so that the vowels
always come together?
A) 181045
B) 201440
C) 12880
D) 504020

www.ibpsguide.com | estore.ibpsguide.com | www.sscexamguide.com


46
Complete Quantitative Aptitude Questions

E) 151200

7) In how many different ways can the letters of the word 'POTENCY' be arranged in such a way that the
vowels always come together?
A) 1360
B) 2480
C) 3720
D) 5040
E) 1440

8) In how many different ways can the letters of the word 'RAPINE' be arranged in such a way that the vowels
occupy only the odd positions?
A) 32
B) 48
C) 36
D) 60
E) 120

9) In how many different ways can the letters of the word 'SPORADIC' be arranged so that the vowels always
come together?
A) 120 2
B) 1720
C) 4320
D) 2160
E) 2400

10) In how many different ways can the letters of the word 'VINTAGE' be arranged such that the vowels
always come together?
A) 720
B) 1440
C) 632
D) 364
E) 546

1) Answer: C)
The word 'NOMINATION' contains 10 letters, namely
3N, 2O, 1M, 2I,1A, and 1T.

www.ibpsguide.com | estore.ibpsguide.com | www.sscexamguide.com


47
Complete Quantitative Aptitude Questions

Required number of ways = 10 ! / (3!)(2!)(1!)(2!)(1!)(1!)


= 151200
2) Answer: C)
The word 'CABIN' has 5 letters and all these 5 letters are different.
Total number of words that can be formed by using all these 5 letters
= 5P5
= 5!
= 5×4×3×2×1
= 120
3) Answer: C)
The word 'BIGBOSS' has 7 letters
In these 7 letters, B(2) , I(1), G(1) , O(1),S(2)
Hence, number of ways to arrange these letters
= {7!} / (2!)(1!)(1!)(2!)}
= 5040/4
= 1260
4) Answer: A)
In these 7 letters, 'R' occurs 2 times, and rest of the letters are different.
Hence, number of ways to arrange these letters
= {7!} / {(2!) }
= {7×6×5×4×3×2×1} / {2×1}
= 2520.

5) Answer: B)
There are 7 different letters in the word 'ABOLISH'.
Therefore,
The number of arrangements of any 4 out of seven letters of the word = Number of all permutations
of 7 letters, taken 4 at a time =
nPr = n(n - 1)(n - 2) ... (n - r + 1)
Here, n = 7 and r = 4, then we have
7p4 = 7 x 6 x 5 x 4 = 840.
Hence, the required number of ways is 840.

6) Answer: E)
In the word 'ABOMINABLES', we treat the vowels AOIAE as one letter.
Thus, we have BMNBLS (AOIAE).
This has 7 (6 + 1) letters of which B occurs 2 times and the rest are different.

www.ibpsguide.com | estore.ibpsguide.com | www.sscexamguide.com


48
Complete Quantitative Aptitude Questions

Number of ways arranging these letters = 7! / 2!


= (7×6×5×4×3×2×1) / (2×1)
= 2520

7) Answer: E)
The word 'POTENCY' has 7 different letters.
When the vowels EO are always together, they can be supposed to form one letter.
Then, we have to arrange the letters PTNCY (EO).
Now, 6 (5 + 1 = 6) letters can be arranged in 6! = 720 ways.
The vowels (EO) can be arranged among themselves in 2! = 2 ways.
Required number of ways = (720 x2)
= 1440.
8) Answer: C)
There are 6 letters in the given word, out of which there are 3 vowels and 3 consonants.
Let us mark these positions as under:
(1) (2) (3) (4) (5) (6)
Now, 3 vowels can be placed at any of the three places out 4, marked 1, 3, 5.
Number of ways of arranging the vowels = 3P3 = 3! = 6.
Also, the 3 consonants can be arranged at the remaining 3 positions.
Number of ways of these arrangements = 3P3 = 3! = 6.
Total number of ways = (6 x 6) = 36

9) Answer: C)
The word 'SPORADIC' contains 8 different letters.
When the vowels OAI are always together, they can be supposed to form one letter.
Then, we have to arrange the letters SPRDC (OAI).
Now, 6 letters can be arranged in 6! = 720ways.
The vowels (OAI) can be arranged among themselves in 3! = 6 ways.
Required number of ways = (720 x 6) = 4320.

10) Answer: A)

www.ibpsguide.com | estore.ibpsguide.com | www.sscexamguide.com


49
Complete Quantitative Aptitude Questions

It has 3 vowels (IAE) and these 3 vowels should always come together. Hence these 3 vowels can be
grouped and considered as a single letter.
That is, VNTG(IAE).
Hence we can assume total letters as 5.
Number of ways to arrange these letters
5!=5×4×3×2×1=120
In the 3 vowels (IAE), all the vowels are different. Number of ways to arrange these vowels among
themselves
3! = 3×2×1=6
Total number of ways 120×6=720

11) How many different possible permutations can be made from the word ‘WAGGISH’ such that the vowels
are never together?
A) 3605
B) 3120
C) 1800
D) 1240
E) 2140

12) In how many different ways can the letters of the word "ZYMOGEN" be arranged in such a way that the
vowels always come together?
a) 1440
b) 1720
c) 2360
d) 2240

13) In how many different ways can the letters of the word "XANTHOUS" be arranged in such a way that the
vowels occupy only the odd positions?
a) 2880
b) 4320
c) 2140
d) 5420

14) In how many different ways can the letters of the word "POMADE" be arranged in such a way that the
vowels occupy only the odd positions?
a) 72
b) 144

www.ibpsguide.com | estore.ibpsguide.com | www.sscexamguide.com


50
Complete Quantitative Aptitude Questions

c) 532
d) 36

15) In how many different ways can the letters of the word 'DILUTE' be arranged such that the vowels may
appear in the even places?
a) 36
b) 720
c) 144
d) 24

Choosing from N things M things


16) How many 3 letters words (with or without meaning) can be formed out of the letters of the word,
"PLATINUM", if repetition of letters is not allowed?
a) 742
b) 850
c) 990
d) 336

17) How many 3-letter words can be formed with or without meaning from the letters A , G , M , D , N , and J
, which are ending with G and none of the letters should be repeated?
a) 20
b) 18
c) 25
d) 27

18) Find out the number of ways in which 12 Bangles of different types can be worn in 2 hands?
A) 1260
B) 2720
C) 1225
D) 4096

19) In how many different ways can 6 apple and 6 orange form a circle such that the apple and the orange
alternate?
A) 82880
B) 86400
C) 71200
D) 63212

www.ibpsguide.com | estore.ibpsguide.com | www.sscexamguide.com


51
Complete Quantitative Aptitude Questions

20) There are 7 periods in each working day of a college. In how many ways can one organize 6 subjects
such that each subject is allowed at least one period?
A) 33200
B) 15120
C) 10800
D) 43600

11) Answer: C)
The word ‘WAGGISH’ contains 7 letters of which 1 letter occurs twice
= 7! / 2!
= 2520
No. of permutations possible with vowels always together = 6! * 2! / 2!
= 1440 / 2
= 720
No. of permutations possible with vowels never together = 2520-720
= 1800.

12) Answer: A)
The arrangement is made in such a way that the vowels always come together.
i.e., "ZYMGN(OE)".
Considering vowels as one letter, 6 different letters can be arranged in 6! ways; i.e., 6! = 720 ways.
The vowels "AE" can be arranged themselves in 2! ways; i.e.,2! = 2 ways
Therefore, required number of ways = 720 x 2 = 1440 ways.

13) Answer: A)
There are 8 different letters in the given word "XANTHOUS", out of which there are 3 vowels and 5
consonants.
Let us mark these positions as under:
[1] [2] [3] [4] [5] [6] [7]
Now, 3 vowels can be placed at any of the three places out of 4 marked 1, 3, 5, and 7.
Number of ways of arranging the vowels = 4P3
= 24 ways.
Also, the 5 consonants at the remaining positions may be arranged in 5P5 ways = 5! Ways
= 120 ways.
Therefore, required number of ways = 24 x 120 = 2880 ways.

www.ibpsguide.com | estore.ibpsguide.com | www.sscexamguide.com


52
Complete Quantitative Aptitude Questions

14) Answer: D)
There are 6 different letters in the given word, out of which there are 3 vowels and 3 consonants.
Let us mark these positions as under:
[1] [2] [3] [4] [5] [6]
Now, 3 vowels can be placed at any of the three places out of 3 marked 1, 3 and 5.
Number of ways of arranging the vowels = 3P3
= 3!
= 6 ways.
Also, the 3 consonants can be arranged at the remaining 3 positions.
Number of ways of these arrangements = 3P3
= 3!
= 6 ways.
Therefore, total number of ways = 6 x 6 = 36.

15) Answer: A)
There are 3 consonants and 3 vowels in the word DILUTE.
Out of 6 places, 3 places odd and 3 places are even.
3 vowels can arranged in 3 even places in 3p3 ways = 3! = 6 ways.
And then 3 consonants can be arranged in the remaining 3 places in 3p3 ways = 3! = 6 ways.
Hence, the required number of ways = 6 x 6 = 36.

16) Answer: D)
The word PLATINUM contains 8 different letters.
Required number of words = number of arrangements of 8 letters taking 3 at a time.
= 8p3
=8x7x6
= 56×6
= 336

17) Answer: A)
Since each desired word is ending with G, the least place is occupied with G. So, there is only 1 way.
The second place can now be filled by any of the remaining 5 letters (A , M , D , N , J ). So, there are 5 ways
of filling that place.
Then, the first place can now be filled by any of the remaining 4 letters. So, there are 4 ways to fill.
Required number of words = (1 x 5 x 4) = 20.

18) Answer: D)

www.ibpsguide.com | estore.ibpsguide.com | www.sscexamguide.com


53
Complete Quantitative Aptitude Questions

The first bangle can be worn in any of the 2 hands (2 ways).


Similarly each of the remaining 11 bangles also can be worn in 2 ways.
Hence total number of ways=2×2×2×2×2×2×2×2×2×2×2×2
=2^12
=4096
19) Answer: B)
6 apples can be arranged in (6-1)! Ways
Now there are 6 positions in which 6 orange can be placed.
This can be done in 6! ways.

Required number of ways = (6-1)! × 6!


= 5! × 6!
= 120 × 720
= 86400

20) Answer: B)
6 subjects can be arranged in periods in 7P6 ways.
Remaining 1 period can be arranged in 6P1 ways.
Two subjects are alike in each of the arrangement. So we need to divide by 2! to avoid over counting.
Total number of arrangements = (7P6 x 6P1)/2!
= 5040 × 6 / 2
= 30240 / 2
= 15120

21) How many 5-letter code words are possible using last 10 letter of the English alphabet , if no letter can be
repeated ?
a) 30240
b) 25440
c) 45640
d) 32940

22) In how many ways can a group of 10 men and 5 women be made out of a total of 12 men and 10 women?
A) 16632
B) 15290
C) 25126
D) 34845
E) 38135

www.ibpsguide.com | estore.ibpsguide.com | www.sscexamguide.com


54
Complete Quantitative Aptitude Questions

23) A box contains 2 pink balls, 3 brown balls and 4 blue balls. In how many ways can 3 balls be drawn from
the box, if at least one brown ball is to be included in the draw?
A) 32
B) 48
C) 64
D) 96
E) None

24) Pramoth has 12 friends and he wants to invite 7 of them to a party. How many times will 4 particular
friends never attend the party?
A) 8
B) 7
C) 12
D) 15

25) In how many ways can 8 different ballons be distributed among 7 different boxes when any box
can have any number of ballons?
A) 5^4-1
B) 5^4
C) 4^5-1
D) 7^8

Problems on Selecting Items


26) A question paper has two parts A and B, each containing 12 questions. If a student needs to choose 10
from part A and 8 from part B, in how many ways can he do that?
A) 32670
B) 36020
C) 41200
D) 29450

27) A bowl contains 8 violet, 6 purple and 4 magenta balls. Three balls are drawn at random. Find out the
number of ways of selecting the balls of different colours?
A) 362
B) 2 48
C) 122
D) 192

www.ibpsguide.com | estore.ibpsguide.com | www.sscexamguide.com


55
Complete Quantitative Aptitude Questions

28) An shopkeeper has 15 models of cup and 9 models of saucer. In how many ways can he make a pair of
cup and saucer?
A) 100
B) 80
C) 110
D) 135

29) There are 10 orange, 2 violet and 4 purple balls in a bag. All the 16 balls are drawn one by one and
arranged in a row. Find out the number of different arrangements possible.
A) 25230
B) 23420
C) 120120
D) 27720

30) In how many ways can a team of 6 persons be formed out of a total of 12 persons such that 3 particular
persons should not be included in any team?
A) 56
B) 112
C) 84
D) 128

21) Answer: A)
The number of 5 letter code words out of the last 10 letters of the English alphabets are = 10× 9×
8 × 7× 6
= 80 × 63× 6
= 30240 ways.
22) Answer: A)
Required number of ways = 12C
10 x 10C5
= 66 × 252
= 16632
23) Answer: C)

www.ibpsguide.com | estore.ibpsguide.com | www.sscexamguide.com


56
Complete Quantitative Aptitude Questions

24) Answer: A)
Remove the 4 particular friends and invite 7 friends from the remaining 8 (12-4) friends . this can be
done in 8C7 ways.
Therefore , required number of ways = = 8C7
= 8C1
=8
25) Answer: D)
Here n = 7, k = 8.
Hence, required number of ways = n^k
=7^8

26) Answer: A)
Number of ways to choose 10 questions from part A = 12C10
Number of ways to choose 8 questions from part B = 12C8
Total number of ways= 12C10 × 12C8
= 12C2 × 12C4 [∵ nCr = nC(n-r)]
={12×11}/{2×1}X{12×11×10×9} / {4×3×2×1}
=66×495
=32670
27) Answer: D)
1 violet ball can be selected is 8C1 ways.
1 purple ball can be selected in 6C1 ways.
1 magenta ball can be selected in 4C1 ways.
Total number of ways = 8C1 × 6C1 × 4C1
= 8×6×4
= 192
28) Answer: D)
He has 15 patterns of cup and 9 model of saucer
A cup can be selected in 15 ways.
A saucer can be selected in 9 ways.

www.ibpsguide.com | estore.ibpsguide.com | www.sscexamguide.com


57
Complete Quantitative Aptitude Questions

Hence one cup and one saucer can be selected in 15×9 ways =135 ways

29) Answer: C)
Number of different arrangements possible
= {16!} / {10! 2! 4!}
= {16×15×14×13×12×11×10×9×8×7×6×5×4×3×2 } /
{(10×9×8×7×6×5×4×3×2 ) (2) (4×3×2)}}
= {16×15×14×13×12×11} / {(2)(4×3×2)}
= {8×5×7×13×3×11}
= 120120
30) Answer: C)
Three particular persons should not be included in each team.
i.e., we have to select remaining 6- 3= 3 persons from 12-3 = 9 persons.
Hence, required number of ways = 9C3
= {9×8 × 7} / {3 × 2 × 1}
= 504 / 6
= 84
Circular Permutations
There are two cases of circular-permutations:-
(a) If clockwise and anti clock-wise orders are different, then total number of circular-permutations is given by
(n-1)!
(b) If clock-wise and anti-clock-wise orders are taken as not different, then total number of circular-
permutations is given by (n-1)!/2!
31) In how many ways can 6 girls be seated in a rectangular order?
A) 60
B) 120
C) 5040
D) 720

32) In how many ways can 10 stones can be arranged to form a bangles?
A) 267720
B) 284360
C) 125380
D) 181440

33) In a birthday party, every person shakes hand with every other person. If there was a total of 66
handshakes in the party, how many persons were present in the party?

www.ibpsguide.com | estore.ibpsguide.com | www.sscexamguide.com


58
Complete Quantitative Aptitude Questions

A) 9
B) 8
C) 7
D) 12

34) A school has 9 maths teachers and 6 science teachers. In how many ways can a team of 4 maths
teachers be formed from them such that the team must contain exactly 1 science teacher?
A) 800
B) 720
C) 680
D) 504

35) In how many ways can 8 different balls be distributed in 6 different boxes can contain any number of balls
except that ball 4 can only be put into box 4 or 5 ?
A) 2×5^6
B) 2×6^7
C) 2×5^4
D) 2×4^7

31) Answer: B)
Number of arrangements possible = (6-1)!
= 5!
= 5×4×3×2×1
= 120
32) Answer: D)
Number of arrangements possible = {1} / {2} X (10-1)!
= {1} / {2}X 9!
= {1} / {2} X 9×8×7× 6×5×4×3×2×1
= {1 / 2 } ×362880
= 181440
33) Answer: D)
Assume that in a party every person shakes hand with every other person
Number of hand shake = 66
Total number of hand shake is given by nC2
Let n = the total number of persons present in the party
nC =66
2

n (n-1) / 2 = 66

www.ibpsguide.com | estore.ibpsguide.com | www.sscexamguide.com


59
Complete Quantitative Aptitude Questions

n² - n = 2 × 66
n² - n – 132= 0
n= 12 , -11
But we cannot take negative value of n
So, n = 12
Therefore number of persons in the party = 12

34) Answer: D)
The team should have 4 maths teachers. But the team must contain exactly 1 science teacher.
Hence, select 3 maths teachers from 9 maths teachers and select 1 science teachers from 6 science
teachers.
Number of ways this can be done = 9C3 × 6C1
={9×8×7}/{3×2×1}X6
= 504 / 6 × 6
= 84 × 6
=504
35) Answer: B)
1st ball can be put in any of the 6 boxes.
2nd ball can be put in any of the 6 boxes.
3rd ball can be put in any of the 6 boxes.
Ball 4 can only be put into box 4 or box 5. Hence, 4th ball can be put in any of these 2 boxes.
5th ball can be put in any of the 6 boxes.
6th ball can be put in any of the 6 boxes.
7th ball can be put in any of the 6 boxes.
8th ball can be put in any of the 6 boxes.
Hence, required number of ways = 6×6×6×2×6×6×6×6
= 2×6^7

4. MIXTURE AND ALLIGATION


MIXTURE
When two or more components are mixed in a certain ratio, a mixture is created.

Types of mixtures:

Simple Mixtures:-
When two or more different ingredients are mixed together, a simple mixture is formed.
Eg,. sugar and water, wine with water, milk with water etc.

www.ibpsguide.com | estore.ibpsguide.com | www.sscexamguide.com


60
Complete Quantitative Aptitude Questions

Compound Mixtures:-
When two or more simple mixtures are mixed together, a compound mixture is formed.
Eg., three or more than types of tea variety, rice variety etc.
Basically mixing of two quantity i.e. cheaper quantity and nearer quantity mean price always
falls between these two quantities from which respective ratio is calculated.

Mean Price:
Mean price is also known as median price or cost price of mixture of quantities taken, which lies
between quantities taken for value consideration.

Allegation:
Allegation is the rule that enables us to find the ratio in which two or more ingredients at the given
price must be mixed to produce a mixture of a desired price.

Basic formula

TYPE 1:
1) In what ratio should a shopkeeper mix two types of rice, one costing 40 rupees/kg and another costing 20
rupees/kg to get a rice variety costing 28 rupees/kg
a) 3:2
b) 2:3
c) 4:3
d) 2:7
e) None

www.ibpsguide.com | estore.ibpsguide.com | www.sscexamguide.com


61
Complete Quantitative Aptitude Questions

2) Some amount out of Rs.8500 was lent at 5% per annum and the remaining was lent at 3% per annum. If
the total simple interest from both the fractions in4 years was Rs.1500, the sum lent at 5% per annum was
a) Rs. 2400
b) Rs. 2200
c) Rs. 2000
d) Rs. 6000

3) In what ratio should wheat at Rs12.60 per kg be mixed with wheat at Rs. 14.60 per kg so that the mixture
be worth Rs.13 per kg ?
a) 6:8
b) 8:7
c) 4:3
d) 2:7
e) 4:1

4) In what ratio must water be mixed with milk costing Rs.30 per litre in order to get a mixture worth of Rs.18
per litre?
a) 3:2
b) 2:3
c) 4:3
d) 2:7
e) 1:2

5) How many kg of dal at Rs.8.40 per kg be mixed with32 kg of dall at Rs10.20 per kg to get a mixture worth
Rs.9.60 per kg?
a) 23
b) 49
c) 43
d) 72
e) 16

6) In 1 kg mixture of iron and carbon, 40% is carbon. How much iron should be added so that the proportion
of carbon becomes 20%
a)1.5kg
b)2kg
c)3kg
d)5kg

www.ibpsguide.com | estore.ibpsguide.com | www.sscexamguide.com


62
Complete Quantitative Aptitude Questions

e)1kg

7) Find the ratio in which Nuts at Rs. 5.40 a kg be mixed with Nuts at Rs. 3.20 a kg to produce a mixture
worth Rs. 4.60 a kg.
a) 9:2
b) 2:3
c) 4:3
d) 3:2
e) 7:4

8) In 20 litres of a mixture, the ratio of sugar syrub to honey syrub is 5:1. In order to make the ratio of sugar
syrub to honey syrub as 3:1, the quantity of honey syrub that should be added to the mixture will be
A) 5 2/3
B) 4 1/3
C) 6 2/3
D) 2 2/9

1) B

Here also we can use Alligation as follows


X = 40-28 = 4 ; y = 28-20 = 8
The ratio between the type 1 and type 2 rice is 4:6 or 2: 3

2) D
Total simple interest received , I = Rs.1500
Principal , p = 8500
period, n = 4 years
Rate of interest, r = ?
Simple Interest, I=PNR/100
1500 = ( 8500 ×4× r )/ 100
= 1500/ 85×4
= 75/17

www.ibpsguide.com | estore.ibpsguide.com | www.sscexamguide.com


63
Complete Quantitative Aptitude Questions

required ratio is: 12:5


Given that total amount is Rs.8500.
Therefore, the amount lent at 5% per annum (part1 amount) = 8500x12/17
= 6000
3) E

Thus the required ratio = 160 :40 = 4:1

4) B
By the rule of allegation

Ratio of water to milk = 12:18 = 2:3

www.ibpsguide.com | estore.ibpsguide.com | www.sscexamguide.com


64
Complete Quantitative Aptitude Questions

5) E
By the rule of allegation ,we have

Quantity of 1st kind dall : Quantity of 2nd kind dall = 60 : 120


=> Quantity of 1st kind dall:32 = 1:2
Quantity of 1st kind dall =32 *1/2 = 16

6) E

Quantity of the mixture : Quantity of iron = 20 : 20 = 1 : 1


Given that quantity of the mixture = 1 kg
Hence quantity of iron to be added = 1 kg

7) E
By the rule of allegation

www.ibpsguide.com | estore.ibpsguide.com | www.sscexamguide.com


65
Complete Quantitative Aptitude Questions

Required ratio = 140 : 80


= 7:4
8) D
By the rule of alligation

Quantity of honey : Quantity of mixture =1/12:3/4}=1:9

Given that quantity of mixture = 20 litre


=>Quantity of honey : 20 = 1 : 9
=> Quantity of honey = 20 x1/9
= 2 2/9 litre
Type II: Percentage of profit or loss

9) How many kg of cumin seeds at 62 rs per kg. must a man mix with 15 kg of cumin seeds at 20 rs per kg.
So that he may , on selling the mixture at 40 rs per kg ,gain 25% on the outlay ?
a) 62
b) 25
c) 42
d) 15

www.ibpsguide.com | estore.ibpsguide.com | www.sscexamguide.com


66
Complete Quantitative Aptitude Questions

10) A merchant has 1500 kg of salt part of which he sells at 12% profit and the rest at 18% profit. He gains
16% on the whole. The Quantity sold at 18% profit is
a) 3020
b) 2000
c) 1000
d) 2700
e) 6003

11) How many kgs of Ponni rice costing Rs.21/kg should a shopkeeper mix with 12.5 kgs of ordinary rice
costing Rs.12 per kg so that he makes a profit of 25% on selling the mixture at Rs.20/kg?
a) 20
b) 23
c) 43
d) 27
e) 10

12) How many litres of water should be added to a 30 litre mixture of milk and water containing milk and
water in the ratio of 3:7 such that the resultant mixture has 40% water in it?
a) 3
b) 5
c) 6
d) 7
e) 9
.
13) How many kilograms of sugar costing Rs. 17.2 per kg must be mixed with 54 kg of sugar costing Rs.14
per kg so that there may be a gain of 20 % by selling the mixture at Rs. 18.48 per kg?
a) 63
b) 83
c) 73
d) 27
e) 42

14) In what ratio must coffee powder worth Rs. 40 per kg be mixed with coffee powder worth Rs. 45 a kg such
that by selling the mixture at Rs. 48.40 a kg ,there can be a gain 10%?
a) 3:2
b) 2:3
c) 4:3

www.ibpsguide.com | estore.ibpsguide.com | www.sscexamguide.com


67
Complete Quantitative Aptitude Questions

d) 2:7
e) 1:4

15) 5 litre of coconut oil is added to 15 litre of a solution containing 40% of groundnut oil. The percentage of
groundnut in the new mixture is
a) 32
b) 23
c) 43
d) 33
e) 30

16) Suprith bought 40 kg of rava at the rate of Rs.8.50 per kg and 55 kg at the rate of Rs.8.75 per kg. He
mixed the two. Approximately at what price per kg should he sell the mixture to make 40% profit at the cost
price?
a) Rs.12
b) Rs.14
c) Rs.25
d) Rs.21

9) A
Cost price of the mixture = S.P × 100 / (100+gain %)
= 40 × 100/(100+25)
= 40 × 100 / 125
= 32 rs per kg

Required ratio = 12/ 30 = 2:5


2:5=x:15
So 6 kg of cumin at rs. 62

www.ibpsguide.com | estore.ibpsguide.com | www.sscexamguide.com


68
Complete Quantitative Aptitude Questions

10) C

So , ratio of 1st and 2nd parts = 2:4 = 1:2


Quantity of 2nd kind = (2/3 x 1500) kg
= 1000 kg

11) E
As the trader makes 25% profit by selling the mixture at Rs.20/kg, his cost per kg of the mixture = Rs.16/kg.
C.P of 1 kg of rice of 1st kind = Rs. 21
C.P of 1 kg of rice of 2nd kind = Rs. 12
Mean price = Rs. 16
By the rule of alligation

Let the amount of ponni rice being mixed be x kgs


‘ 4:5 = x : 12.5
X = 10 kg

12) B
60 litres of the mixture has milk and water in the ratio 3:7.
i.e. the solution has 21 litres of milk and 9 litres of water.
When you add more water, the amount of milk in the mixture remains constant at 21 litres.
In the first case, before addition of further water, 21 litres of milk accounts for 70% by volume. After water is
added, the new mixture contains 60% milk and 40% water.
Therefore, the 21 litres of milk accounts for 60% by volume.

www.ibpsguide.com | estore.ibpsguide.com | www.sscexamguide.com


69
Complete Quantitative Aptitude Questions

Hence, 100% volume =21/0.6 =35 litres.


We started with 30 litres and ended up with 35 litres.
Therefore, 5 litres of water was added.

13) E
Selling Price (SP) of 1 kg mixture = Rs. 18.48

Profit = 20%

Cost Price(CP) of 1 kg mixture =(s.p*100)/120

= ( 18.48 × 100) /120


= ( 184800/120)
= 15.4

i.e., to get a cost price of 15.4


The sugars of kind1 and kind2 should be mixed in the ratio 1.4 : 1.8
= 14 : 18
=7:9
Suppose X kg of kind1 sugar is mixed with 54 kg of kind2 sugar.
Then x : 54 = 7 : 9
9x=54×7
9x = 378
X = 42
14) E
Cost Price(CP) of 1 kg mixture = Rs. 68.20
Profit = 10%
Cost Price(CP) of 1 kg mixture = 48.40×100/110
= 4840 /110
= 44

www.ibpsguide.com | estore.ibpsguide.com | www.sscexamguide.com


70
Complete Quantitative Aptitude Questions

Hence required ratio = 1 : 4

15) E
We have a 15 litre solution containing 40% of groundnut in the coconut oil.
=> Quantity of groundnut in the solution =(15* 40)/100= 6
Now 5 litre of coconut oil is added to the solution.
=> Total quantity of the new solution = 15 + 5= 20
Percentage of groundnut in the new solution = 6*100/20 = 600/20
= 30%

16) A
CP = 40 × 8.50 + 55 ×8.75
= 340 + 481.25
= 821.25
Profit = 40%
SP= (100 +40) /100 × c.p
= 1.4 × 821.25
Total quantity = 40 + 55 = 95 kg
SP per kg =(1.4 x821.25) / 95} = 12

Type: III Drawn and replacement


17) A vessel is filled with liquid, 4 parts of which are water and 5 parts syrup. How much of the mixture must
be drawn off and replaced with water so that the mixture may be half water and half syrup?
a) 1/3
b) 1/5
c) 1/4
d) 1/7
e) 1/10

18) From a cask of milk containing 45 litres, 9 litres are drawn out and the cask is filled up with water.If the
same process is repeated a second, then a third time, what will be the number of litres of milk left in the cask?
a) 31.2

www.ibpsguide.com | estore.ibpsguide.com | www.sscexamguide.com


71
Complete Quantitative Aptitude Questions

b) 20.9
c) 23.04
d) 12.39
e) 15.36

.
19) How many litres of a 36 litre mixture containing milk and water in the ratio of 4 : 6 be replaced with pure
milk so that the resultant mixture contains milk and water in equal proportion?
a) 3
b) 2
c) 4
d) 7
e) 6

20) A container contains 55 litres of milk. From this container 5.5 litres of milk was taken out and replaced by
water. This process was repeated further two times. How much milk is now contained by the container?
a) 32
b) 29.16
c) 43
d) 27.12
e) 40.09

21) 12 litres are drawn from a cask full of wine and is then filled with water. This operation is performed three
more times. The ratio of the quantity of wine now left in cask to that of the water is 81 : 256. How much wine
did the cask originally hold?
a) 32
b) 23
c) 43
d) 27
e) 48

22) A jar full of whiskey contains 52% alcohol. A part of this whisky is replaced by another containing 31%
alcohol and now the percentage of alcohol was found to be 38%. The quantity of whisky replaced is
a) 3/2
b) 2/3
c) 4/3
d) 2/7

www.ibpsguide.com | estore.ibpsguide.com | www.sscexamguide.com


72
Complete Quantitative Aptitude Questions

e) None

23) A sample of x litres from a container having a 60 litre mixture of milk and water containing milk and water
in the ratio of 2 : 3 is replaced with pure milk so that the container will have milk and water in equal
proportions. What is the value of x?
a) 6 litres
b) 10 litres
c) 30 litres
d) None of these

17) E
Suppose the vessel initially contains 9 litres of liquid.
Let x litres of this liquid be replaced with water.
Quantity of water in new mixture = 4 – 4x/9 + x litres
Quantity of syrup in new mixture = 5 – 5x /9
(4 - 4x /9 + x) = ( 5 – 5x / 9)
36-4x + 9x = 45 – 5x
36 + 5x = 45 – 5x
10 x = 45 – 36
10 x = 9
X = 9 / 10
So part of the mixture replaced = 9 / 10 × 1 / 9
= 1 / 10

18) C
where x is the initial quantity of milk in the cask y is the quantity of milk withdrawn in each process and n is the
number of processes.
Initial quantity:45
Withdrawn quantity=9
number of times repeated=3

Hence from the above rule it can be say that,


Formula,=[a(1-b/a)n]
Quantity of milk left after the 3rd operation =45*(45-9)/45)^3
= 23.04 litres.

19) E

www.ibpsguide.com | estore.ibpsguide.com | www.sscexamguide.com


73
Complete Quantitative Aptitude Questions

The mixture contains 40% milk and 60% water in it.


That is 14.4 litres of milk and 21.6 litres of water.
Now we are replacing the mixture with pure milk so that the amount of milk and water in the mixture
is 50% and 50%.
That is we will end up with 18 litres of milk and 18 litres of water.
Water gets reduced by 3.6 litres.
To remove 3.6 litres of water from the original mixture containing 60% water,
we need to remove ={3.6}/{0.6} litres of the mixture
= 6 litres

20) E
Where x is the initial quantity of milk in the cask y is the quantity of milk withdrawn in each process and n is
the number of processes.
Initial quantity:55
Withdrawn quantity=5.5 number of times repeated=3
Hence from the above rule it can be say that,
Formula,=[a(1-b/a)n]
Quantity of milk left after the 3rd operation =55*((55-5.5)/55) ^3
=40.09litres.
21) E
Let initial quantity of wine = x litre

After a total of 4 operations, quantity of wine


x(1-(y/x))^n = x(1-12/x})^4

Given that after a total of 4 operations, the ratio of the quantity of wine left in cask to that of water = 81 : 256
x (1-12 / x)4/ x = 81/ 256
(1-12/x)^4 = (3 /4)^4
X – 12 / x = 3 / 4
4x – 48 = 3x
X = 48

22) B
Concentration of alcohol in 1st Jar = 52%
Concentration of alcohol in 2nd Jar = 31%
After the mixing, Concentration of alcohol in the mixture = 38%

www.ibpsguide.com | estore.ibpsguide.com | www.sscexamguide.com


74
Complete Quantitative Aptitude Questions

By rule of allegation,
52% 31%
38%
7 14

Hence ratio of 1st and 2nd quantities = 7 : 14 = 1 : 2


i.e., 2/3 part of the whisky is replaced.

23) C
Another way for solving this sum is by choice
The mixture of 60 litres has in it 24 litres of milk and 36 litres of water. (2 : 3 :: milk
water)
When you remove x litres from it, you will remove 0.4 x litres of milk and 0.6 x litres of water from it.
assume(2). According to this choice, x = 10.
So, when one removes, 10 litres of the mixture, one is removing 4 litres of milk and 6 litres of water.
Therefore, there will be 20 litres of milk and 30 litres of water in the container.
Now, when you add 10 litres of milk, you will have 30 litres of milk and 30 litres of water – i.e. milk and water
are in equal proportion.

Type IV: Two variety of mixture with ratio

24) Two vessels A and B contain sugar and rava in the ratio 4 : 3 and 2 : 3 respectively. Find the ratio in
which these mixtures be mixed to obtain a new mixture in vessel C containing sugar and rava in the ratio 1 :
2?
a) 2:9
b) 3:2
c) 7:9
d) 2:7
e) 7:4

25) Vessel A contains milk and water in the ratio 3:2. Vessel B contains milk and water in the proportion 4:5.In
what proportion should quantities be taken from A & B to form a mixture in which milk and water are in the
ratio 7:2?
a) 3:2
b) 15:2
c) 4:3

www.ibpsguide.com | estore.ibpsguide.com | www.sscexamguide.com


75
Complete Quantitative Aptitude Questions

d) 8:15
e) 6:18

26) In what ratio must a person mix three kinds of dall costing him Rs 12.25,Rs 12.50 and Rs 12.75 per Kg so
that the mixture may be worth Rs 12.65 per Kg?
a) 11:7:1
b) 22:3:6
c) 14:3:4
d) 2:7:4
e) 3:12:32

27) A milk vendor has 2 cans of milk. The first contains 25% water and the rest milk. The second contains
50% water. How much milk should he mix from each of the containers so as to get 12 litres of milk such that
the ratio of water to milk is 3 : 5?
a) 2
b) 3
c) 4
d) 7
e) 6

28) The cost of Type 1 material is Rs. 50per kg and Type 2 material is Rs.75 per kg. If both Type 1 and Type
2 are mixed in the ratio of 3:2 then what is the price per kg of the mixed variety of material?
a) 32
b) 38
c) 53
d) 62
e) 60

29) A container contains a mixture of two liquids P and Q in the ratio 7 : 5. When 9 litres of mixture are drawn
off and the container is filled with Q, the ratio of P and Q becomes 7 : 9. How many litres of liquid P was
contained in the container initially?
a) 12
b) 21
c) 93
d)17
e) None

www.ibpsguide.com | estore.ibpsguide.com | www.sscexamguide.com


76
Complete Quantitative Aptitude Questions

24) E
Let Cost Price(CP) of 1 kg sugar be Rs.1

Quantity of sugar in 1kg mixture from vessel A=4/7


Cost Price(CP) of 1kg mixture from vessel A = Rs. =4/7

Quantity of sugar in 1 kg mixture from vessel B =2/5


Cost Price(CP) of 1 kg mixture from vessel B = Rs. =2/5

Quantity of sugar to be obtained in 1 kg mixture from vessel C=1/2


Cost Price(CP) of 1 kg mixture from vessel C(Mean Price) = Rs.=1/2

Mixture from Vessel A : Mixture from Vessel B


1/10 : 1/14
7:4

25) D
For this question, we will consider the proportion of milk in each mixture.
In Vessel A, the proportion of milk in 3/((3+2) )=3/5.
vessel B, the proportion of milk is 4/(4+5) = 4/9

The ratio is (3/9) : (8/45) = 8:15

www.ibpsguide.com | estore.ibpsguide.com | www.sscexamguide.com


77
Complete Quantitative Aptitude Questions

26) E
Step1: Mix dall of first and third kind to get a mixture worth Rs 12.25 per Kg.
C.P of 1 Kg dall of 1st kind 1225p
C.P of 1 Kg dall of 3rd kind 1275p

So they must be mixed in the ratio 1 : 4


Step 2: similarly Mix dall of 1st kind and 2nd kind to obtain a mixture worth of Rs.12.65per Kg

So the Mixed ratio = 3:8


Thus the quantities of dall 1st : 2nd : 3rd = 3:12:32

27) E
Let cost of 1 litre milk be Rs.1
Milk in 1 litre mix in 1st can =3/4 litre
Cost Price(CP) of 1 litre mix in 1st can = Rs. 3/4

Milk in 1 litre mix in 2nd can 1/2 litre.


Cost Price(CP) of 1 litre mix in 2nd can = Rs. 1/2

Milk in 1 litre of the final mix = 5/8


Cost Price(CP) of 1 litre final mix =Rs. 5/8
=> Mean price 5/8

By rule of allegation,

www.ibpsguide.com | estore.ibpsguide.com | www.sscexamguide.com


78
Complete Quantitative Aptitude Questions

mix in 2nd can :mix in 1st can =1/8:1/8=1:1


ie, from each can,1/2* 12 = 6 litre should be taken.

28) E
Cost Price (CP) of Type 1 material is Rs. 50 per kg
Cost Price (CP) of Type 2 material is Rs. 75 per kg
Let Cost Price(CP) of resultant mixture be Rs.x per kg

Type 1 material : Type 2 material = 3/2


75-x : x-50 =3/2
X = 60

29) B
Let initial quantity of P in the container be 7x
and initial quantity of Q in the container be 5x

Now 9 litres of mixture is drawn off from the container.


Quantity of P in 9 litres of the mixture drawn off
=9*(7/12) = 63/12=21/4
Quantity of Q in 9 litres of the mixture drawn off
=9*(5/12) =45/12=15/4
Hence,

www.ibpsguide.com | estore.ibpsguide.com | www.sscexamguide.com


79
Complete Quantitative Aptitude Questions

Quantity of P remaining in the mixture after 9 litres is drawn off


=7x-21/4
Quantity of Q remaining in the mixture after 9 litres is drawn off
=5x-15/4

Since the container is filled with Q after 9 litres of mixture is drawn off, quantity of Q in the mixture
= 5x-(15/4)+9=5x+(21/4)
Given that the ratio of P and Q becomes 7 : 9
(7x-21/4) : (5x+(21/4)) = 7 : 9
x=16* 21/4*28
Litres of P contained in the container initially
=7x = 21

Type IV: Three variety


30) Redgram worth Rs. 112 per kg and Rs.123 per kg are mixed with a third variety in the ratio 1: 1 : 2. If the
mixture is worth Rs. 142 per kg, the price of the third variety per kg will be
a) 153
b) 175.50
c) 43.9
d) 227.9
e) 166.5

31) In what ratio must a person mix three kinds of musted seeds costing Rs.65/kg, Rs.70/kg and Rs.105 /kg
so that the resultant mixture when sold at Rs.96/kg yields a profit of 20%?
a) 1:2:4
b) 3:7:6
c) 1:4:2
d)1:8:6
e) 40 : 8 : 25

32) A merchant mixes three varieties of rice costing Rs.40/kg, Rs.48/kg and Rs.60/kg and sells the mixture at
a profit of 40% at Rs.60 / kg. How many kgs of the second variety will be in the mixture if 4 kgs of the third
variety is there in the mixture?
a) 1 kg
b) 20 kgs
c) 3 kgs
d) 6 kgs

www.ibpsguide.com | estore.ibpsguide.com | www.sscexamguide.com


80
Complete Quantitative Aptitude Questions

e) 8kgs

30) E
Since first second varieties are mixed in equal proportions
so their average price = Rs.(112 + 123/2)
= Rs. 117.5
So, the mixture is formed by mixing two varieties, one at Rs. 117.50 per kg and the other at say, Rs.
x per kg in the ratio 2 : 2, i.e., 1 : 1. We have to find x.
By the rule of allgation

X – 142 / 24.5 = 1
X = 24.5 + 142
X = 166.5 per kg.
31) E
The resultant mixture is sold at a profit of 20% at Rs.96/kg
i.e. 1.2 (cost) = Rs.96 => Cost = = Rs.80 / kg.
Let the three varities be A, B, and C costing Rs.65, Rs.70 and Rs.105 respectively.
The mean price falls between B and C.
Hence the following method should be used to find the ratio in which they should be mixed.

www.ibpsguide.com | estore.ibpsguide.com | www.sscexamguide.com


81
Complete Quantitative Aptitude Questions

A:C=4:5 B:C=2:5
The resultant ratio A : B : C :: 40 : 8 : 25.

32) D
If the selling price of mixture is Rs.60/kg and the merchant makes a profit of 20%, then the cost price of the
mixture = 60/1.2 = Rs.50/kg.
We need to find out the ratio in which the three varieties are mixed to obtain a mixture costing Rs.50 /kg.
Let variety A cost Rs.40/kg, variety B cost Rs.48 / kg and variety C cost Rs.60/kg. The mean desired price
falls between B and C.
Step 1: Find out the ratio A : C
Using allegation rule

A:C = 1:1 B:C = 5:1


Step 2: QC is found by adding the value of QC in step 1 and step 2 = 1 + 5 = 6
Therefore, the ratio = 1 : 25 : 5

5. BOATS AND STREAMS

Type 1
1) A boat takes 38 hrs for travelling downstream from point p to point q and coming back to a point r midway
between p and q. If the velocity of the stream is 8 km/hr and the speed of the boat in still water is 28 km/hr.
What is the distance between p and q?
A) 720
B) 640
C) 510
D) 450

2) A boat takes 27 hrs to travel a distance upstream and takes 9hrs to travel the same distance downstream.
If the speed of the boat in still water is 12km/hr, then what is the velocity of the stream?
A) 8km/hr
B) 6km/hr

www.ibpsguide.com | estore.ibpsguide.com | www.sscexamguide.com


82
Complete Quantitative Aptitude Questions

C) 4km/hr
D) None of these

3) There is a bridge besides a river. Two friends Arun and varun started their journey from place L, moved to
the garden located at another place M & then returned to place L. Arun moves by swimming at a speed of 30
km/hr while Varun sails on a boat at a speed of about 24km/hr. If the flow of water current is at the speed of
12km/hr, what will be the average speed of boat sailor?
A) 36
B) 18
C) 24
D) 48

4) A boat covers 12 km upstream and 18km downstream in 3hrs while it covers 18 km upstream and 12km
downstream in 3 ¼ hrs the velocity of the the boat upstream and downstream?
A) 4,8
B) 8,12
C) 12,16
D) 3,9

5) At the same speed a boat travelling 50km upstream and 78km downstream in 16hrs. Also it can travel
70km upstream and 104km downstream, in 22hrs at the same speed of the stream is.
A) 5km/hr
B) 3km/hr
C) 4km/hr
D) 2km/hr

TYPE 2:
6) Thanu can travel 24 miles downstream in a certain river in 12hrs less than it takes him to travel the same
distance upstream. But if he could double his usual rowing rate for his 48 mile round trip, the downstream 24
miles would then take only 2hr less than the upstream 24miles. What is the speed of the current in miles per
hour?
A) 2 / 3 miles/hr
B) 8 / 3 miles/hr
C) 7 / 5 miles/hr
D) 3 / 5 miles/hr

7) A boat takes 24 hours to cover 128 km downstream and 16 hours to cover 64 km

www.ibpsguide.com | estore.ibpsguide.com | www.sscexamguide.com


83
Complete Quantitative Aptitude Questions

upstream. Then the speed of the boat in still water is:


A) 14/3
B) 8/7
C) 3/2
D) 9/5

8) If a boy rows 8 km downstream in 6 hours and 4 km upstream in 4 hours then how long
will he take to cover 16 km in stationary (still) water?
A) 8
B) 14
C) 22
D) 16

9) A pedal boat goes 12km upstream and 14km downstream in 3hrs. It goes 15km upstream and 10.5km
downstream in 3 hrs 15mints. The speed of the boat in still water is
A) 10
B) 15
C) 20
D) 25

10) A boatman can take the same time to row6.5km downstream and 3.5km upstream. His
speed in still water 2.5 km/hr. The speed of the stream is.
A) 0.75 km/hr
B) 2.5 km/hr
C) 1.5km/hr
D) 7.5km/hr

1) ANSWER: A
Explanation:
Speed downstream = (28 + 8)km/hr
= 36 km/hr
Speed upstream = (28 – 8)km/hr
= 20 km/hr
Let the distance between p and q be ‘x’km
Then, x/36 + (x/2)/20 = 38
x/36 +x/40 = 38
19x = 13680

www.ibpsguide.com | estore.ibpsguide.com | www.sscexamguide.com


84
Complete Quantitative Aptitude Questions

X = 720 km
Therefore the distance between p and q is 720km

2) ANSWER : B
Explanation:
Let the velocity of the stream be ‘ y’ km/hr
Then the speed of the downstream = (12 + y)km/hr
The speed of the upstream = (12 – y)km/hr
9 (12 + y) = 27 (12 – y)
108 + 9y = 324 – 27y
27y + 9y = 324 - 108
36y = 216
y = 6 km/hr

3) ANSWER:B
Explanation:
As Arun swims both the ways at the speed of 30km/hr,
the average speed of swimming is 30 km/hr
Being a boat sailor, varun moves downstream at speed =24 +12=36km/hr
and upstream at speed = 24-12=12km/hr
Therefore, Average speed of boat sailor = Downstream speed x Upstream
speed / speed in still water
= [Downstream Speed x Upstream Speed] /
[(1/2)xdownstream Speed + Upstream Speed])]
= (36*12)/0.5(36+12) km/hr
= 432/24 km/hr
= 18 km/hr

4) ANSWER : B
xplanation:
Let rate of upstream be ‘x’km/hr and downstream be ‘y’ km/hr
Then, 12/x + 18/y = 3 --1
18/x + 12/y =13/4 --2
Adding 1 and 2 we get,
30/x + 30/y = 25/4
1/x + 1/y +=5/24 --3
Subtracting 1 and 2 we get,

www.ibpsguide.com | estore.ibpsguide.com | www.sscexamguide.com


85
Complete Quantitative Aptitude Questions

1/x – 1/y = 1/24 --4


Adding 3 and 4 we get,
2/x = 6/24 ; x= 8 -5
Substitute 5 in 3 we get, y=12
Speed of upstream = 8 km/hr and downstream = 12 km/hr

5) ANSWER : C
Explanation:
Let speed of the boat in still water be ‘x’ km/hr
Speed of the stream be ‘y’ km/hr
50 / (x –y) + 78 / (x +y) = 16 ---------1
70 / (x –y) + 104 / (x +y) =22 ---------2
By equation 1 * 7 and 2 * 5 we get,
350 / (x – y) + 546 / (x + Y) = 112
350 / (x – y) + 520 / (x + Y) = 110
Subtracting the above equation we get
26 / x + y = 2
x + y = 13 -------3
substitute 3 in 1
50 / x – y = 10
x – y = 5 -----------4
solving 3 and 4 we get
2x = 18
x = 9 km/hr
y = 4 km/hr

TYPE 2

6) ANSWER : B
Explanation:
Let the speed in still water be ‘x’ m/hr and
The speed of current be ‘y’ m/hr
Speed upstream = x – y
Speed downstream = x + y
24 / (x – y) – 24/ (x + y) = 12
24 [(x +y) – (x + y)/ x^2 – y^2] =12
2y / (x^2 – y^2) =12 / 24 = 1 / 2

www.ibpsguide.com | estore.ibpsguide.com | www.sscexamguide.com


86
Complete Quantitative Aptitude Questions

x^2 – y^2 = 4y
x^2 = 4y + y^2 ---------1
24 / (2x – y) – 24 / (2x + y) = 2
24[(2x + y) – (2x + y) / 4x^2 – y^2 = 2
2y / (4x^2 – y^2) = 1 / 12
4x^2 – y^2 = 24y
x^2 = (24y + y^2) / 4 ----2
From 1 and 2 we get
4y + y^2 = 24y + Y^2 / 4
16y + 4 y^2 = 24y + y^2
3 y^2 = 8y
y=8/3
Speed of the current is 8 / 3 m/hr.

7) Answer: A
Explanation:
Distance covered in downstream = 128km
Time taken in downstream = 24 hours.
Rate of downstream = distance / time = a = 128 km /24 hours = 16/3km/hr
Distance covered in upstream = 64km
Time taken in upstream = 16 hours.
Rate of upstream = distance / time = b = 64km /16hours = 4 km/hr.
Speed in still water = (a + b) / 2 = (1/2)(16/3+4) km/hr = (1/2)(28/3)km/hr
= 14/3km/hr.

8) Answer: B
Explanation:
Distance covered in downstream = 8 km
Time taken in downstream = 6 hours.
Rate of downstream = 8/6=4/3 km/hr
Distance covered in upstream = 4 km
Time taken in upstream = 4hours.
Rate of upstream = 4/4 = 1 km/hr
Speed in still water = (1/2)(4/3+1) = 7/6km/hr
Time Taken to cover 16 km in still water = 16*6/7=14hrs (approximately)

www.ibpsguide.com | estore.ibpsguide.com | www.sscexamguide.com


87
Complete Quantitative Aptitude Questions

9) ANSWER: A
Explanation :
Let ‘x’ be the speed of the boat in still water
Let ‘y’ be the speed of the current.
Pedal boat will travel downstream at (x + y)km/hr and upstream (x – y)km/hr
Therefore 14/x +y + 12/ x –y = 3
10.5/(x+y) + 15/(x-y) = 3 ¼ = 13/4
70/(x+y) +60 /(x-y) =15 --1
42 /(x+y) + 60/(x-y) = 13 --2
From 1 and we get,
x+y = 14 ; x-y = 6
Therefore x= 10 ; y = 4
The speed of the pedalboat is 10 km/hr

10) ANSWER: A
Explanation:
Given that the speed in still water = 2.5 km/hr
Let the speed of the stream be ‘x; km/hr
speed in upstream = (2.5 – x)km/hr
Then time taken to cover 6.5 km downstream = 6.5 / (2.5+x)
Then time taken to cover 3.5 km upstream = 3.5 / (2.5 – x)
6.5 / (2.5 + x) = 3.5 / (2.5 – x)
6.5 (2.5 – x) = 3.5 (2.5 + x)
16.25 – 6.5x = 8.75 + 3.5x
10 x = 7.5
x = 0.75 km/hr
The speed of the stream is 0.75 km/hr

11) A mototboat can row to a place 112 km away and come back in 44 hours. The time to
row 42 km with the stream is same as the time to row 24 km against the stream. Find the
speed of boat in still water.
A) 5.5 km/hr
B) 7.5km/hr
C) 10.5 km/hr
D) 3.5 km/hr

12) A boy rows 1500m in 1350 seconds against the stream and returns in 15 minutes. His

www.ibpsguide.com | estore.ibpsguide.com | www.sscexamguide.com


88
Complete Quantitative Aptitude Questions

rowing speed in still water is.


A) 5km/hr
B) 4km/hr
C) 7km/hr
D) 9km/hr

TYPE:3
13) A woman rows to a place 24km distant and come back in 7 hrs. She finds that she can
row 2km with the stream in the same time as 1.5km against the stream. The rate of the
stream is,
A) 2
B) 4
C) 1
D) 3

14) A girl can row 15 kmph in still water and he finds that it takes him thrice as long to row up as to row down
the river. Find the rate of stream.
A) 2.5
B) 5.5
C) 7.5
D) 12.5

15) A fisherman rows to a place 24km distance and back in 7 hours. He finds that he can row 2km with the
stream in the same time 1.5 km against the stream.The rate of the stream is?
A) 7
B) 5
C) 3
D) 1

16) A pedal boat can cover 20km in 1 hr in still water. And it takes thrice as much as time to cover up than as
to cover down the same distance in running water. The speed of the current is.
A) 7.5 km/hr
B) 20 km/hr
C) 5 km/hr
D) 10 km/hr

17) A man can row 4 km against the stream in 40mins and return in 36mins. Find the rate of current.

www.ibpsguide.com | estore.ibpsguide.com | www.sscexamguide.com


89
Complete Quantitative Aptitude Questions

A) 1/2 km/hr
B) 1/3 km/hr
C) 2/3 km/hr
D) 1/4 km/hr

18) A boat goes 3.5km upstream in 24 mins and the speed of the stream is 1.5km per hour,
then the speed of the boat in still water is.
A) 4.3 km/hr
B) 7.4 km/hr
C) 5.2km/hr
D) 10.25km/hr

19) A boat man lakes 5hrs 30 mins to row a boat 30km downstream of a river and 4 hrs
15mints to cover a distance of 10km upstream. Find the speed of the river current in km/hr.
A) 1.5 km/hr
B) 3km/hr
C) 5km/hr
D) 7 km/hr

11) ANSWER: A
Explanation:
Downstream speed = 42/x km/hr
Upstream speed = 24/x km/hr
112 / (42 / x) + 112 / (24 / x) = 44
112[x /42 + x / 24] = 44
112 / 3 [x / 14 + x / 8] = 44
11x / 56 = 44 * 3 / 112
x = 44 * 3 * 56 / 11 * 112
x = 6 km/hr
So, downstream speed = 7 km/hr,
upstream speed = 4 km/hr
Speed of boat = 1/2 * (7 + 4) km/hr = 11 /2
speed of boat in still water = 5.5 km/hr

12) ANSWER : A
Explanation:
Rate upstream = (1500 / 1350) m/s = (150/135) m/s

www.ibpsguide.com | estore.ibpsguide.com | www.sscexamguide.com


90
Complete Quantitative Aptitude Questions

= 10/9 m/s
Rate downstream = (1500 / 900)m/s = (15/9)m/s
= 5/3 m/s
Rate in still water = ½ [ a+ b]
= ½[10 / 9 + 5 /3]
= ½[10 +15 / 9]
= ½[25/9]
= 25/18 m/s
= (25/18) * (18/5)km/hr
speed in still water = 5km/hr

13) ANSWER: C
Explanation:
Suppose she move 2km downstream in x hrs
Then, speed of downstream = (2/x) km/hr
Speed of upstream = (1.5/x) km/hr
Therefore 24/(2/x) + 24/(1.5/x) = 7
12x + 16x = 7
X=¼
So speed of downstream = 8km/hr and upstream = 6km/hr
Rate of the stream = ½ (8 - 6 )km/hr
= 1 km/hr

14) Answer: C
Explanation:
Given that, time taken to travel upstream = 3 × time taken to travel downstream
When distance is constant, speed is inversely proportional to the time
Hence, 3 × speed upstream = speed downstream
Let speed upstream =x
Then speed downstream =3x
we have, 1/2(x+3x) = speed in still water
i.e., 2x =15 → x=7.5
i.e., speed upstream =7. 5 km/hr
Rate of stream =1/2(3x−x)= x = 7.5km/hr

15) Answer: D

www.ibpsguide.com | estore.ibpsguide.com | www.sscexamguide.com


91
Complete Quantitative Aptitude Questions

Explanation:
Let be moves 2 km downstream in x Hours
Then in speed downstream = 2/x kmph
Speed in upstream = 1.5/x kmph
==> 24/2/X + 24/1.5/X = 7
12x+16x=7
Therefore x=1/4
Speed in downstream = 8 Kmph
Speed in upstream = 6 Kmph
Then the Rate of stream = 1/2 (8 - 6) = 1 Kmph

16) ANSWER : D
Explanation:
Let the speed of upstream be ‘x’ km/hr
Then speed in downstream = 3x km/hr[thrice as much as time to cover up than as to cover down the same
distance in running water]
Speed in still water = 3x + x/2 km/hr
= 4x /2 = 2x km/hr
Given that boat covers 20km in 1 hr in still water
Therefore 2x = 20
x = 10
So speed in upstream = 10km/hr and
Speed in downstream = 30 km/hr
Hence speed of the current = (30 -10)/2
= 10 km/hr.

17) ANSWER: B
Explanation:
Speed of the man (upstream) = (4 / 40 * 60) km/hr
= 6 km/hr
Speed of the man (downstream) = ( 4 / 36 * 60) km/hr
= (60 / 9)km/hr
= 20/3 km/h
Rate of the current = ½[downstream speed – upstream speed]
= ½ [ 20 / 3 – 6] km/hr
= ½ [ 20 – 18/3] km/hr
= ½ [2/3]km/hr

www.ibpsguide.com | estore.ibpsguide.com | www.sscexamguide.com


92
Complete Quantitative Aptitude Questions

Rate of the current = 1/3 km/hr

18) ANSWER: D
Explanation:
Speed of upstream = 3.5 / 24 km/min
= 3.5 / 24 * 60 km/hr
= 8.75 km/hr
Speed of the stream = ½ (a – b)km/hr = 1.5 km/hr
1.5 = ½ ( a- 8.75)km/hr
a = 11.75 km/hr
speed of the boat in still water = ½(a + b) km/hr
= ½ (11.75 + 8.75)
= ½(20.5)
The speed of the boat in still water = 10.25 km/hr

19) ANSWER: A
Explanation:
Rate downstream = (30 / 5 ½ ) km/hr
= 30 / (11/2) km/hr
= 30 * 2 / 11 km/hr
= 60/11 km/hr
Rate upstream = (10 / 4 ¼)km/hr
= 10 / (17/4)km/hr
= 10 * 4 / 17 km/hr
= 40 / 17 km/hr
Speed of current = 1 / 2 (a – b) km/hr
=1 / 2 (60 / 11 – 40 / 17)
= 1.5 km/hr (approx.)
TYPE: 4

20) Sakthi rows a boat at 4 km upstream in 1hour and 1 km downstream in 20 minutes.


How long will he take to reach 3.5km in still water?
A) 1
B) 2
C) 3

www.ibpsguide.com | estore.ibpsguide.com | www.sscexamguide.com


93
Complete Quantitative Aptitude Questions

D) 4

21) A motor boat sails 30 km of a river towards upstream in 10hrs. How long will it take to
cover the same distance downstream, if the speed of the current is ½ of the speed of the boat
in still water.
A) 1.8hrs
B) 3.33hrs
C) 5hrs
D) None of these

22) Faucet 'P' can fill the tank completely in12 hrs while faucet'q' can empty it by 24 hrs. By mistake, Anitha
forgot to close the faucet ‘q’, As a result, both the faucet remained open. After 8 hrs, faucet realized the
mistake and immediately closed the faucet 'q'. In how much time now onwards, would the tank be full?
A) 4
B) 8
C) 12
D) 16

23) A man goes 4km upstream of the stream in 2hr and goes 2km downstream of the stream in 20mints. How
long will it take to go 10km in stationary water?
A) 1hr 15mits
B) 2hrs 30mints
C) 5hrs 30mints
D) 3hrs 45mints

24) A boys rows to a certain place and comes back, but by mistake he covers 2/3rd more
distance while coming back. The total time for this journey is 20 hours. The ratio of speed of boat to that of
stream is 2 : 1. If the difference between upstream and downstream speed is 24km/hr, then how much time
will the man take to reach to starting point from his present position?
A) 2hr 13mints
B)1hr 30 mins
C) 2hr 30mins
D)1hr 40 mins

25) Dhanvanth can swim at 15 km/hr in stagnant water. In a river with 3 km/hr current, he
swims to a certain distance and comes back within 100 min. What is the distance between
the two points?

www.ibpsguide.com | estore.ibpsguide.com | www.sscexamguide.com


94
Complete Quantitative Aptitude Questions

A) 11km
B) 13km
C) 19km
D) 12km

TYPE 5

26) X, Y , Z are three towns on a lake which flows uniformly. Y is equidistant from X and Z. A man rows from
X to Y and returns in 20hrs. He can row from X to Z in 8 hr. The ratio of speed of the man in still water to the
speed of the current is.
A) 3:5
B) 5:3
C) 2:3
D) None of these

27) A motorboat can cover 80 km upstream and 120 km downstream together in 26 hours.
Also it can cover 100 km upstream and 144 km downstream together in 32 hours. What is
the speed of the motorboat in still water?
A) 12 km/hr
B) 15km/hr
C) 8.5km/hr
D) 19km/hr

28) There are 3 poles M, N and O in a straight line such that point N is equidistant from points M and O. A
boat can travel from point M to O downstream in 6 hours and from N to M upstream in 4 hours. Find the ratio
of boat in still water to speed of stream.
A) 2:3
B) 7:1
C) 3:2
D) 1:7

29) Mohana can row 80km upstream and 110km downstream in 26 hrs. Also she can 60km upstream and
88km downstream in 20 hrs. Find the speed of the girl in still water and the speed of the current in ratio:
A) 5:6
B) 3:6
C) 7:9
D) 8:3

www.ibpsguide.com | estore.ibpsguide.com | www.sscexamguide.com


95
Complete Quantitative Aptitude Questions

30) A motorboat running upstream takes 4 hrs 24 mins to cover a certain distance, while it takes 2hrs to cover
the same distance running downstream. What is the ratio between the speed of the boat and speed of the
water current respectively?
A) 8:3
B) 5:2
C) 3:7
D) 2:4

20) ANSWER:A
Explanation:
Upstream Speed = 4/1 = 4km/hr
Downstream Speed = 1/20 = 0.05 km/min
= 0.05*60 = 3 km/hr
Hence, speed of boat = 1/2( Upstream Speed + Downstream Speed)
= 1/2 (4+3)km/hr =3.5 km/hr
Thus, the time required to reach the distance of 3.5 km=DistanceCovered/Speed of boat
= 3.5/3.5km/hr =1 km/hr

21) ANSWER : B
Explanation:
Upstream speed = x –y
Downstream speed = x + y
x– y = 30/10 = 3km/hr
Again x = 2y
Therefore x – y = 3
y= 3km/hr ; x = 6km/hr
Therefore x + y = 9 km/hr
Time during downstream = 30/9 = 3.33hrs

22) Answer: B
Explanation:
Faucet P can fill the tank completely in 12 hours
=> In 1 hour, Faucet P can fill 1/12of the tank
Faucet q can empty the tank completely in 24 hours
=> In 1 hour, Faucet q can empty 1/24 of the tank
i.e., In one hour, Tank p and q together can effectively fill (1/12-1/24)=1/24of the tank

www.ibpsguide.com | estore.ibpsguide.com | www.sscexamguide.com


96
Complete Quantitative Aptitude Questions

=> In 8 hours, Tank p and q can effectively fill 1/24×8=1/3 of the tank.
Time taken to fill the remaining (1−1/3)=2/3of the tank =(2/3)/(1/12) = 8 hours

23) ANSWER : B
Explanation:
Rate downstream = (2/20 * 60) km/hr
= 6 km/hr
Rate upstream = 2 km/hr
Speed in still water = ½(6+2)km/hr
= 4 km/hr
Required time = distance / speed
= (10/4) hrs =(5/2)hrs =2 ½ hrs
= 2hrs 30mits.

24) ANSWER : A
Explanation:
let speed of boat and stream be 2x and x respectively
So downstream speed = 2x+x = 3x, and
upstream speed = 2x-x = x
Let total distance between points is d km
So he covered d km downstream, and while coming back
i.e. upstream he covers d + 2/3 *d = 5d/3 km
Total time for this journey is 20 hrs.
So d/3x + (5d/3)/x = 20
6d / 3x = 20
d = 10x
Now also given, that (2x+x) – (2x-x) = 24
2x + x - 2x + x = 24
2x = 24
x = 12
So d = 120km
So to come to original point, he will have to cover 2/3 * 120 = 80km
And with speed 3x = 48 km/hr(downstream)
So time is 80/36 * 60 = 133.33 minutes = 2hr 13mins

25) ANSWER: D
Explanation:

www.ibpsguide.com | estore.ibpsguide.com | www.sscexamguide.com


97
Complete Quantitative Aptitude Questions

Speed in still water (a) = 15 km/hr


Speed in current( b) = 3 km/hr
Upstream speed = a – b
= 15 – 3
= 12 km/hr
Downstream speed = a + b
= 15 + 3
= 18 km/hr
Let the distance between the 2 points be s km.
Total journey time = s /12 + s / 18 = 100 / 60
3s + 2s / 36 = 5 / 3
5s / 36 = 5 / 3
15x = 36 * 5
x = 36 * 5 / 15
= 12 km
Distance between the two points is 12km

26) ANSWER: A
Explanation:
Let the speed of man in still water = x km/hr
Speed of the current = y km/hr
Speed of downstream = (x+ y) km/hr
Speed of upstream = ( x – y) km/hr
Let the lake be flowing from X to Z and
xy = yz a
then xz =2a.
a / (x + y )+ a / (x – y) = 20 -----1
and 2a / x + y = 8 ----2
a/x+y=4 -------3
substitute 3 in 1
4 + a / x – y = 20
a / x – y = 16 ----------4
dividing 3 and 4 we get,
a / (x +y) * (x –y) / a = 4 / 16
x–y/x+y=1/4
4x – 4y = x + y
3x = 5y

www.ibpsguide.com | estore.ibpsguide.com | www.sscexamguide.com


98
Complete Quantitative Aptitude Questions

x/y=3/5 x:y = 3:5

27) ANSWER :C
Explanation:
Upstream speed in both cases is 80 and 100.
Ratio is 80 : 100 = 8:10 = 4 : 5.
So let times in both cases be 4x and 5x
Downstream speed in both cases is 120 and 144 resp.
Ratio is 120 : 144 = 5 : 6.
So let times in both cases be 5y and 6y
So 4x + 5y = 26
an 5x + 6y = 32
Solve both, x = 4, y = 2
So upstream speed is = 80/4x = 80/16 = 5 km/hr
And downstream = 120/5y = 120/10 = 12 km/hr
So speed of boat = 1/2 * (5+12) =17/2km/hr
speed of boat = 8.5 km/hr

28) ANSWER: B
Explanation:
Let speed in still water = x km/hr, of current = y km/hr
Downstream speed = (x+y) km/hr
Upstream speed = (x – y) km/hr
Let MO = 2p km. So MN = NO = p km.
So 2p/(x+y) = 6 --------1
p/(x-y) = 4 --------2
Divide both equations, and solve
(2p / x+y) * (x – y / p) = 6/4
4 (2x – 2y) = 6x + 6y
8x – 8y = 6x +6y
2x = 14y
x/y = 14 / 2 = 7/1
x : y = 7 :1

29) ANSWER : D
Explanation:

www.ibpsguide.com | estore.ibpsguide.com | www.sscexamguide.com


99
Complete Quantitative Aptitude Questions

Let rate upstream = ‘x’ km/hr and


Rate downstream = ‘y’ km/hr
Then 80/x + 110/y = 26 ---1
60/x + 88/y = 20--2
By solving 1 and 2
y= 11 ; x = 5
Rain in still water = ½ (11 + 5) km/hr = 8 km/hr
Rain of current = ½ (11 – 50) km/hr = 3 km/hr
The required answer is 8:3

30) ANSWER : A
Explanation:
Let the man’s rate upstream be ‘x’km/hr and downstream be ‘y’km/hr
Then distance covered upstream by 4hrs 24mints = distance covered by downstream in 2hrs
(x * 4 2/5) = y *2
22x/5 = 2y
Y = 11x/5
Required ratio = (y+x/2) : (y –x/2)
= 16x/10 : 6x/10
= 16x : 6x
= 8:3

6. PROBABILITY
In mathematics too, probability indicates the same – the likelihood of the occurrence of an event.
Examples of events can be :
 Tossing a coin with the head up
 Drawing a red pen from a pack of different coloured pens
 Drawing a card from a deck of 52 cards etc.
An event that occurs for sure is called a Certain event and its probability is 1.
An event that doesn’t occur at all is called an impossible event and its probability is 0.
This means that all other possibilities of an event occurrence lie between 0 and 1.
0 ≤P(A) ≤ 1
where A is an event and P(A) is the probability of the occurrence of the event.
This also means that a probability value can never be negative.
Every event will have a set of possible outcomes. It is called the ‘sample space’.
Consider the example of tossing a coin.

www.ibpsguide.com | estore.ibpsguide.com | www.sscexamguide.com


100
Complete Quantitative Aptitude Questions

When a coin is tossed, the possible outcomes are Head and Tail. So, the sample space is represented as {H,
T}.
Similarly when two coins are tossed, the sample space is {(H,H), (H,T), (T,H), (T,T)}.
The probability of head each time you toss the coin is 1/2. So is the probability of tail.
Basic formula of probability
the Probability of the occurrence of an event A is defined as:
P(A) = (No. of ways A can occur)/(Total no. of possible outcomes)
Compound probability:
Compound probability is when the problem statement asks for the likelihood of the occurrence of more than
one outcome.
Formula for compound probability
 P(A or B) = P(A) + P(B) – P(A and B)
where A and B are any two events.
P(A or B) is the probability of the occurrence of atleast one of the events.
P(A and B) is the probability of the occurrence of both A and B at the same time
Mutually exclusive events:
Mutually exclusive events are those where the occurrence of one indicates the non-occurrence of the other
OR
When two events cannot occur at the same time, they are considered mutually exclusive.
Note: For a mutually exclusive event, P(A and B) = 0.
Independent and Dependent Events
Independent Event
When multiple events occur, if the outcome of one event DOES NOT affect the outcome of the other events,
they are called independent events.
Say, a die is rolled twice. The outcome of the first roll doesn’t affect the second outcome. These two are
independent events.
Dependent Events
When two events occur, if the outcome of one event affects the outcome of the other, they are called
dependent events.
Conditional probability
Conditional probability is calculating the probability of an event given that another event has already occured .
The formula for conditional probability P(A|B), read as P(A given B) is
P(A|B) = P (A and B) / P(B)
Complement of an event
A complement of an event A can be stated as that which does NOT contain the occurrence of A.
A complement of an event is denoted as P(Ac) or P(A’).
P(Ac) = 1 – P(A)

www.ibpsguide.com | estore.ibpsguide.com | www.sscexamguide.com


101
Complete Quantitative Aptitude Questions

or it can be stated, P(A)+P(Ac) = 1


For example,
if A is the event of getting a head in coin toss, Ac is not getting a head i.e., getting a tail.
if A is the event of getting an even number in a die roll, Ac is the event of NOT getting an even number i.e.,
getting an odd number.
if A is the event of randomly choosing a number in the range of -3 to 3, Ac is the event of choosing every
number that is NOT negative i.e., 0,1,2 & 3 (0 is neither positive or negative).
Problems:
1) When a single die is rolled, the sample space is {1,2,3,4,5,6}.What is the probability of getting a 3 when a
die is rolled?
A) 1/2
B) 1/6
C) 6/1
D) 3/6

2) When two dice are thrown, find the probability of getting a greater number on the first die than the one on
the second, given that the sum should equal 9.
A) 1/2
B) 1/5
C) 2/5
D) 4/2

3) A carton contains 12 green and 8 blue bulbs .2 bulbs are drawn at random. Find the probability that they
are of same colour.
A) 91/47
B) 47/105
C) 47/95
D) 95/47

4) Tickets numbered 1 to 37 are mixed up and then a ticket is drawn at random. What is the probability that
the ticket drawn has a number which is a multiple of 4 or 10?
A) 11/37
B) 37/11
C) 12/37
D) 37/12

www.ibpsguide.com | estore.ibpsguide.com | www.sscexamguide.com


102
Complete Quantitative Aptitude Questions

5) In a Coupon, there are 30prizes and 75blanks. A Coupon is drawn at random. What is the probability of
getting a prize?
A) 2/7
B) 5/7
C) 1/5
D) 1/2

6) Two dice are thrown together. The probability that the total score is a composite number is:
A) 5/12
b) 12/7
c) 7/12
d) 12/5

7) A Receptacle contains 3violet, 4purple and 5 black balls. Three balls are drawn at random from the
receptacle. The probability that all of them are purple, is:
A)3/55
B)7/55
C)1/55
D)9/55

8) Two dice are rolling simultaneously .What is the probability that the sum of the number on the two faces is
divided by 5 Or 7.
A) 13/36
B) 14/36
C) 11/36
D) 9/36

9) In a batch, there are 22 boys and 18 girls. Three students are selected at random. The probability that 1 girl
and 2 boys are selected, is:
a) 3754/8854
b) 4158/9880
c) 8514/9880
d) 2078/4920

10) What is the probability of getting a 4 or a 6 when a die is thrown together?


a) 2/3
b) 1/3

www.ibpsguide.com | estore.ibpsguide.com | www.sscexamguide.com


103
Complete Quantitative Aptitude Questions

c) 3/6
d) 4/6

1) Answer: B)
No. of ways it can occur = 1
Total no. of possible outcomes = 6
So the probability of rolling a particular number (3) when a die is rolled = 1/6.

2) Answer: A)
Let the event of getting a greater number on the first die be G.
There are 4 ways to get a sum of 9 when two dice are rolled = {(3,6),(4,5),(5,4), (6,3)}.
And there are two ways where the number on the first die is greater than the one on the second given that the
sum should equal 9, G = {(5,4), (6,3)}.
Therefore, P(Sum equals 9) = 4/36 and P(G sum equals 9) = 2/36.
Now, P(G) = P(G sum equals 9)/P(sum equals 9)
= (2/36)/(4/36)
= 2/4 =>1/2

3) Answer: C)
Let S be the sample space
Then n(S) = no of ways of drawing 2 bulbs out of (12+8) = 20c2=20*19/2*1=190
Let E = event of getting both bulbs of same colour
Then, n(E) = no of ways (2 bulbs out of 12) or (2 bulbs out of 8)
=12C2+ 8C2=(132/2)+(56/2) = 66+28 = 94
Therefore, P(E) = n(E)/n(S) = 94/190 = 47/95

4) Answer: A)
Here, S = {1, 2, 3, 4, ...., 36,37}.
Let E = event of getting a multiple of 4 or 10= {4,8,12,16,20,24,28,32,36,10, 30}.
P(E) = n(E)/n(S) = 11/37

5) Answer: A)
Total number of outcomes possible, n(S) = 30+75 = 105
Total number of prizes, n(E) = 30
P(E)=n(E)/n(S)=30/105=2/7

6) Answer: C)

www.ibpsguide.com | estore.ibpsguide.com | www.sscexamguide.com


104
Complete Quantitative Aptitude Questions

Clearly, n(S) = (6 x 6) = 36.


Let E = Event that the sum is a composite number
Then E= { (1, 3), (1, 5), (2, 2), (2, 4), (2, 6), (3, 1), (3, 5), (3, 3), (3, 6), (4, 2), (4,4),(4, 5), (4, 6), (5, 1),
(5,3),(5,4),(5,5),(6,2),(6,3),(6,4),(6,6) }
n(E) = 21
P(E) = n(E)/n(S) = 21/36 = 7/12.

7) Answer: C)
Let S be the sample space.
Then, n(S) = number of ways of drawing 3 balls out of 12 = 12C3 = 220
Let E = event of getting all the 3 purple balls.
n(E) = 4C3= 4
P(E) = n(E)/n(S) = 4/220 = 1/55

8) Answer: C)
Clearly, n(S) = 6 x 6 = 36
Let E be the event that the sum of the numbers on the two faces is divided by 5or 7.
Then,E = {(1,4),(1,6),(2,3),(2,5),(3,2),(3,4),(4,1),(4,3),(4,6),(5,2),(5,5),(6,1),(6,4)}
n(E) = 11.
Hence, P(E) = n(E)/n(S) = 11/36

9) Answer: B)
Let , S - sample space E - event of selecting 1 girl and 2 boys.
Then, n(S) = Number ways of selecting 3 students out of 40
= 40C3
= 9880
n(E) = 18C1 *22C2
= 18*231
= 4158
P(E) = n(E)/n(s) = 4158/9880

10) Answer: B)
Taking the individual probabilities of each number, getting a 4 is 1/6 and so is getting a 6.
Applying the formula of compound probability,
Probability of getting a 4 or a 6,
P(4 or 6) = P(4) + P(6) – P(4 and 6)
==> 1/6 + 1/6 – 0

www.ibpsguide.com | estore.ibpsguide.com | www.sscexamguide.com


105
Complete Quantitative Aptitude Questions

==> 2/6 = 1/3

11) Consider the example of finding the probability of selecting a red card or a 9 from a deck of 52 cards.
A) 15/26
B) 26/15
C) 7/13
D) 13/7

12) What is the probability of the occurrence of a number that is even or less than 3 when a fair die is rolled.
a) 2/3
b)3/2
c)5/6
d)6/5

13) Two cards are drawn at random from a pack of 52 cards.what is the probability that either both are Red or
both are king?
A) 52/221
B) 55/190
C) 55/221
D) 19/221

14) In a batch, 40% of the students offered Maths, 30% offered science and 15% offered both. If a student is
selected at random, what is the probability that they has offered science or maths?
A) 0.55
B) 0.65
C) 0.45
D) 0.75

15) In a hostel, 40% of the students play cricket, 20% play chess and 10% both. If a student is selected at
random, then the probability that he plays cricket or chess is:
a) 1/2
b) 3/5
c) 1/4
d) 4/7

16) one rupee coin is tossed twice. What is the probability of getting two consecutive heads ?
A)1/2

www.ibpsguide.com | estore.ibpsguide.com | www.sscexamguide.com


106
Complete Quantitative Aptitude Questions

B)1/4
C)3/4
D)4/3

17) Consider a pack contains 2black, 9 white and 3 pink pencils. If a pencil is drawn at random from the pack,
replaced and the process repeated 2 more times, What is the probability of drawing 2 black pencils and 1 pink
pencil?
a)3/ 49
b)3/686
c)3/14
d)3/545

18) A box contains 5 cone and 4 chocobar ice-creams. Preethi eats 3 of them, by randomly choosing. What
is the probability of choosing 1 chocobar and 2 cone
ice-creams?
a) 63/10
b) 20/63
c) 10/63
d) 63/20

19) A box contains 3red, 8 blue and 5 green marker pens. If 2 marker pens are drawn at random from the
pack, not replaced and then another pen is drawn. What is the probability of drawing 2 blue marker pens and
1 red marker pen?
a) 3/20
b) 1/20
c) 7/20
d) 9/20

20) What is the probability of drawing a jack and a queen consecutively from a deck of 52 cards, without
replacement?
a) 4/664
b) 8/52
c) 4/663
d) 4/52

11) Answer: C)
We need to find out P(R or 6)

www.ibpsguide.com | estore.ibpsguide.com | www.sscexamguide.com


107
Complete Quantitative Aptitude Questions

Probability of selecting a Red card = 26/52


Probability of selecting a 9 = 4/52
Probability of selecting both a red card and a 9 = 2/52
P(R or 9) = P(R) + P(9) – P(R and 9)
= 26/52 + 4/52 – 2/52
= 28/52
= 7/13.

12) Answer: A)
Let the event of the occurrence of a number that is even be ‘A’ and the event of the occurrence of a number
that is less than 3 be ‘B’. We need to find P(A or B).
P(A) = 3/6 (even numbers = 2,4,6)
P(B) = 2/6 (numbers less than 3 = 1,2)
P(A and B) = 1/6 (numbers that are both even and less than 3= 2)
Now, P(A or B) = P(A) + P(B) – P(A or B)
= 3/6 + 2/6 – 1/6
P(A or B) = 4/6=2/3

13) Answer: C)
We have n(s) = 52C2 = 1326.
Let A = event of getting both red cards
B = event of getting both king
A∩B = event of getting king of red cards
n(A) = 26C2 = 325, n(B)= 4C2= 6 and n(A∩B) = 2C2 = 1
P(A) = n(A)/n(S) = 325/1326;
P(B) = n(B)/n(S) = 6/1326 and
P(A∩B) = n(A∩B)/n(S) = 1/1326
P(A∪B) = P(A) + P(B) - P(A∩B)
= (325+6-1) / 1326 = 330/1326 = 55/221

14) Answer: A)
P(M) = 0.40
P(S) =0.30 and
P(M∩S) = 0.15
P(M∪S) = P(M) + P(S) - P(M∩S) = 0.55

15) Answer: A)

www.ibpsguide.com | estore.ibpsguide.com | www.sscexamguide.com


108
Complete Quantitative Aptitude Questions

Given that, 40% play cricket; that is, P(C) = 40/100=4/10


20% play chess; that is, P(c) = 20/100 =2/10
And, 10% play both cricket and chess; that is, P(C And c) = 10/100 = 1/10
Now, we have to find the probability that 1 student plays cricket or chess; that we have to find, P(C or c)
We know that, P(C Or c) = P(C) + P(c) - P(C And c)
= 4/20+2/10-1/10 =5/10=1/2
Hence, the required probability 1/2

16) Answer: B)
Probability of getting a head in one toss = 1/2
The coin is tossed twice. So 1/2 * 1/2 = 1/4 is the answer.
Here’s the verification of the above answer with the help of sample space.
When a coin is tossed twice, the sample space is {(H,H), (H,T), (T,H), (T,T)}.
Our desired event is (H,H) whose occurrence is only once out of four possible outcomes and hence, our
answer is 1/4.

17) Answer: B)
Here, total number of pencils = 14
Probability of drawing 1 black pencil = 2/14
Probability of drawing another black pencil = 2/14
Probability of drawing 1 pink pencil = 3/14
Probability of drawing 2 black pencils and 1 pink pencil = 2/14 * 2/14 * 3/14 = 3/686

18) Answer: C)
Probability of choosing 1 cone= 5/9
After taking out 1 cone, the total number is 8 .
Probability of choosing 2nd cone = 4/8
Probability of choosing 1chocobaricecream out of a total of 7 = 4/7
So the final probability of choosing 2 cone and 1chocobar ice cream = 5/9*1/2*4/7 =10/63

19) Answer: B)
Probability of drawing 1 blue marker pen =8/16
Probability of drawing another blue marker pen = 7/15
Probability of drawing 1 red marker pen = 3/14
Probability of drawing 2 blue marker pens and 1 red marker pen = 8/16*7/15*3/14=1/20

20) Answer: C)

www.ibpsguide.com | estore.ibpsguide.com | www.sscexamguide.com


109
Complete Quantitative Aptitude Questions

Probability of drawing a jack = 4/52 = 1/13


After drawing one card, the number of cards are 51.
Probability of drawing a queen = 4/51.
Now, the probability of drawing a jack and queen consecutively is 1/13 * 4/51 = 4/663

21) There are 2 vessels. 1st vessel contains 5white and 5 blue thread roll. 2nd vessel contains 4 white and 6
black thread roll. One roll is taken at random from first vessel and put to second vessel without noticing its
color. Now a roll is chosen at random from 2nd vessel. What is the probability of the second roll being a white
colored roll?
A) 11/13
B) 9/11
C) 13/11
D) 5/12

22) In MSM college, 35% of the students study Tamil and English. 40% of the students study English. What is
the probability of a student studying Tamil given he/she is already studying english?
A) 0.675
B) 0.580
C) 0.875
D) 0.725

23) A single coin is tossed 7 times. What is the probability of getting at least one tail?
a) 127/128
b) 128/127
c) 2/128
d) 4/128

24) A question (sum) is given to three boys whose chances of solving it are 1/3,1/4 and 1/5 respectively.
What is the probability that the question will be solved?
A) 4/5
B) 3/5
C) 3/4
D) 7/5

25) Murali and his wife appear in an interview for two vacancies in the same post. The probability of murali's
selection is (1/6) and the probability of wife's selection is (1/4). What is the probability that only one of them is
selected ?

www.ibpsguide.com | estore.ibpsguide.com | www.sscexamguide.com


110
Complete Quantitative Aptitude Questions

A) 8/25
B) 1/7
C) 3/4
D) 1/3

26) M speaks truth in 45% of cases and N in 65% of cases. In what percentage of cases are they likely to
contradict each other, narrating the same incident?
A) 57.5
B) 55.5
C) 53.8
D) 51.5

27) A Package contains 12 pack of variety1 drink, 6 pack of variety2 drink and 8pack of variety3 drink. Three
packsof them are drawn at random, what is the probability that the three are not of the same variety?
a) 37/325
b) 288/325
c) 188/325
d) None of these

28) There are two groups, X and Y wrote an examination. The probability of X's pass is 3/5 and the probability
of Y's pass is 5/7. What is the probability that only one of them is passed out?
a) 15/16
b) 16/35
c) 12/43
d) 18/35

29) Pradeesh gets a chance of 40% to win 1st round of a game and a Priya gets a chance of 55% to win 2nd
round of the game. In what % of cases are they likely to contradict each other, narrating the same incident?
a) 49%
b) 54%
c) 51%
d) 38%

30) A cartoon contains 15 torch lights out of which 3 are defective. Two torch light are chosen at random from
this cartoon. The probability that at least one of these is defective is.
A) 13/35
B) 14/35

www.ibpsguide.com | estore.ibpsguide.com | www.sscexamguide.com


111
Complete Quantitative Aptitude Questions

C) 11/35
D) 17/35

21) Answer: B)
Case 1: first was a white roll
Now it is put in second vessel, so total white rolls in second vessel = 4+1 = 5, and total rolls in second vessel
= 10+1 = 11
So probability of white roll from second vessel = 5/11
Case 2: first was a blue roll
Now it is put in second vessel, so total white rolls in second vessel remain 4, and total rolls in second vessel
= 10+1 = 11
So probability of white rolls from second vessel = 4/11
So required probability = 5/11+4/11 = 9/11 (added the cases because we want one of these cases to happen
and not both)

22) Answer: C)
P(T and E) = 0.35
P(E) = 0.40
P(T/E) = P(T and E)/P(E) = 0.35/0.40 = 0.875

23) Answer: A)
Consider solving this using complement.
Probability of getting no tail = P(all heads) = 1/128
P(at least one tail) = 1 – P(all heads) = 1 – 1/128 = 127/128

24) Answer: B)
Let A, B, C be the respective events of solving the problem and Ac ,Bc,Cc be the respective events of not
solving the problem. Then A, B, C are independent events
Ac ,Bc,Cc are independent events
Now, P(A) = 1/3 , P(B) = 1/4 and P(C)=1/5
P(Ac)=2/3, P(Bc)=3/4 and P(Cc)=4/5
P( none solves the problem) = P(not A) and (not B) and (not C)
= P(AcnBcnCc)
= P(Ac) P(Bc) P(Cc)
= 2/3*3/4*4/5 =2/5
Hence, P(the problem will be solved) = 1 - P(none solves the problem)
= 1-2/5=3/5

www.ibpsguide.com | estore.ibpsguide.com | www.sscexamguide.com


112
Complete Quantitative Aptitude Questions

25) Answer: D)
A= Event that the husband is selected
B =Event that the wife is selected
P(A)=1/6,P(B)=1/4
P(Ac)=1-1/6=5/6
P(Bc)=1-1/4=3/4
Required Probability=P[ (A and notB)or(B and not A)]
= P(A). P(Bc) + P(B) P(Ac)
=1/6*3/4 + 1/4 * 5/6 = 1/3

26) Answer: D)
Let M = Event that M speaks the truth
N = Event that N speaks the truth

Then P(M) = 45/100 = 9/20


P(N) = 65/100 = 13/20
P(M-lie) = 1-9/20 = 11/20
P(N-lie) = 1-13/20 = 7/20
Now, M and N contradict each other =[M lies and N true] or [M true and N lies]
= P(M).P(N-lie) + P(M-lie).P(N)
= 9/20*7/20 + 11/20*13/20 =206/400
= 206/400*100= 51.5%

27) Answer: B)
Total number of drink pack= 12+6+8= 26.
Let S be the sample space.
Then, n(S) = number of ways of taking 3 drink pack out of 26.
Therefore, n(S) = 26C3 = 2600
Let Ebe the event of taking 3 pack of the same variety.
Then, E = event of taking (3 pack out of 12) or (3 packout of 6) or (3 packout of 8)
n(E) = 12C3 + 6C3 + 8C3
= 220+20+56
= 296
The probability of taking 3 packof the same variety = n(E)/n(S) = 296/2600=37/325
Then, the probability of taking 3 pack are not of the same variety = 1 – 37/325= 288/325

www.ibpsguide.com | estore.ibpsguide.com | www.sscexamguide.com


113
Complete Quantitative Aptitude Questions

28) Answer: B)
Let X be the event of the group X pass
Let Y be the event of the group Y pass
Then, X'= Event of the group X's fail and Y'= event of the group Y's fail.
Therefore, p(X) = 3/5 and p(Y) = 5/7,
P(X') = 1 - P(X) = 1- 3/5 = 2/5 and P(Y') = 1- P(Y) = 1- 5/7= 2/7
Required probability = P[( X And Y') Or (Y And X')]
= P[( X And Y') Or (Y And X')]
= P[( X And Y') + (Y And X')]
= P[( X And Y')] + p[(Y And X')]
= p(X) * p(Y') + P(Y) *P(X’)
= (6/35 + 10/35) = 16/35

29) Answer: C)
Let A be the event that a pradeesh wins 1st round
Let B be the event that a priya wins 2nd round.
Then, A' = Event that the pradeesh losses 1st round
and B' = event that the priya losses 2nd round.
Therefore, P(A) = 40/100 = 8/20, P(B) = 55/100 = 11/20
P(A') = 1- (8/20) = 12/20 and P(B') = 1- (11/20) = 9/20
First, we have to find the probability that they contradict each other;
That is, P( A And B contradicts each other)
= P[(pradeesh win in 1st round And priya losses in 2nd round) (Or)
(pradeesh losses in 1st round And priya wins in 2nd round)
= P[(A And B') Or (A' And B)] = P[(A And B')] + p[(A' And B)]
= P(A) x P(B') + P(A') x P(B)
=72/400+132/400
= 204/400
We have to find the %.
Required % = (204/400)x100 = 51%.

30) Answer: A)
P (none is defective) = 12c2/15c2
= (12*11/2*1)/(15*14/2*1)
= 66/105=22/35
P (at least one is defective) = (1 – 22/35)
=13/35

www.ibpsguide.com | estore.ibpsguide.com | www.sscexamguide.com


114
Complete Quantitative Aptitude Questions

7. PROBLEMS ON TRAIN
TYPE :1
1) A train 170 m long is running at a speed of 22.5 km/hr. what time will it take to cross a 80m long bridge?
A) 20 sec
B) 40 sec
C) 15 sec
D) 30sec

2) A train 560 m long crosses the platform of length 340 m in 45seconds. Find the speed of the train in km/hr
A) 72km/hr
B) 36km/hr
C) 48km/hr
D) 15km/hr

3) Length of train is 510 meters and speed of train is 63 km/hour. This train can pass a pole in 90 seconds,
then find the length of the pole?
A) 575mts
B) 2500mts
C) 1065mts
D) 876mts

4) A train crosses a platform and a scooter standing on the platform in 20 seconds, 12seconds
respectively.What is the length of the platform if the speed of the train is 25 km/hr?
A) 55.8 m
B) 75m
C) 124m
D) 68.5m

5) A 240m train crosses a standing object in 6 seconds. Find the time taken by the train to cross a long pole
of length 162.5 m.
A) 25
B) 45
C) 10
D) 15

www.ibpsguide.com | estore.ibpsguide.com | www.sscexamguide.com


115
Complete Quantitative Aptitude Questions

6) A train crosses a long bridge of length 12 km and a standing girl in 600 seconds and 24 seconds
respectively. What will be the length of the train?
A) 275
B) 480
C) 389
D)225

7) A train, 1800 meters long running at the rate of 204 km/hr will cross a platform in:
A) 16
B) 32
C) 45
D) 64

8) If a Rajdhani train running at the speed of 25 m/s and covers 0.8 km long chunk in 6 minutes, then the
length of the train is:
A) 8.2
B) 6.7
C) 7.5
D) 9.3

9) A shatabadi express takes 240 seconds to cover a distance of length 700 meters at a speed of 270 km/hr.
What will be the time taken by the train to cross a standing pole?
A) 160sec
B) 465sec
C) 310sec
D) 231sec

10) If a train crosses a tree in 24 seconds while travelling at a speed of 90km/hr, then in how much time will
the train cross a telegraph post of length 1200m at same speed?
A) 26 sec
B) 48 sec
C) 72 sec
D) 38 sec

1) ANSWER:B
Explanation:
Length of the train = 170 m

www.ibpsguide.com | estore.ibpsguide.com | www.sscexamguide.com


116
Complete Quantitative Aptitude Questions

Length of the bridge = 80 m


Therefore, length of the train + length of the bridge = (170 + 80) m = 250m
Speed of the train = 22.5 km/hr
Speed of the train = 22.5 × 5/18 m/sec
= 112.5/18m/sec
= 6.25m/sec
Therefore, time taken by the train to cross the bridge = 250 m/6.25 m/sec. = 40 seconds.

2) ANSWER:A
Explanation:
Length of the train = 560 m
Length of the platform = 340 m
Therefore, length of the train + length of the platform = 560 m + 340 m = 900 m
Time taken by the train to cross the platform = 45 sec
Therefore, speed of train = 900 m / 45 m/sec = 20 m/sec
To convert the speed from m/sec to km/hr, multiply by 18/5
Therefore, speed of the train = 20 × 18/5 km/hr = 72 km/hr

3) ANSWER: C
Explanation:
Given speed of the train = 63 km/hr
= 63*5/18 m/s =17.5m/s
Let the length of the pole = x mts
Given time taken to cover the distance of (510+ x)mts is 90 sec.
We know speed = distance/time m/s
17. 5 = 510+x/90
1575 = 510+x
x = 1065
Therefore the length of the pole is 1065 mts

4) Answer: A
Explanation:
Given that,
The speed of the train = 25km/hr
= 25x 5/18 m/sec
= 125/18 m/sec
The train crosses a scooter in 12 seconds. Then,

www.ibpsguide.com | estore.ibpsguide.com | www.sscexamguide.com


117
Complete Quantitative Aptitude Questions

Length of the train = 125/18*12= 83.33


Now, let the length of the platform is X m.
the train crosses the platform in 20 seconds.
Then, (X +83.3)/20 = 125/18
18(X +83.3)= 2500
18X=1000.6
X=55.58 m
Hence the platform is 55.58 m long.

5) ANSWER:C
Explanation:
Length of the train = 240 m
Time taken to cross an object = 6 seconds
Speed of the train = Length of the train / Time taken to cross an object
= 240/6 m/s
= 40 m/s
Length of the pole = 162.5 m
Time taken to cross the pole= (Length of the train + Length of the pole) / Speed of the train
= (240 + 162.5)/40
= 402.5/40
= 10.06 seconds
Hence the answer is 10 seconds.

6) ANSWER:B
Explanation:
Time taken to cross the bridge(12 km) = 600 seconds.
Speed of the train = Distance / Time
= 12/600 km/s
= 12000/600 m/s
= 20m/s
Train crosses a girl in 24 seconds.
Length of the train = Speed of the train x Time Taken
= (20 *24)m
= 480m
Hence the required length is 480 m.

7) ANSWER: B

www.ibpsguide.com | estore.ibpsguide.com | www.sscexamguide.com


118
Complete Quantitative Aptitude Questions

Explanation:
Speed of the train = 204 km/hr.
We have to find the time in seconds, so convert the speed in the unit of m/sec.
Therefore 204 km/hr = 204 x 5/18 m/s
= 1020/18 m/s
Length of the train = 1800 meters.
we have to find the time taken by the train to cover 1800 meters at 1020/18 m/sec
Time = distance/speed
= 1800/(1020/18)sec
= (1800*18/1020)sec
= 32 sec (approx.)

8) ANSWER:A
Explanation:
Speed of the train = 25 m/s.
Length of the chunk = 0.8 km
= 800 meters
Time taken to cross the chunk = 6 minutes
= (6*60) seconds
= 360 seconds
Let the length of the train be y meters.
Time taken by the train to cover (800 + y) meters at 25 is 360 seconds
i.e., distance = speed x time
(800+y) = 25 x 360
800+y = 9000
y = 8200 m
i.e., y = 8.2 km
Hence the length of the train is 8.2 km

9) ANSWER:D
Explanation:
Speed of the train = 270 km/hr.
Converting the speed into m/sec
Speed = 270 km/hr = 270 x 5/18 m/sec = 75 m/sec
First, we have to find the length of the train
Let the length of the train be X km.
Given that, the time taken to cover 700 meters = 240 seconds.

www.ibpsguide.com | estore.ibpsguide.com | www.sscexamguide.com


119
Complete Quantitative Aptitude Questions

We can have, distance/speed = time


x+700/75 = 240
x+700 = 18000
x = 17300 m
The time taken by the train of length 17300 meters to cross a standing Pole at 75 m/s =( 17300/75)seconds
= 231 seconds.
Hence the answer is 231 sec(approx.)

10) ANSWER:C
Explanation:
Speed of the train = 90 km/hr
Converting the speed into m/sec
90 km/hr = (90* 5/18)m/s
= 25m/s
length of the train = distance travelled
we know distance = speed * time= (25*24)m
= 600 m
Distance travelled = Train length + telegraph post length
= (600 +1200)m
= 1800 m
Time = distance / speed
= ( 1800/25) sec
= 72 seconds
Hence the required answer is 72 seconds
TYPE 2

11) A passenger train 300m long is running with a speed of 136 km/hr. In what time will it pass a boy who is
running at 6 km/hr in opposite direction in which the train is moving?
A) 8.6sec
B) 7.6sec
C) 3.6sec
D) 2.6sec

12) A person runs opposite to that of chandigarh train at a speed of 10km/hr. If the relative speed between
train and the boy running in opposite direction is 25km/hr. What is the length of the train, if it takes 10
seconds to cross a boy, when is at rest?
A) 36.6 m

www.ibpsguide.com | estore.ibpsguide.com | www.sscexamguide.com


120
Complete Quantitative Aptitude Questions

B) 40.6 m
C) 43.6m
D) 41.6m

13) A mail train 220 m long is running with a sped of 120 km/hr. What is the time in which it will pass a man
who starts from the engine running at the speed of 12km/hr in the direction opposite to that of the train.
A) 6sec
B) 5sec
C) 4sec
D) 3sec

14) A boy sitting in a train which is travelling at 25 km/hr observes that a goods train travelling in opposite
direction, takes 4.5 seconds to pass him. If the train is 140 m long, find its speed.
A) 77km/hr
B) 79km/hr
C) 87km/hr
D) 89km/hr

15) A train 135m long is moving at a speed of 12.5 km/h. It will cross a girl coming from the opposite direction
at a speed of 1km/hr in.
A) 36sec
B) 46sec
C) 56sec
D) 66sec

16) The rajdhani express of 400m runs at a speed of 124 km/hr and a person runs on the platform at a speed
of 40km/hr in the direction opposite to that of the train. Find the time taken by the train to cross the running
person?
A) 7 sec
B) 8.78sec
C) 7.78sec
D) 8 sec

17) A passenger train 165 m long which is running at a speed of 30km/hr. In what time will it pass a man who
is running at a speed of 3 km/hr in the opposite direction. In which the train is moving.
A)14 sec
B) 17sec

www.ibpsguide.com | estore.ibpsguide.com | www.sscexamguide.com


121
Complete Quantitative Aptitude Questions

C)16sec
D)18sec

18) A metro train 200 metres long takes 12 seconds to cross a man walking at 10 km/hr in a opposite to that
of the train. Find the speed of the train?
A) 50 km/hr
B) 60km/hr
C) 130km/hr
D) 140km/hr

19) A goods train 110m long is running with the speed of 30 km/hr. in what time will it pass a boy who is
running at 3.5 km/hr in the direction opposite to that in which the train is going.
A) 9 sec
B) 10sec
C) 12sec
D) 13sec

20) A train 330 m long passes a man, running at 18 km/hr in the direction opposite to that of the train in 18
seconds. The speed of the train is.
A) 38 km/hr
B) 29km/hr
C) 48km/hr
D) 44km/hr

11) ANSWER: B
Explanation:
Speed of the train = 136 km/hr
Speed of boy = 6 km/hr
Speed of the train relative to boy = (13 + 6)km/hr
= 142 km/hr
Convert km/hr into m/s 142 km /hr = (142 * 5 / 18)
= 710 / 18 m/s
= 39.44 m/s
Time taken by the train to cross a boy = time taken bt it to cover 300 m at 39.44 m/s
=300 / 39.44 [speed time= distance / speed]
= 7.6 sec

www.ibpsguide.com | estore.ibpsguide.com | www.sscexamguide.com


122
Complete Quantitative Aptitude Questions

12) ANSWER : D
Explanation:
Speed of person = 10km/hr
As the train and the running person are moving in opposite direction,therir speed values are added to find
the relative speed.
Relative speed = speed of train + speed of person
25 = speed of train
Speed of train = 25 – 10
= 15 km/hr
Convert km/hr into m/s
15 km/hr = (15 * 5 / 18)m/s
= 25 / 6 m/s
Distance = speed * time
= 25 / 6 * 10
= 250/6 m/s
= 41.6 m

13) ANSWER: A
Explanation:
We know as the train and the running person are moving in opposite direction,therir speed values are added
to find the relative speed.
Relative speed = speed of mail train + speed of running engine
= (120 +12)km/hr
= 132 km/hr
Convert km/hr into m/s
132 km/hr = (132 * 5 /18) m/s
= (660 / 18)m/s
= 36.66 m/s
Time taken by train to pass a man = distance / speed
[ from the length of the mail train is the distance]
= 220 / 36.66
= 6 sec.

14) ANSWER : C
Explanation:
Relative speed = distance / time
= (140 / 4.5) m/s

www.ibpsguide.com | estore.ibpsguide.com | www.sscexamguide.com


123
Complete Quantitative Aptitude Questions

= 31.11 m/s
Convert m/s into km/hr
31.11 m/s = (31.11 * 18 / 50 km/hr
= (560 / 5) km/hr
= 112 km/hr
Relative speed = speed of train + speed of train (by sitting)
speed of train = Relative speed - speed of train (by sitting)
= (112 – 25) km/hr
= 87 km/hr
Therefore the speed of goods train is 87 km/hr.

15) ANSWER : A
Explanation:
Speed of the train = 12.5 km/hr
Speed of girl = 1 km/hr
Relative speed = speed of train + speed of girl
= (12.5 + 1) km/hr
= 13.5 km/hr
Convert km/hr into m/s
13.5 km/hr = ( 13.5 * 5 /18)m/s
= 67.5 / 18 m/s
= 3.75 m/s
Time = distance / speed
Time taken by the train to pass the man = 135 / 3.75
= 36 sec.

16) ANSWER : B
Explanation:
Length of the train = 400m
Speed of train = 124 km/hr
Speed of person = 40 km/hr
Relative speed (speed of trai relative to man) = (124 + 40) km/hr
= 164 km/hr
[As the train and the running person are moving in opposite direction,therir speed values are added to find
the relative speed.]
Convert km/hr into m/s
164 km/hr = 164 * 5 / 18

www.ibpsguide.com | estore.ibpsguide.com | www.sscexamguide.com


124
Complete Quantitative Aptitude Questions

= 820/18
= 45.55 m/s
We know speed = distance / time
Therefore time taken by the train to cross the running person = time taken by the train to cover 400m at a
relative of 45.55 m/s
= 400 / 45.55
= 8.78 sec.

17) ANSWER: D
Explanation:
Speed of the train = 30 km/hr
Speed of man = 3 km/hr
Length of the train = 165m
If direction is given in opposite direction then we add both speed.
That is (30 + 3 )= 33 km/hr
Convert km/hr into m/s
33km /hr = (33 * 5 / 18) m/s = 55/6 m/s
If the train time taken to passing a man who running in opposite direction
That is 55/6 m/s
So we can easily get the distance of m/s
So it cover 165 m
i.e., 165 * 6 /55 = 18 sec.

18) ANSWER: A
Explanation:
Length of the train = 200 m
Man walking speed = 10 km/hr
Let the speed of the train be ‘x’ km/hr
Speed of the train relative to man = (x +10) km/hr
Convert km/hr into m/s
(x +10)km/hr = (x + 10) * 5 / 18 m/s
Speed = distance / time
200 / (x + 10)* 5 / 18 = 12
18 * 200 / 5(x + 10) = 12
3600 = 60(x +10)
3600 = 60x +600
60x = 3000

www.ibpsguide.com | estore.ibpsguide.com | www.sscexamguide.com


125
Complete Quantitative Aptitude Questions

X = 50
So speed of the train is 110km/hr.

19) ANSWER : C
Explanation:
Length of the train = 110 m
Speed of the train = 30 km/hr
Speed of the boy = 3.5 km/hr
Speed of the train relative to man = (30 + 3.5) km/hr
= 33.5 km/hr
Convert km/hr into m/s
33.5 km/hr = (33.5 * 5 /18) m/s
= 167.5 / 18
= 9.30 m/s
Time taken by the train to cross the man = time taken by it to cover 110 m at 9.30 m/s
= 110 / 9.30
= 11.8 seconds
= 12 sec(approx.).

20) ANSWER: C
Explanation:
Length of the train = 330 m
Speed = distance / time.
Speed of the train relative to man = 330 / 18 m/s
= 110 / 6 m/s
Convert m/s into km/hr
Therefore 110 / 6 m/s = (110 / 6 * 18 / 5) km/hr
= 22 * 3 km/hr
= 66 km/hr
Let the speed of the train be ‘x’ km/hr
Relative speed = (x +18) = 66 km/hr
X + 18 = 66
X = 48
Therefore the speed of the train is 48 km/hr.

TYPE: 3

www.ibpsguide.com | estore.ibpsguide.com | www.sscexamguide.com


126
Complete Quantitative Aptitude Questions

21) A girl is walking at a speed of 15 km/hr along a railway track. If she is 600 m ahead of the train which is
300 m long and runs at a speed of 180 km/hr in same direction, then what is the time required to pass a girl?
A) 19.63sec
B) 32sec
C) 8.2sec
D) 28.3sec

22) A person runs on the platform of 90 m at a speed of 5km/hr in the same direction of the passenger train.
Find the time taken by the train to cross the running person if speed of the train is 35.5 km/hr? (Length of train
= Length of platform)
A) 5.32sec
B) 7.5sec
C) 12.6sec
D) 10.62sec

23) A goods train overtakes two boys who are walking in the same direction in which the train is going, at the
rate of 4 kmph and 8 kmph and passes them completely in 18 and 20seconds respectively. The length of the
goods train is :
A) 50m
B) 200m
C) 160m
D)80 m

24) A train overtakes two girls walking along a railway track. The first girl walks at 9 km/hr. The other girl
walks at 10.8 km/hr. The train needs 16.8 and 17 seconds respectively to overtake them. What is the speed
of the train if both the girls are walking in the same direction as the train?
A) 25 m/s
B) 45 m/s
C) 30m/s
D) 40m/s

25) Sakthi running at 27 kmph alongside a railway track in 272 metres ahead of the engine of a 240 metres
long train running at 135 kmph in the same direction. In how much time will the train pass sakthi?
A) 24
B) 29
C) 36
D) 17

www.ibpsguide.com | estore.ibpsguide.com | www.sscexamguide.com


127
Complete Quantitative Aptitude Questions

26) A train overtakes two persons like A and B who are walking in the same direction in which the train is
going, at the rate of 8 kmph and 16 kmph and passes them completely in 36 and 40seconds respectively. The
length of the train is:
A) 800m
B) 750m
C) 250m
D) 450m

27) A Superfast train overtakes two persons walking along a railway track. The first one walks at 9 km/hr. The
other one walks at 10.8 km/hr. The train needs 16.8and 17seconds respectively to overtake them. What is the
speed of the train if both the persons are walking in the same direction as the train?
A) 250 kmph
B) 487kmph
C) 162kmph
D) 312kmph

28) A 250 m goods train takes 50 s to cross a person who is going in the same direction with the speed of 8
km/h. After crossing that person, the train can reach next station in 2 hr. How long that person takes to reach
that station after being crossed by them?
A) 2 ½ hr
B) 5hr
C) 3 1/2hr
D) 1hr

29) The Mumbai express 500m long passes a man running at 20km/hr in the same direction in which the
express is going in 40sec. The speed of the express is.
A) 50km/hr
B) 45km/hr
C) 25km/hr
D) 65km/hr

30) A train 200m in length, travels at 120 km/hr. In what time it will pass a boy who is walking at 12km an
hour.
a) against(opposite) it:
b) in the same direction?
A) 5.4,6.66

www.ibpsguide.com | estore.ibpsguide.com | www.sscexamguide.com


128
Complete Quantitative Aptitude Questions

B) 7.2,5.2
C) 3.5,6
D) 12,16

21) ANSWER:A
Explanation:
Speed of a girl = 15 km/hr
speed of train = 180 km/hr
By using the condition,
The speed values of train and the moving object are subtracted if they are moving in same direction.
Speed of train relative to walking person (girl) = (180 -15)kmph= 165 kmph
Convert km/hr into m/s, 165 kmph = (165*5/18) m/s
= 45.83 m/s
Distance to be covered by the train = (600+300)m
= 900m
Therefore, time taken by the train to cross the girl = Distance over speed
= 900 /45.83
= 19.63
Hence the time taken by the train to cross the girl is 19.63seconds

22) ANSWER:D
Explanation:
speed of the train = 35.5 km/hr
speed of the person = 5 km/hr
As the train and the running person move in same direction, their speed values are subtracted to find the
relative speed.
Relative speed (Speed of train relative to man) = (35.5-5)kmph = 30.5 kmph
Convert km/hr into m/s, 30.5 kmph = (30.5*5/18) m/s
= 8.47 m/s
We know, speed = distance/time
Therefore, time taken by the train to cross the running person = Time taken by the train to cover
90 m at a relative of 8.47 m/s
= 90/8.47
=10.62 sec
Hence, the time taken by the train to cross the running person is 10.62 sec

www.ibpsguide.com | estore.ibpsguide.com | www.sscexamguide.com


129
Complete Quantitative Aptitude Questions

23) ANSWER:B
Explanation:
Let the length of the goods train be x km and its speed be y km/hr.
Then, speed relative to first boy = (y-4) km/hr.
Speed relative to second boy = (y-8) km/hr.
x/y-4 = 18/60*60 and x/y-8 = 20/60*60
3600x =18y -72 ………….(1)
3600x = 20y – 160……….. (2)
From (1) and (2),
20y- 160 = 18y -72
2y = 88
y =44………………….(3)
substitute y value in (1) ,
x= (18*44)- 72 / 3600
= 720/3600
= 0.2*1000
= 200 m
Therefore the length of the goods train is 200m

24) ANSWER:B
Explanation:
First girl speed = 9km/hr
= 9 * 5 / 18 m/s
= 5/2 m/s
= 2.5 m/s
second girl speed = 10.8 km/hr
= 10.8 * 5/18 m/s
= 54/18
= 3 m/s
Let the speed of the train be x m/s and speed of the train = distance / time
(x – 2.5 ) * 16.8 = (x – 3) * 17
16.8x – 42 = 17x – 51
0.2x = 9
x = 45
Therefore the speed of the train is 45 m/s

25) ANSWER:D

www.ibpsguide.com | estore.ibpsguide.com | www.sscexamguide.com


130
Complete Quantitative Aptitude Questions

Explanation:
Speed of the train = 135 kmph
Speed of sakthi = 27 kmph
Speed of train relative to sakthi = (135-27) kmph
=108 kmph
We have to find the time in seconds, so convert the speed in the unit of m/sec.
108 kmph = (108 * 5/18) m/s
= 30 m/s
Length of the train = 240 m
Distance covered by sakthi = 272m
Total Distance to be covered = (240 +2 72) m
= 512 m
Speed of the train = Distance / Time
Therefore Time taken = ( 512 / 30) sec
= 17 sec

26) ANSWER:A
Explanation:
person A at speed = 8kmph
convert the speed in the unit of m/sec
8kmph =( 8*5/18) m/s
= 20/9 m/s
person B at speed = 16 kmph
convert the speed in the unit of m/sec
16 kmph = ( 16*5/18) m/s
= 40/9 m/s
Let the length of the train be x metres and its speed by y m/sec.
Then (x/y-20/9)=36 and (x/y-40/9) = 40
(9x/9y-20) =36 and (9x/9y-40) = 40
36y-x = 80 …………(1)
360y-9x = 1600……….(2)
Solving (1)and (2),we get
x =800 m
Therefore length of the train is 800 m
27) ANSWER:C
Explanation:
Given that the first one walks at 9 km/hr

www.ibpsguide.com | estore.ibpsguide.com | www.sscexamguide.com


131
Complete Quantitative Aptitude Questions

convert the speed in the unit of m/sec


9 km/hr = (9*5/18) m/s
= 5/2 m/s
= 2.5 m/s
Another one walks at 10.8 km/hr
convert the speed in the unit of m/sec
10.8 km/hr = (10.8*5/18)m/s
= 3 m/s
Let the speed of the train be x m/sec.
(x- 2.5)*16.8 = ( x-3)*17
16.8x - 42 = 17x -51
0.2 x =9
x = 45 m/s
convert the speed in the unit of km/hr
Speed of the train = (45*18/5)kmph
Speed of the train = 162 km/ hr

28) ANSWER:A
Explanation:
Speed of the person = 8 kmph
= (8*5/18)m/s
= 20/9 m/s
Then relative speed of train = (x-20/9)m/s
As train takes 50 sec to cross the person
Therefore 50 =250/(x-20/9)
50 = 2250/ 9x-20
450x –1000 = 2250
450x = 1250
x = 125/45
x= 25/9 m/s
Now distance covered by the train in 2 hr
= 25/9*60*120
= 20,000 m
= 20 km
Thus time taken by the person to cover the distance of
20km= 20/8 hr=2 ½ hr

www.ibpsguide.com | estore.ibpsguide.com | www.sscexamguide.com


132
Complete Quantitative Aptitude Questions

29) ANSWER : D
Explanation:
Length of the train = 500m
Speed of the expess relative to man = (500 / 40)m/s
= 25/2 m/s
Convert m/s into km/hr
25 / 2 m/s = 25 / 2 * 18 / 5 = 45 km/hr
Let the speed of the express be ‘x’ km/hr
Then the relative speed = (x -20)km/hr
Therefore x – 20 = 45
x = 45 + 20
= 65km/hr
Speed of the express = 65 km/hr.

30) ANSWER: A
Explanation:
I) Speed of the train = 120 km/hr
Speed of the boy = 12 km/hr
Relative speed = (120 + 12) km/hr
Convert km/hr into m/s
132 km/hr = (132 * 5/ 18)m/s
= 36.66 m/s
Time = distance / speed
= 200 / 36.6
= 5.4 sec
II) Relative speed for same direction = (120 – 12)km/hr
= 108 km/hr
Convert km/hr imto m/s
108 km/hr = (108 * 5 / 18)m/s
= 30 sec
Time = distance / speed
= 200 / 30
= 6.66 sec

Type 4

www.ibpsguide.com | estore.ibpsguide.com | www.sscexamguide.com


133
Complete Quantitative Aptitude Questions

31) A passenger train travelling at 96 kmph completely crosses another train having half its length and
travelling in opposite direction at 84 kmph, in 24 seconds. It also passes a railway platform in 90 seconds.
The length of the platform is
A) 1250 m
B) 2258m
C) 1600 m
D) 3725m

32) Two goods trains having equal lengths, take 30 seconds and 45 seconds respectively to cross a telegraph
post. If the length of each train is 360 meters, in what time (in seconds) will they cross each other when
traveling in opposite direction?
A) 36
B) 12
C) 24
D) 48

33) The distance between two stations ooty and chennai is 615km. A train with speed of 75 km/h leaves ooty
at 8:00 am towards chennai. Another train with speed of 105 km/h leaves Chennai at 9 : 00 am towards ooty
. Then, at what time both trains meet?
A) 6
B) 12
C) 18
D) 24

34) A train leaves Mumbai for chennai at 6 : 45 a.m. and goes at the rate of 100 km/h. Another train leaves
Chennai for Mumbai at 4.30a.m. and goes at the rate of 120 km/h. If the distance between both is 1240 km, at
what distance from Mumbai will the two trains meet?
A) 210km
B) 324km
C) 115km
D) 440km

35) Two trains A and B are moving in opposite directions at 180 km/hr and 270 km/hr. their lengths are 3.30
km and 2.7 km respectively. The time taken by the slower train to cross the faster train in second is.
A) 48sec
B) 13 sec
C) 38sec

www.ibpsguide.com | estore.ibpsguide.com | www.sscexamguide.com


134
Complete Quantitative Aptitude Questions

D) 24sec

36) Two trains, both 50m long, moving in opposite directions cross each other in 4 seconds. If one is moving
thrice as fast the other, then the speed of the faster train is.
A) 45km/hr
B) 19 km/hr
C) 95 km/hr
D) 67.5 km/hr

37) A and B are 2 trains running in opposite direction cross a man standing on the platform in 54sec and 34
sec respectively and they cross each other in 46sec. The ratio of their speed is.
A) 1:3
B) 3:2
C) 2:3
D) 2:2

38) A train p starts from delhi at 8pm and reaches kanyakumari at 10pm. While another train q starts from
kanyakumari at 8pm and reaches delhi at 11pm. The two trains will cross each other at.
A) 9.24pm
B) 9.10pm
C) 9.02pm
D) 9.12pm

39) Two mail trains Pand Q of 300 m and 600m, run at speed of 130 km/hr and 160 km/hr respectively, in the
direction opposite direction to each other. Find the time required to cross each other after the moment they
met?
A) 11.17 sec
B) 12sec
C) 9.10sec
D) 21sec

40) A goodstrain of 700 m runs at a speed of 165 km/hr. A person traveling in it observes that the mail train
moving in opposite direction takes 30 seconds to cross him. Find the speed of the mail train, if it is 750 m
long.
A) 8.4 m/s
B) 5.3 m/s
C) 2.5 m/s

www.ibpsguide.com | estore.ibpsguide.com | www.sscexamguide.com


135
Complete Quantitative Aptitude Questions

D) 6.8 m/s
31) ANSWER:C
Explanation:
Let the length of the first train be x metres
Then, is x /2metres
Relative speed = (96+84)kmph
= 180 kmph
Convert km/hr into m/s,
= 180*5/18
= 50 m/s
Therefore (x+x/2)/50 = 24
x+x/2 =1200
3x/2= 1200
3x = 2400
x = 800
Length of first train = 800 m.
Let the length of platform be y metres.
Speed of the first train = 96*5/18 =480/18 m/s
Therefore (800+y)/(450+18) = 90
(800+y) = 90*480/18
y = 2400- 800
= 1600
Hence, the length of the platform is 1600 m

32) ANSWER:A
Explanation:
Speed of train = distance /time
= 360/30 m/s
= 12 m/s
Speed of train 2 = distance /time
= 360 /45 m/s
= 8 m/s
If 2 trains are moving in opposite direction then their speed values are added to find the relative speed.
Relative speed = 12+8 =20 m/sec
distance covered = 360+360 = 720 m
Time = distance/speed
= (720/20)

www.ibpsguide.com | estore.ibpsguide.com | www.sscexamguide.com


136
Complete Quantitative Aptitude Questions

= 36 Seconds
Hence the required answer is 36 seconds

33) ANSWER:B
Explanation:
First train leaves at 8.00am and second leaves at 9.00am
So,first train that is , from ooty to Chennai has covered
75 km distance in 1 hr.
So , distance left by between the station = 615-75=540km
Now trains are travelling in opposite directions.
We know that if 2 trains are moving in opposite direction then their speed values are added to find the
relative speed.
So,Relative speed = 75+105 kmph
= 180 km/hr
Time taken to cover 180kmph = 540/180
= 3 hrs
Therefore, the time at which both the trains will meet , 3 hr after second train left
i.e., 9.00 am +3 = 12.00 pm

34) ANSWER: D
Explanation:
Given speed of the first train = 100kmph
speed of the second train = 120kmph
The second train leaving Chennai starts its journey earlier and it travels = 120 *(6.45a.m.
- 4.30a.m.)
= 120*2.15
= 120 * 2 ¼
= 120 * 9/4
= 270 km, when the first trains starts its journey
Now both the trains covers(1240-270) = 970km with relative speed(100+120)= 220 kmph
Therefore thetrains after meet 970/220 =4.40 hrs
After the first starts at 6.45a.m,
Now the first train covers 4.40 *100 = 440 km
Therefore the first train meet to the second train at 440km

35) ANSWER: A
Explanation:

www.ibpsguide.com | estore.ibpsguide.com | www.sscexamguide.com


137
Complete Quantitative Aptitude Questions

Speed of 1st train = 180 km/hr


Speed of 2nd train = 270 km/hr
Relative speed = (180 + 270)km/hr
= 450 km/hr
= (450 * 5 / 180)m/s
= (2250 / 18)m/s
=125 m/s
Distance covered = (3.30 + 2.70)km
= 6km = 6000m
Required time = 6000 / 125
= 48sec
36) ANSWER:D
Explanation:
Let the speed of the slower train be ‘x’ m/sec
The speed of the sfaster train = 3x m/s
Relative speed = (x +3x)m/sec
= 4x m/sec
50 + 50 / 4 = 4x
100 / 4 = 4x
100 = 16x
x = 100 / 16 = 25/4
so speed of the faster train = 3 * 25 / 4
= 75 / 4 m/s = (75/4 *18 /5)km/hr
= 67.5 km/hr

37) ANSWER:B
Explanation:
Let the speed of the 1st train be ‘x’ m/sec and
the speed of the 2nd train be ‘y’ m/sec
then the length of the 1st train = 54 x metres
length of the 2nd train = 34 y metres
54x + 34y / x+ y = 46
54x + 34y = 46x + 46y
54x – 46x = 46y – 34y
8x = 12y
x/y = 12 / 8 = 6/4 = 3/2
x:y = 3:2

www.ibpsguide.com | estore.ibpsguide.com | www.sscexamguide.com


138
Complete Quantitative Aptitude Questions

38) ANSWER: D
Explanation:
Suppose the distance between delhi and kanyakumari is x km
The time taken by p to cover x km = 2hrs
The time taken by q to cover x km = 3hrs
Therefore speed of p = distance / time
= x / 2 km/hr
Speed of q = distance / time
= x / 3 km/hr
Let them meet y hrs after 8pm
Then, xy / 2 + xy / 3 = x
Y (1/2 + 1/3) = 1
5/6 = 1/y
5y = 6
y = (6/5 * 60)min= 72min
So the two trains meet at 9.12pm

39) ANSWER:A
Explanation:
Length of train P = 300 m
Length of train Q = 600 m
Speed of train p = 130 kmph
Convert km/hr into m/s,
Speed of train p =(130 *5/18)m/s
= 650/18 m/s
= 36.11 m/s
Speed of train Q = 160 kmph
Convert km/hr into m/s,
Speed of train Q = (160*5/18)m/s
= 800 / 18 m/s
= 44.44 m/s
As both trains move opposite to each other,
relative speed = 36.11+44.44 = 80.55 m/s
Distance = (Length of train A + Length of train B)
= (300+600)m
= 900 m

www.ibpsguide.com | estore.ibpsguide.com | www.sscexamguide.com


139
Complete Quantitative Aptitude Questions

We know, time =distance /speed


= 900/ 80.55
= 11.17seconds

40) ANSWER:C
Explanation:
Speed of goods train = 165km/hr
length of mail train (P) = 750m
length of goods train (Q) = 700m
Alternately, we can directly use the formula:
Time = ( p+q) / (v1+v2) sec
(here P and Q are length of trains and V1 and V2 are speeds of two trains)

Goods train and the mail train move in opposite direction.


Hence, the relative speed is the addition of two speeds.
Convert 165 km/hr into m/s
165 km/hr = (165*5/18)m/s
= 45.833 m/s
Therefore,
Time = ( p+q) / (v1+v2) sec
30 = (750+ 700) /(45.833+ v2)
= 1450 / 45.833 +v2
45.833+v2 = 48.333
V2 = 2.5 m/s
Therefore the speed of the mail train is 2.5 m/s

TYPE 5:
41) Two metro train 360 meters and 360 meters in length respectively and running in same directions, one at
the rate of 116 km and other at the rate of 100 km an hours. What time will they be cross of each other?
A) 100 sec
B) 162sec
C) 132sec
D) 126sec

42) Let p and q be two trains 65m and 70m long are running on parallel tracks in the same direction with a
speed of 34km/hr and 25 km/hr. how long will it take to clear off each other from the moment they met?
A) 54sec

www.ibpsguide.com | estore.ibpsguide.com | www.sscexamguide.com


140
Complete Quantitative Aptitude Questions

B) 56sec
C) 45sec
D) None of these

43) X and y are two trains of length 500m and 400m run on parallel lines. When they run in the same direction
it will take 60 sec to cross each other. When they run in opposite direction, it will take 20 sec to cross each
other. Find the speed of 2 trains (in km/hr)
A) 116 and 42 km/hr
B) 54 and 78km/hr
C) 104 and 60km/hr
D) 108 and 54km/hr

44) Two passenger trains are running in the same direction on parallel tracks at 210km/hr and 150km/hr
respectively. The faster train passes a man 81 sec faster than the slower train. Find the length of the faster
train.
A) 1250m
B) 1150m
C) 1350m
D) 1450m

45) It takes 60 sec for a train running at 162 km/hr to cross a bridge. And it takes 36 sec for the same speed
to cross a boy walking at the rate of 18km/hr in the same direction in which the train is running. What is the
length of the train and length of the platform.
A) 1440 &1260m
B) 1220 &60m
C) 1000 & 1460m
D) 1260 &1450m

46) Find the time taken by mailtrain 200m long running at a speed of 120 km/hr to cross another train of
length 160m running at a speed of 96km/hr in the same direction.
A) 34sec
B) 52sec
C) 45sec
D) 54sec

47) A train M speeding with 60km/hr crooses another train N, running in the same direction in 1 min. If the
lengths of the trains M and N be 50m and 100m respectively. What is the speed of train N?

www.ibpsguide.com | estore.ibpsguide.com | www.sscexamguide.com


141
Complete Quantitative Aptitude Questions

A) 15 km/hr
B) 61km/hr
C) 51km/hr
D) 50k/hr

48) Two trains 300 metres and 360 metres long are running in the same direction with speeds of 216 kmph
and 162 kmph .In how much time will the first train cross the second?
A) 54sec
B) 44sec
C) 40sec
D) 14sec

49) A and B are two trains 150m and 100m long are running on parallel rails at the rate of 30 kmph and 40
kmph respectively.In how much time will they cross each other if they are running in the same direction.
A) 80 sec
B) 90sec
C) 105sec
D) 45sec

50) Two trains are running at 120 km/hr and 60km/hr respectively in the same direction. Fast train completely
passes a boy sitting in the slower train in 15 seconds. What is the length of the fast train?
A) 150m
B) 550 m
C) 250m
D) 215m

41) ANSWER: B
Explanation:
Length of the 1st metro train = 360m
Length of the 2nd metro train = 360m
Total length of the both metro train = 360 + 360 = 720 m
speed of the 1st metro train = 116 km/hr
speed of the 2nd metro train = 100 km/hr
here two fast trains are running in same direction and their,
relative speed = (116 -100)km/hr
= 16km/hr
Convert km/hr into m/s

www.ibpsguide.com | estore.ibpsguide.com | www.sscexamguide.com


142
Complete Quantitative Aptitude Questions

16km/hr = (16 * 5 / 18)m/s


= 40/9 m/s
Speed = distance / time
Required time = distance / speed
= (720/(40/9))sec
= 720 * 9 /40
= 162sec.

42) ANSWER : A
Explanation :
Speed of the train p = 34km/hr
Speed of the train q = 25 km/hr
If the direction is given in the same direction then we subtract
So relative speed of trains = (34 – 25)km/hr
= 9 km/hr
Convert km/hr into m/s
9 km/hr = (9 * 5 / 18)m/s
= 5/2 m/s
Time = distance / speed
Here,distance = sum of length of trains
= (65 + 70)m
= 135m
Therefore time = (135 / (5/2))sec
= 270/5 sec
=54 sec.

43) ANSWER : D
Explanation:
Let the speed of 2 trains be a1 and a2
Total distance covered to cross each other = (500 + 400)m
= 900m
When they run in the same direction,
Relative speed = a1 – a2
= 900 /60
A1 – a2 = 15 ------1
When they run in the opposite directions,
Relative speed = a1 + a2

www.ibpsguide.com | estore.ibpsguide.com | www.sscexamguide.com


143
Complete Quantitative Aptitude Questions

= 900 / 60
A1 + a2 = 45 -----2
Solving 1 and 2 we get
2a1 = 60 a1 = 30 m/s ---------------3
Convert m/s into km/hr
a 1= (30 * 18 / 5)km/hr
= 108 km/hr
Put 3 value in 1 to get a2 value,
a 2 = 30 – 15 15 m/s ----4
Convert m/s into km/hr
a 2= (15 * 18 / 5)m/s
= 54km/hr
Speed of two trains are 108 and 54 km/hr

44) ANSWER: C
Explanation:
Speed of 1st train = 210km/hr
Speed of 2nd train = 150km/hr
Here the two passenger trains are running in same direction, so
Relative speed = (210 – 150)km/hr
= 60km/hr
Convert km/hr into m/s
60 km/hr = (60 * 5 / 18) m/s
= 50 / 3 m/s
Time = distance (length)/ speed
Length of the faster train = relative speed * time taken by train to pass
= 50/3 * 81
= 1350m
45) ANSWER: A
Explanation:
Relative speed of the train to man = (162 – 18)
= 144km/hr
Convert km/hr into m/s
144km/hr = (144 * 5 /18) m/s
= 40 m/s
When the train passes a boy, it covers the distance which is equal to its own length in the above
relative speed

www.ibpsguide.com | estore.ibpsguide.com | www.sscexamguide.com


144
Complete Quantitative Aptitude Questions

Given that it takes 24se for the train to cross a boy


So length of the train = relative speed * time
= 40 *36
= 1440 m
Speed of train = 162 km/hr
= (162 * 5 / 18)m/s
= 45 m/s
When the train crosses the bridge it over the distance which is equal to the sum of length of trains & bridge
So the sum of length of train & bridge = speed * time
= 45 * 60
= 2700 m
i.e., length of the train + length of bridge = 2700m
1440 + length of bridge = 2700
length of bridge = 1260 m
the length of the bridge & train are 1260 and 1440m

46) ANSWER: D
Explanation:
Total distance covered to cross each other = 200 + 160
= 360m
If the direction is given in the same direction then we subtract
Relative speed of two trains (same direction) = (120 – 96)km/hr
= 24 km/hr
Convert km/hr into m/s
24 km/hr = (24 * 5 / 18) m/s
= 20 / 3 m/s
Time taken to cross = distance / speed
= 360 / (20/3)
= 360 * 3 / 20
= 54sec
Time taken by the faster train to cross the slower train is 54sec.

47) ANSWER: C
Explanation:
Let the speed of be train N be ‘x’km/hr
Speed of M relative to N = (60 – x)km/hr
= (60-x) * 5/18]m/s

www.ibpsguide.com | estore.ibpsguide.com | www.sscexamguide.com


145
Complete Quantitative Aptitude Questions

= (300 – 5x / 18)m/s
150 / (300 – 5x / 18) =60
2700/300-5x=60
2700=18000-300x
300x=15300
x= 153/3
x=51km/hr
Therefore the speed of the train is 51 km/hr

48) ANSWE: B
Explanation:
Length of the 1st train=300m
Length of the 2nd train=360m
Speed of the first train=216kmph
Speed of the second train=162kmph
Relative speed =(216-162)kmph
=54kmph.
Convert km/hr into m/s
=(54*5/18) m/s
=15 m/sec.
Time taken by the trains to cross each other=Time taken to cover (300+360)m at 15 m/s
=(660/15) sec
=44 sec
Therefore the first train cross the second at 44 sec.

49) ANSWER: B
Explanation:
Length of first train=150m
Length of second train=100m
Speed of first train =30kmph
Speed of second train =40kmph
If the direction is given in the same direction then we subtract
Relative speed = (40-30)kmph
=10kmph
Convert km/hr into m/s
=(10*5/18)m/s

www.ibpsguide.com | estore.ibpsguide.com | www.sscexamguide.com


146
Complete Quantitative Aptitude Questions

=25/9m/s
Total distance covered=sum of the length of trains
=(150+100)m
=250m
Time = distance/speed
Time taken =(250*9/25)sec
=90 sec.

50) ANSWER: C
Explanation:
Speed of the 1st train = 120km/hr
Speed of the 2nd train = 60km/hr
If the direction is given in the same direction then we subtract the relative speed.
Relative speed = (120 – 60)km/hr
= 60km/hr
Convert km/hr into m/s
= (60 * 5 / 18)m/s
= 50 / 3 m/s
Length = speed *time
Length of the faster train = (50 / 3 * 15)m
=(750/3)m
= 250m
Therefore the length of the faster train is 250m

8. AVERAGE

1) The average age of a family of 24 members is 132 years. If the age of the youngest member is 22 years,
the average age of the family at the birth of the youngest member was?
A) 75
B) 110
C) 100
D) 95

2) There are three different categories of jobs A, B and C. The average salary of the students who got the job
of A and B categories is Rs.32 lakh per annum. The average salary of the student who got the job of B and C
category is 54 lakh per annum. And the average salary of those students who got the job of A and C is Rs.44

www.ibpsguide.com | estore.ibpsguide.com | www.sscexamguide.com


147
Complete Quantitative Aptitude Questions

lakh per annum. The most appropriate (or closet) range of average salary of all the three categories (if it is
known that each student gets only one category for jobs i.e, A, B and C):
A) lies between 30 & 44
B) lies between 48 & 56
C) lies between 38 & 45
D) lies between 49 & 50

3) In shewak's opinion, his weight is greater than 76 kg but less than 83 kg. His sister doest not agree with
shewak and she thinks that shewak’s weight is greater than 71 kg but less than 81 kg. His father's view is that
his weight cannot be greater than 79 kg. If all are them are correct in their estimation, what is the average of
different probable weights of shewak?
A) 67kg
B) 68kg
C) 72kg
D) 78kg

4) In a post graduate examination the marks obtained by a student is 75 per paper. If he had obtained 33
marks more in Evs paper & 27 more marks in science paper, then his average per paper is increased by 3
marks. Then how many papers were there in exam?
A) 10
B) 12
C) 14
D) 20

5) Students of two university appeared for a common test of maximum 60 marks. The average of their marks
for university 1 & university 2 are 39 & 42 respectively. If the no of students of university 1 is twice the no of
students of university 2. Then what is the average marks of all students of both the university?
A) 40
B) 42
C) 26
D) 36

6) Find the average of even numbers above 1000 & upto 1 lakh?
A) 50500
B) 50000
C) 49500
D) 51500

www.ibpsguide.com | estore.ibpsguide.com | www.sscexamguide.com


148
Complete Quantitative Aptitude Questions

7) The average age of 11 daughters of a family is 12yrs. Average age of the daughters together with their
parents is 27yrs. If mother is 14yrs younger than father. Then find the father's age
A) 127 yrs
B) 116.5 yrs
C) 168yrs
D) 170yrs

8) The average price of 12 note books is Rs.15 while the average price of 10 of these note books is Rs.13. Of
the remaining 2 note books, if the price of one note book is 50% more than the price of the other, what is the
price of each of these two note books?
A) Rs. 30, Rs.20
B) Rs. 8, Rs. 30
C) Rs. 10, Rs. 16
D) Rs. 12, Rs. 20

9) The average monthly salary of 24 labours and 6 supervisor in a factory was Rs. 1200. When one of the
supervisor whose salary was Rs. 1440, was replaced with a new supervisor, then the average salary of the
team went down to 1160. What is the salary of the new supervisor?
A) 170
B) 420
C) 390
D) 240

10) The average scores of a batch of students in a test is 84. The 36% of students's average score is 57 and
42% of students's score is 84. Then find the average score of remaining 22% of students approximately,
A) 128
B) 131
C) 119
D) 106

1) Answer: B)
Total age of the family of 24 members = 132* 24 = 3168 yrs
Total age of the family members 22 yrs ago = 3168 - (24*22) = 2640
at the time total members in a family = 24
average age of the family at the birth of the youngest member = 2640/24 = 110yrs.

www.ibpsguide.com | estore.ibpsguide.com | www.sscexamguide.com


149
Complete Quantitative Aptitude Questions

2) Answer: C)
Let the number of students who got the jobs of A, B and C categories is a, b and c respectively,
∴Total salary of Aand B = 32 (a +b)
∴Total salary of B and C = 54 (b+ c)
∴ Total salary of A and C = 44 (a +c)
Then the total salary = 32(a+b)+54(b+c)+44(a+c)/2(a+b+c)
= 76a+86b+98c / 2(a+b+c)
= 38a+43b+49c /(a+b+c)
= 38 (a+b+c)+(5b+11c)/(a+b+c)
= 38 +some positive value
∴ So the minimum salary must be Rs.38 lakh and the maximum salary cannot exceed 45, which is the highest
of the three..

3) Answer: D)
Let shewak's weight be X kg.
According to shewak , 76 < X < 83
According to shewak 's sister, 71 < X < 81.
According to shewak 's father, X ≤ 79
The values satisfying all the above conditions are 77, 78and 79.
Required average=[ (77+78+79)/3] = 201/ 3 = 78kg.

4) Answer: D)
Let the number of paper be A.
Then total marks earned him = 75A
from questions,
75A + 33+ 27= 78A
3A = 60=> A=20 = number of subjects

5) Answer: A)
Let number of students of university2 be N and
the no of students of university 1 be 2N
the average of university1 and university 2 is 39 and 42
total marks of university 1 students and university 2 students,
= 2N*39= 78N and N*42 =42N
average of both university,
=(78N+42N) /(N+2N) = 40 marks

www.ibpsguide.com | estore.ibpsguide.com | www.sscexamguide.com


150
Complete Quantitative Aptitude Questions

6) Answer: A)
No of even numbers upto 1lkah = 50000
No of even numbers upto 1000 = 500
No of even numbers above 1000& upto 1 lakh= 49500
sum of first N even numbers = N*N
sum of even numbers above 1000 & upto 1 lakh = 50000*50000 - 500*500 = 2499750000
average of even numbers above 1000& upto 1 lakh = 2499750000/ 49500 = 50500

7) Answer: B)
Total age of 11 daughters of a family = 11*12 = 132
father age = 14 +mother age
total age of the family
(11 daughters + father + mother) is,
132+mother +14+ mother = 27 * 13
mother = 102.5
father age = 14+102.5 =116.5years

8) Answer: A)
Total price of 12 note books = Rs. 180
Total price of 10 note books= Rs. 130
⇒ The price of 2 note books = Rs. 50
Let the price of each book be x and y.
⇒ x + y = 50 ---------------- (1)
Given that the price of 1 note book is 50% more than the other price
150y/100+y=50
y=20
Substituting y value in (1) we get, x=30

9) D)
The total salary amount = 30×1200=36000
The salary of the exiting supervisor = 1440
Therefore, the salary of 24 labours and the remaining 5 supervisors:
=36000-1440=34560
When a new supervisor joins, the new average salary drops to Rs.1160 for the total team of 30of them.
The total salary for the 30people i.e., 24 labours, 5old supervisors and 1 new supervisor = 1160×30= 34800
Therefore, the salary of the new supervisor is 36000-34800=1200 less than that of the old supervisor who left
the company, which is equal to 1440−1200=240

www.ibpsguide.com | estore.ibpsguide.com | www.sscexamguide.com


151
Complete Quantitative Aptitude Questions

10) Answer: A)
let the number of students be 100,
then, 100*84 = 36*57 + 42*84 + 22*x
22x = 8400-2052-3528
x = 128.18

11) In a famous restaurant rooms were numbered from 401 to 430, each room gives an earning of Rs. 4675/
day for the first 15 days a month and for the later half, Rs. 5370/ day per room. Find the average income room
per day over the month of 30 days?
A) 5700.5
B) 5872.7
C) 5900.5
D) 5022.5

12) The average height of the students in a group was 360cm. When 10 students whose height is 292.8cm
are newly admitted. The average height of the group was reduced by 24cm.How many students are present
in the group?
A) 29
B) 25
C) 28
D) 14

13) Rohit married 21 years ago at the age of 90 years. His wife's present age 99years. If 24 years later the
average age of rohit, his wife and their son was 90 years, then what is son's present age?
A) 15yr
B) 14yr
C) 13yr
D) 12yr

14) The average height of a batch is 344 cm. 16more students with an average height of 320 cm joined the
batch therefore decreasing the average height of the batch by 12 cm. Find the total strength of the batch?
A) 12
B) 11
C) 14
D) 16

www.ibpsguide.com | estore.ibpsguide.com | www.sscexamguide.com


152
Complete Quantitative Aptitude Questions

15) The average age of the assembalage of people was 114 yrs. When the 2 people of the assembalage
went out with age of age of 84 yrs and 108yrs, then the average of the assembalage increased by 12yrs. How
many people are there initially?
A) 3
B) 4
C) 5
D) 6

16) The monthly expenditure of Moorthy family was Rs 3120 during the first 4 months, Rs 3390 during the
next 3 months and Rs 3618 during the last 5 months of a year. If the total saving during the year is Rs 3540,
find the average monthly income of moorthy family.
A) Rs 3690
B) Rs 3785
C) Rs 3670
D) Rs 3875

17) Saravanan has scored an total of 90% in an examination with five subjects in the ratio 18:14:15:21:22. If
85% is the marks required to get an ‘A’ grade in each subject, then find in how many subjects did he get an
‘A’ grade. Given that the maximum marks in each subject is 120.
A) 2
B) 3
C) 4
D) 5

18) The average age of K and L is 48 years. If M replaced K, the average age would be 40 and if M replaced
L, the average would be 44. What are the ages of K,L,M repectively?
A) 58, 46, 22
B) 32, 44, 36
C) 52,44,36
D) 56, 22,38

19) There were 126 people in a Inn. Due to the entry of 39 new people, the expenses of the inn increase by
Rs. 93 per day while the average expenditure per head diminished by Rs. 9. What was the original
expenditure of the inn?
A) Rs. 6343
B) Rs.5098
C) Rs.6215

www.ibpsguide.com | estore.ibpsguide.com | www.sscexamguide.com


153
Complete Quantitative Aptitude Questions

D) Rs. 5476

20) A batch of 50 people has the oldest person with 160 years of age. The average of the batch is reduced by
2, if the oldest person is replaced by someone new, Find the age of the new person.
A) 45
B) 30
C) 60
D) 75

11) Answer: D)
Total number of rooms = 30
earning in 1st 15 days for 30 rooms
= 15 * 4675* 30 = Rs. 2103750
earning in 2nd 15 days for 30 rooms
= 15 * 5370 * 30 = Rs. 2416500
Average income per room per day over the month of 30 days
= [ 2103750 + 2416500] / [ 30 * 30]
= Rs. 5022.5

12) Answer: C)
let the number of students initially in the class be A,
then, total height = A * 360------ 1
again the no of students increased = A + 10
then, total height A + 10 student = (A +10) * 336----2
the total height of 10 student of 10 new students = 10* 292.8 = 2928cm-----3
from 1,2 , 3
(A+10) *336-2928 =A*360
336A+3360-2928=360A
A=18
Number of students in class = 18+10=28

13) Answer: D)
Rohit present age = 111years
total age of family after 24 yrs = 90*3= 270
present age of rohit & his wife = 111+99 = 210yrs
present age of rohit, his wife & his son= 270- 24*3 =198
present age of son= 210-198=12yrs

www.ibpsguide.com | estore.ibpsguide.com | www.sscexamguide.com


154
Complete Quantitative Aptitude Questions

14) Answer: D)
let the initial strength of the batch be X
total height of the batch initially = 344X
total height of 16 new students = 320 * 16 = 5120 cm
Then, the average height of X+ 16students = 332
332 = [5120 + 344 X]/ [X + 16]
X= 16 students

15) Answer: C)
let the number of people initially be N
total age of N people = 114N
total age of N-2 people = 114N - ( 84+108) = 114N -192
average age of N-2 people = 126 = [ 114N - 192] / [ N - 2]
N=5
Hence th required answer is 5

16) Answer: A)
Total expenditure during the year
= Rs [3120 × 4 + 3390 × 3 + 3618 × 5]
= Rs [12480+10170 +18090 ]
= Rs 40740.
Total income during the year = (40740 + 3540) = 44280.
Average monthly income = (44280/12) = Rs3690.
Hence, the average monthly income of moorthy family is 3690

17) Answer: B)
Maximum total marks = 5 × 120 = 600
Marks scored by Saravanan = 90/100*600=540
Let the marks scored by him in the 5 subjects be 18x,14x,15x,21x and 22x respectively.
18x + 14x + 15x + 21x + 22x = 540 ⇒ 90x = 540 ⇒ x = 6.
So, The marks scored by him in each of the subjects are 18x = 108, 14x = 84, 15x = 90, 21x = 126, 22x = 132
minimum 80% marks are required for ‘A’ grade i.e. 0.85 × 120 = 102
Therefore, he has got an ‘A’ grade in 3 subjects.

18) Answer: C)
Total age of K& L = 48*2 = 96 yrs----->1

www.ibpsguide.com | estore.ibpsguide.com | www.sscexamguide.com


155
Complete Quantitative Aptitude Questions

Total age of L& M = 40*2 = 80yrs----->2


Total age of K& M = 44*2 = 88 yrs----->3
from 1, 2, 3
K+L+M =132---->4
from 1 & 4, M= 36
from 2 & 4, K=52
from 3 & 4, L=44
Hence the required answer is 52,44,36

19) Answer: B)
Let per day average expense = x
126x + 93 = 165(x-9)
126x+93=165x-1485
1578=39x
X=40.46
original expenditure = 126*40.46 = Rs 5098

20) Answer: C)
Average=Sum of observations/Number of observations
Given, a batch of 50 people has the oldest person with 160 years of age.
Let the average of the group be ‘a’.
Sum total of ages = 50 a
Given, average of the batch is reduced by 2, if the oldest person is replaced by someone new.
Let the age of the new person be ‘b’ .
50a – 160 + b = 50(a – 2)
b= 160-100
b = 60 years

21) There were 32 boys in a hostel. If the number of boys be increased by 15, then the expenditure on food
increases by Rs. 43 per day while the average of expenditure of boys is reduced by Rs. 2. What was the
initial expenditure on food per day?
A) Rs.108.2
B) Rs.125.6
C) Rs.135.8
D) Rs.144.5

www.ibpsguide.com | estore.ibpsguide.com | www.sscexamguide.com


156
Complete Quantitative Aptitude Questions

22) The average monthly salary of directors & office boys of an organization Rs. 3750. Then the average
salary of all directors is Rs. 6000 while that of all office boys is Rs. 3000 per month. If there are 120
employees in the organization then find the ratio of manager and office boy.
A) 2:5
B) 3:1
C) 1:3
D) 5:2

23) There are two groups of a class consisting of 72 and 88 students respectively. If the average weight of 1st
group is 80kg and the weight of 2nd group is 70kg. Find the average weight of whole class?
A) 64kg
B) 74.5kg
C) 84.5kg
D) 54kg

24) A person’s age is 150% of what it was 15 years ago, but 76% of what it will be after 15years. What is his
present age?
A) 39.7
B) 45.8
C) 42.8
D) 37.8

25) The average marks scored by two Class A1 and A2 students are 120 and 130 respectively. If 8 students
are moved from Class A2 to Class A1 and the average marks of the two Class get interchanged. Find the
total number of students in two Class put together, if the average marks scored by the 8 students who moved
is 150
A) 25
B) 30
C) 35
D) 40

26) When a girl weighing 90 kgs left a class, the average weight of the remaining 119 students increased by
400 g. What is the average weight of the remaining 119 students?
A) 124
B) 138
C) 145
D) 116

www.ibpsguide.com | estore.ibpsguide.com | www.sscexamguide.com


157
Complete Quantitative Aptitude Questions

27) In an competitive examination, the average was found to be 72 marks. After detecting the errors the
marks of 94 candidates had to be charged from 90 to 66 each, and the average is reduced to 64 marks. Find
the total number of candidates who took the exam.
A) 282
B) 382
C) 828
D) 200

28) In an exam the average marks obtained by a candidate is 82 per paper. If he had obtained 32 marks more
in science paper & 28more marks in social paper, then his average per paper is increased by 15 marks. Then
how many papers when there in examination?
A) 10
B) 6
C) 4
D) 8

29) 18 friends went to a restaurant for taking their breakfast. 17 of them spent RS.24 each on their breakfast
and the last one spent RS.16 more then the average expenditure of all the 18. What was the total money
spent by them?
A) 228
B) 448.9
C) 458.6
D) 428.0

30) The average weight of x,y,z is 90kg. If the average weight of x and y be 80kg and that of y and z be
86kg. Find the weight of y
A) 66kg
B) 63kg
C) 62kg
D) 70kg

21) Answer: A)
Let initial expense of 32 boys = Rs. x
average expense of 32 boys = x/32
average expense of 47 boys = (x + 43)/ 47
then,

www.ibpsguide.com | estore.ibpsguide.com | www.sscexamguide.com


158
Complete Quantitative Aptitude Questions

(x/32) - ((x+43)/47) = 2
x= Rs.108.28
22) C)
let the number of director be X and office boy be 120-x
total salary of director and office boy = 120*3750 =
Rs.450000
total salary of director = 6000x
total salary of office boy = 3000* (120 -x)
total salary of director + total salary of office boy=450000
450000 = 6000x + 3000 * (120 -x)
3000x =90000
x = 30 number of director
number of office boy = 120-30 = 90
ratio of director to office boy = 30:90 = 1:3

23) B)
Total weight of (72 + 88) = (72*80) + (88 * 70)kg
= (5760 + 6160)kg
= 11,920kg
Average weight of the whole class = 11920 / 160
= 74.5 kg
24) B)
let the present age be X yrs then 150% of (X-15)=X; and 76 % of (x+15)= X
150% of (X-15)=76% of(X+15)
3/2(X-15)=19/25(X+15)
X=45.8yrs

25) D)
let the number of students in Class A1 be x and class A2 be y.
Total marks scored by the students will be 120x and 130y, the average gets interchanged after moving
student from y
Thus we get,
130y-8*150=120(y-8)
130y-1200=120y-960
10y=240
Y=24
Similarly, 120x +1200 =130(x+8)

www.ibpsguide.com | estore.ibpsguide.com | www.sscexamguide.com


159
Complete Quantitative Aptitude Questions

120x+1200=130x+1040
10x=160
X=16
Thus the total number of students =24+16=40

26) B
Let the average weight of the 119 students be X.
Therefore, the total weight of the 119 of them will be 119X.
The questions states that when the weight of this student who left is added, the total weight of the class
= 119X+90
When this student is also included, the average weight decreases by 0.4 kgs.
(119X+90)/120=X−0.4
⇒119X+90=120X−48
⇒X=138

27) A)
let the number of candidates be A
total marks of candidates = 72A
after detecting error the change of marks for one candidate = 90 - 66 = 24 marks
change of marks for 94 candidates = 94*24 =2256
after detecting error of N candidates = 64A
then, 72A- 2256= 64A ,
8A=2256
A=282

28) C)
let the number of paper be x
total mark earned him = 82x
then,, 82x+32+28= 97x
15x=60
x= 4 =number of subjects

29) B)
Let the average expenditure of all the 18 be Rs ‘x’.
Then (24 * 17) + (x + 16) = 18x
424 + x = 18x
x = 24.9

www.ibpsguide.com | estore.ibpsguide.com | www.sscexamguide.com


160
Complete Quantitative Aptitude Questions

therefore total money spent 18x


= 18 * 24.9
= Rs.448.9

30) C)
Let x ,y ams z represent their individual weights. Then,
X + y + z = ( 90 * 3) = 270kg
X+ y = 80 *2 = 160kg
Y + z = 86 * 2 = 172kg
Therefore y = (x + y) + ( y+z) – (x+y+z)
= 160 + 172 – 270
Y= 62kg.

31) The average age of 78 students of a batch is 30 years. If the age of the head master be included, then the
average increases by 6 months. Find the age of head master?
A) 56yrs 5 months
B) 66yrs 6 months
C) 56yrs 5 months
D) 69yrs 5 months
32) The average of 12 numbers is 7.9. The average of 5 of them is 6.8, while the average of other five is 7.7.
what is the average of the remaining 2 numbers?
A) 10.25
B) 11.15
C) 12.65
D) 13.25

33) Harish buys petrol at rs 21, Rs.24 and Rs.27 per litre for 3 successive years. What approximately is the
average cost per litre of petrol if he spends Rs.12000 each year?
A) 23.74
B) 54.76
C) 24.66
D) 28.99

34) A batsman average for 20 innings is 25 runs. His highest score exceeds his lowest score by 64 runs. If
these 2 innings are excluded, the average of the remaining 18 innings is 24 runs. The highest score of the
player is.
A) 66

www.ibpsguide.com | estore.ibpsguide.com | www.sscexamguide.com


161
Complete Quantitative Aptitude Questions

B) 72
C) 77
D) 88

35) The batting average of runs of a cricket player of 30 innings was 96. How many runs must he make in his
next innings so as to increase his average of runs by 12?
A) 468
b) 668
C) 648
D) 486

36) A students maks were wrongly entered as 72 instead of 52. Due to that the average marks for the class
got increased by half. The number of students in the class is.
A) 50
B) 56
C) 46
D) 40

37) The average state of the city in the first 4 days of a month was 116 degrees. The average for the second,
third, fourth and fifth days was 120 degrees. If the state of the first and fifth days were in the ratio 7:8 then
what is the state on the fifth day.
A) 112degrees
B) 128degrees
C) 132degrees
D) 130degrees
.
38) 4 years ago, the average age of x and y was 22 years. With z joining them, the average becomes 26
years. How ols is z now?
A) 62yrs
B) 50yrs
C) 38 yrs
D) 54yrs

39) The average age of Krishnan family of 10 members was 34 yrs, 3 yrs ago. A baby having been born, the
average age of the family is the same today. The present age of the baby.
A) 1yr
B) 3yrs

www.ibpsguide.com | estore.ibpsguide.com | www.sscexamguide.com


162
Complete Quantitative Aptitude Questions

C) 4yrs
D) 2yrs

40) An industry employed 1200 men and 800 women and the average salary was Rs 51 per day. If a woman
got Rs 10 less than a man , then what are their daily salary?(mens and women respectively)
A) men Rs 25 and women Rs 35
B) men Rs 45 and women Rs 55
C) men Rs 35 and women Rs 25
D) men Rs 55 and women Rs 45

31) D)
Total age of 78 students = (780 * 30)yrs
= 2340 yrs
Average of 79 persons = 30 yrs 6 months
= 6 ½ yrs
Total age of 79 persons = 61/2 * 79
= 2409.5
Age of headmaster = (2409.5 – 2340)yrs
= 69.5 yrs
= 69 yrs 5 months.

32) B)
Sum of the remaining 2 numbers = [(12 * 7.9) – (586.8) – (5*7.7)]
= 94.8 – 34 – 38.5
= 22.3
Required average = 22.3 / 2
= 11.15

33) A)
Total quantity of petrol consumed in 3 yr = (12000/21 +12000/24+12000/27)
=12000(1/2 + 1/24 + 1/27)
=12000(72+63+56/1512)
=12000(191/1512)
=(2292000/1512)litres
Total amount spent = rs(3 *12000)
= Rs.36000
Average cost = Rs(36000*1512/2292000)

www.ibpsguide.com | estore.ibpsguide.com | www.sscexamguide.com


163
Complete Quantitative Aptitude Questions

=Rs.23.74

34) A)
Let the highest score be ‘x’
Then lowest score = x-64
Then (25 *20)- 18 * 24 = 432
X+X-64=68
X=66

35) A
Given that the total average runs of a cricket player (30 innings) = 96
After 31 innings = 108
Required number of runs = (108 * 31 – (96 *30)
=3348 – 2880
=468

36) D)
Let there be y students in the class
Total increase in marks = y *1/2
= y/2
Therefore y/2 = (72 – 52)
y/2 = 20
y = 40

37) B)
Sum of states on 1st, 2nd, 3rd, 4th days = (116 *4) = 464 =======>A
Sum of states on 2nd, 3rd, 4th and 5th days = (120 * 4)= 480 ======>b
From a and b
Subtracting each other we get,
State on 5th day – state on 1st day = 16 degrees
Let the state on 1st and 5th days be 7x and 8x degrees respectively
Then 8x – 7x = 16
x = 16
therefore state on the 5th day = 8x = 128 degrees.

38) C)
age of (x+y) 4 yrs ago= (22 * 2)yrs

www.ibpsguide.com | estore.ibpsguide.com | www.sscexamguide.com


164
Complete Quantitative Aptitude Questions

=44
age of (x+y+z)=(26 *3)yrs
=78yrs
Therefore z’s age = (78-44)yrs
=34 yrs.
Current age of z =38 yrs

39) C)
Total age of 10 members , 3 yrs ago = (34*10)yrs = 340 yrs
Total age of 10 members now = (340 +3*10)
=370yrs
Total age of 11 members now = (34*11)yrs
=374yrs
Therefore age of the baby= 374 – 370
=4yrs.

40) D)
Let the daily salary of man be Rs’x’
Then the daily salary of a women = Rs (x-100
Now 1200x + 800(x-10) = 51 *(1200+800)
1200x+800x -8000 = 51*2000
2000x = 102000+8000
=110000
X=55
Therefore mens daily salary Rs 55 and womens daily salary Rs 45

41) A group of students of arithmetic mean of the marks in a test was 63. The brightest 30% of them secured
a mean score of 70 and the dullest 15% a mean score of 41. The mean score of remaining 55 % is
A) 63.675
B) 61.785
C) 65.181
D) 66.67

42) Find the average of first 85 natural numbers?


A) 43
B) 41
C) 45

www.ibpsguide.com | estore.ibpsguide.com | www.sscexamguide.com


165
Complete Quantitative Aptitude Questions

D) 47

43) The average of 7 consecutive odd numbers is 41. Find the largest of these numbers
A) 40
B) 57
C) 47
D) 43

44) The average of 3,8,7 and a is 6 and the average of 19,2,7, a and b is 11. What is the value of b?
A) 11
B) 21
C) 31
D) 41

45) If the average of 4 observations x, x+1, x+2, x+3 is 12 then the average of last 2 observations is
A) 10
B) 11
C) 12
D) 13

46) The average of a positive numbers (non-zero) and its square is 8 times the number. The number is
A) 15
B) 17
C) 13
D) 19

47) A zoo has an average of 1020 visitors on Sundays and 480 on other days. The average number visitors
per day in a month of 30 days beginning with a Sunday is.
A) 5200
B) 570
C) 530
D) 450

48) 11 girls went to a canteen. 10 of them spent 10 each and the 11th girl spent 25 more than the average
expenditure of all. Find the total money spent by them?
A) 172 .5
B) 178 .5

www.ibpsguide.com | estore.ibpsguide.com | www.sscexamguide.com


166
Complete Quantitative Aptitude Questions

C) 137.5
D) 165.5

49) An Aided school has only three classes which contain 70, 36 and 40students respectively. The pass
percentage of these classes are30,25 and 20 respectively. The pass percentage of the aided school is
A) 23 %
B) 15%
C) 26 %
D) 30%

50) The average weight of M,N and O is 168 kg. If P joins the group, the average weight of the group
becomes 160 kg. If another man Q who weighs 6kg more than P replaces M, then the average of N,O,P and
Qbecomes158 kg. What is the weight of M?
A) 175
B) 140
C) 145
D) 150

41) C)
Let the requird mean score be ‘x’
Then (30*70)+(15*41)+55x = 63*100
2100+615+55x=6300
2715+55x=6300
55x=3585
X=65.181

42) A)
Sum of 1st n natural numbers = n(n+1)/2
So, sum of 1st 85 natural numbers = 85(85+1)/2
=85(86)/2
=7310/2
=3655
Required average = 3655/85
=43.

43) C)
Let the numbers be x , x+2,x+4,x+6, x+8, x+10 and x+12

www.ibpsguide.com | estore.ibpsguide.com | www.sscexamguide.com


167
Complete Quantitative Aptitude Questions

Then x+x+2+x+4+x+6+x+8+x+10+x+12/7
7x+42=287
7x=245
X=35
Largest number = x+12 = 35+12
=47

44) B)
We have (3+8+7+a+6/5)=6
24 =a = 30
A=6
Also (19+2+7+a+b/5)=11
19+2+7+6+b=55
34+b=55
b=21

45) D)
We have (x+x+1+x+2x+3/40=12
4x+6=48
4x=42
x=10.5
so the numbers are 10.5,11.5, 12.5, 13.5
required average = 12.5+13.5/2
=26/2
=13.

46) A)
Let the number be ‘x’
rThen x+x^2/2 = 8x
X+x^2=16x
X^2=15x
X^2 – 15x = 0
X(x-15)=0
X=0 or x= 15
So the number is 15 (because given [non-zero] positive).

47) B)

www.ibpsguide.com | estore.ibpsguide.com | www.sscexamguide.com


168
Complete Quantitative Aptitude Questions

Since the month begins with a Sunday


So there will be 5 Sundays I the month
Therefore required average = [(1020*5)+(480*25)/30]
=(5100+12000)/30
=17100/30
= 570

48) C)
Let the average money spent by the 11girls = x
Money spent by the 11th girl = (x + 25)
Money spent by the other 10 girls = (10*10) = 100
Total money spent by the 11 girls = (100 + x + 25) =(x + 125)
x = (x + 125)/ 11 =>10x=125
x =12.5 Total money spent by the 11 girls = 11*12.5 = 137.5.

49) C)
Total number of students passed in class1 = 30/100 *70 = 21
Total number of students passed in class 2= 25/100 *36 = 9
Total number of students passed in class 3= 20/100 *40 = 8
Total number of students passed in the school = 38
Thus, pass percentage of the school = 38 /146*100 = 26 %

50) D)
M+N+O =168 * 3 =504
M+N+O+P=160*4=640……(1)
P=136 kg and Q =142 kg
N+O+P+Q =158*4 =632
N+O+P =632-142 =490…..(2)
from 1 & 2,
M=640-490 =150kg

51) 1/2of a certain travel is covered at the rate of 30km/hr, one-third at the rate of 40 km/hr and the rest at
35km/hr. Find the average speed for the whole travel.
A) 33 1/3
B) 33 3/5
C) 34 3/7
D) None of these

www.ibpsguide.com | estore.ibpsguide.com | www.sscexamguide.com


169
Complete Quantitative Aptitude Questions

52) After replacing an old person by a new person, it was found that the average age of five persons of a
assembly is the same as it was 6 years ago. What is the difference between the ages of the replaced and the
new persons?
A) 24
B) 36
C) 30
D) 15

53) There are 50 compartments in a Chennai express carrying an average of 70 passengers per
compartments. At least 24 passengers were sitting in each compartment, not any compartment has equal
number of passengers, and any compartment does not exceed the number of average passengers expect
50th compartment. Find how many passengers can be accommodated in 50th compartment?
A. 748
B. 705
C.739
D.cannot be determined

54) Students of two colleges appeared for Talent test carrying 250marks as maximum. The average of their
marks for college1 & college 2 are 160 & 180respectively. If the number of students of college 1 is the half of
the number of students of college 2, then what is the average marks of all students of both the college?
A) 170
B) 110
C) 173.33
D) 177.33

55) If the average of two numbers are 138& product is 2241. Then find the difference of both numbers?
A) 225.40
B) 259.25
C) 267.81
D) 294.57

56) In an examination mahi scores 64% of marks, nitesh scores 52% of marks and ritesh scores 48% of
marks. The maximum mark of the exam is a three digit number, whose sum is 10 and the middle digit is equal
to the sum of the other two digits. The number will decreased by 297, if its digit are reversed. approximately
what is the average mark obtained by mahi, nitesh and ritesh?
A) 247

www.ibpsguide.com | estore.ibpsguide.com | www.sscexamguide.com


170
Complete Quantitative Aptitude Questions

B) 248
C) 264
D) 284

57) At the average age of siva and sasi is equal to the average age of somu and ramu. if the ratio of age of
siva and sasi 1:3 and somu age is equal to the three by two of siva age. What is the present age of somu if
ramu age is 25years?
A) 12
B) 15
C) 18
D) 21

58) The average height of 60 girls was calculated to be 300 cm. It was detected later that one value of 330 cm
was wrongly copied as 270 cm for the computation of the mean. Find the correct mean.
A) 301cm
B) 300.5cm
C) 307cm
D) 311cm

59) The mean of 8 article was found to be 15. On rechecking, it was found that two article were wrongly taken
as 11 and 9 instead of 16 and 14 respectively. Find the correct mean.
A) 17.25
B) 13.65
C) 16.54
D) 16.25

60) The average height of 50 students in a class is 182 cm. 40 students whose average height is 182.5 cm
left the class and 50 students whose average height is 180.5 cm joined the class. Find the average height of
the present?
A) 180.41
B) 175.5
C) 180.55
D) 185.5

51) B)
let the total travel be X km.

www.ibpsguide.com | estore.ibpsguide.com | www.sscexamguide.com


171
Complete Quantitative Aptitude Questions

Then X/2 km at the speed of 30 km/hr and X/3 km at 40 km/hr and the rest distance( X - X/2 – X/3) =1/6 X at
the speed of 35km/hr.
Total time taken during the travel of X km
= X/2*30 hrs+ X/3*40hrs+
X/6*35hrs
= 5X/168 hrs
Average speed =X/(5x/168) = 168/5 =33 3/5km hr

52) C)
Let the ages of the five persons at present be a, b, c, d & e years.
And the age of the new persons be f years.
So the new average of five members' age = (a + b + c + d + f)/5 ------- (1)
Their corresponding ages 6 years ago = (a-6), (b-6), (c-6), (d-6) & (e-6) years
So their average age 6 years ago = (a + b + c + d + e - 30)/5 = x ----- (2)
==> a + b + c + d + e = 5x + 30
==> a + b + c + d = 5x + 30 - e ------ (3)
Substituting this value of a + b + c + d = 5x + 30 - e in (1) above,
The new average is: (5x + 30 - e + f)/5
Equating this to the average age of x years, 6yrs, ago as in (2) above,
(5x + 30 - e + f)/5 = x
==> (5x + 30 - e + f) = 5x
Solving e - f = 30 years.
Thus the difference of ages between replaced and new person = 30years.

53) C)
Total number of passengers in Chennai express = 50*70 = 3500
Total number of candidates from 1 to 49 compartments = 24+25+....+70
= (70*71)/2 +(23*24)/2 = 2761
number of passengers in 50th compartment = 3500 - 2761= 739

54) C)
let the number of students of college 2 be '2N'
then the number of students of college1 is 'N'
the average marks for college1 is 160
the average marks for college 2 is 180
total marks of college 1 students = N*160 = 160N
total marks of college 1 students = 2N *180 = 360N

www.ibpsguide.com | estore.ibpsguide.com | www.sscexamguide.com


172
Complete Quantitative Aptitude Questions

average marks of all students of both the colleges =


(160N + 360N)/N+2N = 173.33marks

55) B)
let the two number be a & b
sum of two numbers, a+b = 138*2 =276---1
product of two numbers, a*b=2241
(a+b)2 = a2 + 2ab + b2 ------2
(a-b)2 = a2 -2ab +b2 ------3

solving 2 & 3,
76176 - (a-b)2 = 4*2241
a-b = 259.25

56) A)
Maximum mark consist of a three digit number let’s consider
Unit digit place is Z,ten's place digit is Y and hundred's place digit is X.
According to the question,
Y=x+z---------1
X+Y+Z=10, substitute equation 1
2y=10
Y=5
X+Z=5--------2
Number=100z+10y+x
100z+10y+x=100x+10y+z-297
99x-99z=297
X-Z=3---------3
Solve equation 2 and 3,
X =4,z=1
Original number =451=maximum mark
Total number of Mahi, Nitesh and Ritesh
=164/100×451=739.61
Required average =739.64/3=246.54~247.

57) B)
siva:sasi=x:3x
siva+sasi/2=somu + ramu/2
X+3X=3/2×X+25

www.ibpsguide.com | estore.ibpsguide.com | www.sscexamguide.com


173
Complete Quantitative Aptitude Questions

5X=50
X=10
Siva’s age is 10 then somu's age = 3/2×10=15 years

58) A)
Calculated average height of 60 girls = 300cm.
Incorrect sum of the heights of 60 girls
= (300 × 60)cm
= 18000 cm.
Correct sum of the heights of 60 girls
= (incorrect sum) - (wrongly copied item) + (actual item)
= (18000 - 270 + 330) cm
= 18060cm.
Correct mean = correct sum/number of girls
= (18060/60) cm
= 301 cm.

59) D)
Calculated mean of 8 articles = 15
Incorrect sum of these 8 articles = (15*8) = 120.
Correct sum of these 8 articles
= (incorrect sum) - (sum of incorrect articles) + (sum of actual articles)
= [120 - (20) + (30)]
= 130
Therefore, correct mean = 130/8 = 16.25
Hence, the correct mean is 16.25.

60) A)
The average of the students leaving the class as well as joining the class to be 182 so that the average
remains the same.
But it is given that the average of the 40 students leaving the class is 182.5 (more than 182).
So we will incur a loss of 0.5 cm in the average upon 40 students.
Hence the loss in the sum = 0.5x 40 = 20 cm
Also, since the average of the 50 students joining the class is 180.5 (less than 182) we will incur a loss in this
case as well. The loss in the average is 1.5 cm upon 50 students.
Hence the loss in the sum = 1.5x50 = 75 cm

www.ibpsguide.com | estore.ibpsguide.com | www.sscexamguide.com


174
Complete Quantitative Aptitude Questions

Thus the total loss in the sum = 95 cm. This loss will be shared by 60 students which is the present strength of
the class.
Hence the average of the present class = 182 – 95/60 = 180.5 cm

61) A cricketer played 3 matches in tournament. The respective ratio between the scores of first and second
matches was 3:7 and that between the scores of second and third matches between was 7 :2. the difference
between first and third matches was 84 runs, what was the cricketer average score in all the matches
together?
A) 336
B) 146
C) 168
D) 189

62.The average marks obtained by a student in Tamil, english, maths, science, and social together is 65%
above the average mark obtained in maths and science together. How many more marks exceeded by the
average of maths and science together than the sum of the tamil and english together?
A) 50
B) 65
C) Data insufficient
D) None of these

63) There are five numbers, the second number is 25% more than the first or third number, the 4th number is
5/4 of the third number and the fifth number is 3/2of the 3rd number. What is the average of 5 numbers if the
first number is 10?
A) 12.5
B) 12
C) 13
D) 10

64) The average age of 150 students in a class is 40% of the number of students in the class and the average
age of a group of 50 students present in the class is 32yrs and the average age of another 50 students in the
class is 36yrs. What is the average age of the remaining students in the class?
A) 102
B) 118
C) 112
D) 108

www.ibpsguide.com | estore.ibpsguide.com | www.sscexamguide.com


175
Complete Quantitative Aptitude Questions

65) Average age of A, B, C is 90yrs, however when D joins them, then the average comes down to 80 yrs.
Now a new person E, whose age is six-fifth of the age of D, replaces A and the new average is 78. What is
age of A?
A) 64
B) 68
C) 62
D) 66

66) Mr. Sebastine, a famous author, recently got his new novel released. To his utter dismay, he found that
for the 1974 pages on an average there were 3 mistakes in every page. While, in the first 1024 pages there
were only 2122 mistakes, they seemed to increase for latter pages. Find the average number of mistakes per
page for the remaining pages.
A) 5
B) 2
C) 3
D) 4

67) Gowshik is going to market from his home by bike at a speed of 20kmph. While he comes back to his
home with a speed of X kmph, what should be the value of x so that his average speed is 24kmph ?
A) 24
B) 34
C) 30
D) 32

68) There are some middle level workers in steel factory. The average monthly salary of the 70 middle level
workers is Rs. 3250. and that of higher level workers is Rs. 3750., if the average monthly salary of higher and
middle level workers is 3400. Find the total number of workers in the factory, if the number of middle level and
higher level workers in factory from 50% of the total number of workers?
a) 200
b) 150
c) 250
d) 100

69) The average weather(rainfall) for the first 6 days out of 8 days recorded to be 9 cm. The rainfall on last 2
days was in the ratio 2:3. the average of 8 days was 14.2 cm. What was the rainfall on the last day?
A) 7.8
B) 34.4

www.ibpsguide.com | estore.ibpsguide.com | www.sscexamguide.com


176
Complete Quantitative Aptitude Questions

C) 35.76
D) 8.9

70) A cricket player had a certain average of runs for his 43 innings. In his 44th innings, he is bowled out for
no score on his part. This brings down his average by 4runs. His new average of runs is ?
A) 172
B) 182
C) 166
D) 162

61) A)
The ratio between first and second matches equal to 3 :7
The ratio between second and third matches=7:2
The difference between first and third matches=3x-2x=84 runs
= 84
Required average=1008/3=336 .

62) C)
According to the question
There is no other information about the marks in any one of the subject.
The data gives is insufficient to answer the question.

63) B)
Let us consider 5 numbers are A,B,C,D and E
B=125/100×A
B=125/100×C
125/100×A=125/100×C
A/C=1/1
D=5/4×c
E=3/2*C
First number is 10,
A=10=x
B=25% ×10+10=12.5
C=10
D=5/4×10=12.5
E=3/2×C=15
Required average=10+12.5+10+12.5+15/5=12

www.ibpsguide.com | estore.ibpsguide.com | www.sscexamguide.com


177
Complete Quantitative Aptitude Questions

64) C)
150 students average 150×40/100=60 years
According to the question,
150×60=50×32+50×36+50×X
50×X=9000-1600-1800
50x=5600
X=112

65) B)
according to the question
A+B+C/3=90
=> A+B+C =270…………..(1)
A+B+C +D/4=80
=> A+B+C +D=320………(2)
From the equation 1 and 2
D=50
E =6/5×50=60
E+B+C +D/4=78
=> E+B+C +D =312
B+C +D =312-60=252……………..(3)
From 2and 3
The age of A=320-252=68

66) D
According to the question
X is the remaining pages average.
2122+(1974-1024)×x/1974=3
950x=5922-2122=3800
X=3800/950=4

67) C)
Let speed be X
Average speed =2xy/(x+y)
=>2×X×20/(x+20)=24
40x=24x+480
16x=480

www.ibpsguide.com | estore.ibpsguide.com | www.sscexamguide.com


178
Complete Quantitative Aptitude Questions

X=30 kmph.

68) A)
(70×3250+x×3750)/(70+x)=3400
227500+3750x=238000+3400x
350x=10500
x=10500/350
X=30
Total workers=100/50×(70+30)
=200workers are there in factory

69) C)
according to question
(6×9+3x+2x)/8=14.2
54+5x=113.6
X=11.92
Last day rainfall=3×11.92=35.76cm

70) A)
Let the cricket player’s average of runs for his 43 innings be X runs.
Total number of runs in 43innings=43x
According to the question,
[43x + 0]/ 44= x-4
43x = 44x – 176
x = 176
new average of runs = 176-4
=>172

71) The average age of Mr and Mrs Rahim at the time of their marriage in 1981 was 56 yrs. On the occasion
of their anniversary in 1986, they observed that the average age of their family had come down by 15 yrs
compared to their age at the time of their marriage. This was due to the fact that their daughter varshini was
born. What was the age of varshini in 1990?
A) 6
B) 5
C) 4
D) 1

www.ibpsguide.com | estore.ibpsguide.com | www.sscexamguide.com


179
Complete Quantitative Aptitude Questions

72) The Cricketer average score in 40 over is 21, if the man scores 23runs in 8over, 26 runs in 10 over and
18runs in 14 over. What is the average score in remaining over?
A) 18
B) 19
C) 16
D) 20

73) The average age of a morning class is 108, if the average age of 72 ladies in the class is as same as the
total average and the number of gents in the class is 2 more than the ladies in the class, what is the average
age of gents in the class?
A) 152
B) 108
C) 156
D) 154

74) The average marks of a class of 90 students is 126. Out Of them, 4 scores zero, first 60 students scored
an average of 116, next 24 scored an average of 118. What is the mark obtained by the remaining student in
the class?
A) 750
B) 862
C) 774
D) 875

75) The average salary per head of all the employees in a company is Rs. 180. The average salary of 24
employees is Rs. 1040and the average salary per head of the rest is Rs. 160. Find the total number of
employees in the workshop.
A) 2841
B) 1056
C) 1195
D) 2145

76) The average score in a bank examination of 13 students of a class is 50. If the scores of the top five
students are not considered, the average score of the remaining students falls by 5. The pass mark was 35
and the maximum mark was 100. It is also known that none of the students failed. If each of the top five
scorers had distinct integral scores, the maximum possible score of the topper is.
A) 85
B) 90

www.ibpsguide.com | estore.ibpsguide.com | www.sscexamguide.com


180
Complete Quantitative Aptitude Questions

C) 95
D) 100

77) The average age of three -seventh of class is 49, what should be the average of remaining four-seventh
students so that the average of the entire class is 63?
A) 24.5
B) 86.5
C) 73.5
D) 25.5

78) Ronit spends his money from his saving in different way that is he spends 25% on travel,15% on
food,20% on wages and 17% on shopping a clothes and after that all expenditure he saved 6900.Find the
how much he spent on clothes.
A) 4400
B) 5100
C) 4000
D) 6000

79.The average expenditure of A, Band C is Rs 10000 per month. Also, the average expenditure of B, C and
D is Rs 14000 per month. If the average expenditure of D is thrice of that of A then the average expenditure of
B and C is:
A) 12000
B) 15000
C) 18000
D) 21000

80) The average monthly income of M and N is Rs. 12100. The average monthly income of N and O is Rs.
10500 and the average monthly income of O and M is Rs. 14400. What is the monthly income of N?
A) 4200
B) 8200
C) 5600
D) 7600

71) B)
Sum of the ages of Mr and Mrs Rahim=56×2=112years
Sum of the ages in 1986=41×3=123 years
Sum of the ages of Mr and Mrs Rahim =112+10=122

www.ibpsguide.com | estore.ibpsguide.com | www.sscexamguide.com


181
Complete Quantitative Aptitude Questions

Daughters age in 1986=123-122=1 years


Daughters age in 1990 =1+4=5 years.

72) A)
According to the question
40*21=23*8+26*10+18*14+8×X
840=184+260+252+8x
8x=144
X=144/8=18 runs

73) B)
According to question,
(72+74)X=72×108+74X
146x-74x=7776
72x=7776
X=7776/72= 108years

74) C)
According to the question,
90*126=(4*0)+(60*116)+(24*118)+2y
11340=6960+2832+2y
2y=1548
y=774
75) B)
Average = Sum of observations/Number of observations
Let the number of employees be x.
Given, the average salary per head of all the employees in a company is Rs. 180
So, the sum of salary of all the employees in the company = Rs.180x
Given, the average salary of 24 employees is Rs. 1040
So, the total salary of 24employees = 1040 × 24 = Rs. 24960
Given, the average salary per head of the result is Rs. 160.
Number of remaining employees = x – 24
Then, total salary of the remaining employees = 160× (x – 24)
∴ 24960 + 160 × (x – 24) = 180x
⇒ 24960+ 160x - 3840 = 180x
⇒ 20x = 2112
⇒ x = 1056

www.ibpsguide.com | estore.ibpsguide.com | www.sscexamguide.com


182
Complete Quantitative Aptitude Questions

76) D
Average = Sum of observations/Number of observations
Given, average score in a bank examination of 13 students of a class is 50.
Sum of total scores = 13 × 50= 650
Given, if the scores of the top five students are not considered, the average score of the remaining students
falls by 5.
Sum of scores of remaining 8 students = 8× 45= 360
Sum of scores of the top 5 students = 650-360= 290
Let the scores of the top 5 students be a, b, c, d and e.
Let ‘e’ be the maximum possible score of the topper.
Also, each of the top five scorers had distinct integral scores.
Thus, for ‘e’ to be the maximum possible score, the collective score of (a + b + c + d. should be least possible.
Since average given was 45, so minimum score for highest scorers will be 46 atleast.
Thus,
a = 46, b = 47, c = 48, d = 49
⇒a + b + c + d + e = 290
⇒46+ 47 + 48+ 49 +e = 290
⇒e = 100

77) C)
According to the question,
63=3/7×49+4/7×y
63=147/7+4y/7
441=147+4y
4y=294
Y=73.5

78) B)
Let the total income of Ronit x then total expenditure from income
X x (25%+15%+20%+17%)=X*77%
X x 77% =Total savings= X x 23%
X = 6900 x 100 / 23 = 30000
expenditure on clothes = 17% so,30000 x 17 / 100 = 5100

79) A)
Total expenditure of A, B & C=3×10000=30000

www.ibpsguide.com | estore.ibpsguide.com | www.sscexamguide.com


183
Complete Quantitative Aptitude Questions

Total expenditure of B, C & D =3×14000=42000


D-A=42000-30000=12000
Also, we are given D=3A. so 3A-A =12000 & A=6000
Total expenditure of B & C=30000-6000=24000
Average expenditure of B& C=24000/2 =12000

80) B)
The average monthly income of M and N is Rs. 12100.
The average monthly income of N and O is Rs. 10500 and
the average monthly income of O and M is Rs. 14400.
M+ N = 2 × 12100 = 24200 …. (Equation 1)
N+ O = 2 × 10500 = 21000 …. (Equation 2)
O + M = 2 × 14400 = 28800 …. (Equation 3)
by solving,
(Equation 1) + (Equation 2) – (Equation 3)
=> M+N+N+O – (O+M) = 24200+21000-28800
=> 2xN = 16400
then N= 8200
N’s monthly income = Rs. 8200

81) The batting average for 20 innings of a cricketer is 25 runs. His highest score exceeds his lowest score
by 86 runs. If these two innings are excluded, the average of the remaining 18 innings is 22runs. Find out the
highest score of the player.
A) 85
B) 90
C) 95
D) 100

82) The average age of 14 mens is increased by 1 year when one of them whose age is 44 years is replaced
by a lady. What is the age of the lady?
A) 54
B) 58
C) 50
D) 56

www.ibpsguide.com | estore.ibpsguide.com | www.sscexamguide.com


184
Complete Quantitative Aptitude Questions

83) on analyzing the result of an competitive exam the teacher found that the average for the entire the class
was 69 marks. If we say that average of 10 % of the students scored 77 marks and average of 28 % of the
students scored 66 marks, then calculate average marks of the remaining students of the class
A) 67.54
B) 68.26
C) 66.91
D) 69.06

84) The average weight of a group of 22students is 34kg. When the weight of the staff is also included, the
average weight increases by 2kg. What is the weight of the staff?
A. 80 kgs
B. 54 kgs
C. 47 kgs
D. 33kgs

85) The average of marks obtained by 60 students in a computer examination is 18. If the average marks of
passed students is 20and that of the failed students is 8, what is the number of students who passed the
examination?
A) 100
B) 75
C) 50
D) 85

86) The average age of 160 boys in a class is 58 yrs. The average group of 30 boys in the class is 42 yrs and
the average of another group of 50 boys in the class is 36 years. What is the average age of the remaining
boys?
A) 72.58
B) 74.25
C) 77.75
D) 75.68

87) When the average age of a father, mother and their son was 90 years, the son got married and a child
was born just 6 year after the marriage when child turned 14 years the average age of the family is 80yrs.
Find the age of daughter - in - law at present?
A) 55
B) 56
C) 57

www.ibpsguide.com | estore.ibpsguide.com | www.sscexamguide.com


185
Complete Quantitative Aptitude Questions

D) 58

88) The average age of Anu, banu and tonu is 74 yrs. 10years hence the average age of anu and tonu is 86
yrs.6yrs ago the average age of aarthi and banu was 72 yrs. Find the present age of aarthi.
A) 80
B) 86
C) 84
D) 83

89) The average monthly expenditure of Mr. Abi family for the first 4 months is Rs.4210, next 4 month
expenditure Rs.4450 for the last 4 months Rs. 4360 . If his family saves Rs. 6800 for 12 months, find the
average monthly income of the family for the 12 months?
A) Rs. 4707.25
B) Rs.4564.75
C) Rs. 4906.66
D) Rs. 4806.50

90) The average of 35 results is 28. The average of first 17 of them is 24 and that of last 17 is 21. Find the
18th result?
A) 125
B) 215
C) 512
D) 521

81) C)
Total runs scored by the player in 20 innings = 20*25
Total runs scored by the player in 18 innings after excluding two innings = 18 × 22
Sum of the scores of the excluded innings = 20 × 25 – 18× 22 = 104
Given that the scores of the excluded innings differ by 86. Hence let’s take
the highest score as x + 86 and lowest score as x
Now x + 86 + x = 104
=> 2x = 18
then x = 9
Highest score = 9+ 86 = 95

82) B)
suppose the average age of 14 men is x years and the age of lady is y years.

www.ibpsguide.com | estore.ibpsguide.com | www.sscexamguide.com


186
Complete Quantitative Aptitude Questions

the total age of 14 mens will be = 14x years


the average age of 14 mens is increased by 1 years who of them whose age is 44 years is replaced by a lady
So, the new average is = x +1 years
total age of them will be = [14* (x +1)]years
14x - 44 + y = 14* (x+1)
14x - 44 + y = 14x + 14
y = 14+ 44
y=58

83) D)
average of entire class = 69marks
average of 28 % of the students = 66 marks
average of 10% of the students = 77 marks
then % of remaining students = (100- 10 - 28) = 62%
let the average of 62% of the students be x
(62*x)+(10*77)+(28*66) = 100 *69
(62*x)+770+1848= 6900
62 * x = 6900-770-1848
62*x = 4282
x = 69.06

84) A)
The average weight of a group of 22students = 34 kgs.
Therefore, the total weight of the group = 22*34 = 748 kg
When the weight of the staff is included, there are 23 individuals.
The average weight increases by 2kg. That is the new average weight = 36 kgs.
Therefore, the total weight of the 22 students plus the staff = 23* 36= 828
828-748=80 kg.

85) C
Let the number of passed students be x.
Then total marks = 60 × 18= 20x + (60– x) × 8
1080= 20x + 480– 8x
12x = 600
∴ x = 50
∴ number of passed students = 50

www.ibpsguide.com | estore.ibpsguide.com | www.sscexamguide.com


187
Complete Quantitative Aptitude Questions

86) C)
Total age of 160 boys = 160* 58= 9280
total age of 30 boys = 30 * 42= 1260
total age of next 50boys = 50 * 36= 1800
average of the remaining boys =
[(9280-{1260+1800})/[160 - (30 + 50)]
=>9280-3060/80
=>6220/80
=77.75yrs

87) B)
total age of father, mother and son at the time of son's marriage = 90*3 =270
present age of family father, mother, son, daughter-in-law, child = (father, mother, son age at the time of
marriage) + daughter-in-law present age + child present age
= (270+60)+ daughter-in-law present age + 14 = 80*5 =400
daughter-in-law present age= 400-344 = 56yrs

88) B)
anu+ banu+ tonu = 74 * 3 = 222yrs
10yrs hence,
anu + 10 + tonu +10 = 86 * 2
anu + tonu= 152
banu = 222-152 = 70 yrs
aarthi -6 + banu - 6= 72 * 2
aarthi + banu = 144 + 12 = 156
aarthi age = 156-70
aarthi age = 86yrs

89) C)
Mr Abi family first 4 month expenditure=4210*4=16840
Mr Abi family next 4 month expenditure=4450*4=Rs17800
Mr Abi family last 4 month expenditure=4360*4=Rs.17440
Mr Abi family total expenditure in 12 months=16840+17800+17440+6800=Rs.58880.
Mr Abi family average expenditure in 12 months =58880/12=Rs.4906.66

90) B)
Clearly 18th result = (sum of 35 results)- (sum of 34 results)

www.ibpsguide.com | estore.ibpsguide.com | www.sscexamguide.com


188
Complete Quantitative Aptitude Questions

=(35 * 28) – { (17 *24) +(17 *21)}


= 980 – (408 + 357
=980 – 765
=215

91) Distance between two stations P & Q is 1556km. A passenger train covers the journey from P to Q at
168km per hr and return back to P with a uniform speed of 112km/hr. Find the Average speed of the train
during the whole journey?
A) 124.4 km/hr
B) 130.4km/hr
C) 134.4 km/hr
D) 130.0 km/hr

92) If the mean of P, Q,R is A and PQ + QR + RP =0, then the mean of p^2,Q^2,R^2 is.
A) 2A^2
B) 4A^2
C) A^2
D) 3A^2

93)The average of the 3 digit number, which remain the same when the digits interchange their positions is.
A) 444
B) 555
C) 666
D) 777

94) The average age of the men in a team is 76 years and that of the women is 75 years. The average age
for the whole team is.
A) 75 yrs
B) 75.5 yrs
C) Cannot be computed with the given information
D) None of the above

95) The average monthly income of certain fashion designers is F and that other workers is N. The number of
fashion designer workers is 33 times that of other workers. The average monthly income (in rs) of all the
worker is.
A) F+N/2
B) F+33N/12

www.ibpsguide.com | estore.ibpsguide.com | www.sscexamguide.com


189
Complete Quantitative Aptitude Questions

C) 33F+N/22
D) (33F+N)/34

96) Of the 5 numbers, the first is twice the second, the second is one – third of the third, the third is 5 times of
the fourth, and the fourth is three – seventh of fifth. The average of the numbers is 35. The largest of these
number is.
A) 30.63
B) 65.625
C) 43.75
D) 75.356

97) If the arithmetic mean of 150 numbers is calculated 70. If each number is increased by 10, then mean of
new number is.
A) 70
B) 80
C) 90
D) 60

98) A group of 4 persons joins in javelin throw competition. The best player scored 42.5 points. If he had
scored 46 points, the average score for the team would have been 42. The number of points the team scored.
A) 164.5
B) 166.5
C) 169.7
D) 162.5

99) 20 years ago, the average age of a family of 4 member was 48 years. 2 children having been born (with
the age difference of4 yrs) the present average age of the family is the same. The present age of the
youngest child is.
A) 2yrs
B) 4 yrs
C) 6yrs
D) 8yrs

100) The average age of students of a college is 31.6 yrs. The average age of boys in the class is 32.8 yrs
and that of girls is 30.8. The ratio of number of boys to the number of girls in the class.
A) 2:3
B) 3:1

www.ibpsguide.com | estore.ibpsguide.com | www.sscexamguide.com


190
Complete Quantitative Aptitude Questions

C) 1:3
D) 3:2

91) C)
Given x= 168 , y = 112
Required average speed = (2xy / x + y) km/hr
= 2 * 168 *112 / 168 + 112
= 37632 / 280
=134.4 km/hr

92) D)
We have (P + Q + R)/3 = A
P+Q+R = 3A
(P+Q+R)^2 = 9A^2
P^2+Q^2+R^2 + 2 (PQ + QR + PR) = 9A^2
P^2+Q^2+R^2 = 9A^2
Required mean = (P^2+Q^2+R^2)/3 = 9A^2/3= 3A^2

93) B)
Average = (111+222+333+444+555+666+777+888+999)/9
=(111+999)+(222+888)+(333+777)+(444+666)+555/9
=(4*1110)+555/9
=4440 + 555/9
=4995/9
=555.

94) C)
Clearly to find the average, we ought to number of men, women or total persons in the class, neither of which
has been given.
So the data provided is inadequate.

95) D)
Let the number of other workers be ‘x’.
The number of fashion design workers = 33x
Total number of workers = 34x
Average monthly income = (F * 33x)+(N*x)/34x
=x(33F + N)/34x

www.ibpsguide.com | estore.ibpsguide.com | www.sscexamguide.com


191
Complete Quantitative Aptitude Questions

=(33F + N)/34.

96) B)
Let the fifth number be ‘x’.
4th number = 3/7 x
3rd number = 5 (3/7 x) = 15/7x
2nd number = 1/3 (15/7 x) = 15 /21 x
1st number = 2 (15/21 x) = 30/21 x
X + 3/7 x + 15 /7 x + 15/21 x + 30/21 x = 35 * 5
120x = 3675
X = 30.625
So the numbers are 30.62, 13.125, 65.625, 21.875, 43.75.
Therefore largest number is 65.625.

97) B)
Arithmetic mean of numbers = 70
Sum of 150 numbers = (70 * 150)
=10500
Total increase = (150 * 10) = 1500
Increased sum = 10500 + 1500 = 12000
Increased average = 12000/150
=80

98) A)
Let the total score be ‘x’.
(X+46 – 42.5)/4 = 42
X + 3.5 = 42 * 4
X + 3.5 = 168
X = 168 – 3.5
X = 164.5

99) C)
Total age of 4 members, 20 yrs ago = (48 * 4)=192 yrs
Total age of 4 members now = 192 + 20 * 4 = 272 yrs
Total age of 6 members now = (48*6)=288yrs
Sum of the age of children = 288 – 272 = 16 yrs
Let the age of the younger child be x.

www.ibpsguide.com | estore.ibpsguide.com | www.sscexamguide.com


192
Complete Quantitative Aptitude Questions

Then the age of the elder child 4 +x


So x + 4 +x = 16
2x=12
X=6
Age of younger child = 6years

100) A)
Let the ratio be D:1.then,
(D * 32.8) + ( 1* 30.8)= (D+1) * 31.6
32.8 D +30.8 = 31.6 D + 31.6
32.8 D – 31.6D = 31.6 – 30.8
1.2D = 0.8
D = 0.8/1/2 = 0.4/0.6
D = 2/3
Required ratio = 2/3 :1 = 2:3.

9. PROFIT AND LOSS


1) Tata Docomo charged Rs 540 for 1gb(1day)and aircel charged 90% of RS 540 for the same 1gb for same
days. Gowri uses docomo for 1st 15days and remaining 15days uses aircel. If she uses aircel for whole
30days, how much amount she save?

2) A vendor bought 35kg of wheat at the rate of Rs 50per kg. He sold 45%of the total quantity at the rate of
Rs55 per kg. Approximately, at what price per kg should he sell the remaining quantity to make 40%overall
profit?

3) If 20% discount is allowed on the marked price then the profit is 40%. If the discount is increased to 40%
then what will be the profit percentage?

4) After selling a fan at 6% gain and a fridge at 9% gain, a shopkeeper gains Rs 5100. But if he sells the fan
at 9% gain and the fridge at 6% loss, he gains Rs 1800 on the whole transaction. Find the original price of the
fan.
5) A shopkeeper announced 35% discount on an item. A customer bought the item from the shop for Rs.
21000 after getting discount. That person sells the item to another person in such a way that he earned a
profit of 32% on the original price. What is the selling price for the another person?

6) A seller sold a cloth for Rs. 720 after giving 20% discount on the labelled price. Had he not given the
discount, he would have earned a profit of 40% on the CP. what was the Cp of the cloth?

www.ibpsguide.com | estore.ibpsguide.com | www.sscexamguide.com


193
Complete Quantitative Aptitude Questions

7) A salesperson has goods of worth Rs.12000. He sold half of the goods at a gain of 24%. At what profit per
cent should he sell the remaining half of the stock so that he gets36% profit on the whole?
8) Kannan sells laddu at Rs.30 per kg. A laddu is made up of flour and sugar in the ratio of 5:3. The ratio of
price of sugar and flour is 7:3 (per kg). Thus, he earns 67% profit. What is the cost price of sugar?

9) Two items X and Y are sold at a profit of 30% and 45% respectively. If the amount of profit received is the
same, then the cost price X and Y may be in ratio?

10) Sai purchased a DVD player at an additional 20% discount on the reduced price after deducting 40% on
the labelled price. If the labelled price of the DVD player was Rs. 5600, then at what price did he purchase the
DVD player?

1) for 1st 15days= 15*540=8100rs


for 2nd 15days=90/100*540*15=7290
for 30days=8100+7290=15390
if she uses aircel for 30days=30*540*90/100=14580.
Amount save=15390-14580=Rs.810

2) CP of wheat=35*50=Rs.1750
45 of 35kg=45/100*35=15.75kg
SP of 15.75kg=15.75of55=Rs.866.25
For 40%profit, total Sp of all the wheat is 1750*140/100=Rs.2450
Remaining sugar(35-15.75)=19.25kg
Rate of remaining sugar per kg=2450-866.25/19.25
= Rs.82.27

3) cp=100% profit=40% selling price= 140% marked price = 140%


mp=140*100/80= 175
40%discount in mp= 175 *60/100
New sp=105
new profit%= new sp-cp/cp *100
=105-100/100*100
=5

4) let the price of fan be X and fridge be Y


6x/100+9y/100 = 5100

www.ibpsguide.com | estore.ibpsguide.com | www.sscexamguide.com


194
Complete Quantitative Aptitude Questions

6x+9y=510000------(1)
then 9x/100-6y/100=1800
9x-6y=180000-------(2)
Calculate both, the we will get
x=Rs40000

5) Let original price=x


X*65/100=21000
X=21000*100/65=32307.6Rs.
SP=32307.6*132/100=Rs.42646.15

6) Let CP= x
Sp of each cloth =80/100*x =720
x= 720*100/80 =900
CP= 900* 100/140
=Rs 643.

7) CP of goods = 12000
For 36% gain, total SP =12000 + (36/100*12000) = 16320
SP of goods worth 6000 at 24% profit
= 6000 + (24/100*6000) = 7440
Therefore, SP of remaining goods = (16320-7440) = 8880
Let the gain % for remaining goods be x. Then,
=>6000*(100+x)/100=8880
=>100+x=8880/60
x=148-100=48

8) Since, the profit is 67, so the selling price will be 167% of the cost price
∴Ratio of selling price to cost price will be 5:3
∴Cost price of laddu =30*3/5 = 18
Let cost of flour and sugar is 3s and 7s respectively
Since laddu is made up of flour and sugar in the ratio of 5:3 = (5*3s)+(3*7s)/8=18
15s+21s=144
36s=144
s=4
∴Price of sugar = 7s = 7 × 4
= Rs. 28

www.ibpsguide.com | estore.ibpsguide.com | www.sscexamguide.com


195
Complete Quantitative Aptitude Questions

9) Let same profit be Rs 45.


30% = Rs 45 for item X ⇒ CP of item X= 150 Rs
45% = Rs 45 for item Y ⇒ CP of item Y = 100 Rs
Ratio = 150:100 = 3:2

10) Let the labelled price be Rs. 100


Reduced price = (100 – 40)% of 100 = Rs. 60
20% additional discount = 20% of 60 = Rs. 12
Net CP = 60 – 12 = Rs. 48
Therefore, sai’s cost price = 5600/100 *48 = Rs. 2688

11) By selling a mobile at 52% discount, a seller incurs a loss of 8%. The marked price of the product is
18000 Rs. At what percent discount the seller should sell the mobile so as to have a profit of 33%.

12) In a certain mall, the profit is 160% of the cost. If the cost increases by 12.5% but the selling price remains
constant, approximately what percentage of the selling price is the profit?

13) On selling 9 pens at Rs. 360, there is a loss equal to the cost price of 3 pens. The cost price of a pen is:

14) Sekar gives the discount of 42% for nokia phone, after he marked the price 29% more than its cost price.
find his loss or profit %?

15) A seller mixes 52 kg of ragi at Rs. 40 per kg with 60 kg of ragi of other variety at Rs.72per kg and sells the
mixture at Rs.60 per kg. His profit percent is:

16) A decrease of 40 % in the price of coffee enables a person to buy 8 kg more for Rs 400 find the original
and reduced price per kg of coffee.

17) Varsha purchased a speaker of Rs 10,800 and a phone Rs 19,200. she sold speaker at four-fifth of its
cost price and the phone at 5/4 of its cost price. What was the profit?

18) Deepak selling a product at 48% discount, he incurs a loss of 10%. The marked price of the product is
15000 Rs. At what percent discount he should sell the product so as to have a profit of 30%.

19) Dhuviksha sells his two camera one at 45% loss and another at 30% profit. If the cost prices of the two
cameras are in the ratio of 1:2, what is his percent profit or loss?

www.ibpsguide.com | estore.ibpsguide.com | www.sscexamguide.com


196
Complete Quantitative Aptitude Questions

20) By selling 90 oranges for Rs 80, a man loses 40 %. How many oranges should he sell approximately for
Rs 48 to gain 40 % in the transaction?

11) Since, Marked Price = 18000


Therefore, Selling Price = 18000*0.48 = 8640
Loss of 8% is there
Therefore, Cost price = 8640/0.92 = 9391.30Rs
To have profit of 33%
The Selling Price should be = 9391.30*1.33 = 12490.42
Hence, Discount should be 18000 – 12490.42 = 5509.58Rs
Discount% should be [5509.58/18000]*100 = 30.60 ≈ 31%.

12) Let C.P.= Rs. 100.


Then, Profit = Rs. 160, S.P. = Rs. 260.
New C.P. = 112.5% of Rs. 100 = Rs. 112.5
New S.P. = Rs. 260.
Profit = Rs. (260– 112.5) = Rs. 147.5
Required percentage = 147.5/260*100=57% (approx.)

13) CP of 9 pens - Sp of 9 pens = Cp of 3 pens


Cp of 6pens = Sp of 9 pens = 360
Cp of 6pens =360
CP of 1 pen= 360/6= 60.

14) Cp= 100


MP=129/100*100=129
Sp=129*42/100 =129-54.18 =74.82
Loss% = (100-74.82)
= 25.18%

15) C.P. of 112 kg rice = Rs. (52x 40 + 60 x 72)


= Rs. (2080+4320) = Rs. 6400.
S.P. of 112 kg rice = Rs. (112 x 60) = Rs. 6720
Gain=320
Gain %= 320*100/6400
Gain %=5

www.ibpsguide.com | estore.ibpsguide.com | www.sscexamguide.com


197
Complete Quantitative Aptitude Questions

16) Price of 8kg of coffee= 40% of 400


Reduced price of 8kg=160Rs
Reduced price of 1kg= 20Rs
Original price= 20*100/60 = Rs 33.33

17) CP of speaker = Rs 10800


SP = 10800× 4/5 = Rs 8640
CP of the phone = Rs 19200
SP = 19200× 5/4= Rs 24000
Total CP = 10800+19200= Rs 30000
Total SP = 8640+24000= Rs 32640
Profit = 32640– 30000 = Rs 2640

18) Mp=15000==>Sp= 15000*0.52= 7800.


Loss of 10% is there==>Cp= 7800/ 0.90= RS.8666.66
Profit of 30% = 6000*1.15=11266.66
Discount should be Rs2600
Discount % should be {2600/15000}*100
=17.3%

19) Given that CPs are in the ratio 1:2


Therefore let the CPs be Rs.100 &Rs. 200 respectively,
1st SP = 100-45% of 100 = Rs. 55.
2nd SP = 200 + 30% of 200 = Rs. 260.
Total CP = Rs.300.
Total SP = 55 + 260 = Rs.315.
Profit = Rs.315-300 = Rs.15.
Profit percent = 15*100/300 = 5% profit.

20) Let S.P. of 90 oranges be Rs. x.


Then, 60 : 80 = 140 : x
x = 80*140/60= 186.66
For Rs.186.66, oranges sold = 90
For Rs.48, oranges sold = 90/186.66* 48 = 23(approx.)

www.ibpsguide.com | estore.ibpsguide.com | www.sscexamguide.com


198
Complete Quantitative Aptitude Questions

21) If 7/8 part of a books is sold at 60% profit, 1/12 part at 32% profit and remaining part at 24% profit and
finally, there is a profit of Rs.150, then find the cost price of the books approximately

22) Gokul purchases 10 goats at Rs. 1500 each. 1 goat died. He sold 2 goats at 5% loss, at what rate he
should sale the remaining goat, so as to gain a Profit of 10 % on the total Cost?

23) Geetha groups sell their two hotels which are made up of teak woods. One at 25% loss and other at 20%
profit. If the cost prices of the two hotels are in the ratio of 3:4, what is his % profit or loss?

24) Jai earns 20% on investment but loses 15% on another investment .If the ratio of the two investments be
3:5, what is the gain or loss on the two investments taken together?

25) Reeta buys an old cycle for Rs.2500 and spends Rs.300 on its repairs. If he sells the cycle for Rs.3200,
his gain percent is?

26) Kathir bought some bananas at Rs.20 per dozen and bought the same number of bananas at Rs.16 per
dozen. He sold these bananas at Rs.22 per dozen and gained Rs. 240. The total number of dozen bananas
bought by him was:

27) P sells a bike to Q at a profit of 25%. Q sells it to R at a profit of 30%.If R pays Rs.273 for it, the cost price
of the bike for P is:

28) Profit after selling a product for Rs. 850 is the same as the loss after selling it for Rs.710. What is the cost
of the product?

29) Nivetha buys a tape recorder for Rs.450. His overhead expenses are Rs.30
she sells the tape recorder for Rs.600. The profit percent of nivetha is

30) By selling a pen for Rs.15, a man loses one sixteenth of what it costs him. The cost of the pen is?

21) a = 7/8 , x = 60 % ,
b =1/12 , y = 32 % ,
z = 24 % and R = Rs150
Required CP of books = (150*100)/(7/8*60+1/12*32+1/24*24)
=15000/(52.5+2.66+1)
= 15000/56.16
= Rs267

www.ibpsguide.com | estore.ibpsguide.com | www.sscexamguide.com


199
Complete Quantitative Aptitude Questions

22) Selling Price with Profit of 10 % of total cost = 1500*10*110/100


= 16500
Selling Price of 2 goat with 5% loss= 3000*95/100
= 2850
Difference = 16500-2850= 13650
So rate of the goats for selling to gain 10% profit on total = 13650/7
= Rs. 1950

23) CP are in the ratio 3:4 which is Rs.300& Rs.400


1st SP= 300-25% of 300 = Rs.225
2nd Sp=400 +20% of 400 = Rs.480
Total Cp= Rs 700
Total SP= 225+480 = Rs 705
Profit = Rs. 705 - 700 = Rs .5
then % profit= (5*100)/700 = 5/7%profit.

24) Suppose he invested 300 & 500 respectively


profit : 20% of 300 = 60
loss = 500 *15/100 = 75
net loss = - 15
=>15/800 *100 = 1.8%.

25) Profit Percentage = Profit / cost price x 100


Cost price = 2500+300 = 2800
Profit = sp-cp
=3200-2800=> 400
Profit percentage = 400 / 2800 x 100
= 14.28%

26) Cost price :


1 dozen bananas = Rs. 20
1 dozen bananas = Rs. 16
2 dozen bananas= Rs. 36
1 dozen bananas = Rs. 18
Selling price:
1 dozen bananas = Rs.22

www.ibpsguide.com | estore.ibpsguide.com | www.sscexamguide.com


200
Complete Quantitative Aptitude Questions

Profit per dozen is Rs. 4


Therefore x* profit = Rs. 240
x = 240/4
X = 60 dozen

27) Selling price for q is Rs. 273


Cost price + profit = 273
X + 30x/100= 273
X = 210
Cost price of q is 210
Selling price of p is 210
Cost price + profit = 210
Y + 25y/100= 210
Y = 168
The cost price of p is Rs. 168/-

28) Assume cost Price = x


P = 850 –x
P  850 – x = 710 +x
850 – 710 = 2x
140 = 2x
X = 70
P = 850– 70
= 780

29) Cost price + profit = selling price


Cost price = 450 + 30= 480
Profit=sp-cp
=600-480
=120
Profit%= 120/480*100
X = 12000/480
X = 25 %.

30) Let us assume cost price is x


Ie., x – 15 =1/ 16x
X – 15-x/16

www.ibpsguide.com | estore.ibpsguide.com | www.sscexamguide.com


201
Complete Quantitative Aptitude Questions

X - x/16 = 15
16x - x/16 = 15
15x = 15/16
X = 15x16/15
X = 16.

31) By selling a marker for Rs.18, a boy loses one nineteenth of what it costs
him. The cost of the marker is?

32) Selva purchased 60 chairs at a price of Rs. 55 per chair. He sold 15 chairs at a profit of Rs. 6 per chair
and 37 chair at a profit of Rs. 7 per chair. The remaining chairs were sold at a loss of RS.3 per chair. What is
the average profit per chair?

33) Reenu sold an article at a loss of 40%. If the selling price had been increased by Rs.200, there would
have been a gain of 10%. The cost price of the article was

34) kalyan bought 16 types of rava of certain rupees. After a week, he sold 6 types of rava at 20%profit,
6types of rava with neither profit nor loss and 4 types at 10%loss. In this transaction, what is the profit %?

35) A merchant purchases a table and a fan for Rs.450. He sells them making a profit of 15% on the table and
20% on a fan. He earns a profit of Rs.76.50. The difference between the original prices of the table and fan is
equal to

36) When a manufacturer allows 38% commission on the retail price of his product he earns a profit of 9%.
What would be his profit percent if he commission is reduced by 22%?

37) Sridevi sells an article at a profit of 50%. If she had bought it at 40% less and sold it for Rs.21.50 less,
She would have gained 60%. Find the cost price of the article.

38) If the cost price of shoes doubles, then the loss gets tripled of what it was initially. The initial loss % was ?

39) Gowtham started a business, investing Rs.25000. After 4 months and 5 months respectively, Rithick and
Sudhir joined him with capitals of 15000 and 12000. At the end of the year the total profit was Rs.4632. What
is the difference between Rithick’s and Sudhir’s share in the profit approximately?

40) Bhuvana bought 50 dozen of bangles at Rs5 per dozen. She spend Rs.125 on a particular tax and she
sold them at 50p per each bangle. What was her profit or loss percent?

www.ibpsguide.com | estore.ibpsguide.com | www.sscexamguide.com


202
Complete Quantitative Aptitude Questions

31) Let us assume cost price is x


Ie., x – 18=1/ 19*x
X – 18=x/19
X –x/19= 18
18x=18*19
18x=342
X=342/18
X=19

32) Total cost price = Rs(60*55) = Rs.3300


Total selling price = Rs [(15*55 + 15 * 6)+(37*55+37*7)+(8*55 - 8*30)]
= Rs. [ 915 + 2294 + 416]
= Rs.3625
Average profit = Rs[3625 – 3300/60]
= Rs.(325/60)
= Rs. 5.416
Therefore the average profit per chair is Rs 5.41.

33) Let the C.P. of article be Rs. x.


110% of x - 60% of x = Rx. 200
50% of x = Rx. 200
x = Rs. (200×100)/50
= Rs. 400

34) Let CP of 16 types =x


CP of 1 type= x/16
Sp of rice 20%=6x/16+6x/16*20/100=9x/20
Spof rice without profir or loss=6x/16=3x/8
Sp of rice at 10% loss= 4x/16-4x/16*10/100=9x/40
Total seling price=9x/20+3x/8+9x/40 = 42x/40
now profit%= [42x/40-x]/x *100
=5%

35) Let cost price of table be Rs X


Cost price of fan = Rs(450-X)
[15% of X]+[20% of (450-X)] = 76.50

www.ibpsguide.com | estore.ibpsguide.com | www.sscexamguide.com


203
Complete Quantitative Aptitude Questions

15/100*X+20/100*(450-X) = 76.50
X = 270
So cost price of table = Rs.270
Cost price of fan = Rs.180
Difference=Rs(270-180) =Rs 90

36) Let retail price =Rs.100, then commission=Rs.38


Selling price=Rs(100-38)
=Rs.62, but profit=9%
Cost price=Rs[100/109*62] =56.88
New commission=Rs.16, New selling price=Rs(100-16) =Rs.84
Gain=Rs[84 - 54.88]=27.12
Gain%=[27.12*100/56.88]
=47.6%

37) 1st selling price=150% of x = 150x/100 = 15x/10


New Cost price=60% of x = 60x/100 = 6x/10
2nd selling price = 160% of 6x/10 = 160/100 * 6x/10 = 96x/100
Therefore 15x/10 – 96x/100 = Rs.21.50
X=Rs.39.81
Therefore cost price of the article = Rs.40 (approx.)

38) Let the cost price be y and selling price be z.


Loss = y-z
When the cost price doubles, the loss gets tripled.
So it becomes like this, 2y – z = 3(y-z)
y=2z
Loss % = (2z-z/2z*100) = 50 %

39) Gowtham investment =25000*12=3000000


Rithick investment = 15000*8 =120000
Sudhir investment = 12000*7 =84000
Ratio of their investment= 25:10:7
The difference between Rithick’s and Sudhir’s share=3
That is= 4632*3/42 = Rs 331 (approx.)

40) Cost price of 1 dozen bangles = Rs.5

www.ibpsguide.com | estore.ibpsguide.com | www.sscexamguide.com


204
Complete Quantitative Aptitude Questions

Cost price of 50 dozen bangles = 50*5=Rs. 250


Amount of tax paid = Rs.125
Total cost price=250+125 = 375
Selling price of total number of bangles=50 * 12 * ½ = 300
Loss = 375 – 300 = 75
loss percentage = (75*100)/375 = 20%

41) Profit earned by an corporation is distributed among total officers and clerks in the ratio of 7:5
respectively. If the number of officers is 60 and the number of clerks is 100 and the amount received by an
each officer is Rs.35,000. What was the total amount of profit earned?

42) Dharan bought a donkey and a carriage for Rs.3000. He sold the donkey at a gain of 20% and the
carriage at a loss of 10%, thereby gaining 2% on the whole. Find the cost of the donkey?

43) In a certain office, the profit is 160% of the cost. If the cost increases by 12% but the selling price remains
constant, approximately what is the percentage profit?

44) Vaishnavi purchased 35kg of wheat at the rate of Rs.19.50 per kg and another 35kg of wheat at a certain
rate. She mixed the two and sold the entire quantity at the rate of the Rs.21.25 per kg and made 20%. Overall
profit at what price per kg did she purchase the lot of another 35kg wheat?

45) A car worth Rs.2,50,000 is sold by usha to nandhini at 10 % profit. Nandhini sells the car back to usha at
4% loss. Then in the entire transaction total gain by usha is?

46) A shopkeeper sells a cricket bat whose marked price is Rs.60 at a discount of 20% and gives a ball
costing Rs.3 free with each bat. Even then he makes a profit of 25%. His cost per bat is.

47) Murugan sold a book at a loss of 40%. If the selling price had been increased by Rs.200, there would
have been a gain of 10%. What was the cost price of the book?

48) Swetha purchased 160kg of ragi at Rs.27 per kg and mixed it with 240kg ragi at Rs.32 per kg. At what
rate should she sell the mixture to gain 32% profit?

49) In a shop the profit is 160% of the cost. If the cost increased by 12.5% but the selling price remains
constatnt, approximately what percentage of the selling price is the profit?

www.ibpsguide.com | estore.ibpsguide.com | www.sscexamguide.com


205
Complete Quantitative Aptitude Questions

50) By mixing two brands of dal and selling the mixture at the rate of Rs.354 per kg a shopkeeper makes a
profit of 36%. If to every 4 kg of one brand costing Rs.400 per kg, 6kg of other brand is added. Then how
much per kg does the other brand cost?

41) The total amount distributed among 60 officers=35000*60 = 21,00,000


Let the total amount distributed to 100 clerks be ‘Y’
Then 21,00,000/Y = 7/5
Y = 15,00,000
Therefore total profit = 21,00,000 + 15,00,000 = 36 lakhs

42) Let the cost price of the donkey be Rs ‘X’


Then cost price of the carriage = Rs(3000-X)
20% of X – 10% of (3000 – X) = 2% of 3000
X/5 – (3000-X)/10 = 60
2X – 3000 + X = 600
X = 1200
Hence cost price of the donkey = Rs.1200

43) Let cost price = Rs.100 ; then profit = 160


Selling price = Rs.260 ; new cost price = 112% of Rs 100
New cost price = 122 ; new selling price = Rs 260
Profit = 260 – 112 = Rs.148
Required percentage = (148/112 *100)%
= 132.14%

44) Let the required price per kg be Rs’X’


Then cost price of 70 kg wheat = Rs(35 * 19.50 + 35 * X)
= Rs (682.5 + 35X)
Selling price of 70kg wheat = Rs (70 * 21.25)
= Rs.1487.5
Total cost price=1487.5*100/120=1239.58
35x= 1239.58-682.5=557.08
X=15.91
So the cost price of second lot is Rs15.91 per kg

45) Money spent by usha = Rs.2,50,000


Money received by usha = 110% of Rs.2,50,000

www.ibpsguide.com | estore.ibpsguide.com | www.sscexamguide.com


206
Complete Quantitative Aptitude Questions

= 2,75,000
Cost price to usha = 96% of Rs.2,75.000 = Rs.2,64,000
Therefore usha gains Rs(2,75,000 – 2,64,000) = Rs.11000

46) Marked price = Rs.60


Selling price = Rs [(80/100 * 60) – 3]
= Rs.45
Let cost price brRs ‘x’.
Then 125% of x = 45
(125/100 * x) = 45
Cost price of one bat = Rs.36

47) Let cost price be Rs ‘x’


Then (110% of x) – (60 % of x) = 200
(110x/100) - (60x/10) = 200
110x – 60x = 20000
X = 400
So cost price of the book = Rs.400

48) Cost price of 400kg of mixture = Rs(160 * 27 = 240 *32)


= Rs12000
Selling price = 132% of Rs 12000
= 132/100 * 12000
= 15,840
Rate of selling price of the mixture = Rs 15840/400
= Rs.39.60 per kg

49) Let cost price be ‘x’


Then profit = Rs 160
Selling price = Rs.260
New cost price = 112.5% 0f Rs.100
= (112.5x/100 * 100)
= Rs.112.5
New selling price = Rs.260
Profit = Rs(260 – 112.5)
= Rs.147.5
Required percentage = (147.5/260 * 100) % =56.7%

www.ibpsguide.com | estore.ibpsguide.com | www.sscexamguide.com


207
Complete Quantitative Aptitude Questions

50) Let the cost price of other brand be Rs ‘x’ per kg


cost price of 10 kg = Rs (4 * 400 + 6 * x)
= Rs (1600 +6x)
Selling price of 10 kg = Rs (10 * 354 )
= Rs.3540
Therefore 3540 –(1600 + 6x)/1600 + 6x * 100 =36
1940 – 6x/1600 + 6x = 0.36
1940 – 6x = 576 + 2.16x
x=167.15
so the cost of other brand = Rs.167.15

51) Bharani sells a novel at a gain of 40%. if he had bought it at 20% less and sold it for Rs. 70 more which
equals to 350% of CP. The CP of the novel is

52) A salesman sold an book at a loss of 25%. If the selling price had been increased by Rs. 150, there would
have been a gain of 15%. The cost price of the book was:

53) An product is sold at a loss of 10%. Had it been sold for Rs. 9 more, there would have been a gain of 12
1/2% on it. The cost price of the product is

54) Fresh grapes contains 80% of water by weight, where as dry grapes contains 50% of water by weight.
Yamini buys 100kg of fresh grapes for a total price of Rs. 300. At what price should she sell the dry grapes
(fresh grapes are dried up) to earn 80%profit?

55) Anamika purchased 60 tablets at a price of Rs.60 per tablet. she sold 15 tablets at profit of Rs.6per tablet
and 38 tablets at a profit of Rs. 7per tablet. The remaining tablets were sold at a loss of Rs. 4 per tablet. What
is the average profit percentage per tablets?

56) In order that there may be a profit of 15% after allowing a discount of 10% on the marked price, The
marked price of an book has to be increased by %?

57) What is the highest % discount(approximately) that a trader can offer on his marked price, so that he
ends up selling at no profit or loss, if he initially marked his goods up by 60%?

www.ibpsguide.com | estore.ibpsguide.com | www.sscexamguide.com


208
Complete Quantitative Aptitude Questions

58) A shopkeeper sells speaker at the rate of Rs 914each and earns the commission of 4%. He also sells
phones at the rate of Rs 160each and earns a commission of 20%. How much amount of commission will he
earn in two weeks if he sells 20 speaker and 12phones per day?

59) The price of a necklace, passing through a 3 persons, rises on the whole by 32.5%. If the 1st and the 2nd
sellers earned 10% and 12.5% profit respectively. Find the percentage profit earned by the 3rd seller.

60) Varshini sold one – fourth of her books at a gain of 10% and the remaining at cost price. Find the gain
earned by her in the whole transaction.

51) Let CP= x


SP= 140x/100 = 14x/10
CP= 80% 0f x= 8x/10
then, SP =14x/10+70
14x/10 + 70= 350%of 8x/10
14x/10+70=350/100*8x/10=28x/10
70=28x/10-14x/10
70=14x/10
X=700/14
X=50

52) Let the C.P. of book be Rs. x.


115% of x - 75% of x = Rx. 150
40% of x = Rx. 150
x = Rs. (150×100)/40
x= Rs. 375

53) Let the cost price of the product = Rs. x


S.P. at 10% loss
= x×90/100= Rs. 9x/10
S.P. at 12 1/2 % gain
x × (100+12 1/2)/100 = Rs. 225x/200
9x/10 + 9 = 225x/200
180x + 1800 = 225x
x = Rs. 40

54) In fresh grapes ratio of water to grapes is 80:20= 4:1

www.ibpsguide.com | estore.ibpsguide.com | www.sscexamguide.com


209
Complete Quantitative Aptitude Questions

In dry grapes ratio is = 50:50 =1:1


Weight of grapes is = 100*[1/5]= 20kg
Weight of dry grapes = 20*2 =40 kg
Required amount is = 300*100[180/100]/40 = Rs. 1350

55) Total CP= 60*60=3600


Total Sp= [(15*60+15*6)+(38*60+38*7)+(7*60-7*4)] = 990+2546+392=3928
Average profit = 3928-3600/60
=328/60
=>5.46%

56) Mp= Rs x
Sp= 90x/100= 9x/10
Mp= 9x/10*100/115=18x/23
Mp has increased by
(x-18x/23)/(18x/23)*100
=27.7%

57) Let CP= Rs 100


MP= Rs100+60=160
Discount= x%
then, 160*[100-x/100] =100
100-x=100*100/160
x=37.5%

58) Sp of modem=Rs 914


Commission of one speaker =4*914/100
Commission of 20 speakers = 4*914*20/100=Rs 731.2
Commission of one phone = 20*160/100
Commission of 12 phones = 20*160*12/100=384
Total commission on one day= (RS.731.2+384) = 1115.2
The amount of commission will he earn in 2 weeks = 1115.2*14= Rs 15,612.8

59) Let the original price of the necklace be Rs x and


Let the profit earned by the 3rd seller be P%
Then (100+P)% of 112.5% of 110% of x = 132.5% of x
(100+P/100)*(112.5/100)*(x) = 132.5/100*x

www.ibpsguide.com | estore.ibpsguide.com | www.sscexamguide.com


210
Complete Quantitative Aptitude Questions

(100+P)*112.5*110 = 132.5 *100 *100*100/100


100+P = 132.5 *100*100/112.5 *110
=1325000/12375
100+P = 107.07
P= 107.07 – 100 = 7.07%
The percentage profit earned by the 3rd seller = 7%

60) Let cost price of whole be Rs x


Cost price of 1 / 4 th =Rs x/4
Cost price of 3 / 4 th = Rs 3x/4
Total selling price = Rs [(110% of x/4)+3x/4)]
= Rs [(110/100 * x/4) + 3x/4]
= Rs [11x/40 + 3x/4]
= Rs (11x+30x/40)
= Rs 41x/40
Gain = Rs 41x/40 – x = x/40
Gain % = x/40 * 100 *1/x = 100/40
Gain % = 2.5%

61) Find the single discount equivalent to a series discount of 40%, 20% and 10%

62) Kumar buys a car marked price at Rs 500000 with 35% and 30% off. He spends Rs 75000 for
maintenance sell it for Rs 500000 What is the gain or loss percentage?

63) Rita bought a computer with a discount of 10% on its marked price. she sold it at a price 25% more than
the price at which she bought it. The new SP is how much percentage more than marked price?

64) Terms of percentage loss, which of the following is the best transaction?
No options here

65) Ganesh purchased a bed and a washing machine for Rs 40,000. Later , he sold the bed at 40% profit and
the washing machine at 20% loss. Thus, he gained 4% in the whole transaction. Find cost price of the bed?

66) A substance is purchased for Rs. 300. If one fourth of the substance is sold at a loss of 10% and the
remaining at a gain of 5%, Find out the overall gain or loss percentage.

www.ibpsguide.com | estore.ibpsguide.com | www.sscexamguide.com


211
Complete Quantitative Aptitude Questions

67) A vendor sells a radio at Rs. 1680 at a gain of 40% and another for Rs. 1920 at the loss of 8%. Find his
total gain percent

68) A sells a gold ring to B for Rs 90,000 losing 20% in the transaction. B sells it to C at a price which would
have given a profit of 20% to A. By what percent does B gain?

69) Aruna silks has announced 50% discount on prices of fancy sarees at the time of sale. If a purchase
needs to have a discount of Rs 960, then how many sarees, each costing Rs 640 should be purchase?

70) The market price of a novel was 50% more than its cost price. Vadi was going to sell it at market price to
a customer, but he showed vadi some defects in the novel, due to which vadi gave him a discount of
33%.Next day he came again and showed vadi some more defects, hence he gave him another discount that
was equal to 14.5% of the cost price. What was the approximate profit/loss to vadi?

61) Let marked price be Rs 100


Then net selling price = 90% of 80% of 60% of Rs 100
= 90/100 *80/100 *60/100 *100
= 90*80*60/10000
= Rs 43.2
Required discount = (100 – 43.2) %
= 56.8%.

62) Total CP= (65%0f 70%of 500000)+75000


=0.65*0.7*500000+75000
CP=Rs 302500
SP=Rs5,00,000
gain%=(500000-302500)/302500*100
=65.28%

63) Mp=Rs x
Sp=Rs 90/100*x=9x/10
Sp=9x/10*125/100=9x/8
Required %= (9x/8-x)/x *100 = 12.5%

64) %loss= loss/CP*100


A. Cp= Rs 15, loss= Rs 3
%loss=3/15*100=20%

www.ibpsguide.com | estore.ibpsguide.com | www.sscexamguide.com


212
Complete Quantitative Aptitude Questions

B. CP=Rs18, loss=Rs4
%loss=4/18*100=22.2%
C. CP=Rs 14, loss= Rs 3
%loss=Rs3/14*100=21.4%
D.CP=Rs 30,loss=Rs 5
%loss=5/30*100=16.6%
Option D is the best transaction

65) CP of bed=x
x*40/100-(40,000-x)*20/100=40,000*4/100
40x-(800000-20x)=160000
60x-800000=160000
60x=960000
X=Rs16000

66) Price Received by selling one fourth of the substance at a loss of 10% =
(1/4) * 300 * (90/100) = Rs. 67.5
Price Received by remaining substance at a gain of 5% =
(3/4) * 300 * (105/100) = Rs.236.25 [Note: 1-(1/4) = 3/4]
Total Selling Price =RS303.75
Profit = 303.75 - 300 = 3.75
Profit%=(Gain/Cost∗100)%
=(3.75/300∗100)%
=1.25%

67) So, C.P. of 1st radio= (100/140∗1680)=1200


C.P. of 2nd radio= (100/92∗1920)=2086.95
Total C.P. = 3286.95
Total S.P. = 3600
Gain = 3600 – 3286.95 = 313.05
Gain% = 9.5%

68) A sells to B at 20% loss


So CP=90000*100/80=112500
20%profit to A= 112500*120/100=135000
Total gain= (135000-90000)=45000
Gain%= 45000/90000*100=50%

www.ibpsguide.com | estore.ibpsguide.com | www.sscexamguide.com


213
Complete Quantitative Aptitude Questions

69) The cost of discount on a saree= 50/ 100*640= 320


Number of sarees for taking the discount of Rs.960
=>960/320
=>3 sarees

70) Let the cost price be Rs. 100


Then, market price is Rs. 150
Now, the first discount is of 33% Rs.150 =Rs. 50
Hence, its selling price = 150 - 50 = Rs. 100
since vadi’s selling at cost price, any further discount will be equal to loss %
The next discount of 14.5% will be the loss percentage to vadi

71) Kabi buys wheat at Rs. 15/kg and puts a price tag on it so as to earn a profit of 10%. However, his faulty
balance shows 1000 gm when it is actually 900 gm. What is his actual gain percentage?

72) A taxi driver makes a profit of 25% on every trip when he carries 4 passengers and the price of petrol is
Rs. 35 a litre. Find the approximate % profit for the same journey if he goes for 5 passengers per trip and the
price of petrol reduces to Rs. 28 litres?

73) On selling a fan at 8% loss and an iron box at 18% gain, a man gains rs.346.If he sells the fan at 7% gain
and the iron box at 15% gain, then he gains rs.450. The actual price of the iron box is approximately?

74) A merchant marked his goods at 16% above the cost price. He sold half of the stock at the marked price,
one quarter at a discount of 16% on the marked price and the rest at a discount of 32% on the marked price.
His total gain is,

75) In a certain occupation, the profit is 110% of the cost. If the cost increases by 13% but the selling price
remains constant, approximately what percentage of the selling price is the profit?

76) Hytech electronics is in recession. Find the marked price is what percentage of cost price, if there may be
a profit of 10% after allowing a commission of 5% to middle man?

77) Harshith is a successful milk vector in salem. He sold half of the milk at 40% profit, half of the remaining
milk at 40% loss and he rest was sold at his cost price. In the total transaction, his gain or loss will be

www.ibpsguide.com | estore.ibpsguide.com | www.sscexamguide.com


214
Complete Quantitative Aptitude Questions

78) Usha purchased 10 fans for Rs 1000 each. She spent Rs 4000 on the maintenance of all fans. She sold
five of them for Rs 1500 each and the remaining for 1100 each. Then the total gain or loss % is?

79) Preethi owns lenova mobile in a showroom. she labelled the price of mobile to earn a profit of 20% on the
cost price. she sold the mobile by offering a discount of 10% on the labelled price. What is the actual percent
profit earned?

80) Wipro is the leading IT company then the profit earned by same organization is distributed among HR and
officers in the ratio of 13:7 respectively. If the number of HR is 104 and the number of officers is 156 and the
amount received by HR is Rs 6000. What was the total amount of profit earned (in lakhs )?

71) CP of 1000gm = Rs. 15


SP of 900gm = Rs. 16.5
SP of 1000gm =16.5x1000/900 = Rs. 18
Now take 1000gm as reference to calculate profit.
Profit=SP-CP=18-15=Rs. 3
Profit % = 3x100/15 = 20%

72) When 4 passengers income was 4x


Expense= Rs.35.
Profit =25% of 35+35= Rs.43.75
That means his earning is Rs.44.775.so that per passenger fare must be Rs.10.93
When 5 passengers earning = 10.99x5=Rs.54.68
Expense =Rs.28
Profit = 95.28 % approx.

73) Let the cost price of the fan be Rs X and Cost of the iron box beRs Y
Then 18%Y – 8% X=346 ; That is 18Y-8X=34600 1
And 15%Y+8% X=450 ; that is 15Y+8X=450002
Solving 1 and 2 we get X=1102.275 Y=2412.12
therefore cost of ironbox is = 2412(approx.)

74) Let cost price whole stock = Rs.100


then marked price of whole stock=Rs.116
Marked price of half stock=Rs.58
marked price of onefourth stock=Rs.29
Total selling price=Rs[58+(84% of 29)+(68% of 29)]

www.ibpsguide.com | estore.ibpsguide.com | www.sscexamguide.com


215
Complete Quantitative Aptitude Questions

=Rs[58+24.36+19.72]
=Rs.102.08
Hence gain %=(102.08-100)%=2.08%

75) Let Cp= RS.100


Profit= Rs 110
Sp= Rs 210
New Cp= 113%0f Rs.100=Rs. 113
New SP= Rs. 210
Profit= Rs. (210-113)= Rs. 97
Required %= 97/210*100=46%

76) Let the CP= Rs.100


Then, SP=Rs.110
MP=Rs. x
Then, 95% of x= Rs 110
x= 110*100/95 = 115.7%

77) total CP= Rs 100


total SP=[ 140/2+ 60/4+100/4]
= 280+60+100/4
=440/4
=110
Gain=sp-cp=110-100=10
Gain% = 10/100*100=10%

78) Total actual CP


= Rs. (1000*10+4000)=Rs. 14000
Total Sp
= Rs.(5* 1500+5*1100) =Rs. 13000
Loss= cp-sp=1000
Loss %= (1000/14000)*100 = 7.14%

79) Let the CP of mobile= Rs 100


Then the labeled price= Rs.120
SP= Rs. 120-10%of 120 = RS120-12= Rs 108
Gain= Rs 108- Rs 100 = Rs 8

www.ibpsguide.com | estore.ibpsguide.com | www.sscexamguide.com


216
Complete Quantitative Aptitude Questions

Gain%= 8/100*100=8%

80) Total amount distributed among 104 HR’s= Rs. 104*6000 =Rs. 624000
Let the amount distributed to 156 officers be x
Then 624000/x = 13/ 7
Then x = Rs, 336000
The total profit = Rs. 624000+336000= Rs. 9.6lakhs

81) Ananya invest in two schemes, in one scheme she get 15% profit and in other scheme he get 30% profit,
what is his total profit if he invest in the ratio of 2:3 respectively.

82) Preethi bought 240 reams of brown sheet at Rs 200 per ream and the expenditure on transport Was Rs
960. she had to pay an octroi duty of 1Rs per ream and the labour charges were Rs 120. What should she
charge per ream to gain 80%(in Rs)?

83) Mani went to purchase a mi mobile handset, the shopkeeper told him to pay 25% tax if he asked the bill.
Mani manages to get the discount of 10% on the actual saleprice of the mobile and he paid the shopkeeper
Rs.3275 without tax. Besides he manages to avoid to pay 25% tax on the already discounted price, what is
the amount of discount that he has gotten?

84) Every year before the Diwali festival, the kumaran stores increases the price of the product by 48% and
then introduce two successive discount of 12% and 13% respectively. What is percentage loss and
percentage gain ?

85) A dealer allows a discount of 40 % on the marked price to the retailer. The retailer sells at 10% below the
marked price. If the customer pays Rs.38 for an article, what profit is made by the retailer on it?

86) Sasi is a badham merchant in kerala. He has badham in sealed wooden boxes of 15kg each. The price of
the badham increases by Rs.30 per kg for every year, but at the same time, 10% of the badham are eaten by
rodents every year. If the price of a 1 kg of fresh badham is Rs.240, what is the change in his profits if he sells
a sealed box after one year of storage, rather than selling it fresh ?( In Rs.)

87) A seller mixes 30% water to milk and then he sells the whole mixture at the price of milk. If the cost price
of water be 50% of the cost price of milk, what is the net profit percentage?

www.ibpsguide.com | estore.ibpsguide.com | www.sscexamguide.com


217
Complete Quantitative Aptitude Questions

88) The profit percentage of banu and renu is same on selling the articles at Rs 900 each but banu calculates
his profit in the selling price while renu calculates it correctly on the cost price which is equal to 10%. What is
the difference in their profits?

89) P and Q both are dealers of cars. The price of a car is Rs 56,000. P gives a discount of 10% on whole,
while Q gives a discount of 12% on the first Rs 40,000 and 8% 0n the rest Rs 16000. What is the difference
between their selling prices?

90) A Barbie doll is available at Ratna stores in salem at 20% discount and the same is available at only 18%
discount at Royal stores in attur. Deepika has just sufficient amount of Rs 800 to purchase it at Ratna stores
in salem. What is the amount that deepika has less than required amount to purchase it at Royal stores in
attur?

81) Total profit = (2/5*15)+(3/5*30)


=30/5+90/5
=6+18
=>24%

82) Total CP= Rs(240*200)=Rs. 48000


Total expenditure=(960+1*240+120)= Rs. 1320
totalCp= 48000+1320=Rs. 49320
gain= 80%
sp of 240reams = 49320*180/100 = Rs.88776
Sp per ream= 88776/240= Rs 369.9

83) CP = 100, SP (with tax) =125


New SP = 100 - 10 = 90
Effective discount = 125-90 = 35
So, at SP of 90 ----> discount = 35
and at SP of 3275-----> discount = 35/90*3275= Rs 1274(approx.)

84) Let CP = 100,


48 % increase =>SP = 148
12 % discount in SP => ((148 x 12)/100) = 17.76
So 1st SP = (148 – 17.76) = 130.24

www.ibpsguide.com | estore.ibpsguide.com | www.sscexamguide.com


218
Complete Quantitative Aptitude Questions

13 % discount in 1st SP ((130.24x13)/100) = 16.93


2nd SP = 130.24-16.93= 113.31
So finally CP = 100, SP = 113.31 => gain = 13.31%

85) Let dealer marked price = 100%, Retailer's C.P = 60%


And the retailer sells at 10% less than the marked price =>S.P = 90%
If S.P of 90% of the retailer costs Rs.38 to customer,
So its C.P of 60% will cost 60*38/90 = 25
Profit made by the retailer = 38-25
= Rs.13

86) Price of 1kg fresh batham= Rs.240


Therefore, price of 15kg = Rs.15*240= 3600
10% of 15kg which eaten by rodents = 10 x 15/100 = 1.5kg.
So, End of Year he had 15kg – 1.5kg = 13.5kg
So, he sells that 13.5kg with Rs.30 per kg profit = 13.5*270=3645
He buy 15kg wooden box with badham for Rs.3600 and sell that for Rs. 3645
So, profit = 3645-3600= Rs.45 more money he get.

87) Let CP of 1litre of milk = Rs.200 & CP of 1litre of water = Rs. 100
CP of solution=140+30=170
Sp of 1litre solution= 200
Profit%= (200-170)*100/170 = 17.64%

88) when profit calculate on SP the profit = 10% of 900 = 90


when profit calculate on CP(x) = x+X/10 = 900
11X=9000
X=818.18
profit= 81.82
required diffference= 90-81.82 = Rs 8.18

89) discount offer by by P= 10 % of 56000 = Rs. 5600


total discount offer by Q= 12% of 40,000+8% of 16000 =4800+1280=Rs. 6080
required difference= 6080-5600= Rs.480

90) 80%= 800


100%= 1000

www.ibpsguide.com | estore.ibpsguide.com | www.sscexamguide.com


219
Complete Quantitative Aptitude Questions

when discount 18%,then,


SP= 1000-180=820
required difference= 820-800 = Rs. 20

91) If novels bought at prices ranging from Rs 300 to Rs 470 are sold at prices ranging from Rs 400 to Rs
520, what is the greatest possible profit that might be made in selling 12 books?

92) A seller buys three types of apples at Rs 150, Rs 120, and Rs 110 per kg. He mixes them in the ratio of
5:6:7 by weight and sells them at a profit of 50%. At what price per kilogram does he sell the apples?

93) The ratio of the cost price of product A to that of B is 5:7. product A was sold at a profit of 80% and
product B was sold at a profit of 20%. If the total profit earned after selling both the (products A and B is Rs
296) what is the difference between the cost prices of product A and B?

94) Femina is the famous magazine in India. In that company there is a profit of Rs 150. If 2/3 part of n
magazine is sold at 60% profit, 1/4 part at 32% profit and remaining part at 24% profit and the find the cost
price of the magazine?

95) Rahim is dealer of magnetic components. He imports the components from Russia. Rahim sells a radio
valve at profit of 20%. If he bought it at 20% less and sold it for Rs 5 less than the previous selling price, he
would have gained 25%. Find the cost price?

96) Helen went to a mall and then she bought earings at Rs 120 per dozen. The selling price of hundred
earings so as to gain 30% will be(in Rs)?

97) Kumaran stores offered 60% discount on the marked price and there is a loss of 30%. If it is sold the MP,
find the profit percentage?

98) Rithick wants to buy a computer. He bought a computer listed at Rs 6000 with discount 30% offer on the
list price. What additional amount must be offered to rithick to bring the net price to Rs3950?

99) A marked price of a Chair is Rs 480. The seller offered the discount of 25% and gain 50%. If no discount
is allowed, find his gain percentage?

100) Selvi, Deepi and Dinesh invest in the ratio of 4 : 5: 6. The percentage of return on their investments are
in the ratio of 7 : 6 : 5. Find the total earnings, If Deepi earns Rs. 350 more than Selvi:

www.ibpsguide.com | estore.ibpsguide.com | www.sscexamguide.com


220
Complete Quantitative Aptitude Questions

91) least cost price= 300*12 =3600


greatest sold price= 520 *12= 6240
profit required= 6240-3600=Rs 2640

92) suppose the seller purchases 5kg , 6kg 7 kg apples.


then the price of the total apple
(5*150)+(6*120)+(7*110)=750+720+770=2240
as the total apples weights 18kg and the dealer makes a profit of 50%
the selling price of apples per kg= 1.5* 2240/ 18 = 3360/18=Rs. 186.66

93) CP of product A= 5X
CP of product B=7x
Total price = (5x*0.8)+(7x*0.2) =296
4X+1.4X=296
5.4X=296
x=54.81=55(approx.)
difference of the CP= 7x-5x=2x=2*55
= Rs. 110

94) S1= 2/3 P1=60%


S2=1/4, P2= 32%
S3=1-[2/3 +1/4] =1/12, P3=24%
total profit = Rs 150
CP of an entire magazine = (total profit *100)/( S1*P1+S2*P2+S3*P3)
CP of an entire magazine = (150*100)/(2/3*60+1/4*32+1/12*24)
=15000/(40+8+2)
=15000/50
=RS.300

95) let CP= Rs 100 then SP= 120


New CP= Rs. 80 then New SP=80* 125/ 100 = 100
from question,
Rs 120- Rs 100= Rs 20 is equivalent to Rs 5
Rs 100 is equivalent to Rs 25
CP = Rs 25

96) 12 earings cost= 120per dozen

www.ibpsguide.com | estore.ibpsguide.com | www.sscexamguide.com


221
Complete Quantitative Aptitude Questions

1 earing cost= 120/12 = Rs. 10


sp CP of 100 earings = 100* 10 = Rs 1000
so SP= Rs x
from question,
SP-CP/CP* 100 =profit %
(x-1000)/1000*100= 30
300=x-1000
x=Rs1300

97) MP= Rs X
Sp= 40/100*x= 2x/5
CP= 2x/5*100/70=4x/7
Profit when sold at MP= x-4x/7 =3x/7
Profit% = (3x/7)/(4x/7)*100=75%

98) Mp= 6000


Sp= 6000*70/100 =4200
net amount=3950
required % = (4200-3950)/4200*100
=250/42=>5.95%

99) MP= Rs. 480,discount= 25%


SP= 480*75/100 = 360
CP= 360* 100/150= 240
Required %= (480-240)/240*100 = 100%

100) Selvi deepi dinesh

Total = 88xy/100
deepi earnings – selvi earnings = 2xy/100 =Rs 350
Total earning = 88xy/100 = Rs 15,400

10. PARTNERSHIP

www.ibpsguide.com | estore.ibpsguide.com | www.sscexamguide.com


222
Complete Quantitative Aptitude Questions

1). 3 people A, B, C invested in a business in the ratio of 5:6:9. After 3months B withdraw half of her capital. If
the Sum invested by A is 32000, then what is the profit earned by B at the end of the year out of the total
profit of Rs. 48250?
A) 17965.33
B) 70767.50
C) 46189.25
D) 10193.66

2). Two people abi and krish invested in a business with 7 lakh and 8 lakh rupees respectively. They agree
that 46% of the profit should be divided equally among them and rest is divided between them according to
their investment. If krish got Rs. 2100 more than abi, Then the total profit is,
A) 55000
B) 56000.
C) 58333
D) None

3). P, Q, R invested in the ratio of 15:16:17. After the end of the business teram they receives the profit in the
ratio 9:10:11. Find the ratio of time in which they invested in the business.
A) 934:1699:1578
B)1224:1275:1320
C)1578:1668:8096.
D) None

4). Anu and Banu invested rupees Rs.26000 and Rs.22000 respectively. Anu being an active partner will get
Rs.350 every month extra for running the business. In two years if Anu receives a total of Rs. 18000, then
what is profit earned by Banu in two years.
A) 2200
B) 3300
C) 4400
D) None

5). M and N invested in a business in which M invests Rs.385 more than N. N invested for 9months while M
invested for 5months. If M get Rs.65 more than N out of profit of Rs.1350. Then then total amount invested in
the business approximately is,
A) 1452
B) 7413
C) 6298

www.ibpsguide.com | estore.ibpsguide.com | www.sscexamguide.com


223
Complete Quantitative Aptitude Questions

D) 5778

6). 3 partners A , B, C starts a business. 4 times of A's capital is equal to 6 times of B's Capital and B's Capital
is 8 times of C's capital if profit of B is Rs.6300. Then find the average profit A and C
A) 7000.50
B) 5118.75
C) 4475.25
D) none

7). Three partners A, B, C invest a total sum of Rs,114000. At the end of the year A gets Rs 6000 and B gets
5000 and C gets Rs 8000 as profit. Find the average amount invested by A and C.
A) 46677
B) 42000.
C) 53000.
D) None

8). X invested 13% more than Y's investment. Y invested 18% less than investment of Z. X's investment is
what percent of investment made by both Y and Z?
A) 47.5.
B) 50.9.
C) 56.5
D) None

9. Two persons P and Q invested in a business with 21 lakh and 28 lakh rupees. They agree that 30% of the
profit should be in tha ratio 2:3 for P and Q and rest is divided between them according to their investment. If
Q got Rs.1200 more than P, then then total profit Q is
A) 4350
B) 4567
C) 4467
D) None

10). Three persons enter into a partnership by investing in the ratio of 9:8:1. After one year P double its
investment and R puts another Rs.6000 to the initial investment. Now the ratio of investment changes to
9:8:2. What is total investment P,Q, R after 2years?
A) 108000.
B) 114000.
C) 126000.

www.ibpsguide.com | estore.ibpsguide.com | www.sscexamguide.com


224
Complete Quantitative Aptitude Questions

D) None

1). Answer: D)
Let A, B, C put amounts be 5x,6x,9x respectively
Then A's investment is
5x=32000=> x=Rs.6400
Then, B, C puts amount is Rs.38400 & 57600
=32000*12 : 38400 *3 +19200 * 9:57600*12
=>384000:288000:691200
=20:15:36
B’s profit is 48250* 15/71
=Rs. 10193.66

2). Answer: C)
Ratio of profit abi and krish is,
=>7:8
46% of then profit should be divided equally among them, remaining share is 54% of x
Abi share =54/100 * x * 7/15
Krish share =54/100 * x* 8/15
54/100 * x * 8/15 - 54/100 * x* 7/15 =2100
X =58333

3). Answer: B)
P, Q, R invest the amount in the ratio 15:16:17
T1, T2, T3 are time for investment profit gained =investment × time
Profit ratio= 9: 10: 11=15 × T1 :16 × T2 : 17 × T3
T1:T2:T3 =1224:1275:1320

4). Answer: C)
Profit ratio at Anu &Banu is ,
Anu: Banu=26000 :22000.
Anu :Banu =13:11
Anu being an active e partner will get Rs 350 / every month.
2 years receive = Rs. 8400
Balance amount of Anu =18000 -8400 = 9600
Profit gained Banu in 2 years =9600/24 *11 = Rs.4400

www.ibpsguide.com | estore.ibpsguide.com | www.sscexamguide.com


225
Complete Quantitative Aptitude Questions

5). Answer: B)
Let investment of M is X + 385
Let investment of N is X
Profit earned of M&N is,
M/N=(X+385)*5 /x *9-------> (1)
Profit of M =65 +profit of N
Total profit = 1350=65 +profit of N + profit of N
Profit of N =Rs.642.5
Profit of M =65 +642.5= Rs.707.5
Profit ratio of both = 707.5 :642.5
=283 :257-------> (2)
From 1& 2, 283 /257 =(x + 385) *5/(x*9)
X =392.01
Total investment =(392+385)*5+(392*9)
=Rs.7413

6). Answer: B)
Let the capital of C be y
Let the capital of B be 8y
Let the capital of A be 48y/4=12y
Ratio of their profit is ,
A:B:C = 12y : 8y :y
=12:8:1
Total of A&C =6300/8 *13
=Rs .10237.5
Average profit of A&C
=10237.5/2 =Rs.5118.75

7). Answer: B)
Profits of A,B &C is 6000,5000, 8000 rupees
Ratio of their profits is,
A:B:C =6:5:8
Investment of A,B,C be
6x,5x,8x
6x+5x+8x=114000
=>x=6000
Average amount invested by A &C is

www.ibpsguide.com | estore.ibpsguide.com | www.sscexamguide.com


226
Complete Quantitative Aptitude Questions

6000/2 *14 =Rs.42000

8). Answer: B)
Let investment of Z be 100%
Let Y’s investment is 82%
Also A’s investment is 92.66%
Required % =92.66/182*100= 50.9%

9). Answer: A)
Ratio of profit of P&Q is,
P:Q=21:28 => 3:4
Let total profit gained be X
Since ,30% of profit should be divided in ratio 2:3 for P&Q,
Remaining share is =70% of x
P’s share =70/100 *x* 3/7 +30/100*x*2/5
Q’s share =70/100 *x* 4/7 +30/100*x*3/5
(70/100 *x* 4/7 +30/100*x*3/5) – (70/100 *x* 3/7 +30/100*x*2/5)
=1200
X =Rs 7500
Q’s total profit =70/100 *7500 *4/7 + 30/100 *7500*3/5
=Rs 4350

10). Answer: B)
Let investment of P, Q & R are 9x,8x & x for 1year.
After 1 year ratio of their investments.
9:8:2 =9 x +18x : 8x: x+6000=27x :8x: x+6000
X =Rs.6000
Total investments of P, Q & R
After 2 years
=[9+8+2]*6000=Rs.114000.

11). Mahesh and Devi invested Rs.20979 and Rs. 22977 respectively in a business. Devi being an active
partner will get Rs.750 every month extra for running the business. In 18 months if Devi received a total
amount of Rs.18250, then what is total profit earned by both in one and half years is,
A) 26888.
B) 22500.
C) 22587

www.ibpsguide.com | estore.ibpsguide.com | www.sscexamguide.com


227
Complete Quantitative Aptitude Questions

D) 25874

12). X invested three- nineth, Y invested three-seventh of the remaining and Zth remaining. If Y earned
Rs.840 as profit pr each year. Find the average monthly profit of all.
A) 510
B) 490
C) 630
D) 370

13). A Shop makes a profit of Rs. 85000 of which 15% as paid as taxes. If the rest is divided among the
partners A,B,C in the ratio 8:7:6 then the total share of A and B is
A) 54780
B) 51607
C) 56803
D) None

14). In a partnership P invests 2/6 th of the capital for 2/3rd of the time, Q invests 3/5th of the capital for 1/3rd
of the time and R invests for 2/5th of the capital for 3/4th of the time. Out of the profit of Rs. 64375at the end
of the year, find R's share in the profit
A) 27854.
B) 23000.
C) 26740.
D) 25784

15). Rs.10500 is divided among P, Q and R. So that P receives half as much as Q and Q receives half as
much as R. Then then total share of P and R is
A) 4500
B) 3000
C) 5000
D) 7500

16). P, Q, R enter into a partnership. P initially invests Rs 15000 and withdrawn Rs. 7500 after 7 months. Q
initially invests Rs 12500 and withdraws Rs 10000 after 5 months and R invests Rs 10000 and adds another
Rs 5000 after few months. At the end of 1 year and 3 months profits of P and R is twice the profit of Q. Then
after how many months did R invests Rs 10000?
A) 2.5
B) 2.6

www.ibpsguide.com | estore.ibpsguide.com | www.sscexamguide.com


228
Complete Quantitative Aptitude Questions

C) 2.7
D) None

17). Ravi and kavi are partners in a business. Ravi contributes 3/7th of the total capital for 20 months and kavi
received 3/4 th of the profit. Then how long kavi invest the money in the business?
A) 5
B) 11
C) 6
D) 9

18). M and N invested in a work which M invest Rs 3015 more than N. N invested for 7 months while M
invested for 5 months. If M gets Rs 425 more than N, out of a total profit of Rs 3400. Then the amount
invested in the business by M approximately is,
A) 6784
B) 6258
C) 7524
D) 7275

19) X is a working and Y is sleeping partners in a business. X puts in Rs. 2500 and Y puts in Rs.3000. X
receives 25/2% of the profit for managing the business and the rest is divided in proportion to their capital.
What does Y get out of a profit of Rs. 440?
A) 210
B) 200
C) 250
D) 275

20). P and Q entered into a partnership with captials in the ratio 4:5. After 3 months, P withdrew 1/3 of his
capital and Q withdrew 1/5 of his capital . The gain at the end of 9 Months was Rs. 775. P's shares of profit is
A) 358
B) 362
C) 337
D) 345

11). Answer: C)
Profit ratio of Mahesh: Devi =20979 :22977=:21:23
Devi is an active partner will get Rs,750/month
For 18 months ,she receives

www.ibpsguide.com | estore.ibpsguide.com | www.sscexamguide.com


229
Complete Quantitative Aptitude Questions

Rs.13500
Balance amount of Devi
=18250 -13500 =Rs 4750
Then profit gained by Devi in 18 months =4750/23 *21 =Rs.4337
Total profit earned by both in one &half years =18250+4337
=Rs 22587

12). Answer: B)
Let total amount puts by X, Y& Z be P
X’s investment = 3/9 P
Balance investment =6/9 p
Y’s investment =6/9*P*3/7=2/7 P
Remaining investment =8 p/21 =Z’s amounts
Profit ratio of X,Y &Z,
X:Y:Z=3/9 *P:2/7 *P:8/21*p=7:6:8
1month share =840/12=Rs .70
Average monthly profit of X, Y, Z=70/3 *21
=Rs.490

13). Answer: B)
Profit by company =Rs.85000
After paying tax of 15%
Remaining profit =85000*85/100
=Rs 72250
Profit ratio of X, Y, Z
=8:7:6
Total share of X& Y
=72250*15/21
=Rs.51,607

14). Answer: C)
Ratio of their profits
(2/6*2/3)*12: (3/5*1/3)*12: (2/5*3/4) *12 =40:36:54
R’s share =64375*54/130
=Rs.26740

15). Answer: D)

www.ibpsguide.com | estore.ibpsguide.com | www.sscexamguide.com


230
Complete Quantitative Aptitude Questions

Let amount received by R is X


Amount received by Q is X/2
Amount received by P is X/4
Profit ratio of
P,Q &R =X/4:X/2:X
=1:2:4
Total share of P&R
=10500*5/7 =Rs 7500

16). Answer: A)
Ratio of P, Q &R is ,
15000*7+7500*8:12500*5+2500*10:10000*x+15000*(15-x)
P: Q: R=66:35:9-2x
Profit of Q = (profit of P + profit of R)/2
35 = (66 +9 -2x)/2
X=2.5 months

17). Answer: A)
Total profit be X
kavi’s profit is 3/4x
Ravi’s profit is 1/4x
Ratio of ravi & kavi profit =3/4 x: 1/4x=3:1
Let Ravi's investment =3/7 *P(for 20 months then, kavi’s investment =4/7 P)
Ratio of ravi & kavi
Profit =3:1=3/7*P*20 :4/7*p*T
T=5 months= kavi investment time

18). Answer: A)
Let investment of N be x
Profit of N be y
Investment of M is x+3015
Profit ratio of M&N is,
M:N =[x+3015]*5:x*7-------->(1)
Profit of M =425 +profit of N
Total profit =3400 =425+profit of N+profit of N
Profit of N =Rs 1487.5
Profit of M=Rs 1487.5 +425

www.ibpsguide.com | estore.ibpsguide.com | www.sscexamguide.com


231
Complete Quantitative Aptitude Questions

=Rs 1912.5
Profit ratio of
M&N=1912.5:1487.5
=153:119------------>(2)
153/119=(x+3015)*5/x*7
X=Rs.3769
M’s investment =x+3015
=3769+3015
=Rs6784

19). Answer: A)
Total profit = Rs. 440
X's share for managing the business i.e
25/2 % = {25/ 100}{440/2}=Rs.55
Remaining profit of X and Y as per their capital = 440 - 55= Rs. 385
Ratio of amounts = 2500 : 3000 = 5 : 6
Sum of ratios = 5 + 6 = 11
X's share = 385{5}/{11}= Rs.175
X's total share = 175 + 55= Rs. 230
Y's share = 385{6}/{11}=Rs.210

20). Answer: C)
Investment initial of P=4x
Investment initial of Q=5x
After 3 months of P=4x*(1/3)=4x/3
After 3 months of Q= 5x*(1/5)= x
Ratio of capitals = (4x*3+3x*6: 5x*3+4x*6)=30x:39x=10:13
Total profit =775
P’s share =10/23*775=337(approx.)

21). Agalya started a work with Rs. 144000 and after 3months Bhuvana joined with her the amount of Rs.
116000. At the end of the year bhuvana received a profit of Rs.20960 including 16% as a commission for
maintain the work. Find the profit earned by agalya at the end of the year approximately?
a) 22045
b) 23040
c) 21450
d) 24500

www.ibpsguide.com | estore.ibpsguide.com | www.sscexamguide.com


232
Complete Quantitative Aptitude Questions

22). A started a business with Rs. 28000 and is later joined by B with Rs. 49000. After how many months did
B joined if the profit after 3 yrs divided in the ratio for 7:6 for B and A?
a) 12
b) 17
c) 16
d)15

23). Ajay and vivek are active partner in a particular work. Ajay receive 1/7th of the capital for 84 months and
vivek receives 1/8 th of the profit, for how long vivek amount was used?
a) 11
b) 8.
C) 4
d) 2

24). In a work A and B invested amounts in the ratio 7:9, whereas the ratio between amounts invested by A
and C was 5:11. If Rs. 12750 was their profit, how much amount did B gained?
a) 2546
b) 2475
c) 2842
d) 2656

25). M, N and O amounted plot. M invests 24 flat for 27months, N invests 18 flat for 24 months and O puts 48
plat for 27 months. If the rent of the plot is Rs.44649 then what is the average amount paid by M & O?
a) 18116.5
b) 18265.5
c) 16878.5
d) 12688.5

26). P and Q established a shop. P puts 7 times that of Q. Q also kept the amount puts for 5 as much time as
P. If Q got a profit of Rs.12360. What was the total profit?
A) 25748
B) 27856
C) 29664
D) 28458

www.ibpsguide.com | estore.ibpsguide.com | www.sscexamguide.com


233
Complete Quantitative Aptitude Questions

27). R being the silent partner contributes 2/9 th of profit and the remaining is divided between S and T in the
ratio of 7:5. If the difference between the profit shares of R and T is Rs.6600. What is the S’s share?
a) 27900
b) 28000
c) 29400
d) 26889

28). L, M, N enter into a partnership work with L receives Rs. 54000. If out of a total profit of Rs. 4500. L gets
Rs. 1500 and M gets Rs.2100. N gets Rs.900, then M capital is
a) 70500
b) 75600.
c) 72700
d) 74900

29). X, Y, Z started a business by investing Rs.60,000, Rs.70,000 and Rs.80,000 respectively. Find the share
of each out of an annual profit of Rs.28350
A) 8000, 9367, 11800
B) 8500, 9400, 11000
C) 8308, 9530, 10346
D) 8100, 9450, 10800

30). Tonu started a business investing Rs. 90,000. After 3 months, reetu joined him with a capital of Rs.
1,20,000. After 6 months, shalu joined them with a capital of Rs.1,80,000. At the end of the year, they made a
profit of Rs.33000. find the share of shalu.
A) 13200
B) 6600
C) 7500
D) 12000

21). Answer: B)
Ratio of agalya & bhuvana
144000*12: 116000*9= 48:29
Let the total profit be Z.
Profit earned by bhuvana =16 % of Z+ 29/77 of 84 % of Z=20960
524Z= 20960 * 100 *11 => Z= Rs.44000.
Total profit by agalya
=48/77 *84/100 *44000 = Rs.23040.

www.ibpsguide.com | estore.ibpsguide.com | www.sscexamguide.com


234
Complete Quantitative Aptitude Questions

22). Answer: A)
Let number of months A invest by B be Y
Ratio of A after 2 yrs is,
6/7 = (2800*36)/(4900*Y)
y = 24 months
No of month after A joined =36 -24 =12 months

23). Answer: D)
Let total profit be A.
Profit earned by vivek =1/8*A.
Profit earned by Ajay =7/8 *A
Ratio of vivek & Ajay =1:7
Let total capital investment be Z Ajay receive 1/7th of Z for 84 months
Vijay receive 6/7th of Z for Y months
7/1=(1/7*Z*84)/(6/7*Z*Y) => Y = 2month

24). Answer: C)
A &B ratio =7:9
B & A post ratio = 9:7
A & C post ratio=5:11
A:B:C = 45:35:77
Amount received by B =35/157*12750=Rs.2842

25). Answer: B)
Ratio of M, N, O is,
24*27:18*24:48*27 =3:2:6
Amount paid by M&O=44649*3/11+ 44649*6/11
=Rs.36531
Average amount paid by M&O =36531/2 =Rs.18265.5

26). Answer: C)
Let amount put by Q be A
Let amount put by P be 7A
Let amount time of P be T
Let amount time of Q is 5T
Ratio of P &Q

www.ibpsguide.com | estore.ibpsguide.com | www.sscexamguide.com


235
Complete Quantitative Aptitude Questions

7A*T:A*5T=>7:5
Total received amount =12360/5*12=29664

27). Answer: C)
Let total profit be x
R’s share =2/9*x
Balance share =7/9 *x
Share of T=7/9 *x*5/12=35/108*x
Difference of R &T is,
35/108*X-2/9 *x=6600=>x=64,800
Share of S =7/9*64800*7/12
=Rs.29400.

28). Answer: B)
Profit ratio is L:M:N=1500:2100:900
L:M:N=5:7:3.
Let L,M,N be 5x,7x,3x
L’s receives,5x=54000,x =10800
M’s receives,7 *10800 = Rs.75600

29). Answer: D)
Ratio of shares = 60,000 : 70,000 : 80,000
=6:7:8
A’s share = Rs(28350 *6/21)
= Rs.8100
B’s share = Rs(28350 * 7/21)
= Rs.9450
C’s share = Rs (28350* 8/21)
= Rs. 10,800

30) Answer: B)
Ratio of their capitals = (90000*12):(1,20,000*9): (1,80,000*3)
= 2:2:1
Tonu’s share = Rs(33000*2/5) = Rs.13200
Reetu’s share = Rs(33000*2/5) = Rs.13200
Shalu’s share = Rs(33000*1/5) = Rs.6600

www.ibpsguide.com | estore.ibpsguide.com | www.sscexamguide.com


236
Complete Quantitative Aptitude Questions

31). P, Q & R enter into partnership. P invests 6 times as much as Q and Q invests two- sixth of what R
invests. At the end of the year, the profit earned is Rs.7200. what is the share of Q?
A) 720
B) 550
C) 670
D) 480

32). Meenu invested Rs.38000 in a business. After few months, ruba joined her with Rs.28500. At the end of
the year, the total profit was divided between them in the ratio 2:1. After how many months did ruba joined?
A) 2 months
B) 3 months
C) 4 months
D) 5 months

33). Four persons rented a grassland. P grazed 48 goats for 3 months, Q grazed 20 goats for 5 months, R
grazed 70 goats for 4 months and S grazed 42 goats for 3 months. If p’s share of rent is Rs.1420. Find the
total rent of the field?
A) 6410
B) 6200
C) 6400
D) 6300

34) Suja and Madhavi are partners in a business, Suja invests Rs.42000 for 9 months and Madhavi invests
Rs.54000 for 12 months. Then, out of a profit of 38750 find suja’s profit?
A) Rs.14330
B) Rs.14590
C) Rs.14650
D) Rs.14276

35) X, Y, Z subscribe is Rs.25000 for a business. X subscribes Rs.2000 more than Y and Y Rs.2500 more
than Z. out of a total profit of Rs.17500, X receives
A) Rs.7700
B) Rs.7490
C) Rs.7350
D) Rs7320

www.ibpsguide.com | estore.ibpsguide.com | www.sscexamguide.com


237
Complete Quantitative Aptitude Questions

36) P, Q & R are three partners start a business. Thrice P’s capital is equal to four times Q’s capital and Q’s
capital is 6 times R’s capital. Out of a total profit Rs.35860 at the end of the year, Q’s share is
A) Rs.12334
B) Rs.14344
C) Rs. 21500
D) Rs.17600

37) If 12 (M’s capital) = 18 (N’s capital) = 30 (O’s capital) then out of a profit of Rs.13950, How much amount
O will receive?
A) Rs.2700
B) Rs.2600
C) Rs.2500
D) Rs.2400

38). Mano, kajal, kavin enter into partnership. Mano invests some money at the beginning kajal invests double
the amount for 8 months and kavin invests thrice the amount for 10 months. If the annual profit be Rs.35000
then kavin’s share is approximately
A) Rs. 17567
B) None
C) Rs. 12985
D) Rs. 19274

39). Three active partners A, B, C start a work. Twice of A's capital is equal to 6 times of B's capital is nine
times C's capital. If B's profit is Rs. 7500. Then find the total profit.
a) 30750
b) 30833
c) 45250
d) 32745

40). Shyam and Ram are active partners in a business. Shyam contributes 2/7th received capital of the profit
for 21 month and Ram received 1/5th of the profit. Then how many long Ram invest the money in the
business.
a) 8months.
b) 7months.
c) 3months
d) 1.5 months

www.ibpsguide.com | estore.ibpsguide.com | www.sscexamguide.com


238
Complete Quantitative Aptitude Questions

31). Answer: A)
Let R’s capital = Rs.x
Q’s capital = Rs.2x/6 = x/3
P’s capital = 6x/3 = 2x
Ratio of their capitals = 2x : x/3 : x
=6x:x:3x
=6:1:3
Hence Q’s share = (7200*1/10) = Rs.720

32). Answer: C)
Suppose Ruba joined after x months
Then Ruba’s money was invested for (12-x) months.
38000 *12/28500*(12-x) = 2/1
456000 = 684000 – 57000x
X=4
Hence Ruba joined after 4 months.

33). Answer: A)
Ratio of shares P,Q, R, S = 48*3 : 20 *5 : 70*4 :42*3
= 144 : 100: 280 : 126
= 72:50:140:63
Let total rent be Rs x
Then P’s share = Rs. 72x/325
72x /325 = 1420
X = Rs.6410

34). Answer: D)
Ratio of their shares = (42000*9) : (54000*12)
= 378 : 648
= 7:12
Suja’s share = Rs(38750 * 7/19)
= Rs.14276

35). Answer: C)
Let Z = y, then Y = y+2500, X = y+2500+2000
X= y+4500
So y+y+2500+y+4500 = 25000

www.ibpsguide.com | estore.ibpsguide.com | www.sscexamguide.com


239
Complete Quantitative Aptitude Questions

3y = 18000 => y = 6000


X:Y:Z = 10500: 8500 : 6000
= 105 : 85 : 60
= 21:17 : 12
X’s share = Rs.(17500*21/50)
= Rs.7,350

36). Answer: B)
Let R = c
Q = 6c
P = 24c/3 = 8c
P : Q : R= 8c:6c:c = 8: 6: 1
So Q’s capital = Rs. 35860 * 6/15
= Rs.14344

37). Answer: A)
Let 12 M = 18N = 30 O = y
M = y/12, N = y/18 O = y/30
Therefore M : N: O = y/12 : y/18 :y/30
= 15 : 10 : 6
Hence O’s share = Rs (13950 * 6/31)
= Rs.2700

38). Answer: B)
Let mano’s investment be Rs x
Then Ratio of capitals = (x*12) : (2x:8) : (3x*10)
= 12x : 16x : 30x
= 6:8: 15
Kavin’s share = Rs.(35000*15/29)
= Rs.18103.

39). Answer: B)
The capital of C be y.
The capital of B be 9y.
The capital of A be 54y/2 = 27y
Ratio of A:B:C =27y:9y:y
Total =7500/9*37=Rs 30833.

www.ibpsguide.com | estore.ibpsguide.com | www.sscexamguide.com


240
Complete Quantitative Aptitude Questions

40). Answer: D)
Let total profit =P
Ram’s profit =1/5 P
Shyam profit =4/5 p
Ratio of Shyam & Ram =4/5 p: 1/5
=4:1
Total amount put be X
Shyam put =2/7*p for 21 months.
Ram put =5/7 * p
Ratio of Shyam & Ram =4:1=2/7 *p*21:5/7*p*T
T= 1.5months = Ram amount investments time.

41). X, Y, Z started a business with their investment in the ratio of 4/5: 4/7: 5/9, after 6months. X has
withdrawn half of his investment. Also X will get 30% of profit for being an active partner. At the end of a year
total profit earned was Rs. 54750. Then find the total profit earned by Y and Z?
A) 25200
B) 28600
C) 25010
D) 26867

42). L, M and N started a business in which L invested Rs.15000/- for 2year, M invested Rs 25000/- for 3
years, N invested Rs.35000/- for 4 years. At the end of the profit received by them is Rs.9800/-. What is N’s
share?.
A) 5600
B) 5000
C) 5500
D) 4500

43) Q and K are partners in a Company. Q receives 4/7th of the capital for 9months and K received 1/5th of
the profit,for how long K money used?
A) 3
B) 4
C) 5
D) 6

www.ibpsguide.com | estore.ibpsguide.com | www.sscexamguide.com


241
Complete Quantitative Aptitude Questions

44) Sharuk being the active partner receives 3/5th of profit and the remaining is divided between sanjay and
Karthi in the ratio of 2:1. If the difference between the profit shares of Sharuk and sanjay is Rs.3400. What is
Karthi share in Rs?
A) 1540
B) 1360
C) 1698
D) None

45). X, Y, Z are becomes a partner in their business with X's share receives Rs.117000. If out of a total profit
of Rs 8100,X gets Rs.3900 and Y gets Rs. 2700, Z gets Rs.1500 then total capital of Y and Z is,
A) 36900
B) 146000
C) 126000
D) 247000

46) The amounts of P, Q and R in the ratio 3:4:5 and their spending are in the ratio 4:5:6. If P saves 1/6th of
his income then then savings of P, Q, R are in the ratio
A) 18:11:3.
B) 12:13:4.
C) 4:11:18.
D) None

47). Divide Rs 1728 among P, Q, R in such way that 8 times P's share is 12times Q's share and is 6 times R's
share. How much does each get?
A) Rs. 576,384,768
B) Rs. 588,392,484
C) Rs. 592, 384, 652
D) None of the above

48). The monthly income of M and N is in the ratio 5:3 and their expenses in the ratio 4:2. Both of them save
Rs. 12000 each. Find the monthly income of M
A) 68000
B) 72000
C) 60000
D) 64000

www.ibpsguide.com | estore.ibpsguide.com | www.sscexamguide.com


242
Complete Quantitative Aptitude Questions

49. X, Y, Z are engaged to do a certain piece of work for Rs.8200.Y and Z are to execute 15/26 of the work
together. Amount to be paid to X is
A) 4287
B) 3469
C) 6124
D) 2865

50). Tarani and Ruba invest in a business ratio 7:8. If 24% of the total profit goes to charity and Ruba 's share
is Rs 1506. Then then profit goes to Tarani and Ruba is
A) 1675
B 2688
C) 2823
D) 1495

41). Answer: C)
Let investment of X, Y, Z be 4/5 x,4/7x &5/9 x =>252x, 180x & 175x
Profit ratio of X, Y, Z is
252x*6+126x*6:180x*12 :175x *12
=189:180:175
Active partner X’s amount= 54750*30/100
=Rs.16425
Remaining profit =54750-16425
=Rs.38325
Total profit earned by
Y*Z= 38325*355/544 =Rs. 25010

42). Answer: A)
L = Rs.15000/- per 2 years
M= Rs.25000/- per 3 years
N = Rs.35000/- per 4 years
= 15000 × 2 : 25000 × 3 : 35000 × 4
=30000:75000:140000=30:75:140
=6:15:28
Total = 49 parts -----> Rs.9800/- (profit)
= 1 part ------> Rs.200/-
Then, N's share is
= 28 parts = Rs.200 × 28 parts

www.ibpsguide.com | estore.ibpsguide.com | www.sscexamguide.com


243
Complete Quantitative Aptitude Questions

= Rs.5600

43). Answer: A)
Let total profit gained be A
Profit earned by K =1/5 *A
Profit earned by Q =4/5*A
Ratio of Q & K profit
=4/5 *A:1/5*A=4:1
Total investment be P
Q receives 4/7 th of p for 9 months
K receives 3/7 th of P for Y months.
Ratio of K & Q is,
1/4= (3/7*P*Y)/(4/7 *P*9) =>Y= 3 months =>K

44). Answer: B)
Total profit =Y
Sharuk’s share =3/5 * Y
Balance share =2/5 * Y
Share of sanjay =2/5 *Y*2/3
=4/15 * Y
Difference of sanjay & Sharuk share,
3/5* Y- 4/15* Y =3400 =>Y= Rs 10200
Karthi share =2/5 *10200 *1/3 =1360

45). Answer: C)
Profit ratio is, X:Y:Z=3900: 2700:1500
X:Y:Z =13 :9:5
Then, X,Y,Z capitals are
13x,9x &5x
X’s receives amount is,
13x=117000
=>x=9000
Total capital of Y&Z is,
=9000*14=126000

46). Answer: C)
Income of P,Q,R = 3x:4x:5x

www.ibpsguide.com | estore.ibpsguide.com | www.sscexamguide.com


244
Complete Quantitative Aptitude Questions

Expense of 4:5:6=4y:5y:6y.
Savings [income –expenditure]
=3x-4y :4x-5y :5x-6y------->1
Given 3x-4y=x/6
y=17x/24
Sub if (1) is,
=3x-4(17x/24) :4x-5(17x/24) :5x-6(17x/24)
=8x:22x:36x
= 4:11:18

47). Answer: A)
Given, 8P=12Q=6R
4P=6Q=3R=>4P=6Q=>6/4Q=>P=3/2Q
6Q=3R R=6Q/3=2Q
Then, P+Q+R=1728
3/2Q+Q+2Q=1728
3Q+2Q+4Q=3456
9Q=3456
Q=384
P=3/2Q =3/2*384=576,R=2Q=768
A:B:C=576:384:768

48). Answer: C)
Income ratio of M=5x
Income ratio of N=3x
Savings of M=12000
Savings of N=12000
Expense of M=5x-12000
Expense of N =3x-12000
Expense order in ration 4:2
5x-12000:3x-12000=4:2
X=12000
M’s income =5x=5*12000
=60000

49). Answer: B)
Y and Z are execute 15/26 of week

www.ibpsguide.com | estore.ibpsguide.com | www.sscexamguide.com


245
Complete Quantitative Aptitude Questions

Balance =1-15/26
11/26 of work completed by X
X’s share of revenue
=11/26 *8200
=Rs 3469

50). Answer: C)
Let total profit =X
Balance profit after =76%of X
Ruba’s share =76/100 *x*8/15=1506
X=Rs.3715
Then the profit =3715*76/100=Rs2823

51) X, Y, Z jointly engaged in a business. It was agreed that X would invest Rs. 14500 for 4 months,Y Rs
19000 for 7 months and Z Rs 25000 for 6 months. X wants to be a working partner and he was to receive
14% of the profit and total profit earned by X was Rs 43929. Calculate share of Z
A) 55750
B) 58472
C) 67500
D) 65250

52) Agalya and Bhairavi enter into a partnership and Agalya invests Rs 14000 in the partnership. At the end
of 3months she invests Rs 5000. At the end of another 4 months, she invests the another Rs 6000. If Bhairavi
invests a certain sum in the partnership at the beginning of the year and leaves it and receives Rs 9720 as
her share if the total of Rs 31320 a year, how much did Bhairavi invest in the company?
A) 37850
B) 42500
C) 41400
D) 35478

53). Among 3 Persons P,Q,R the profit Of Q is equal to one- third of the difference between the profit of R
and triple of P. If at the end of the year, total profit is Rs 27000. Then find the profit of R?
A) 13000
B) 17850
C) 14500
D) 20250

www.ibpsguide.com | estore.ibpsguide.com | www.sscexamguide.com


246
Complete Quantitative Aptitude Questions

54). In a business, X,Y and Z inversed Rs.20,000/-,Rs.30,000/- and Rs.40,000/- respectively. After 1 year X
adds Rs.10000/- to the initial investment.Y and Z withdrew Rs.10000/- and Rs.15,000/- respectively. After 2
years the total profit is Rs.9900/-. What is X’s share?.
A) 3000
B) 2700
C) 3600
D) 4000

55). Two persons Ashok and Amit enter into a business. Ashok invests Rs 12500 in the first time and after
6months withdraws Rs 7000. Amit withdraws Rs 3000 after 8 months. Both have equal profit at the end of the
year. What is the amount invested by Amit initially?
A) 15000
B) 20000
C) 10000
D) 16000

56). Jaya and sona invested amount of Rs 10000 and Rs 5000 respectively in their business. What
percentage of the share of the profit that should be given to Jaya such that ratio of income is equal for both at
the end of year?
A) 5
B) 7
C) 4
D) 6

57). Murugan, Anwar, Joseph started a business by a share of 7:8:9. The time for which they invested was
4:2:3. The difference in the profit of Murugan and Joseph is what % of Anwar?
A) 7.5
B) 8.2
C) 6.4
D) 6.25

58). P, Q, R enter in business partnership. P invests some money at the beginning, Q invests thrice times the
amount after 4 months and R invests 6 times the amount after 8 months. If the annual profit be Rs 61260, R's
share is
A) 41700
B) 45870
C) 47860

www.ibpsguide.com | estore.ibpsguide.com | www.sscexamguide.com


247
Complete Quantitative Aptitude Questions

D) 40840

59). Among 3 persons, P, Q, R the profit of R is equal to two-fifth of the sum of one-third profit of P and one-
third of Q. If at the end of the year total profit is Rs 12750, then find the average profit of P and Q?
A) 4750
B) 5625
C) 7425
D) 6520

60). Durga invested Rs X for 5 months and Y for remaining time. Moushsree invested Rs Y for 9.months and
X for the remaining time. At the end of the year profits are in the ratio 4:5. Then find what % of X is Y?
A) 6.2 %
B) 7.69%
C) 5.25%
D) 3.2%

51). Answer: C)
Ratio of X, Y, Z profit is 14500*4:19000*7:25000*6=58:133:150
Total profit be’ a’
X receives 14% of a as profit for active partner
Remaining profit=86% of a
X’s total profit=[86/100*a*58/341]+14/100*a=43929
9762/34100*a=43929 => a= Rs.153450
Share of Z =153450/341*150 =Rs 67500

52). Answer: C)
Let investment of Bhairavi be X rupees for 12 months
Agalya’s investment for 12 months =14000*3+5000*4+6000*5
=Rs 92000
Share received by Bhairavi = Rs 9720
Share received by Agalya =Rs 21600
Ratio of their profit 92000/X=21600/9720
X=Rs 41400

53). Answer: D)
Q=1/3(R-3P)
3Q=R-3P

www.ibpsguide.com | estore.ibpsguide.com | www.sscexamguide.com


248
Complete Quantitative Aptitude Questions

R=3[P+Q]
R/(P+Q) =3/1
R/(P+Q+R)=3/4
R/27000 =3/4
R=Rs.20250

54). Answer: A)
X's Share = Rs.20000/- per first 1 year, Next year 10000/- added
= Rs.20000/- × 1 + Rs.30000/- × 1 = Rs.50000/-
Y's Share = Rs.30000/- per first 1 year, Next year 10000/- reduced
= Rs.30000/- × 1 + Rs.20000/- × 1 = Rs.50000/-
Z's Share = Rs.40000/- per first 1 year, Next year 15000/- reduced
= Rs.40000/- × 1 + Rs.25000/- × 1 = Rs.65000
X's part = 50 : Y's part = 50 : Z's part = 65
===> 10 : 10 : 13
Total 33 parts -----> Rs.9900/-
1 part------> Rs300/-
Then, X's share is
10 parts = Rs.300/- × 10 parts = Rs.3000/-

55). Answer: C)
Ashok’s investment =12500*6+5500*6=108000
Amit’s investment=x*8+(x-3000)*4
108000=8x+4x-12000=>X=Rs 10000
Amount invest by Amit is Rs 10,000

56). Answer: C)
Ratio of invest of jaya & sona =10000:5000 =2:1
Income of both at end of year is equal, then
Income is divided into 50 parts.
Income be 100%, then 1st part =25 ,2 nd part=25,
Percentage=100/25=4%

57). Answer: D)
Ratio of investment =7:8:9
Ratio of time=4:2:3
Ratio of profit =28:16:27

www.ibpsguide.com | estore.ibpsguide.com | www.sscexamguide.com


249
Complete Quantitative Aptitude Questions

= (Joseph - Murugan)/ Anwar =(28-27)/16*100


=6.25%

58). Answer: D)
Let initial investment of P be X
Q’s initial investment after 4 month is 3 X
R’s initial investment after 8 months is 6X
Profit ratio of P,Q&R is,
X*12:3x*4:6x*8=12:12:48=1:1:4
R’s share =61260*4/6=Rs.40840

59). Answer: B)
R=2/5(1/3 P+1/3 Q)
15R=2(P+Q)
R/(P+Q)= 2/15
Take R=2x, P+Q=15x
Total profit =15x+2x =12750
=>X=Rs.750
Average profit of P&Q =750/2 *15=5625

60). Answer: B)
Durga investment =5X+7Y
Moushsree investment =9Y+3X
(5X+7Y)/(9Y+3X)=4/5
X/Y=1/13 *100=7.69%

61). Bharathi started a business with Rs.50000 and after 4 months, sathya joined him with Rs.120000.
Bharathi received Rs.116000 including 20% of profit as commission for managing the business. What amount
did sathya receive?
A) 112484.
B) 145826.
C) 134972.
D) 112540

62) X, Y, Z started a business by a share of 8:6:9. The time for which they invested was 6:7:8. The difference
in the profit of X and Z is what % of difference in the profit of Zand Y?
A) 65

www.ibpsguide.com | estore.ibpsguide.com | www.sscexamguide.com


250
Complete Quantitative Aptitude Questions

B) 80
C) 55
D) 75

63) P,Q, R enter into a partnership.P initially invests Rs 54 lakh and withdraws Rs 18 lakhs after 4 years. Q
initially Rs 72 lakh ans adds another Rs 18 lakhs after 6 years and R invests Rs 108 lakh and adds another
Rs 18 lakh few years. At the end of 10 years profit Of R is equal to sum of profit of P and Q. Then for how
many years did R invests Rs 126 lakh per annum
A) 5
B) 7
C) 8
D) 11

64) Saranya began a business with Rs 84000 and he joined by shalu with Rs 72000. At the end of the year
their profit ratio 14:4 then after how many months does shalu joins the business?
A) 5
B) 6
C) 8
D) 9

65). Ajay and vijay entered into business with capital in ratio of 5:7.after 5 months Ajay withdraws 4/7 th of his
capital and vijay withdraws 2/7th of his capital. At the end of 8months there was a profit of Rs 4470. Then find
the profit
A) 3678.
B) 2000.
C) 4000.
D) 1500

66) P , Q, R hired a van for Rs. 640 and used it for 11,12,13 hours respectively. Hire changes paid by Q
were.
A) 213
B) 312
C) 123
D) 412

67) X, Y & Z rent a pasture. X puts 20 cows for 5 months , Y puts 24 cows for 7 months and Z puts 30 cows
for 3 months for grazing. If the rent of the pasture is Rs. 150, how much must z pay as his share of rent?

www.ibpsguide.com | estore.ibpsguide.com | www.sscexamguide.com


251
Complete Quantitative Aptitude Questions

A) 32
B) 39.65
C) 37.70
D) 35.20

68). In a company M and O invested amount in the ratio 3 : 2 whereas the ratio between amounts invested by
M and N was 4 : 3. If Rs.1,80,000 was their profit, how much did N receive?(calculate approximate value)
A) 75432
B) 74483
C) 76854
D) 76443

69). Ananya and bheeman started a partnership business investing some amount in the ratio of 3:7. Chandru
joined them after 6 months with an equal amount of bheeman. In what proportion should thr profit at the end
of 1 year be distributed among ananya, bheeman, and chandru?
A) 5:6:7
B) 7 :12;4
C) 6:11:5
D) 6:14:7

70). Dhivya, Dinesh and Deepi enter into a partnership and their shares are in the ratio 1/3 :1/4 :1/5. After 2
months, dhivya withdraws half of her capital and after 10 months, a profit of Rs. 462 is divided among them,
what is Deepi’s share?
A) 123.56
B) 165.45
C) 143.3
D) 146.43

61). Answer: A)
Ratio of the profits = 50000 × 12 : 120000 × 8
= 5:8
Let the total profit = x.
Then Bharathi received 20x/100 = x/5 as commission for managing the business
Remaining profit = x- x/5 = 4x/5 which is shared in the ration 5:8
Bharathi 's share = x/5 + (4x/5) × (5/13) = 116000
=x/5+4x/13=116000
=33x/65=116000

www.ibpsguide.com | estore.ibpsguide.com | www.sscexamguide.com


252
Complete Quantitative Aptitude Questions

=> x = 228484
Sathya 's share = 228484 - 116000 = 112484

62). Answer: B)
Ratio of investment =8:6:9
Ratio of time =6:7:8
Ratio of X,Y,Z=48:42:72
=8:7:12
(X-Z)/(Z-Y) = 4/5 *100 =80%

63). Answer: C)
Ratio of their profit P, Q, R is
54*4+36*6:72*6+90*4:108*x+126*(10-x)
=12:22:35 - 0.5x
Profit of R=profit of P +profit of Q
35-0.5x=12+22
X=2 year
The number of years for R invests Rs 126 lakh
=10-2=8years

64). Answer: C)
Saranya invests Rs 84000 for 12 months
shalu invests Rs 72000 for X months
Ratio of both profits,
Saranya: shalu =84000*12:72000*x
=14:X
14/4=14/X => X = 4months
=12-4=>8months

65). Answer: B)
Profit ratio of Ajay & vijay,
5x*5+3x*3 :7x*5+5x*3
=34:50=17:25
Profit of vijay =3360/42 *25
=Rs 2000

66). Answer: A)

www.ibpsguide.com | estore.ibpsguide.com | www.sscexamguide.com


253
Complete Quantitative Aptitude Questions

P : Q : R = 11 : 12 : 13
Hire changes paid by Q = Rs.(640 * 12/36)
= Rs. 7680 / 36
= Rs.213 (approx.)

67). Answer: C)
X : Y : Z = 20* 5 : 24* 7 : 30 * 3
= 100 : 168 : 90
=50 : 84 : 45
Therefore Z’s rent = Rs (150 * 45 / 179)
= Rs.(6750/179)
Rs.37.70

68). Answer: B)
M : O = 3:2
O: M= 2:3 = 8:12
M:N = 4 :3 =12:9
O:M:N = 8:12:9
Therefore N’s share = Rs (1,80,000 * 12/29)
=Rs(2160000/29)
= Rs.74483(approx.)

69). Answer: D)
Let the investments of ananya,bheeman be 3x and 5x
Ananya : bheema : chandru = (3x*12 ) : (7x *12 ) : (7x *6)
=36x : 84x :42x
=6 : 14 :7

70). Answer: C)
Ratio of initial investments = 1/3 : 1/4 : 1/5 = 20 :15 :12
Let their initial investments be 20x, 15x and 12x respectively
Dhivya: Dinesh :Deepi = (20x *2 +10x *10) : (15x *12) : (12x*12)
=140x : 180x ;144x
= 70:90:72 = 35 : 45 :36
Deepi ‘s share = Rs.462 * 36/116
= Rs(16632/116)
=Rs.143.3

www.ibpsguide.com | estore.ibpsguide.com | www.sscexamguide.com


254
Complete Quantitative Aptitude Questions

71). Aakash and Aravind are partners in a business. Aakash contributes 1/5 of the capital of 16 months and
aravind received ¾ of the profit. For how long Aravind money was used?
A) 11 months
B) 12 months
C) 13 months
D) 14 months

72) Two friends M & N started a business Investing in the ratio of 6 :7 . O joined then after 6 months investing
an amount equal to that of N. at the end of the year 30% profit was equal to 105000. What was the amount
invested by O?
A) 148484
B) 154326
C) 146738
D) 145783

73) Anu and Thanu start a business. Anu invests Rs.8000 for 8 months and Thanu remains in the business
for 4 months out of total profit, Thanu claims 3/7 of the profit. How much money was contributed by Thanu?
A) Rs6000
B) Rs.7000
C) Rs.8000
D) Rs.9000

74. Three partners shared the profit in a business in the ratio 7 :8 :9. They had partnered for 16 months, 10
months and 9 months respectively. What was the ratio of their investments?
A) 2286:5599:7643
B) 2346:76543:8754
C) 5643;1254 :2345
D) 2205 :4032:5032

75). Keerthi and Kavi invest in a business in the ratio 7:5. If 10% of the total profit goes to charity and
Keerthi’s share is Rs.2625 the total profit is.
A) 5000
B) 2000
c) 3000
D) 4000

www.ibpsguide.com | estore.ibpsguide.com | www.sscexamguide.com


255
Complete Quantitative Aptitude Questions

76) Three friends started a Company, let there names are P, Qand R. What profit Q will get, if,
1. R invested Rs. 4000 for nine months, his profit was 3/2 times that of Q's and his investment was four times
that of P.
2. P and Q invested for one year in the proportion 1 : 2 respectively.
3. The three together got Rs. 500 as profit at the year end.
A) Only 1 and 3
B) Only 1 and 2
C) All 1, 2 and 3
D) None of above

77). X, Yand Z enter into a partnership by investing in the ratio of 6 : 4: 8. After 1 year, Y invests another Rs.
540,000 and Z, at the end of 2 years, also invests Rs.540,000. At the end of three years, profit is are shared
in the ratio of 6 : 8 : 10. Find initial investment of Z.
A) 7,20,000
B) 8,80,000
C) 6,50,000
D) 5,50,000.

78) P, Q and R jointly thought of engaging themselves in a business venture. It was agreed that P would
invest Rs. 3250 for 6 months, Q, Rs. 4200 for 5 months and R, Rs. 5,000 for 3 months. P wants to be the
working member for which, he was to receive 10% of the profits. The profit earned was Rs. 3700. Calculate
the share of Q in the profit.
A) Rs. 1800
B) Rs. 1260
C) Rs. 1580
D) Rs. 1940

79). A, B and C are 3 partners in a business. Their investments are respectively Rs 2000, Rs 4,000 and Rs
3,000. A gets 30% of total profit for managing the business. The remaining profit is divided among them in
the ratio of their investments. At the end of the year, the profit of Ais Rs 1100 less than the sum of the profit of
B and C. What amount of income will C get?
A) Rs 2100.75
B) Rs 2887.5
C) Rs 2705.75
D) Rs 2546.25

www.ibpsguide.com | estore.ibpsguide.com | www.sscexamguide.com


256
Complete Quantitative Aptitude Questions

80). P started a business in 1990 by investing Rs.25,000. She invested Rs. 10,000 as additional amount in
1991 and her friend Q joined her with an amount of Rs.35,000. P invested another Rs. 10,000 in 1992 and R
joined them with Rs. 35,000. At the end of these 3 years, they earned a profit of Rs. 150,000. Find Q's share?
a) Rs.50,000
b) Rs.65,000
c) Rs.75,000
d) Rs.15,000

71). Answer: B)
Let the total profit be Rs z
Then, Aravind share = 3z/4
Aakash share = z – 3z/4
= z/4
Aakash : Aravind = Z/4 : 3z/4
= 1:3
Let the total capital be Rs x
Suppose Aravind’s money was used for y months.then,
1/5x * 16/4/5x * y = 1/3
3 (16x/5) = 4x/5 * y
48 = 4y
Y = 48/4 = 12 months.

72). Answer: A)
Let the total profit be x
Then 30% of x = 105000
X = 105000 * 100/30
X=350000
Let the capitals of M,N,O be Rs. 6x ,7x and 7x respectively. Then,
(6x*12) + (7x *12) +(7x *6) = 350000*12
72x +84x +42x = 4200000
198x = 4200000
X=21212
O’s investment = Rs.148484

73). Answer: D)
Let the total profit be Rs.x
Then thanu = 3x/7

www.ibpsguide.com | estore.ibpsguide.com | www.sscexamguide.com


257
Complete Quantitative Aptitude Questions

Anu = (x – 3x/7) = 4x/7


So, Anu : thanu = 4x/7 : 3x/7
= 4:3
Lets thanu’s capital be y. then,
8000*8/ (y *4) = 4/3
24000 * 24 = 4y * 16
Y = 24000*24/(16*4)
Y = Rs.9000

74). Answer: D)
Let their investments be Rs.x for 16 months, y for 10 months , Z for 9 months
Then 16x : 10y :9z = 7:8:9
Now 16x / 10y = 7/8
128x = 70y
Y=64/35x
16x/9z = 7/9
144x = 63z
Z = 144x/63
X:Y:Z = x:64x/35 :144x/63
X:Y:Z = 2205:4032:5032.

75). Answer: A)
Let the total profit be Rs.100
After playing to charity, keethi’s share = Rs(90 * 7/12)
=Rs.52.5
If keerthi’s sghare is Rs.52.5 total profit = Rs 100
If keerthi’s share is Rs. 2625
Total profit = (100/52.5 *2625)
=Rs.5000

76). Answer: C)

1 and 2 will give :


R = Rs. (4000 x 9) for 1 year = Rs. 36000 for 1 year.
P = Rs. (1/4 * 4000 * 12 ) for 1 year = Rs 12000 for 1 year
Q = Rs 24000 for one year
R:P:Q = 36000 : 12000 : 24000 = 3 : 1 : 2

www.ibpsguide.com | estore.ibpsguide.com | www.sscexamguide.com


258
Complete Quantitative Aptitude Questions

From 3, we will get total Profit = 500


Now from the ratio and total profit we can get Share of R.
R share will be = 500 * 2/6 = 166.66

77). Answer: A)
Let the initial investments of X, Yand Z be Rs. 6x, Rs. 4x and Rs. 8x respectively.
Then, (6x * 36) : [(4x * 12) + (4x + 540000) x 24] : [(8x * 24) + (8x +540000) x 12]=6:8:10
216x : (144x + 12960000) : (288x + 6480000) = 6 : 8 : 10
216x /(144x+12960000)=3/4
=> 864x = 432x + 38880000
=> 432x = 19440000
x=90000
Z's initial investment = 8x = Rs. 7,20,000.

78). Answer: B)
For managing, P received = 10% of Rs. 3700 = Rs. 370.
Balance = Rs. (3700 - 370) = Rs. 3330.
Ratio of their investments = (3250 x 6) : (4200 x 5) : (5000 x 3)
= 19500 : 21000 : 15000
= 13 : 14 : 10
Q's share = Rs. 3330x 14 / 37
= Rs. 1260

79). Answer: B)
The ratio of profit of A, B and C is 2000:4000:3000=2:4:3.
Let the annual profit be P.
Then, A will get 0.3p for managing the business.
And, remaining 0.7p will be distributed in the ratio of their investment.
So, from the remaining investment, A will get,
=2/(2+4+3)×0.7p=2/9×0.7p
B gets=4/(2+4+3)×0.7p=4/9×0.7p
and c Gets=3/(2+4+3)×0.7p=3/9×0.7p
A's total profit=0.3p+(2/9)×0.7p
Given, at the end of the year, the profit of A is Rs 1100 less than the sum of the profit of B and C
⇒ 4/9×0.7p+3/9×0.7p−1100=0.3p+2/9×0.7p
⇒ 7/9×0.7p−2/9×0.7p−0.3p=1100
⇒ p=12,375

www.ibpsguide.com | estore.ibpsguide.com | www.sscexamguide.com


259
Complete Quantitative Aptitude Questions

So, C's share=3/9×0.7p=Rs 2887.5

80). Answer: A)
P invested
Rs.25,000 for 12 months, Rs.(25000 + 10000) for 12 months, , Rs.(25000 + 10000 + 10000) for 12 months.
i.e., P invested Rs.25,000 for 12 months, Rs.35000 for 12 months Rs.45000 for 12 months.
Q invested Rs. 35000 for 2 years;
i.e., Rs.35000 for 24 months
And, R invested Rs.35000 for 1 year; i.e., Rs. 35000 for 12 months.
Their investing ratio: P : Q : R
= (25,000 x 12 + 35000 x 12 + 45000 x 12):(35000 x 24):(35000 x 12)
= 252:168:84
= 3:2:1
Total profit for 3 years = Rs.1,50,000
them, Q's share = Rs.(1,50,000 x 2 /(3+2+1))
= Rs.(1,50,000 x 2/6) = Rs.50,000

11. PIPES AND CISTERN


1) Two faucet can fill a tank in 12hours and 16 hours .While a third faucet empties the full tank in 24 hours.If
all the three faucet are operate simultaneously, In how much time will the tank be filled ?
A) 4 hours 12mins
B) 4hours 48min
C) 9hours 36mins
D) 5hours 48min

2) A tube can fill a cistern in 18hrs.After half the cistern is filled, three more similar tubes are opened. What is
the total time taken to fill the cistern completely ?
A) 9hrs 52min
B) 10hrs 15 min
C) 9hrs 45 min
D) 10hrs 30min

3) Two taps can fill a tank in 4 hours and 5 hours .If two taps are operate simultaneously, In how much time
will the tank be filled ?
A) 4hrs 18min
B) 10min 12min
C) 2hrs 13min

www.ibpsguide.com | estore.ibpsguide.com | www.sscexamguide.com


260
Complete Quantitative Aptitude Questions

D) 12hrs 10min

4) Two faucet p and q can fill a tank in 10 minutes and 20 minutes.If both faucet are opened simultaneously ,
after how much time should q be closed so that the tank is full in 9 minutes ?
A) 2 min
B) 9 min
C) 4 min
D) 7min

5) A pipe can fill the tank in 12 hours.Because of a leak in the tank it took 16(1/2) hours to fill the tank.If the
tank is full,how much time will the leak take to empty it?
A) 18hrs51mins
B) 18hrs 20min
C) 18hrs 40min
D) 18hrs 45min
E) None of these

6) One tap can fill a tank thrice as fast as another tap. If together the two taps can fill the tank in 12 minutes,
then the slower tap alone will be able to fill the tank in
A) 30min
B) 33min
C) 32min
D) 35min

7) Two pipes can fill a dumper in 8 hours and 24 hours .If two pipes are operate simultaneously, In how much
time will the dumper be filled ?
A) 3 hrs
B) 4 hrs
C) 8 hrs
D) 6 hrs

8) A tube can fill a tank completely in 18 hours. After half the tank is filled , one more similar tube is opened.
What is the total time taken to fill the tank completely ?
A) 14hrs 20min
B) 13hrs 30min
C) 13hrs 10min
D) 14hrs 30min

www.ibpsguide.com | estore.ibpsguide.com | www.sscexamguide.com


261
Complete Quantitative Aptitude Questions

9) A tap can fill a cistern in 11hours, but due to a leakage it took 13hours to fill the cistern. If the cistern is full,
in what time will the cistern become empty due to leakage ?
A) 73.30hrs
B) 77.50hrs
C) 71.5 hrs
D) 72.30hrs
E) None of these

10) Two faucet P and Q can fill a tank in 10 min. and 20 min. respectively. A water faucet R can empty the
tank in 10 min. First P and Q are opened. After 3 min, R is also opened. In how much time, the tank is full?
A) 17m
B) 15m
C) 11m
D) 19m

1) C
1/12+1/16 – 1/24 = (4+3-2)/48
=>5/48
Time taken to fill the tank = 48/5
= 9hours 36min

2) D
Time taken to fill the half cistern = 18/2 = 9 hrs
Remaining part = 1/2
Time taken to fill the remaining part = [1/(4×1/12)] ×(1/2) = 3/2hrs
3/2 hrs = 90 min = 1hr 30mints
Total time = 9hrs+90 min = 10hrs 30mins

3) C
1/(A+B) =1/4 + 1/5
= (5+4)/20
= 9/20
=>20/9=2hrs 13min to filled the tank
4) A
p fill the tank in 1 minute (10×2 = 20) = 2 units
q fill the tank in 1 minute (20×1 = 20) = 1units

www.ibpsguide.com | estore.ibpsguide.com | www.sscexamguide.com


262
Complete Quantitative Aptitude Questions

For 9 min(p) = 9×2 = 18 units


Remaining = 20 – 18 = 2 units
Time for q be closed so that the tank is full in 9 minutes = 2/1 = 2 min

5) A
=>1/12-2/33 = 11-4/132.
==>7/132
=>132/7 hrs
=>132/7 hrs=>1131 mins =>18hrs 51min

6) C
(1/x)+(1/3x)=(1/12)
3+1/3x = 4/3x = 1/12
3x = 48
x=32mins

7) D
(A+B) =1/8 + 1/24
=> (3+1)/24
=> 4/24 =>1/6
6 hrs to filled the dumper

8) C
A tube can fill the half tank in 9hrs
Now another similar tube opened
1/18+1/18 = 2/18 = 1/9
Remaining half tank filled in 4.5hrs
Total time = 9+4.5 =13.5 = 13hrs 30min

9) C
1/11-1/13 = 13-11/143
=>2/143
=>1/71.5
=>71.5hours

10) C
Part filled in 3 min. = 3*((1/10)+(1/20))

www.ibpsguide.com | estore.ibpsguide.com | www.sscexamguide.com


263
Complete Quantitative Aptitude Questions

= 3 * (2+1/20)
=>3(3/20) = 9/20
Remaining part=(1-(9/20))=(11/20).
Net part filled in 1min. when P,Q and R are opened=(1/10)+(1/20)-(1/10)=(1/20).
Now,(1/20) part is filled in one minute.
(11/20) part is filled in = (20*(11/20))=11minutes.

11) Two filling tap can fill a dumper in 14 and 28 min. respectively and when the waste tap is open, they can
together fill it in 35 min. The waste pipe can empty the full dumper in –
A) 152/9 hrs.
B) 137/7 hrs.
C) 60/9 hrs.
D) 140/11 hrs.

12) Three taps P,Q,R can fill a bunker in 3 hrs. After working at it together for 1 hr, R is closed and P,Q can fill
the remaining part in 3 hrs. The number of hrs taken by R alone to fill the bunker.
A) 9hrs
B) 7hrs
C) 11hrs
D) 5hrs

13) two pipes X and Y can fill a cistern in 18 hrs and 24 hrs respectively. If both the pumps are opened
simultaneously. How much time will be taken to fill the cistern?
A) 10hrs 50mints
B) 10 hrs 5mints
C) 10hrs 17mints
D) 10hrs 44mints

14) A tank has two faucet which fill it in 15 mins and 18 mins respectively. There is also a waste faucet in the
tank. When all the 3 are opened the empty tank is full in 25 mins. How long the waste faucet take to empty
the full tank?
A) 450/17 mins
B) 450/28 mins
C) 450/37 mins
D) 450/22 mins

www.ibpsguide.com | estore.ibpsguide.com | www.sscexamguide.com


264
Complete Quantitative Aptitude Questions

15) An electric pump can fill a cistern in 6 hrs. Because of a leak in the cistern it took 7 hrs to fill the cistern. If
the cistern is full, how much time will the leak take to empty it?
A) 42hrs
B) 24hrs
C) 32hrs
D) 16hrs

16) Two tap can fill a dumper in 7 hrs and 8 hrs respectively. The taps are opened simultaneously and it is
found that due to leakage in the bottom it took 16 mins more to fill the dumper. When the dumper is full, in
what time will the leak empty it?
A) 56hrs
B) 56hrs
C) 46hrs
D) 36hrs

17) Two tube A and B can fill a cistern in 48 mins and 64 mins. If both the tubes are opened simultaneously,
after how much time B should be closed so that the cistern is full in 36 minutes?
A) 20 mins
B) 16mins
C) 22mins
D) 14mins

18) A tank can be filled by a tap in 8 hrs while it can be emptied by another tap in 18 hrs. if both the taps are
opened simultaneously, then after how much time will the cistern get filled?
A) 14hrs 24mins
B) 14 hrs 12 mins
C) 14hrs 56 mins
D) 14hrs 30mins

19) A pump can fill a cistern in 12 hrs. After half the cistern is filled, 3 more similar pumps are opened. What is
the total time taken to fill the cistern completely?
A) 8 hrs
B) 4 hrs
C) 2hrs
D) 7.5 hrs

www.ibpsguide.com | estore.ibpsguide.com | www.sscexamguide.com


265
Complete Quantitative Aptitude Questions

20) Tap 1 can fill a tank in 10 hrs , tap 2 in 20 hrs and tap 3 in 60 hrs. if all the taps are open, in how many
hour will the tank be filled?
A) 3 hrs
B) 5 hrs
C) 7hrs
D) 6 hrs

11) D
1/14 + 1/28 – 1/35
=>10 + 5 – 4/140
=>11/140
=>140/11 => hence answer is 140/11 hours.

12) A
Part filled in 1 hr = 1/3
Remaining part = 1 – 1/3 = 2/3
(P + Q)’s 3 hours work = 2/3
(P+Q)’s 1 hour work = 2/9
R’s 1 hr work = [(P+Q+R)’s 1 hr work – (P+Q)’s 1 hr work]
= 1/3 – 2/9 = 3-2/9 = 1/9
R alone can fill the tank in 9 hours.

13) C
Part filled by x in 1hour = 1/18
Part filled by y in 1 hour = 1/24
Part filled by (x+y) in 1 hour = 1/18 + 1/24
=>4+3/72
=>7/72
The cistern will be full in 72/7 hrs = 10 hrs 17mins.

14) C
Work done by the waste faucet in 1 minute
=>1/25 – (1/15 + 1/18)
=>1/25 – (6+5/90) = 1/25 – 11/90
=> - 37/450
Here negative sign means emptying
Therefore waste faucet will empty the full tank in 450/37 mins.

www.ibpsguide.com | estore.ibpsguide.com | www.sscexamguide.com


266
Complete Quantitative Aptitude Questions

15) A
Work done by the leak in 1 hr = [1/6 – 1/7]
= 7-6/42
=1/42
Therefore the leak will empty the cistern in 42 hrs.

16) A
Work done by 2 taps in 1 hr = (1/7 + 1/8) = 15/56
Time taken by these taps to fill the dumper = 56/15 hrs
=3 hrs 44 mins
Due to leakage, time taken = 3 hrs +44mins+16 mins
=4hrs
Work done by (2 pipes + leak) in 1 hr = ¼
Work done by the leak in 1 hr = (15/56 – ¼)
=15 – 4/56
=1/56
Leak will empty the full cistern in 56 hrs.

17) B
Let B be closed after ‘x’ mins.
Then part filled by (A+B) in x mins + part filled by A in (36 – x) mins = 1
Therefore x(1/48 + 1/64) + (36 – x)* 1/48 = 1
7x/192 + 36 – x/48 = 1
7x + 4(36-x)/192 = 1
7x+144 – 4x = 192
3x = 48 =>x = 16
Here B must be closed after 16 mins.

18) A
Net part filled in 1hr = (1/8 – 1/18)
= 9 – 4/72
= 5/72
Therefore the tank will be filled in 72/5 hrs i.e., 14 hrs 24mins

19) D
Time taken by 1 pump to fill tha half tank = 6 hrs

www.ibpsguide.com | estore.ibpsguide.com | www.sscexamguide.com


267
Complete Quantitative Aptitude Questions

Part filled by the 4 pumps in 1 hr = 4 * 1/12


=1/12 =>1/3
Remaining part = (1 – 1/2)= 1/2
Therefore 1/3 :1/2 :: 1:x
X=1.5 hrs
So total time takem = 6hrs + 1.5 hrs = 7.5 hrs.

20) D
Net part filled in 1 hr = 1/10 + 1/20 + 1/60
= 6 + 3 + 1/60
= 10/60
= 1/6
Therefore all the 3 taps together fill tank in 6 hrs.

21) A water cistern is two-fifth full. Tap A can fill a cistern in 20 mins and tap B can empty it in 12mins. If both
the taps are open, how long will take to empty or fill the cistern completey?
A) 10 mins
B) 12mins
C) 14mins
D) 16 mins

22) 3 faucet A,B,C can fill a cistern from empty to full in 45mins, 35mins, and 25 mins respectively. When the
cistern is empty all the 3 faucets are opened. A,B,c discharge chemical solutions A,B,C respectively. What is
the proportion of solution C in the liquid in the tank after 5 mins?
A) 63/143
B) 72/151
c) 67/173
D) 81/167

23) Two pipe P & Q fill a tab in 5 hrs and 20 hrs respectively. If both the pipes are open then due to the
leakage, it took 30mins more to fill the tab. If the tab is full, how long will it take for the leakage alone to empty
the tank?
A) 28
B) 24
C) 36
D) 12

www.ibpsguide.com | estore.ibpsguide.com | www.sscexamguide.com


268
Complete Quantitative Aptitude Questions

24) One tap can fill a cistern four times as fast as another tap. If together the two taps can fill the cistern in 48
mins then the slower tap alone will be able to fill the cisterns.
A) 1hr
B) 2hr
C) 3hr
D) 4hr

25) A dumper is filled in 15hrs by 3 tubes P,Q,and R. the tube R is thrice as fast as Q and Q is thrice as fast
as P. How much time will tube Q alone take to fill the tank?
A) 195hrs
B) 190hrs
C) 185hrs
D) 180hrs

26) A dumper is normally filled in 18 hours but takes 6 hours longer to fill because of a leak in the bottom of
the dumper. If the dumper is full the leak will empty it in how many hours?
A) 76 hours
B) 78 hours
C) 72 hours
D) 74 hours

27) 24 pumps are connected to a tank. Some of them are inlet pumps and the others are outlet pumps. Each
of the inlet pumps an fill the tank in 16hours and each of the outlet pumps can empty the tank completely in
12hours. If all the pumps are kept open, the empty tank gets filled in 48 hours. How many inlet pumps are
there?
A) 16
B) 18
C) 17
D) 14

28) A barrier has four inlets A, B, C and D. The barrier can be filled in 24 minutes through the first three inlets
and it can be filled in 30 minutes through the second, the third and fourth inlet also it can be filled through the
first and the fourth inlet in 40 minutes. How much time required to fill up the barrier by all the four inlets?
A) 10 mins
B) 15 mins
C) 20 mins
D) 25 mins

www.ibpsguide.com | estore.ibpsguide.com | www.sscexamguide.com


269
Complete Quantitative Aptitude Questions

E) None of the Above

29) A tank can be filled by an inlet tap at the rate of 8 litres per minute. A leak in the bottom of a tank can
empty the full tank in 16 hours. When the tank is full, the inlet is opened and due to the leak, the tank is
empty in 80 hours. How many litres does the tank hold?
A) 8000 litre
B) 9560 litre
C) 8525 litre
D) 9600 litre

30) Two faucet A and Bcan fill a cistern in 6 hours and 2 hours respectively. If they are opened on alternate
hours and if faucet A is opened first, in how many hours will the cistern be full?
A) 4 hours
B) 5 hours
C) 7 hours
D) 6 hours

21) B
Clearly tap B is faster than the Tap A and so, the cistern will be emptied
Part to be emptied = 2/5
Part emptied by (A+B) in 1 min = (1/12 – 1/20)
= 5-3/60
= 2/60
=>1/30
Therefore 1/30 : 2/5 :: 1;x
x= 1*2/5 *30/1 = 12mins
so the tank will be emptied in 12mins.

22) A
Part filled by (A+B+C) in 5 mins = 5(1/45 +1/35 + 1/25)
= 5 (35+45+63/1575)
= 143/315
Part filled by c in 5 mins = 5/25 = 1/5
Required ratio = 1/5 * 315/143 =63/143.

23) C
Part filled by(P+Q) in 1 hr = 1/5 + 1/20

www.ibpsguide.com | estore.ibpsguide.com | www.sscexamguide.com


270
Complete Quantitative Aptitude Questions


So P & Q together can fill the tank in 4 hrs.
Work done by the leak in 1 hr = 1 / 4 - 2/9
Leak will empty the tank in 36 hrs.

24) D
Let the slower tap alone fill the cistern in x mins
Then faster tap will fill it in x/4 mins
Therefore 1/x +4/x = 1/48
5/x = 1/48
X = 48*5
x = 240 mins
x = 4 hrs.

25) A
Suppose tube P alone takes x hrs to fill the tank
Then tubes Q and R will take x/3 and x/9 hrs respectively to fill the dumper.
Therefore 1/x + 3/x+ 9/x = 1/15
13x = 1/15
X = 195 hrs

26) C
Work done by leak in 1 hr=(1/18-1/24)
=>4-3/72
=>1/72
Leak will empty the dumper in 72 hours

27) D
(x/16)-[(24-x)/12] = 1/48
x/16-24/48+x/12=1/48
(3+4)x/48=97/48
7x/48=97/48
7x=97
X=14

28) C
(1/A + 1/B + 1/C) = 1/24 …(i)

www.ibpsguide.com | estore.ibpsguide.com | www.sscexamguide.com


271
Complete Quantitative Aptitude Questions

(1/B + 1/C + 1/D) = 1/30 …(ii)


(1/A + 1/D) = 1/40…(iii)
From eqn (i) and (ii)
(1/A – 1/D) = 1/120…(iv)
From eqn (iii) and (iv)
A=60,D=120.
Let the time taken to full the tank = s
S(1/A + 1/B +1/C +1/D)= 1
S(1/24+1/120) = 1
S= 20 mins

29) D
Part emptied by the leak in 1 hour = 1/16
part filled by (leak & inlet open) in 1 hour = 1/80
Part filled by the inlet tap in 1 hour = 1/16 – 1/80 = 1/20
Inlet tap fills the tank in = 20 hours
Inlet tap fills water at the rate of 8 litres a minute.
Capacity of tank = 20 * 60 * 8= 9600 litre

30) A
Faucet A can fill = 1/6
faucet B can fill = 1/2
For every two hour, 1/6 + 1/2 = 1+3/6
=>4/6=2/3 Part filled
Total filled in 3 hours = 2/3+1/6= 5/6
In next hour it will be fille full. So total time will be 4 hours.

31) Two pumps M and N can fill a cistern in 10m and 15m respectively. If both the pumps are opened
simultaneously, after how much time should N be closed so that the cistern is full in 8 minutes ?
A) 5min
B) 6min
C) 3min
D) 7min

32) Two pipes X and Y can fill a bunker in 24 hours. If only pipe X is open then it would take 12 hours longer
to fill the bunker. Find how much longer would it take if only pipe Y is open.
A) 46hrs

www.ibpsguide.com | estore.ibpsguide.com | www.sscexamguide.com


272
Complete Quantitative Aptitude Questions

B) 32hrs
C) 48hrs
D) 34hrs

33) A tank is filled by 3 taps K, L and M with uniform flow. The second tap L takes 3 times the time taken by K
to fill the tank, while M takes two times the time taken by L to fill the tank. If all the three taps can fill the tank
in 14 hours, find the time required by tap K alone to fill the tank.
A) 10hrs
B) 21hrs
C) 14hrs
D) 22hrs

34) Taps P, Q and R can fill a tank in 3, 4 and 5 hours respectively. If all the taps are opened together and
after 30 minutes taps Q and R are turned off, find the total time in which the tank is full.
A) 2(3/8)hrs
B) 1(1/7)hrs
C) 2(13/40)hrs
D) 3(13/43)hrs

35) Faucet A and B can fill a tank in 15 and 9 hrs respectively. Faucet C can empty it in 45 h. The tank is half
full. All the three faucets are in operation simultaneously. After how much time the tank will be full ?
A) 1(7/15)hrs
B) 2(1/11)hrs
C) 3(3/14)hrs
D) 3(3/11)hrs

36) Three tubes A, B and C can fill a tank in 12hours. After working at it together for 4 hours, C is closed and
A and B can fill the remaining part in 12 hours. The number of hours taken by C alone to fill the tank is
A) 32hrs
B) 30hrs
C) 38hrs
D) 36hrs

37) Two taps P and Q can fill a cistern in 24 min and 32 min respectively. Both the taps are opened together
for a certain time but due to some obstruction the flow of water was restricted to 7/4 of full flow in tap P and
5/3 of full in tap Q. This obstruction is removed after some time and cistern is now filled in 6min from that
moment. How long was it before the full flow.

www.ibpsguide.com | estore.ibpsguide.com | www.sscexamguide.com


273
Complete Quantitative Aptitude Questions

A) 8 min
B) 3 min
C) 5.6 min
D) 4.5 min

38) Two pipes, Xand Y can fill a tank in 36 and 30 minutes respectively. Both are opened together, but at the
end of 3minutes, X is turned off. In how many more minutes will Y fill the cistern?
A) 7(24/5)
B) 7(1/2)
C) 24(1/3)
D) 8(1/4)

39) A Bunker has a leak which would empty the completely filled bunker in 20 hours. If the bunker is full of
water and a tap is opened which admits 4 litres of water per minutes in the bunker, how many litres does the
bunker holds?
A) 2400
B) 4500
C) 4800
D) 7200

40) Two pumps M and N can separately fill a dumper in 36 minutes and 45 minutes respectively. Both the
pumps are opened together but 12 minutes after the start the pump M is turned off. How much time will it take
to fill the dumper?
A) 9 min
B) 10 min
C) 30 min
D) 32 min

31) C
X(1/10+1/15) +(8-x)1/10 = 1
5x/30+8-x/10 =1
5x+24-3x/30 =1
2x+24 = 30
2x=6
X=6/2 = 3

32) C

www.ibpsguide.com | estore.ibpsguide.com | www.sscexamguide.com


274
Complete Quantitative Aptitude Questions

X= 24+12 = 36
X+Y= 1/24
Y= 1/24 – 1/36 = 3-2/72 = 1/72
Y= 72
X alone= 72-24 = 48 HRS

34) B
1/x + 1/ (3x) + (1/6x) = 1/14
9/6x= 1/14
6x/9 = 14
6x=126
X=126/6
X=21

34) C
In 1 hr P, Q, R = 1/3+1/4+1/5 = 20+15+12/60 = 47/60
Filled in 30m = 47/120
Remaining = 1-47/120 =73/120
Tap P = 3*73/120= 219/120
Total = 219/120+1/2 =219+60 /120= 279/120= 2 13/40 hrs

35) C
In 1 hr = 1/15+1/9 – 1/45
=>3+5-1/45 = 7/45
½ tank filled by 3 Faucets = 45/7*1/2
=> 45/14 =>3(3/14)

36) D
A+B+C in 1h = 1/12
A+B+C in 4h = 4/12 = 1/3
Remaining = 1-1/3 = 2/3
A+B in 12hrs = 2/3
A+B in 1hr = 2/36=1/18
C alone to fill the tank= 1/12– 1/18 = 3-2/36= 1/36=36hrs

37) D
Let the obstruction remain for X min.

www.ibpsguide.com | estore.ibpsguide.com | www.sscexamguide.com


275
Complete Quantitative Aptitude Questions

Hence,
Part of cistern filled in X min + part of cistern filled in 6 min = full cistern
[(7X/4*24)+(5X/3*32)]+[(6/24)+(6/32)] = 1
(12X/96)+(7/16) = 1
12x/96=9/16
Thus,
X = 4.5 min.

38) C
X can fill cistern in 36 minutes.
X fills cistern in 1 minute = 1/36
Y can fill cistern in 30 minutes.
Y fills cistern in 1 minute = 1/30 part.
Xand Y together can fill cistern in 1 minute,
= {(1/36) + (1/30)} = 11/180 part.
So, they can together fill cistern in 3 minute,
= 3* (11/180) = 33/180 part.
Rest Cistern = 1 - (33/180) = 147/180 part.
147/180 part cistern must be filled by Y in,
[(147/180)/(1/30)] = 24(1/3) minutes.

39) C
Leak emptied the bunker per hour = 100/20 = 5% of water per hour;
Quantity of water emptied per hour = 4*60 = 240;
Thus, 5% = 240 liter;
Hence, capacity of water,
100% = 240*100/5= 4800 liter.

40) C
12/36 + x/45 = 1
(12*5)/180+4x/180=1
60+4x=180
4x=120
X=120/4=30min

41) A dumper has a leak which would empty the dumper in 10 minutes. A tap is turned on which admits 4
liters a minute into the dumper, and it is emptied in 12 minutes. How many liters does the dumper hold?

www.ibpsguide.com | estore.ibpsguide.com | www.sscexamguide.com


276
Complete Quantitative Aptitude Questions

A) 480 liters
B) 600 liters
C) 320 liters
D) 240 liters

42) Two faucet can separately fill a cistern 30minutes and 45 minutes respectively and when the waste faucet
is open, they can together fill it in 54 minutes. The waste faucet can empty the full cistern in?
A) 27 min
B) 23 min
C) 23 min
D) 29 min

43) One tube can fill a tank five times as fast as another tube. If together the two tubes can fill tank in 42 min,
then the slower tube alone will be able to fill the tank in?
A) 252 min
B) 208 min
C) 244 min
D) 192 min

44) A cistern is filled in 15 hours by three pipes P, Q and R. The pipe R is thrice as fast as Q and Q is thrice
as fast as P. How much time will pipe P alone take to fill the tank?
A) 120 hrs
B) 195 hrs
C) 135 hrs
D) Cannot be determined

45) A cistern is filled by three faucets with uniform flow. The first two faucets operating simultaneously fill the
cistern in the same during which the cistern is filled by the third faucet alone. The second faucet fills the
cistern 5 hours faster than the first faucet and 4 hours slower than the third faucet. The time required by the
first faucet is?
A) 6 hrs
B) 10 hrs
C) 15 hrs
D) 30 hrs

46) Pipes P, Q and R which fill the tank together in 12 hours are opened for 2hours after which pipe R was
closed. Find the number of hours taken by pipe R to fill the tank if the remaining tank is filled in 14 hours.

www.ibpsguide.com | estore.ibpsguide.com | www.sscexamguide.com


277
Complete Quantitative Aptitude Questions

A) 16
B) 14
C) 20
D) 42

47) A tank is 1/4th full. Two pipes which fill the tank in 30minutes and 40 minutes respectively are opened
simultaneously. After 10 minutes, a third pipe which empties the full tank in 60 minutes is also opened. In how
many minutes the tank will be full?
A) 14
B) 12
C) 15
D) 17

48) Two taps M and N can alone fill a tank in 80 minutes and 120 minutes respectively. But due to a leakage
of tank, it took 12 more minutes to fill the tank. In how many hours, the leak can alone empty the full tank?
A) 240
B) 230
C) 248
D) 256

49) Three pumps, M, N and O are opened to fill a tank such that M and N can fill the tank alone in 18 min. and
23 min. respectively and O can empty it in 15 min. After 3 minutes the emptying pipe is closed. In how many
minutes the tank will be full in this way?
A) 20
B) 25
C) 18
D) 12

50) Two faucet A and B can fill a tank in 20 hours and 40 hours respectively. If they are opened
simultaneously. Sometimes later, tap B was closed, then it takes total 14 hours to fill up the whole tank. After
how many hours B was closed?
A) 4 hours
B) 15.2 hours
C) 12 hours
D) 17.6 hours

41) D

www.ibpsguide.com | estore.ibpsguide.com | www.sscexamguide.com


278
Complete Quantitative Aptitude Questions

1/x - 1/10 = -1/12


10-x/10x=-1/12
12(10-x)=-10x
120-12x = -10x
2x=120
X=60
Therefore 60 * 4 = 240 litres

42) A
1/30 + 1/45 - 1/x = 1/54
1/x=1/30+1/45-1/54
1/x=9+6-5/270
1/x=10/270
1/x=1/27
x = 27

43) A
Let the slower tube alone fill the tank in x min.
Then, faster tube will fill it in x/5 min.
1/x + 5/x = 1/42
6/x = 1/42
=> x = 252min.

44) B
Suppose pipe P alone takes x hours to fill the tank.
Then, pipes Q and R will take x/3 and x/9hours respectively to fill the tank.
1/x + 3/x + 9/x = 1/15
13/x = 1/15
=> x = 195 hrs.

45) C
Suppose, first faucet alone takes x hours to fill the cistern.
Then, second and third faucets will take (x - 5) and (x - 9) hours respectively to fill the cistern.
1/x + 1/(x - 5) = 1/(x - 9)
(2x - 5)(x - 9) = x(x - 5)
x2 - 18x + 45 = 0
(x- 15)(x - 3) = 0 => x = 15

www.ibpsguide.com | estore.ibpsguide.com | www.sscexamguide.com


279
Complete Quantitative Aptitude Questions

46) D
1/P + 1/Q + 1/R = 1/12
Now given that first all open for 2hours, then R closed and P+Q completes in 14 hours, so
(1/P + 1/Q + 1/R) *2 + (1/P + 1/Q)*14 = 1
Put 1/P + 1/Q = 1/12 – 1/R
(1/12 – 1/R + 1/R) *2 + (1/12 – 1/R)*14 = 1
1/6+ 14/12 – 14/R = 1
Solve, R = 42

47) A
Since 1/4th is already filled, 3/4th is to filled now.
(1/30 + 1/40)*(10+x) – (1/60)*x = ¾
7/120(10+x)-x/60=3/4
7x-2x/120=3/4-7/12
5x/120=1/6
30x=120
X=4
So total 10+4=14minutes

48) A
A and B can fill tank in (1/80 + 1/120) = 1/48 so 48minutes
But it took 12 more minutes, this means the tank got full in 48+12= 60 minutes
So (1/80 + 1/120 – 1/x) = 1/60
1/X=1/80+1/120-1/60
Solve, x = 240

49) D
Let the tank full in x minutes, then M and N opened for x minutes and O for 3 minutes.
(1/18 + 1/23)*x – (1/15)*3 = 1
(23+18/414)X=1+1/5
Solve, x = 12

50) A
Let x is the time when B is closed
X(1/20+1/40)+14/20=1
X=4 hours

www.ibpsguide.com | estore.ibpsguide.com | www.sscexamguide.com


280
Complete Quantitative Aptitude Questions

51) Two taps A and B are opened together to fill a tank. Both the taps fill the tank in time “x” If B separately
took 25 minutes more time than “x” to fill the tank and B took 49 minutes more time than “x” to fill the tank,
then find out the value of x?
A) 48 minutes
B) 24 minutes
C) 54 minutes
D) 35 minutes

52) A tap can fill a bunker in 4 hours. After half the bunker is filled, three more similar tap are opened. What is
the total time taken to fill the bunker completely?
A) 3 hours
B) 2.5 hours
C) 5 hours
D) 4.2hours

53) 24 tins of water fill a bunker when the capacity of each bunker is 27litres. How many tins will be needed to
fill the same bunker, if the capacity of each tin is 18litres?
A) 36
B) 26
C) 46
D) 56

54) Bucket A has twice the capacity as bucket B. It takes 120 turns for bucket A to fill the empty dumper. How
many turns it will take for both the buckets A and B, having each turn together to fill the empty dumper.
A) 40
B) 50
C) 70
D) 80

55) Two taps P and Q can separately fill a tank in 120mins and 150 mins respectively. There is a 3rd tap in the
bottom of the tank to empty it. If all of the 3 taps are simultaneously opened, then the tank is full in 100 mins.
In how much time the 3rd pipe alone can empty the tank?
A) 100
B) 200
C) 300
D) 400

www.ibpsguide.com | estore.ibpsguide.com | www.sscexamguide.com


281
Complete Quantitative Aptitude Questions

56) A large cistern can be filled by 2 pipes P & Q in 120 mins and 80 mins respectively. How many mins will it
take to fill the cistern from empty state if Q is used for half the time and P & Q fil it together for the other half?
A) 2hrs
B) 1hr
C) 3hrs
D) 4hrs

57) A leak in the bottom of a bunker can empty the full bunker in 4 hrs. An inlet pump fills water at the rate of
3 litres a min. when the bunker is full, the inlet is opened and due to the leak, the bunker is empty in 6 hrs.
How many litres does the bunker field?
A) 1160 litrs
B) 1610 litres
C) 2160 litres
D) 2610 litres

58) Two taps can fill a bunker in 40 and 48 mins respectively and a waste tap can empty 6 gallons per
minute. All the 3 taps working together can fill the bunker in 30 mind. The capacity of the bunker is.
A) 480
B) 480
C) 680
D) 780

59) Two pumps P & Q can fill a tank in 12 mins and 15 mins respectively while a third pipe R can empty the
full tank in 6 mins. P & Q kept open for 5 mins in the beginning and then R is also opened. In what time is
tank emptied?
A) 30mins
B) 35mins
C) 40mins
D) 45 mins

60) 3 faucet P,Q & R can fill a cistern in 12 hrs. After working together for 4hrs, R is closed P and Q can fill
the remaining part in 14 hrs. The number of hrs taken by R alone to fill the cistern is.
A) 18 hrs
B) 20 hrs
C) 28hrs
D) 30hrs

www.ibpsguide.com | estore.ibpsguide.com | www.sscexamguide.com


282
Complete Quantitative Aptitude Questions

51) D
Time is taken to fill the tank by both taps x = √a*b
x = √25*49
=> 5*7
=> 35

52) B
In One hour tap can fill = 1/4
Time is taken to fill half of the bunker = 1/2 * 4= 2hours
Part filled by four taps in one hour = (4*1/4) = 1
Required Remaining Part = 1/2
Total time = 2+ 1 /2=2.5 hrs

53) A
Capacity of the bunker = (24 * 27)litres
=648litrs
Capacity of each tin = 18litres
Number of tins needed = 648/18
=36.

54) D
Let capacity of A be x litres
Then capacity of B = x/2 litres
Capacity of the dumper = 120x litres
Required number of turns = 120x/ x =x/2
= 2 (120x)/3x
=2430x/3x
=80.

55) B
Work done by the 3 tap in 1 min
= 1/100 – ( 1/120 + 1/150)
= 1/100 – (5+4/600)
=1/100 – 9/600
=6-9/600 = - 3/600
= - 1/200(negative signs mean empting)

www.ibpsguide.com | estore.ibpsguide.com | www.sscexamguide.com


283
Complete Quantitative Aptitude Questions

Therefore the 3rd tap alone can empty the tank in 200 mins

56) B
Part filled by (P+Q) in 1 min = 1/120 + 1/80
=2+3/240 = 5/240
= 1/48
Suppose the cistern is filled in x mins then, x/2 (1/48 + 1/80) = 1
x/2(5+3/240)=1
8x/480 = 1
x=480/8
x= 60 mins or 1 hr

57) C
Work done by the inlet in 1 hr = (1/4 – 1/6)
= (3-2/12)
= 1/12
Work done by the inlet in 1 min = 1/12 * 1/60 = 1/720
Volume of 1/720 part = 3 litres
Volume of whole = (720 * 3 )=2160 litres.

58) A
Work done by the waste tap in 1 min = 1/30 – (1/40 + 1/48)
= 8 – (6+5)/240
= 8-11/240
= -3/240
= -1/80 (negative sign means emptying)
Volume of 1/80 part = 60 gallons
Volume of whole = (6*80) = 480 gallons

59) D
Part filled in 5 min = 5(1/12 +1/15)
= 5*9/60 = ¾
Part emptied in 1 min when all the pumps are opened,
= 1/6 – (1/12 + 1/15)
= 1/6 – 3/20
= 1/60
Now 1/60 is part emptied in 1 min

www.ibpsguide.com | estore.ibpsguide.com | www.sscexamguide.com


284
Complete Quantitative Aptitude Questions

Therefore ¾ part will be emptied in 60 * ¾ = 45 mins

60) C
Part filled in 4 hrs = 4/12 = 1/3
Remaining part = 2/3
(P+Q)’s 14 hrs work = 2/3
(P+Q)’s 1 hr work = 2/ 3 *14 = 1/21
R’s 1 hr work = (P+Q+R)’s 1 hr work – (P+Q)’s 1 hr work
= 1/12 – 1/21
= 7-4/84 = 3/84 = 1/28
R alone can fill the tank in 28 hrs.

12. SIMPLE INTEREST AND COMPOUND INTEREST

1) A sum of money invested for 7years in Scheme 1 which offers SI at a rate of 8% pa. The amount received
from Scheme 1 after 7 years invested for 2 years in Scheme 2 which offers CI rate of 10% pa. If the interest
received from Scheme B was Rs.1638. What was the sum invested in Scheme 1?
A) Rs.7500
B) Rs.5000
C) Rs.8200
D) Rs.9000
E) None of these

2) Rs.5200 was partly invested in Scheme A at 10% pa CI for 2 years and Partly invested in Scheme B at
10% pa SI for 4 years. Both the schemes earn equal interests. How much was invested in Scheme A?
A) Rs.1790
B) Rs.2200
C) Rs.3410
D) Rs.2670
E) None of these

3) A sum of rupees 4420 is to be divided between rakesh and prakash in such a way that after 5 years and 7
years respectively the amount they get is equal, if compounded annually. The rate of interest is 10 percent.
Find the share of rakesh and prakash
a) 2000, 2420
b) 2420, 2000
c) 2480, 2420

www.ibpsguide.com | estore.ibpsguide.com | www.sscexamguide.com


285
Complete Quantitative Aptitude Questions

d) 2210, 2210
e) None of these

4) A sum of rupees 3200 is compounded annually at the rate of 25 paise per rupee per annum. Find the
compound interest payable after 2 years.
a) 1200
b) 1600
c) 1800
d) 2000
e) None of these

5) Aishwarya saves an amount of 500 every year and then lent that amount at an interest of 10 percent
compounded annually. Find the amount after 3 years.
a) 1820.5
b) 1840.5
c) 1920.5
d) 1940.5
e) None of these

6) What sum(principal) will be amount to Rs.34536.39 at compound interest in 3 years, the rate of interest for
1st, 2nd and 3rd year being 5%, 6% and 7% respectively?
a) Rs.25576
b) Rs.29000
c) Rs.28012
d) Rs.24000
e)none of these

7) A sum of money was put at SI at a certain rate for 2 years. Had it been at 1% higher rate, it would have
fetched Rs 24 more. Find the sum.
a) 1500
b) 1200
c) 1300
d) 1600
e) none of these

8) The difference between simple interest and compound on Rs. 1200 for one year at 10% per annum
reckoned half-yearly is:

www.ibpsguide.com | estore.ibpsguide.com | www.sscexamguide.com


286
Complete Quantitative Aptitude Questions

a) 5
b) 6
c) 3
d) 4
e) None of these

9) A man lends a certain sum of money at simple interest. Rate of interest for first one and half years is 6%,
for next 9 months is 5% and after that 4%. If he received Rs. 11496 at the end of 4 years, his capital was
a) Rs. 10000
b) Rs. 9000
c) Rs. 9600
d) Rs. 9200
e) None of these

10) Kriya deposits an amount of Rs. 65800 to obtain in a simple interest at the rate of 14 p.c.p.a. for 4 years.
What total amount will Kriya get at the end of 4 years?
a) Rs. 102648
b) Rs. 115246
c) Rs. 125578
d) Rs. 110324
e) None of these

1) B
SI =>Amount = x*8*7/100 + x = 56x+100x/100 = 156x/100 = 39x/25
CI=> 39x/25[(1+10/100)2 – 1] 1638 = 39x/25[121/100 – 1] = 39x/100[21/100] X = 1638*100*25/21*39 = 5000

2) C
Amount invested in Scheme B = X
Amount invested in Scheme A = 5200 – x
X*10*4/100 = (5200-x)*21/100……………………[(1-10/100)2-1] = 21/100
40x/100 = (5200-x)*21/100
2x/5 = (5200-x)*21/100
200x = 5200*21*5 – x*5*21
200x = 546000 – 105x
305x = 546000
X = 1790
Scheme A = 5200 – 1790 = 3410

www.ibpsguide.com | estore.ibpsguide.com | www.sscexamguide.com


287
Complete Quantitative Aptitude Questions

3) B
R*(1+10/100)^ 5 = (4420 – R)*(1+10/100)^ 7
We get R = 2420, so P = 2000

4) C
Rate of interest is 25 paise per rupee per annum.
So for 100 rupees it is 2500 paise i.e. 25 percent
Now, CI = 3200(1+25/100)^ 2 – 3200 = 1800

5) A
Total amount = 500*(1+10/100)^ 3 + 500*(1+10/100)^ 2 + 500*(1+10/100)
= 1820.5

6) B
34536.39 = p(1 + 5/100)(1 + 6/100)(1 + 7/100)
= p (105/100) x (106/100) x (107/100)
p = 34536.39 x 100 x 100 x 100 / 105 x 106 x 107
p = Rs.29000

7) B
2 years ,Rs 24 more
1 years , 24/ 2, ie 12 more
1 % of P = 12 (since the rate increased by 1%)
p * 1/100 = 12
P = 1200

8) C
Solution: SI =Rs. (1200 ×10×1)/100= Rs. 120
CI = Rs. [ 1200×(1+5/100) ^2 - 1200] = Rs.123
So CI-SI = Rs. 3

9) C
p*6*18/100*12+p*5*9/100*12+p*4*(48-27)=11496-p
or 108p=45p=84p=1200p=11496*1200
or p=11496*1200/1437= Rs 9600

www.ibpsguide.com | estore.ibpsguide.com | www.sscexamguide.com


288
Complete Quantitative Aptitude Questions

10) A
100===156
65800===?
102648

11) Adam borrowed some money at the rate of 6% p.a. for the first two years, at the rate of 9% p.a. for the
next three years, and at the rate of 14% p.a. for the period beyond five years. If he pays a total interest of Rs.
11,400 at the end of nine years , how much money did he borrow ?
a) 13000
b) 14000
c) 12000
d) 15000
e) None of these

12) The simple interest on a certain sum of money for 2.5 years at 12% per annum is Rs. 40 less than the
simple interest on the same sum for 3.5 years at 10% per annum. Find the sum.
a) 700
b) 800
c) 1000
d) 600
e) None of these

13) A certain sum of money amounts to Rs. 1008 in 2 years and to Rs. 1164 in 3.5 years. Find the rate of
interest?
a) 12
b) 14
c) 13
d) 18
d) None of these

14) The least number of complete years in which a sum of money put out at 10% CI will be more than
doubled is
A) 8yrs
B) 6yrs
C) 4yrs
D) 7yrs
E) None of these

www.ibpsguide.com | estore.ibpsguide.com | www.sscexamguide.com


289
Complete Quantitative Aptitude Questions

15) If Rs. 1,200 amounts to rs 1,323 in two years at compound interest, then what will be the amount of rs
1,600 in three years at compound interest at the same rate per cent ?
a) 1832.20
b) 1852.20
c) 1862.20
d) 1872.20
e) None of these

16) A and B each borrowed equal sums for 3 years at the rate of 5% simple and compound interest
respectively. At the time of repayment B has to pay Rs. 76.25 more than A. The sum borrowed and the
interest paid by A (in Rs.) is:
a) Rs. 10,000, Rs. 1,500
b) Rs. 11,000, Rs. 1,100
c) Rs. 10,000, Rs. 1,400
d) Rs. 9,000, Rs. 200
e) None of these

17) A sum was put at 5% at a certain rate for 5 Years. Had it been put at 3% Per Annum higher rate, it would
have fetched Rs. 900 more. Find the Sum?
a) 5000
b) 3000
c) 6000
d) 7000
e) None of these

18) Manish took a loan of Rs. 4000 at S.I. After 2 Years he cleared the loan by paying Rs. 5600. Find the
Rate % P.A?
a) 10
b) 20
c) 30
d) 40
e) None of these

19) A lent Rs. 25000 to B for 4 years and Rs. 40,000 to C for 3 1/2 years and got Rs. 24,000 S.I from both B
and C. Find the rate PCPA
a) 10

www.ibpsguide.com | estore.ibpsguide.com | www.sscexamguide.com


290
Complete Quantitative Aptitude Questions

b) 20
c) 30
d) 15
e) None of these

20) Equal amounts of each Rs. 43,892 is lend to two persons for 3 years. One at the rate of 30% S.I. and
second at the rate of 30% C.I. annually. By how much percent the C.I. is greater than the simple interest
received in this 3 years duration?
a) 33%
b) 35%
c) 37%
d) 30%
e) 43%

11) C
12+27+56=95
95===11400
100===?
12000

12) B
7x/20 - 3x/10 = 40 ⇒ x = ( 40 × 20 ) ⇒ x = 800

13) C
[ 1164-1008 = 156 ] ⇒ 156/3×4 = 208 ; R = 208/2×800×100 ⇒ 13

14) A
P(110/100)n > 2P
(11/10) n > 2P
1.1*1.1*1.1*1.1*1.1*1.1*1.1*1.1 = 2.14 > 2
N=8

15) B
1323 = 1200 [ 1 + r/100 ]3
441/100 = [21/20]2 = [1+r/100]2
R =5%
A = P [ 1+ r/100 ]n

www.ibpsguide.com | estore.ibpsguide.com | www.sscexamguide.com


291
Complete Quantitative Aptitude Questions

A = 1600 [ 21/20]3 = 1852.20

16) A
CI-SI=rs76.25
p[1+r/100]^n-p-PRN/100=76.25
p[1+5/100]^3-p-p*5*3/100=76.25
p[21/20*21/20*21/20]-p-15p/100=76.25
9261p-8000p-1200p/8000=76.25
61p=8000*76.25
p=10000
SI paid by A=PNR/100=10000*5*3/100=rs1500

17) C
P = (100 X 900) / (5X3) = 6000

18) B
Interest will be 5600-4000 = 1600
R = (100 X 1600) / (400 X 2)= 20

19) A
= 250 X 4R = 400 X (7/2) R = 2400R
=> R = 10

20) A
SI=43892*30*3=43892[9/10]
CI=43892[(1+30/100)^3-1]=43892(2197-1000/1000)=43892(1197/1000)
CI-SI=43892(297/1000)
Desired%=43892(297/1000)/43892(900/1000) = 33%

21) The present population of a village is 9,261. If the annual birth rate is 8(1/2)% and the annual death rate is
3.5%, then calculate the population 3 years ago.
a) 10,721
b) 11,363
c) 11,391
d) 8,000
e) 10,561

www.ibpsguide.com | estore.ibpsguide.com | www.sscexamguide.com


292
Complete Quantitative Aptitude Questions

22) Two equal sums of money are lent at the same times at 8% and 7% per annum simple interest. The
former is recovered 6 months earlier than the latter and the amount in each case is Rs. 2560. The sums of
money are lent out are :
a) Rs. 2000
b) Rs. 1500
c) Rs. 2500
d) Rs. 3000
e) None of these

23) In what time will a man receive Rs. 85 as compound interest on Rs. 320 at 12(1/2)% p.a. compounded
yearly?
a) 4(1/2) yrs.
b) 2(1/2) yrs.
c) 2 yrs.
d) 5 yrs.
e) 3(1/2) yrs.

24) If the compound interest on a certain sum for two years at 10% p.a. is Rs. 2,100 the simple interest on it
at the same rate for two years will be
a) Rs. 1,980
b) Rs. 1,760
c) Rs. 2,000
d) Rs. 1,800
e) Rs. 1,805

25) If Rs. 1,200 amounts to Rs. 1,323 in two years at compound interest, then what will be the amount of Rs.
1,600 in three years at compound interest at the same rate per cent?
a) Rs. 1,850
b) Rs. 1,852.20
c) Rs. 1,752.20
d) Rs. 1,905.50
e) Rs. 1,951

26) A person invested some amount at the rate of 12% simple interest and a certain amount at the rate of
10% simple interest. He received yearly interest of Rs. 130. But if he had interchanged the amounts invested,
he would have received Rs. 4 more as interest. How much did he invest at 12% simple interest?

www.ibpsguide.com | estore.ibpsguide.com | www.sscexamguide.com


293
Complete Quantitative Aptitude Questions

a) Rs. 700
b) Rs. 500
c) Rs. 800
d) Rs. 400
e) None of these

27) A tree increases annually by 1/8 th of its height. By how much will it increase after 2(1/2) years, if it stand
today 10 ft. high?
a) data insufficient
b) less than 12 ft.
c) more than 3 ft.
d) more than 2 ft.
e) slightly more than 13 ft.

28) On a certain sum of money, compound interest earned at the end of three years = Rs. 1456. Compound
interest at the end of two years is Rs. 880. Compute the principal invested.
a) Rs. 2,400
b) Rs. 2,800
c) Rs. 2,000
d) Rs. 1,600
e) None of these

29) Sashidharan took a loan of Rs. 20,000 to purchase a colour TV set from Royal Finance Co. He promised
to make the payment after three years. The company charges compound interest @ 10% p.a. for the same.
But, suddenly the company announces the rate of interest as 15% p.a. for the last one year of the loan period.
What extra amount Sashidharan has to pay due to his announcement of new rate of interest?
a) Rs. 7,830
b) Rs. 6,620
c) Rs. 4,410
d) Rs. 1,210
e) Rs. 3,000

30) A bank offers 5% annually compound interest calculated on half-yearly basis. A customer deposits Rs.
1600 each on 1st January and 1st July of a year. At the end of the year, the amount he would have gained by
way of interest is:
A) Rs. 120
B) Rs. 121

www.ibpsguide.com | estore.ibpsguide.com | www.sscexamguide.com


294
Complete Quantitative Aptitude Questions

C) Rs. 122
D) Rs. 123
e) None of these

21) D
Eff rate=8.5-3.5=5%
X(1+5/100)^3=9261
X=8000

22) A
X+(t-1/2)*x*8/100=x+x*t*7/100
7xt/100=2xt/25-x/25
xt/100=x/25
t=4yrs
A=x+x*8*3.5/100=2560
x=2000

23) C
By using option
Ci=2(320*1/8)40*1/8
=80+5=85

24) C
10%100=10
10%10=1=>10+1=11
Ci=10+11=21
21==2100
20==2000

25) B
1323/1200=(1+r/100)^2
R=5%.
Ci=3(1600*1/20)+2(80*1/20)+(4+4*1/20)=252.2
Req amt=1600+252.2=1852.2

26) B
x*12/100=y*10/100=130

www.ibpsguide.com | estore.ibpsguide.com | www.sscexamguide.com


295
Complete Quantitative Aptitude Questions

12+10y=13000 is equ (1)


x*10/100+y*12/100=134
10x+12y=13400 is equ (2)
Solve both equ we get
x=rs500

27) C
=>10(1+1/8)^2(1+1/8)^1/2=10*81/64*17/16=13.44
=>13.44-10=3.44

28) C
1456-880=576/880=36/55
6²/5(5+6)=1/5*100=20%
20% for 2 years=44%
44%==880
100%==2000

29) D
R=10,t=3
Ci=20000*33.1/100=6620
Ci for 2yrs,r=10
Ci=20000*21/100=4200
Ci for 3rd,r=15
24200*15/100=3630
Total ci=7830
Amt=7830-6620=1210

30) B
A={1600(1+2.5/100)^2 + 1600(1+2.5/100)]
rs.3321
Ci=3321-3200=121

31) There is 60% increase in an amount in 6 years at simple interest. What will be the compound interest of
Rs. 12,000 after 3 years at the same rate?
A) Rs. 2160
B) Rs. 3120
C) Rs. 3972

www.ibpsguide.com | estore.ibpsguide.com | www.sscexamguide.com


296
Complete Quantitative Aptitude Questions

D) Rs. 6240
E) None of these

32) What is the difference between the compound interests on Rs. 5000 for 1 years at 4% per annum
compounded yearly and half-yearly?
A) Rs. 2.04
B) Rs. 3.06
C) Rs. 4.80
D) Rs. 8.30
E) None of these

33) The least number of complete years in which a sum of money put out at 20% compound interest will be
more than doubled is:
A) 3
B) 4
C) 5
D) 6
E) None of these

34) Hari took an educational loan from a nationalized bank for his 2 years course of MBA. He took the loan of
Rs.5 lakh such that he would be charged at 7% p.a. at CI during his course and at 9% CI after the completion
of the course. He returned half of the amount which he had to be paid on the completion of his studies and
remaining after 2 years. What is the total amount returned by Hari?
A) Rs. 626255
B) .Rs. 626277
C) Rs. 616266
D) Rs. 626288
E) None of these

35) Rs.200,000 was invested by Mahesh in a FD @ 10% pa at CI. However every year he has to pay 20% tax
on the CI. How much money does Mahesh have after 3 years?
A) 215662.4
B) 216662.4
C) 217662.4
D) 218662.4
E) None of these

www.ibpsguide.com | estore.ibpsguide.com | www.sscexamguide.com


297
Complete Quantitative Aptitude Questions

36) Leela takes a loan of Rs. 8400 at 10% p.a. compounded annually which is to be repaid in two equal
annual installments. One at the end of one year and the other at the end of the second year. The value of
each installment is?
A) 4200
B) 4140
C) 4840
D) 5640
E) None of these

37) A sum of money lent at compound interest for 2 years at 20% per annum would fetch Rs.723 more, if the
interest was payable half yearly than if it was payable annually. The sum is ____
A) Rs. 20000
B) Rs. 15000
C) Rs. 30000
D) Rs. 45000
E) None of these

38) A sum of Rs.7140 is to be divided between Anita and Bala who are respectively 18 and 19 yr old, in such
a way that if their shares will be invested at 4% per annum at compound interest, they will receive equal
amounts on attaining the age of 21 year. The present share of Anita is
A) 4225
B) 4352
C) 3500
D) 4000
E) None of these

39) Suresh borrows Rs.6375 to be paid back with compound interest at the rate of 4 % pa by the end of 2
year in two equal yearly installments. How much will each installment will be?
A) 3840
B) 3380
C) 4800
D) Data inadequate
E) None of these

www.ibpsguide.com | estore.ibpsguide.com | www.sscexamguide.com


298
Complete Quantitative Aptitude Questions

40) During the first year the population of a village is increased by 5% and the second year it is diminished by
5%. At the end of the second year its population was 31500. What was the population at the beginning of the
first year?
A) 35500
B) 31578
C) 33500
D) 33000
E) None of these

31) C
Si=100r=10%
Ci12000(1+10/100)^3-12000=3972

32) E
yearly=5000*4*1/100=200
half yearly=>2%5000=100
2%100=2=>100+2=102
Ci=202
Diff=202-200=2

33) B
=>(6/5)^n>2=>(6/5*6/5*6/5*6/5)
N=4

34) D
5,00,000 * (1.07)² = 572450
Returned amount = 286225
After two years = 286225 * (1.09)² = 340063
Total amount = 286225 + 340063 = 626288

35) E
P I T Total
1st year - 200000 - 20000 – 4000 = 216000
2nd year - 216000 – 21600 – 4320 = 233280
3rd year - 233280 – 23328 - 4665.6 = 251942.4

36) C

www.ibpsguide.com | estore.ibpsguide.com | www.sscexamguide.com


299
Complete Quantitative Aptitude Questions

8400 = x*(210/121) => 4840

37) C
C.I. compounded half yearly = (4641/10000)x
C.I. compounded annually = (11/25)x
(4641/10000)x – (11/25)x = 723
x = 30000

38) C
Amount got by Anita after 3 yr = Amount got by Bala after 2 yr
x*(26/25)³ = (7140 – x)*(26/25)
26/25 = 7140 – x / x
x = 3500

39) B
25x/26 + 625/676x = 6375
x = (6375 * 676)/1275 = 3380

40) B
x * 105/100 * 95/100 = 31500
x = 31500 * 100/105 * 100/95
D = 31578

41.If Rs. 7200 amounts to Rs.10368 at compound interest in a certain time , then Rs. 7200 amounts to what
in half of the time?
A) 3400
B) 3600
C) 38000
D) 3520
E) None of these

42) The compound interest on a certain sum for 2 years is Rs. 756 and S.I. is Rs. 720. If the sum is invested
such that the S.I. is Rs. 1296 and the number of years is equal to the rate per cent per annum, Find the rate
of interest?
A) 4%
B) 5%
C) 6%

www.ibpsguide.com | estore.ibpsguide.com | www.sscexamguide.com


300
Complete Quantitative Aptitude Questions

D) 8%
E) 2%

43) The difference between the total simple interest and total compound interest compounded annually at the
same rate of interest on a sum of money at the end of two years is Rs. 50. What is definitely the rate of
interest?
A) 10
B) 4
C) Data provided are not adequate to answer the question
D) 5
E) 7.5

44) The compound interest on a certain sum of money for 2 years at 4% per annum be Rs. 2448, what would
be the simple interest on the same sum for 2 years at the same rate?
a) Rs2500
b) Rs2400
c) Rs2360
d) Rs2250
e) None of these

45) If the rate increases by 2%, the simple interest received on a sum of money increases by Rs. 108. If the
time period is increased by 2 years, the simple interest on the same sum increases by Rs. 180.The sum is :
a) Rs. 1800
b) Rs. 3600
c) Rs. 5400
d) Data inadequate
e) None of these

46. A person invested in all Rs. 2600 at 4%, 6% and 8% per annum simple interest. At the end of the year, he
got the same interest in all the three cases. The money invested at 4% is :
a) Rs. 200
b) Rs. 600
c) Rs. 800
d) Rs. 1200
e) None of these

www.ibpsguide.com | estore.ibpsguide.com | www.sscexamguide.com


301
Complete Quantitative Aptitude Questions

47) Arun invested a sum of money at a certain rate of simple interest for a period of 4 yrs. The total interest
earned by him would have been 50% more than the earlier interest amount when invested for 6 years. What
was the rate of interest per cent per annum ?
a) 4
b) 8
c) 5
d) Can’t be determined
e) None of these

48) Find the compound interest on Rs. 64,000 for 1 year at the rate of 10% per annum compounded quarterly
(to the nearest integer).
a) Rs. 8215
b) Rs. 8205
c) Rs. 8185
d) Can’t be determined
e) None of these

49) Amal borrowed a sum of money with simple interest as per the following rate structure:
a. 6 p.c. p.a. for the first three years
b. 8 p.c. p.a. for the next five years
c. 12 p.c. p.a. for next eight years
If he paid a total of Rs. 5,040 as interest at the end of twelve years, how much money did he borrow?
a) Rs. 8,000
b) Rs. 10,000
c) Rs. 12,000
d) Rs. 6,000
e) None of these

50) On a certain rate of interest a sum of Rs 5000 becomes Rs 16,200 in certain years at compound interest.
In half of the time given, this sum will become?
A) Rs 10,000
B) Rs 5,600
C) Rs 9,000
D) Cannot be determined
E) None of these

41) E

www.ibpsguide.com | estore.ibpsguide.com | www.sscexamguide.com


302
Complete Quantitative Aptitude Questions

Let rate = R% and time = n year


Then, 10368 =7200(1+R/100)n
⇒ (1+R/100)n = 10368/7200 = 1.44
∴ (1 + R/100)n/2 = √1.44 = 1.2
∴ Required amount for n/2 yr
= 7200(1+ R/100)n/2
= 7200 x 1.2 = Rs. 8640

42) C
CI for 2 years = Rs. 756
SI for 2 years = Rs. 720
36/360 * 100 = 10%
P for first year = 3600
P*x*x/100 = 1296
x = 6%

43) C
Data provided are not adequate to answer the question.

44) B
let p=100
=>4%100=4
4%4=0.16=>4+0.16=4.16
Ci=8.16,si=8
8.16===2448
8===?
2400

45) D
The given date is not enough to solve this

46) D
=>1/4:1/6:1/8=>6:4:3
13===2600
6===?
1200.

www.ibpsguide.com | estore.ibpsguide.com | www.sscexamguide.com


303
Complete Quantitative Aptitude Questions

47) D
=>cannot be determined

48) E
=>64000*(1.025)^4=70644.025
Ci=6644.025

49) E
Let x be the amount Amal borrowed.
∴ 18% of x + 40% of x + 48% of x = 5040
106%=5040
100%=4754

50) C
a————————b—————c
5000—————– X ————16200
———–t——————- t ————
As we have to calculate the sum for half time, both time period is same, and hence
a:b = b:c
5000:x = x:16200
x=Rs 9000

51) If a certain sum becomes double in 3 years at certain rate of interest at C.I. Then in how many years it will
become 16 times?
A) 12 years
B) 24 years
C) 8 years
D) Cannot be determined
E) None of the above

52) Manivel invests two sum of money A and B at 10% p.a. and 20% p.a respectively at CI for 2 years. IF the
total interest on both the sum is Rs 5350 then find the sum invested in A if the total sum of A and B was Rs
20,000?
A) Rs 5,000
B) Rs 10,000
C) Rs 12,000
D) Rs 15,000

www.ibpsguide.com | estore.ibpsguide.com | www.sscexamguide.com


304
Complete Quantitative Aptitude Questions

E) None of these

53) The compound interest on a certain sum for 2 years at a certain rate of interest is Rs 1025 and Simple
Interest on the same sum, same time and same rate of interest is Rs 1,000. Then find the C.I for same sum in
3 years.
A) Rs 1575.25
B) Rs 1576.25
C) Rs 1576.75
D) Rs 1575.75
E) None of these

54) A sum becomes triple in 6 years at S.I. The same sum will become 19 times in how many years?
A) 50 years
B) 48 years
C) 54 years
D) 57 years
E) None of these

55) A sum of Rs 343 becomes 512 in 3 years at C.I. Find the rate of interest.
A) 14 (2/7) %
B) 12.5 %
C) 8 (2/3) %
D) 16 (2/3) %
E) None of these

56) Find the C.I on Rs 20,000 at 10% rate of interest in 2 years if compounded half yearly. (Approximately)
A) Rs 4210
B) Rs 4310
C) Rs 4410
D) Rs 4510
E) None of these

57) A sum of Rs 6,000 was taken as a loan. This is to be repaid in two equal annual installments. If the rate of
interest is 20% compounded annually then find the value of each installment.
A) Rs 4400
B) Rs 2220
C) Rs 4320

www.ibpsguide.com | estore.ibpsguide.com | www.sscexamguide.com


305
Complete Quantitative Aptitude Questions

D) Rs 4420
E) None of these

58) If the ratio of difference between CI and SI for 3 years and 2 years is 31:10, then find the Rate of Interest.
A) 11.11%
B) 10%
C) 20%
D) 25%
E) None of these

59) If a sum of RS 2744000 becomes Rs 3176523 in three years on Compound Interest then find the rate of
interest.
A) 10%
B) 5%
C) 8%
D) 20%
E) None of these

59) B
Find the cube root of both numbers. Cube root-> 3 years
cube root(2744000): cube root(3176523)
140:147
rate=(147-140)/140*100==5

60) If the difference between Simple Interest and Compound Interest at 20% rate of Interest in 3 years is
5120, then find the sum.
A) Rs 40,000
B) Rs 50,000
C) Rs 60,000
D) Rs 30,000
E) None of these

51) A
In C.I P increases like
P——-2P——–4P———8P——–16P
—-3yrs—–3yrs—–3yrs——-3yrs
total=3+3+3+3=12 years

www.ibpsguide.com | estore.ibpsguide.com | www.sscexamguide.com


306
Complete Quantitative Aptitude Questions

52) D
At 10% CI in 2 years=21 %
At 20% Ci in 2 years =44%
and 5350 is 107/4% of 20000, by using allegation
A B
21 44
107/4
3 1
A=3/4*20000= Rs 15000

53) B
SI for 2 years = Rs 1000 =.> Si 1 year = Rs 500
In the second years Rs 25 is added in CI (1025-1000) which is 5% of 500
Hence R=5%
5%=500
100%=10000
sum=10000
CI for 3 years= RS 1576.25

54) C
SI=A-P=> A=3P as sum triples
SI=3P-P=2P in 6 years
In 19 times SI=18 P—54 years (2:6 hence 18=54)

55) A
Sum=353; Amount=512
cuberoot(343): cuberoot(512)
7:8
rate=(8-7)/7 *100 =14 (2/7)%

56) B
In half yearly=> Time-double; Rate= half
Rate=5% ; Time=4 years; Sum = Rs 20,000
1 years————–2 years————3 years———-4 years
1000—————–1000—————1000————-1000
————————50——————50—————–50

www.ibpsguide.com | estore.ibpsguide.com | www.sscexamguide.com


307
Complete Quantitative Aptitude Questions

———————————————50—————–50
———————————————2.5—————-50
—————————————————————–2.5
—————————————————————–2.5
—————————————————————–2.5
—————————————————————-0.125
Total = Rs 4000 +300 + 10+0.125= Rs 4310.125

57) C
x/ (1+20/100)^1 + x/(1+20/100)^2 = 6600=4320

58) B
Sum= A
Interest= B
A——–A———A
———-B———B
———————B
———————C
CI for 3 years=3A+3B+C
SI for 3 years =3A
Diff= 3B+cCI for 2 years=2A+B
SI for 2 years=2A
diff=B
ratio=(3B+C)/B=31/10
B=10; C=1
Rate=C/B=1/10=10%

59) B
Find the cube root of both numbers. Cube root-> 3 years
cube root(2744000): cube root(3176523)
140:147
rate=(147-140)/140*100==5

60) A
On SI interest=20% *3 =60%
On CI interest =20%= 1/5
5——-6

www.ibpsguide.com | estore.ibpsguide.com | www.sscexamguide.com


308
Complete Quantitative Aptitude Questions

5——-6
5——-6
________
125—-216
(216-125)/125*100=72.8%
diff=72.8-60=12.8%
12.8%=5120
100%=40,000

61) Find the Compound Interest on Rs 30,000, if the rate of interest for first year is 5% second year is 10%
and on the third year is 20%
A) 11580
B) 11500
C) 10500
D) 10000
E) None of these

62) What is the difference between Simple Interest and Compound Interest on Rs 70,000 ar 20% rate of
interest in one and a half year if Compound Interest is compounded half yearly.
A) Rs 2070
B) Rs 2160
C) Rs 2170
D) Rs 2060
E) None of these

63) Divide Rs 20,816 between A and B so that A’s share at the end of 7 years is equal to B’s share at the end
of 9 years with compound interest being 4% p.a
A) 10716, 10100
B) 10616, 10200
C) 10816, 10000
D) 10800, 10016
E) None of these

64) Find the simple interest and compound interest of Rs 15000 at 20% rate of interest after 3 years.
A) 9000, 11000
B) 8000, 11920
C) 9000, 10920

www.ibpsguide.com | estore.ibpsguide.com | www.sscexamguide.com


309
Complete Quantitative Aptitude Questions

D) 6000, 9000
E) None of these

65) A man borrows Rs 8000 at 10% compounded rate of interest. At the end of each year he pays back Rs
2200. How much amount should he pay at the end of the third year to clear all his dues?
A) Rs 5500
B) Rs 5466
C) Rs 5666
D) Rs 5566
E) None of these

66) What sum of money at compound interest will amount to Rs 32000 in 3 years at the rate of interest 20% in
first years, 16 (2/3)% in second year and 14 (2/7)% in third year.
A) Rs 18,000
B) Rs 20,000
C) Rs 22,000
D) Rs 25,000
E) None of these

67) S.I on certain sum for 3years at any rate of interest is Rs225 while C.I on the same sum at the same rate
for 2yrs is 153. Find the rate%?
A) 5%
B) 6%
C) 4%
D) 7%
E) 8%

68) If a sum of Rs216 becomes 343 in 3yrs. Then find C.I on Rs43,200 in 3yrs at the same rate of interest?
A) 20,000
B) 25,000
C) 24,400
D) 25,400
E) 26,000

69) Aishwarya invested Rs. 20,000 with rate of interest at 20 p.c.p.a. The interest was compounded half
yearly for first year and in the next year it was compounded yearly. What will be the total interest earned at
the end of two years?

www.ibpsguide.com | estore.ibpsguide.com | www.sscexamguide.com


310
Complete Quantitative Aptitude Questions

a) Rs. 8,800
b) Rs. 9,040
c) Rs. 8,040
d) Rs. 9,800
e) None of these

70) A sum of money is lent for 2 years at 20% p.a. compound interest. It yields Rs 482 more when
compounded semi-annually than compounded annually. What is the sum lent?
A) Rs 25,600
B) Rs 20,000
C) Rs 26,040
D) Rs 40,500
E) None of these

61) A
1st year 5%=1/20————20———21
2nd year 10% =1/10———10———11
3rd year 20% =1/5———–5———-6
——————————–=1000——1386
(1386-1000)/1000*200=38.6%
38.6% of 30000=11580

62) C
SI on 1 (1/2) year= 20*1.5=30%
SI on 1 (1/2) years of compounded half yearly make rate half yearly and time double
r=10%=1/10 ; t=3 years
10——11
10——11
10——11
1000—-1331
r=331/1000*100=33.1
33.1%of 70,000 = 2170

63) C
Second part + 4% ci for 2 years of second part = first part
Second part + 8.16% of second part= first part
First part/second part= 108.16/100 = 10816/10000

www.ibpsguide.com | estore.ibpsguide.com | www.sscexamguide.com


311
Complete Quantitative Aptitude Questions

64) C
SI= 20*3=60%=9000
CI=
3000 ———— 3000 ———3000
——————-600————–600
————————————–600
————————————–120
=> 9000+1800+120=10920

65) D
First year=8000+800=8800-2200=6600
Second year=6600+660=7260-2200=5060
Third year=5060+506=5566

66) B
1st year = 20% =1/5————-5 ————-6
2nd year = 16 (2/3)= 1/6——-6————–7
3rd year = 14 (2/7) =1/7 ——-7————–8
———————————-= 210———336 on simplifying = 5:8
r=(8-5)/5*100=60%
160%=32000
100%=20000

67) C
S.I for 1yr = 225/3 =75
S.I for 2nd yr= 150
S.I 75 75
C.I 3(153-150=3)
3/75 * 100 = 4%

68) D
(216)1/3 : (343)1/3
6 : 7 – difference is 1. Now 1/6*100 = 16(2/3)%
Now according to question C.I for 3 yrs
16(2/3)% = 1/6 . 6 …………7
. . 6 …………7

www.ibpsguide.com | estore.ibpsguide.com | www.sscexamguide.com


312
Complete Quantitative Aptitude Questions

. . 6 …………7
216 − 343 = 127
216 =43,200
1 = 200
127 = 25,400

69) B
A=20000(110/100)(110/100)(120/100)
=29040
ci=29040-20000
=9040

70) B
P[1 + (r/2)/100]4 – P[1 + r/100]2 = 482
P[1 + 10/100]4 – P[1 + 20/100]2 = 482
Solve, P = 20,000

71) A sum of money is accumulating at compound interest at a certain rate of interest. If simple interest
instead of compound were reckoned, the interest for the first two years would be diminished by Rs. 20 and
that for the first three years by Rs. 61. Find the sum.
a) Rs. 7000
b) Rs. 8000
c) Rs. 7500
d) Rs. 6500
e) None of these

72) The difference between compound interest earned after 3 years at 5% p.a. and simple interest earned
after 4 years at 4% p.a. is Rs 76. Find the principal amount.
A) Rs 32,000
B) Rs 28,000
C) Rs 31,500
D) Rs 32,500
E) None of these

73) A sum of money is lent at simple interest and compound interest. The ratio between the difference of
compound interest and simple interest of 3 years and 2 years is 35 : 11. What is the rate of interest per
annum?

www.ibpsguide.com | estore.ibpsguide.com | www.sscexamguide.com


313
Complete Quantitative Aptitude Questions

A) 20 3/4%
B) 17 2/5%
C) 18 2/11%
D) 22 1/5%
E) 24 5/6%

74) Rs 3903 is to be divided in a way that A’s share at the end of 7 years is equal to the B’s share at the end
of 9 years. If the rate of interest is 4% compounded annually, find A’s share.
A) Rs 2475
B) Rs 1875
C) Rs 2175
D) Rs 1935
E) Rs 2028

75) A sum of rupees 3903 is divided between P and Q such that the share of P at the end of 8 years is equal
to the share of Q after 10 years. Find the share of P if rate of interest is 4% compounded annually.
a) 2012
b) 2029
c) 2028
d) 2081
e) None of these

76) A man borrows Rs. 4000 from a bank at 7.5% compound interest. At the end of every year, he pays Rs.
1500 as part repayment of loan and interest. How much does he still owe to the bank after three such
installments?
a) Rs. 123.25
b) Rs. 125
c) Rs. 400
d) Rs. 469.18
e) None of these

77) A sum of money is lent for 2 years at 10% p.a. compound interest. It yields Rs 8.81 more when
compounded semi-annually than compounded annually. What is the sum lent?
a) 1000
b) 1200
c) 1400
d) 1600

www.ibpsguide.com | estore.ibpsguide.com | www.sscexamguide.com


314
Complete Quantitative Aptitude Questions

e) None of these

78) A sum of rupees 4420 is to be divided between venki and Kavi in such a way that after 5 years and 7
years respectively the amount they get is equal. The rate of interest is 10 percent compounded. Find the
share of venki and kavi
a) 2000, 2420
b) 2420, 2000
c) 2480, 2420
d) 2210, 2210
e) None of these

79) A part of 70000 is lent out at 10% annum. The rest of the amount is lent out at 5% per annum after one
year. The ratio of interest after 3 years from the time when first amount was lent out is 1:2. Find the second
part that was lent out at 5%.
A) 40000
B) 50000
C) 60000
D) 48000
E) 55000

80) Vijay lends a certain amount to Vignesh on simple interest for two years at 20%. Vignesh gives this entire
amount to kishore on compound interest for two years at the same rate annually. Find the percentage earning
of Vijay at the end of two years on the entire amount.
A) 3%
B) 3(1/7)%
C) 4%
D) 5(6/7)%
E) None of these

71) B
p(r/100)^2=40
p(r/100)^2*(300+r)/100=61
solving both we will get p=8000

72) A
P[1 + 5/100]3 – P] – P*4*4/100 = 76
P [9261/8000 – 1 – 16/100] = 76

www.ibpsguide.com | estore.ibpsguide.com | www.sscexamguide.com


315
Complete Quantitative Aptitude Questions

P=32000

73) E
Difference in 3 yrs = Pr^2(300+r)/100^3
Difference in 2 yrs = Pr^2/100^2
So Pr^2(300+r)/100^3 / Pr^2/100^2 = 35/11
= (300+r)/100 = 35/11
= 18 2/11%

74) B
A’s share = (1 + 4/100)^7
B’s share = (1 + 4/100)^9
Divide both, B/A = (1 + 4/100)2 = 676/625
So A’s share = 625* 3903/(676+625) =1875

75) C
P*(1 + 4/100)^8 = (3903 – P)*(1 + 4/100)^10
p(104/100)^8=(3903-P)*(104/100)^10
solving it we get P=2028

76) A
Balance = Rs.[{4000 x (1 x 15/2 x 100)3} - {1500 x (1 + 15/2 x 100)2 + 1500 x (1 + 15/2 x 100) + 1500}]
= Rs. 123.25

77) D
8.81 = p*(1+5/100)^4 – p*(1+10/100)^2
8.81=p(105/100)^4-p(110/100)^2
solving it we will get p=1600

78) B
R*(1+10/100)^ 5 = (4420 – R)*(1+10/100)^ 7
We get R = 2420, so P = 2000

79) C
10*3*x/5*2*y = 1/2
x/y = 1/6
6/7*70000 = 60000

www.ibpsguide.com | estore.ibpsguide.com | www.sscexamguide.com


316
Complete Quantitative Aptitude Questions

80) C
SI=20*2=40%
CI=20+20+(400/100)=44%
Diff = 44-40=4%

81) The difference between the total simple interest and the total compound interest compounded annually at
the same rate of interest on a sum of money at the end of two years is Rs. 450. What is definitely the rate of
interest per cent per annum?
A) 8400
B) 4800
C) 7800
D) Data inadequate
E) None of these

82) Venkat and Vidhya have to clear their respective loans by paying 2 equal annual instalments of Rs.30000
each. Venkat pays at 10% pa of SI and Vidhyapays at 10% CI pa. What is the difference in their payments ?
A) 200
B) 300
C) 400
D) 500
E) None of these

83) A sum is divided A and B in the ratio of 1:2.A purchased a car from his part which depreciates 14 2/7%per
annum and B deposited his amount in a bank,which pays him 20%interest per annum compounded
annually.By what % will the total sum of money increases after two years due to this investment
pattern(approx)?
a) 20
b) 26.66
c) 30
d) 25
e) None of these

84) A sum of Rs.7140 is to be divided between Anita and Bala who are respectively 18 and 19 yr old, in such
a way that if their shares will be invested at 4% per annum at compound interest, they will receive equal
amounts on attaining the age of 21 year. The present share of Anita is
A) 4225

www.ibpsguide.com | estore.ibpsguide.com | www.sscexamguide.com


317
Complete Quantitative Aptitude Questions

B) 4352
C) 3500
D) 4000
E) None of these

85) During the first year the population of a village is increased by 5% and the second year it is diminished by
5%. At the end of the second year its population was 31500. What was the population at the beginning of the
first year?
A) 35500
B) 31578
C) 33500
D) 33000
E) None of these

86) In how much time will the simple interest on $3,500 at the rate of 9% p.a be the same as simple interest
on $4,000 at 10.5% p.a for 4 years?
A) 5years 4 month
B) 5 years
C) 6 years
D) 7 years
E) None

87) A father left a will of Rs.35 lakhs between his two daughters aged 8.5 and 16 such that they may get
equal amounts when each of them reach the age of 21 years. The original amount of Rs.35 lakhs has been
instructed to be invested at 10% p.a. simple interest. How much did the elder daughter get at the time of the
will?
a) Rs. 17.5 lakhs
b) Rs. 21 lakhs
c) Rs. 15 lakhs
d) Rs. 20 lakhs
e) None of these

88) What will Rs.1500 amount to in three years if it is invested in 20% p.a. compound interest, interest being
compounded annually?
a) 2400
b) 2592
c) 2678

www.ibpsguide.com | estore.ibpsguide.com | www.sscexamguide.com


318
Complete Quantitative Aptitude Questions

d) 2540
e) None of these

89) Rs.100 doubled in 5 years when compounded annually. How many more years will it take to get another
Rs.200 compound interest?
a) 10 years
b) 5 years
c) 7.5 years
d) 15 years
e) 8 years

90) Rs. 5887 is divided between Shyam and Ram, such that Shyam's share at the end of 9 years is equal to
Ram's share at the end of 11 years, compounded annually at the rate of 5%. Find the share of Shyam.
a) 2088
b) 2000
c) 3087
d) 3057
e) None of these

81) D
Difference = Pr2/(100)2
= (450×100×100)/(P×r2)
P is not given

82) B
D =[(30,000 *110/100*110/100) – 30,000] – 30,000 *10*2/100
=[36300-30000]- 6000
=6300 – 6000
D = 300

83) A
let amt be 100and 200
the value of 100 become 100*6/7*6/7=3600/49=73.46
the value of 200 becomes 200*1.2*1.2=288
total=288+73.46=361.46
=>approx 20%increases

www.ibpsguide.com | estore.ibpsguide.com | www.sscexamguide.com


319
Complete Quantitative Aptitude Questions

84) C
x*(26/25)³ = (7140 – x)*(26/25)
26/25 = 7140 – x / x
x = 3500

85) B
x * 105/100 * 95/100 = 31500
x = 31500 * 100/105 * 100/95
D = 31578

86) A
S.I on $4,000 at rate 10.5% = 10.5/100 = 0.105 for 4 years
S.I = ( P x R x T) /100
= 4000 x 0.105 x 4
S.I = $ 1,680
The interest of $1,680 is the same as that on $3,500 at 9% p.a for suppose 't' years.
S.I x 100
Time = t = ------------
PxR
1680 x 100
Time = t = ------------
3500 x 9
168,000
Time = t = ------------
31,500
Time = t = 5.33years= 5 year 4 months

87) B
x +50x/100 = (3,500,000 - x) + 12.5*10*(3500000-x)/100
=> 2x +50x/100 +125x/100 = 3,500,000 (1 +5/4 )
=> 200x+50x+125x/100=9/4(3500000)
=> x = 2,100,000 = 21 lakhs.

88) B
20%1500=300
20%300=60=>300+60=360
20%360=72=>360+72=432

www.ibpsguide.com | estore.ibpsguide.com | www.sscexamguide.com


320
Complete Quantitative Aptitude Questions

Ci=432+360+300=1092
A=1500+1092=2592

89) B
Rs.100 invested in compound interest becomes Rs.200 in 5 years.
The amount will double again in another 5 years.
=>the amount will become Rs.400 in another 5 years.
So, to earn another Rs.200 interest, it will take another 5 years.

90) C
Shyam's share * (1+0.05)9 = Ram's share * (1 + 0.05)11
Shyam's share / Ram's share = (1 + 0.05)11 / (1+ 0.05)9 = (1+ 0.05)2 = 441/400
Therefore Shyam's share = (441/841) * 5887 = 3087.

91) A sum of money invested for a certain number of years at 8% p.a. simple interest grows to Rs.180. The
same sum of money invested for the same number of years at 4% p.a. simple interest grows to Rs.120. For
how many years was the sum invested?
a) 15 years
b) 40 years
c) 33 years and 4 months
d) 23years
e) Cannot be determined

92) A man invests Rs.5000 for 3 years at 5% p.a. compound interest reckoned yearly. Income tax at the rate
of 20% on the interest earned is deducted at the end of each year. Find the amount at the end of the third
year.
a) 5624.32
b) 5630.50
c) 5788.125
d) 5627.20
e) None of these

93) How long will it take a certain amount to increase by 30% at the rate of 15% simple interest?
a) 3
b) 2
c) 6
d) 4

www.ibpsguide.com | estore.ibpsguide.com | www.sscexamguide.com


321
Complete Quantitative Aptitude Questions

e) None of these

94) A money lender lent Rs. 1000 at 3% per year and Rs. 1400 at 5% per year. The amount should be
returned to him when the total interest comes to Rs. 350. Find the number of years.
A) 3.5
B) 3.75
C) 4
D) 4.5
E) None of these

95) Apersom invests ₹ 12000 as fixed deposit at a bank at the rate of 10% per annum simple interest. But
due to some pressing needs, he has to withdraw the entire money after 3 yr for which the bank allowed him a
lower rate of interest. If he gets ₹ 3320 less than, what he would have got at the end of 5 yr then rate of
interest allowed by bank is
a) 7 8/9%
b) 8 7/9%
c) 8 8/9%
d) 7 4/9%

96) Ajay takes some loan from Rashmi at the rate of 5% per annum and after 2 yr, Ajay gave back ₹ 8800 to
Rashmi and this way paid his whole loan. Find the interest paid by Ajay.
a) 825
b) 975
c) 800
d) 850
e) None of these

97) Ramesh invested an amount that is 10% of ₹ 10000 at simple interest. After 3 yr, the amount becomes
₹ 2500. Find out the 4 times of actual interest rate.
a) 5000%
b) 250%
c) 200%
d) 600%
e) None of these

98) A sum of ₹ 1550 was lent partly at 5% and partly at 8% per annum simple interest. The total interest
received after 4 yr was ₹ 400, The ratio of the money lent at 5% to that lent at 8% is

www.ibpsguide.com | estore.ibpsguide.com | www.sscexamguide.com


322
Complete Quantitative Aptitude Questions

a) 16 : 15
b) 17 : 15
c) 16 : 13
d) 16 : 19
e) None of these

99) The simple interest of a sum of money is 1/144 of the principal and the number of years is equal to the
rate per cent per annum. What will be the rate per cent per annum ?
a) 3/5%
b) 5/6%
c) 7/6%
d) 1/6%
e) None of these

100) A sum of money amounts to ₹ 2240 at 4% per annum simple interest in 3 yr. The interest on the same
sum for 6 months at 3.5% per annum is
a) 30
b) 50
c) 35
d) 150
e) None of these

91) A
Principal + 8% p.a. interest on principal for n years = 180 …….. (1)
Principal + 4% p.a. interest on principal for n years = 120 ……… (2)
4% p.a. interest on principal for n years = Rs.60.
Principal + 60 = 120
= Principal = Rs.60.
n = 60/4 = 15 years.

92) A
5% is the rate of interest. 20% of the interest amount is paid as tax. That is 80% of the interest amount stays
back. Therefore, if we compute the rate of interest as 80% of 5% = 4% p.a., we will get the same value.
The interest accrued for 3 years in compound interest = 3*simple interest on principal + 3*interest on simple
interest + 1*interest on interest on interest. = 3*(200) + 3*(8) + 1*0.32 = 600 + 24 + 0.32 = 624.32
The amount at the end of 3 years = 5000 + 624.32 = 5624.32

www.ibpsguide.com | estore.ibpsguide.com | www.sscexamguide.com


323
Complete Quantitative Aptitude Questions

93) B
Simple interest = x*30/100 = 3x/10
T = 100*SI/PR = 100*3x/10 / x*15 = 2 years

94) A
1000*t*3/100) + (1400*t*5/100) = 350 → t =3.5

95) D
Let the rate of interest allowed by bank be r%
According to the question,
[(12000 x 5 x 10)/100] - [(12000 x 3 x r)/100] = 3320
⇒ 6000 - 360r = 3320
⇒ 360r = 6000 - 3320 = 2680
⇒ r = 2680/360 = 7 4/9%

96) C
SI = (8800 x 5 x 2) / (100 + 5 x 2)
= (8800 x 10) / 110
= ₹ 800

97) C
Investment of Ramesh = 10% of 10000 = ₹ 1000
After 3 yr = 2500 - 1000 = ₹ 1500
⇒ 1500 = (1000 x R x 3)/100
∴ R = (1500 x 100) / (1000 x 3) = 50%
∴ 4 times of 50% = 200%

98) A
Let the sum lent at 5% = P
∴ Sum lent at 8% = (1550 - P)
Then, [(P x 5 x 4)/100] + [{(1550 - P) x 8 x 4}/100] = 400
⇒ 20P - 32P + 1550 x 32 = 40000
⇒ - 12P + 49600 = 40000
⇒ - 12P = - 9600
∴ p = ₹ 800
Sum lent at 8% = 1550 - 800 = ₹ 750
∴ Required ratio = 800 : 750 = 16 : 15

www.ibpsguide.com | estore.ibpsguide.com | www.sscexamguide.com


324
Complete Quantitative Aptitude Questions

99) B
Let the principal be P.
Then, according to the question,
(P x T x T) / 100 = P/144 [∵ time and rate are equal]
⇒ T^2 = 100/144
∴ T = 10/12 = 5/6 %

100) C
If the sum be ₹ P, then
(2240 - P) = (P x 4 x 3)/100
⇒ 2240 = 12P/100 + P
⇒ 2240 = 112P/100
∴ P = (2240 x 100)/112 = ₹ 2000
Now, required interest,
SI = PRT/100 = [{2000 x (7/2) x (1/2)} /100]
= ₹ 35

101) 2/3 part of my sum is lent out at 3%, 1/6 part is Lent out at 6% and remaining part is lent out 12% All the
three parts are lent out at simple interest. If the annual income is ₹ 25, what is the sum?
a) 500
b) 650
c) 600
d) 450
e) None of these

102) A sum of ₹ 1521 lent out two parts in such a way that the interest on one part at 10% for 5 yr is equal to
that of another part at 8% for 10 yr. What will be the two parts of sum ?
a) 926 and 595
b) 906 and 615
c) 916 and 605
d) 936 and 585
e) None of these

103) Pratap borrowed some money from Arun at simple interest. The rate of interest for the first 3 years was
12% for the next 5 years was 16% and beyond this it was 20%. If the simple interest for 11 years was more
than the money borrowed by Rs. 6080. What was the money borrowed?

www.ibpsguide.com | estore.ibpsguide.com | www.sscexamguide.com


325
Complete Quantitative Aptitude Questions

a) Rs. 7550
b) Rs. 8500
c) Rs. 8000
d) Rs. 9000
e) None of these

104) A person closes his account in an investment scheme by withdrawing ₹ 10000. One year ago, he had
withdraw ₹ 6000. Two years ago, he had withdrawn ₹ 5000. Three years ago, he had not withdrawn any
money. How much money had he deposited approximately at the time of opening the account 4 yr ago, if the
annual compound interest is 10% ?
a) 15600
b) 16500
c) 17280
d) 16780
e) None of these

105) The simple interest on a certain sum of money for 2 1/2 yr at 12% per annum is ₹ 20 less than the
simple interest on the same sum for 3 1/2 yr at 10% per annum. Find the sum.
a) 800
b) 750
c) 625
d) 400
e) None of these

106) Harsha makes a fixed deposit of ₹ 20000 in Bank of India for a period of 3 yr. If the rate of interest be
13% SI per annum charged half - yearly, what amount will he get after 42 months?
a) 27800
b) 28100
c) 29100
d) 30000
e) None of these

107) Subraja borrowed some money at the rate of 6% per annum for the first 3 yr. at the rate of 9% per
annum for the next 5 yr and a the rate of 13% per annum for the period beyond 8 yr. If she pays a total
interest of ₹ 8160 at the and of 11 yr how much money did she borrow ?
a) 12000
b) 10000

www.ibpsguide.com | estore.ibpsguide.com | www.sscexamguide.com


326
Complete Quantitative Aptitude Questions

c) 8000
d) 12000
e) Data is inadequate

108) Reena had ₹ 10000 with her, out of this money she lent some money to Akshay for 2 yr at 15% simple
interest. She lent remaining money to Brijesh for an equal number of years at the rate of 18%. After 2 yr
Reena found that Akshay had given her 360 more as interest as compared to Brijesh. The amount of money
which Reena had lent to Brijesh must be
a) 4000
b) 2500
c) 3500
d) 4200
e) None of these

109) Mr. Pawan invests an amount of ₹ 24200 at the rate of 4% per annum for 6 yr to obtain a simple
interest, later he invests the principal amount as well as the amount obtained as simple interest for another 4
yr at the same rate of interest. What amount of simple interest will be obtained at the end of the last 4 yr ?
a) 4800
b) 4850.32
c) 4801.28
d) 4700
e) None of these

110) Harish invested certain sum in three different schemes P, Q and R with the rates of interest 10% per
annum, 12% per annum and 15% per annum, respectively. If the total interest accrued in 1 yr was ₹ 3200
and the amount invested in scheme R was 150% of the amount invested in scheme P and the amount
invested in R is 240% of amount inveseted in P. what was the amount invested in scheme Q ?
a) 8000
b) 9000
c) 5000
d) 3050
e) None of these

101) A
[(2P/3) x 3%] + [(P/6) x 6%] + [1 - ( 2/3 + 1/6 )] P x 12% = 25
⇒ [(2P/3) x (3/100)] + [(P/6) x (6/100)] + [1 - 4 + 1/6] x (12P/100) = 25
⇒ 2P/100 + P/100 + 2P/100 = 25

www.ibpsguide.com | estore.ibpsguide.com | www.sscexamguide.com


327
Complete Quantitative Aptitude Questions

⇒ 5P = 2500,
∴ P = 500

102) D
(P x 5 x 10)/100] = [{(1521 - P) x 10 x 8}/100]
⇒ 5P = 12168 - 8P
⇒ 13P = 12168
⇒ P = ₹ 936
So second part = 1521 - 936 = ₹ 585

103) C
⇒ P + 6080 = (P x 12 x 3) / 100 + (P x 16 x 5) / 100 + (P x 20 x 3) / 100
⇒ P + 6080 = (36P + 80P + 60P) / 100
⇒ 100 x (P + 6080) = 176P
∴ P = 608000 / 76 = 8000

104) A
After one year he had P + (P x 10 x 1)/100 = ₹ 11P/10
After two years, he had
11P/10 + (11P/10 x 10 x 1)/100 = ₹ 121P/100 ...(i)
After withdrawn ₹ 5000 from ₹ 121P/100, the balance
= ₹ (121P - 500000)/100
After 3 yr, he had
(121P - 500000)/100 + [(121P - 500000)/100 x 10 x 1]/100
= 11(121P - 500000)/100 ... (ii)
After withdrawn ₹ 6000 from amount (ii) the balance
= (1331P/1000 - 11500)
∴ After 4 yr, he had ₹ (1331P - 5500000)/1000 + 10% of ₹ (1331P - 5500000)/1000
= ₹ (11/10) x (1331P/1000 - 11500) ... (iii)
After withdrawn ₹ 10000 from amount (iii) the balance =0
∴ 11/10(1331P/1000 - 11500) - 10000 = 0
⇒ P = ₹ 15470

105) D
[(P x 10 x 7)/(100 x 2)] - [(P x 12 x 5)/(100 x 2)] = 20
⇒ (7P/20) - (3P/10) = 20
∴ P = 20 x 20 = ₹ 400

www.ibpsguide.com | estore.ibpsguide.com | www.sscexamguide.com


328
Complete Quantitative Aptitude Questions

106) C
SI = (20000 x 13 x 7)/(100 x 2) = ₹ 9100
∴ Amount (A) = 20000 + 9100 = 29100

107) C
(P x 6 x 3)/100] + [(P x 9 x 5)/100] + [(P x 13 x 3)/100] = 8160
⇒ (18P + 45P +39P) / 100 = 8160
⇒ 102P/100 = 8160
⇒ P = (8160 x 100)/102= 8000

108) A
SI for Akshay = (P x 15 x 2)/100 = 3P/10
SI for Brijesh = {(10000 - P) x 18 x 2}/100 = 9/25 (10000 - P)
According to the given condition, (3P/10) - [(9/25) x ( 10000 - P ) = 360
[as SI (Akshay) - SI (Brijesh) = 360]
⇒ (3P/10) - 3600 + 9P/25 = 360
⇒ 3P/10 + 9P/25 = 360 + 3600 = 3960
⇒ 33P/50 = 3960
⇒ P = 3960 x 50/33
⇒ P = 6000
∴ The amount of money lent to Brijesh
= 10000 - 6000 = 4000

109) C
SI = (P x R x T) /100 = (24200 x 4 x 6) / 100 = 5808
SI = 24200 + 5808 = 30008
In the case II,
SI = (30008 x 4 x 4) / 100 = 4801.28

110) C
[(a x 10 x 1)/100] + [(b x 12 x 1)/100] + [(c x 15 x 1)/100] = 3200
⇒ 10a + 12b + 15c = 320000 .....(i)
Now, c = 240% of b = 12b/5 ....(ii)
and c = 150% of a = 3a/2
⇒ a = 2c/3 = (2/3 x 12/5) b = 8b/5 .....(iii)
From Eqs. (i), (ii) and (iii), we get

www.ibpsguide.com | estore.ibpsguide.com | www.sscexamguide.com


329
Complete Quantitative Aptitude Questions

16b + 12b + 36b = 320000


⇒ 64b = 320000
∴ b = 5000
∴ Sum invested in scheme Q = ₹ 5000

111) Income of Shantanu was ₹ 4000. In the first 2 yr. his income decreased by 10% and 5% respectively
but in the third year, the income increased by 15%. What was his income at the end of third year?
a) 3933
b) 4000
c) 3500
d) 3540
e) None of these

112) An amount is invested in a bank at compound rate of interest. The total amount, including interest, after
first and third year is ₹ 1200 and ₹ 1587, respectively. What is the rate of interest?
a) 10 %
b) 3.9 %
c) 12 %
d) 15 %
e) None of these

113) Find what is that first years in which a sum of money will become more than double in amount if put out
at compound interest at the rate of 10% per annum ?
a) 6th years
b) 7th years
c) 8th years
d) Data inadequate
e) None of these

114) The difference of compound interest on Rs.800 for 1 year at 20% per annum when compound half yearly
and quarterly is ?
a) Nil
b) Rs. 2.50
c) Rs. 4.40
d) Rs. 6.60
e) None of these

www.ibpsguide.com | estore.ibpsguide.com | www.sscexamguide.com


330
Complete Quantitative Aptitude Questions

115) The population of a country is 10 crore and it is the possibility that the population will become 13.31
crore in 3 yr. What will be the annual rate percent on this growth?
a) 8%
b) 12.7%
c) 10%
d) 15%
e) None of these

116) A sum of ₹ 8448 is to be divided between A and B who are respectively 18 and 19 yr old, in such a way
that if their shares be invested at 6.25 % per annum at compound interest, they will receive equal amounts on
attaining the age of 21 yr. The present share of A is
a) 4225
b) 4352
c) 4096
d) 4000
e) None of these

117) On a certain sum of money, compound interest earned at the end of three years = Rs. 1456. Compound
interest at the end of two years is Rs. 880. Compute the principal invested.
a) Rs. 2,400
b) Rs. 2,800
c) Rs. 2,000
d) Rs. 1,600
e) None of these

118) The cost of car, purchased 2 years ago, depreciates at the rate of 20% every year. If its present worth is
Rs.315600, find :
(i) its purchase price
(ii) its value after 4 years.
a) 493125 and 201884
b) 582125 and 221384
c) 482612 and 332172
d) 493215 and 210884
e) None of these

www.ibpsguide.com | estore.ibpsguide.com | www.sscexamguide.com


331
Complete Quantitative Aptitude Questions

119) Ram invests Rs. 10,000 for 1 year at a rate of 10% per annum compounded yearly and Sita invests the
same amount for same time at same rate per annum compounded half yearly. What is the difference between
the interests earned by both?
A) Rs. 25.50
B) Rs. 25
C) Rs.20.50
D) Rs.23.75
e) None of these

120) What sum will be amount to Rs.30000 at CI in 3 years, if the rate of interest for 1st, 2nd and 3rd year
being 10%, 20% and 30% respectively?
A) 17482.5
B) 20145
C) 16524
D) 17000
E) None of these

111) A
= P(1 - R1/100)(1 - R2/100)(1 + R3/100)
= 4000(1- 10/100) (1 - 5/100) (1 + 15/100)
= 4000 x (9/10) x (19/20) x (23/20)
= 9 x 19 x 23
= 3933

112) D
Amount after 1st yr = ₹ 1200
⇒ P(1 + R/100) = 1200 ...(i)
Amount after 3rd yr = 1587
⇒ P(1 + R/100)3 = 1587 ...(ii)
On dividing Eq. (ii) from Eq. (i), we get
(1 + R/100)2 = 1587/1200 = 529/400
⇒ 1 + R/100 = 23/20
⇒ R/100 = 3/20
∴ R = 15 %

113) C
Here, P(1 + 10/100)t > 2P

www.ibpsguide.com | estore.ibpsguide.com | www.sscexamguide.com


332
Complete Quantitative Aptitude Questions

⇒ (11/10)t> 2
When t = 8 → (11/10)8 = 2.14358
t =7 → (11/10)7 = 1.9487
By trial, [11 x 11 x 11 x 11 x 11 x 11 x 11 x 11] / [10 x 10 x 10 x 10 x 10 x 10 x 10 x 10] >2
Hence, the first years in which sum of money will become more than double in amount is 8th year

114) C
C.I. when reckoned half yearly
= Rs.[800 x (1 + 10/100)2 - 800]
= Rs. 168
C.I.when reckoned quarterly
= Rs.800[(1 + 5/100)4 - 1]
= 800[(194481 - 160000)/160000]
= 34481/200
= Rs.172.40
∵ Required difference =Rs.(172.40 - 168)
= Rs.4.40

115) C
13.31 = 10 (1 + R/100 )3
⇒ 1331/1000 = (1 + R/100 )3
⇒ (11/10)3 = (1 + R/100 )3
⇒ 1 + R/100 = 11/10
⇒ R/100 = 11/10 - 1 = 1/10
∴ R = 10%

116) C
N(1 + 6.25/100)3 = (8448 - N) x (1 + 6.25/100)2
⇒ 1 + 6.25/100 = (8448 - N)/N
⇒ 1 + 1/16 = (8448 - N)/N
⇒ 17/16 = (8448 - N)/N
⇒ 17N = 135168 - 16N
⇒ N = 4096

117) C
r^2 + 3r + 3) x 440 = (r + 2) x 728
(r^2 + 3r + 3) x 55 = (r + 2) x 91

www.ibpsguide.com | estore.ibpsguide.com | www.sscexamguide.com


333
Complete Quantitative Aptitude Questions

55r^2 + 165r + 165 = 91r + 182


55r^2 + 74r -17 = 0
55r^2 + 85r - 11r - 17 = 0
5r (11r + 17) -1 (11r + 17) = 0
r = 0.2 or a negative number. Or, r has to be 20%.
2000

118) E
Here, V = present value = Rs315600, t = 2 years, r = 20%,
V0 = purchase price = ?
Using V = V0(1 – r /100)n,
315600 = V0 (1 – 20/100)2
or, 315600 = V0 (80/100)2 = V0 (4×4) (5×5) =V0(16/25)
or, V0 = 315600×25/16 = Rs.493125. [Ans.]
(ii) Again, V0 = Rs.493125, V = value after 4 years, t = 4years, n = 4, r = 20%.
Using, V = V0(1 – r /100)n
V = 493125 (1 – 20/100)4
= 493125 (80/100)4
= 493125 (256/625)
= Rs.201984.

119) B
Amount = 10000.[1 + 10/100]1 = Rs.10000 x 11/10 = Rs.11000
For Sita:
Amount= Rs. 10000.[1 +(10/2)/100]2 = 10000 x (21/20)2 = Rs.11025
Difference = Rs.(11000-11025) = Rs.25

120) A
30000= p(1 + 10/100)(1 + 20/100)(1 + 30/100)
= p (110/100) x (120/100) x (130/100)
p = 30000x 100 x 100 x 100 / (110 x 120 x 130)
p = 17482.5

121) David invested certain amount in 3 different schemes A,B and C with the rate of interest 10% p.a,12%
p.a and 15% p.a(simple interest) respectively. If total accrued in one year was Rs.3200 and the amount
invested in C was 150% of the amount invested in A and 240% of the amount invested inscheme B, what was
the amount invested in scheme B?

www.ibpsguide.com | estore.ibpsguide.com | www.sscexamguide.com


334
Complete Quantitative Aptitude Questions

a) Rs.5000
b) Rs.6500
c) Rs.8000
d) Cannot be determined
e) None of these

122) bank offers 5% compound interest calculated on half-yearly basis. A customer deposits Rs. 1600 each
on 1st January and 1st July of a year.
At the end of the year, the amount he would have gained by way of interest is:
A) 123
B) 122
C) 121
D) 120
E) None of these

123) A lent Rs. 5000 to B for 2 years and Rs. 3000 to C for 4 years on simple interest at the same rate of
interest and received Rs. 2200 in all from both of them as interest. The rate of interest per annum is:
A) 5 %
B) 7%
C) 10 %
D) 12%
e) None of these

124) person borrows Rs. 5000 for 2 years at 4% p.a. simple interest. He immediately lends it to another
person at 6.25% p.a. for 2 years.Find his gain in the transaction per year.
A) Rs. 112.50
B) Rs. 175
C) Rs. 150
D) Rs. 125.50
E) None of these

125) Vinay deposited Rs. 3,000 at 10% simple interest for 2 years. How much more money will Vijay have in
her account at the end of two years, if it is compounded semi annually.
a) Rs. 50
b) Rs. 40
c) Rs. 77.50
d) Rs. 85.50

www.ibpsguide.com | estore.ibpsguide.com | www.sscexamguide.com


335
Complete Quantitative Aptitude Questions

e) None of these

126) here is 100% increase to an amount in 8 year, at simple interest. Find the compound interest of Rs. 8000
after 2 year at the same rate of interest .
a) Rs. 2500
b) Rs. 2000
c) Rs. 2250
d) Rs. 2125
e) None of these

127) sum of money becomes 25 times of itself in 20 years at compound interest, compounded yearly. In how
many years can the same sum become 5 times of itself?
a) 4 years
b) 10 years
c) 5 years
d) 15 years
e) None of these

128) a sum of money, simple interest for 2 years is Rs 660 and compound interest is Rs 696.30, the rate of
interest being the same in both cases.
a) 5 %
b) 12%
c) 10%
d) 11%
e) None of these

129) What would be the compound interest accrued on an amount of Rs. 7,400 @13.5 p.c.p.a.at the end of
two years? (Rounded off to two digits after decimal)
a) Rs. 2,136.87
b) Rs. 2,306.81
c) Rs. 2,032.18
d) Rs. 2,132.87
e) None of these

130) Determine the compound amount and compound interest on Rs.1000 at 6% compounded semi-annually
for 6 years. Given that (1 + i)n = 1.42576 for i = 3% and n = 12.
a) 420

www.ibpsguide.com | estore.ibpsguide.com | www.sscexamguide.com


336
Complete Quantitative Aptitude Questions

b) 500
c) 600
d) 425.76
e) None

121) A
x,y and z be the amounts invested in A,B and C respectively. Then
10x+12y+15z/100=3200
=>z=150/100 x,
z=240/100 y
Solving and substituting we get=>64y=3200*100=>5000

122) C
= Rs. [1600×(1+ 5/200)^2 + 1600 × (1+5/200)]
= Rs. 3321
So CI = Amount- Principal
= Rs. 3321 – Rs. 3200 = Rs. 121

123) C
5000 x r x 2)/100 +(3000 x r x 4)/100 = 2200.
100R + 120R = 2200
R = 2200/220= 10.
Rate = 10%.

124) A
= Rs. [{(5000×6.25×2)/100} – {(5000×4×2)/100}]
= Rs. (625- 400) = Rs. 225.
So gain in 1 year = Rs.225/ 2 = Rs. 112.50

125) E
=3000(1+5/100)^4 =3000(21/20)^4=3646.5
C.I.=3646.5-3000=646.5
Required Difference=646.5-600=46.5

126) D
P = (PXRX8)/100, R=12.5%
A=8000(1+12.5/100)^2=10125

www.ibpsguide.com | estore.ibpsguide.com | www.sscexamguide.com


337
Complete Quantitative Aptitude Questions

C.I.=10125-8000=2125

127) B
The sum is becoming 25 times of itself in 20 years, so it will become 5 times of itself in 10 years

128) D
Difference between C.I and S.I for 2 years = 36.30
S.I. for one year = 330
S.I. on Rs 330 for one year = 36.30
So R% = {100*36.30}/{330*1} = 11%

129) D
=7400[{1+(13.5/100)²}-1]
=7400[1.288225-1]=7400×0.288225
=Rs 2132.87

130) D
i =0.06/2
= 0.03; n = 6 × 2 = 12
P = 1,000
Compound Amount (A12) = P ( 1 + i )n
= Rs. 1,000(1 + 0.03)12
= 1,000 × 1.42576
= Rs. 1,425.76
Compound Interest = Rs. (1,425.76 – 1,000)
= Rs. 425.76

131) What annual rate of interest compounded annually doubles an investment in 7years? Given that 1+i 2
1/7= 1.104090
a) 10.41%
b) 10%
c) 11%
d) 12%
e) None

www.ibpsguide.com | estore.ibpsguide.com | www.sscexamguide.com


338
Complete Quantitative Aptitude Questions

132) A person opened an account on April, 2001 with a deposit of Rs.800. The account paid 6% interest
compounded quarterly. On October 1 2001 he closed the account and added enough additional money to
invest in a 6 month time-deposit for Rs. 1,000, earning 6% compounded monthly.
(a) How much additional amount did the person invest on October 1?
a) 175.82
b) 180
c) 190
d) None
e) None

133) Rs. 5,000 is invested in a Term Deposit Scheme that fetches interest 6% per annum compounded
quarterly. What will be the interest after one year? What is effective rate of interest?
a) 6.13
b) 6.03
c) 6.23
d) 6.33
e) None of these

134) Shantha bought Rs 7000 at simple interest from village moneylender. At the end of 3 years he again
borrows Rs3000and closes his account after paying Rs 4615 as interest after 8years from the time he made
the first borrowing. Find the rate of interest.
a) 3.5
b) 4.5
c) 5.5
d) 6.5
e) None

135) Raghav borrows Rs.2550 to be paid back with compound interest at the rate of 4% per annum by the
end of 2 years in two equal yearly instalments. How much will each instalment be?
a) Rs.1275
b) Rs.1283
c) Rs.1352
d) Rs.1377
e) None of these

www.ibpsguide.com | estore.ibpsguide.com | www.sscexamguide.com


339
Complete Quantitative Aptitude Questions

136) The difference between simple interest and compound interest on Rs.1200 for one year at 10% per
annum reckoned half-yearly is :
a) Rs.2.50
b) Rs.3
c) Rs.3.75
d) Rs.4
e) None of these

137) if the annual rate of simple interest increases from 10% to 12.5% .Then a man’s yearly income from an
investment increases by Rs.1250. His principle amount is:
a) Rs,45000
b) Rs.50,000
c) Rs. 60,000
d) Rs.65,000
e) None of these

138) Simple interest on a certain sum of money for 3 years at 8% per annum is half the compound interest on
Rs. 4000 for 2 years at 10% per annum. The sum placed on simple interest is:
a) Rs.1550
b) Rs.1650
c) Rs.1750
d) Rs.2000
e) None of these

139) If the simple interest on a sum of money for 2 years at 5% per annum is Rs.50, what is the compound
interest on the same at the same rate and for the same time?
a) Rs. 52
b) Rs. 51.25
c) Rs. 54.25
d) Rs. 60
e) None of these

140)The rate of simple interest in two banks is in the ratio of 4 : 5 . Amith wants to deposit his total saving in
these two banks in such a way that he should receive equal half yearly interest from both. He should deposits
the saving in the banks in the ratio of:
a) 2 : 5
b) 5 : 4

www.ibpsguide.com | estore.ibpsguide.com | www.sscexamguide.com


340
Complete Quantitative Aptitude Questions

c) 5 : 3
d) 4 : 5
e) None of these

131) A
Solution:
If the principal be P then An
= 2P.
Since An
= P(1+ i)n
2P = P (1 + i )7
2 1/7 = ( 1 + i )
1.104090 = 1 + i
i = 0.10409
Required rate of interest = 10.41% per annum

132) A
Given that (1 + i)n
is 1.03022500 for i=1½ % n=2 and (1+ i)n
is 1.03037751 for i = ½ % and
n = 6.
initial investment earned interest for April-June and July- September quarter
i.e. for two quarters. In this case i = 6/4 = 1½ % = 0.015, n
n=
12
64=2
and the compounded amount = 800(1 + 0.015)2
= 800 × 1.03022500
= Rs. 824.18
The additional amount invested = Rs. (1,000 - 824.18)
= Rs. 175.82

133) A
5000(1+6/400)^4=306.82
For effective rate of interest using I = PEt we find
306.82 = 5,000 × E × 1.
= 0.0613 or 6.13%

www.ibpsguide.com | estore.ibpsguide.com | www.sscexamguide.com


341
Complete Quantitative Aptitude Questions

134) D
The interest would be paid on 7000 for 3years+10000 for 5years 6.5%
=>7000*3*6.5/100+10000*6.5*3/100=1365+3250=4615
= 6.5%

135) C
x/(1+4/100) + x/(1+4/100)2 = 2550 = 25x/26 + 625x/676 = 2550
1275x = 2550 *676
x = (2550*676)/1275 = 1352.
:. Value of each instalment = Rs.1352

136) B
S.I = (1000 *10 *4)/100 = Rs.400
C .I =1200 *1+5/100)2 –1200 = 123.
Difference = Rs.(123-120) = Rs.3

137) B
(x*25/2*1/100) – (x*10*1/100)= 1250
25x-20x = 250000 ; x=50000

138) C
C.I = 4000 * (1+10/100)2 –4000
= 4000 *11/10 + 11/10 –4000 = Rs.840
:. Sum = Rs.(420 * 100)/ (3 *8) = Rs.1750

139) B
Simple interest for 2 years = Rs.50 ie. For 1 years Rs. 25. . In the first year the S.I and C.I are
Same ie.Rs. 25. So in the 2nd year in C.I calculated for 1 years interest also. So in second years for
Rs.25 interest is 25*5/100 = 1.25 . So total C.P = 51.25.

140) B
X +4x * 1/2 *1/100 = Y + 5x * 1/2 *1/100 or X/Y = 5/4 i.e. X : Y = 5 : 4

www.ibpsguide.com | estore.ibpsguide.com | www.sscexamguide.com


342
Complete Quantitative Aptitude Questions

141) Rakesh invested an amount of Rs.12000 at the rate of 10% simple interest and another amount at the
rate of 20% simple interest. The total interest earned at the end of one year on the amount invested became
14 p.c.p.a. Find the total amount invested .
a) Rs.20000
b) Rs.22000
c) Rs.24000
d) Rs.25000
e) None of these

142) If the simple interest on a sum of money at twelve percent per annum for two years is Rs.3800,
compound interest on the same sum for the same period at the same rate of interest is
a) Rs.4028
b) Rs.4100
c) Rs.4128
d) 4228
e) None of these

143) A sum of Rs.100 is lent at simple interest of 3% p.a. for the first month, 9% p.a. for the second month,
27% p.a. for the third month and so on. What is the total amount of interest earned at the end of the year
approximately
a) Rs.797160
b) Rs.791160
c) Rs.65930
d) Rs.66430
e) None of these

144) Mr.Govind invested an amount of Rs.13900 divided in two different schemes S1 and S2 at the simple
interst rate of 14% p.a. and 11% p.a. respectively. If the total amount of simple interest earned in two years
was Rs.3508, what was the amount invested in Scheme S2?
a) Rs.6400
b) Rs.6500
c) Rs.7200
d) Rs.7500
e) None of these

145) The simple interest on a certain sum of money for 2 l/2 years at 12% per annum is Rs. 40 less than the
simple interest on the same sum for 3 ½ years at 10% per annum. Find the sum.

www.ibpsguide.com | estore.ibpsguide.com | www.sscexamguide.com


343
Complete Quantitative Aptitude Questions

A) 800
B) 850
C) 900
D) 950
E) None of these

146) There is 80% increase in an amount in 8 years at simple interest. What will be the compound interest of
Rs. 14,000 after 3 years at the same rate?
A) Rs.3794
B) Rs.3714
C) Rs.4612
D) Rs.4634
E) None of these

147) Simple interest on a certain sum of money for 4 years at 5% per annum is half the compound interest on
Rs. 3000 for 2 years at 10% per annum. The sum placed on simple interest is:
A) Rs.1575
B) Rs. 2200
C) Rs. 1200
D) Rs. 1625
E) None of these

148) A man borrows Rs. 20,000 at 10% compound interest. At the end of every year he pays Rs. 2000 as part
repayment. How much does he still owe after three such installments?
A) Rs.24000
B) Rs.15000
C) Rs.20000
D) Rs.10000
E) None of these

149) If the compound interest on a certain sum for 2 years in Rs. 80.80 and the simple interest Rs. 80; then
the rate of interest per annum is
A) 2%
B) 1%
C) 3%
D) 4%
E) None of these

www.ibpsguide.com | estore.ibpsguide.com | www.sscexamguide.com


344
Complete Quantitative Aptitude Questions

150) A sum of Rs. 6600 was taken as a loan. This is to be repaid in two equal annual instalments. If the rate
of interest be 20% compounded annually then the value of each instalment is
A) Rs. 4320
B) Rs. 2220
C) Rs. 4400
D) Rs.4500
E) None of these

141) A
(12000* 10 * 1)/100 + (x*20 *1)/100 = ((12000 +x)* 14*1)/100
= 1200 +x/5 = (168000 +14x)/100
600000 + 100x = 840000 +70 x
30x = 240000; X = 8000
Total investment = 12000 +8000 = Rs.20000

142) A
S I for 2 years = 3800 ie for one year =1900
The compound interest for Rs.1900 for the second year = 1900*12/100 =228
The CI for two years 3800+228 = 4028

143) D
I = P/100 * 1[3/12 + 9/12 +27/12…. 312/12
Where P = 100; I = 1/12 (3+9+…..312)
I = 1/12(3(312-1))/3-1
= 531440* 3/12*2 =Rs.66430

144) A
(x*14*2)/100 + ((13900-x)*11*2)/100 = 3508;
28x-22x= 3350800-(13900*22);
6x = 45000; x= 7500
So sum invested in Scheme S2 = Rs. (13900-7500) = Rs.6400

145) A
(x*10*7)/(100*2))-( (x*12*5)/(100*2)) = 40
7x/20)-(3x/10)=40
x = (40 * 20) = 800

www.ibpsguide.com | estore.ibpsguide.com | www.sscexamguide.com


345
Complete Quantitative Aptitude Questions

146) D
P=100,si=80
=>r=10%
Ci:=>10%14000=1400
10%1400=140=>1400+140=1540
10%1540=154=>1540+154=1694
Ci=1400+1540+1694=4634`

147) A
CI:=>10%3000=300
10%300=30=>300+30=330
Ci=300+330=630
SI=630/2=315
20===100
315===?=>315*5=1575

148) C
2000(110/100)^3-[2000(110/100)^2+2000*(110/100)+2000)
=>20000

149) A
Shortly=>200+r/200=CI/SI
=>200+r/200=80.80/80
=>r=2%`

150) A
=>x [5/6+25/36]=6600
=>x=120*36=4320.

13. RATIO AND PROPORTION

1) Three friends Alice, Bond and Charlie divide $1105 amongst them in such a way that if $10, $20 and $15
are removed from the sums that Alice, Bond and Charlie received respectively, then the share of the sums
that they got will be in the ratio of 11 : 18 : 24. How much did Charlie receive?
a) $495
b) $510

www.ibpsguide.com | estore.ibpsguide.com | www.sscexamguide.com


346
Complete Quantitative Aptitude Questions

c) $480
d) $375
e) $360

2) The ratio of marks obtained by Vinod and Basu is 6:5. If the combined average of their percentage is 68.75
and their sum of the marks is 275, find the total marks for which exam was conducted.
A) 150
B) 200
C) 400
D) 450
E) None

3) In a pocket of A, the ratio of Rs.1 coins, 50p coins and 25p coins can be expressed by three consecutive
odd prime numbers that are in ascending order. The total value of coins in the bag is Rs 58. If the number of
Rs.1, 50p, 25p coins are reversed, find the new total value of coins in the pocket of A?
A) Rs 68
B) Rs 43
C) Rs 75
D) Rs 82
E) NONE

4) Joseph bought two varieties of rice, costing 5 cents per ounce and 6 cents per ounce each, and mixed
them in some ratio. Then he sold the mixture at 7 cents per ounce, making a profit of 20 percent. What was
the ratio of the mixture?
A) 1:10
B) 1:5
C) 2:7
D) 3:8
E) NONE

5) Manish, Rahul and Bharti have some stones with each of the. Five times the number of stones with Rahul
equals seven times the number of stones with Manish while five times the number of stones with Manish
equals seven times the number of stones with Bharti. What is the minimum number of stones that can be
there with all three of them put together?
A) 113
B) 109
C) 93

www.ibpsguide.com | estore.ibpsguide.com | www.sscexamguide.com


347
Complete Quantitative Aptitude Questions

D) 97
E) NONE

6) Rs.432 is divided amongst three workers A, B and C such that 8 times A’s share is equal to 12 times B’s
share which is equal to 6 times C’s share. How much did A get?
A) Rs.192
B) Rs.133
C) Rs.144
D) Rs.128
E) None

7) An outgoing batch of students wants to gift a PA system worth Rs 4,200 to their school. If the teachers,
offer to pay 50% more than the students and an external benefactor gives three times the teacher's
contribution, then how much should the teachers donate?
A) Rs 600
B) Rs 840
C) Rs 900
D) Rs 1,200
E) NONE

8) Two cogged wheels of which one has 32 cogs and other 54 cogs, work into each other. If the latter turns 80
times in three quarters of a minute, how often does the other turn in 8 seconds?
A) 48
B) 24
C) 38
D) 39
E) None of these

9) The monthly incomes of A and B are in the ratio 4:5, their expenses are in the ratio
5 : 6. If 'A' saves Rs.25 per month and 'B' saves Rs.50 per month, what are their respective incomes?
A) Rs.400 and Rs.500
B) Rs.240 and Rs.300
C) Rs.320 and Rs.400
D) Rs.440 and Rs.550
E) NONE

www.ibpsguide.com | estore.ibpsguide.com | www.sscexamguide.com


348
Complete Quantitative Aptitude Questions

10) IBM and KTC quote for a tender. On the tender opening day, IBM realizes that their quotations are in the
ratio 7 :4 and hence decreases its price during negotiations to make it Rs 1 Lakh lower than KTC's quoted
price. KTC realizes that the final quotes of the two were in the ratio 3:4. What was the price at which IBM won
the bid?
A) Rs 7 Lakh
B) Rs 4 Lakh
C) Rs 3 Lakh
D) Rs 1.5 Lakh

1) Answer: A
Let the sums of money received by A, B and C be x, y and z respectively.
Then x – 10 : y – 20 : z -15 is 11a : 18a : 24a
When $10, $20 and $15 are removed, we are removing a total of $45 from $1105.
Therefore,
11a+18a+24a=1105-45=1060
53a=1060
or a= 20
We know that z – 15 = 24a = (24 * 20) = 480
Therefore, z = 480 + 15 = $495

2) Answer: B
The sum of the marks=6x+5x=11x
But the sum of the marks is given as 275=11x. We get x=25x=25 therefore, Vinod’s marks is 6x=1506x=150
and Basu’s marks = 5x=125.
Therefore, the combined average of their marks % = 150+1252=137.5
If the total mark of the exam is 100 then their combined average of their percentage is 68.75
Therefore, if their combined average of their percentage is 137.5 then the total marks would be
137.568.75×100=200

3) Answer: D
Since the ratio of the number of Rs. 1, 50p and 25p coins can be represented by 3 consecutive odd numbers
that are prime in ascending order, the only possibility for the ratio is 3:5:7.
Let the number of Re1, 50p and 25p coins be 3k, 5k and 7k respectively.
Hence, total value of coins in paise
⇒ 100×3k+50×5k+25×7k=725k=5800
⇒ k=8.

www.ibpsguide.com | estore.ibpsguide.com | www.sscexamguide.com


349
Complete Quantitative Aptitude Questions

If the number of coins of Rs. 1,50p and 25p is reversed, the total value of coins in the
Bag (in paise) =100×7k+50×5k+25×3k=1025k (In above we find the value of k).
⇒ 8200p
= Rs 82.

4) Answer: B
1) 20% profit at the price of 7 cents means that cost of the mixture should be 70/12 cents per ounce (7 =
120% and x = 100% => x = 7*100/120)
2) Let the amount of 5 cent rice will be x and the amount of 6 cent one – y. The price of ounce of mixture will
be: (5x + 6y)/(x + y), which as we know should equal to 70/12
3) (5x + 6y)/(x + y) = 70/12 => x/y = 1/5

5) Answer: B

6) Answer: C
8 times AA’s share = 12 times BB’s share = 6 times CC’s share
Note that this is not the same as the ratio of their wages being 8:12:68:12:6
In this case, find out the L.C.M of 8, 12 and 6 and divide the L.C.M by each of the above numbers to get the
ratio of their respective shares.
The L.C.M of 8, 12 and 6 is 24.
Therefore, the ratio A:B:C= 24/8: 24/12: 24/6
A:B:C=3:2:4
The sum of the total wages = 3x + 2x + 4x = 432 => 9x = 432 or x = 48.
Hence, A who gets 3x will get 3 * 48 = Rs.144.

7) Answer: C
The ratio of the share students : teacher: benefactor=1:1.5 : 4.5
So the proportion to teacher's share = 1.5/7
Hence, the teachers would donate 1.5/7×4200=Rs 900

8) Answer: B

www.ibpsguide.com | estore.ibpsguide.com | www.sscexamguide.com


350
Complete Quantitative Aptitude Questions

Number of turns required= 80 × 54/32 × 8/45 = 24


24 times

9) Answer: A
Let AA's income be = 4x
AA's expenses, therefore = 4x–25
Let BB's income be = 5x
BB's expenses, therefore = 5x–50
We know that the ratio of their expenses = 5:6
⇒24x−150=25x−250
⇒ Therefore, x=100
A's income =4x=400 and B's income =5x=500

10) Answer: C
IBM initially quoted Rs 7x lakh. KTC quoted 4x lakh.
IBM's final quote =(4x−1) Lakh
Thus, (4x-1/4x) = 3/4 = x = 1
IBM's bid winning price = Rs 3 Lakh
So IBM wins the bid at 4x−1= Rs 3 lakh.

11) One year ago the ratio between Laxman and Gopal salary was 3:4. The individual ratios between their
last year’s and this year’s salary are 4:5 and 2:3 respectively. At present the total of their salary is Rs.4160.
The salary of laxman now is?
a) 1800
b) 1500
c) 2160
d) 2560
e) 1600

12) Rs 4830 is divided among Abhishek, Dishant and Prashant such that if Abhishek's share diminishes by Rs
5, Dishant's share diminishes by Rs 10 and Prashant's share diminishes by Rs 15, their shares will be in the
ratio 5:4:3. Find the Dishant's original share
A) 1610
B) 2010
C) 2410
D) 1590
E) NONE

www.ibpsguide.com | estore.ibpsguide.com | www.sscexamguide.com


351
Complete Quantitative Aptitude Questions

13) A, B and C play cricket. A's runs are to B's runs and B's runs are to C's as 3:2 , 3:2. They get altogether
342 runs. How many runs did A make?
A) 162
B) 108
C) 72
D) 78
E) None

14) On a certain day, the ratio of the passenger in the 1st class and the second class travelling by train is 1:3.
the ratio of the fares collected from each first class and second class passengers is 30:1. If the total amount
collected from all the passengers is Rs 1,320. Find the amount in Rs, collected from the second class
passengers.
A) 240
B) 360
C) 480
D) 120
E) None

15) If a carton containing a dozen mirrors is dropped, which of the following cannot be the ratio of the broken
mirror to the unbroken mirror?
a) 2 :1
b) 7:5
c) 3:2
d) 1:5
e) None of these

16) One year ago, the ratio between A's and B's salary was 4:5. The ratio of their individual salaries of last
year and present year are 3:5 and 2:3 respectively. If their total salaries for the present year is Rs 6800, the
present salary of A is
A) Rs 4080
B) Rs 3200
C) Rs 4533.40
D) Rs 2720
E) NONE

www.ibpsguide.com | estore.ibpsguide.com | www.sscexamguide.com


352
Complete Quantitative Aptitude Questions

17) A noodles merchant buys two varieties of noodles the price of the first being twice that of the second. He
sells the mixture at Rs 17.50 per kilogram thereby making a profit of 25% . If the ratio of the amounts of the
first noodles and the second noodles in the mixture is 2:3, then the respective costs of each noodles are
A) Rs 20, Rs 10
B) Rs 24, Rs 12
C) Rs 16, Rs 8
D) Rs 26, Rs 13
E) NONE

18) The ratio of incomes of Pankaj and Gauri is 3:5 and the ratio of their expenditures is 2:3. Who does save
more? (You have to assume that no one takes any loan from anywhere)
A) Pankaj
B) Gauri
C) Both save equally
D) Depends upon the incomes of Pankaj and Gauri
E) NONE

19) In a house, there are dogs, cats and parrot in the ratio 3:7:5. If the number of cats was more than the
number of dogs by a multiple of both 9 and 7, what is the minimum of pets in the house?
A) 945
B) 630
C) 252
D) 238
E) NONE

20) A sum of money is to be distributed among A, B, C, D in the proportion of 5:2:4:3. If C gets Rs. 1000 more
than D, what is B's share?
A) Rs. 500
B) Rs. 1500
C) Rs. 2000
D) Rs.2500
E) NONE

11) Answer: E
Let the salaries of Laxman and Gopal one year before be x1 ,y1 respectively.
∴ x1/y1 = 3/4 ......(1)
x2 + y2 = 4160 .....(2)

www.ibpsguide.com | estore.ibpsguide.com | www.sscexamguide.com


353
Complete Quantitative Aptitude Questions

From equations (1) and (2)


x2 = Rs. 1600.

12) Answer: A
Let actual share of Abhishek, Dishant and Prashant be A, D , P respectively.
A+D+P=4830
Hence, A's, D's and P's share are diminished by Rs 5, Rs 10 and Rs 15, their net share will be Rs.4800.
Dishant's diminished share = (4/12) * 4800 = Rs1600
Hence, Dishant actual share =Rs1600+Rs10= Rs 1610.

13) Answer: A
A:B = 3:2 = 9:6;
B:C = 3:2 = 6:4 (making B equal)
Therefore, A:B:C = 9:6:4
Therefore, the runs made by A = (9/19) X 342 = 162.

14) Answer: D
Let the number of passengers travelling by first class and second class be x and 3x respectively.
Lets the fares collected from each of the first class and second class passengers be 30y and y respectively.
Hence x(30y)+3x(y)=30xy+3xy=33xy=1320
xy=40.
Total amount collected from the second class =3xy=3×40= Rs 120.

15) Answer: C
The carton contains a dozen mirror Hence, when dropped, a few mirrors may break. Here, the ratio obtained,
no matter whatever is the number of broken mirrors, will always sum up its terms such that they divide 12
exactly. From the given choices, we add up terms of each ratio to check, if they divide 12 or not.
For 2 : 1, 2 + 1 = 3, which divides 12
For 7 : 5, 7 + 5 = 12, which divides 12
For 1 : 5, 1 + 5 = 6, which divides 12
For 11 : 1, 11 + 1 = 12, which divides 12
But for 3 : 2, 3 + 2 = 5, which does not divides 12.
Hence, 3 : 2 cannot be the ratio.
For dividing 12 into two whole numbers the sum of the terms of the ratio must be a factor of 12. So they
cannot be in the ratio of 3 : 2

www.ibpsguide.com | estore.ibpsguide.com | www.sscexamguide.com


354
Complete Quantitative Aptitude Questions

16) Answer: B
Given, the Ratio of A's last year and present year salary = 3:5.
Let salary be 3x and 5x respectively.
Also, the Ratio of B's in last year and present year salary = 2:3
Let salary be 2y and 3y respectively.
Given, 3x/2y=4/5.
=>15x=8y . --------------- (i)
Also, 5x+3y=6800. --------------- (ii)
From equation (i) and (ii), we get :-
y=1200 and x=640.
Hence, A's present salary is 5x=5×640=3200.

17) Answer: A

18) Answer: B
The ratio of income of Pankaj and Gauri is 3:5 Ratio of their expenditures is 2:3 i.e 3:4.5
Had the ratio of expenditures been 3:5, ratio of savings also would have been 3:5, but since ratio of their
expenditures is 3:4.5 only. Obviously
Savings of Gauri will be something more than 5/3 of savings of Pankaj.
Thus Gauri is the answer.

19) Answer: A
If three kinds of pets are taken be 3k,7k and 5k respectively, then 7k−3k=63p (where pp is any positive
integer).
As the number is a multiple of both 9 and 7, it has to be multiple of 63.
k = 63p/4
Minimum value of pp for which kk is a natural number is 4.

www.ibpsguide.com | estore.ibpsguide.com | www.sscexamguide.com


355
Complete Quantitative Aptitude Questions

Thus, k =63
Hence, the number of pets =15k= 945

20) Answer: C
Let the shares of A, B, C, D be Rs. 5x, Rs. 2x, Rs. 4x and Rs. 3x respectively.
Then, 4x−3x=1000
⇒x=1000.
B's share =Rs.2x=Rs.(2×1000)= Rs. 2000

21) The salary of two friends Ramu and Raju are in the ratio 4:5. If the salary of each one increases by
Rs.6000, then the new ratio becomes 48:55. What is Raju's present salary?
a) Rs.10500
b) Rs.10500
c) Rs.11500
d) Rs.12500

22) The number of candidates writing three different entrance exams is in the ratio 4:5:6. There is a proposal
to increase these numbers of candidates by 40%, 60% and 85% respectively. What will be the ratio of
increased numbers?
a) 14:15:16
b) 12:15:19
c)13:19:21
d) 14:16:19
E) None of these

23) The ratio of salary of two persons X and Y is 5:8. If the salary of X increases by 60% and that of Y
decreases by 35% then the new ratio of their salaries become 40:27. What is X's salary?
a) Rs.15000
b) Rs.12000
c) Rs.19500
d) Data inadequate.
e) NONE

24) Seven men, five women and eight children were given as assignment of distributing 2000 books to
students in a school over a period of three days. All of them distributed books on the first day. One of the
second day two women and three children remained absent and on the third day three men and five children

www.ibpsguide.com | estore.ibpsguide.com | www.sscexamguide.com


356
Complete Quantitative Aptitude Questions

remained absent. If the ratio of the number of books distributed in a day by a man, a woman and a child was
5 : 4 : 2 respectively, a total of approximately how many books were distributed on the second day?
a) 1000
b) 800
c) 650
d) 900
e) Cannot be determined

25) A bag contains one rupee, 50 paise and 25 paise coins in the ratio 2 : 3 : 5. Their total value is Rs. 144.
The value of 50-paise coins is:
a) Rs. 24
b) Rs. 36
c) Rs. 48
d) Rs. 72
e) None of these

26) Determine the ratio of the number of people having characteristic X to the number of people having
characteristic Y in a population of 100 subjects from the following table
People having X and Y are 20
People having X but not Y are 10
People having Y but not X are 30
People having neither X nor Y are 40.
a) 3 : 5
b) 3 : 2
c) 1 : 2
d) 2 : 3
e) None of these

27) The students in three classes are in the ratio 4 : 6 : 9. If 12 students are increased in each class, the ratio
changes to 7 : 9 : 12. Then the total number of students in the three classes before the increase is:
a) 95
b) 76
c) 100
d) 114
e) None of these

www.ibpsguide.com | estore.ibpsguide.com | www.sscexamguide.com


357
Complete Quantitative Aptitude Questions

28) If x is subtracted from the numbers 7, 31 and 199, then the remainders will be in continued proportion.
What is the value of x?
a) 5
b) 3
c) 8
d) 4
e) None of these

29)Two jars having a capacity of 3 and 5 litres respectively are filled with mixtures of milk and water. In the
smaller jar 25% of the mixture is milk and in the larger 25% of the mixture is water. The jars are emptied into a
10 litre cask whose remaining capacity is filled up with water. Find the percentage of milk in the cask.
a) 55%
b) 50%
c) 45%
d) 25%
e) None of these

30) Three friends A, B and C started a business by investing a sum of money in the ratio of 5 : 7 : 6. After 6
months C withdraws half of his capital. If the sum invested by ‘A’ is Rs 40,000, out of a total annual profit of
Rs 33,000, C’s share will be
a) Rs 9,000
b) Rs 12,000
c) Rs 11,000
d) Rs 10,000
e) None of these

21) Answer : B
Ratio their salary is 4:5
Let the original salary of Ramu and Raju be 4k and 5k respectively.
After increasing Rs.6000, the ratio becomes 48:55
That is,
(4k+6000)/(5k+6000) = 48/55
55(4k+6000) = 48(5k+6000)
220k+330000 = 240k+288000
20k= 42000
We have to find the original salary of Raju; that is, 5k.
If 20k = 42000 then 5k = 10500.

www.ibpsguide.com | estore.ibpsguide.com | www.sscexamguide.com


358
Complete Quantitative Aptitude Questions

Hence the required answer is Rs.10500

22) Answer : E
Given ratio of number of candidates is 4:5:6
Let the number of candidates for 3 exams be 4k, 5k and 6k respectively.
After increasing, number of candidates become (140% of 4k), (160% of 5k) & (185% of 6k)
That is, (140x4k)/100, (160x5k)/100 and (185x6k)/100
= 56k/10, 80k/10 and 111k/10
Now, the required new ratio is: 56k/100 : 80k/10 : 111k/10
= 56 : 80 : 111

23) Answer : D
Ratio of salary of X and Y is 5:8
Let the original salary of X and Y be Rs.5k and Rs.8k respectively.
After increasing 60%, new salary of X = 160% of 5k = 160x5k/100 = 80k/10 ...(1)
After decreasing 35%, new salary of Y = (100-35)% of 8k = 65% of 8k = 52k/10 ...(2)
Given that, new ratio is 40:27
That is, 80k/10 : 52k/10 = 40/27
This does not give the value of k; so that we cannot find X's exact salary.
Hence the answer is data inadequate.

24) Answer: C
Sol. Let the books distributed by man, a woman and a child be 5x, 4x and 2x respectively.
∴ No. of books distributed in 1st day
= 7×5x+5×4x+8×2x=71x
No. of books distributed in 2nd day
=7×5x+3×4x+5×2x=57x
And no. of books distributed in IIIrd day
=4×5x+5×4x+3×2x=46x
71x + 57x + 46x = 2000, x = 2000/174
57x=2000/174×57= 655
650 (Approx)

25) Answer: B
Sol. Let the number of one-rupee coins, 50-paise coins and 25-paise coins be 2k, 3k and 5k, respectively.
∴ 2k × 1 + 3k × 0.50 + 5k × 0.25 = 114
⇒ 2k + 1.50k + 1.25k = 114

www.ibpsguide.com | estore.ibpsguide.com | www.sscexamguide.com


359
Complete Quantitative Aptitude Questions

⇒ 4.75k = 114
⇒ k = 24.
50 paise value = 1.5x = 1.5*24 = 36

26) Answer: A
Number of people having characteristic X = 10 + 20 = 30
Number of people having characteristic Y = 20 + 30 = 50
Therefore Required ratio = 30 : 50 = 3 : 5

27) Answer: B
Let originally were 4x, 6x and 9x student there in classes receptively.
After 12 students increase in each student then students were 7x, 9x and 12x in each class respectively.
Now,
Total Students = 7x + 9x + 12x
4x + 6x + 9x + 3*12 = 28x
9x = 3*12
x = 4.
Then toatl number of student in three classes,
= 4x + 6x + 9x = 19x = 19 * 4 = 76.

28) Answer: B
(7-x)/(31-x)= (31-x)/(199-x)
gives x=3

29) Answer: D
0.75+3.75=4.5/10+8*100=4.5/18*100=25%

30) Answer: A
Sum invested by A , B and C is
5 * 12:7 * 12:6 * 6 + 3 * 6
or 60 : 84 : 54
or 10 : 14 : 9
:: Share of C = ( 9 / 33 ) * 33000 = Rs. 9000

31) In 1 kg mixture of sand and iron, 20% is iron. How much sand should be added so that the proportion of
iron becomes 10%?
a) 1 kg

www.ibpsguide.com | estore.ibpsguide.com | www.sscexamguide.com


360
Complete Quantitative Aptitude Questions

b) 200 gms
c) 800 gms
d) 1.8 kg
e) None of these

32) When 30 percent of a number is added to another number the second number increases to its 140 per
cent. What is the ratio between the first and the second number?
a) 3 : 4
b) 4 : 3
c) 3 : 2
d) 4 : 5
e) None of these

33) In Ram nagar Colony, the ratio of school going children to non-school going children is 5 : 4. If in the next
year, the number of non-school going children is increased by 20%, making it 35,400 what is the new ratio of
school going children to non-school going children?
a) 4 : 5
b) 3 : 2
c) 25 : 24
d) 6 : 7
e) None of these

34) The salaries of A,B and C are in the ratio 5:3:2.If the increments of 20% ,10% and 20% are allowed in
their salaries, then what will be the new ratio of their salaries ?
A) 22:11:9
B) 22:10:8
C) 20:11:8
D) 20:10:9
E) None of these

35)Two numbers are respectively 40% and 60% more than than the third number. The ratio of the two
number is
A) 6:7
B) 7:8
C) 8:7
D) 7:5
E) None of these

www.ibpsguide.com | estore.ibpsguide.com | www.sscexamguide.com


361
Complete Quantitative Aptitude Questions

36) Ratio of earnings of A and B is 3:5. If the earnings of A increase by 25% and those of B decrease by
30%, the new ratio of their earnings become 6:8.What is A’s earnings ?
A) 16,000
B) 34,000
C) 15,000
D) 32,000
E) Data inadequate

37) The sum of the three number is 68.If the ratio of the first to second is 3:2 and that of the second to the
third is 5:3 , then the second number is approximately
A) 21
B) 22
C) 23
D) 24
E) None of these

38) The ratio of prices of 2 dresses is 10:15.If the price of the first dress is increased by 10% and that of the
second dress by Rs.400 then the ratio of their prices is 4:7.What are the initial prices of the dresses ?
A) 910,1375
B) 920,1380
C) 930,1395
D) 940, 1410
E) None of these

39) The ratio of the number of boys to that of girls in a school is 3:2. If 30% of boys and 70% of girls appeared
in an examination, the ratio of students appeared in the examinations to that not appeared in the examination
is
A) 23:27
B) 22:25
C) 21:17
D) 18:17
E) None of these

40) In a bag there are coins of 25p, 10p and 5p in the ratio 1:2:3.If there are Rs.45 in all then find how many
25p coins are there?
A) 60

www.ibpsguide.com | estore.ibpsguide.com | www.sscexamguide.com


362
Complete Quantitative Aptitude Questions

B) 65
C) 70
D) 75
E) None of these

31) Answer: A
If we add x grams of sand, the total amount would be 1,000 + x grams of mixture and we need 200 grams of
iron to be 10% of that: 0.1(1,000 + x) = 200 --> x = 1,000.

32) Answer: B
of x=140% of y
or, y + 0.3x = 1.4y
or, 0.3x = 0.4y
∴ x : y = 0.4 : 0.3 = 4 : 3

33) Answer: C
5 : 4 → 5 : 4.8 → 25 : 24

34) Answer: C

35) Answer: B
Let third number = x
(140/100)x = (7/5)x
(160/100)x = (8/5)x
(7/5) : (8/5) = 35 : 40 = 7 : 8

36) Answer: E
Data inadequate
We cannot derive the earnings from the given details

37) Answer: B
A: B = 3:2
B:C = 5 : 3

www.ibpsguide.com | estore.ibpsguide.com | www.sscexamguide.com


363
Complete Quantitative Aptitude Questions

B:C =(5×[2/5]) : (3×[2/5])= 2: (6/5)


A:B:C = 3:2:(6/5) = 15:10:6
Second number = (10/31)×68 = 21.9 = 22

38) Answer: D
Let the initial price of the dress = 10x and 15x
Price of the 1st dress after increment = 10x × (110/100) = 11x
Price of the 2nd dress after increment = 15x+400
[11x/(15x+400)] = 4/7
77x = 60x + 1600
17x = 1600
X = 1600/17 = 94.11 = 94
So dresses will be 940, 1410

39) Answer: A
3:2 = 30:20
30% boys = (30/100) × 30= 9
70% girls = (70/100) × 20 = 14
(9+14) : (50-23) = 23 : 27

40) Answer: D
[(25x) + (10x × 2) + (5x × 3)] / 100 = 45
[25x + 20x + 15x] / 100 = 45
60 x = 4500
X = 4500/60 = 75

41) The ratio of two persons A and B salaries in the ratio 12:7 if 12% increased salary yearly then what will be
the B Salary after 2 yrs if salary of A is 12000.
A) 7808
B) 8700
C) 7,500
D) 8781

42) The price of a diamond is proportional to the square of its weight. The diamond accidentally fell and broke
into four pieces whose weights are in the ratio of 1:2:3:4. If the price fetched is Rs. 70,000 less than the
original price, find the original price?
a) Rs. 100,000

www.ibpsguide.com | estore.ibpsguide.com | www.sscexamguide.com


364
Complete Quantitative Aptitude Questions

b) Rs. 70,000
c) Rs. 160,000
d) Rs. 10800
e) Rs. 150,000

43) In a mixture of milk and water, their ratio is 5:6 respectively in the first container. And the same mixture
has ratio 7:2 respectively in the second container. What is the ratio should the mixture be extracted from each
container and poured in to the third container, so that the ratio of milk and water comes to 6:5 respectively in
the third container?
a) 99:253
b) 253:99
c) 243:88
d) 88:243
e) None of these

44) The ratio of the price of two houses was 17:24. Two years later, when the price of the first had risen by
20% while the price of second house increases by Rs.500 and their prices become 16:25. Find the original
prices of the two houses together?
a) Rs.2603
b) Rs.2503
c) Rs.2403
d) Rs.2303
e) None of these

45) A working partner gets 25% as his commissions after his commissions paid that is equal to Rs.7500, then
what is the total profit ?
A) Rs.32,000
B) Rs.30,000
C) Rs.37,500
D) Rs.40,000
E) None of these

46) Equal quantities of 3 mixtures of milk and water are mixed in the ratio 1:3, 2:3 and 3:4.The ratio of water
and milk in the new mixture is
A) 45:76
B) 151:269
C) 123:154

www.ibpsguide.com | estore.ibpsguide.com | www.sscexamguide.com


365
Complete Quantitative Aptitude Questions

D) 145:245
E) None of these

47) The ratio of income of X and Y is 4:3. The sum of their expenditure is Rs.12,000 and the amount of
savings is X is equal to the amount of expenditure of Y.What is the salary of Y?
A) 9000
B) 7000
C) 12000
D) 15000
E) None of these

48) When 7 is added to the numerator and denominator of the fraction, then the new ratio of numerator and
denominator becomes 13:19, what is the original ratio ?
A) 11:13
B) 7:9
C) 4:7
D) Can’t be determined
E) None of these

49) A school has 4 sections of class 12, such that half the number of students of 1st section, 1/3rd of 2nd
section, 1/4th of 3rd section and 1/5th of the 4th section are equal. If total number of students in class 12 is
420, find the number of students in sections 1st and 2nd.
A) 180
B) 120
C) 240
D) 150
E) 260

50) Brother A and B had some savings in the ratio 5:6. They decided to buy a gift for their sister, sharing the
cost in the ratio 4:5. After they bought, A is left with three-fourth of his amount, while B is left with Rs.497.
Then, the value of the gift is
a) 215
b) 115
c) 315
d) 415
e) None of these

www.ibpsguide.com | estore.ibpsguide.com | www.sscexamguide.com


366
Complete Quantitative Aptitude Questions

41) Answer: D
12:7 = 12,000 : 7,000
12% = B = 840……1st yr and B = 941 …..2nd yr
New inc B = 7000+840+941 = 8781

42) Answer: A
let price of diamond as kx^2 where k is a constant
total price for 4 pieces
kx^2[1+4+9+16]=30kx^2
price of original diamond=100kx^2
difference 70kx^2=$70000 or kx^2=1000
original price of the diamond=100*1000=100000

43) Answer: B
x litres of mixture be taken from the first container and poured into the third container.
Let y litres of mixture be taken from the second container and poured into the third container.
Quantity of milk in x litres=5/11 x
Quantity of milk in y litres=7/9 y
Quantity of milk in third container = 5/11 x+7/9 y
Similarly, quantity of water in third container,
=6/11 x+2/9 y
(5x/11+7y/9) :(6x/11+2y/9)=6 :5
((45x+77y)/99)/((54x+22y)/99)=6/5
=>324x-225x=385y-132y =>99x=253y =>x/y=253/99
=253 :99

44) Answer: A
Let, new price of the first house=17x×120/100=102x/5
Let, new price of the second house=24x+500
According to the question,
(102x/5) :(24x+500)=16 :25
((102x/5))/((24x+500 ) )=16/25 =>102x×5=(24x+500)16 =>x=8000/126=4000/63
Original price of first house=17×4000/63=1079.37~ Rs.1079
Original price of second house=24×4000/63=1523.81 ~ Rs.1524
Required answer, =1079+1524=2603
Answer=d) Rs.2603

www.ibpsguide.com | estore.ibpsguide.com | www.sscexamguide.com


367
Complete Quantitative Aptitude Questions

45) Answer: C
X=total profit
25/100*[x-7500] = 7500
x-7500 = 7500*100/25 =30,000
x = 37,500

46) Answer: B
Milk = 1/4 : 2/5 :3/7
= 35/140 :56/140 : 60/140
Quantity of milk in new mix = 35+56+60 = 151
Quantity of water in new mix = 140*3 = 420-151 = 269
M:W = 151:269

47) Answer: A
X’s saving = Expenditure of Y = S
4x-S + S = 12000
X = 3000
3x =3*3000 = 9000

48) Answer: D
X+7/y+7 = 13/19 x and y are different variable, so original fraction cannot be determined

49) Answer: D
Let number of students in 4 sections be A, B, C, D respectively. Then
1/2 of A = 1/3 of b = 1/4 of C = 1/5 of D
So A : B : C : D = 2 : 3 : 4 : 5 [When A/2 = B/3 = C/4, then ratio A: B : C = 2 : 3 : 4] So students in 1st
and 2nd section = [(2+3)/(2+3+4+5)] * 420 = 150

50) Answer: C
Let the savings of A and B are 5x, 6x and the share cost of gift are 4y, 5y respectively.
According to question,
For A, 5x-4y=3/4×5x =>x=16y/5
For B, 6x-5y=497 =>6×16y/5-5y=497 =>y=35
Cost of gift=4y+5y=9×35 =315

51)The ratio of the monthly salaries of A and B is in the ratio 15 : 16 and that of B and C is in the ratio 17 : 18.
Find the monthly income of C if the total of their monthly salary is Rs 1,87,450.

www.ibpsguide.com | estore.ibpsguide.com | www.sscexamguide.com


368
Complete Quantitative Aptitude Questions

A) Rs 66,240
B) Rs 72,100
C) Rs 62,200
D) Rs 65,800
E) Rs 60,300

52)The ratio of the incomes of A and B last year was 9 : 13. Ratio of their incomes of last year to this year is 9
: 10 and 13 : 15 respectively. The sum of their present incomes is Rs 50,000. What is the present income of
B?
A) Rs 32,000
B) Rs 24,000
C) Rs 20,000
D) Rs 30,000
E) None of these

53) A sum of Rs 315 consists of 25 paise, 50 paise and 1 Re coins in the ratio 3 : 4 :6. What is the number of
each kind of coin respectively?
A) 216, 144, 27
B) 108, 144, 216
C) 27, 72, 216
D) 120, 35, 108
E) 102, 150, 210

54) Rs 650 was divided among 3 children A,B,C in the ratio 2 : 4 : 7. Had it been divided in the ratio 1/2 : 1/4 :
1/7, who would have gained the most and by how much?
A) C, Rs 246
B) C, Rs 264
C) B, Rs 18
D) A, Rs 246
E) A, Rs 264

55) The ratio of the number of boys to the number of girls in a school is 6 : 5. If 20% of boys and 45% of girls
come by bus to school, what percentage of students opt transport other than bus to come to school?
A) 68 9/11%
B) 68 7/11%
C) 72 7/11%
D) 73%

www.ibpsguide.com | estore.ibpsguide.com | www.sscexamguide.com


369
Complete Quantitative Aptitude Questions

E) 73 5/11%

56) The incomes of A and B are in the ratio 1 : 2 and their expenditures are in the ratio
2 : 5. If A saves Rs 20,000 and B saves Rs 35,000, what is the total income of A and B?
A) Rs 30,000
B) Rs 70,000
C) Rs 90,000
D) Rs 60,000
E) Rs 80,000

57) Rs 5750 is divided among A, B, and C such that if their share be reduced by Rs 10, Rs 15 and Rs 25
respectively, the reminder amounts with them shall be in the ratio 4 : 6 : 9. What was C’s share then?
A) Rs 2700
B) Rs 2725
C) Rs 2750
D) Rs 2625
E) None of these

58) Two candles of same height are lighted at the same time. The first is consumed in 3 hours and second in
2 hours. Assuming that each candles burns at a constant rate, in how many hours after being lighted, the ratio
between the first and second candles becomes 2:1?
A) 2 hour
B) 2.5 hour
C) 4 hour
D) 1.5 hour
E) None of these

59) A bag contains 25p coins, 50p coins and 1 rupee coins whose values are in the ratio of 8:4:2.If the total
values of coins is X and the total amount in rupees is Y,then which of the following is true
A) X = 840; Y = 360
B) X = 966; Y = 345
C) X = 840; Y = 280
D) X = 740; Y = 260
E) None of these

60) Nandita scores 60% marks in five subjects together, viz., Hindi, Science, Mathematics, English and
Sanskrit, where in the maximum marks of each subject were 105. How many marks did Nandita score in

www.ibpsguide.com | estore.ibpsguide.com | www.sscexamguide.com


370
Complete Quantitative Aptitude Questions

Science, if she scored 69 marks in Hindi, 62 marks in Sanskrit, 68 marks in Mathematics and 51 marks in
English?
a) 66
b) 68
c) 55
d) 65
e) None of these

51) Answer: A
A/B = 15/16 and B/C = 17*18
So A : B : C = 15*17 : 16*17 : 16*18
= 255 : 272 : 288
So C’s salary = [288/(255+272+288)] * 1,87,450= Rs 66,240

52) Answer: D
ratio of the incomes of A and B last year was 9x : 13y
Now given that ratio of the incomes of A and B last year was 9 : 13.
So 9x/13y = 9/13
This gives x = y
Total of incomes of A and B this year = 10x+15y = 10x+15x = 25 x (because x=y)
So 25x = 50,000
This gives x = 2,000
So present income of B = 15y = 15x = 15*2000 = 30,000

53) Answer: B
25 paise = 25/100 Rs, 50 paise = 50/100 Rs
So value ratio of these coins become = 3*(25/100) : 4*(50/100) : 6*(1)
= 3/4 : 2 : 6 = 3 : 8 : 24
So 25 paise coins value= [3/(3+8+24)] * 315 = Rs 27, so coins = 27 * (100/25) = 108
So 50 paise coins value= [8/(3+8+24)] * 315 = Rs 72, so coins = 72 * (100/50) = 144
So 1 paise coins value= [24/(3+8+24)] * 315 = Rs 216, so coins = 215 * (100/100) = 216

54) Answer: E
New ratio = 1/2 : 1/4 : 1/7 = 14 : 7 : 4
So both ratio suggests that C has not gained any money, rather he has lose the money.
For both ratio find the shares of A and B
With ratio 2 : 4 : 7, A gets = [2/(2+4+7)] * 650 = 100, B gets = [4/(2+4+7)] * 650 = 200

www.ibpsguide.com | estore.ibpsguide.com | www.sscexamguide.com


371
Complete Quantitative Aptitude Questions

With ratio 14 : 7 : 4, A gets = [14/(14+7+4)] * 650 = 364, B gets = [7/(14+7+4)] * 650 = 182
B has also lose the money, A gain the money and = 364 – 100 = 264

55) Answer: B
If 20% of boys and 45% of girls come by bus, then 80% of boys and 55% of girls opt transport other than bus.
Let total number of students in school = x
So boys who opt other transport are (80/100) * 6/(6+5) * x = 24x/55
And girls who opt other transport are (55/100) * 5/(6+5) * x = x/4
So total students who opt other transport = (24x/55) + (x/4) = 151x/220
So required % = [(151x/220)/x] * 100 = 68 7/11 %

56) Answer: C
Income of A = x, of B = 2x
Expenditure of A = 2y, of B = 5y
Savings is (income – expenditure). So
x – 2y = 20,000
2x – 5y = 35,000
Solve the equations, x = 30,000
So total = x+2x = 3x = 3*30,000 = 90,000

57) Answer: B
When the shares reduce, the total amount will also reduce which is to be divided among them. So after
reducing shares by Rs 10, Rs 15 and Rs 25 respectively, total amount is 5750 – (10+15+25) = 5700
So C’s share shall be [9/(4+6+9)] * 5700 = 2700
Actually C would have received = 2700 + 25=2725

58) Answer: D
Height of both candles are same i.e. h
First one takes 3 hours to burn completely, so in one hour = h/3
Similarly second one will burn in one hour = h/2
Let after t time, ratio between their height is 2:1
so, remaining height of first candle = h – t*(h/3)
similarly for second candle = h – t*(h/2)
ratio given 2:1,
h – t*(h/3) / h – t*(h/2) = 2/1
Solving we get t = 1.5

www.ibpsguide.com | estore.ibpsguide.com | www.sscexamguide.com


372
Complete Quantitative Aptitude Questions

59) Answer: A
Value is given in the ratio 8:4:2.
By option,
(8x*0.25) + (4x*0.5) + (2x*1) = 840.
So X=840 Y=360

60) Answer: D
Total of maximum marks of all subjects=105×5=525
75% of 525=525 ×60/100= 315
Obtained marks of foru subjects (Hindi, Sanskrit, mathematics and English)
=69+62+68+51=250
So, the obtained marks in Science=315-250=65

61) Mr. Shrimant inherits 4325 gold coins and divides them among his three sons; Bharat, Parat and Marat; in
a certain ratio. Out of the total coins received by each of them, Bharat sells 40 coins; Parat donates his 20
coins and Marat loses 30 coins. Now, the ratio of gold coins with them is 41:34:46, respectively. How many
coins did Parat receive from his father?
a) 1210
b) 1211
c) 1212
d) 1213
e) None of these

62) The sum of the ages of the 4 members of Sinha family is 172 years. 8 years ago the ages of the 4
members Nishu, Vicky, Mrs.Sinha and Sinha were in the ratio of 2:3:7:8. After how many years would Nishu
be as old as the present age of his mother?
a) 33 years
b) 35 years
c) 36 years
d) 37 years
e) None of these

63) Seats for Mathematics, Science and arts in a school are in the ratio 5:7:8. There is a proposal to increase
these seats by X%, Y% and Z% respectively. And the ratio of increased seats is 2:3:4, which of the following
is true?
A) X = 50; Z = 40
B) Y = 40; Z = 50

www.ibpsguide.com | estore.ibpsguide.com | www.sscexamguide.com


373
Complete Quantitative Aptitude Questions

C) X = 40; Z = 75
D) X = 50; Z = 40
E) Y = 50; X = 75

64) If a certain amount X is divided among A, B, C in such a way that A gets 2/3 of what B gets and B gets 1/3
of what C gets, which of the following is true
A) C’s Share = 1053 and X = 1666
B) A’s Share = 238 and X = 1638
C) B’s Share = 234 and X = 1666
D) C’s Share = 1053 and X = 1638
E) A’s Share = 351 and X = 1638

65) A bus and a truck are available to cross a jungle. The speed of the truck is thrice that of the bus. The
capacity of the truck is 60 persons and that of bus is 40 persons. The average occupancy of the bus is twice
that of the truck. The tickets for the bus and the truck cost Re 1 and Re 1.50 respectively. What is the ratio of
the average rupee collection of the truck to that of the bus in a day? Assume there is no wastage time
between trips and the occupancy of the bus/truck is defined as the ratio of the actual number of persons
boarding it and its capacity.
a) 9:17
b) 17:9
c) 8:27
d) 27:8
e) None of these

66) Ramana divides two sums of money among his four sons Ganesh, Mahesh, Anil and Sunil. The first sum
is divided in the ratio 4: 3: 2: 1 and second in the ratio 5: 6: 7: 8. If the second sum is twice the first, the
largest total is received by
a) Ganesh
b) Mahesh
c) Anil
d) Sunil
e) Both Ganesh and Mahesh

67) An amount of money is to be distributed among P, Q and R in the ratio of 5:4:7 respectively. If the total
share of P and R is 3 times the share of Q, what is definitely Q’s share?
a) 2000

www.ibpsguide.com | estore.ibpsguide.com | www.sscexamguide.com


374
Complete Quantitative Aptitude Questions

b) 4000
c) 6000
d) Data inadequate
e) None of these

68) Two candles of same height are lighted at the same time. The first is consumed in 6 hours and second in
4 hours. Assuming that each candles burns at a constant rate, in how many hours after being lighted, the ratio
between the first and second candles becomes 2:1?
a) 1 hour
b) 2 hour
c) 3 hour
d) 4 hour
e) None of these

69) An employer reduces the number of his employees in the ratio of 7:4 and increases their wages in the
ratio 3:5. State whether his bill of total wages increases or decreases and in what ratio.
a) increases 20:21
b) decreases 21:20
c) increases 21:22
d) decreases 22:21
e) None of these

70) A vessel contains milk and water in the ratio of 4:3. If 14 litres of the mixture is drawn and filled with water,
the ratio changes to 3:4. How much milk was there in the vessel initially?
a) 24
b) 32
c) 40
d) 48
e) None of these

61) Answer: A
41x+40+34x+20+46x+30=4325 =>121x=4325-90 =>x=35
Required answer,
number of coins received by parat
=34x+20=34×35+20=1210

62) Answer: B

www.ibpsguide.com | estore.ibpsguide.com | www.sscexamguide.com


375
Complete Quantitative Aptitude Questions

Let their ages 8 years ago be 2x, 3x, 7x, and 8x.
Their ages now 2x+8, 3x+8, 7x+8, 8x+8.
According to the question,
=2x+8+3x+8+7x+8+8x+8=172
20x+32=172 =>x=140/20=7
Present age of Nishu=2×7+8=22 years
Present age of mother=7×7+8=57 years
Hence, required years (57-22)=35 years

63) Answer: C
Number of increased seats are (140% of 5x), (150% of 7x) and (175% of 8x)
i.e., (140/100 * 5x), (150/100 * 7x) and (175/100 * 8x)
i.e., 7x, 21x/2 and 14x
Required ratio = 7x:21x/2:14x
= 14x : 21x : 28x = 2:3:4
40% of 5 = 2, total seats 5+2=7
50% of 7 = 7/2 total seats 7+7/2=21/2
75% of 8 = 6 total seats 8+6=14
Now compare ratios of all 7:21/2:14=>14:21:28
=>2:3:4

64) Answer: D
A= 2/3 B; B= 1/3C;
A:B = 2:3 ; B:C = 1:3;
A:B:C = 2:3:9
C = 9/14 * 1638 = 1053
X=1638

65) Answer: D
Average Rupee collection = Speed× capacity × Occupancy × Ticket rate
Ratio of average Rupee collection of truck to that of bus= product of above rate
According to question,
(3×60×1×1.5):(1×40×2×1)= 270:80=27:8

66) Answer: A
Let the first sum be 4x, 3x, 2x, x
Second sum be 5y, 6y, 7y, 8y

www.ibpsguide.com | estore.ibpsguide.com | www.sscexamguide.com


376
Complete Quantitative Aptitude Questions

The second sum is twice the first sum


26y = 20x; 13y = 10x
Take y=10 x= 13
G:M:A:S
4X+5Y: 3X+6Y: 2X +7Y: X+8Y
102: 99: 96: 93

67) Answer: D
Data inadequate

68) Answer: C
Let height of both candles is ‘h’ and let after t times ratio between the height be 2:1
h – t*h/6 : h – t*h/4 = 2:1
t=3

69) Answer: B
Let initial employees be 7x and then 4x similarly initial wages be 3y and then 5y
so total wage = 21xy initially and then 20xy
so wages decreases and ratio = 21:20

70) Answer: B
milk = 4x and water = 3x
milk = 4x – 14*4/7 and water = 3x – 14*3/7 + 14
4x – 8: 3x + 8 = 3:4
X = 8, so milk = 8*4 = 32 litres

71) The sum of three numbers is 210. If the ratio between the first and second number be 2:3 and that
between the second and third be 4:5, then the difference between the first and third number?
a) 21
b) 35
c) 42
d) 56
e) None of these

72) 180 sweets are divided among friends A, B, C and D in which B and C are brothers also such that sweets
divided between A and B are in the ratio 2 : 3, between B and C in the ratio 2 : 5 and between C and D in ratio
3 : 4. What is the number of sweets received by the brothers together?

www.ibpsguide.com | estore.ibpsguide.com | www.sscexamguide.com


377
Complete Quantitative Aptitude Questions

A) 78
B) 84
C) 92
D) 102
E) 88

73) Two alloys contain platinum and gold in the ratio of 1:2 and 1:3 respectively. A third alloy C is formed by
mixing alloys one and alloy two in the ratio of 3:4. Find the percentage of gold in the mixture
a) 79.2/7%
b) 71.2/7%
c) 73.2/7%
d) 71.3/7%
e) None of these

74) One year ago the ratio between rahul salary and rohit salary is 4:5. The ratio between their individual
salary of the last year and current year is 2:3 and 3:5 respectively. If the total current salary of rahul and rohit
is 4300. Then find the current salary of rahul.
a) 1200
b) 1800
c) 2400
d) 3600
e) None of these

75) The sum of the squares between three numbers is 5000. The ratio between the first and the second
number is 3:4 and that of second and third number is 4:5. Find the difference between first and the third
number.
A) 20
B) 30
C) 40
D) 50
E) None of these

76) A sum of 12600 is to be distributed between A, B and C. For every rupee A gets, B gets 80p and for every
rupee B gets, C get 90 paise. Find the amount get by C.
A) 3200
B) 3600
C) 4200

www.ibpsguide.com | estore.ibpsguide.com | www.sscexamguide.com


378
Complete Quantitative Aptitude Questions

D) 4600
E) None of these

77) Rs.1870 is divided into three parts in such a way that half of the first part, one-third of the second part and
one-sixth of the third part are equal. The third part is
A) Rs.510
B) Rs.680
C) Rs.850
D) Rs.1020
E) None of these

78) The ratio between the number of boys and girls in a school is 4:5. If the number of boys are increased by
30 % and the number of girls increased by 40 %, then what will the new ratio of boys and girls in the school.
A) 13/35
B) 26/35
C) 26/41
D) 23/13
E) None of these

79) If 40 percent of a number is subtracted from the second number then the second number is reduced to its
3/5. Find the ratio between the first number and the second number.
A) 1:3
B) 1:2
C) 1:1
D) 2:3
E) None of these

80) Two vessels contains equal quantity of solution contains milk and water in the ratio of 7:2 and 4:5
respectively. Now the solutions are mixed with each other then find the ratio of milk and water in the final
solution?
A) 11:7
B) 11:6
C) 11:5
D) 11:9
E) None of these

www.ibpsguide.com | estore.ibpsguide.com | www.sscexamguide.com


379
Complete Quantitative Aptitude Questions

71) Answer: C
a: b = 2:3 and b:c = 4:5
a:b:c = 8:12:15
Difference between first and third number = (7/35)*210 = 42

72) Answer: B
A:B:C:D
2*2*3 : 3*2*3 : 3*5*3 : 3*5*4
4 : 6 : 15 : 20
B and C together = [(6+15)/(4+6+15+20)] * 180 = 84

73) Answer: D
Platinum = 1/3 and 1/4
gold = 2/3 and 3/4
Alloy one and two are mixed in the ratio of 3:4, so ratio of platinum and gold in final ratio – 2:5
So gold % = (5/7)*100
71.3/7%

74) Answer: B
4x and 5x is the last year salry of rahul and rohit respectively
Rahul last year to rahul current year = 2/3
Rohit last year to rohit current year = 3/5
Current of rahul + current of rohit = 4300
(3/2)*4x + (5/3)*5x = 4300.
X = 300.
So rahul current DDsalary = 3/2 * 4* 300 = 1800

75) Answer: A
a^2 + b^2 + c^2 = 5000
a:b:c = 3:4:5
50x^2 = 5000.
X = 10.
5x – 3x = 2*10 = 20

76) Answer: B
Ratio of money between A and B – 100:80 and that of B and C – 100:90

www.ibpsguide.com | estore.ibpsguide.com | www.sscexamguide.com


380
Complete Quantitative Aptitude Questions

so the ratio between A : B :C – 100:80:72


so 252x = 12600, x = 50. So C get = 50*72 = 3600

77) Answer: D
F/2 = S/3= T/6
T = 2S = 3F
F + S + T = 1870
T( 1/3 +1/2 + 1 ) = 1870
T= 1870 x 6/11
= 170 x 6 = 1020
Or the third part is a multiple of 6 and is large

78) Answer: B
boys = 4x and girls = 5x.
Required ratio = [(130/100)*4x]/ [(140/100)*5x]
26/35

79) Answer: C
[ b – (40/100)a] = (3/5)b.
So we get a = b.
1:1

80) Answer: A
milk = 7/9 and water = 2/9 – in 1st vessel
milk = 4/9 and water = 5/9 – in 2nd vessel
(7/9 + 4/9)/ (2/9 + 5/9) = 11:7

81) The ratio between the present age of Maha and Deepa is 5:X. Maha is 9yrs younger than
Parveen.Parveen’s age after 9yrs will be 33yrs. The difference between the Deepa and Maha age is same as
the present age of Parveen.Find X.
a) 13
b) 10
c) 11
d) 14
e) None of these

www.ibpsguide.com | estore.ibpsguide.com | www.sscexamguide.com


381
Complete Quantitative Aptitude Questions

82) 180 sweets are divided among friends A, B, C and D in which B and C are brothers also such that sweets
divided between A and B are in the ratio 2 : 3, between B and C in the ratio 2 : 5 and between C and D in ratio
3 : 4. What is the number of sweets received by the brothers together?
A) 78
B) 84
C) 92
D) 102
E) 88

83) Number of students in 4th and 5th class is in the ratio 6 : 11. 40% in class 4 are girls and 48% in class 5
are girls. What percentage of students in both the classes are boys?
A) 62.5%
B) 54.8%
C) 52.6%
D) 55.8%
E) 53.5%

84) Consider two alloys A and B. 50 kg of alloy A is mixed with 70 kg of alloy B. A contains brass and copper
in the ratio 3 : 2, and B contains them in the ratio 4 : 3 respectively. What is the ratio of copper to brass in the
mixture?
A) 8 : 5
B) 7 : 5
C) 5 : 11
D) 4 : 9
E) 5 : 7

85) In a mixture of milk and water, their ratio is 5:6 respectively in the first container. And the same mixture
has ratio 7:2 respectively in the second container. What is the ratio should the mixture be extracted from each
container and poured in to the third container, so that the ratio of milk and water comes to 6:5 respectively in
the third container?
a) 99:253
b) 253:99
c) 243:88
d) 88:243
e) None of these

www.ibpsguide.com | estore.ibpsguide.com | www.sscexamguide.com


382
Complete Quantitative Aptitude Questions

86) The ratio of the price of two houses was 17:24. Two years later, when the price of the first had risen by
20% while the price of second house increases by Rs.500 and their prices become 16:25. Find the original
prices of the two houses together?
a) Rs.2603
b) Rs.2503
c) Rs.2403
d) Rs.2303
e) None of these

87) Brother A and B had some savings in the ratio 5:6. They decided to buy a gift for their sister, sharing the
cost in the ratio 4:5. After they bought, A is left with three-fourth of his amount, while B is left with Rs.497.
Then, the value of the gift is
a) 215
b) 115
c) 315
d) 415
e) None of these

88) Nandita scores 60% marks in five subjects together, viz., Hindi, Science, Mathematics, English and
Sanskrit, where in the maximum marks of each subject were 105. How many marks did Nandita score in
Science, if she scored 69 marks in Hindi, 62 marks in Sanskrit, 68 marks in Mathematics and 51 marks in
English?
a) 66
b) 68
c) 55
d) 65
e) None of these

89) The ratio of income of A to that of B is 5: 4 and the expenditure of A to that of B is 3: 2. If at the end of the
year, each saves Rs.800, the income of A is
a) Rs.1600
b) Rs.1800
c) Rs.2000
d) Rs.2200
e) None of these

www.ibpsguide.com | estore.ibpsguide.com | www.sscexamguide.com


383
Complete Quantitative Aptitude Questions

90) The sum of the ages of the 4 members of Sinha family is 172 years. 8 years ago the ages of the 4
members Nishu, Vicky, Mrs.Sinha and Sinha were in the ratio of 2:3:7:8. After how many years would Nishu
be as old as the present age of his mother?
a) 33 years
b) 35 years
c) 36 years
d) 37 years
e) None of these

81) Answer: A
P present age = 33-9 = 24
M = 24 – 9 =15
D – M = 24
D – 15 = 24
D = 24+15 = 39
15:39 = 5:13
X = 13

82) Answer: B
A/B = N1/D1 B/C = N2/D2 C/D = N3/D3
A : B : C : D = N1*N2*N3 : D1*N2*N3 : D1*D2*N3 : D1*D2*D3
A/B = 2/3 B/C = 2/5 C/D = 3/4
A:B:C:D
2*2*3 : 3*2*3 : 3*5*3 : 3*5*4
4 : 6 : 15 : 20
B and C together = [(6+15)/(4+6+15+20)] * 180=84

83) Answer: B
Total students in both = 6x+11x = 17x
Boys in class 4 = (60/100)*6x = 360x/100
Boys in class 5 = (52/100)*11x = 572x/100
So total boys = 360x/100 + 572x/100 = 932x/100 = 9.32x
% of boys = [9.32x/17x] * 100=54.8%

84) Answer: E
Brass in A = 3/5 * 50 = 30 kg, Brass in B = 4/7 * 70 = 40 kg
Total brass = 30+40 = 70 kg

www.ibpsguide.com | estore.ibpsguide.com | www.sscexamguide.com


384
Complete Quantitative Aptitude Questions

So copper in mixture is (50+70) – 70 = 50 kg


So copper to brass = 50 : 70=5:7

85) Answer: B
x litres of mixture be taken from the first container and poured into the third container.
Let y litres of mixture be taken from the second container and poured into the third container.
Quantity of milk in x litres=5/11 x
Quantity of milk in y litres=7/9 y
Quantity of milk in third container = 5/11 x+7/9 y
Similarly, quantity of water in third container,
=6/11 x+2/9 y
(5x/11+7y/9) :(6x/11+2y/9)=6 :5
((45x+77y)/99)/((54x+22y)/99)=6/5
=>324x-225x=385y-132y =>99x=253y =>x/y=253/99
=253 :99

86) Answer: A
Let, new price of the first house=17x×120/100=102x/5
Let, new price of the second house=24x+500
According to the question,
(102x/5) :(24x+500)=16 :25
((102x/5))/((24x+500 ) )=16/25 =>102x×5=(24x+500)16 =>x=8000/126=4000/63
Original price of first house=17×4000/63=1079.37~ Rs.1079
Original price of second house=24×4000/63=1523.81 ~ Rs.1524
Required answer, =1079+1524=2603

87) Answer: C
Let the savings of A and B are 5x, 6x and the share cost of gift are 4y, 5y respectively.
According to question,
ForA, 5x-4y=3/4×5x =>x=16y/5
For B, 6x-5y=497 =>6×16y/5-5y=497 =>y=35
Cost of gift=4y+5y=9×35 =315

88) Answer: D
Total of maximum marks of all subjects=105×5=525
75% of 525=525 ×60/100= 315
Obtained marks of foru subjects (Hindi, Sanskrit, mathematics and English)

www.ibpsguide.com | estore.ibpsguide.com | www.sscexamguide.com


385
Complete Quantitative Aptitude Questions

=69+62+68+51=250
So, the obtained marks in Science=315-250=65

89) Answer: C
Income - expenditure for A is 5x – 3y
Income - expenditure for B is 4x – 2y
5x – 3y = 800; 4x – 2y = 800
Subtracting x –y =0; x = y
2x =800; x =400; 5x =2000

90) Answer: B
Let their ages 8 years ago be 2x, 3x, 7x, and 8x.
Their ages now 2x+8, 3x+8, 7x+8, 8x+8.
According to the question,
=2x+8+3x+8+7x+8+8x+8=172
20x+32=172 =>x=140/20=7
Present age of Nishu=2×7+8=22 years
Present age of mother=7×7+8=57 years
Hence, required years (57-22) = 35 years

91) A bus and a truck are available to cross a jungle. The speed of the truck is thrice that of the bus. The
capacity of the truck is 60 persons and that of bus is 40 persons. The average occupancy of the bus is twice
that of the truck. The tickets for the bus and the truck cost Re 1 and Re 1.50 respectively. What is the ratio of
the average rupee collection of the truck to that of the bus in a day? Assume there is no wastage time
between trips and the occupancy of the bus/truck is defined as the ratio of the actual number of persons
boarding it and its capacity.
a) 9:17
b) 17:9
c) 8:27
d) 27:8
e) None of these

92) Mani, Ram and Bhuvana have some stones with each of them. Seven times the number of stones with
Ram equals nine times the number of stones with Mani while seven times the number of stones with Mani
equals nine times the number of stones with Bhuvana. What is the minimum number of stones that can be
there with all three of them put together?
a) 193

www.ibpsguide.com | estore.ibpsguide.com | www.sscexamguide.com


386
Complete Quantitative Aptitude Questions

b) 195
c) 197
d) 199
e) None of these

93) A, B and C invest in a business. If the ratio of their time periods are 4 : 5 : 6 and their profits are in the
ratio 7 : 8 : 9. Find the ratio in which the investment are made by A, B and C.
a) 45 : 35:27
b) 35 : 32 : 30
c) 27 : 37 : 44
d) 30 : 32 : 35
e) 41 : 42 : 34

94) A certain amount to be distributed among A,B, and C in the ratio 2:3:4 respectively but was erroneously
distributed in the ratio 7:2:5 respectively. As a result of this ,B got Rs. 40 less. What is the amount?
a) Rs.210
b) Rs.270
c) Rs.230
d) Rs.280
e) None of these

95) Mr. Venku invested a certain amount in Debit and Equity funds in the ratio of 4 : 5 respectively. At the end
of one year, he earned a total dividend of 30% on his investment. After one year he reinvested the amount
including dividend in the ratio of 6 : 7 in Debt and Equity Funds. If the amount reinvested in Equity Funds was
Rs. 94,500/-, what was the original amount invested in Equity Funds?
a) Rs. 75,000/-
b) Rs. 81,000/-
c) Rs. 60,000/-
d) Rs. 65,000/-
e) None of these

96) The ratio of the monthly salaries of A and B is in the ratio 15 : 16 and that of B and C is in the ratio 17 :
18. Find the monthly income of C if the total of their monthly salary is Rs 1,87,450.
a) Rs 66,240
b) Rs 72,100
c) Rs 62,200
d) Rs 65,800

www.ibpsguide.com | estore.ibpsguide.com | www.sscexamguide.com


387
Complete Quantitative Aptitude Questions

e) Rs 60,300

97) The income of A, B, and C are in the ratio 3 : 4 : 7. If their incomes be changed such that the new income
of A is 50% increased, 25% increased for B and 25% decrease for C. Find the ratio of their new incomes.
a) 18 : 40 : 23
b) 17 : 12 : 21
c) 18 : 20 : 21
d) 28 : 20 : 21
e) None of these

98) If Rs. 1540 be divided amongst A, B and C in such a way that the share of B is equal to of what A and C
together receive. Then, B’s share will be:
a) Rs. 770
b) Rs. 420
c) Rs. 880
d) Rs. 1210
e) None of these

99) RS.73,689/- are divided between A and B in the ratio 4:7. What is the difference between thrice the share
of A and twice the share of B?
a) Rs. 35699/-
b) Rs. 46,893/-
c) Rs. 20,097/-
d) Rs. 26796/-
e) Rs. 13,398/-

100) The price of diamond is directly prortional to the square of its weight. The diamond accidentally fell and
broke into four pieces whose weights are in the ratio of 1:2:3:4. If the price fetched is Rs. 70,000 less than the
original price, find the original price?
a) Rs. 100,000
b) Rs. 70,000
c) Rs. 160,000
d) Rs. 10800
e) Rs. 150,000

91) Answer: D
(3×60×1×1.5):(1×40×2×1)= 270:80=27:8

www.ibpsguide.com | estore.ibpsguide.com | www.sscexamguide.com


388
Complete Quantitative Aptitude Questions

92) Answer: A
Let the stones with Mani, Ram and Bhuvana be M, R and B respectively.
Given, 7R=9M; 7M=9B
R/M=9/7; M/B=9/7 =>so, R:M=9:7; M:B=9:7
The least possible integral values for R, M, B will be R=81, M=63 and B=49
=>Total=81+63+49=193

93) Answer: B
Ratio of their investment
=7/4 : 8/5 : 9/6 =>take LCM
=7×15 :8×12 :9×10
A :B :C= 35 :32 :30

94) Answer: A
Let the amount be X the distruibution For B is 3/9
The error amount calculated for B 2/14
By this we got Rs.40 Less
x/3-x/7=40
4x=40*210
x=210

95) Answer: A
Amount invested in Debit funds = 4.
Amount invested in equity funds = 5.
After 1 yr Mr. Xaviers total amount =4x*130/100 +5x*130/100 => 11.7,
Amount invested in equity funds = 11.7 x 7/13
11.7 x 7/13= 94500
= 15000
The original amount invested in equity funds = 5 x 15000
= Rs. 75000

96) Answer: A
A: B = 15: 16
B: C = 17: 18
A: C = 255: 288
A: B: C = 255: 272: 288

www.ibpsguide.com | estore.ibpsguide.com | www.sscexamguide.com


389
Complete Quantitative Aptitude Questions

C = (288/815)*187450
= 66240

97) Answer: C
A: B: C :: 3: 4: 7
New income ratio = 4.5: 5: 5.25
= 18: 20: 21

98) Answer: A
if a get 100 and c get 200 then b get 300.
So the ratio is 1:3:2
B’s share= 1540*3/6 = 770

99) Answer: E
The A’s Share is = 73687 *4/11=26795.27
The B’s Share is = 73687*7/11=46891.73
Difference between Twice B’s Share & thrice A’s Share = 93783.45-80385.82=13397.64
A’s Share = 93783.45-80385.82=13397.64 =13398

100) Answer: A
If we assume weight of diamond 10x, will I come up with 10x because the ratio, given 1: 2: 3: 4 add upto 10.
So, individual weights of diamonds will be x, 2x, 3x and 4x
Original price = K (10x)2
Price for pieces = K (x2 + 4x2 + 9x2 + 16x2)
= K 30x2
Thus change in price us see in 100 Kx2 30Kx2 = 70Kx2 which is given is 70,000
So, the original price is Rs. 100,000

14. PROBLEMS ON AGES

1) If the current age of a person be X, then


- age after n years = X + n
- age n years ago = X – n
- n times the age = nX
- If ages in the numerical are mentioned in ratio A : B, then A : B will be AX and BX

www.ibpsguide.com | estore.ibpsguide.com | www.sscexamguide.com


390
Complete Quantitative Aptitude Questions

2) If sum of ages of x and y is A and ratio of their ages is p : q respectively, then u can determine age of y by
using the formula shown below:
Age of y =( Ratio of y/Sum of ratios ) *sum of ages
Age of y = (q/(p+q))*A

Type1: Calculate Present age

1) What is Kishore present age, if after 30 years his age will be 15times of his age 15 years back.
a) 16.2 years
b) 17.7 years
c) 18.2years
d) 15.4 years

2) Amit is 18 years elder than Rohit. If 8 years ago, Amit was 6 times as old as Rohit, then find Amit’s present
age.
a). 32.5 years
b) 27.5 years
c) 29.6 years
d) 24.9 years

3) The age of vaishu is 18 times that of her daughter raiza. If the age of raiza is 6years, what is the age of
vaishu?
a) 98
b) 110
c) 90
d) 108

4) The father’s age 7years ago was 11 times the age of his daughter. At present the father’s age is 7 times
that of his daughter. Find the daughter’s present age.
a) 17.5
b) 18.2
c) 23
d) None

5)12 years ago M was half of N in age. If the ratio of their present ages is 4 : 5, what will be the total of their
present ages
a) 30

www.ibpsguide.com | estore.ibpsguide.com | www.sscexamguide.com


391
Complete Quantitative Aptitude Questions

b) 35
c) 36
d) 32

6) The sum of the ages of a Mother and son is 54 years. Eight years ago, the product of their ages was 6
times the mother’s age at that time. The present age of mother and son
a) 34,17
b) 14,40
c) 40,14
d) 50,17

7) Mani is 32years older than his son Dinesh. In 4 years, his age will be twice the age of his son. The present
age of his son is
a) 26 years
b) 28years
c) 22 years
d) 24 years

8) The sum of the current ages of a father and his son is 84 years. 7 years ago, father's age was 3 times the
age of the son. After 7 years, son's age will be
a) 35 years
b) 28 years
c) 56years
d) 53 years

9) The total age of x and y is 48 years more than the total age of y and z. z is how many year younger than x
a) 44
b) 24
c) 96
d) 48

10) The ages of two sisters differ by 32 years. 12 years ago, the elder one was 7 times as old as the younger
one. What are their present ages of the elder person
a) 46.9
b) 49.3
c) 42.7
d) 45.4

www.ibpsguide.com | estore.ibpsguide.com | www.sscexamguide.com


392
Complete Quantitative Aptitude Questions

1) C
Let kishore’s present age be x
kishore’s age before 15 years = (x - 15)
kishore’s’s age after 30 years = (x + 30)
We are given that,
kishore’s’s age after 30 years (x + 30) is 15 times his age 15 years back (x – 15)
Therefore,
(x + 30) = 15 (x – 15)
Solving the equation, we get
x + 30 =15x –225
14x = 255
x = 18.2 years

2) C
Let age of Rohit be y
Amit is 18 years elder than Rohit = (y + 18).
SoAmit’s age 8 years ago = (y + 18 – 8)
Rohit’s age before 8 years = (y – 8)
8 years ago, Amit is 6 times as old as Rohit
(y + 18 – 8) = 6 (y – 8)
(y + 10) = (6y – 48)
5y = 58
y = 11.6
Rohit’s age = 11.6years
Amit’s age = (y + 18) = (11.6 + 18) = 29.6years

3) D
vaishu’s present age = x
vaishu’a age is 18 times her daughter’s age.
Daughter’s age = 6.
Therefore, 18 times of 6 = x
18 x 6 = x
x= 108 years = vaishu’s age.

4) A
daughter’s present age = x

www.ibpsguide.com | estore.ibpsguide.com | www.sscexamguide.com


393
Complete Quantitative Aptitude Questions

At present the father’s age is 7 times that of his daughter


i.e., Father’s present age = 7x.
‘Father’s age 7 years ago was 11 times the age of his daughter’
(7x – 7) = 11 * (x – 7)
x = 17.5 years,
The daughter’s present age = 17.5 years

5) C
Let M's age 12 years ago = x years.
Then, N's age 12 years ago = 2x years.
(x + 12) / (2x+ l2) = 4/5
=> x = 4.
So, the total of their present ages =(x + 12 + 2x + 12)
= (3x + 24) = 36 years.

6) C
Let sons age = x years. Then mothers age = (54 - x)years.
(x-8)(54-x-8) = 6(54- x - 8) hence (x-8) = 6 so x = 14
Their ages are 40 years and 14 years.

7) B
Let the son's present age be x years.
Then, mani's present age = (x + 32) years
=> (x + 32) + 4 = 2(x + 4)
=> x + 36 = 2x + 8
So, x = 28

8) A
Son’s current age = x years.
Then father’s age = (84 - x)years
(84 - x) - 7 = 3(x - 7)
2x=56
x =28; after 7years it will be turn on 35

9) D
Given that x+y = 48 + y+ z
x – z= 48 + y – y = 48

www.ibpsguide.com | estore.ibpsguide.com | www.sscexamguide.com


394
Complete Quantitative Aptitude Questions

z is younger than x by 48 years

10) B
Let's take the present age of the elder person = x
and the present age of the younger person = x – 32
(x – 12) = 7*(x-32-12)
=> x – 12 = 7x – 308
=> 6x = 296
=> x = 49years 3months

Type 2: Numerical to Determine Ages in ratio form

11) Three year ago, ratio of Tom and Ram age’s was 5 : 6 respectively. After 12 years, this ratio becomes 6 :
7. How old is Ram at present?
a) 39years
b) 93 years
c) 78 years
d) 87 years

12) Age of father 30 years ago was 9 times the age of her son. After 30 years, father’s age will be thrice that
of his son. Find the ratio of their present ages.
a) 21 : 5
b) 29 : 5
c) 27 : 4
d) 21 : 3

13) Ratio of ages of three girls are 5:8:10, six years ago, the sum of their ages was 28. Find their present
ages.
a) 16,10,20
b) 12,28,36
c) 10,16,20
d) 16,28,36

14) Eight years ago, the ratio of the ages of micky and Donald was 6:5, 12years hence, the ratio of their ages
will be 11:10. What is Donald age at present
a) 30 years
b) 28 years

www.ibpsguide.com | estore.ibpsguide.com | www.sscexamguide.com


395
Complete Quantitative Aptitude Questions

c) 34 years
d) 36 years

15) Shyam is younger than Sharuk by 22 years. If the ratio of their ages is 5:7, find the age of Shyam
a) 55
b) 77
c) 66
d) 88

16) Keerthi's grandfather was 16 times older to her 32 years ago. He would be 6times of her age 16 years
from now. What was ratio of ages of keerthi and her grandfather 16 years ago.
a) 1:10
b) 11:10
c) 10:11
d) 10:1

17) The ratio between the present ages of M and N is 8:9. If N is 6 years old than M, what will be the ratio of
the ages of M and N after 6 years
a) 9:10
b) 7:9
c) 9:8
d) 11:10

18) Total of the ages of P, Q and R at present is 99 years. Eight years ago, the ratio of their ages was 4: 5: 6.
What is the age of Q at present
a) 44
b) 30
c) 45
d) 33

19) Ratio between vinodha and swetha is 7:6, After 8 Years vinodha’s age will be 29 years. What is swetha
present age.
a) 16
b) 18
c) 20
d) 22

www.ibpsguide.com | estore.ibpsguide.com | www.sscexamguide.com


396
Complete Quantitative Aptitude Questions

20) The ratio of the ages of Bala and Babu is 13: 11. The total of their ages is 3.6decades. The proportion of
their ages after 0.95decades will be [1 Decade = 10 years]
a) 24:23
b) 12:11
c) 27:24
d) 29:26

11) B
We are given that age ratio of tom and ram = 5 : 6
Tom’s age = 5x and Ram’s age = 6x
3 years ago, their age was 5x and 6x.
Hence at present, Tom’s age = 5x +3 and Ram’s age = 6x +3
After 12years,
Tom’s age = (5x +3) + 12 = (5x + 15)
Ram’s age = (6x +3) + 12= (6x + 15)
After 12years, this ratio becomes => 6 : 7. Therefore,
Tom’s Age/ram’s Age =6/7
(5x + 15) / (6x +15) = 6 / 7
7 (5x + 15) = 6 (6x + 15)
X = 15
Ram’s present age = (6x + 3) = 93years

12) A
At age of son be x and as father’s age is 9 times the age of her son, let it be 9x, nine years ago.
At present: father’s age will be (9x + 30) and son’s age will be (x + 30)
After 30 years: father’s age will be (9x + 30) +30 and son’s age will be (x + 30) + 30
father’s age is thrice that of son
(9x + 30) +30 = 3 [(x + 30) + 30]
(9x + 60) = 3[x + 60]
Solving the equation, we get x = 20
We are asked to find the present ratio.
(9x + 30) : (x + 30) = 210 : 50 = 21:5

13) C
Let the present ages are 5x, 8x, 10x.
=> (5x-6) + (8x-6) + (10x-6) = 28
=> 23x = 46

www.ibpsguide.com | estore.ibpsguide.com | www.sscexamguide.com


397
Complete Quantitative Aptitude Questions

=> x = 2
So their present ages are: 10,16,20

14) B
Let 8 years ago the age of Micky and Donald be 6x and 5x resp.
then,
((6x+8)+12) / ((5x+8)+12) = 11/10
10(6x+20) = 11(5x+20)
5x = 20 => x = 4
So Donald age is (5x+8) = 28

15) A
If sharuk age is x, then Shyam age is x-22,
so (x-22)/x = 5/7
=> 7x-154 = 5x
=> 2x = 154
=> x = 77
So shyam age is 77 - 22 = 55

16) A
Let, keerthi’s age 32 years ago = x,
Grandfather's age 32 years ago = 16x.
16 years from now, 6(x+32+16) = (16x+32+16)
=> x = 24
16 years ago ratio was:
(x+16)/ (16x+16) = (24+16) / (16*24+16)
40 /400 = >1 / 10

17) A
Let M age and N age be 8x years and 9x years.
Then 9x - 8x = 6 <=> x = 6
So required ratio will be (8x+6): (9x+6)
54:60
9:10

18) D
Let their ages 8 years ago is4x, 5x and 6x years.

www.ibpsguide.com | estore.ibpsguide.com | www.sscexamguide.com


398
Complete Quantitative Aptitude Questions

4x+8+5x + 8 + 6x + 8= 99
hence x= 5
Q’s present age = (5x + 8)
=33 years

19) B
Present age is 7x and 6x,
=> 7x + 8 = 29 => x = 3
So swetha age is = 6(3) = 18

20) D
Let, Bala’s age = 13A and babu’s age = 11A
Then 13A + 11A = 36
A = 1.5
Bala’s age = 19.5years
and Babu’s age = 16.5 years
Proportion of their ages after 9.5 is = (19.5+9.5) : (16.5 + 9.5)
= 29 : 26

Type 3: Determine Age of a Person after / before x Years

21) Nandhu is 80years old and Nalini is 90 years old. How many years ago was the ratio of their ages 8 : 10?
a) 40 years
b) 35 years
c) 20 years
d) 25 years

22) The ratio of kabil’s age 6 years ago and sahul’s age after 6 years is 3 : 3. If at present, the ratio of their
ages is 7: 5, then find the ratio between kabil’s age 6 years hence and sahul’s age 6 years ago.
a) 1 : 3
b) 2 : 1
c) 4 : 3
d) 3 : 4

23) The sum of the current ages of two persons M and N is 129. If the age of M is twice that of N, find the sum
of their ages 8 years hence.
a)125

www.ibpsguide.com | estore.ibpsguide.com | www.sscexamguide.com


399
Complete Quantitative Aptitude Questions

b)145
c)137
d)149

24) The ratio of the age of a rithick and his wife is 12:9. At the time of marriage the ratio was 10:6 and After 6
years this ratio will become 18:14. How many years ago were they married?
a) 27
b) 12
c) 18
d) 14

25) The sum of ages of 4 puppies born at the intervals of 2 years each is 48 years. What is the age of the
youngest puppy?
a) 9years
b) 8 years
c) 10 years
d) None of these

26) Vasu said to his son, "I was as old as you at the time of your birth". If vasu’s age is 76 years now, the
son's age ten years back was:
a)14 years
b)19 years
c)33 years
d)28 years

27) P is 6 years older than Q who is twice as old as R. If the total of the ages of P, Q and R be 81, the how
old is Q?
a) 27
b) 28
c) 29
d) 30

28) Fathima's father was 76 years of age when she was born while her mother was 72years old when her
brother 8 years younger to her was born. What is the difference between the ages of her parents?
a)12 years
b)14 years
c)16 years

www.ibpsguide.com | estore.ibpsguide.com | www.sscexamguide.com


400
Complete Quantitative Aptitude Questions

d)18 years

29) Mr. Nagar is 6 times more aged than his daughter. If after 10 years, he would be 6 times of daughter’s
age, then further after 10 years, how many times he would be of his daughter’s age?
a) 5 .5 times
b) 5.8 times
c) 2.5 times
d) 5.1 times

30) 15 years ago, vaishali age was 15 times the age of her brother and the sum of present ages of vaishali
and brother is 94years. What will be the age of her brother after 18 years?
a) 32 years
b) 33.5 years
c) 37 years
d) 30 years

21) A
Let us assume x years ago
At present: Nandhu is 80 years and Nalini is 90 years
x years ago: Nandhu’s age = (80 – x) and Nalini’s age = (90 – x)
Ratio of their ages x years ago was 8 : 10
(80– x)/ (90– x) = 8/10
800-10x =720-8x
2x=80
x=40
Therefore, 40 years ago, the ratio of their ages was 8:10

22) B
At present: Ratio of their ages = 7 : 5. Therefore, 7: 5 will be 7x and 5x.
kabil’s age 6 years ago = 7x – 6
sahul’s age after 6years = 5x + 6
Ratio of kabil’s age 6years ago and sahul’s age after 6 years is 3 : 3
Therefore,
(7x – 6)/(5x+6) =3/3
Solving, we get x = 6

We are asked to find the ratio between kabil’s age 6 years hence and sahul’s age 6 years ago.

www.ibpsguide.com | estore.ibpsguide.com | www.sscexamguide.com


401
Complete Quantitative Aptitude Questions

kabil’s age : (7x + 6)


sahul’s age: (5x – 6)
Ratio of sahul’s age and kabil’s age
(7x + 6) / (5x – 6) = 48/24 =2/1 =2:1
23) B
M+N=129
Given M=2N
From Which N =43,M=86
8 years their age will be 51 and 94 ,so the sum of their ages is 145

24) C
Let the present age of the rithick and his wife be 12x and 9x respectively.
After 6 years this ratio will become 18:14 =>9:7
(12x+6): (9x+6) = 9:7
84x+42 =81x+54
x=4
Present age of the rithick =12x=48
Present age of his wife =9x=36
Assume that they got married before t years. Then,
(48-t):(36-t)=10:6
144-3t=180-5t
2t=36
t=18

25) A
Let the ages of puppies be x, (x + 2), (x + 4)and (x + 6) years.
Then, x + (x + 2) + (x + 4) + (x + 6) = 48
4x = 36,x = 9
Age of the youngest puppy = x = 9 years

26) D
Let the son's present age be x years.
Then, (76 - x) = x
2x = 76
x = 38
Son's age 10 years back (38-10) = 28 years

www.ibpsguide.com | estore.ibpsguide.com | www.sscexamguide.com


402
Complete Quantitative Aptitude Questions

27) D
Let R's age be x years.
Then, Q's age = 2x years.
P's age = (2x + 6) years.
(2x + 6) + 2x + x = 81
5x = 75=> x =15.
Hence, Q's age = 2x = 30 years

28) A
Mother's age when Fathima 's brother was born = 72 years.
Father's age when Fathima 's brother was born = (76+8) years = 84 years.
Required difference = (84-72) years = 12 years

29) D
Let daughter’s age be x and father’s age be 6x.
Father’s age is 6 times more aged than his daughter, therefore father’s present age = x + 6x = 7x
After 10years, father’s age is 6 times more than his daughter age.
(7x + 10) = 6 (x + 10)
(7x+10)=6x+60
x = 50
After 10 years it was (7x + 10), then after further 10 years, father’s age = (x +20) and daughter’s age = (x +
20)
(7x+10) /(x+20) = ?
Substitute the value of x, we get
= (350+10) /70
= 360 /70
=5.14
After further 10 years, father will be 5.1 times of daughter’s age

30) C
Let present age of brother be x and vaishali’s age be 94 – x.

We are given,15 years ago sister’s age was 15 times the age of her brother.

www.ibpsguide.com | estore.ibpsguide.com | www.sscexamguide.com


403
Complete Quantitative Aptitude Questions

Therefore,
(94 – x) – 15 = 15 (x – 15)
94 – x – 15 = 15x – 225
15x + x = 94 – 15 +225
16x = 304
x = 19
Future age (after 18 yrs) = (x + 18) = (19 + 18) = 37 years

15. TIME, SPEED AND DISTANCE

1) In a 2280 m race Dinesh beats Aarav by 360 m or 6 seconds. In another race on the same track at the
same speeds. Aarav and karthick start at one end while Dinesh starts at the opposite end. How many metres
would Aarav have covered ,by the time Dinesh meets karthick given that Dinesh speed is 16 m/sec more
than that of karthick
a) 1140 m
b) 2280 m
c) 2460 m
d) 1180 m

2) kajal rides his scooter 14km at an average speed of 16 km/hr and again travels 16km at an average speed
of 14 km/hr. What is her average speed for the entire trip approximately?
a) 15.36 km/hr
b) 17.46 km/hr
c) 13.56 km/hr
d) 14.86 km/hr

3) A bus can travel 25% faster than a jeep. Both start from point P at the same time and reach point Q, 225
kms away from P, at the same time. On the way, however, the bus lost about 37.5 minutes while stopping at
the certain place. What is the speed of the jeep?
a) 64 km/hr
b) 72 km/hr
c) 68 km/hr
d) 56 km/hr

4) A race course is 250 m long. P and Q run a race and P wins by 4m. Q and R run over the same course and
Q win by 3m. R and S run over it and S wins by 12m. If P and S run over it, then who would win and by how
much?

www.ibpsguide.com | estore.ibpsguide.com | www.sscexamguide.com


404
Complete Quantitative Aptitude Questions

a) P by 4.82m
b) S by 5.30m
c) P by 7.25m
d) Q by 6.54m

5) Kishore is travelling on his bike and has calculated to reach point P at 4 pm if he travels at 20 kmph. He will
reach there at 12 noon if he travels at 30 kmph. At what speed must he travel to reach P at 2 pm?
a) 18 kmph
b) 12 kmph
c) 24 kmph
d) 16 kmph

6) The speed of a van increases by 5 kmph after every one hour. If the distance travelled in the first one hour
was 55 km, what was the total distance travelled in 8 hours?
a) 580km
b) 116km
c) 147km
d) 240km

7) The distance between salem and trichy is 170 km. A bus starts from salem at 6 a.m. and travels towards
trichy at 20 km/hr. Another bus starts from trichy at 7 a.m. and travels towards salem at 30 km/hr. At what
time will they meet?
a) 7a.m
b) 8a.m
c) 9a.m
d) 10a.m

8) Tharun has to cover a distance of 84 km in 40 minutes. If he covers one-half of the distance in one-fourth
of the total time, to cover the remaining distance in the remaining time, what should be his speed in km/hr?
a) 42 km/hr
b) 64km/hr
c) 84km/hr
d)76km/hr

9) In a 2700 m race around a round track of length 300m, abi and kabi meet at the end of the 3rd minute, for
the first time after the start of the race. All the runners maintain uniform speed throughout the race. If abi runs
at twice the speed of the kabi. Find the time taken by abi to finish the race.

www.ibpsguide.com | estore.ibpsguide.com | www.sscexamguide.com


405
Complete Quantitative Aptitude Questions

a) 27 mints
b) 32 mints
c) 5 mints
d) 23 mints

10) Three friends X, Y and Z run a running race, Y finished 12 meters ahead of Z and 18 m ahead of X, while
Z finished 8m ahead of X. If each friends runs the entire distance at their respective constant speeds, what is
the length of the race?
a) 16m
b) 48m
c) 24m
d) 12m

1) B
Aarav 's speed =360/6 =60 m/s
Time taken by Aarav to cover 2280 m =2280/60= 38 seconds
Dinesh 's speed = 2280/60=38 m/s
karthick 's speed = 22m/s
Time taken by Dinesh to meet karthick in 2280m race in opposite direction:
Distance covered by Aarav:
=2280/(38+22)
=2280/60
=38 seconds
=(38*60)m
=2280 m

2) D
Total distance travelled
=14+16=30Km
Time taken to travel 14 km at an average speed of 16 km/hr
=14/16=7/8 Hr
Time taken to travel 16 km at an average speed of 14 km/hr
=16/14=8/7 hr
Total time taken =7/8+8/7
Average speed =Total distance travelled /Total time taken
=30/(7/8+8/7)
=30/(113/56)

www.ibpsguide.com | estore.ibpsguide.com | www.sscexamguide.com


406
Complete Quantitative Aptitude Questions

=30*56/113=1680/113
=14.86 km/hr

3) B
Let speed of the jeep =x kmph
Then, speed of the bus =(100+25)x/100
=125x/100 =5x/4 kmph
Time taken by the jeep to travel from P to Q
=225/x hours
Time taken by the bus to travel from p to Q
=225/(5x/4) +(37.5/60)
Since both start from P at the same time and reach point Q at the same time,
225/x = 225/(5x/4) +(37.5/60)
225/x=900/5x+37.5/60
225/x=180/x+37.5/60
45/x =37.5/60
37.5x=45*60
X=72 km/hr

4) B
If P covers 250m, Q covers 246 m
If Q covers 250m, R covers 247 m
If S covers 250m, R covers 238m
Now if Q covers 246 m, then R will cover =247/250*246=243.048
If R covers 243.048 m, then S will cover=250/238*243.048=255.30
If P and S run over 250 m, then S win by =(255.30-250)
S win by = 5.30 m

5) C
Let the distance be x km
Travelling at 20 kmph, Kishore will reach point P at 4pm.
Travelling at 30 kmph, Kishore will reach point P at 12noon.
Therefore, time taken when travelling at 20km - time taken when travelling at 30 km =4hours
X/20-X/30=4
(3x-2x)/60=4
x/60=4
x=240

www.ibpsguide.com | estore.ibpsguide.com | www.sscexamguide.com


407
Complete Quantitative Aptitude Questions

Time needed if travelled at 20 kmph =240/20=12hours Therefore, to reach at 2 pm, his travelling time must
be (12-2)=10 hours
Hence, required speed =240/10=24 kmph

6) A
Distance travelled in 1ST hour =55 km
Speed of the van increases by 5 kmph after every one hour. Hence,
distance travelled in 2nd hour =60 km
distance travelled in 3rd hour =65 km
and so on
Total distance travelled
=(55+60+……..+(8 terms))
=8/2(2*55+(8-1)5)
=4(110+35)
=580km

7) D
Assume that they meet x hours after 6 a.m.
Then, Bus 1, starting from salem, travels x hours till the bus meet.
Distance travelled by bus 1 in x hours =20x km
bus 2, starting from trichy , travels (x−1) hours till the bus meet.
Distance travelled by bus 2 in (x−1)hours =30(x−1) km
Total distance travelled
= Distance travelled by bus 1 + Distance travelled by bus 2
⇒170=20x+30(x−1)
170=20x+30x-30
170+30=50x
200=50x
X=4
Hence, the bus meet 4 hours after 6 a.m., i.e. at 10a.m

8) C
Tharun needs to cover 84 km in 40minutes
Given that he covers one-half of the distance in one-fourth of the total time
⇒ he covers half of 84km in one-fourth of 40 minutes
⇒ He covers 42 km in ¼ * 40minutes

www.ibpsguide.com | estore.ibpsguide.com | www.sscexamguide.com


408
Complete Quantitative Aptitude Questions

⇒ He covers 42 km in 10 minutes
Now he needs to cover the remaining 42 km in remaining 30minutes
Distance =42km
Time =30minutes =1/2 hr
Required Speed = Distance / Time
=42/(1/2)=84 km/hr

9) A
As, abi is twice as fast as the kabi, abi would have completed two rounds by the time kabi completes one
round.
And that is their second meeting.
Their first meeting takes place after the abi takes 3 min to complete one round.
300*1=300m
⇒He takes 2700/300 *3 =27 mints
Hence he takes 27 minutes to finish the race.

10) B
Let the length of the race track be 'd'.
When Y finished the race, X and Z would have run (d−18) and (d−12) meters respectively.
When Z finishes the race, X would have run (d-8) meters.
The ratio of speeds of Z and X is:
(d-12)/(d-18) =d/(d-8)
(d-8)(d-12)=d(d-18)
d2-12d-8d+96 = d2-18d
d2-20d+96= d2-18d
20d-18d=96
d=96/2=48m
hence the length of the race track is 48m

11) A car covers a distance of 1120 metres in 1 minute whereas a bus covers a distance of 56 kms in 44
minutes. What is the ratio of their speed?
a) 22:25
b) 25:22
c) 11:15
d) 15:11

www.ibpsguide.com | estore.ibpsguide.com | www.sscexamguide.com


409
Complete Quantitative Aptitude Questions

12) Anushiya covered a definite distance at some speed. If she had moved 5 kmph faster, she would have
taken 55minutes less. If he had moved 4 kmph slower, she would have taken 55 minutes more. What is the
distance in km?
a) 110 Km
b) 220 km
c) 330 km
d) 440 km

13) Shreya travel the first part of her journey at 160 kmph and the second part at 240 kmph and cover the
total distance of 3840 km to her destination in 20 hours. How long did the first part of her journey last?
a) 8 hrs
b) 12 hrs
c) 16 hrs
d) 10 hrs

14) Ravi walks to and fro to a Gym. He spends 30 minutes in gym. If he walks at speed of 20 km an hour, he
returns to home at 8.00 a.m. If he walks at 30 km an hour, he returns to home at 7.30 a.m. How fast must he
walk in order to return at 7.15 hours?
a) 40 km/hr
b) 30 km/hr
c) 60 km/hr
d) 50 km/hr

15) A passenger train without stopping running at an average speed of 120 km/hr and with stoppages at an
average speed of 80 km/hr. What is the total time taken by the train for stoppages on a route of length
480km?
a) 2 hours
b) 3 hours
c) 4 hours
d) 5 hours

16) Siva leaves chennai at 3 am and reaches Hyderabad at 11 am . Anirudh leaves Hyderabad at 5 am
and reaches chennai at 12:00 noon. At what time do they cross each other?
a) 7 : 52am
b) 8 : 25am
c) 6 : 52am
d) 9 : 25am

www.ibpsguide.com | estore.ibpsguide.com | www.sscexamguide.com


410
Complete Quantitative Aptitude Questions

17) A monkey takes 16 jumps for every 20 jumps of a dog but 12 jumps of a monkey are equal to 16 jumps of
the dog. Compare their speeds
a) 15:16
b) 17:16
c) 16:15
d) 16:17

18) A hare is spotted by a tiger from a distance of 50 metres. When the tiger starts a chase, the hare also
starts running. If the speed of the hare be 4 km/hr and that of a tiger 5 km/hr, how far the hare will have run
before its over taken?
a) 100m
b) 200m
c) 300m
d) 400m

19) A royal enfield bike starts with the speed of 210 km/hr with its speed increasing every 2 hrs by 30km/hr, in
how many hrs will it cover 1035km?
a) 2 1/2 hr
b) 4 1 /2 hr
c) 6 1 /2 hr
d) 7 1 /2 hr

20) Jackson travels a distance of 100km in 5 hrs. How much faster in kilometer per hr, on an average, must
he travel to make a journey in 5/3 hr less time?
a) 5 km/hr
b) 10km/hr
c) 6 km/hr
d) 8 km/hr

11) A
Speed of the Car = 1120 m/minute
Speed of the bus=56 kms in 44 minutes
=56/44 km/m
=56000/44 m/mints
Speed of the car : Speed of the bus,
=1120:56000/44

www.ibpsguide.com | estore.ibpsguide.com | www.sscexamguide.com


411
Complete Quantitative Aptitude Questions

=280:14000/44
=20:1000/44
=880:1000
=88:100
Required ratio =22:25

12) C
speed=2V1V2/(V1-V2)
= 2*5*4/(5-4)
=40 km/hr
Distance= vt1 (1+v/v1)
=40* 55/60(1+40/5)
=40*11/12(1+8)
=40*11/12*9
=330 km
Therefore the distance is 330km

13) B
The total time of journey = 20 hours.
Let 'x' hours be the time that shreya travelled at 160 kmph
Therefore, 20-x hours would be time that she travelled at 240 kmph.
Hence, she would have covered x*160+(20-x)*240 kms in the 20 hours = 3840 kms
x*160+(20-x)*240 =3840
160x +4800-240x =3840
240x-160x=4800-3840
80x= 960
X=960/80=12 hrs

14) A
As per the question, let D be the total distance and
‘t’ is the time taken.
So we have:
D=20t
20t =30(t-0.5)
20t=30t-15
10t=15

www.ibpsguide.com | estore.ibpsguide.com | www.sscexamguide.com


412
Complete Quantitative Aptitude Questions

t=3/2
D= 30 km
Now, for the condition given we have:
30=S(t-3/4)
30=S(3/2-3/4)
30=S((6-3)/4)
30=S(3/4)
S=40 km/hr

15) A
Let r = running time of the train
s= stoppage time of the train
D= total distance travelled by train
We have:
D/r= 120 and………..(1)
D/(r+s) =80………..(2)
Diviving (1)&(2),
D/r *(r+s)/D = 120/80
(r+s)/r =3/2
1 +s/r =3/2
s/ r = 3/2-1
s/ r = ½……………(3)
As, D=480 kms
480/r= 120
r=4
put r value in (3),we get
s/4 =1/2
S = 2 hours

16) C
Time taken by Siva = 8 h
Time taken by Anirudh = 7 h
For your convenience take the product of times taken by both as a distance.
Then the distance = 56km
Since, Siva covers half of the distance in 2 hours(i.e at 5 am)
Now, the rest half (i.e 28 km) will be coverd by both Anirudh and Siva
Time taken by them = 28/15 = 1 hr 52 min

www.ibpsguide.com | estore.ibpsguide.com | www.sscexamguide.com


413
Complete Quantitative Aptitude Questions

Thus , they will cross each other at 6 : 52am.

17) C
Let the distance covered in 1 jump of the monkey be x and,
Distance covered in 1 jump of the dog be y
Then 12 x = 16y
X = 16 / 12 y ---------->1
Ratio of speed of monkey and dog = Ratio of distances covered by them in the same time
=16x : 20y
=256 /12 y : 20y (from 1)
=256 : 240
= 16:15

18) B
Relative speed of the tiger = (5-4)km/hr
=1 km/hr
Time taken by tiger to cover 50 m,
=(50/1000)hr =1/20hr
In 1/20 hrs, the tiger cover a distance of (4*1/20)km
=1/5km =>(1/5*1000)m
=200m

19) B
Distance covered in 1st 2 hrs = (210*2)km
=420km
Distance covered in next 2 hrs = (240*2)km
=480km
Remaining distance = 1035 – (420+480)
=1035 – 900
=135km
Speed in 5th hr = 270 km/hr
Time taken to cover 135 km = 135/270 = ½ hr
Total time taken = (2+2+1/2)
=4 ½ hrs

20) B
Time required = (5 hrs – 5/3 hrs)

www.ibpsguide.com | estore.ibpsguide.com | www.sscexamguide.com


414
Complete Quantitative Aptitude Questions

=(5 hrs – 100 mints)


= (5 hrs – 1hr 40 mints)
= 3 hrs 20 mints
= 3 1/3 hrs
Required speed = (100*3/10) km/hr
=30 km/hr
Original speed = (100/5)km/hr = 20 km/hr
Difference speed = required speed – original speed
=(30-20)
=10km/hr

21) Anitha takes 8 hrs 20 mints in walking to a temple and riding back. She would have gained 3 hrs by riding
both ways. The time she would take to walk both ways is.
a) 15hrs 40mints
b) 16hrs 40 mints
c) 18 hrs 40 mints
d) 19 hrs 40 mints

22) A taxi driver make a trip from the plains to ooty which are 340km apart at an average speed of 60km/hr. In
the return trip, he covers the speed distance at an average of 30 km/hr. the average speed of the taxi over the
entire distance of 680 km is
a) 30
b) 40
c) 50
d) 60

23) Robert travels from his home to anna park at a distance of 125 miles in 11/4 hrs. He returns to home in 2
hrs 15 mints. His average speed is.
a) 50
b) 40
c) 30
d) 20

24) The average speed of a bus in the upward journey is 50% more than that in the return journey. The bus
halts for 2 hrs on reaching the destination. The total time taken for the complete to and from trip is 34 hrs
covering a distance of 1600 km. the speed of the bus in the upward journey is.
a) 63.24

www.ibpsguide.com | estore.ibpsguide.com | www.sscexamguide.com


415
Complete Quantitative Aptitude Questions

b) 65.67
c) 63.49
d) 62.49

25) Senthil started bike at 5 a.m to reach a temple. After going temple, his bike went out of order.
Consequently he rested for 45 mins and came back to his house walking all the way. Senthil reached home at
8 a.m. if bike speed is 20 km/hr and his walking speed is 1 km/hr, then on bike he covered a distance of.
a) 3.12
b) 2.76
c) 4.13
d) 2.14

26) P,Q & R are on a journey by scorpio. P drives during the 1st 2 hrs at an average speed of 60 km/hr. Q
drives during the next 3 hrs at an average speed of 58 km/hr R drives for the next 4 hrs at an average speed
of 62 km/hr. They reached their destination after exactly 5 hrs. Their main speed is.
a) 60 2/9
b) 62 2/9
c) 60 4/8
d) 62 4 /8

27) Karthiga jogs a speed of 18 km/hr at a distance of 27 km. at what speed would she need to jog during the
next 4.5 hrs to have an average of 27km/hr for the entire jogging session.
a) 40km/hr
b) 30 km/hr
c) 20 km/hr
d) 35 km/hr

28) Ganesh is travelling on his motorbike and has calculated to reach golden temple at 8 p.m if he travels at
30 km/hr; he will reach there at 6 p.m if he travels at 45 km/hr. At what speed must he travel to reach the
golden temple at 7 p.m?
a) 36
b) 46
c) 32
d) 30

29) A girl covered a definite distance at same speed. Had she moved 6 km/hr faster, she would have taken 30
mins less. If she had moved 4 km/hr slower, she would have taken 30 mins more. The distance (in km) is.

www.ibpsguide.com | estore.ibpsguide.com | www.sscexamguide.com


416
Complete Quantitative Aptitude Questions

a) 50
b) 40
c) 70
d) 60

30) Anitha travels 400 km by bus at 70 km/hr, 600 km by train at 80km/hr, 250 km by bike at 60 km/hr and 50
kn by car at 40 km/hr. what is the average speed for the total distance?
a) 68.45
b) 69.45
c) 67.45
d) 69.77

21) D
Let the distance be x km. then,
(time taken to walk x km)+(time taken to ride x km) = 25/3 hrs
(time taken to walk 3x km)+(time taken to ride 3x km) = 25/3*3 =25hrs
But the time taken to ride 3xkm=(25/3 – 3)hrs
=(25 – 9)/3 hrs
=16/3 hrs
Time taken to walk 3x km = (25 – 16/3)hrs
=(75 – 16)/3
=59/3
=19 hrs 40mins

22) B
Average speed =(2xy/( x+y)) km/hr
Given x =60 km/hr y = 30km/hr
Average speed = (2*60*30/ (60+30)) km/hr
=(3600/ 90)
=40 km/hr

23) A
Speed from home to annapark:
(125*4/11)mph = (500/11)mph
Speed from annapark to home:
(125*4/9)mph = (500/9) mph
Average speed = 2xy / (x+y) mph

www.ibpsguide.com | estore.ibpsguide.com | www.sscexamguide.com


417
Complete Quantitative Aptitude Questions

=[(2*500/11 *500/9) / (500/11+500/9)]mph


=(500000/99) / ((4500+5500)/99)
=(500000/99) / 10000 /99
=50mph

24) D
Let the speed in return journey be x km/hr
Then speed in upward journey = 150 /100 x = (3/2 x)km/hr
Average speed = (2*3x/2*x) / (3x/2 +x)
=3x2 / (5x/2)
=6x2 / 5x
=6x/5 km/hr
Therefore (1600 * 5/6x) = 32
1600*5 = (32*6x)
192x = 8000
X = 41.66
Speed in upward journey = (3/2 x ) km/hr
=(3/2*41.66)
=62.49 km/hr

25) D
Time taken = 2 hr 15 mins = 2 ¼ hrs
Time = 9/4 hrs
Let the required distance be x km
Then x/20 + x/1 = 9/4
(X+20x) / 20 = 9/4
21x / 20 = 9/4
84x = 180
X= 2.14 km
Therefore the required diatance is 2.14 km

26) A
Distance covered by P = 2*60 = 120 km
Distance covered by Q = 3* 58 = 174 km
Distance covered by R = 4*62 = 248 km
Total distance = (120+174+248)km
=542 km

www.ibpsguide.com | estore.ibpsguide.com | www.sscexamguide.com


418
Complete Quantitative Aptitude Questions

Total time taken = 2+3+4 = 9 hrs


Mean speed = diatance/ time
=(542 / 9)km/hr
=60 2/9 km/hr

27) B
Let the speed of jagging be x km/hr
Total time taken = (27/18 hrs + 4.5 hrs)
=(1.5 hrs + 4.5 hrs)
=6 hrs
Total distance covered = (27+4.5 x) km
Therefore (27 +4.5x) / 6= 27
27+4.5x = 162
4.5x = 135
X=135/4.5
=30
So jagging speed is 30 km/hr

28) A
Let the distance travelled be x km
x/30 – x/45 = 2
(3x – 2x) / 90 = 2
x/90 = 2
x = 180km
Time taken to travel 180 km at 30 km/hr = 180 / 30 = 6 hrs
So ganesh started 6 hrs before 8 pm
That is 2 pm
Required speed = distance / time
=(180/5)
=36 km/hr

29) D
Let distance = x km
Rate = y km/hr
x/y – x / (y+6) = 30/60
(x(y+6)-xy) / y(y+6) = ½
(xy +6x – xy) / y(y+6) = ½

www.ibpsguide.com | estore.ibpsguide.com | www.sscexamguide.com


419
Complete Quantitative Aptitude Questions

12x = y(y+6) ------------>1


And x/ (y-4) – x/y = 30/60
(Xy – x(y-4)) / y(y-4) = ½
2(xy – xy +4x) = y(y-4)
8x = y (y-4) ------------>2
Dividing 1 by 2
12x / 8x = y(y+6) / y(y-4)
3/2 = (y+6) / (y-4)
3y – 12 = 2y +12
Y = 24km
Put y value in 1 we get
12x = 24 (24+6)
12x = 24 *30
X=60km

30) D
Total distance = (400+250+50+600)km
=1300 km
Total time taken = (400/70 + 600/80+250/60+50/40) hrs
=(40/7 +60/8+25/6+5/4)
=(24*40)+(21*60)+(28*25)+(42*5) / 168
=960+1260+700+210 / 168
=(3130 / 168)
Average speed =(1300*168 / 3130) km/hr
=218400/3130
=69.77 km/hr

31) A van travels from namakkal to Madurai at a constant speed. If its speed were increased by 40km/hr it
would have taken 1 hr lesser to cover the distance. It would have taken further 45 mins lesser if the speed
was further increased by 40km/hr. what is distance between the 2 cities.
a) 1680
b) B1660
c) 1670
d) 1675

32) Raghu starts a bike at 40 km/hr and he increases his speed in every hour by 4 km/hr. Then the maximum
distance covered by him in 12 hours is:

www.ibpsguide.com | estore.ibpsguide.com | www.sscexamguide.com


420
Complete Quantitative Aptitude Questions

a) 744 km
b) 658 km
c) 436 km
d) 512 km

33) A bike starts with a speed of 60 km/hr at 8a.m. Due to the problem in engine it reduces its speed as 20
km/hr for every 1 hour. After 9 am, the time taken to covers 15 km is:
a) 13 minutes and 20 seconds
b) 15 minutes and 09 seconds
c) 18 minutes and 15 seconds
d) 22 minutes and 30 seconds

34) Mr.Aneesh left for city 1 from city 2 at 3.00 pm. He travelled at the speed of 60km/hr for 1 and half hours.
After that the speed was reduced to 15 km/hr. If the distance between two cities is 120 kms, at what time did
Mr.Aneesh reach city 1 ?
a) 6.30 pm.
b) 5.30 pm.
c) 7.30 pm.
d) 8.30 pm.

35) Geetha runs at a speed of 12 km per hour and she increases her speed in every hour by 1 km per hour.
In how many hours will he covers 27.8km ?
a) 1 1/4 hours
b) 2 1/3 hours.
c) 3 1/2 hours.
d) 2 1/5 hours.

36) A bus starts at bus stop and reaches a destination in 4 hours. If it travels first and second half at the
speed of 40 km/hr and 50 km/hr respectively then the distance between bus stop and destination is
a) 177.7 km
b) 154.2 km
c) 163.5 km
d) 147.3 km

37) Anand takes 9 hours more than Babu to cover 150 km. Suppose the time taken by Anand is 30 minutes
less than Babu he must double his speed. Then the speed of Anand will be:

www.ibpsguide.com | estore.ibpsguide.com | www.sscexamguide.com


421
Complete Quantitative Aptitude Questions

a) 7.64 km/hr.
b) 7.43km/hr.
c) 7.89 km/hr.
d) 7.25 km/hr.

38) Find the speed and average speed of a bus which leaves salem at 4 p.m. and reaches madurai in the
same day at 8 p.m. The distance between the two stations is 216 km and the total time for stoppage is 1 hour
between these stations.
a) 54,72
b) 62,84
c) 72,54
d) 82,64

39) I reach the bus stop 15 min late if I walk at 2 kmph Starting from my office . Instead, if I walk at 3 kmph, I
reach the bus stop 10 min early. How far is bus stop from my office?
a) 1.5 km
b) 2.5 km
c) 3.5 km
d) 4.5 km

40) If Gowtham ride a bike at 30 km/hr, then he arrives at a certain place at 3 p.m. If he ride at 45 km/hr, he
will arrive at the same place at 1 P.m. At what speed must he ride to get there at noon?
a) 18 km/hr
b) 24 km/hr
c) 36 km/hr
d) 28 km/hr

31) A
Let distance = x km
Usual rate = y km/hr
Then x/y – x/(y+40) =1
X(y+40) / y(y+40) = 1
Xy+40x – xy = y(y+40)
40x = y(y+40) ---------->1
x/y – x/y+80 = 7/4
x(y+80) – xy / y(y+80) = 7/4
xy +80x – xy / y(y+80) = 7/4

www.ibpsguide.com | estore.ibpsguide.com | www.sscexamguide.com


422
Complete Quantitative Aptitude Questions

320x = 7y(y+80) ----------->2


Dividing 1 by 2 we get
8 = 7y(y+80) / y(y+40)
8(y+40) = 7y + 560
8y +320 = 7y+560
Y=240
Substitute y value in 1
40x = 240*280
X=1680 km

32) A
Speed of the rider = 40km/hr.
Distance covered in 1st hour = 40 km.
He increased his speed in every 1 hour by 4 km/hr.
Distance covered in every 1 hours will be, 44, 48, ,... upto 12 terms.(for 12 hours).
The above series is an A.P series;
Sum of first n terms = (n/2)(2a+(n-1)d)
Here, a = 40, d = 4 and n = 12.
Sum of first 12 terms = (12/2)(2(40)+(11)4) = 6(80 + 44)
= 6(124) = 744.
Hence, he covers 744 km in 12 hours.

33) D
Initial speed of the bike= 60 km/hr
Due to engine problem, speed is reduced to 20km for every 1 hour
Speed of the car at 9 am = (60 - 20) = 40km/hr
Time to cover 15 km at 40 km/hr = distance/speed
= 15/40 hours. = 3/8 hours
= 3/8 x 60 minutes = 45/2 minutes = 22 minutes + 1/2 minutes
= 22 minutes + 1/2 x 60 seconds
= 22 minutes and 30 seconds.

34) A
Mr.Aneesh travelled 60 km/hr for 1 1/2 hours (3/2 hours).
Distance covered in 3/2 hours = 60 x 3/2 = 90 km.
Therefore, remaining distance = 120 -90 = 30 km

www.ibpsguide.com | estore.ibpsguide.com | www.sscexamguide.com


423
Complete Quantitative Aptitude Questions

After 3/2 hours, the speed was reduced to 15 km/hr.


Time taken to cover the remaining 30 km = 30/15 = 2 hours
Total time taken = 3/2 + 2 hours = 7/2 hours = 3 hours 30 minutes.
So, Mr.Aneesh reached village 2 at 3.00 pm + 3 hours 30 minutes
= 6.30pm
Hence the required answer is 6.30 pm.

35) D
Geetha starts with 12 kmph.
Distance covered in first 1 hour = 12 km
she increases her speed in every hour by 1 km.
Speed in 2nd hour = 13 km/hr
Distance covered in 2nd hour = 13 km
Remaining distance = 27.8 - (12+13) = 2.8 km
Speed in the third hour = 14 km/hr
Time taken to cover 2.8 km at 14 km/hr = 2.8/14 = 1/5 hour.
Therefore, total time = 1 + 1 + 1/5 hours = 2 1/5 hours.
Hence the answer is 2 1/5 hours.

36) A
Let the distance between bus stop and destination be X.
The total time taken by the bus to cover X = 4 hours
Since X/2 by 40km/hr and remaining X/2 by 50km/hr
Then by Time = distance/speed, we have
(X/2)/40 + (X/2)/50 = 4
X/80 + X/100 = 4
5X + 4X = 1600
9X = 1600
X = 1600/9 = 177.7
Hence 177.7 km is the required answer.

37) C
Let Anand 's speed be X km/hr.
And let the time taken by Babu be Y.
Since Anand takes 9+Y hours to cross 150km at X km/hr.
i.e., 150 / X = 9+Y ---eqn1
And he takes Y - 1/2 hours to cross 150km at 2X km/hr.

www.ibpsguide.com | estore.ibpsguide.com | www.sscexamguide.com


424
Complete Quantitative Aptitude Questions

i.e., 150/2X = Y- 1/2 ---eqn2


Subtract eqn2 from eqn1, we have
150/X - 150/2X = 9+Y - Y + 1/2 = 19/2
(300-150 )/ 2x = 19/2
150/2X =19/2
150/X = 19
X = 7.89
Hence Anand 's speed is 7.89 km/hr.

38) C
Total time taken = 4 hours;
Time of stoppage = 1 hour, that is, actual time taken = 4 hours - 1 hours = 3 hours
Speed = Distance/Time
= 216/3
= 72 km/hr
Average speed = Total Distance/ Total Time
= 216/4
= 54 km/hr

39) B
Let the distance between the office and the bus stop be 1 km
Time required at 2 kmph
=1/2 =30 mins
Time required at 3 kmph
=1/3 =20 mins
Difference = 30−20=10 min.
Actual difference in timings = 15+10 =25 min
If difference is 10 min, distance is 1 km
⇒ If difference is 25 min, distance is 2.5 km
Hence the required answer is 2.5 km

40) C
When speed of gowtham = 30 km/hr
=d/t and,
When speed of the gowtham = 45 km/hr
=d/(t−2)
Equating the value of d:

www.ibpsguide.com | estore.ibpsguide.com | www.sscexamguide.com


425
Complete Quantitative Aptitude Questions

30t =45(t-2)
30t=45t-90
15t=90
t=6 hours
Finally desired speed
=d/(t-1)
=30t/(t-1)
=30*6/(6-1)
=180/5
=36 km/hr

41) A certain thing is thrown twice from a place with the gap of 45 minutes between the two shots. A girl
approaching this point in a train heard the second shot 44 minutes after she heard the first shot. What is the
speed of train (in kmph) if sound travels at 660 m/s?
a) 62 km/hr
b) 54 km/hr
c) 48 km/hr
d) 27 km/hr

42) A bus can travel 25% faster than a van. Both start from a certain shop at the same time and reach poin P,
37.5 kms away from the shop at the same time time. On the way, however, the bus lost about 6.25 mins while
shopping at the some such places. The speed of the van is.
a) 72
b) 73
c) 74
d) 75

43) In covering a definite distance, the speeds of P & Q are in the ratio of 5:6. P takes 45 mins more than Q to
reach the target. The time taken by P to reach the target is.
a) 4 hrs 20 mins
b) 4 hrs 45 mins
c) 2hrs 30 mins
d) 4hrs 30 mins

44) A 1800 km journey of 24 hrs, if 360km is done by van and the rest by train. It takes 60 mins more, if
600km is done by van and the rest by train. The ratio of the speed of the van to that of the train is.
a) 2:4

www.ibpsguide.com | estore.ibpsguide.com | www.sscexamguide.com


426
Complete Quantitative Aptitude Questions

b) 4:3
c) 3:4
d) 5:3

45) P is thrice as fast as Q and Q is 4 times as fast as R. If the journey covered by R is 48 mins, then the time
to be covered by Q.
a) 12
b) 14
c) 10
d) 6

46) A goat is noticed by a lion from a distance of 350m. the goat starts running and the lion chases him. The
goat and the lion run at the ratio of 30km and 31km per hr respectively. What is the distance between them
after 12 mins?
a) 120m
b) 100m
c) 150m
d) 170m

47) 2 pistol were shoot from the same place at an interval of 15 mins and 15 secs but a boy in the train
approaching the place hears the second shot 15 mins after the first. The speed of the train(in
km/hr),supposing that speed travels at 320 metres per second is.
a) 15.166
b) 17.144
c) 18.563
d) 19.188

48) Arun steals a van at 8.30 am and drives it at 40km/hr. The theft is discovered at 9 a.m and the owner
sets off in another van at 55km/hr. when will he overtake arun?
a) 10.20 am
b) 12.20am
c) 11.20am
d) 10.15am

49) Two bike A & B start at the same time from salem to Rasipuram which are 60km apart. If the 2 bikes
travel in opposite directions, they meet after 1 hr and if they travel in same direction(from salem towards
Rasipuram), then A meets B after 3 hrs. what is the speed of bike A?

www.ibpsguide.com | estore.ibpsguide.com | www.sscexamguide.com


427
Complete Quantitative Aptitude Questions

a) 50
b) 70
c) 90
d) 30

50) 2 bus start from A & B respectively and travel towards each other at a speed of 150 km/hr and 120 km/hr
respectively. By the time they meet, the first bus has travelled by 300km more than the second. The distance
between A & B is
a) 2700
b) 2800
c) 2750
d) 2457

41) B
Actual time between the two shots being fired = 45 minutes.
If a girl was stationary she would have heard the shots after 44 minutes. But since the train was moving
towards the source,
she heard the second shot after only 44 minutes = Distance travelled by the sound in 1 min.
the train speed .
=1/44 * (Speed of sound)
= 1/44 *(660)
=15 m/s
Convert m/s into km/hr
15 m/s = (15 *18/5) km/hr
=54 km/hr
Hence the speed of the train is 54 km/hr

42) A
Let speed of the car be x kmph
The speed of the train = 125x/100
=5x/4 km/hr
Therefore 37.5 / x – 37.5 /(5x/4) = 6.25 /60
37.5 / x – (37.5*4)/5x = 6.25 / 60
37.5 / x – 150 / 5x = 6.25 / 60
37.5 / x – 30/x = 6.25 / 60
7.5 / x = 6.25 / 60
6.25x = 7.5*60

www.ibpsguide.com | estore.ibpsguide.com | www.sscexamguide.com


428
Complete Quantitative Aptitude Questions

X=72 km/hr

43) D
Ratio of speeds = 5:6
Ratio of time =6:5
Suppose P takes 6x hrs and Q takes 5x hrs to reach the target. Then,
6x – 5x = 45/60
X=3/4
Time taken by P = 6x hrs
=(6*3/4)hrs
=4.5 hrs
i.e., the time taken by P to reach the target is 4hrs 30mins

44) C
Let the speed of the van be x km/hr
And that of the train be y km/hr
360/x+1440/y = 24 --------------->A
Dividing by 24 on both sides
15/x+60/y = 1 ---------1
And 600/x +1200/y =25
Dividing by 25 on both sides
24/x+48/y = 1 -----------2
Multiply 1 by 24, 360/x +1440/y = 24
Multiply 2 by 15 360/x+720/y = 15
Subtract the above equation
720/y =9
Y=80
Put y value in A
360/x = 24-18
X=360/6
X=60
Ratio of speeds = 60:80 = 3:4

45) A
Let R’s speed = x km/hr
Then Q’s speed = 4x km/hr
P’s speed = 12x km/hr

www.ibpsguide.com | estore.ibpsguide.com | www.sscexamguide.com


429
Complete Quantitative Aptitude Questions

Ratio of speeds of P<Q<R = 12x :4x:x


=12:4:1
Ratio of time taken = 1/12: ¼:1
=1:3:12
If R takes 12 mins, then Q takes 3 mins
If R takes 48 mins, then
Q takes =(3/12 *48)mins
=(144/120mins
=12mins

46) C
To find out the relative speed of the goat and lion
(31-30) = 1 km/hr
Distance covered in 12 mins =(1/60*12)km
=1/5 km
=(1000/5)m
=200m
Distance between the goat and lion = (350 – 200)m
=150m

47) D
Let the speed of the train be x m/sec
Then distance travelled by the train in 15mins = distance travelled by sound in 15secs
X*15*60 = 320*15
900x = 4800
X=4800/900
X=5.33
Speed of the train 5.33 m/s
Convert m/s into km/hr
5.33m =(5.33*18/5)
=(95.94/5)
=19.188 km/hr

48) A
Suppose the arin is overtaken x hrs after 8.30 a.m
Then distance covered by arun in x hrs = distance covered by the owner in(x-1/2) hrs
40x =55(x-1/2)

www.ibpsguide.com | estore.ibpsguide.com | www.sscexamguide.com


430
Complete Quantitative Aptitude Questions

40x = 55x -55/2


55x – 40x =55/2
15x=55/2
X = 55/2*1/15
x=55/30
x= 1hr 50mins
so the thief is overtaken at 10.20 a.m

49) B
Let their speed be x km/hr and y km/hr respectively
Then 60 / (x+y) = ½
X+y = 120 ------------->1
Now when they move in same direction
(distance travelled by A in 3 hrs) – (Distance travelled by B in 3 hrs) = 60km
3x – 3y =60
X – y =20 ------------>2
Adding 1 & 2 we get
2x = 140
X=70
Put x value in 1 we get
Y=120 -70=50
Y=50
Therefore A’s speed = 70 km/hr

50) A
At the time of meeting,
Let the distance travelled by the second bus be xkm
Then distance covered by the 1st bus is (x+300)km
x/120= (x+300) /150
150x = 120(x+300)
150x = 120x +36000
30x = 36000
X=36000/30
X=1200
So the distance covered between A & B =(x+x+300)
=(1200+1200+300)
=2700km.

www.ibpsguide.com | estore.ibpsguide.com | www.sscexamguide.com


431
Complete Quantitative Aptitude Questions

16. TIME AND WORK

1) P and Q together can complete a job in 30 days. Q and R together can complete the same job in 40 days.
P and R together can complete the same job in 40 days. What is the respective ratio of the number of days
taken by P when completing the same job alone to the number of days taken by R when completing the same
job alone?
a) A.2:5
b) B.1:2
c) C.3:4
d) D.2:3

2) 6 boys and 5girls can do a job in 8 days. When 7 boys and 10 girls work on the same job, the work gets
completed in 5 days. How many days will a boy take to do the job, if he works alone on it?
a) A.25
b) B.50
c) C.75
d) D.100

3) Among 4 persons A, B, C and D. A takes twice as much time as B to complete a piece of work. B takes
twice as much time as C and C takes twice as much time as D to complete the same work. One group of
three of the four men can complete the work in 11 days while another group of three can do so in 7 days.
Which is the group that takes 11 days?
a) A.B.C
b) A,B,D
c) B,C,D
d) None of the above

4) A task is done by 46 persons not all of them have the same capacity to task. Every day exactly 2 persons,
do the task with no pair of persons working together twice. Even after all possible pairs have worked once, all
the persons together works for two more days to finish the task. Find the number of days in which all the
persons together will finish the whole task?
a) 44
b) 45
c) 46
d) 47

www.ibpsguide.com | estore.ibpsguide.com | www.sscexamguide.com


432
Complete Quantitative Aptitude Questions

5) A officer undertakes to complete a job in 250 days. He employs 300 male for 50 days and they complete
1/2 of the work . He then reduces the number of male to 100, who work for 120 days, after which there are 20
days holidays. How many male must be employed for the remaining period to finish the work?
a) 25
b) 50
c) 45
d) 35

6) Hanisha can do a work in 25 days, while sudha can do the same work in 50 days. They started the work
jointly.Few days sughana also joined them and thus all of them completed the whole work in 12 days. All of
them were paid total Rs.900. What is the Share of sughana?
a) 252
b) 346
c) 454
d).359

7) X does half as much work as Yand Z does half as much work as X and Y together. If Z alone can finish the
work in 120 days, then together ,all will finish the work in ?
a) 14 1/2
b) 40
c) 24 1/2
d)17 1/3

8) P and Q can do a piece of job in 24 days; Q and R can do it in 30 days while R and P can finish it in 40
days. If P, Q, Rworks together, in how many days will they finish the job? In how many days will each one of
them finish it, working alone?
A) 15,25,35,45
B) 20,30,40,120
C) 20,30,40,50
D) 20,25,30,60

9) P,Q, & R can complete a piece of job together in 20 days. All the 3 started working at it together and after
8 days P left. Then Q & R together completed a job in 20 more days. P alone could complete the work in.
a) 60
b) 50
c) 40
d) 30

www.ibpsguide.com | estore.ibpsguide.com | www.sscexamguide.com


433
Complete Quantitative Aptitude Questions

10) M do a piece of work in 45 days and N can finish it in 40 days. They work together for 5 days and then M
leaves. In how many days will N finish the remaining work?
a) 30 5/9
b) 29 4/9
c) 31 3/5
d) 29 5/9

1) B
Efficiency of P and Q= 1/30 per day ___________1
⇒ Efficiency of Q and R = 1/40 per day_________2
⇒ Efficiency of R and P = 1/40 per day _________3
Taking equation 2 and 3 together
⇒ Q + R = 1/40 and R + P = 1/40
⇒ R and 1/40 will be removed. Hence P = Q
⇒ Efficiency of P = Q =1/60
⇒ Efficiency of R = 1/40-1/60=1/120
⇒ P can do the job in 60 days and R can do the job in 120 days he they work alone.
⇒ Ratio of number of days in which P and R can complete the job 1:2.

2) D
Let the amount of work done by a boy in a day be ‘B′ and the amount of work done by a girl in a day be ‘G′.
Therefore, 6 boys and 5 girls will do 6B+5G amount of work in a day.
If 6 boys and 5 girls complete the entire work in 8 days, they will complete 1/8 of the work in a day.
6B+5G = 1/8………(1)
7B+10G=1/5………….(2)
Solving eqn (1) and eqn (2), we get
12B-7B=1/4-1/5
5B=1/20
B=1/100
i.e. a boy does 1/100th of the work in a day.
Hence he will take 100 days to do the work.

3) B
From the given information B is twice as efficient as A.
C is twice as efficient as B.
D is twice efficient as C.

www.ibpsguide.com | estore.ibpsguide.com | www.sscexamguide.com


434
Complete Quantitative Aptitude Questions

If work done by A in a day is 'n' units, the work done in a day by B, C and D would be 2n, 4n and 8n units
respectively.
It can be seem that, A, B and C working together can do 7n units in a daywhile A, B and D working together
can do 11n units in a day.
Hence, the ratio of times taken to complete the work by the former and later groups is 11:7
A, B and D take 11 days.

4) D
46 persons task in pairs, with no same pair of persons working together.
Each person will be working with other 45 which means each persons will work for 45 days in pair.
Let the time taken by each person be T1 ,T2 ,T3……T45 respectively
According to Question
{ task done when the persons work in pairs}+{ task done when all the persons work together for two days} = 1
45(1/ T1 + 1/T2 +1/T3+……+1/T46+)+ 2(1/ T1 + 1/T2 +1/T3+……+1/T46)=1
47(1/ T1 + 1/T2 +1/T3+……+1/T46)=1
(1/ T1 + 1/T2 +1/T3+……+1/T46)= 1/47
If all the persons work together they will finish the whole task in 47 days.

5) B
300 male in 50 days do = 1/2work
1 male in 1 day does = ½*1/50*1/300 work
100 male in 120 days do = ½ *1/50*1/300 *100*120 =2/5 work
Total work done = ½ + 2/5 =(5+4)/10=9/10
Remaining work = 1-9/10=1/10
Remaining time = (250-50-120-20)=60days
½ work is done in 50 days by 300 male
1/10 work is done in 60 days by
=300*50*1*2/10*60
=50 male

6) A
Efficiency of Hanisha = 1/100*25
=25/100=1/4=>4%
Efficiency of sudha = 2%
Thus, in 12 days working together they will complete only 72% of the work.
[(4+2)*12] =72
Hence, the remaining work will surely done by sughana, which is =(100-72)=28%

www.ibpsguide.com | estore.ibpsguide.com | www.sscexamguide.com


435
Complete Quantitative Aptitude Questions

Thus, sughana will get 28% of Rs. 900,


=28/100*900
which is Rs.252

7) B
Z alone can finish the work in 120 days.
As given Z does half as much work as X and Y together
=> (X + Y) can do it in 60 days
(X+ Y)s 1 days wok = 1/60.
X's 1 days work : Y's 1 days Work = 1/2 : 1 = 1:2(given)
X's 1 day’s work = (1/60) x (1/3) = (1/180)
X's 1 days work = (1/60) x (2/3) = 1/90
(X+Y+Z)'s 1 day's work = (1/180) + (1/90) + (1/120)
=(2+4+3)/360
=9/360
= 1/40
All the three together will finish it in 40 days

8) B
Time taken by (P + Q) to finish the job = 24days.
(P + Q)’s 1 day’s job = 1/24
Time taken by (Q +R) to finish the job = 30days.
(Q + R)’s 1 day’s job = 1/30
Time taken by (R + P) to finish the job = 40 days.
(R + P)’s 1 day’s job = 1/40
Therefore, 2(P + Q + R)’s 1 day’s job = (1/24+1/30+1/40)
=(5+4+3)/120
=12/120=1/10
⇒ (P + Q + R)’s 1 day’s job = ½*1/10 =1/20
Therefore, P, Q, R together can finish the job in 20 days.
Now, P’s 1 day’s job
= {(P + Q + R)’s 1 day’s job } - {(Q + R)’s 1 day’s job }
= (1/20 -1/30)=2/60=1/30
Hence, P alone can finish the job in 30 days.
Q’s 1 day’s work
{(P + Q + R)’s 1 day’s job } - {(R + P)’s 1 day’s job
= (1/20 -1/40)=1//40

www.ibpsguide.com | estore.ibpsguide.com | www.sscexamguide.com


436
Complete Quantitative Aptitude Questions

Hence, Q alone can finish the job in 40 days.


R’s 1 days work
= {(P + Q + R)’s 1 day’s job } - {(P + Q)’s 1 day’s job }
= (1/20 -1/24)=1/120
Hence, R alone can finish the job in 120 days.

9) B
Work done by P,Q & R in 8 days = 1/20 * 8
=2/5
Remaining work = (1 – 2/5) = 3/5
Now 3/5 work is done by Q & R in 20 days
Whole work will be done by Q & R in(20*5/3) =100/3 days
(P+Q+R)’s 1 day work = 1/20
(Q+R)’s 1 day work = 3/100
P’s 1 day work =(P+Q+R)’s 1 day work – (Q+R)’s 1 day work
=1/20 – 3/100
=5-3/100
=2/100 =1/50
P alone could complete the work in 50 days..

10) A
Time taken by M to finish the work = 45 days.
M’s 1 day’s work = 1/45
Time taken by N to finish the work = 40 days.
N’s 1 day’s work = 1/40
(M + N)’s 1 day’s work = (1/40+1/45) = (9+8)/360=17/360
(M + N)’s 5 day’s work (5 × 17/360) = 85/360=17/72
Remaining work (1 – 17/72) = 55/72
Now, 55/72 work is done by N in 1 day
Therefore, 55/72 work will be done by N in (55/72*40) days = 30and 5/9 days.
Hence, the remaining work is done by B in 30 and 5/9 days.

11) After finishing the work by Aswin for 9 days, Ajay finds that only 1/4 of the work has been done. He
employs Ajay who is 80% as efficient as Aswin. How many days more would Ajay take to complete the work?
a) 32
b) 28
c) 34

www.ibpsguide.com | estore.ibpsguide.com | www.sscexamguide.com


437
Complete Quantitative Aptitude Questions

d) 26

12) Adam and Smith are working on a project. Adam takes 18 hrs to type 108 pages on a computer, while
Smith takes 15 hrs to type 120 pages. How much time will they take, working together on two different
computers to type a project of 360 pages?
a) 23 hrs 24 min
b) 45 hrs 42 min
c) 25 hrs 42 min
d) 42 hrs 45 min

13) Arfath is thrice as good workman as deekshith and together they finish a piece of work in 27 days. In how
many days will arfath alone finish the work?
a) 9 days
b) 10 days
c) 11 days
d)12 days

14) Pranesh can do a piece of job in 60 days. He works at it for 4 days and then sarvesh alone finishes the
remaining work in28 days. In how much time will pranesh and sarvesh working together, finish the work?
a) 35 days
b) 30 days
c) 25 days
d) 20 days

15) 90 persons can complete a job in 32 days. 12 days after they started working, 60 more persons joined
them. How many days will they now take to complete the remaining work?
a) 10 days
b) 12 days
c) 14 days
d) 16 days

16) 4 male and 6 female do a piece of work in 20 days while 6 male and 4 female can do the same work in16
days. In how many days can 4 male and 2 female do the work?
a) 3 62/142
b) 2 18/124
c) 4 81/165
d) 1 67/133

www.ibpsguide.com | estore.ibpsguide.com | www.sscexamguide.com


438
Complete Quantitative Aptitude Questions

17) Rishwi can finish a work in 6 days working 4 hours a day. Dhiviksha can complete the same work in 4
days working 5 hrs a day. If both rishwi and dhiviksha work together, working 4 hrs a day, in how many days
can they finished the work?
a) 30/11
b) 11/120
c) 11/30
d) 30/13

18) Nandhu, naren and abishek can do a piece of work in 10,15, and 30 days respectively. In how many days
can Nandhu do the work if he is assisted by naren and abishek on alternate days?
a) 6.6 days
b) 7.5 days
c) 8.4 days
d) 9.1 days

19) X,Y and Z can do a certain job in 72,108 and 144 days respectively. They started the job but X left 16
days before the completion of the job while Y left 24 days before the completion. The number of days for
which Z worked is.
a) 12 days
b) 24 days
c) 36 days
d) 48 days

20) X and Y can do a job in 24 days, Y & Z can do the same job in 36 days. X,Y,Z together can finish it in
18 days. X and Z together will do it in.
a) 24
b) 36
c) 12
d) 26

11) C
Aswin has completed 1/4 of the work in 9 days
Then he can complete the total work in
1/4 ---- 9
1 ---- ?
= 36 days

www.ibpsguide.com | estore.ibpsguide.com | www.sscexamguide.com


439
Complete Quantitative Aptitude Questions

But given Ajay is only 80% as efficient as Aswin


Ajay = 1/36*80/100=1/45
Ajay can complete the total work in 45 days
Now, remaining ¾ of work can be completed in
1 ------ 45
3/4 ------ ?
=3/4*45 =33.75=34 days (approx.)

12) C
Number of pages typed by Adam=108
Number of pages typed by Adam in 1 hour = 108/18=6
Number of pages typed by Smith =120
Number of pages typed by Smith in 1 hour = 120/15 =8
Number of pages typed by both in 1 hour = (6 + 8) = 14
Time taken by both to type 360 pages,
= (360 * 1/14) =25 hrs 42 min

13) A
Arfath’s 1 days work = 3
Deekshith 1 day’s work = 1
Arfath 1 day’s work : Deekshith 1 days work = 3:1
(Arfath + deekshith )’s 1 day’s work = 1/27
Divide 1/27 in the ratio 3:1
Arfath’s 1 days work = 1/27 *3/1 = 3/27
=1/9
Hence Arfath’s alone can finish the work in 9 days

14) D
Work done by pranesh in 4 days = 1/60 *4
=1/15
Remaining work = 1-1/15
=(15 – 1)/15
=14/15
Now 14/15 work done by sarvesh in 28 days
Whole work will be done by sarvesh in(28*15 /14) = 2* 15 =30 days
Pranesh 1 day’s work = 1/60
Sarvesh 1 day’s work = 1/30

www.ibpsguide.com | estore.ibpsguide.com | www.sscexamguide.com


440
Complete Quantitative Aptitude Questions

(pranesh + sarvesh )’s 1 day’s work = 1/60 + 1/30


= (1+2) /60 = 3/60
=1/20
Hence both will finish the work in 20 days

15) B
(90 * 32) persons can complete the work in 1 day.
1 person 1 day’s work = 1/(90 *32)
i.e., 1/2880
90 person’s 12 day’s work = 1/32 * 12 = 3/8
Remaining work =(1- 3/8) = 5/8
150 persons 1 day work = 150 / 2880
=5/96
Now 5/96 work is done by them in 1 day .
Therefore 5/8 work is done by them,
= (96/5*5/8)
=12 days.

16) D
Let 1 male’s 1 day’s work = x
1 female’s 1 day’s work = y
Then 4x + 6y = 1/20 ---------->A
80x + 120y = 1 -----------1
6x + 4y = 1/16 ----------B
96x + 64y = 1 --------2
Multiply 1 by 96  7680x + 11520y = 96 ----3
Multiply 2 by 80 => 7680x + 5120 y = 80 -----4
Subtracting above equation
6400y = 16
Y= 16/6400 = 1/400
Substitute y value in B
6x +4 (1/400) = 1
6x+1/100 = 1
6x = 1 – 1/100 = 99/100
X=99/(100*6) = 33/200
(4 male + 2 female)’s 1 day work = 4x+ 2y
=4(33/200)+2(1/400)

www.ibpsguide.com | estore.ibpsguide.com | www.sscexamguide.com


441
Complete Quantitative Aptitude Questions

=132/200+1/200 = 133/200
So 4 male and 2 female together can finish the work in 200/133 = 1 67 /133 days

17) A
Rishwi can complete the work in (6*4) = 24 hrs
Dhiviksha can complete the work in (4*5) = 20 hrs
Rishwi’s 1 hr work = 1/24
Dhiviksh’s 1 hr work = 1/20(Rishwi and dhivikasha)’s 1 hr work = (1/24 + 1/20)
= (5+6)/120 = 11/120
So both rishwi and dhiviksha will fininsh the work in (120/11)hrs
Number of days of 4 hrs each = 120/11*1/4
=120/44
=30/11 days

18) B
Nandhu’s 1 day work = 1/10
Naren’s 1 day work = 1/15
Abishek’s 1 day work =1/30
Nandhu’s 2 day’s work =(1/10*2) =1/5
(nandhu + naren+abishek)’s 1 day work = 1/10+1/15+1/30
=3+2+1/30
=6/30
=1/5
Workdone in 3 days = 1/5 +1/5
= 2/5
Now 2/5 work is done in 3 days
Whole work will be done in 3 *5/2 = 15/2 = 7.5 days

19) D
Suppose the work was finished in ‘a’ days
Then X’s (a – 16) day’s work + Y’s (a – 24) days + Z’s a days work = 1
(a-16)/72 + (a-24)/108 + a/144 =1
lcm of 72,108,144 is 432
(6(a-16) + 4(a-24) +3a) /432 = 1
6(a-16) + 4(a-24) +3a = 432
6a – 96 +4a – 96 =3a = 432
13a - 192 = 432

www.ibpsguide.com | estore.ibpsguide.com | www.sscexamguide.com


442
Complete Quantitative Aptitude Questions

13a = 624 => a =48


Therefore Z worked in 48 days.

20) A
(X+Y+Z)”s 1 day work =1/18
(X+Y)’s 1 day work = 1/24
(Y+Z)’s 1 day work = 1/36
(X+Z)’s 1 day work = 2[X+Y+Z]’s 1 day work - [(X+Y)’s 1 day work + (Y+Z)’s 1 day work]= 2 [1/18]- (1/24
+1/36)
=1/9 – (1/24 + 1/36) = 1/9 -1/24 – 1/36
=(8-3-2)/72
=3/72 = 1/24
So X & Z together will do the work in 24 days

21) P and Q finished a work in 36 days; Q and R can do it in 60 days; P and R can do it in 45 days. In what
time can P alone do it.
a) 30 days
b) 60 days
c) 90 days
d) 120 days

22) Karthik is 2 times as good workman as Raghu and together they complete a piece of work in 42 days. The
number of days taken by Karthik alone to finish the work is.
a) 55 days
b) 60 days
c) 63 days
d) 67 days

23) M & N can do a work together in 12 days. M is 2 ½ time as efficient as N. the same work can be done by
M alone in.
a) 84/5
b) 6/84
c) 82/5
d) 6/82

24) Vinay can do a work in 28 days. Vikram is 75% more efficient than vinay. The number of days taken by
vikram to do the same piece of work.

www.ibpsguide.com | estore.ibpsguide.com | www.sscexamguide.com


443
Complete Quantitative Aptitude Questions

a) 12 days
b) 14 days
c) 16 days
d) 18 days

25) Sai and Ram working together completed a job in 10 days. If sai worked twice as efficiently as he actually
did and ram worked 2/3 as efficiently as he actually did, the work would have been completed in 6 days. Sai
alone could complete the work in.
a) 40/3 days
b) 42/4 days
c) 44/6 days
d) 43/3 days

26) A bakery P can sold 500 breads in 4 hours, bakery Q can sold the same number of bread in 5 hrs while
bakery R can sold them in 6 hrs. All the bakeries are opened at 8 a.m while bakery P is closed at 9 p.m and
the remaining 2 bakeries complete their target. Approximately at what time will the work be finished?
a) 10
b) 8
c) 12
d) 6

27) M & N do a certain work in 40 days and 24 days respectively. M started the work alone and then after 8
days N joined him till the completion of the work. How long did the work last?
a) 16days
b) 18days
c) 20 days
d) 22 days

28) Arul can do a certain work in 9 days while Ravi can do the same work in 7 days. Both of them complete
the work together and get Rs.288. what is the share of Arul?
a) Rs.124
b) RS.126
c) Rs.128
d) Rs.130

29) 60 girls can do a work in 48 days. 48 boys can finish the same work in 45 days. What is the ratio between
the capacity of a boys and girls?

www.ibpsguide.com | estore.ibpsguide.com | www.sscexamguide.com


444
Complete Quantitative Aptitude Questions

a) 2:1
b) 3:6
c) 2:4
d) 4:3

30) 6 male , 8 female and 12 children can complete a work in 14 days .a female does twice the work of a
male does and a child does half the work a male does . how many female alone can complete this work in 14
days?
a) 15
b) 10
c) 12
d) 14

21) B
(P+Q)’s 1 day work = 1/36 ---1
(Q+R)’s 1 day work = 1/60 ----2
(P+R)’s 1 day work = 1/45 ----3
Adding 1 & 2 & 3 we get
2(P+Q+R)’s 1 day work = 1/36 + 1/60+ 1/45
=(5+3+4)/180 = 12/180 = 1/15
(P+Q+R)’s 1 day work = 1/15 * ½ = 1/30
So P’s 1 day work = (P+Q+R)’s 1 day work – (Q+R)’s 1 days work
= 1/30 – 1/60 = 2-1/60
= 1/60
P alone can do the work in 60 days.

22) C
Karthik’s 1 day work : Raghu’s 1 day work = 2:1
(Karthik + Raghu)’s 1 day work = 1/42
Divide 1/42 in the ratio 2:1
Therefore karthick’s 1 day work = 1/42 * 2/3
=1/63
Hence karthik alone can finish the work in 63 days.

23) A
M’s 1 day work = 2 ½ = 5/2
N’s 1 day work = 1

www.ibpsguide.com | estore.ibpsguide.com | www.sscexamguide.com


445
Complete Quantitative Aptitude Questions

Ratio of M and N = 5/2 : 1= 5:2


Let M’s and N’s 1 day work be 5x and 2x respectively.
Then 5x+2x = 1/12
7x=1/12
X= 1/(12*7)
X=1/84
M’s 1 day work = 1/84 * 5 = 5/84
Hence M alone can finish the work in 84/5 days.

24) C
Given a certain work done by vinay = 28 days
Efficient percentage of vikram = 75%
Ratio of time taken by vinay and vikram = 175 : 100 = 7:4
Suppose vikram takes ‘x’ days to do the work.
7:4 :: 28 : x
7x = 4*28
X= 4*28/7
X= 4*4 = 16 days
So vikram takes 16 days to do the work.

25) A
Let sai’s 1 day work = x
Ram’s 1 day work = y
Then x+y = 1/10 ===>1 and 2x +2y /3 = 1/6
(6x+2y )/3 = 1/6
6x + 2y =1/2 ====>2
Multiply 1 by 6 =>6x+6y = 6/10
2 =>6x+2y = 1 /2
Subtracting above equation we get,
6y – 2y = 3/5 – ½
4y = 6-5/10 = 1/10
4y = 1/10
Y=1/40
Substitute y value in equ 1
X=1/10 – 1/40 = 4-1/10
=3/40
Sai’s 1 day work = 3/40

www.ibpsguide.com | estore.ibpsguide.com | www.sscexamguide.com


446
Complete Quantitative Aptitude Questions

So sai alone could complete the work in 40/3 days.

26) A
(P+Q+R)’s 1 hour work = (1/4 +1/5+ 1/6)
=(15+12+10)/60
=37/60
Remaining work = 1- 37/60 = 23/60
(Q+R)’s 1 hour work = (1/5 +1/6)
=11/30
Now 11/30 work is done by Q and R in 1 hour
So 23/60 work will be done by Q and R in(30/11*23/60)
=23/22 hrs = 1hr
So the work will be finished approximately 1 hr after 9.am i.e., around 10 a.m

27) C
Work done by M in 8 days = 1/40*8
=1/5
Remaining work = 1-1/5 = 4/5
(M+N)’s 1 day work = 1/40 + 1/24
=(3+5)/120 =8/120
=1/15
Now 1/15 work is done by M & N in 1 day.
So 4/5 work will be done by X and Y is,
=15 / 1 * 4/5
=12 days
Hence total time taken = 12+8 = 20 days

28) B
Arul’s 1 day work = 1/9
Ravi’s 1 day work = 1/7
Arul’s salary : Ravi’s salary = Arul’s 1 day work : Ravi’s 1 day work
= 1/9 : 1/7
= 7 :9
Arul’s share = Rs.(7/16 *288)
=Rs.126

29) D

www.ibpsguide.com | estore.ibpsguide.com | www.sscexamguide.com


447
Complete Quantitative Aptitude Questions

Total number of girls =60


Total number of boys =48
(60*48) girls can complete the work in 1 day
2880 girls can complete the work in 1 day
1 girls 1 day work =1/2880
(48*45) boys can complete the work in 1 day
2160 boys can complete the work in 1 day
1 boys 1 day work =1/2160
So required ratio =1/2160 : 1/2880
=2880 : 2160
=4:3

30) D
Let 1 female ‘s 1 day work= x
Then 1 male’s 1 day work =x/2
And 1 child’s 1 day work =x/4
So 6x/2 + 8x +12x/4 =1/14
3x+8x+3x = 1/14
14x = 1/14
X=196
1 female alone can complete the work in 196 days
So, to complete the work in 14 days number of female required = 14.

31) 8 male can finish a work in 6 days.12 female can complete the same work in 9 days. 6 male and 4 female
started working and after 4 days 3 more female joined them. How many days will they now take to complete
the remaining work?
a) 76/41 days
b) 25/62days
c) 3/31days
d) 4/77days

32) Praneetha and Pooja together can do a piece of work in 24 days ,which pooja and priya can do in 32
days. After praneetha has been working at it for 10 days and pooja for 14 days, Priya finishes it in 26 days. In
how many days Priya alone will do the work?
a) 12 days
b) 24 days.
c) 48 days

www.ibpsguide.com | estore.ibpsguide.com | www.sscexamguide.com


448
Complete Quantitative Aptitude Questions

d) 50 days

33) A sum of amount of money is adequate to pay peter’s salary for 42 days and Harish’s salary for 56 days.
The same money is adequate to pay the salary of both for.
a) 12 days
b) 24 days
c) 36 days
d) 48 days

34) If 24 women and 32 girls can do a piece of work in 5 days; 26 women and 48 girls can do it in 4 days then
the ratio of the daily work done by a women to that of a girl is.
a) 2:1
b) 1:3
c) 3:1
d) 1:2

35) 2 male, 6 female, and 8 girls can do a work in 192 hrs, 4 male and 16 girls can do it in 160 hrs, 4 male
and 6 female can do it in 240 hrs.10 male and 24 girls can do it in.
a) A.87 3/ 11 hrs
b) B.78 4 /11 hrs
c) C.88 6 /12 hrs
d) 80 4 /5 hrs

36) P and Q undertake to do a piece of job for Rs.1200. P alone can do it in 12 days while Q alone can do it in
16 days. With the help of R , they finish it in 6 days. Find the share of P,Q & R.
a) 400,600,200
b) 200,400,600
c) 450,400,250
d) 600,450,150

37) A cistern can be filled in 25 minutes. There is a leakage which can empty it in 50 minutes. In how many
minutes cistern can be filled?
a) 20 mints
b) 25mints
c) 45 mints
d) 50 mints

www.ibpsguide.com | estore.ibpsguide.com | www.sscexamguide.com


449
Complete Quantitative Aptitude Questions

38) Rakchita can do a piece of work in 28 days, while Shreya can do the same work in 42 days.They started
the work together but 6 days before the completion of the work, Rakchita left the work. The total number of
days to complete the work is
a) 3
b) 4
c) 5
d) 6

39) 8 men and 8 boys can finish a piece of work in 10 days. 9 women and 7 boys can finish the same work in
10 days. Also 7 men and 6 women can finish the same work in 10 days. In how many days 1man, woman
and one boy can finish the work, at their double efficiency?
a) 25 days
b) 20 days
c) 15 days
d) 10 days

40) A motor bike tyre has two punctures. The 1st puncture alone would have made the tyre flat in 27 minutes
and the 2nd alone would have done it in 18 minutes. If air leaks out at a constant rate, how long does it take
both the punctures together to make it flat?
a) 20 4/3 minutes
b) 15 4/7 minutes
c) 10 4/5 minutes
d) 25 4/3 minutes

31) A
1 male’s 1 day work = 1/48
1 female’s 1 day work = 1/108
Work done in 4 days = 4 (6/48 +4/108)
=4(1/8+1/27)
=1/2+4/27
=(27+8)/54
=35/54
Remaining work = (1 – 35/54) = 19/54
(6 male + 7 female)’s 1 day work = 6/48 + 7/108
=1/8 +7/108
=(27+14)/216
=41/216

www.ibpsguide.com | estore.ibpsguide.com | www.sscexamguide.com


450
Complete Quantitative Aptitude Questions

Now 41/216 work is done by them in 1 day


Therefore time taken = 216/41 * 19/54 = 76/41 days

32) C
Praneetha’s 10 day’s work + Pooja 14 day’s work + Priya’s 26 days work = 1
(Praneetha’s +pooja’s) 10 day’s work + (pooja + priya)’s 4 day’s work +priya’s 22 days work = 1
=>10/24 + 4/32 + priya’s 22 day’s work = 1
Priya’s 22 day’s work = 1 – (10/24 + 4/32)
= 1-(5/12+1/8)
=1-((10+3)/24)
=1-13/24
=11/24
Priya’s 1 day’s work = 11/24 * 1/22
=1/48
Priya can finish the work in 48 days.

33) B
Let the total money be Rs.x
Peter’s 1 day salary = Rs. x/42
Harish 1 day salary = Rs. x/56
(peter+harish)’s 1 day salary = Rs(x/42+x/56)
=(4x+3x)/168
=7x/168
=x/24
Money is adequate to pay the salary of both for 24 days.

34) A
Let 1 women’s 1 day work = x
1 girl’s 1 day work = y
Then 24x+32y = 1/5 =>6x+8y = 1/20 ------->1
26x + 48y = 1 / 4 => 13x + 24y = 1/8 -------->2
Multiply 1 by 3
18x + 24y = 3/20
13x + 24y = 1/8
Subtracting above equations we get
5x = 3/20 – 1/8
= (6-5)/40 =1/40

www.ibpsguide.com | estore.ibpsguide.com | www.sscexamguide.com


451
Complete Quantitative Aptitude Questions

X = 1/(40*5)
X=1/200
Put x value in 1 we get
8y = 1/20 – 6/200
= (10 – 6)/ 200 = 4/200
8y = 1/50
Y=1/400
Required ratio = x:y = 1/200: 1/400
=400:200
=2:1

35) A
Let 1 males 1 hr work = x
1 female 1 hour work = y
1 girl 1 hour work = z
Then 2x + 6y + 8z = 1/192 ------>1
4x + 16z = 1/160 --------->2
4x + 6y = 1/240 ------------->3
Adding 2 and 3
8x+6y + 16z = 1/160 + 1/240
= (3+2) / 480 = 5/480
8x + 6y+16z = 1/96 ----4
Subtract 1 from 4
6x+8z = 1/96 – 1/192
= (2-1) / 192
6x+8z = 1/192 ----5
Solving 2 and 5 we get
X= 1/1920 ; y = 1/2880 ; z = 1/3840
(10 male + 24 girl)’s 1 hr work = 10/1920 + 24 /3840
= 1/192 + 1/160
=(5+6) / 960 = 11/960
10 male and 24 girls can do work in 960/11 hrs
i.e., 87 3 /11 hrs

36) D
R;s 1 day work = 1/6 – (1/12 + 1/16)
=1/6 – 1/12 – 1/16

www.ibpsguide.com | estore.ibpsguide.com | www.sscexamguide.com


452
Complete Quantitative Aptitude Questions

= (8-4-3)/48 = 1/48
P:Q:R = ratio of 1 day work = 1/12:1/16 : 1/48
=4:3:1
P’s share = Rs(1200*4/8)
=Rs.600
Q’s share = Rs(1200 * 3/8)
=Rs.450
R’s share = Rs(1200 * 1/8)
=Rs.150

37) D
Efficiency of filling faucet= 100/25 = 4%
⇒ Efficiency of leakage faucet = 100/50= 2%
⇒ Net filling efficiency = (4-2)%
=2%
So, t cistern can be filled in = 100/2
= 50 minutes

38) D
6 days before the completion of the work Rakchita left the work means in last 6 days only Shreya has worked
alone
So, in last 6 days worked done by Shreya = 6*1/42=6/42=1/7
So, the rest = 1-1/7 =6/7
So 6/7work was done by Rakchita
and Shreya worked together=6/7*70/10
=420/70=6 days

39) A
Efficiency of 8 men and 8 boys = 10%
Efficiency of 9 women and 7boys = 10%
Efficiency of 7 men and 6 women = 10%
So, Efficiency of 15 men, 15women and 15boys = 30%
So, efficiency of 1 man, 1 woman and 1 boy = 2%
Now, since they will work at double their efficiency
Efficiency of 1 man, 1 woman and 1 boy = 4%
Required number of days = 100/4=25 days

www.ibpsguide.com | estore.ibpsguide.com | www.sscexamguide.com


453
Complete Quantitative Aptitude Questions

40) C
1st puncture made the tyre flat in 27 mins
2nd puncture made the tyre flat in 18 mins
1 minute’s work of both the punctures = (1/27 + 1/18)
=(2+3)/54
=5/54
So, both the punctures will make the tyre flat in 54/5 mins
=>10 4/5 minutes

41) A single petrol tank supplies the petrol to the whole city, while the petrol tank is fed by a single pipeline
filling the tank with the stream of uniform volume. When the petrol tank is full and if 20,000 litres of petrol is
used daily, the supply fails in 45 days. If 16,000 litres of petrol is used daily, it fails in 30 days. How much
petrol can be used daily without the supply every failing?
a) 28000
b) 47000
c) 24000
d) 42000

42) The total number of male, female and children working in a factory is 72. They earn Rs.16000 in a day. If
the sum of the wages of all male, all female and all children is in the ratio of 36 : 20 : 24 and if the wages of an
individual male, female and child is in the ratio 6 : 5 : 3, then how much a female earn in a day?
a) Rs 500
b) Rs 1000
c) Rs 1500
d) Rs 1200

43) Rubika can do the 12 times the actual work in 72 days while sathya can do the one-half of the original
work in 6 days. In how many days will both working together complete the 6 times of the original work?
a) 20 days
b) 25 days
c) 30 days
d) 35 days

44) Prabu and balaji are two workers. Working together they can complete the whole work in 20 hours. If the
Prabu worked for 5 hours and balaji worked for 17 hours, still there was half of the work to be done. In how
many hours Prabu working alone, can complete the whole work?
a) 17 hours

www.ibpsguide.com | estore.ibpsguide.com | www.sscexamguide.com


454
Complete Quantitative Aptitude Questions

b) 23 hours
c) 34 hours
d) 37 hours

45) A and B together can complete a work in 24 days. A alone can complete it in 40 days. If B does the work
only for half a day daily, then in how many days A and B together will complete the work?
a) 24 days
b) 15 days
c) 21 days
d) 19 days

46) P, Q and R together earn Rs.420 per day, while P and R together earn Rs.296 and Q and R together earn
Rs.164. The daily earning of R is :
a) Rs.50
b) Rs.80
c) Rs.40
d) Rs.60

47) 48 boys complete a work in 36 days. After they have worked for 24 days, 24 more boys join them. How
many days will they take to complete the remaining work?
a) 8 days
b) 6 days
c) 4 days
d) 3 days

48) Faucet A basically used as inlet pipe and Faucet B is used as outlet pipe. Faucet A and B both are
opened simultaneously, all the time. When Faucet A fills the tank and Faucet B empty the tank, it will take
thrice the time than when both the pipes fill the tank. When Faucet B is used for filling the tank, its efficiency
remains constant. What is the ratio of efficiency of Faucet A and Faucet B respectively?
a) 1:2
b) 2:1
c) 1:3
d) 3:1

49) Pump A can fill the empty bunker in 6 hours, but due to a leak in the bottom it is filled in 7.5 hours, if the
bunker is full and then Pump A is closed then in how many hours the leak can empty it?
a) 30 hours.

www.ibpsguide.com | estore.ibpsguide.com | www.sscexamguide.com


455
Complete Quantitative Aptitude Questions

b) 45 hours.
c) 50 hours.
d) 35 hours.

50) Madhesh does half work as much as vinith in three eighth of the time. If together they take a day to
complete the work, how much time shall vinith take to do it?
a) 21
b) 22
c) 23
d) 24

41) A
Let x litre be the per day filling and
y litre be the capacity of the tank, then
45x + y = 20000 × 45 =900000…. (1)
30x + y = 16000 × 30 =480000…. (2)
Solving eq. (1) and (2), we get
15x=420000
X=420000/15
x = 28000
Hence, 28000 litres per day can be used without the failure of supply

42) B
Ratio of number of male, female and children =36/6:20/5:24/3
= 6x:4x:8x
So, (6x + 4x + 8x) = 72
18x=72
X=72/18=4
So, x = 4
Therefore, number of female = 16
Share of all female =20/80*16000
=Rs 4000
So, Share of each female = 4000/4
= Rs 1000

43) C
Efficiency of Rubika = 16.666%

www.ibpsguide.com | estore.ibpsguide.com | www.sscexamguide.com


456
Complete Quantitative Aptitude Questions

Efficiency of sathya = 4.166%


Total efficiency of rubika and sathya =(16+4) =20%
Actual work 100/20=5
So, they can do actual work in 5 days
So, 6 times work requires 30 days.

44) C
Efficiency of Prabu and balaji = 5%
Prabu worked for 5 hours and balaji worked separately 17 hours.
Which means it can be considered that Prabu and balaji worked together for 5 hours and balaji worked alone
for 12 hours.
Thus, Prabu and balaji in 5 hours can complete 25% work.

It means the remaining (50 ⎯25) = 25% of the work was done by balaji in 12 hours.
Therefore, balaji can do 100% work in 48 hours.
It means the efficiency of balaji = 2.08%
Therefore, efficiency of prabu = (5-2.08) = 2.92%
Thus, prabu require . = 34 hours to complete the work alone.

45) A
B’s 1 day’s work = (1/24 -1/40)
=(5-3)/120
=2/120=>1/60
Now, (A + B)’s 1 day’s work = (1/40 +1/60)
=(3+2)/120
=5/120
So, A and B together will complete the work in 120/5 = 24 days.

46) D
(P+Q+R)’s earning = Rs. 420
(P+R)’s earning=Rs 296
(Q+R)’s earning=Rs 164
Q’s daily earning = Rs. (420 ⎯296) = Rs.124
P’s daily earning = Rs. (420 ⎯164) = Rs.236
Now (P+Q)’s earning=Rs 360
R’s daily earning = Rs. [420 ⎯360] = Rs.60

www.ibpsguide.com | estore.ibpsguide.com | www.sscexamguide.com


457
Complete Quantitative Aptitude Questions

47) A
1 boy’s 1 day’s work = 1/(36*48)
=1/1728
48 boy’s 24 day’s work=48*24/1728=2/3
Remaining work = 1-2/3=1/3
72 boy’s 1 day’s work =72/1728=1/24
1/24 work is done by them in 1 day.
So, 1/3 work is done by them in 24/3= 8 days

48) B
Efficiency when both pipes used to fill = A + B
And efficiency when Faucet A is used to fill and Faucet B is used to empty the tank = A-B
So (A+B)/(A-B)=3/1
A/B+1/(A/B-1)=3/1
A/B+1=3(A/B-1)
3A/B-A/B=4
2A/B=4
A/B=2
Thus, the ratio of efficiency of Faucet A and B =2:1

49) A
Efficiency of A = 16.666%
Effective of leak = 13.333%when there is leakage
So, efficiency of leakage =( Efficiency of A- Effective of leak)
=16.666%-13.333%= 3.333%
It means due to leakage a full bunker will be empty in 30 hours.

50) A
Suppose vinith takes x days to do the work
Madhesh takes (2*3/8 x) = 6/8x
i.e., 3/4x days to do it
(madhesh + vinith’s) 1 day work = 1/9
Therefore 1/x +4/3x = 1/9
(3+4) / 3x = 1/9
7/3x = 1/9
3x = 9*7
X = 63/ 3

www.ibpsguide.com | estore.ibpsguide.com | www.sscexamguide.com


458
Complete Quantitative Aptitude Questions

=21
Vinith takes 21 days to do the work.

www.ibpsguide.com | estore.ibpsguide.com | www.sscexamguide.com


459
1 Adda247 | No. 1 APP for Banking & SSC Preparation
Website:store.adda247.com | Email:ebooks@adda247.com
2 Adda247 | No. 1 APP for Banking & SSC Preparation
Website:store.adda247.com | Email:ebooks@adda247.com
Directions (1–5): This data is regarding total number of employees working in Administration
(admin), Operations (Ops.) and other departments of corporate divisions of Companies A and B.

The total number of employees working in both the companies together is 4800. The respective ratio
of number of employees in Companies A and B is 5 : 7. Each employee works in only one of the 3
Departments
i.e. “ops”, “Admin” and “others”.
In company A, 70% of the total employees are males. 60% of the total male employees work in ‘Ops’
1
out of the remaining male employees, 8 th work in ‘Admin’. Out of the total female employees, 24%
5
work in ‘Admin’ and 8th of the remaining female employees work in ‘Ops’.
In company B, 80% of the total employees are males. 65% of the total male employees work in ‘Ops’.
Number of male employees who work in other departments in Company B is 20% more than the
male employees who work in ‘Other Departments’ in company A. Number of female employees who
work in Ops in Company B are less than the number of male employees who work for ‘Ops’ in the
1
same company by 75%. Out of the remaining female employees, 4 work in ‘Admin’.

Q1. What per cent of the total number of male employees in company A work in ‘other’
departments?
(a) 45 (b) 25 (c) 30
(d) 35 (e) 40

Q2. What per cent the total number of female employees in company B work in administration
department?
(a) 18.5 (b) 8.75 (c) 14
(d) 16 (e) 19

Q3. What is the total number of female employees who work on Ops in Company A and B
together?
(a) 681 (b) 781 (c) 689
(d) 649 (e) 788

Q4. What is the difference between the average number of males working in ‘Admin’ in both the
companies together and average number of females working ‘Other Departments’ in both the
companies together?
(a) 26 (b) 36 (c) 16
(d) 24 (e) 14

3 Adda247 | No. 1 APP for Banking & SSC Preparation


Website:store.adda247.com | Email:ebooks@adda247.com
Q5. In company B, what is the respective ratio between the total number of employees (both male
and female) who work in ‘Admin’ and the total number of employees (both male and female) who
work in ‘Other Department’ in the same company?
(a) 2 : 3 (b) 1 : 3 (c) 1 : 4
(d) 3 : 5 (e) 1 : 5

Solution(1-5):
For company A , Total = 2000
Male (1400) Female (600)
Ops 840 285
Admin 70 144
Other 490 171

For Company B –, Total = 2800


Male (2240) Female (560)
Ops 1456 364
Admin 196 49
Other 588 147

S1. Ans.(d)
490
Sol. Required % = × 100 = 35%
1400

S2. Ans.(b)
49
Sol.Required % = 560× 100 = 8.75%

S3. Ans.(d)
Sol.Required no. of female = 285 + 364 = 649

S4. Ans.(a)
171+147 70+196
Sol. Required difference = −
2 2
= 159 – 133
= 26

S5. Ans.(b)
Sol. Required Ratio = (196 + 49) : (588 + 147)
= 245 : 735
=1:3

4 Adda247 | No. 1 APP for Banking & SSC Preparation


Website:store.adda247.com | Email:ebooks@adda247.com
Directions (6-10): Study the table and answer the given questions.

Data related to Human Resource Dept. of a multinational company (X) which has 145 offices across 8
countries.
Respective Ratio of male
Countries Offices Total Employees % of post graduate
& female employees
A 16 2568 5:7 75
B 18 2880 11 : 5 65
C 14 2310 10 : 11 40
D 22 3575 3:2 60
E 13 2054 7:6 50
F 17 2788 20 : 21 75
G 24 3720 8:7 55
H 21 3360 8:6 80

Q6. The number of male post graduate employees in country H is 1800. If number of female post
graduates increase by 50% in the next year, what % of female employees in that particular country
is post graduate? (Given that all other data remain same)
(a) 76.8% (b) 74% (c) 92.5%
(d) 90% (e) 80%

Q7. In which country, is the percentage of women employees to number of employees (both male
& female) is ranked third lowest?
(a) E (b) B (c) H
(d) F (e) A

Q8. What is the ratio between total number of male employees in countries B and H together and
total number of post graduate employees in same countries?
(a) 76 : 65 (b) 86 : 85 (c) 75 : 76
(d)65 : 76 (e)12 : 33

Q9. What is the difference between average number of post graduate employees in countries A, B
and D together and average number of post graduate employees in countries F, G and H together?
(a) 294 (b) 282 (c) 284
(d) 280 (e) 200

Q10. Which country has the 2nd highest number of average employees per office?
(a) D (b) H (c) G
(d) A (e) F

5 Adda247 | No. 1 APP for Banking & SSC Preparation


Website:store.adda247.com | Email:ebooks@adda247.com
S6. Ans.(c)
Sol.
4
Graduate = 3360 × = 2688
5
Female graduate = 2688 – 1800 = 888
6
Female employee = 3360 × 14 = 1440
3
Female graduate next year = 888 × 2 = 1332
1332
% of female graduate = 1440 × 100 = 92.5%

S7. Ans.(c)
Sol.
7 5
A ⇒ 12 × 100 = 58.34% B ⇒ 16 × 100 = 31.25%
11 2
C ⇒ 21 × 100 = 52.4% D ⇒ 5 × 100 = 40%
6 21
E ⇒ 13 × 100 = 46.15% F ⇒ 41 × 100 = 51.22%
7 6
G⇒ 15 × 100 = 46.67% H ⇒ 14 × 100 = 42.86%
Clearly, H is the third lowest.

S8. Ans.(d)
Sol.
11 8
× 2880 + 14 × 3360 1980 + 1920 3900
16
65 4= =
1872 + 2688 4560
2880 × 100 + 3360 × 5
= 65 : 76

S9. Ans.(a)
Sol.
3 3
A ⇒2568 × 4 = 1926 F ⇒2788 × 4 = 2091
65 55
B ⇒2880 × 100 = 1872 G ⇒3720 × 100 = 2046
3 4
D ⇒3575 × 5 = 2145 H ⇒3360 × 5 = 2688
A + B + D = 5943 F + G + H = 6825
Diff. = 6825 – 5943 = 882
882
Avg. = = 294
3

S10. Ans.(e)
Sol.
2568 2880 2310
A⇒ = 160.5 B⇒ = 160 C⇒ = 165
16 18 14
3575 2054 2788
D⇒ = 162 E⇒ = 158 F⇒ = 164
22 13 17
3360 3720
H⇒ 21
= 160 G⇒ 24
= 155
2nd highest avg. no. of employees per office = F

6 Adda247 | No. 1 APP for Banking & SSC Preparation


Website:store.adda247.com | Email:ebooks@adda247.com
Directions (11-15): The table given below shows the no. of units produced of six different items by
a company, the mark-up % on each unit and the discount offered on the marked-up price of each
unit.

The cost price of all the items is same and fixed at Rs 100.
The line graph shows the estimated percentage of items sold by the company on the normal
discounted price.
The bar graph shows the percentage of cost price at which the company sold the remaining no. of
items.( means the company sold the remaining no. of items at a price lower than the cost price)

A B C D E F
Production 200 160 80 140 180 150
Unit
Mark Up % 50 60 80 40 60 45
Discount % 20 25 40 15 20 20

Series 1

100 90
Estimated Sales % out of Total

90 80 80
75
80 70
70 60
Production

60
50
40
30
20
10
0
A B C D E F

90
80
70
60

40
30

7 Adda247 | No. 1 APP for Banking & SSC Preparation


Website:store.adda247.com | Email:ebooks@adda247.com
Q11. Find profit % of A and B together in year 2015if it is known that B sold only 90% of goods of
what he actually estimated to sell in year 2015.
(a) 0.33% (b) 0.44% (c) 0.55%
(d) 0.60% (e) None of these

Q12. In year 2016 E has increased his discount % by 10 basis point and which lead to increasein its
estimated salesby 20 basis point. Apart from that everything else remains same then what is the
difference in profit in year 2015 to 2016.
(a) 504 (b) 508 (c) 512
(d) 516 (e) 520

Q13. In year 2016 C decreased the discount % by 10 basis point due to which its sales reduced to
80% of total production of year 2015. By how much % profit will increase or decrease in 2016
compare to year 2015. (Approximately)
(a) Increased by 200% (b) Decreased by 210% (c) Increased by 203%
(d) Increased by 207% (e) Decreased by 207%

Q14. What is the difference between the absolute profit of A, B and C together and D, E and F
together in year 2015
(a) 750 (b) 800 (c) 900
(d) 1000 (e) None of these

Q15. What is the profit % of all the companies together in year 2015. (Approximately)
(a) 4.9% (b) 4.6% (c) 5.1%
(d) 4.7% (e) 5.4%

S11. Ans.(a)
Sol.
100×150 80
A’s selling price per unit = × 100 = 120
100
80
A’s sales in unit = 200 × 100 = 160
A’s SP of 160 Unit = 160 × 120 = 19200
60
A’s SP of Remaining unit = (200 − 160) × 100 × 100 = 2400
100×160 75
B’s Selling price per unit = × 100 = 120
100
75 90
B’s Sales in Unit = (160 × 100) × 100 = 108
30
B’s total SP = 108 × 120 + [(160 − 108) × 100 × 100] = 14520
Total SP (A + B) = 19200 + 2400 + 14520 = 36120
Total CP (A + B) = (200 + 160) × 100= 36000
36120−36000
Required Profit % = ( ) × 100 = 0.33%
36000

8 Adda247 | No. 1 APP for Banking & SSC Preparation


Website:store.adda247.com | Email:ebooks@adda247.com
S12. Ans.(a)
Sol.
E’s total SP in year 2016
160 70 90 70 180 × 10
= [(100 × )× ] × (180 × ) + (100 × × )
100 100 100 100 100
= 19404
E’s total SP in year 2015
160 80 70 70 (180 × 30)
= [(100 × )× ] × (180 × ) + (100 × × )
100 100 100 100 100
= 19908
Required answer = 19908 − 19404
= 504

S13. Ans.(c)
Sol.
SP of C in year 2016
180 70 80 20 90
= [(100 × )× ] × (80 × ) + (80 × ) × (100 × )
100 100 100 100 100
= 9504
Profit of C in year 2016 = 9504 − 80 × 100
= 1504
180 60 90 10 90
SP of C in year 2015 = [(100 × 100) × 100] × (80 × 100) + (80 × 100) × (100 × 100)
= 8496
Profit of C in year 2015 = 8496 − 80 × 100
= 496
1504−496
Required % = = 203.22% ~ 203%
496

S14. Ans.(c)
Sol.
A’s SP in year 2015
80 150 80 20 60
= (200 × ) × (100 × × ) + (200 × ) × (100 × )
100 100 100 100 100
= 21600
B’s SP in year 2015 =
75 160 75 25 30
= (160 × ) × (100 × × ) + (160 × ) × (100 × )
100 100 100 100 100
= 15600
C’s SP in year 2015 s
90 180 60 10 90
= (80 × ) × (100 × × ) + (80 × ) × (100 × )
100 100 100 100 100
= 8496

9 Adda247 | No. 1 APP for Banking & SSC Preparation


Website:store.adda247.com | Email:ebooks@adda247.com
Total (A + B + C) = 21600 + 15600 + 8496 = 45696
Profit = 45696 − 20000 − 16000 − 8000 = 1696
80 140 85 20 40
D’s SP in year 2015 = (140 × ) × (100 × × ) + (140 × ) × (100 × )
100 100 100 100 100
= 14448
70 160 80 30 70
E’s SP in year 2015 = (180 × 100) × (100 × 100 × 100) + (180 × 100) × (100 × 100)
= 19908
60 145 80 40 80
F’s SP in year 2015 = (150 × 100) × (100 × 100 × 100) + (150 × 100) × (100 × 100)
= 15240
Total SP (D + E + F) = 1448 + 19908 + 15240
= 49596
Profit = 49596 − 14000 − 18000 − 15000
= 2596
Required answer = 2596 − 1696
= 900

S15. Ans.(d)
Sol.
total profit = 2596+ 1696
= 4292
Total CP = 20000+ 16000+ 8000 + 14000 + 18000+ 15000
= 91000
4292
Required profit % = 91000 × 100
= 4.716 ~ 4.72%

Direction (16-20): Study the graph to answer the questions.

Total investment (in Rs. thousand) of Gaurav and Rishabh in 6 schemes (M, N, O, P,Q and R)
investment

120

100
Total investment

80

60

40

20

0
M N O P Q R
Schemes

10 Adda247 | No. 1 APP for Banking & SSC Preparation


Website:store.adda247.com | Email:ebooks@adda247.com
Percentage of Gaurav’s Investment out of total investment

70

60

50
% of investment

40

30

20

10

0
M N O P Q R

Schemes

Q16. Scheme M offers simple interest at a certain rate of interest (per cent per annum). If the
difference between the interest earned by Gaurav and Rishabh from scheme M after 4 yr is Rs.
4435.20, what is the rate of interest (per cent per annum)?
(a) 17.5 (b) 18 (c) 16.5
(d) 20 (e) 15

Q17. What is the respective ratio between total amount invested by Gaurav in schemes O and Q
together and total amount invested by Rishabh in the same scheme together?
(a) 31 : 44 (b) 31 : 42 (c) 27 : 44
(d) 35 : 48 (e) 29 : 38

Q18. If scheme O offers compound interest (compounded annually) at 12% per annum, then what
is the difference between interest earned by Gaurav and Rishabh from scheme O after 2 yr?
(a) Rs. 1628.16 (b) Rs. 1584.38 (c) Rs. 1672.74
(d) Rs. 1536.58 (e) Rs. 1722.96

Q19. Rishabh invested in scheme R for 4 yr. If scheme R offers simple interest at 7% per annum
for the first two years and then compound interest at 10% per annum (compound annually) for the
3rd and 4th year, then what will be the interest earned by Rishabh after 4 yr?
(a) Rs. 13548.64 (b) Rs. 13112.064 (c) Rs. 12242.5
(d) Rs. 12364 (e) Rs. 11886

11 Adda247 | No. 1 APP for Banking & SSC Preparation


Website:store.adda247.com | Email:ebooks@adda247.com
Q20. Amount invested by Gaurav in scheme S is equal to the amount invested by him in scheme
N. The rate of interest per annum of schemes S and N are same. The only difference is scheme S
offers compound interest (compounded annually), whereas the scheme N offers simple interest. If
the difference between the interest earned by Gaurav from both the schemes after 2 yr is Rs.
349.92, then what is the rate of interest?
(a) 9% (b) 5% (c) 13%
(d) 11% (e) 7%

S16. Ans.(c)
Sol.
Amount invested by Gaurav in scheme M = 54% of 84000
= Rs. 45360
∴ Amount invested by Rishabh in scheme M = 84000 – 45360
= Rs. 38640
Let the required rate be r% per annum. Then,
45360 × 𝑟 × 4 38640 × 𝑟 × 4
= − = 4435.20
100 100
⇒6720 × r × 4 = 443520
⇒ r = 16.5%

S17. Ans.(a)
Sol.
Required ratio = (Total amount invested by Gaurav in schemes O and Q together) : (Total amount
invested by Rishabh in schemes O and Q together)
= (40% of 32000 + 42% of 64000) : (60% of 32000 + 58% of 64000)
= 39680 : 56320 = 31 : 44

S18. Ans.(a)
Sol.
Difference of amount invested by Gaurav and Rishabh in
Scheme O = 60% of 32000 – 40% of 32000 = 20% of 32000
= Rs. 6400
∴ Required difference in their interest
12 2
= 6400 [(1 + ) − 1] = 6400 × 0.2544 = 𝑅𝑠. 1628.16
100

S19. Ans.(b)
Sol.
Amount invested by Rishabh in investment R
= (100 – 64)% of 96000 = 36% of 96000 = Rs. 34560
Then, total interest earned by Rishabh after 4 year
34560 × 7 × 2
= + 21% of (34560 + SIoffirst 2 years)
100
= 4838.40 + 8273.664 = Rs. 13112.064

12 Adda247 | No. 1 APP for Banking & SSC Preparation


Website:store.adda247.com | Email:ebooks@adda247.com
S20. Ans.(a)
Sol.
Amount invested by Gaurav in each of scheme S and N
= 60% of 72000 = 43200
Let the rate of interest be r% per annum.
Then, according to the question,
43200 × 𝑟 2
349.92 =
1002
2
𝑜𝑟, 𝑟 = 81
∴r = 9%

Directions (21–25): The graph suggests the no. of consumers and consumption of electricity units
in five years. Electricity units are given in Lacs while the no. of consumers aregiven in thousand.
Read the graph and answer the question.

Consumers (In thousand) Units(in Lacs)


700

600 575
550

500 450

400 375
350
325
300
300 250
225
200
200

100

0
2011 2012 2013 2014 2015

Q21. What is the ratio of electricity consumption per consumer in 2012 to the same in 2015?
(a) 39 : 44 (b) 77 : 79 (c) 11 : 19
(d) 9 : 7 (e) None of the above

Q22. If no of consumers in 2016 is 120% more than in 2011 while the consumption remain same as
in 2015, then what will be the impact of no of units consumed by a consumer in 2016 when
compared to electricity consumption per consumer in 2015?
(a) + 42 units (b) + 36 Units (c) – 36units
(d) – 42 units (e) None of the above
13 Adda247 | No. 1 APP for Banking & SSC Preparation
Website:store.adda247.com | Email:ebooks@adda247.com
Q23. Electricity consumption in 2012 will be approximately how many times the total no. of
consumer all over the years?
(a) 3 (b) 21.5 (c) 2.5
(d) 4 (e) None of the above

Q24. Total no of units in 2011 and 2013 are approximately what % more or less than Total units in
2012 & 2014 together?
(a) 20% more (b) 24% more (c) 29% less
(d) 28% less (e) None of the above

Q25. In which of the following year, the ratio of unit consumtion to the no. of consumers is
maximum?
(a) 2011 (b) 2015 (c) 2014
(d) 2013 (e) 2012

S21. Ans.(a)
Sol.
325
250 325 × 375
550 = = 39 ∶ 44
250 × 550
375

S22. Ans.(c)
Sol.
220
2016 : No. of consumers = 100 [225] = 495 thousand
Electricity consumption = 550 Lacs
550×100000
∴ Electricity consumption per consumer = 495×1000
= 111 units per consumer
550×100000
2015 : Electricity consumption per consumer = 375000
≈ 147 units per consumer
Hence, the Impact is reduction of 36 units per consumer

S23. Ans.(b)
Sol.
Total consumer all over the year = 225 + 250 + 300 + 350 + 375 = 1500 thousand
325×100000
Desired value = = 21.5 times approx
1500000

S24. Ans.(d)
Sol.
Total units in 2011 and 2013 = 650 Lacs
Total units in 2012 and 2014 = 900 Lacs
250
Desired value = 900 × 100 ≈ 28%approx
14 Adda247 | No. 1 APP for Banking & SSC Preparation
Website:store.adda247.com | Email:ebooks@adda247.com
S25. Ans.(c)
Sol.It is clear from the graph that unit consumption is highest in 2014 while consumers-electricity
units difference is maximum as well. Hence, Ratio of unit consumption to the number of consumers
is maximum in 2014.

Directions (26-30): The following information is about performance of Akhilesh in SBI PO mains
exam. Read the information carefully and answer the following question.

The exam consists of 200 marks, with 5 sections i.e. Reasoning, quant, English, G.A., Computers.
3
Akhilesh attempted 22 questions in Reasoning with an accuracy of 77 11 % . Each question of
reasoning consists of 2 marks with a negative marking of 25%. (if right question is of 2 mark, then 0.5
mark will be deducted for each wrong answer).
Each section of the exam have the 25% of negative marking for each wrong question. The total
1
number of questions in reasoning is 30. Each question of computer consists of 2 marks and maximum
marks in computer are 10. Total 16 questions are attempted by Akhileshin computer with the ratio of
right questions to wrong questions 3 : 1.
The number of questions in English is equal to maximum marks of English. Akhilesh attempted 26
questions with 50% accuracy. The number of questions attempted in English is 65% of the total
number of questions in English.
GA section consists of 40 questions with each question 0.75 marks. Akhilesh attempted 23 questions
out of which 8 are wrong. Quant section contains 40 questions out of which Akhilesh attempted 35
questions and got 52.5 marks.

Q26. Another student arunoday attempted 70% questions in the same exam, then find the number
of questions left by arunoday.
(a) 119 (b) 68 (c) 51
(d) 65 (e) None of these

Q27. Find the marks obtained by Akhilesh in GA.


(a) 8.75 (b) 9.25 (c) 9.75
(d) 10.75 (e) None of these

Q28. The number of correct questions in reasoning is how much more than the number of
incorrect questions in the same subject?
(a) 12 (b) 7 (c) 18
(d) 9 (e) None of these

Q29. Find the total marks obtained by Akhilesh in the exam.


(a) 101 (b) 105 (c) 109
(d) 102 (e) None of these
15 Adda247 | No. 1 APP for Banking & SSC Preparation
Website:store.adda247.com | Email:ebooks@adda247.com
Q30. Find the total number of incorrect questions attempted by Akhilesh in the exam.
(a) 27 (b) 15 (c) 28
(d) 18 (e) None of these

S26. Ans.(c)
Sol. Total number of question = 170, no of questions left= 170 – 119=51

S27. Ans.(c)
Sol.Marks in GA = 9.75

S28. Ans.(a)
Sol. 17 – 5 = 12

S29. Ans.(c)
Sol.Total marks obtained = 109

S30. Ans.(e)
Sol. Total number of incorrect questions = 122 – 85=37

Directions (31-35): Study the following graph carefully and answer the questions given below it.
Percentage of profit earned by two companies Sony and H.P. over the given years
𝐼𝑛𝑐𝑜𝑚𝑒 − 𝐸𝑥𝑝𝑒𝑛𝑑𝑖𝑡𝑢𝑟𝑒
% 𝑃𝑟𝑜𝑓𝑖𝑡 = × 100
𝐸𝑥𝑝𝑒𝑛𝑑𝑖𝑡𝑢𝑟𝑒
60

50

40

30 Sony
HP
20

10

0
2007 2008 2009 2010 2011 2012

16 Adda247 | No. 1 APP for Banking & SSC Preparation


Website:store.adda247.com | Email:ebooks@adda247.com
Q31. Expenditure of Company HP in 2008 and 2009 are Rs. 12 lakhs and Rs. 14.5 lakh respectively.
What was the total income of Company B in 2008 and 2009 together (in lakh rupees)?
(a)35 lac (b)37.65 lac (c)40 lac
(d)37.95 lac (e)None of these

Q32. Ratio of expenditure of companies Sony and HP in 2011 was 3 : 4 respectively. What was the
respective ratio of their incomes in 2011?
(a)2 : 3 (b)23 : 37 (c)43 : 56
(d)29 : 46 (e)39 : 56

Q33. Total expenditure of Company Sony in all the years together was 82.5 lakhs. What was the
total income of the Company in all the years together?
(a)38 lac (b)40 lac (c)45 lac
(d) Cannot determined (e)None of these

Q34. If the expenditures of Companies Sony and HP in 2012 were equal and the total income of
the two companies was Rs. 5.7 lakh, What was the total expenditure of the two companies in 2012?
(a)4 lac (b)5 lac (c)6 lac
(d)8 lac (e)10 lac

Q35. If the income of Company HP in 2009 and 2010 were in the ratio of 2 : 3 respectively. What
was the respective ratio of expenditure of that Company in these two years?
(a) 2 : 3 (b)4 : 5 (c)29 : 45
(d)39 : 55 (e)None of these

S31. Ans.(d)
Sol.
Income of HP = 𝐼1 in 2008
𝐼1 − 12
∴ 35 = × 100
12
𝐼1 = Rs. 16.2 L
In 2009, Let Income = 𝐼2
𝐼2 − 14.5
∴ 50 = × 100
14.5
𝐼2 = 21.75 L
∴total income = 21.75 𝐿 + 16.2 𝐿 = 37.95 𝐿

17 Adda247 | No. 1 APP for Banking & SSC Preparation


Website:store.adda247.com | Email:ebooks@adda247.com
S32. Ans.(e)
Sol.
Let the respective expenditures of both Sony and HP be Rs. 3𝑥 and Rs. 4𝑥 lakhs.
𝐼1 − 3𝑥
∴ 𝐼𝑠𝑜𝑛𝑦 𝑖𝑛 2011 ⇒ 30 = × 100
3𝑥
or, 𝐼1 = 3.9𝑥
𝐼2 − 4𝑥
𝐴𝑔𝑎𝑖𝑛, 𝐼𝐻𝑃 𝑖𝑛 2011 ⇒ 40 = × 100
4𝑥
⇒ 𝐼2 = 5.6𝑥
Desired ratio ⇒ 𝐼𝑠𝑜𝑛𝑦 ∶ 𝐼𝐻𝑃 = 3.9𝑥 ∶ 5.6𝑥
=39 : 56

S33. Ans.(d)
Sol.It can’t be determined as data given are inadequate.

S34. Ans.(a)
Sol.
Let expenditure of both Sony and HP in 2012 beRs. 𝑥 lakhs & their respective incomes be Rs. 𝐼1 &𝐼2
lakhs.
∴ Profit% for Sony = 40
& Profit% for HP = 45
𝐼1 −𝑥
∴40 = × 100 … (𝑖)
𝑥
𝐼2 −𝑥
&45 = × 100 … (𝑖𝑖)
𝑥
From (i) and (ii)
𝑥 = Rs. 2L
∴ Total expenditure = 2 × 2 = Rs. 4 lakh

S35. Ans.(c)
Sol.
Let the income be Rs. 2𝑥and Rs. 3𝑥lakhs respectively in 2009 and 2010 for HP.
∴In 2009,
2𝑥 − 𝐸1
50 = × 100
𝐸1
⇒ 1.5 𝐸1 = 2𝑥
2𝑥
⇒ 𝐸1 = 𝐿𝑎𝑘ℎ
1.5
In 2010,
3𝑥 − 𝐸2
45 = × 100
𝐸2
3𝑥
⇒ 𝐸2 =
1.45
2𝑥 3𝑥
∴ ∶ = 29 ∶ 45.
1.5 1.45
18 Adda247 | No. 1 APP for Banking & SSC Preparation
Website:store.adda247.com | Email:ebooks@adda247.com
Directions (36-40): Study the given graph carefully to answer the questions that follow:

70

60
No. of People (in thousands)

50

40 IDEA
30 JIO

20
Airtel

10

0
2014 2015 2016 2017 2018 2019
Year

Q36. What is the average number of people using mobile service of JIO for all the years together?
2 1 2
(a) 16 3 (b) 14444 6 (c)16666 3
1
(d) 14 6 (e) None of these

Q37. The total number of people using all the three mobile services in the year 2017 is what per
cent of the total number of people using all the three mobile services in the year 2018?(rounded off
to two digits after decimal)
(a) 89.72 (b) 93.46 (c) 88.18
(d) 91.67 (e) None of these

Q38. The number of people using mobile service of Idea in the year 2016 forms approximately
what per cent of the total number of people using all the three mobile services in that year?
(a) 18 (b) 26 (c) 11
(d) 23 (e) 29

Q39. What is the ratio of the number of people using mobile service of AIRTEL in the year 2015 to
that of those using the same service in the year 2014?
(a) 8 : 7 (b) 3 : 2 (c) 19 : 13
(d) 15 : 11 (e) None of these

19 Adda247 | No. 1 APP for Banking & SSC Preparation


Website:store.adda247.com | Email:ebooks@adda247.com
Q40. What is the total number of people using mobile service ofJIO in the years 2018 and 2019
together?
(a) 35,000 (b) 30,000 (c) 45,000
(d) 25,000 (e) None of these

S36. Ans.(c)
1
Sol. Average = 6 × [5 + 10 + 25 + 20 + 25 + 15] × 1000
100000 2
= = 16666
6 3

S37. Ans.(d)
55
Sol. Required % = 60 × 100 = 91.67%

S38. Ans.(a)
10
Sol. Required % = × 100 = 18% (approx.)
55

S39. Ans.(b)
Sol. Required Ratio = 15 : 10 = 3 : 2

S40. Ans.(e)
Sol. Required no. of people = (25 + 15) × 1000 = 40000

Directions(41-45): Study the given information carefully to answer the questions that follow:

An organization consists of 2400 employees working in different departments, viz HR, Marketing, IT,
Production and Accounts. The ration of male to female employees in the organization is 5 : 3. Twelve
percent of the males work in the HR department. Twenty four percent of the females work in the
Accounts department. The ratio of males to females working in the HR department is 6:11. One-ninth
of the females work in the IT department. Forty two percent of the males work in the Production
department. The number of females working in the production department is 10 percent of the males
working in the same. The remaining females work in the marketing department. The total number of
employees working in the IT department is 285. Twenty two percent of the males work in the
Marketing and the remaining work in the Accounts department.

Q41. The number of males working in the IT department forms approximately what percent of the
total number of males in the organization?
(a) 5 (b) 12 (c) 21
(d) 4 (e) 18

Q42. What is the difference between males in Accounts department and Males in IT department?
(a) 10 (b) 15 (c) 18
(d) 16 (e) None of these

20 Adda247 | No. 1 APP for Banking & SSC Preparation


Website:store.adda247.com | Email:ebooks@adda247.com
Q43. The total number of employees working in the Accounts department forms what percent of
the total number of employees in the organization?
(a) 19.34 (b) 16.29 (c) 11.47
(d) 23.15 (e) None of these

Q44. The number of females working in the Production department forms what percent of the
total number of females in the organization?
(a) 7 (b) 2 (c) 4
(d) 15 (e) None of these

Q45. What is the total number of females working in the HR and Marketing departments together?
(a) 363 (b) 433 (c) 545
(d) 521 (e) None of these

Solutions (41–45):
Total no. of employees = 2400
5
No. of males = 8 × 2400 = 1500
& No. of females = 900
Males (HR) = 12% of 1500 = 180
11
Females (HR) = × 180 = 330
6
∴ Females (Accounts) = 24% of 900 = 216
1
&Females (IT) = 9 × 900 = 100
No. of Males in IT = 285 – 100 = 185
∴ No. of males in Production = 42% of 1500
= 630
Females (Production) = 10% of 630 = 63
22×1500
Males (Marketing) = = 330
100
No. of females in Marketing = (900 – 330 – 216 – 100 – 63)
= 191
No. of Male in Accounts = 1500 – 180 – 185 – 630 – 330
= 175

S41. Ans.(b)
185
Sol. Desired% = 1500 × 100 = 12.33% ≈ 12%

S42. Ans.(a)
Sol. No. of males in accounts = 175
No. of males in IT = 185
Difference = 10

21 Adda247 | No. 1 APP for Banking & SSC Preparation


Website:store.adda247.com | Email:ebooks@adda247.com
S43. Ans.(b)
(216+175)
Sol. Reqd. % = × 100 = 16.29%
2400

S44. Ans.(a)
63
Required %= 900 × 100 = 7%

S45. Ans.(d)
Sol. Females in (HR + Marketing) = 330 + 191 = 521

Directions (46-50): Read the given bar graph and answer the following questions.

Number of Students taking fresh education loan from different Banks


6000

5000

4000 2008
2009
3000
2010
2011
2000
2012

1000

0
SBI PNB BOB UCO OBC

Q46. Approximately how many students taking a loan from UCO in 2009 and PNB in 2010 were
defaulters if 23% from UCO in 2009 and 20% from PNB in 2010 have defaulted?
(a) 630 (b) 650 (c) 600
(d) 750 (e) 840

Q47. In 2007, no of defaulters in SBI was 5%. However each year no of defaulters increases by 10%
in number. What will be the difference between the number of defaulters of SBI in the year 2009
and 2012?
(a) 1500 (b) 2000 (c) 1325
(d) 1456 (e) Cannot be determined

22 Adda247 | No. 1 APP for Banking & SSC Preparation


Website:store.adda247.com | Email:ebooks@adda247.com
Q48. In which of the following years, the difference in no. of students taking loan from Bank BOB
from the previous year is highest?
(a) 2008 (b) 2009 (c) 2010
(d)2012 (e) None of these

Q49. If on average, Rs. 175000 per students education loan sanctioned by OBC bank all over the
year. What will be total amount sanctioned by OBC in all given years?
(a) 1055600000 (b) 1055800000 (c) 1620000000
(d) 1050000000 (e) None of the above

Q50. What is the ratio of Number of students taking Education Loans from SBI and BOB together
in all the years and the total no of students taking Education loans in 2010 and 2011 together?
(a) 8 : 5 (b) 5 : 7 (c) 7 : 5
(d) 9 : 7 (e) None of these

S46. Ans.(a)
Sol.
Students taking loan from UCO in 2009 = 1000
Defaulters (UCO) = 23% of 1000 = 230
Person taking loan from PNB in 2010 = 2000
Defaulters (PNB) = 20% of 2000 = 400
Total desired defaulters = 230 + 400 = 630

S47. Ans.(e)
Sol.Cannot be determined because no. of students taking a loan from SBI in 2007 is unknown.

S48. Ans.(b)
Sol.From graph, it is clear that in 2009, difference between no. of students taking a loan is highest as
compared to previous year.

S49. Ans.(e)
Sol.No. of students taking education loan from OBC bank all over the year
= 1000 + 1000 + 1500 + 2000 + 1500= 7000
Total loan amount sanctioned over the years = 7000 × 1,75,000
= Rs. 1,22,50,00,000

S50. Ans.(c)
Sol.
SBI : 2500 + 3000 + 4500 + 4000 + 5000 = 19000
BOB : 2500 + 3500 + 4000 + 4500 + 5000 = 19500

23 Adda247 | No. 1 APP for Banking & SSC Preparation


Website:store.adda247.com | Email:ebooks@adda247.com
Total no. of students taking loan in 2010 = 13500
Total no. of students taking loan in 2011 = 14000
19000+19500 38500 7
Desired ratio = 13500+14000 = 27500 = 5

Directions (51-55): Study the following line graph carefully and answer the questions given below.

% of literacy rate from 1951 to 2001


Female Persons Males
80 75.26

70 64.13
56.38
60 64.84
45.96
50
40.4 52.21 53.97
40
27.16 43.57
30 39.55 39.29

20 28.3 29.76
18.33 21.97
10 15.35
0 8.86
1951 1961 1971 1981 1991 2001

Q51.What is the approx difference between the percentage literacy rate increased in male from
1951 to 1991 and percentage literacy rate increased in female from 1971 to 2001.
(a) 15% (b) 10% (c) 5%
(d) 20% (e) 2%

Q52.In which of the census years percentage increase given in male literacy rate was the highest
with respect to previous census year?
(a) 1981 (b) 1991 (c) 2001
(d) 1961 (e) None of these

Q53.In which of the given census years was the percentage increase in the literacy rate of females
the lowest with respect to that of previous census year?
(a) 1981 (b) 1991 (c) 2001
(d) Data inadequate (e) None of these

Q54.In which of the given census years was the percentage increase in the number of males the
highest with respect to the previous census year?
(a) 1981 (b) 1991 (c) 2001
(d) Data inadequate (e) None of these

24 Adda247 | No. 1 APP for Banking & SSC Preparation


Website:store.adda247.com | Email:ebooks@adda247.com
Q55.What is the ratio of percentage literacy rate increased of male from (1961-1981) to literacy rate
increased of person in 1971?
(a) 1 : 1 (b) 1 : 2 (c) 2 : 1
(d) 1 : 3 (e) 3 : 1

S51. Ans.(b)
Sol.
53.97−21.97 64.13−27.16
Required difference = ( ) × 100 − ( × 100)
21.97 27.16
32 36.97
=( × 100) − × 100
21.97 27.16
≈ 146 – 136
≈ 10%

S52. Ans.(d)
Sol.
Percentage increase in the literacy rate of male in
1961 = 48.74%
1971 = 13.76%
1981 = 22.67%
1991 = 13.74%
2001 = 17.35%
∴Required year = 1961

S53. Ans.(b)
Sol.
Percentage increase in the literacy rate of female
In 1961 = 73.25%
In 1971 = 43.12%
In 1981 = 35.45%
In 1991 = 32.02%
In 2001 = 37.36%

S54. Ans.(d)
Sol.
Since, the number of males are not specified, we can not get the required value.

S55. Ans.(a)
Sol.
56.38−40.4
Required ratio = ( × 100) ∶ 39.55
40.4
= 39.55 : 39.55
=1:1

25 Adda247 | No. 1 APP for Banking & SSC Preparation


Website:store.adda247.com | Email:ebooks@adda247.com
Directions (56-60): Study the following graph carefully and answer the questions given below.

Number of TV sets sold over the years


60 Type X Type Y

50
Number of TV sets sold

40
(in thousands)

30

20

10

0
2010 2011 2012 2013 2014 2015
Year

Q56. What was the average number of Y-type TV sets sold by the company in 2011, 2012, 2014 and
2015 together?
(a) 32869 (b) 36250 (c) 35600
(d) 39827 (e) 42686

Q57. The number of X-type TV sets sold in 2011 was exactly what percent of the number of Y-type
TV sets sold in 2015?
1 1 1
(a) 33 2 % (b) 323% (c) 372%
1 1
(d) 452% (e) 532%

Q58. What is the percentage increase in the sale of Y-type TV sets from 2011 to 2014?
2 1
(a) 25% (b) 623% (c) 533%
2
(d) 66 % (e) 49%
3

Q59. In which of the following years was the percentage increase/decrease of sale of X-type TV
sets the maximum from the previous year?
(a) 2014 (b) 2012 (c) 2015
(d) 2011 (e) 2010

Q60. In which of the following years was the difference between the sales of X-type TV sets and Y-
type TV sets the maximum?
(a) 2010 and 2012 (b) 2013 and 2014 (c) 2011 and 2012
(d) 2014 and 2013 (e) 2012 and 2013
26 Adda247 | No. 1 APP for Banking & SSC Preparation
Website:store.adda247.com | Email:ebooks@adda247.com
S56. Ans.(b)
(30 + 25 + 50 + 40)
Sol. Required average = × 1000
4
= 36.25 × 1000 = 36250

S57. Ans.(c)
15 1
Sol. Required % = 40 × 100 = 372%

S58. Ans.(d)
50 − 30
Sol. Required % increase = × 100
30
200 2
= % = 66 %
3 3

S59. Ans.(b)
15 − 20
Sol. In year 2011 = × 100
20
5
= − 20 × 100 = −25%
40 − 15 2
In year 2012 = × 100 = 166 3 %
15
45 – 40
In year 2013 = × 100 = 12.5%
40
37.5 − 45
In year 2014 = × 100 = −16.67%
45
30 − 37.5
In year 2015 = × 100 = −20%
37.5

S60. Ans.(c)
Sol. From the graph the maximum difference is in the year 2011 and 2012.

Directions (61-65): Study the following pie-charts carefully and answer the questions given below
them.

The entire fund that an organization gets from different sources is equal to Rs. 16 crore.
Internal
Source, 8%

NGOs,
Government 12%
Agencies,
38%
Ministry of
Home
Affairs, 42%

Sources of funds in the organisation

27 Adda247 | No. 1 APP for Banking & SSC Preparation


Website:store.adda247.com | Email:ebooks@adda247.com
Scholarship,
16%
Payment,
35%
Building
Maintenance,
25%
Reserved,
24%

Use of fund by the organisation

Q61. What is the difference between the fund acquired by the organization from NGOs and that
from Government Agencies?
(a) Rs. 43268000 (b) Rs. 38650000 (c) Rs. 46800000
(d) Rs. 52860000 (e) None of the above

Q62. If the organization managed Building Maintenance from the Ministry of Home Affairs fund
only, how much fund from the Ministry of Home Affairs would still be left for other use?
(a) Rs. 2.72 crore (b) Rs. 7.23 crore (c) Rs. 5.20 crore
(d) Rs. 3.06 crore (e) Rs. 8.03 crore

Q63. If the Scholarship has to be paid out of the fund from Government Agencies, find what is the
approximate percentage of Government Agencies fund used for this purpose.
(a) 42.11% (b) 38.6% (c) 31.23%
(d) 48.3% (e) 52%

Q64. What is the total amount used by the organization for Payment?
(a) Rs. 4.8 crore (b) Rs. 6.3 crore (c) Rs. 5.6 crore
(d) Rs. 9.73 crore (e) None of the above

Q65. What is the amount of fund acquired by the organization from Ministry of Home Affairs?
(a) 6.25 crores (b) 6.2 crores (c) 6.72 crores
(d) 9.25 crores (e) None of the above

S61. Ans.(e)
Sol. Required fund = (38 – 12)% of 160000000
= Rs. 41600000
28 Adda247 | No. 1 APP for Banking & SSC Preparation
Website:store.adda247.com | Email:ebooks@adda247.com
S62. Ans.(a)
Sol. Required remaining amount
= 42% of 16 cr – 25% of 16 cr
= 17% of 16 cr
= 2.72 crore

S63. Ans.(a)
16
Sol. Required % = 38 × 100 = 42.11%

S64. Ans.(c)
Sol. Required amount = 35% of 16 crore
35 × 16
= = Rs. 5.6 crore
100

S65. Ans.(c)
Sol. Fund acquired = 42% of 16 crore
42 × 16
= = Rs. 6.72 crore
100

Directions (66-70): Study the following table carefully and answer the questions given below:

Number of Cars (in thousands) of different Models and colours sold in two Metro Cities in a year
Metro M Metro H
Type
Colour Colour
Black Red Blue White Silver Black Red Blue White Silver
A 40 25 55 75 15 45 32 40 60 20
B 20 35 60 80 20 30 37 39 81 35
C 35 30 50 90 35 40 42 41 6 37
D 45 40 45 85 40 35 39 37 90 42
E 50 35 35 60 30 50 44 43 77 22
F 55 42 40 65 52 47 34 45 87 17

Q66.The difference between the white-coloured cars sold in the two metros of which of the
following models is the minimum?
(a) A (b) C (c) D
(d) F (e) None of these

Q67. The total number of blue-coloured cars of Model E and D sold in metro H is exactly equal to
the number of white-coloured cars of which model in Metro M?
(a) B (b) F (c) C
(d) A (e) None of these
29 Adda247 | No. 1 APP for Banking & SSC Preparation
Website:store.adda247.com | Email:ebooks@adda247.com
Q68. What is the difference between the number of blue-colours cars of model ‘C’ sold in Metro M
and number of red-colour cars of Model ‘F’ sold in Metro H?
(a) 8,000 (b) 10,000 (c) 12,000
(d) 15,000 (e) None of these

Q69. The total number of silver-coloured cars sold in Metro H is approximately what percentage of
that in Metro M?
(a) 130 (b) 140 (c) 90
(d) 100 (e) 110

Q70. In metro M the number of cars sold was maximum for which of thecolour-model
combinations?
(a) White-C (b) Blue-B (c) Silver-B
(d) White-D (e) Silver-F

S66. Ans.(e)
Sol.
A ⇒ 75 – 60 = 15 B ⇒ 81 – 80 = 1 C ⇒ 90 – 6 = 84
D ⇒ 90 – 85 = 5 E ⇒ 77 – 60 = 17 F ⇒ 87 – 65 = 22
Required model = B

S67. Ans.(a)
Sol. Blue coloured car of
Model E and D sold in Metro H = 43 + 37 = 80,000
Which is equal to white coloured car of B model in metro M

S68. Ans.(e)
Sol. Required difference = 50 – 34 = 16000

S69. Ans.(c)
173
Sol. Required % = 192 × 100 ≈ 90%.

S70. Ans.(a)
Sol.
White C – 90
Blue B – 60
Silver B – 20
White D – 85
Silver F – 52

30 Adda247 | No. 1 APP for Banking & SSC Preparation


Website:store.adda247.com | Email:ebooks@adda247.com
Direction (71–75): Study the following table carefully to answer the questions that follow:

Number of Orders cancelled by five different e-commerce companies in six different years
e-Com P Q R S T
Years
2011 240 405 305 365 640
2012 420 600 470 446 258
2013 600 680 546 430 610
2014 160 208 708 550 586
2015 140 640 656 250 654
2016 290 363 880 195 483

Q71. What was the difference between the highest number of Order cancelled by Company -
Q and the lowest number of Order cancelled by Company -T out of all the six years?
(a) 325 (b) 422 (c) 596
(d) 416 (e) None of these

Q72.What was the approximate percentage increase in number of Order cancelled by


Company-S in the year 2014 as compared to previous year?
(a) 57 (b) 44 (c) 125
(d) 28 (e) 95

Q73. What was the average number of Order cancelled by the Companies P,R, S and T in the
year 2014?
(a) 405 (b) 551.5 (c) 501
(d) 488 (e) None of these

Q74.In 2016, 40% Order are cancelled by Company-R due to bad weather and others by
packaging fault. How many orders are cancelled by Company-R due to packaging fault?
(a) 548 (b) 468 (c) 568
(d) 528 (e) None of these

Q75.What is the approximate percentage of cancelled Order by Companies P and R in 2013


as compared to cancelled orders by Company-S in 2011?
(a) 340 (b) 314 (c) 280
(d) 265 (e) 384

S71. Ans.(b)
Sol.Highest number of Order cancelled by Company-Q = 680
Lowest number of Order cancelled by Company-T = 258
Required difference = 680 – 258 = 422
31 Adda247 | No. 1 APP for Banking & SSC Preparation
Website:store.adda247.com | Email:ebooks@adda247.com
S72. Ans.(d)
Sol. Number of Order cancelled by Company-S in the year 2013 = 430
Number of Order cancelled by Company-S in the year 2014 = 550
550−430
Required percentage = × 100= 28 (approx)
430

S73. Ans.(c)
Sol.Required average = (160 + 708 + 550 + 586) ÷ 4
= 2004 ÷ 4 = 501

S74.Ans.(d)
Sol.Total number of Order are cancelled by Company R in 2016 = 880
Order are cancelled by Company-R due to packaging fault = 60%
Required number = 60% of 880
= 528

S75.Ans.(b)
Sol.Cancelled Order by Company’s P and R in 2013 = 600 + 546 = 1146
Cancelled Order by Company-S in 2011 = 365
600+546
Required percentage = × 100 = 314(approx.)
365

Directions (76-80): Study the bar graph carefully and answer the following questions.

The number of male and female probationary officers in various banks

Male Female
10
9
8
7
6
5
4
3
2
1
0
SBI BOI HDFC PNB ICIC
In thousand

32 Adda247 | No. 1 APP for Banking & SSC Preparation


Website:store.adda247.com | Email:ebooks@adda247.com
Q76.What is the total number of employees in the given six banks?
(a) 60000 (b) 56000 (c) 58000
(d) 62000 (e) 59000

Q77.What is the ratio of male to female probationary officers in all six banks?
(a) 5 : 4 (b) 3 : 2 (c) 2 : 3
(d) 7 : 8 (e) 4 : 5

Q78.In HDFC 40% males and 30% females are unmarried, then what is the ratio of the married
males to the married females in HDFC?
(a) 7 : 5 (b) 5 : 7 (c) 12 : 13
(d) 2 : 3 (e) 3 : 5

Q79.If the number of married male probaionary officers in ICICI is euqal to that in PNB, which is
40% of the male probabtionary officers in PNB, then what is the percentage of married male
probationary officers in ICIC with respect to the total number of probationary officers in ICIC?
(a) 25.51% (b) 28% (c) 27.91%
(d) 22% (e) 23.33%

Q80.The male probationary officers in PNB is what per cent more than the female probationary
officers in BOI?
(a) 74.8% (b) 74% (c) 75%
(d) 75.4% (e) 78%

S76. Ans.(a)
Sol. Total employees of the given six banks
= (8 + 9 + 3 + 4 + 6 + 5 + 6 + 7 + 5 + 7) × 1000 = 60000

S77. Ans.(d)
Sol. Ratio of male to female probationary officers in all six banks
= (8000 + 3000 + 5000 + 7000 + 5000) : (9000 + 4000 + 6000 + 6000 + 7000)
= 28000 : 32000 = 7 : 8

S78.Ans (b)
40
Sol. Unmarried males in HDFC = 5000 × 100 = 2000
∴ Married males = (5000 – 2000) = 3000
30
Unmarried females in HDFC = 6000 × 100 = 1800
∴ Married females = (6000 – 1800) = 4200
∴ Required ratio = 3000 : 4200 = 5 : 7

33 Adda247 | No. 1 APP for Banking & SSC Preparation


Website:store.adda247.com | Email:ebooks@adda247.com
S79. Ans.(e)
Sol. Number of male married probationary officers in ICICI
= Number of male married probationary officers in PNB
40 40
= Male probationary officers in PNB × 100 = 7000 × 100 = 2800
∴The percentage of married male probationary officers in ICICI w.r.t to the total probationary officers
in ICICI
2800 × 100
= = 23.33%
7000 + 5000

S80. Ans.(c)
7000 − 4000
Sol. Required% = × 100%
4000
= 75% more than female probationary officers in BOI

Directions (81-85): The table given below shows the monthly salary of six employees working in a
leading manufacturing firm.

Years→ 2011 2012 2013 2014 2015 2016


Employees↓
Richali 19200 20500 23400 25000 26600 28200
Piyush 28500 30100 31800 33000 34900 36000
Ritesh 22600 24000 26400 28100 29800 31000
Aditi 23000 24500 26100 27000 29300 31200
Krishna 24800 26000 27900 29100 30800 33000
Raksha 31500 35800 36600 40200 44000 45800

Q81. What is the difference between average monthly income of Aditi all over the years and
monthly income of Raksha in 2015?
(a) Rs. 17250 (b) Rs. 18150 (c) Rs. 17510
(d) Rs. 17150 (e) None of these

Q82. Monthly salary of Ritesh in 2016 contributes for what percent in total monthly salary of
Richali, Piyush and Krishna together in 2016? (approximately)
(a) 30% (b) 32% (c) 38%
(d) 42% (e) 28%

Q83. Find the ratio of annual salary of Aditi in 2012 and Raksha in 2014 together to that of Piyush
in 2013 and Richali in 2011 together?
(a) 6 :7 (b) 7 : 6 (c) 5 : 4
(d) 3 : 2 (e) None of these

34 Adda247 | No. 1 APP for Banking & SSC Preparation


Website:store.adda247.com | Email:ebooks@adda247.com
Q84. Monthly salary of Piyush and Krishna together in 2013 is by what percent more or less than
that of Aditi and Raksha together in 2015? (approximately)
(a) 19% more (b) 16% less (c) 19% less
(d) 16% more (e) 29% less

Q85. In 2015, Raksha donated 5% of her monthly salary, she then lent out 20% of remaining salary
on CI at 5% for 3 years. Find the interest (approx.) earned by her after 3 years?
(a) Rs. 1381 (b) Rs. 1318 (c) Rs. 1418
(d) Rs. 1315 (e) Rs. 1300

S81. Ans.(d)
1
Sol. Average monthly income of Aditi = 6 × 161100 = 26850 Rs.
∴Required difference = 44000 – 26850 = Rs. 17150

S82. Ans.(b)
31000
Sol. Required percentage = 97200 × 100 ≈ 32%

S83. Ans.(e)
(24500+40200)×12 647
Sol. Required ratio = (31800+19200)×12 = 510

S84. Ans.(c)
Sol. Monthly salary of Piyush and Krishan = 31800 + 27900 = 59700
Monthly salary of Adity and Raksha = 29300 + 44000 = 73300
13600
∴Required percentage = 73300 × 100 ≈ 19% less.

S85. Ans.(b)
1 95
Sol. 20% of amount left after donation = 5 × 100 × 44000
= Rs. 8360
5 3
∴ C.I. after 3 years = 8360 [(1 + 100) − 1]
= 8360 × 0.1576
≈ Rs. 1318

Directions (86-90): A team of 5 players participated in a tournament and played four matches (1 to
4). The following table gives partial information about their individual scores and the total runs
scored by the team in each match.

Each column has two values missing. These are the runs scored by the two lowest scorers
in that match. None of the two missing values is more than 10% of the total runs scored in
that match.
35 Adda247 | No. 1 APP for Banking & SSC Preparation
Website:store.adda247.com | Email:ebooks@adda247.com
Match-1 Match-2 Match-3 Match-4
Ajinkya 100 53
Pandya 88 65 52
Runs scored
Cheteswar 100
by player
Dhawan 72 75 20 56
Virat 60 78
Total 270 300 240 200

Q86. What is the maximum possible percentage contribution of Ajinkya in the total runs
scored in the four matches (approximately)?
(a) 20% (b) 22% (c) 17%
(d) 23% (e) Cannot be determined

Q87.What is the maximum possible percentage contribution of Viratin the total


runsscored in the four matches?
(a) 18% (b) 19.9% (c) 18.6%
(d) 20.2% (e) Cannot be determined

Q88. If the absolute difference between the total runs scored by Ajinkya and Cheteshwar in the
Four matches is minimum possible then what is the ratio of Ajinkya and Cheteshwar’s total runs
scored by them in the four matches.
(a) 187:189 (b) 189:187 (c) 183:187
(d) 189:188 (e) Cannot be determined

Q89.If the absolute difference between the total runs scored by Ajinkya and Cheteshwar in the
four matches is minimum possible then what is the absolute difference between total runs scored
by Pandya and Virat in the four matches?
(a) 32 (b) 37 (c)35
(d) 27 (e) Cannot be determined

Q90. The players are ranked 1 to 5 on the basis of the total runs scored by them in the four
matches, with the highest scorer getting Rank 1. If it is known that no two players scored the same
number of total runs, how many players are there whose ranks can be exactly determined?
(a) 0 (b) 1 (c) 3
(d) 5 (e) Cannot be determined

36 Adda247 | No. 1 APP for Banking & SSC Preparation


Website:store.adda247.com | Email:ebooks@adda247.com
S86. Ans. (a)
Sol.
Maximum possible runs scored by Ajinkya in Match-1 = 27
Maximum possible runs scored by Ajinkya in Match-3 = 19 (less than 20)
Maximum possible percentage contribution:
27 + 100 + 19 + 53 199
× 100% = × 100% = 19.7%
270 + 300 + 240 + 200 1010
= 20% approx.

S87. Ans. (c)


Sol.
Maximum possible runs scored by Virat in Match-2 = 30
Maximum possible runs scored by Virat in Match-4 = 20
Maximum possible percentage contribution:
60 + 30 + 78 + 20
× 100%
270 + 300 + 240 + 200
188
= × 100% = 18.6%
1010

S88. Ans. (b)


Sol.
Maximum possible total runs scored by Cheteshwar in the four matches = 27 + 30 + 110 + 20 = 187.
Total runs scored by Ajinkya in the four matches is in the range of 189 to 199
Hence,
In such a case minimum possible
Total runs scored by Ajinkya in the four matches = 23 + 100 + 13 + 53 = 89
Difference = 189 – 187 = 2 (minimum possible)
So Required ratio is 189: 187

S89. Ans. (b)


Sol.
Maximum possible total runs scored by Cheteshwar in
the four matches= 27 + 30 + 110 + 20 = 187.
In such a case minimum possible total runs scored by
Ajinkya in the four matches
= 23 + 100 + 13 + 53 = 189.
Difference = 189 – 187 = 2 (minimum possible)
Subsequently total runs scored by Pandya in the four
matches = 88 + 65 + 19 + 52 = 224.
Also, total runs scored by Virat in the four matches
= 60 + 30 + 78 + 19 = 187
Absolute difference = 224 – 187 = 37

37 Adda247 | No. 1 APP for Banking & SSC Preparation


Website:store.adda247.com | Email:ebooks@adda247.com
S90. Ans.(c)
Sol.
Individual ranges for total score:
Ajinkya-> 189-199
Pandya-> 218-224
Cheteshwar-> 182-187
Dhawan-> 223
Virat-> 187-188
Least total will be of Cheteshwar (Rank 5)
2nd least will be Virat (Rank 4)
Rank 3 must be of Ajinkya
It is not possible to determine the exact ranks of Pandya and Dhawan

Directions (91-95): The table below shows production of five types of Trucks by a company in the
years 2009 to 2014. Study the table and answer questions.

Production of trucks by a company

Year →
Type ↓ 2009 2010 2011 2012 2013 2014 Total
Minivan 8 20 16 17 21 6 88
Pickup 16 10 14 12 12 14 18
Canopy 21 17 16 15 13 8 90
Panel 4 6 10 16 20 31 87
Cab 25 18 19 30 14 27 133
Total 74 71 75 90 80 86 476

Q91. In which year the production of trucks of all types taken together was approximately equal to
the average of the total production during the period?
(a) 2009 (b) 2011 (c) 2013
(d) 2014 (e) None of these

Q92. In which year, the total production of trucks of types of Minivan and Pickup together was
equal to the total production of trucks of types Canopy and Panel together.
(a) 2010 (b) 2011 (c) 2014
(d) 2013 (e) None of these

Q93. During the period 2009-14, in which type of trucks was a continuous increase in production?
(a) Minivan (b) Pickup (c) Canopy
(d) Panel (e) None of these

38 Adda247 | No. 1 APP for Banking & SSC Preparation


Website:store.adda247.com | Email:ebooks@adda247.com
Q94. The production of which type of trucks was 25% of the total production of all types of trucks
during 2013?
(a)Panel (b) Canopy (c) Pickup
(d)Minivan (e) None of these

Q95. The per cent increase in total production of all types of trucks in 2012 to that in 2011 was?
(a) 15 (b) 20 (c) 25
(d) 30 (e) None of these

S91. Ans.(c)
476
Sol. Average of the total production during the period = 6 ≈ 80 which is equal to the total
production in 2013.

S92. Ans.(d)
Sol. Answer will be 2013.

S93. Ans.(d)
Sol. Answer is Panel

S94. Ans.(a)
Sol. 25% of 80 = 20 = production of Panel’s car in 2013.

S95. Ans.(b)
90−75
Sol. Required percent increase = × 100 = 20%
75

Directions (96-100): Study the following graph to answer the given questions.

Percent profit earned by two companies over the given years


Income − Expenditure
% profit = × 100
Expenditure

Company M Company N

70
60

45
40 40
30 30 30 30
25 20
10

2005-06 2006-07 2007-08 2008-09 2009-10 2010-11

39 Adda247 | No. 1 APP for Banking & SSC Preparation


Website:store.adda247.com | Email:ebooks@adda247.com
Q96. For Company M, its income in 2009-10 was equal to its expenditure in 2010-11, what was the
ratio of its respective incomes in these two years?
(a) 4:5 (b) 3:4 (c) 5:7
(d) Cannot be determined (e) None of these

Q97. If the income of Company M in 2006-07was equal to the expenditure of Company N in 2009-
10 what was the ratio of their respective profits?
(a) 13:15 (b) 15:26 (c) 13:26
(d) Cannot be determined (e) None of these

Q98. What was the difference in the expenditures of the two companies in 2007-08?
(a) 10 (b) 100 (c) 1000
(d) Cannot be determined (e) None of these

Q99. In 2010-11 the income of Company N was Rs. 119 crores. What was its expenditure in that
year?
(a) Rs. 76.8 crore (b) Rs. 64 crore (c) Rs. 70 crore
(d) Cannot be determined (e) None of these

Q100. For Company N, in which year is the percent of increase in percent profit over that of
previous year the highest?
(a) 2011-12 (b) 2007-08 (c) 2010-11
(d) Cannot be determined (e) None of these

S96. Ans.(c)
Sol.
𝐼𝑀 2010−11
𝐼𝑀 2009 − 10 = 𝐸𝑀 2010 − 11 =
1.4
10
𝐼𝑀 2009−10 : 𝐼𝑀 2010−11 = 14 = 5 ∶ 7.

S97. Ans.(e)
Sol. Suppose in the year 2006-07 expenditure of Company M = Rs. a
Then profit earned by Company M in this year = Rs. (10% of a)
Hence, income of Company M = Rs. (110% of a)
𝑎 × 110
Again, expenditure of Company N in 2009-10 = Rs. 100
Hence, profit earned by Company N in 2009-10
𝑎 × 110 60
= 𝑅𝑠. ×
100 100
Thus, required ratio
10
100
×𝑎 10
= 𝑎×110 60 = = 5 ∶ 33.
× 100 66
100

40 Adda247 | No. 1 APP for Banking & SSC Preparation


Website:store.adda247.com | Email:ebooks@adda247.com
S98. Ans.(d)
Sol. The given graph depicts only the percent profit earned by the two companies over the given
years. Hence, these information are insufficient to answer the question.

S99. Ans.(c)
Sol. In 2010-11, profit earned by Company N was 70%
Therefore, 170% of expenditure Rs. 119 crore
119
Thus, required expenditure = 170 × 100 = Rs. 70 Crores

S100. Ans.(e)
Sol. Percent of increase in percent profit over that of the previous year for the given years is as
follows:
Year
(30−25)
2006-07: × 100 = 20%
25
(40−30)
2007-08: × 100 = 33.33%
30
(20−40)
2008-09: = × 100 = -50%
40
(60−20)
2009-10: × 100 = 200%
20
(70−60)
2010-11: × 100 = 16.66%
60

Directions (101-105): Study the graphs carefully to answer the questions that follow.

Total number of children in 6 different schools and the percentage of girls in them

3500

3000
Number of Childerns

2500

2000

1500

1000

500

0
P Q R S T U
Schools

41 Adda247 | No. 1 APP for Banking & SSC Preparation


Website:store.adda247.com | Email:ebooks@adda247.com
50
45
40
Percentage of Girls 35
30
25
20
15
10
5
0
P Q R S T U
Schools

Q101. What is the total percentage of boys in schools R and U together? (rounded off to two digits
after decimal)
(a) 78.55 (b) 72.45 (c) 76.28
(d) 75.83 (e) None of these

Q102. What is the total number of boys in school T?


(a) 500 (b) 600 (c) 750
(d) 850 (e) None of these

Q103. The total number of students in school R, is approximately what per cent of the total
number of students in school S?
(a) 89 (b) 75 (c) 78
(d) 82 (e) 94

Q104. What is the average number of boys in schools P and Q together?


(a) 1425 (b) 1575 (c) 1450
(d) 1625 (e) None of these

Q105. What is the respective ratio of the number of girls in schools P to the number of girls in
school Q? (a) 27 : 20 (b) 17 : 21
(c) 20 : 27 (d) 21 : 17
(e) None of these

S101. Ans.(d)
Sol. Number of boys in school R and U together
2000 × 72.5 1000 × 82.5
= +
100 100
= 1450 + 825 = 2275
2275
∴Required percentage = 3000 × 100 = 75.83%

42 Adda247 | No. 1 APP for Banking & SSC Preparation


Website:store.adda247.com | Email:ebooks@adda247.com
S102. Ans.(c)
1250 × 60
Sol. Number of boys in school T = = 750
100

S103. Ans.(a)
Sol. Total number of students in school R = 2000
Total number of students in school S = 2250
2000
∴Required percentage = 2250 × 100 ≈ 89

S104. Ans.(b)
1 2500 × 60 3000 × 55
Sol. Required average = 2 ( 100 + )
100
1 1
= (1500 + 1650) = × 3150 = 1575
2 2

S105. Ans.(c)
2500 × 40 3000 × 45
Sol. Required ratio = 100 ∶ 100
= 25 ×40 : 30 × 45
= 100 : 135 = 20 : 27

Directions (106-110): Study the graph and answer the following questions.

Power Supply Position in UP (in billion KWH)


Requirement Supply
200

175 170

150 145
137
129 151
118 120
125 109 130
102 124
100 115
84 88 105
78 97 99
75 86
75 83
50 66

25

0
2003-04 2004-05 2005-06 2006-07 2007-08 2008-09 2009-10 2010-11 2011-12 2012-13 2013-14

Q106. What was the approximate percentage increase in supply of power between 2009-10 and
2013-14?
(a) 56% (b) 145% (c) 43%
(d) 85% (e) None of these

43 Adda247 | No. 1 APP for Banking & SSC Preparation


Website:store.adda247.com | Email:ebooks@adda247.com
Q107. The cumulative shortfall between requirement and supply from 2009 to the end of 2014 was
(in billion)
(a) 56 (b) 85 (c) 45
(d) 76 (e) None of these

Q108. The requirement of power in 2013-14 was approximately how many times the availability of
supply in 2007-08?
(a) 2.6 (b) 1.75 (c) 2.75
(d) 2.0 (e) None of these

Q109. The percentage of growth in power requirement from 2008-09 to 2013-14 was less than the
percentage of growth in power requirement from 2003-04 to 2008-09 by what figure?
(a) 3 (b) 4 (c) 15
(d) 7 (e) None of these

Q110. Between 2008-09 and 2012-13, the power generation has generally logged behind power
demand by how many years?
(a) 1 (b) 2 (c) 3
(d) 4 (e) None of these

S106. Ans.(c)
Sol. In 2009-10 is 105 while in 2013-14 is 151.
151−105 46
So percentage increases is × 100 = × 100 = 43%
105 105

S107. Ans.(d)
Sol. Total requirement = 120 + 129 + 137 + 145 + 170 = 701
Total supply = 105 + 115 + 124 + 130 + 151 = 625
Difference = 701 - 625 = 76

S108. Ans.(b)
Sol. 170 = 97 × x
170
So, 𝑥 = = 1.75
97

S109. Ans.(d)
170−118
Sol. In 2008-09 to 2013-14, % Growth = 118 × 100 ≈ 44%
118−78 40
In 2003-04 to 2008-09 growth= = 78 × 100 ≈ 51%
78
So, more ≈ 51 - 44 = 7%

S110. Ans.(c)
Sol. In 2008-09 demand was 118 which completed in 2012-
2013 means 3 years.

44 Adda247 | No. 1 APP for Banking & SSC Preparation


Website:store.adda247.com | Email:ebooks@adda247.com
Directions (111-115): Dominos prepares Pizzas of three different types – Cheese, Onion and
Chicken.
The production of the three types over a period of six Months has been expressed in the bar-graph
provided below. Study the graph and answer the questions based on it.
Order of three different types of Dominos Pizzas over the Months (in lakh orders)

Cheese Onion Chicken


60 60 60 60
60 55 55 55 55
50 50 50 50 50
50 45 45 45
40 40
40

30

20

10

0
January February March April May June

Q111. For which of the following Months the percentage of rise/fall in Order from the previous
Month is the maximum for the Onion flavor?
(a) February (b) March (c) April
(d) May (e) June

Q112. For which type was the average annual Order maximum in the given period?
(a) Cheese only (b) Onion only (c) Chicken only
(d) Cheese and Onion (e) Cheese and Chicken

Q113. The total Order of Chicken type in March and April is what percentage of the total Order of
Cheese type in January and February?
(a) 96.67% (b) 102.25% (c) 115.57%
(d) 120% (e) 133.33%

Q114. What is the difference between the average Order of Cheese type in January, February and
March and the average Order of Onion type in April, May and June?
(a) 50,000 orders (b) 80,000 orders (c) 2,40,000 orders
(d) 3,30,000 orders (e) 5,00,000orders
45 Adda247 | No. 1 APP for Banking & SSC Preparation
Website:store.adda247.com | Email:ebooks@adda247.com
Q115. What was the approximate decline in the Order of Chicken type in June as compared to the
Order in April?
(a) 50% (b) 42% (c) 33%
(d) 25% (e) 22.5%

S111. Ans.(b)
Sol. The percentage rise/fall in Order from the previous Month for Onion type during various
Months are:
(60−55)
In February = [ × 100] % = 9.09% (increase)
55
(60−50)
In March = [ × 100] % = 16.67% (decrease)
60
(55−50)
In April = [ × 100] % = 10% (increase)
55
(55−50)
In May = [ × 100] % = 9.09% (decrease)
55
(55−50)
In June = [ × 100] % = 10% (increase)
50
∴Maximum change is decrease of 16.67% during March.

S112. Ans.(b)
Sol. Average annual Orders over the given period for various types are:
1
For Cheese type = [6 × (50 + 40 + 55 + 45 + 60 + 50)] lakh orders = 50 lakh orders.
1
For Onion type = [6 × (55 + 60 + 50 + 55 + 50 + 55)] lakh orders
= 54.17 lakh orders.
1
For Chicken type = [6 × (45 + 50 + 60 + 60 + 45 + 40)] lakh orders = 50 lakh orders.
∴ Maximum average Order is for Onion type.

S113. Ans.(e)
(60 + 60) 120
Sol. Required percentage = [(50 + 40) × 100] % = ( 90 × 100) % = 133.33%.

S114. Ans.(e)
1
Sol. Average Order of Cheese type in January, February and March = [3 × (50 + 40 + 55)]
145
=( ) lakh orders.
3
1
Average Order of Onion type in April, May and June = [3 × (55 + 50 + 55)]
160
=( ) lakh orders.
3
160 145 15
∴ Difference = ( − )= = 5 lakh orders = 5,00,000 orders.
3 3 3

46 Adda247 | No. 1 APP for Banking & SSC Preparation


Website:store.adda247.com | Email:ebooks@adda247.com
S115. Ans.(c)
Sol. Percentage decline in the Order of type Chicken in June as compared to the Order in April =
(60 − 40) 20
[ × 100] % = (60 × 100) % = 33.33% ≈ 33%.
60

Directions (116-120): Study the following Graph carefully and answer the questions given below:

Preferences of People in Playing Different Games Over the Years (in Hundred)
500
450
400
350
300
Cricket
250
Football
200
Tennis
150
100
50
0
2011 2012 2013 2014 2015 2016

Q116. In the year 2016, the people preferring to play Tennis is what percent of the people prefer to
play Cricket, Football and Tennis together in that year?
(a) 22.76% (b) 20.58% (c) 42. 24%
(d) 25% (e) None of these

Q117. How many people have preferred to play Cricket in all the years together?
(a) 217500 (b) 224500 (c) 247500
(d) 175600 (e) None of these

Q118. What is the respective ratio of the number of people prefer to play cricket in 2011, 2013 and
2015 to the number of people prefer to play Tennis in the year 2013, 2015 and 2016?
(a) 2 : 1 (b) 45 : 33 (c) 44 : 31
(d) 48 : 31 (e) None of these

Q119. From 2011 to 2016, the total number of people who preferred to play Football was what
percent more or less than the total number of people who preferred to play Tennis during same
period?
(a) 5.24% (b) 6.24% (c) 7.24%
(d) 8.24% (e) 10%

47 Adda247 | No. 1 APP for Banking & SSC Preparation


Website:store.adda247.com | Email:ebooks@adda247.com
Q120. The no. of people prefer to play tennis in 2016 is what percent fewer than the number of
people preferring to play tennis in 2015?
4 4 7
(a) 23 % (b) 36 % (c) 42 %
11 11 13
9
(d) 3313 % (e) None of these

S116. Ans.(b)
Sol.
175 175
Desired% = × 100 = × 100 = 20.58%
350 + 325 + 175 850

S117. Ans.(a)
Sol. Total people playing cricket over all years = 2,17,500

S118. Ans.(d)
Sol.
400 + 450 + 350 1200
Ratio = = = 48 ∶ 31
325 + 275 + 175 775

S119. Ans.(c)
Sol.
1850 − 1725 125
Desired % = × 100 = × 100 = 7.24%
1725 1725

S120. Ans.(b)
Sol.
275 − 175 400 4
Desired % = × 100 = = 36 %
275 11 11

Directions (121–125): Read the given bar graph and answer the following questions.

Number of Students taking fresh education loan from different Banks


6000

5000

4000 2008
2009
3000
2010
2000 2011
2012
1000

0
SBI PNB BOB UCO OBC

48 Adda247 | No. 1 APP for Banking & SSC Preparation


Website:store.adda247.com | Email:ebooks@adda247.com
Q121. Approximately how many students taking a loan from UCO in 2009 and PNB in 2010 were
defaulters if 23% from UCO in 2009 and 20% from PNB in 2010 have defaulted?
(a) 630 (b) 650 (c) 600
(d) 750 (e) 840

Q122. In 2007, no of defaulters in SBI was 5%. However each year no of defaulters increases by
10% in number. What will be the difference between the number of defaulters of SBI in the
Month 2009 and 2012?
(a) 1500 (b) 2000 (c) 1325
(d) 1456 (e) Cannot be determined

Q123. In which of the following years, the difference in no. of students taking loan from Bank
BOB from the previous year is highest?
(a) 2008 (b) 2009 (c) 2010
(d) 2012 (e) None of these

Q124. If on average, Rs. 175000 per students education loan sanctioned by OBC bank all over the
years. What will be total amount sanctioned by OBC in all given years?
(a) 1055600000 (b) 1055800000 (c) 1620000000
(d) 1050000000 (e) None of the above

Q125. What is the ratio of Number of students taking Education Loans from SBI and BOB together
in all the Years and the total no of students taking Education loans in 2010 and 2011 together?
(a) 8 : 5 (b) 5 : 7 (c) 7 : 5
(d) 9 : 7 (e) None of these

S121. Ans.(a)
Sol.
Students taking loan from UCO in 2009 = 1000
Defaulters (UCO) = 23% of 1000 = 230
Person taking loan from PNB in 2010 = 2000
Defaulters (PNB) = 20% of 2000 = 400
Total desired defaulters = 230 + 400 = 630

S122. Ans.(e)
Sol. Cannot be determined because no. of students
taking a loan from SBI in 2007 is unknown.

49 Adda247 | No. 1 APP for Banking & SSC Preparation


Website:store.adda247.com | Email:ebooks@adda247.com
S123. Ans.(b)
Sol. From graph, it is clear that in 2009, difference between no. of students taking a loan is highest as
compared to previous year.

S124. Ans.(e)
Sol. No. of students taking education loan from OBC bank all over the year
= 1000 + 1000 + 1500 + 2000 + 1500 = 7000
Total loan amount sanctioned over the years = 7000 × 1,75,000
= Rs. 1,22,50,00,000

S125. Ans.(c)
Sol.
SBI : 2500 + 3000 + 4500 + 4000 + 5000 = 19000
BOB : 2500 + 3500 + 4000 + 4500 + 5000 = 19500
Total no. of students taking loan in 2010 = 13500
Total no. of students taking loan in 2011 = 14000
19000+19500 38500 7
Desired ratio = 13500+14000 = 27500 = 5

Directions (126-130): Study the following Radar graph carefully and answer the questions given
below.
Number of students studying in different universities in a year (Numbers in Lac).

P
50

40 37.5
30
30
20
T Q
35 10
27.5 30 40 Males
0 Females

25 25

37.5
42.5
S R

50 Adda247 | No. 1 APP for Banking & SSC Preparation


Website:store.adda247.com | Email:ebooks@adda247.com
Q126. What is the average number of females in all the universities together?
(a) 3300000 (b) 350000 (c) 320000
(d) 3200000 (e) None of these

Q127. What is the total number of students (males and females together) in University P and R
together?
(a) 1300000 (b) 1350000 (c) 1400000
(d) 14500000 (e) None of these

Q128. What is the respective ratio of the number of females from University P and Q together to
the number of males in the Universities R and T together?
(a) 27 : 32 (b) 27 : 28 (c) 25 : 28
(d) 28 : 27 (e) None of these

Q129. The number of males in University Q are what per cent of the total number of students
(males and females together) in University S?
(a) 68 (b) 62 (c) 66
(d) 64 (e) None of these

Q130. If the total number of males in University T increases by 50%, what would be the total
number of students (males and females together) in that university?
(a) 7526000 (b) 76250000 (c) 7625000
(d) 75260000 (e) None of these

S126.Ans.(a)
Sol.
37.5+30+25+37.5+35 165
Required No. = = lakhs = 3300000
5 5

S127. Ans.(e)
Sol. Required No.(30 + 37.5 + 42.5 + 25)
=135 lakhs = 13500000

S128. Ans.(b)
Sol. Required Ratio = (37.5 + 30) : (42.5 + 27.5)
= 27 : 28

S129. Ans.(d)
40
Sol. Required % = 25 + 37.5 × 100
= 64%

51 Adda247 | No. 1 APP for Banking & SSC Preparation


Website:store.adda247.com | Email:ebooks@adda247.com
S130. Ans.(c)
150
Sol. Required no. = (27.5 × 100) + 35
= 76.25 lakhs
= 7,62,5000

Directions (131-135): Study the graph carefully to answer the questions that follow.

PERCENT INCREASE IN PROFIT OF


THREE COMPANIES OVER THE YEARS
2011
60

50

40

2016 30 2012

20

10
COMPANY X
0 COMPANY Y
COMPANY Z

2015 2013

2014

Q131. If profit for company Y in 2012 is 2000 and expenditure in 2013 for company Y is 50,000, then
what is the total revenue in 2013 for Y? Give that total revenue = expenditure + profit.
(a) 52600 (b) 54200 (c) 53280
(d) 55800 (e) None of these

Q132. If profit in year 2015 for company Z is 3000 and profit of company X in 2013 is equal to profit
of company Z in 2014 then what is the profit of company X in 2013
(a) 1500 (b) 4000 (c) 3500
(d) 2000 (e) 2500

52 Adda247 | No. 1 APP for Banking & SSC Preparation


Website:store.adda247.com | Email:ebooks@adda247.com
Q133. What is the average percentage increase in profit for company Y over all the years.
(a) 49% (b) 32% (c) 23%
(d) 38% (e) 35%

Q134. What was the approximate percent increase in percent increase of profit of company X in the
year 2014 from its previous year
(a) 60% (b) 65% (c) 55%
(d) 50% (e) 70%

Q135. If profit earned by company Y in 2014 is 27,000 and by company Z in 2014 is 43500 then what
is the total profit earned by them in year 2013?
(a) 25,000 (b) 35,000 (c) 40,000
(d) 50,000 (e) None of these

S131. Ans.(a)
130
Sol. Profit in 2013 = 2000 × 100
= 2600
Total revenue = 50,000 + 2600
= 52600

S132. Ans.(d)
3000 × 100
Sol. Profit of company X in 2013 = 150
= 2000

S133. Ans.(e)
45 + 25 + 30 + 35 + 35 + 40
Sol. Required average = 6
210
= 6
= 35%

S134. Ans.(a)
40 − 25
Sol. Required percentage = 25 × 100
15
= 25 × 100
= 60%

S135. Ans.(d)
27000 × 100
Sol. Profit earned by Y in 2013 = 135
= 20,000
43500 × 100
Profit earned by Z in 2013 = 145
= 30,000
Total profit = 50,000

53 Adda247 | No. 1 APP for Banking & SSC Preparation


Website:store.adda247.com | Email:ebooks@adda247.com
Directions (136-140): Study the following graph carefully and answer the following question.

The graph below represents the production (in tonnes) and sales (in tonnes) of a company X from
2010-2015
2010
800
Production Sales
700
600
500
2015 400 2011
300
200
100
0

2014 2012

2013

Q136. If production of company X and another company Y is in the ratio 14 : 13 in year 2014 then
production of company Y in 2014 is what percent more or less than production of company X in
2010.
1 1 2
(a) 13 3 % (b) 33 3 % (c) 66 3 %
2
(d) 16 3 % (e) None of these

Q137. If production of company X in 2016 is 120% of its production in 2015 then what is the ratio of
sales company X in 2010 to the production of company X in 2016.
7 13 20
(a) (b) (c)
9 20 13
5 7
(d) 13 (e) 13

Q138. If production cost is Rs. 1,500 per tonne and sale is at the rate of Rs. 2,800 per tonne over all
years then what is the ratio of profit or loss of company X in 2013 to the profit or loss in year 2014.
(Profit = Income through sales – Production cost)
59 20 53
(a) 70 (b) 23 (c) 94
27
(d) 38 (e) None of these

54 Adda247 | No. 1 APP for Banking & SSC Preparation


Website:store.adda247.com | Email:ebooks@adda247.com
Q139. If production cost in year 2013 is 150 per tonne and production cost increases by 10% every
year after 2013 then what is the average production cost of company X over all years after year
2013?
(a) 12,20,239 (b) 1,16,737.5 (c) 2,22,467
(d) 1,33,647 (e) None of these

Q140. If 35% of production of company X in 2010 is added to the sale of company X in 2012 then
total sale of company X in 2012 is what percent of the total sale of company X over all the years
now? (approximately)
(a) 14% (b) 18% (c) 35%
(d) 28% (e) 24%

S136. Ans.(a)
700
Sol. Production of company Y in 2014 = × 13 = 650
14
100
Required percentage = 750 × 100
40
= %
3
1
13 3 % less

S137. Ans.(d)
120
Sol. Production of company X in 2016 = 100 × 650 = 780
300
Required ratio = 780
5
=
13

S138. Ans.(a)
Sol. Cost of production in 2013 = 1500 × 550
= Rs. 8,25,000
Total Income through sales = 2800 × 400
= Rs. 11, 20,000
Profit in 2013 = 11,20,000 − 8,25,000
= Rs. 2,95,000
Cost of production in 2014 = Rs. 1500 × 700
= Rs. 10,50,000
Total Income through sales = Rs. 2800 × 500
= Rs. 14,00,000
Profit in 2014 = 3,50,000
295 59
Required ratio = 350 = 70

55 Adda247 | No. 1 APP for Banking & SSC Preparation


Website:store.adda247.com | Email:ebooks@adda247.com
S139. Ans.(b)
Sol. Total production cost in 2014 and 2015 = 165 × 700 + 181.5 × 650
= 1,15,500 + 1,17,975
= 2,33,475
2,33,475
Required average = = 1,16,737.5
2

S140. Ans.(e)
35
Sol. Total sale of company X in 2012 = 450 + 100 × 750 = 712.5
712.5
Required percentage = 300 + 550 + 450 + 400 + 500 + 550 + 262.5 × 100
712.5
= × 100 = 23.65% ~ 24%
3012.5

Directions (141-145): Study the following pie-chart and table carefully and answer the questions
given below:
Percentage wise distribution of the number of mobile phones sold by a shopkeeper during six months
Total number of mobile phones sold = 45000

December, July,
16% 17%

November,
12% August,
22%
October,
8%
September,
25%

The ratio between the numbers of mobile phones sold of Company A and Company B during six
months
Month Ratio
July 8:7
August 4:5
September 3:2
October 7:5
November 7:8
December 7:9

Q141. What is the ratio of the number of mobile phones sold of Company B during July to those
sold during December of the same company?
(a) 119 : 145 (b) 116 : 135 (c) 119 : 135
(d) 119 : 130 (e) None of these

56 Adda247 | No. 1 APP for Banking & SSC Preparation


Website:store.adda247.com | Email:ebooks@adda247.com
Q142. If 35% of the mobile phones sold by Company A during November were sold at a discount,
how many mobile phones of Company A during that month were sold without a discount?
(a) 882 (b) 1635 (c) 1638
(d) 885 (e) None of these

Q143. If the shopkeeper earned a profit of Rs. 433 on each mobile phone sold of Company B
during October, what was his total profit earned on the mobile phones of that company during the
same month?
(a) Rs. 6,49,900 (b) Rs. 6,45,900 (c) Rs. 6,49,400
(d)Rs. 6,49,500 (e) None of these

Q144. The number of mobile phones sold of Company A during July is approximately what
percent of the number of mobile phones sold of Company A during December?
(a) 110 (b) 140 (c) 150
(d) 105 (e) 130

Q145. What is the total number of mobile phones sold of Company B during August and
September together?
(a) 10000 (b) 15000 (c) 10500
(d) 9500 (e) None of these

S141. Ans.(c)
Sol.
17
Total number of mobiles sold in the month of July = 45000 × = 7650
100
7
Mobile phones sold by Company B in the month of July = 7650 ×15 = 3570
16
Total numbers of mobile phones sold in the month of December = 45000 ×100 = 7200
9
Mobile phones sold by Company B in the month of December = 7200 ×16 = 4050
3570 357 119
∴ Required ratio = 4050 = 405 = 135 = 119 : 135

S142. Ans.(c)
Sol.
12
Number of mobile phones sold in the month of November = 45000 ×100 = 5400
7
Number of mobile phones sold by Company A in the month of November = 5400 ×15 = 2520
∴ Number of mobile phones sold without discount in the month of November by Company A
65
= 2520 ×100 = 2520 × 0.65 = 1638

S143. Ans. (d)


Sol.
8
Number of mobile phones sold in the month of October = 45000 ×100 = 3600
5
∴ Number of mobile phones sold by Company B in the month of October = 3600 × 12 = 1500
∴ Total profit earned by Company B in the month of October = 1500 × 433 = 649500
57 Adda247 | No. 1 APP for Banking & SSC Preparation
Website:store.adda247.com | Email:ebooks@adda247.com
S144. Ans.(e)
Sol.
17
Number of mobile phones sold in the month of July = 45000 ×100 = 7650
8
Number of mobile phones sold by Company A in the month of July = 7650 ×15 = 4080
16
Number of mobile phones sold in the month of December = 45000 ×100 = 7200
7
Number of mobile phones sold by Company A in the month of December = 7200 ×16 = 3150
4080
∴ Required % = 3150× 100 = 129.52 ≈ 130

S145. Ans.(a)
Sol.
22
Number of mobile phones sold in the month of August = 100× 45000 = 9900
25 1
Number of mobile phones sold in the month of September = 100× 45000 = 4× 45000 = 11250
5
Number of mobile phones sold by Company B in the month of August = 9900 ×9 = 5500
2
Number of mobile phones sold by Company B in September = 11250 ×5 = 4500
Total number of mobile phones sold in August and September by Company B = 5500 + 4500
= 10000

Directions (146-150): These questions based on the following graphs


Classification of appeared candidates in a competitive test from different states and qualified
candidates from those states.

Appeared candidates = 45000.

A
G 15%
22%
B
11%

C
F 8%
18% E D
9% 17%

58 Adda247 | No. 1 APP for Banking & SSC Preparation


Website:store.adda247.com | Email:ebooks@adda247.com
Qualified candidates = 9000

G A
F 13% 18%
11%
B
16%
E
14% D
21% C
7%

Q146. What is the ratio of the number of appeared candidates from states C and E together to that
of the appeared candidates from sates A and F together?
(a) 17 : 33 (b) 11 : 13 (c) 13 : 27
(d) 17 : 27 (e) None of these

Q147. In which state, the percentage of qualifies candidates with respect to that of appeared
candidates is minimum?
(a) C (b) F (c) D
(d) E (e) G

Q148. What is the difference between the number of qualified candidates of states D and those of
G?
(a) 690 (b) 670 (c) 780
(d) 720 (e) None of these

Q149. What is the percentage of qualified candidates with respect to appeared candidates from
states B and C taken together? (rounded to two decimal places)
(a) 23.11 (b) 24.21 (c) 21.24
(d) 23 (e) None of these

Q150. What is the ratio between the number of candidates qualified from states B and D together
to the number of candidates appeared from states ‘C’, respectively?
(a) 8 : 37 (b) 11 : 12 (c) 37 : 40
(d) 7 : 37 (e) None of these

S146. Ans.(a)
8+9
Sol. Required ratio = 15 + 18 = 17: 33.

S147. Ans.(e)
Sol. Here, do not find the ratio of number of qualified candidates that of the appeared. Simply check
the ratio of % qualified candidates with respect to the appeared is the least for which state. Ans. = G.

59 Adda247 | No. 1 APP for Banking & SSC Preparation


Website:store.adda247.com | Email:ebooks@adda247.com
S148. Ans.(d)
Sol. Required difference = (21 - 13)% of 9000 = 720.

S149. Ans.(b)
(16 + 7)% 𝑜𝑓 9000
Sol. Required % = (11 + 8)% 𝑜𝑓 45000 × 100 = 24.21%

S150. Ans.(c)
(16 + 21)% 𝑜𝑓 9000
Sol. Required ratio = 8% 𝑜𝑓 45000 = 37 : 40

Directions (151-155): Study the following pie-charts carefully and answer the questions given
below it.
The entire fund that school gets from different sources in equal to Rs. 500 lakh

NGO's
15%
Donation
35%

Govt. agencies
45%

Internal
sources
5%
Sources of funds in school

School
maintenance
20% Reserved
35%

Payment
30% Scholarship
15%

Uses of funds by school

Q151. What is the difference between the funds acquired by school from NGO’s and internal
sources?
(a) Rs. 50 lakh (b) Rs. 45 lakh (c) Rs. 75 lakh
(d) Rs. 25 lakh (e) None of these

60 Adda247 | No. 1 APP for Banking & SSC Preparation


Website:store.adda247.com | Email:ebooks@adda247.com
Q152. If the school managed school maintenance from the government agencies fund only, then
how much fund from government agencies would still left for other use?
(a) Rs. 120 lakh (b) Rs. 150 lakh (c) Rs. 110 lakh
(d)Rs. 95 lakh (e) None of these

Q153. If scholarship has to be paid out of the donation fund, then what is the approximate per cent
of donation fund used for his purpose?
(a) 43% (b) 53% (c) 37%
(d) 45% (e) 32%

Q154. What is the total amount used by the school for payment?
(a) Rs. 100 lakh (b) Rs. 110 lakh (c) Rs. 150 lakh
(d) Rs. 140 lakh (e) None of these

Q155. What amount of the fund is acquired by the school from government agencies?
(a) Rs. 220 lakh (b) Rs. 310 lakh (c) Rs. 255 lakh
(d) Rs. 225 lakh (e) None of these

S151. Ans.(a)
Sol. Required difference = (Percentage of fund acquired from NGO-Percentage of fund acquired from
internal sources) of 500 lakh
500 × 10
= (15 - 5)% of 500 lakh = 100 lakh = Rs. 50 lakh

S152. Ans.(e)
Sol. Fund from government agencies
500 × 45
= 100 = Rs. 225 lakh
Expenses in school maintenance
500 × 20
= 100 = Rs. 100 lakh
∴ Remaining found = (225 - 100) lakh
= Rs. 125 lakh

S153. Ans.(a)
500 × 35
Sol. Fund from donation = 100 = Rs. 175 lakh
15 × 500
Scholarship amount = = Rs. 75 lakh
100
75
∴ Required percentage = 175 × 100 = 42.85%
= 43% (approx.)

S154. Ans.(c)
Sol. Total amount used by the school for payment
500 × 30
= 100 = Rs. 150 lakh

S155. Ans.(d)
Sol. Fund acquired from government agencies
500 × 45
= 100 = Rs. 225 lakh

61 Adda247 | No. 1 APP for Banking & SSC Preparation


Website:store.adda247.com | Email:ebooks@adda247.com
Directions (156-160): In the following table, the Investment and profit of three Companies in
different countries is given.

Investment (in mn $.) Profit (in mn $.)


State TCS Infosys Accenture TCS Infosys Accenture
Singapore 15000 — 25000 — 8000 12500
UK — 7000 8000 — — 14000
UAE 4000 5000 4500 — — —
Qatar 9000 10000 — 4500 6000 —
Malaysia — — 17000 20000 30000 40000

Note: Some values are missing. You have to calculate these values as per data given in the questions:-

Q156. If TCS invested his amount in SINGAPORE state for 9 years and Accenture invested his
amount in the same country for 10 years then find the total profit made by all of them from
SINGAPORE?
(a) mn $ 29250 (b) mn $ 24250 (c) mn $ 27250
(d) mn $ 31200 (e) None of these

Q157. If the total profit earned from UK by all of them is mn $ 32375 and each invested for 9 years
then find the ratio of investment of TCS in UK to the profit of Infosys from SINGAPORE ?
(a) 16 : 7 (b) 7 : 16 (c) 8 : 13
(d) 13 : 8 (e) None of these

Q158. If TCS, Infosys and Accenture invested in UAE for 5 years, 8 years and 6 years respectively
then profit earned by Accenture from UAE is what % of the profit earned by TCS and Infosys
together from the same Country, if total profit earned by all of them from UAE state is 8700 mn $.
(a) 45% (b) 50% (c) 55%
(d) 40% (e) None of these

Q159. In Malaysia state total Investment of TCS and Infosys is 85000 mn $, while TCS and Infosys
invested their amount for 4 years and 6 years respectively in the same country, then find the
number of years that accenture invested his amount ?
(a) 8 years (b) 9 years (c) 20 years
(d) Can’t be determined (e) None of these

Q160. Average Investment made by all of them in Qatar is $ 10,000 mn and average profit earned
by all of them from the same state is $ 6000 mn, then profit earned by Accenture in the same
country is what percent more/less than the amount invested by Accenture in the same state?
1 6 7
(a) 35 3 % (b) 37 7 % (c) 32 11 %
7 9
(d) 33 11 % (e) 31 11 %

62 Adda247 | No. 1 APP for Banking & SSC Preparation


Website:store.adda247.com | Email:ebooks@adda247.com
S156. Ans.(c)
Sol.
15000 × 9 x
=
25000 × 10 12500
27 x
=
50 12500
x = $ 6750 mn
∴ Required profit = 6750 + 8000 + 12500 = $ 27250 mn

S157. Ans.(b)
Sol.
7000 Pinfosys
=
8000 14000
PInfosys = $ 12250mn
Ptcs = 32375 − 12250 − 14000
Ptcs = $ 6125mn
Let Investment of TCS in UK = x
x 6125
∴ =
7000 12250
x = $ 3500mn
Required Ratio = (3500) : (8000)
= 7 : 16

S158. Ans.(a)
Sol.
TCS : Infosys : Accenture
𝐏𝐫𝐨𝐟𝐢𝐭 ∶ (4000 × 5) : (5000 × 8) : (4500 × 6)
20 : 40 : 27
20
∴Ptcs = 87 × 8700
Trick :
=$ 2000 mn
40
Pinfosys = 87 × 8700 = $ 4000 mn Required value =
27
Paccenture = $ 2700 mn 40+20
× 100 = 45%
2700
Required % = 6000 × 100 = 45%

S159. Ans.(c)
Sol.
x×4 20,000
=
(85,000 − x) 6 30,000
2x 2
=
3 (85,000 − x) 3
6x = 2 × 3 × 85000 − 6x
12x = 6 × 85000
x = $ 42500 mn
Itcs = $ 42500 mn

63 Adda247 | No. 1 APP for Banking & SSC Preparation


Website:store.adda247.com | Email:ebooks@adda247.com
∴ Iinfosys = $ 42500 mn
Let Required years = y
42500× 6 30,000
∴ 17,000× y = 40,000
y = 20 years

S160. Ans.(e)
Sol.
Iaccenture = 30000 − 9000 − 10000
= $ 11000 mn
Paccenture = 18000 − 4500 − 6000
= $ 7500 mn
11000−7500
Required % = × 100
11000
9
= 31 %
11

Directions (161-165): A person purchased 5 Gadgets from a shop and sold them online. Given
below is the data showing cost price, selling price and profit/loss percentage.

C.P. (in Rs.) Profit/Loss% S.P. (in Rs.)


Smartphone 32445 — 40556.25
Laptop — Profit-15% 40940
Tablet 22150 Loss-12% —
Digital camera 28295 — 31140
Smart Watch — Profit-25% 7075

Q161. Cost price of Laptop is what percent of selling price of Tablet? (approximate)
(a) 138% (b) 182% (c) 142%
(d) 154% (e) 186%

Q162. If there has been a profit of 12% on Tablet instead of 12% loss. Then the new S.P. is how
much more than the original S.P.?
(a) 5216 (b) 5396 (c) 5336
(d) 5316 (e) None of these

Q163. Profit percentage on Digital camera is what percent more/less than profit percentage on
Laptop?
(a) 50% more (b) 33.34% less (c) 33.67% more
(d) 50% less (e) 150% less

64 Adda247 | No. 1 APP for Banking & SSC Preparation


Website:store.adda247.com | Email:ebooks@adda247.com
Q164. What is the ratio between profit percentage of Smart Watch to profit percentage of
Smartphone?
(a) 5 : 3 (b) 3 : 2 (c) 3 : 5
(d) 2 : 5 (e) None of these

Q165. What is the overall profit/loss percentage? (approximate)


(a) 22.12% profit (b) 12.12% profit (c) 14.14% profit
(d) 33.12% loss (e) 15.15% loss

S161. Ans.(b)
Sol. Let cost price of Laptop = 𝑥
(100 + 15)
𝑥× = 40940
100
𝑥 = 35600
(100−12)
Selling price of Tablet = 22150 × = 19492
100
35600
Required percentage = 19492 × 100 ≈ 182%

S162. Ans.(d)
88
Sol. Original S.P.= 22150 × 100 = 19492
(100+12)
New S.P.= 22150 × = 24808
100
Difference = 5316

S163. Ans.(b)
31140−28295
Sol. Percentage profit on Digital camera = × 100 = 10.05%
28295
Profit percentage on Laptop = 15%
(15−10) 100
Required percentage = × 100 = = 33.34% 𝑙𝑒𝑠𝑠
15 3

S164. Ans.(e)
40556.25−32445
Sol. Profit percentage on 𝑆𝑚𝑎𝑟𝑡𝑝ℎ𝑜𝑛𝑒 = × 100 = 25%
32445
Profit percentage on Smart Watch= 25%
Required Ratio = 1 ∶ 1

S165. Ans.(b)
Sol. Overall cost price of all items together = 32445 + 35600 + 22150 + 28295 + 5660 = 124150
Overall selling price of all items together = 40556.25 + 40940 + 19492 + 31140 + 7075 = 139203.25
139203.25−124150
Profit percentage = × 100 ≈ 12.12% profit
124150

65 Adda247 | No. 1 APP for Banking & SSC Preparation


Website:store.adda247.com | Email:ebooks@adda247.com
Directions (166-170): Study the table and answer the given questions.

Data related to the number of employees in five different companies in December 2012

Company Total Out of total number of employees


number of
Percentage Percentage of Percentage
Employees
Of Science Commerce of Arts
graduates graduates graduates
M 1050 32% – –
N 700 – 31% 40%
O – 30% 30% –
P – – 40% 20%
Q – 35% 50% –

Note:
(I) Employees of the given companies can be categorised only in three types: Science graduates,
Commerce graduates and Arts graduates
(II) A few values are missing in the table (indicated –). A candidate is expected to calculate the
missing value, if it is required to answer the given question, on the basis of the given data and
information.

Q166. What is the difference between the number of Arts graduate employees and Science
graduate employees in Company N?
(a) 87 (b) 89 (c) 77
(d) 81 (e) 73

Q167. The average number of Arts graduate employees and commerce graduate employees in
Company Q was 312. What was the total number of employees in Company Q?
(a) 920 (b) 960 (c) 1120
(d) 1040 (e) 1080

Q168. If the ratio of the number of Commerce graduate employees to that of Arts graduate
employees in Company M was 10 : 7, what was the number of Arts graduate employees in M?
(a) 294 (b) 266 (c) 280
(d) 308 (e) 322

66 Adda247 | No. 1 APP for Banking & SSC Preparation


Website:store.adda247.com | Email:ebooks@adda247.com
Q169. The total number of employees in Company N increased by 20% from December 2012 to
December 2013. If 20% of the total number of employees in Company N in December 2013 were
Science graduates, what was the number of Science graduate employees in company N in
December 2013?
(a) 224 (b) 266 (c) 294
(d) 252 (e) 168

Q170. The total number of employees in Company P was 3 times the total number of employees in
Company O. If the difference between the number of Arts graduate employees in Company P and
that in Company O was 180, what was the total number of employees in Company O?
(a) 1200 (b) 1440 (c) 720
(d) 900 (e) 1080

S166. Ans.(c)
Sol. Total number of employees in company N = 700
Percentage of Science graduate employees = [100 – (31 + 40)] = 29%
Now, percentage difference between Arts graduate and science graduate employees
= (40 – 29)% = 11%
11% of 700 = 77
Therefore, difference = 77

S167. Ans.(b)
Sol. The percentage of Arts graduate employees in Company Q = 100 – 35 – 50 = 15%
Now, the percentage of Arts graduate employees and Commerce and Arts = 50 + 15 = 65%
Average = 312
Therefore, the total number of employees in commerce and Arts = 2 × 312
Let the total employees in Company Q be x
Then, 65% of x = 2 × 312
x = 960

S168. Ans.(a)
Sol. The percentage of commerce graduate and Arts graduate employees in company M
= 100 – 32 = 68%
68 × 7
Now, the percentage of Arts graduate employees = = 28%
17
68 × 10
the percentage of Commerce graduate employees = = 40%
17
1050 × 28
The number of arts graduate employees in company M = = 294
100

S169. Ans.(e)
Sol. The number of employees in company N in December 2012 = 700
700 × 120
The number of employees in company N in December 2013 = = 840
100
20 × 840
Number of Science graduate employees in company N in December 2013 = = 168
100

67 Adda247 | No. 1 APP for Banking & SSC Preparation


Website:store.adda247.com | Email:ebooks@adda247.com
S170. Ans.(d)
Sol. The percentage of Arts graduate employees in company O = 100 – 30 – 30 = 40%
The percentage difference between Arts graduate employees in company O and P = 40 – 20 = 20%
Now, let the number of employees in company O be x
Then, x × 20% = 180
x = 900

Directions (171-175): Study the table carefully and answer the given questions.
Data related to number of candidates appeared and qualified in a competitive exam from 2 states
during 5 years

State P State Q
Percentage of Percentage of
Number of Number of
Years appeared appeared
appeared appeared
candidates who candidates who
candidates candidates
qualified qualified
2006 450 60% – 30%
2007 600 43% – 45%
2008 – 60% 280 60%
2009 480 70% 550 50%
2010 380 – 400 –

Q171. Out of the number of qualified candidates from State P in 2008, the respective ratio of male
and female candidates is 11 : 7. If the number of female qualified candidates from State P in 2008
is 126, what is the number of appeared candidates (both male and female) from State P in 2008?
(a) 630 (b) 510 (c) 570
(d) 690 (e) 540

Q172. The number of appeared candidates from State Q increased by 100% from 2006 to 2007. If
the total number of qualified candidates from State Q in 2006 and 2007 together is 408, what is the
number of appeared candidates from State Q in 2006?
(a) 380 (b) 360 (c) 340
(d) 320 (e) 300

Q173. What is the difference between the number of qualified candidates from State P in 2006 and
that in 2007?
(a) 12 (b) 22 (c) 14
(d) 24 (e) 16

Q174. If the average number of qualified candidates from State Q in 2008, 2009 and 2010 is 210,
what is the number of qualified candidates from State Q in 2010?
(a) 191 (b) 195 (c) 183
(d) 187 (e) 179

68 Adda247 | No. 1 APP for Banking & SSC Preparation


Website:store.adda247.com | Email:ebooks@adda247.com
Q175. If the respective between the number of qualified candidates from State P in 2009 and 2010
is 14 : 9, what is the number of qualified candidates from State P in 2010?
(a) 252 (b) 207 (c) 216
(d) 234 (e) 198

S171. Ans.(e)
3𝑥
Sol. No. of qualified candidates in 2008 = 5
No. of female qualified from state P
7 3𝑥
= ×
18 5
7𝑥
=
30
7𝑥
= 126
30
𝑥 = 30 × 18 = 540
∴ Required no. of appeared candidates = 540

S172. Ans.(c)
Sol. Let no. of appeared candidates from state Q in 2006 = 100
Let no. of appeared candidates from state in 2007 = 200
∴ 30 + 90 → 408
408
1→
120
408
100 → × 100 = 340
120

S173. Ans.(a)
60 43
Sol. Required difference = 100 × 450 − 100 × 600
= 270 – 258
= 12

S174. Ans.(d)
Sol. Let required no. of candidates = x
28 × 6 + 55 × 5 + 𝑥
∴ = 210
3
168 + 275 + x = 630
x = 630 – 443
x = 187

S175. Ans.(c)
Sol.
48 × 7 14
=
𝑥 9
x = 24 × 9
x = 216

69 Adda247 | No. 1 APP for Banking & SSC Preparation


Website:store.adda247.com | Email:ebooks@adda247.com
Directions (176-180): In the following table, investments and profit of three persons is given for
different years in a joint business.

Investments (in Rs.) Profit (in Rs.)


Year A B C A B C
2012 25500 31500 34500 127500 — 172500
2013 — 7500 — — 18750 138750
2014 — 10050 12000 — — 21000
2015 — — 13500 75000 66000 36000
2016 16500 45000 — — — —

Note:
1. Apart from year 2015, they invested the amounts for same period.
2. Some values are missing. You have to calculate these value as per given data.

Q176. If the total profit in 2014 is 59587.50Rs. , then find the ratio of the investment of B in 2013 to
the investment of A in 2014.
(a) 5 : 8 (b) 10 : 27 (c) 5 : 7
(d) 5 : 9 (e) None of these

Q177. In year 2015, A and B invested their amount for 6 months and 4 months respectively and B
investedRs.24750 then find the number of months that C invested his amount for?
(a) 3 months (b) 6 months (c) 2 months
(d) 4 months (e) 1 month

Q178. Total profit earned by B in year 2012 is how much less (in Rs.) than the profit earned by him
in the year 2014?
(a) Rs. 176575.5 (b) Rs. 139912.5 (c) Rs. 193825
(d) Rs. 185050 (e) None of these

Q179. Investment made by A in 2016 is approximately what % more/less than the investment made
by C in 2013?
(a) 31% (b) 70% (c) 40%
(d) 68% (e) 79%

Q180. Total profit earned by all in 2016 is 578340Rs. and the ratio of investment made by A and B
together and investment made by B and C together is 123 : 137. Then find the difference between
the profit made by A and C in 2016 ?
(a) 47628 (b) 59428 (c) 69478
(d) 45928 (e) None of these

70 Adda247 | No. 1 APP for Banking & SSC Preparation


Website:store.adda247.com | Email:ebooks@adda247.com
S176. Ans.(a)
Sol. Let profit of B in 2014 = x
12000 10.050
∴ =
21000 𝑥
x = 17587.5 Rs.
Profit of A in 2014 = 59587.5 – 178587.5 – 21000
= 21000 Rs.
12×21
∴ Required Ratio = (7500) :( ) × 1000
21
= 7500 : 12000
=5:8

S177. Ans.(d)
Sol. Let C invested for x months, then
24750 × 4 66000
=
13500 × 𝑥 36000
⇒x=4

S178. Ans.(b)
Sol. Let profit earned by B in year 2012 = 𝑥
Profit earned by B in year 2014 = 𝑦
31500 34500
=
𝑥 172500
x = 157500 Rs.
12000 10050
And, 21000 = 𝑦
y = 17587.5 Rs.
Required Difference = 157500 - 17587.5 =139912.5

S179. Ans.(b)
55500−16500
Sol. Required % = × 100
55500
≈ 70%

S180. Ans.(a)
Sol.
(16500 + 45000)
Investment made by C in 2016 = × 137 − 45000
123
= 23500 Rs.
Ratio of their investment = 165 : 450 : 235
= 33 : 90 : 47
(47 − 33)
Difference= × 578340 = 47628
170

71 Adda247 | No. 1 APP for Banking & SSC Preparation


Website:store.adda247.com | Email:ebooks@adda247.com
Directions (181-185): The following bar graph shows the production (in lakh tonnes) of 3
companies A, B and C in different years. Study the graph and answer the following questions:

2005
50
40
30
20
2009 2006
10 A
0 B
C

2008 2007

Q181. The average production (in lakh tonnes) of company A over the given years is:
(a) 32 (b) 36 (c) 38
(d) 35 (e) None of these

Q182. The total production of all 3 companies together in 2008 is what percent more/less than that
in 2006? (rounded off to two decimal points)
(a) 4.67% (b) 5.17% (c) 5.67%
(d) 4.17% (e) 6.67%

Q183. The total production of all 3 companies together is 2nd lowest in


(a) 2005 (b) 2006 (c) 2007
(d) 2008 (e) 2009

Q184. What is the percentage decrease in total production of all 3 companies together in 2007 as
compared to previous year?
(a) 10.5% (b) 11.5% (c) 9.5%
(d) 12.5% (e) None of these

Q185. What is the ratio of total production of company B to that of company C in all years
together?
(a) 38 : 35 (b) 38 : 37 (c) 35 : 38
(d) 37 : 38 (e) None of these
72 Adda247 | No. 1 APP for Banking & SSC Preparation
Website:store.adda247.com | Email:ebooks@adda247.com
S181. Ans.(c)
190
Sol. Required average = = 38 lakh tonnes
5

S182. Ans.(d)
(125−120)
Sol. Required percentage = × 100 = 4.17%
120

S183. Ans.(c)
Sol. Total production is 2nd lowest in 2007 i.e. 105 lakh tonnes

S184. Ans.(d)
15
Sol. Required percentage = 120 × 100 = 12.5%

S185. Ans.(d)
185
Sol. Required ratio = 190 = 37 ∶ 38

Directions (186-190): Read the following graph and table carefully and answer the questions given
below.
Percentage of admitted students in different discipline from 2005 to 2009
Science Commerce Art
2005
60
50
40
30
2009 20 2006
10
0

2008 2007

Total number of admitted students in different years


Year Total number of students
2005 12560
2006 14820
2007 13850
2008 16580
2009 11220

73 Adda247 | No. 1 APP for Banking & SSC Preparation


Website:store.adda247.com | Email:ebooks@adda247.com
Q186. What is the average number of students in Arts in 2008 and 2009 together?
(a) 5182 (b) 5475 (c) 5318
(d) 5267 (e) None of these

Q187. The number of students in Science in 2008 is approximately what percent of the number of
students in commerce in 2006?
(a) 67 (b) 72 (c) 63
(d) 58 (e) 78

Q188. What is the difference between the number of students in Science in 2006 and number of
students of Commerce in 2008?
(a) 1625 (b) 1546 (c) 1871
(d) 1781 (e) None of these

Q189. What is the difference between the number of students in Arts in 2008 and number of
students in Science in 2006?
(a) 3210 (b) 3103 (c) 3325
(d) 3014 (e) None of these

Q190. What is the total number of students in Commerce in all the years?
(a) 28026 (b) 21642 (c) 22510
(d) 19441 (e) None of these

S186. Ans.(d)
10534
Sol. Average = = 5267.
2

S187. Ans.(a)
4974
Sol. Required percent = 7410 × 100 = 67% (approx.)

S188. Ans.(c)
Sol. Difference = 5187 – 3316 = 1871.

S189. Ans.(b)
Sol. Difference = 8290 – 5187 = 3103

S190. Ans.(e)
Sol. Total number = 20826.

74 Adda247 | No. 1 APP for Banking & SSC Preparation


Website:store.adda247.com | Email:ebooks@adda247.com
Directions (191-195): There are five students who appeared for RBI Grade B exam. Paper consists
of 100 questions with 1 mark for each correct answer and 0.25 marks for each wrong answer.

Students Questions attempted Right Questions Wrong Questions Marks obtained


Aditya 78 – – 70.5
Puskar 92 76 – –
Anshuman 98 – 36 –
Alka – 30 – 27.25
Avanish 56 – – 53.50

Q191. Difference between total right number of questions of all students together and total wrong
no. of questions of all students together is
(a) 141 (b) 161 (c) 223
(d) 156 (e) None of these

Q192. Marks obtained by Aditya and Puskar together is what % of the marks obtained by
Anshuman, Avanish and Alka together? (rounded off to 2 decimal places)
(a) 106.54% (b) 91.16% (c) 95.20%
(d) 96.71% (e) 101.71%

Q193. If the penalty of wrong answer is 0.33 then marks obtained by Aditya, Anshuman and
Puskar together is
(a) 192.21 (b) 224.19 (c) 190.86
(d) 219.14 (e) 194.22

Q194. If the passing % marks in the exam is 50 marks than at least how many questions has to be
answered right by Puskar? (He attempted 92 questions)
(a) 58 (b) 56 (c) 59
(d) 55 (e) 60

Q195. What is the percent of marks obtained by all of them together?


(a) 59.03% (b) 53.15% (c) 52.53%
(d) 45.05% (e) 55.25%

S191. Ans.(c)
Sol. Required difference = (72 + 76 + 62 + 30 + 54) – (6 + 16 + 36 + 11 + 2)
= 294 – 71 = 223

S192. Ans.(a)
70.5+72
Sol. Required % = 53+27.25+53.50 × 100
= 106.54%

75 Adda247 | No. 1 APP for Banking & SSC Preparation


Website:store.adda247.com | Email:ebooks@adda247.com
S193. Ans.(c)
Sol. Required marks = (72 + 76 + 62) – 0.33 (6 + 16 + 36) = 190.86

S194. Ans.(c)
Sol. By options
Let right Questions = 59
1
∴ marks = 92 − 4 (92 − 59) = 50.75

S195. Ans.(e)
70.5 + 72 + 53 + 27.25 + 53.50
Sol. Required % = × 100 = 55.25%
500

Directions (196-200): Seven companies A, B, C, D, E, F and G are engaged in production of two


items I and II. Comparative data about production of these items by the companies is given in the
following graph and table. Study them carefully and answer the questions given below.

Percentage of the total production produced by the seven companies

G, 12% A, 15%
F, 5%

B, 11%

E, 27%
C, 22%

D, 8%

Cost of the total production (both items together) by seven companies = Rs 25 crores
Ratio of production between items I and II and the per cent profit earned for the two items

Company Ratio of Per cent profit earned


Production
Item I Item II Item I Item II
A 2 3 25 20
B 3 2 32 35
C 4 1 20 22
D 3 5 15 25
E 5 3 28 30
F 1 4 35 25
G 1 2 30. 24

76 Adda247 | No. 1 APP for Banking & SSC Preparation


Website:store.adda247.com | Email:ebooks@adda247.com
Q196. What is the total cost of the production of item I by companies A and C together in Rs crore?
(a) 9.25 (b) 5.9 (c) 4.1625
(d) 4.9 (e) None of these

Q197. What is the amount of profit earned by company D on item II?


(a) Rs 3.125cr (b) Rs 31.25 cr (c) Rs 3.125 lakhs
(d) Rs 31.25 lakhs (e) None of these

Q198. Cost of production of item I by company F is what per cent of the cost of production of item
II by company D?
(a) 16% (b) 33.33% (c) 66.67%
(d) 20% (e) None of these

Q199. What is the ratio of the cost of production of item I by company A to the cost of production
of item I by company D?
(a) 3 : 5 (b) 1 : 2 (c) 2 : 1
(d) 2 : 3 (e) None of these

Q200. What is the total of the profit earned by company B on production of item I and the profit
earned by company A on production of item II?
(a) Rs 9.78 cr (b) Rs 97.8 lakhs (c) Rs 52.8 lakhs
(d) Rs 5.28 cr (e) None of these

S196. Ans.(b)
15
Sol. Total cost of production by company A = 100 × 25 = 3.75 crores = 3.75 crores
22
Total cost of production by Company C = 100 × 25 = 5.5 crores
2
Cost of production of item I by Company A = 5 × 3.75 = 1.5 crores
4
Cost of production of item I by Company C = 5 × 5.5 = 4.4 crores
∴ Required total cost = 1.5 + 4.4 = 5.9 crores

S197. Ans.(d)
25 5 8
Sol. Required profit earned = 100 × 8 × 100 × 25 = 0.3125 crores
= 31.25 lakhs

S198. Ans.(d)
Sol. Required %
5 1
× 5 × 25
100
= 8 5 × 100
× × 25
100 8
0.25
= × 100 = 20%
1.25
77 Adda247 | No. 1 APP for Banking & SSC Preparation
Website:store.adda247.com | Email:ebooks@adda247.com
S199. Ans.(c)
Sol. Required Ratio
15 2
× 5 × 25
100
= 8 3
× 8 × 25
100
30 800
= ×
500 24
5×8
=
5×4
=2:1

S200. Ans.(b)
Sol. Required total profit
32 3 11 20 3 15
=( × × × 25) + ( × × × 25)
100 5 100 100 5 100
= 0.528 + 0.45
= 0.978 crores
= 97.8 lakhs

78 Adda247 | No. 1 APP for Banking & SSC Preparation


Website:store.adda247.com | Email:ebooks@adda247.com
79 Adda247 | No. 1 APP for Banking & SSC Preparation
Website:store.adda247.com | Email:ebooks@adda247.com
500 Quadratic Equations for IBPS RRB/PO/Clerk

500 Quadratic Equation Questions for IBPS RRB/PO/Clerk 2017

eBook

Direction (Q. 1-5): Two equations (I) and (II) are given in each question. On the basis of these equations.
You have to decide the relation between 'x’and 'y’and give answer .
a) If x < y
b) If x ≥ y
a) If x > y
d) If x = y or no relation can be decided between 'x’and 'y'.
e) If x ≤ y
1). I. 2x2 - 15x + 28 = 0
II. 4y2 - 23y + 30 = 0

2). I. 2x2 - 15x + 27 = 0


II. 5y2 - 26y + 33 = 0

3). I. 6x2 + 25x + 24 = 0


II.12y2 + 13y + 3 = 0

4). I. 12x2 – x - 1 = 0
II. 20y2 - 41y + 20 = 0

5). I. 10x2 + 33x + 27 = 0


II. 5y2 + 19y + 18 = 0

Direction (Q. 6-10): Two equations (I) and (II) are given in each question. On the basis of these
equations. You have to decide the relation between 'x’and 'y’and give answer .
a) If x > y
b) If x < y
c) If x ≥ y
d) If x ≤ y
e) If x = y or no relation can be decided between 'x’and 'y'.

6). I. x = (-7)2
II. y2 - 46y - 147 = 0

www.ibpsguide.com | estore.ibpsguide.com | www.sscexamguide.com 1


500 Quadratic Equations for IBPS RRB/PO/Clerk

7). I. 4x2 - x - 3 = 0
II. 4y2 - 16y + 15 = 0

8). I. 2x2 - 117x + 1701 = 0


II. y2 = 992.25

9). I. 12x2 - 41x + 35 = 0


II. 3y2 - 17y - 28 = 0

10). I. 4x - 3y = 16
II. 7x - 8y = 17

1). D)
I. 2x2 - 15x + 28 = 0
or, 2x2 - 7x - 8x + 28 = 0
or, x (2x -7) - 4 (2x - 7) = 0
or, (2x - 7) (x - 4) = 0
x = 7/2 = 3.5, x = 4
II. 4y2 - 23y + 30 = 0
or, 4y2 – 15y - 8y + 30 = 0
or, y (4y - 15) -2 (4y - 15) = 0
or, (4y - 15) (y - 2) = 0
y = 15/4 = 3.8, y= 2
Hence x = y or no relationship can be established.

2). B)
I. 2x2 - 15x + 27 = 0
or, 2x2 - 9x - 6x + 27 = 0
or, x (2x -9) - 3 (2x - 9) = 0
or, (2x - 9) (x - 3) = 0
x = 9/2 = 4.5, x = 3
II. 5y2 - 26y + 33 = 0
or, 5y2 – 15y - 11y + 33 = 0
or, 5y (y - 3) -11 (y - 3) = 0
or, (5y - 11) (y - 3) = 0
y = 11/5 = 2.2, y= 3
Hence x ≥ y

3). A)
I. 6x2 + 25x + 24 = 0
or, 6x2 + 9x + 16x + 24 = 0
or, 3x (2x +3) + 8 (2x + 3) = 0
or, (2x + 3) (3x + 8) = 0
x = - 3/2 = - 1.5, x = - 8/3 = - 2.6
II.12y2 + 13y + 3 = 0
or, 12y2 + 9y +4y + 3 = 0

www.ibpsguide.com | estore.ibpsguide.com | www.sscexamguide.com 2


500 Quadratic Equations for IBPS RRB/PO/Clerk

or, 3y (4y +3) +1 (4y + 3) = 0


or, (3y +1) (4y + 3) = 0
y = - 1/3 = -0.9, y= - 3/4 = -0.75
Hence x < y

4). A)
I. 12x2 – x - 1 = 0
or, 12x2 -4x + 3x -1 = 0
or, 4x (3x -1) + 1 (3x-1) = 0
or, (4x + 1) (3x - 1) = 0
x = - 1/4 = - 0.25, x = 1/3 = 0.3
II. 20y2 - 41y + 20 = 0
or, 20y2 - 16y -25y + 20 = 0
or, 4y (5y -4) -5 (5y-3) = 0
or, (5y - 4) (4y - 5) = 0
y = 4/5 = 0.8, y= 5/4 = 1.25
Hence x < y

5). B)
I. 10x2 + 33x + 27 = 0
or, 10x2 +15x + 18x +27 = 0
or, 5x (2x +3) + 9 (2x + 3) = 0
or, (5x + 9) (2x +3) = 0
x = - 9/5 = - 1.8, x = 3/2 = -1.5
II. 5y2 + 19y + 18 = 0
or, 5y2 + 10y +9y + 18 = 0
or, 5y (y + 2) +9 (y + 2) = 0
or, (5y + 9) (y +2) = 0
y = - 9/5 = 0.8, y= - 2
Hence x ≥ y

6). C) x = (-7)2
x = 49
= y2 - 46y - 147 = 0
y2 - 49y + 3y - 147 = 0,
= y (y-49) + 3 (y - 49) = 0
(y - 49) (y + 3) = 0 y = 49, -3
x≥y

7). B) 4 x2 - x - 3 = 0
= 4x2 - 4x +3x - 3 = 0
= 4 x (x - 1) + 3 (x - 1) = 0
= (4x + 3) (x - 1) = 0
x = 3/4, 1

www.ibpsguide.com | estore.ibpsguide.com | www.sscexamguide.com 3


500 Quadratic Equations for IBPS RRB/PO/Clerk

4y2 - 16y + 15 = 0
4y2 - 10y - 6y + 15 = 0
2y (2y – 5) –3 (2y - 5) = 0
(2y - 3) (2y - 5) = 0
y = 3/2, 5/2
x<y

8). D) 2x2 - 117x + 1701


= 2x2 - 54x - 63x + 1701
= 2x ( x - 27) -63 ( x - 27)
= (2x - 63) ( x - 27) = 0
x = 27, 63/2
y = √(992.25)
y = 31.5
x≤y

9). E) 12x2 - 41x + 35 = 0


= 12x2 - 20x - 21x + 35 = 0,
= 4x (3x - 5) - 7 ( 3x - 5) = 0,
= (4x - 7) (3x -5) = 0,
x = 7/4, 5/3
3y2 - 17y - 28 = 0
3y2 - 21y +4y - 28= 0
3y ( y - 7) + 4 ( y – 7) = 0
( y -7) ( 3y + 4) = 0
y = 7, -4/3
ie. no relation between ‘x’and ‘y’.

10). A) Equn (I) × 7 - Equn (II) × 4


(28x - 21y = 112) - (28x – 32y = 68)
11y = 44
y = 4,& from this x = 7
x >y

Directions (Q. 11-15): For the two given equations I and II.
Give answer:
a) If p is greater than q.
b) If p is smaller than q.
c) If p is equal to q.
d)If p is either equal to or greater than q.
e) If p is either equal to or smaller than q.
11) I. 6p2+5p+1=0
II. 20q2+9q=-1

www.ibpsguide.com | estore.ibpsguide.com | www.sscexamguide.com 4


500 Quadratic Equations for IBPS RRB/PO/Clerk

12) I. 3p2+2p-1=0
II. 2q2+7q+6=0

13) I. 3p2+15p=-18
II. q2+7q+12=0

14) I. p=√4/√9
II. 9q2-12q+4=0

15) I. p2+13p+42=0
II. q2=36

Directions (Q. 16-20): In each question two ego numbered I and II are given. You have to solve both
equations and mark the appropriate answer.
a) if x >y
b) ifx≥y
c) if x<y
d) ifx≤y
e) if x = y or no relation can be established between x and y.
16). I. 9x2 - 41x+46=0
II. 12y2 + 43y+ 38 = 0

17). I. 6x2 + 13x - 169 =0


II. y2 + 8y - 65 = 0

18). I. 3x + 5y = 4
II. 6x - 7y= 25

19). I. x2 - 5x + 4 = 0
II. y2 + 11y – 12 = 0

20). I. 8x2 + 50x + 57 = 0


II. 6y2 – y – 57 = 0

11).B)
I. 6p2+5p+1=0
(3p+1)(2p+1)=0
p=-1/3,-1/2
II. 20q2+9q+1=0
(4q+1)(5q+1)=0
q=-1/4,-1/5
∴p<q

12). A)
I. 3p2+2p-1=0
(3p-1)(p+1)=0
p=1/3,-1

www.ibpsguide.com | estore.ibpsguide.com | www.sscexamguide.com 5


500 Quadratic Equations for IBPS RRB/PO/Clerk

II. 2q2+7q+6=0
(2q+3)(q+2)=0
q=-3/2,-2

13). D)
I. 3p2+15p+18=0
(3p+6)(p+3)=0
p=-2,-3
II. q2+7q+12=0
(q+4)(q+3)=0
q=-3,-4
∴p≥q

14). C)
I. p=√4/√9=2/3
II. 9q2-12q+4=0
(3q-2)2=0
q=2/3
∴p=q

15). E)
p2+13p+42=0
(p+7)(p+6)=0
p=-6,-7
II. q2=36
q=±6
∴p≤q

16). I 9x2 - 41x + 46 = 0


or, 9x2 - 18x - 23x + 46 = 0
or, 9x(x — 2) - 23(x - 2) = 0
or, (9x - 23) (x - 2) =0
x = 23/9, 2
II. 12y2 - 43y + 38 =0
or, 12y2 + 24y + 19y + 38 = 0
or, 12y(y + 2) + 19(y + 2) = 0
or, (12y + 19) (y+ 2) = 0
y = 19/12, -2
Therefore x > y
Answer: a)

17). I.6x2 + I3x - 169 = 0


or, 6x2 - 26x + 39x - 169 =0

www.ibpsguide.com | estore.ibpsguide.com | www.sscexamguide.com 6


500 Quadratic Equations for IBPS RRB/PO/Clerk

or, 2x(3x - 13) + 13(3x - 13) = 0


or, (2x + 13) (3x - 13) = 0
x = -13/2, 13/3
II. y2 + 8y - 65 = 0
or, y2 + 13y - 5y - 65 = 0
or, y(y + 13) - 5(y + 13) = 0
or, (y - 5) (y + 13) = 0.
y = 5, -13
Therefore relation can't be established between x and y
Answer: e)

18). I. 3x + 5y = 4
II. 6x – 7y = 25
Solving equation (I) and (ii), we get
6x + 10y = 8
6x – 7y = 25
17y = -17
y = -1 and x =3, Therefore x > y
Answer: a)

19). I. x2 - 5x + 4 = 0
or, x2 – 4x – x + 4 = 0
or, x(x - 4) – 1(x -4) = 0
or, (x -1) (x - 4) = 0
x = 1, 4
Il.y2 + 11y - 12= 0
or, y2 + 12y – y -12 = 0
or, y(y + 12) – 1(y + 12) = 0 or, (y - 1) (y + 12) = 0
y = 1, -12
Therefore, x ≥ y
Answer: b)

20). I 8x2 + 50x + 57 = 0


or, 8x2 + 12x + 38x +57=0
or, 4x(2x + 3) + 19(2x + 3) = 0
or, (4x + 19) (2x + 3) = 0
x = -19/4, -3/2
II.6y2 – y - 57=0
or, 6y2 + 18y - 19y - 57 = 0
or, 6y(y + 3) - 19(y + 3) = 0
or, (6y - 19) (y + 3) = 0
y= 19/6, -3

www.ibpsguide.com | estore.ibpsguide.com | www.sscexamguide.com 7


500 Quadratic Equations for IBPS RRB/PO/Clerk

Therefore relation can't be established.

Directions (Q. 21-25): In the following questions numbered I and II are given. You have to solve out the
equations and Give answer
a) if x > y
b) if x ≥ y
c) if x < y
d) if x ≤ y
e) if x = y or the relationship cannot be established
21). I. 4x2 + 27x + 35 = 0
II. 3y2 - 23y +42 = 0

22). I. 9x2 + 15x - 14 =0


II.7y2 – 23y + 16 =0

23). I.x2 = 1156


II. y = 3√(35937)

24). I. 2x - 3y = 43
II. 3x + 4y = -12

25). I. 5x2 - 44x + 63 = 0


II. 10y2 + 23y + 12 = 0

Directions (Q. 26-30): Two equations (I) and (II) are given in each questions. On the basis of these questions,
you have to decide the relation between x and y and give answer
a) if x > y
b) if x < y
c) if x ≥ y
d) if x ≤ y
e) if x =y, or no relation can be established between x and y.
26). I. 5x2 – 87x + 378 = 0
II. 3y2 – 49y + 200 = 0

27). I. 10x2 – x – 24 = 0
II. y2 – 2y = 0

28). I. x2 – 5x + 6 = 0
II. 2y2 – 15y + 27 = 0

29). I. 3x + 2y = 301
II. 7x – 5y = 74

30). I. 14x2 – 37x + 24 = 0

www.ibpsguide.com | estore.ibpsguide.com | www.sscexamguide.com 8


500 Quadratic Equations for IBPS RRB/PO/Clerk

II. 28y2 – 53y + 24 = 0

21). C)
I. 4x2 + 27x + 35 = 0
x = +(20/4), +(7/4)
x = -5, -(7/4)
II. 3y2 - 23y +42 = 0
y = -(14/3), -(9/3)
y = (14/3), 3
hence x < y.

22). C)
I. 9x2 + 15x - 14 =0
x = +(21/9), -(6/9)
x = -(7/3), +(2/3)
II.7y2 – 23y + 16 =0
y = -(16/7), -(7/7)
y = (16/7), 1
hence x < y.

23). E) I.x2 = √1156


x = 1156 = ± 34
II. y = 3√(35937) = 33
Hence no relation can be established.

24). A) 2x - 3y = 43 ..(i)
3x + 4y = -12 …(ii)
Solving equation (i)×3 – (ii) ×2
(6x – 9y = 129) – (6x + 8y = -24) = -17y = 153
Y = -(153/7) = -9
Putting the value of y in equation (i), we get
2x – 3(-9) = 43
Or, 2x + 27 = 43
X = (43-27) / 2 = 16/2 = 8
Hence x > y

25).A) I. 5x2 - 44x + 63 = 0


x = -(35/5), -(9/5)
x = 7, (9/5)
II. 10y2 + 23y + 12 = 0
y = (15/10), (8/10)
y = -(3/2), -(4/5)

www.ibpsguide.com | estore.ibpsguide.com | www.sscexamguide.com 9


500 Quadratic Equations for IBPS RRB/PO/Clerk

hence x > y.

26). I. 5x2 - 45x - 42x + 378 = 0


or,5x(x - 9) - 42(x- 9) = 0
or. (5x - 42)(x -9) = 0
x = 9, 42/5
II. 3y2 - 24y - 25y + 200=0
or, 3y(y - 8) -25(y - 8) = 0 or, (y - 8)(3y-25)=0
y = 8, 25/3
Hence, x>y
Answer: a)

27). I. 10x2 -16x + 15x – 24 = 0


or, 2x(5x -8) + 3(5x 8)=0
or,(2x + 3)(5x +8) = 0
x = -3/2, 8/5
II. y2 – 2y = 0
or, y(y -2) = 0
y = 0, 2
ie no relationship exists between x and y.
Answer: e)

28). x2 - 2x - 3x + 6 =0
or, x(x -2) - 3(x -2) = 0
or, (x -2) (x -3) = 0
x =2, 3
2y2 - 6y - 9y + 27 = 0
or, 2y(y - 3)-9(y -3) = 0
or, (y- 3) (2y -9) = 0
y = 3, 9/2
hence, x ≤ y
Answer: d)

29). I. eqn (I) × 5 + eqn (II) × 2


[15x + 10y = 1505] + [14x – 10y = 148] = 29x = 1653
x = (1653/29) = 57
and y = 65
hence, x< y
Answer: b)

30). 14x2 - 37x + 24 = 0


or, I4x2 - 21x- 16x + 24 = 0
or, 7x(2x -3) -8(2x -3) = 0

www.ibpsguide.com | estore.ibpsguide.com | www.sscexamguide.com 10


500 Quadratic Equations for IBPS RRB/PO/Clerk

or, (2x - 3)(7x - 8) = 0


x = (3/2), (8/7)
II. 28y2 - 53y + 24 = 0
or, 28y2 -21y - 32y + 24 =0
or, 7y(4y - 3) -8(4y - 3) = 0
or, (7y - 8) (4y - 3) = 0
y = 8/7, 3/4
x≥y
Answer: c)

Directions (Q. 31-35): In each of these questions, two equations (I) and (II) are given. Solve both the
equations and give answer
a) if x > y
b) if x < y
c) if x ≥ y
d) if x ≤ y
e) if x = y or no relation can be established between ‘x’and ‘y’.

31).
I. x - 7 √3x + 36 = 0
II. y -12 √2y + 70 = 0

32).
I. 10x + 6y = 13
II. 45x + 24y = 56

33).
I. 63x -194 √x +143 = 0
II. 99y - 255 √y +150 = 0

34).
I. 16x^2 – 40x – 39 = 0
II. 12y^2 – 113y + 255 = 0

35).
I. x^2 - 7 √7x + 84 = 0
II. y^2 - 5 √5y + 30 = 0

Directions (Q. 36-40) In the following questions, two equations I and II are given. You have to solve both the
equations.
Give Answer
a)If x > y
b)If x > = y
c)If x < y
d)If x < = y

www.ibpsguide.com | estore.ibpsguide.com | www.sscexamguide.com 11


500 Quadratic Equations for IBPS RRB/PO/Clerk

e)If x = y or the relation cannot be established


36). I. (25 / x2) – (12 / x) + (9 / x2) = (4 / x2)
II. 9.84 – 2.64 = 0.95 + y2

37). I .√(900)x + √(1296) = 0


II. (256)1/4 y + (216)1/3 = 0

38). I. [(3)5 + (7)3 / 3] = x3


II. 7y3 = - (15 × 2) + 17y3

39). I. (x1/4 / 16)2 = 144 / x3/2


II. y1/3 × y2/3 × 3014 = 16 × y2

40). I. 3x2 –19x + 28 = 0


II. 5y2 – 18y + 16 = 0
31). I. x - 7 √3x + 36 = 0
or x - 7 √3. √x + 36 = 0
or x - 3 √3. √x - 4 √3. √x + 36 = 0
or ( √x - 3 √3)(√x - 4√3) = 0
x = 27, 48
II. y - 5 √2y - 7 √2y + 70 = 0
or y - 5 √2. √y - 7 √2. √y + 70 = 0
or ( y - 5 2)( y - 7 2) = 0
y = 50, 98
x<y
Answer: b)

32). I. 10x + 6y = 13
II. 45x + 24y = 56
On solving both equations,
x = 4/5, y = 5/6
Answer: b)

33). I. 63x -194 √x +143 = 0


or63x -117 √x - 77√x +143 = 0
or (7 √x -13)(9 √x -11) = 0
x = 169/49, 121/81
II. 99y - 225 √y +150 = 0
or 99y - 90 √y -165 √y +150 = 0

www.ibpsguide.com | estore.ibpsguide.com | www.sscexamguide.com 12


500 Quadratic Equations for IBPS RRB/PO/Clerk

or (11 √y -10)(9√y -15) = 0


y = 100/121, 225/81
Therefore relation cannot be established between x and y.
Answer: e)

34). I. 16x^2 - 40x - 39 = 0


or 16x^2 - 52x + 12x - 39 = 0
or (4x- 13) (4x + 3)
x = 13/4, -3/4
II. 12y^2 - 113y + 255 = 0
or 12y^2 - 45y - 68y + 255 = 0
or (4y - 15) (3y - 17) = 0
y = 15/4, 17/3
Therefore y > x or, x < y
Answer: b)

35). I. x^2 - 7 √7x + 84 = 0


or (x - 4 √7)(x - 3√7) = 0
x = 4 √7, 3 √7
II. y^2 - 5 √5y + 30 = 0
or (y - 2 √5)(y - 3 √5) = 0
y = 2 √5, 3 √5
x>y
Answer: a)

36). I. (25/x2 ) + (9/x2 ) – (4/x2 ) = (12/x)


(25 + 9 - 4) / x2 = 12/x = 30/x2 = 12/x
12x = 30
x = 30 / 12 = 5/2 = 2.5
II. 9.84 -2.64 = 0.95 + y2
7.2 – 0.95 = y2
y = √(6.25) = ± (2.5)
clearly x ≥ y
Answer: b)

37). I. √(900)x + √(1296) =0


√(900)x = -√(1296)
30x = -36
x = -36 / 30 = -1.2

www.ibpsguide.com | estore.ibpsguide.com | www.sscexamguide.com 13


500 Quadratic Equations for IBPS RRB/PO/Clerk

II. (256)1/4 y = (216)1/3


(44)1/4 y = - (63)1/3 4y = -6
Y = -(6/4) = -1.5
Clearly, x > y
Answer: a)

38). I. [(3)5 + (7)3] / 3 = x3


(243 + 343) / 3 = x3
(586 / 3) = x3
II. 7y3 = -30 + 17y3 = 10y3 = 30
y3 = 30/10 = 3
clearly, x > y
Answer: a)

39). I. (x1/4 / 16)2 = (144 / x3/2 ) = (x1/2 / 256) = (144 / x3/2 )


(x1/2 ) × (x3/2 ) = 256 × 144
x2 = (256 × 144)
x = √(256 × 144)
x = ± (16 × 12) = ±192
II. y1/3 × y2/3 × 3104 = 16y2
y × 3104 = 16y2
3104 = 16y
Y = 3104 / 16 = 194
Clearly, x > y
Answer: c)

40). I. 3x2 – 19x + 28 = 0


3x2 - 12x – 7x + 28 = 0
3x (x - 4) – 7 (x - 4) = 0
(x - 4) (3x - 7) = 0
x = 4, 7/3
II. 5y2 – 18y + 16 = 0
5y2 – 10y – 8y + 16 = 0
5y (y - 2) – 8 (y - 2) = 0
(y - 2) (5y - 8) = 0
Y = 2, 8/5
Clearly, x > y
Answer: a)

www.ibpsguide.com | estore.ibpsguide.com | www.sscexamguide.com 14


500 Quadratic Equations for IBPS RRB/PO/Clerk

Directions (Q. 41-50) In the following questions, two equations I and II are given. You have to solve both the
equations.
Give Answer
a) If x > y
b)If x >= y
c)If x < y
d)If x < = y
e)If x = y or the relationship cannot be established
41). I. x2 – 1200 = 244
II. y + 122 = 159

42). I. 14x – 25 = 59 – 7x
II. √(y + 222) - √(36) = √(81)

43). I. 144x2 – 16 = 9
II. 12y + √4 = √(49)

44). I. x2 – 9x + 20 = 0
II. y2 – 13y + 42 = 0

45). I. (√(x) / 5) + (3 √(x) / 10) = (1 / √(x))


II. (10 / √(y)) – (2 / √(y)) = 4√(y)

Directions (Q. 46-55): Two equations (I) and (II) are given in each question. On the basis of these equations,
you have to decide the relation between x and y and give answer
46).I. 2x^2 + x - 1 = 0
II. 6y^2 - 13y + 5 = 0
a) x>y
b) x<y
c) x≥y
d) x = y or no relation can be established between ‘x’and ‘y’.
e) x ≤ y
47).I. 21x^2 - 122x + 165 = 0
II. 3y^2 - 2y - 33 = 0
a) x>y
b) x≥y
c) x<y
d) x ≤ y
48). I. 5x^2 - 29x + 36 = 0
II. 10y^2 - 3y - 27 = 0
a) x>y
b) x≤y
c) x<y
d) x≥y

49). I. 7x + 4y = 3
II. 5x + 3y = 3
a) x>y

www.ibpsguide.com | estore.ibpsguide.com | www.sscexamguide.com 15


500 Quadratic Equations for IBPS RRB/PO/Clerk

b) x<y
c) x≤y
d) x≥y

50). I. 7x^2 - 54x + 99 = 0


II. 4y^2 - 16y + 15 = 0
a) x>y
b) x<y
c) x≥y
d) x≤y

41). I. x2 = 1200 + 244


x2 = 1444
x = √(1444) = ± 38
II. y = 159 – 122 = 37
Clearly, x > y or x < y
Hence, the relationship cannot be established.
Answer: e)

42). I. 14x + 7x = 59 + 25
21x = 84; x = 4
II. √(y + 222) = √(36) + √(81)
√(y + 222) = ± 6 ±9
√(y + 222) = ± 15
Taking (+ve) sign,
√(y + 222) = 15
y + 222 = 225
y = 225 – 222 = 3
Taking (-ve) sign,
√(y + 222) = -15
(y + 222) = 225
Y = 225 – 222 = 3
Clearly, x > y
Answer: a)

43). I. 144x2 = 16 + 9
144x2 = 25 = x2 = 25 / 144
x = ± 5 / 12
II. 12y = √49 - √4
12y = ± 7 – (± 2)
12y = ± 5

www.ibpsguide.com | estore.ibpsguide.com | www.sscexamguide.com 16


500 Quadratic Equations for IBPS RRB/PO/Clerk

y = ± 5 / 12
clearly, x = y
Answer: e)

44). x2 – 9x + 20 = 0
x2 – 5x -4x +20 = 0
x(x - 5) – 4 (x - 5) = 0
(x - 5) (x - 4) = 0
x = 5 or 4
II. y2 – 13y + 42 = 0
y2 – 7y – 6y + 42 = 0
y(y - 7) – 6 (y - 7) = 0
(y - 7) (y - 6) = 0
Y = 6 (or) 7
Clearly, x < y
Answer: c)

45). I. (2√x + 3√x) / 10 = 1 / √x


2x + 3x = 10
5x = 10
x=2
II. (10 - 2) / √y = 4 √y
8 = 4y
y=8/4=2
Clearly, x = y
Answer: e)

46). D)
2x^2 + 2x - x - 1 = 0
or, 2x(x + 1) - 1(x + 1) = 0
or, (x + 1) (2x - 1) = 0
x = -1 , 1/2
II. 6y^2 - 3y - 10y + 5 = 0
or, 3y(2y - 1) - 5(2y - 1) = 0
or, (3y - 5)(2y - 1) = 0
y = 0.5, 5/3
hence no relationship.

47). D)
I. 21x2 - 45x - 77x + 165 = 0
or, 3x(7x - 15) - 11 (7x - 15) = 0

www.ibpsguide.com | estore.ibpsguide.com | www.sscexamguide.com 17


500 Quadratic Equations for IBPS RRB/PO/Clerk

or, (3x - 11) (7x - 15) = 0


x = 11/3, 15/7
II. 3y2 + 9y - 11y - 33 = 0
or,3y(y + 3) - 11(y + 3) = 0
or,(3y - 11) (y + 3) = 0
y = -3 , 11/3
x≤y

48). D
I. 5x^2 - 20x - 9x + 36 = 0
or, 5x(x - 4) - 9(x - 4) = 0
or,(x - 4) (5x - 9) = 0
x = 4 , 9/5
II. 10y^2 + 15y - 18y - 27 = 0
or, 5y(2y + 3) - 9(2y + 3) = 0
or, (2y + 3) (5y - 9) = 0
y = 9/5, - 3/2
x≥y

49).B)
eqn (I) × 3 - eqn (II) × 4

21x + 12y = 9
20x + 12y = 12
---
-------------------
x=-3
and y = 6
x<y

50).A)
I. 7x^2 - 21x - 33x + 99 = 0
or, 7x(x - 3) - 33(x - 3) = 0
or, (x - 3) (7x - 33) = 0
x = 3, 33/7
II. 4y^2 - 6y - 10y + 15 = 0
or, 2y(2y - 3) - 5(2y - 3) = 0
or, (2y - 3)(2y - 5) = 0
y = 3/2, 5/2
x>y

51). I. 5x^2 - 87x + 378 = 0


II. 3y^2 - 49y + 200 = 0
a) x>y
b) x<y
c) x≥y

www.ibpsguide.com | estore.ibpsguide.com | www.sscexamguide.com 18


500 Quadratic Equations for IBPS RRB/PO/Clerk

d) x≤y

52). I. 10x^2 - x - 24 = 0
II. Y^2 - 2y = 0
a) x = y or no relation can be established between ‘x’and ‘y’.
b) x<y
c) x≥y
d) x≤y

53). I. x2 + √5x – 10 = 0
II. 2y2 + 9√5y + 50 = 0
A) x > y
B) x < y
C) x ≥ y
D) x ≤ y
E) x = y or relation cannot be established

54). I. 3x2 – 23x + 40 = 0


II. 3y2 – 8y + 4 = 0
A) x > y
B) x < y
C) x ≥ y
D) x ≤ y
E) x = y or relation cannot be established

55). I. 6x2 + x – 2 = 0
II. 3y2 – 22y + 40 = 0
A) x > y
B) x < y
C) x ≥ y
D) x ≤ y
E) x = y or relation cannot be established

Directions (Q. 56-60) In the following questions, two equations I and II are given. You have to solve both the
equations.
Give Answer
a) If x > y
b)If x < y
c)If x ≥ y
d)If x ≤y
e)If x = y or the relationship cannot be established

56. I. 8x+y=10
II. 4x+2y=13

57 I. (x+3) (y+2)=12
II. 2xy+4x+5y=11

58. I. (3x-2)/y = (3x+6)/(y+16)


II.(x+2)/(y+4) = (x+5)/(Y+10)

59. I. x^2+20x+4=50-25x

www.ibpsguide.com | estore.ibpsguide.com | www.sscexamguide.com 19


500 Quadratic Equations for IBPS RRB/PO/Clerk

II. y^2-10y-24=0

60. I. (x^2-10x+16)/(x^2 -12x +24) = 2/3


II. y^2-y-20=0

51).A)
I. 5x^2 - 45x - 42x + 378 = 0
or, 5x(x - 9) - 42(x - 9) = 0
or, (5x - 42) (x - 9) = 0
x = 9 , 8.4
II. 3y^2 - 24y - 25y + 200 = 0
or, 3y(y - 8) - 25(y - 8) = 0
or, (y - 8) (3y - 25) = 0
y = 8, 8.3
x>y

52). A)
I. 10x^2 - 16x + 15x - 24 = 0
or, 2x(5x - 8) + 3(5x - 8) = 0
or, (2x + 3) (5x - 8) = 0
x = 1.63, -1.5

II. y^2 - 2y = 0
or, y(y - 2) = 0
y = 0, 2
ie no relationship exists between x and y
53). C)
x2 + √5x – 10 = 0
x2 + 2√5x – √5x – 10 = 0
Gives x = -2√5, √5
2y2 + 9√5y + 50 = 0
2y2 + 4√5y + 5√5y + 50 = 0
Gives y = -2√5, -5√5/2
Put all values on number line and analyze the relationship
-5√5/2….. -2√5….. √5

54). A)
3x2 – 23x + 40 = 0
3x2 – 15x – 8x + 40 = 0
Gives x = 5, 8/3
3y2 – 8y + 4 = 0
3y2 – 6y – 2y + 4 = 0
Gives y = 2/3, 2

www.ibpsguide.com | estore.ibpsguide.com | www.sscexamguide.com 20


500 Quadratic Equations for IBPS RRB/PO/Clerk

Put all values on number line and analyze the relationship


2/3….. 2….. 8/3….. 5

55). B)
6x2 + x – 2 = 0
6x2 + 4x – 3x – 2 = 0
Gives x = 1/2, -2/3
3y2 – 22y + 40 = 0
3y2 – 12y – 10y + 40 = 0
Gives y = 10/3, 4
Put all values on number line and analyze the relationship
-2/3…… 1/2…… 10/3….. 4

56) B
from both equation
x=7/12, y=16/3
y>x

57) E
xy+3y+2x +6 =12
2xy+6y+4x=12----(i)
2xy+5y+4x=11 ---- (ii)
From eq. (i) --- (ii)
Y=1
From eq. (i)
x=1
x=y

58) B
(3x-2)/y = (3x+6)/(y+16)
48x-8y = 32 ---- (i)
(x+2)/(y+4) = (x+5)/(y+10)
y = 2x ---- (i)
From Equation (i) & (ii)
x=1, y=2
y>x

59) E
From the given Equation
x=1, -46
& y=-2,/2
x#y

60) E
From 1st equation

www.ibpsguide.com | estore.ibpsguide.com | www.sscexamguide.com 21


500 Quadratic Equations for IBPS RRB/PO/Clerk

x^2-6x=0
x=0,6
From 2nd equation
(y+4) (y-5)
y=-4,5
x#y

Directions (Q. 61-65) In the following questions, two equations I and II are given. You have to solve both the
equations.
Give Answer
a) If x < y
b) If x > y
c) If x ≤ y
d) If x ≥ y
e) If x = y or the relationship cannot be established

61. I. 6x^2-49x+99=0
II. 5y^2+17y+14=0

62. I. 5x^2=19x-12
II. 5y^2+11y=12

63. I. x^3=(1331)^1/3
II. 2y^2-21y+55=0

64. I. 5x=7y+21
II. 11x+4y+109=0

65. I. 2x^2-11x+12=0
II. 2y^2-17y+36=0

Direction (Q.66-70): In each of these questions two equations numbered (I) and (II) are given. You have
to solve both the equations and give answer:
a) If x > y
b) If x ≥ y
c) If x < y
d) If x ≤ y
e) If x = y or no relationship can't be established between x and y.

66).I. x²- 4x - 621 = 0


II. y² - 56y + 783 = 0

67). I. 15x² - 34x + 15 = 0


II. 15y² + 22y + 8 = 0

68). I. x² = 121
II. y³ =1331

69). I. x² + 12x + 35 = 0
II. y² + 18y + 45 = 0

www.ibpsguide.com | estore.ibpsguide.com | www.sscexamguide.com 22


500 Quadratic Equations for IBPS RRB/PO/Clerk

70). I. 14x² -41x + 15=0


II. 56y² - 54y+ 10 = 0

61) B
6x^2-49x+99=0
(3x-11) (2x-9)=0
x=11/3, 9/2
5^2+17y+14=>
(5y+7) (y+2)=>
y=-2,-7/5
x>y

62) D
5x^2-19x+12=0
x=3,4/5
5y^2+11y=12
y = 4/5 ,-3

63) B
x=11
2y^2-21y+55= 0
(2y-11) (y-5)= 0
y=5,11/2
x>y

64) B
From given equation
x=-7
y=-8
x>Y

65) C
2x^2-11x+12=>
x=3/2,4
2y^2-17y+36=>
y=4,9/2

66). D)
I. x² - (27-23)x-621=0
x² - 27x - 23x - 621=0
x (x-27) +23 (x-27)=0
(x-27) (x+23)=0
x=27, x= -23
II. y² - (29+27) y+783 = 0
y² - 29y - 27y + 783 =0
y (y-29) -27(y-29)=0

www.ibpsguide.com | estore.ibpsguide.com | www.sscexamguide.com 23


500 Quadratic Equations for IBPS RRB/PO/Clerk

(y-29) (y-27) =0
y=29, y=27

67). A)
I. 15x²- (9+25)x + 15 =0
15x² - 9x - 25x + 15 = 0
3x (5x-3)-5 (5x-3)=0
(3x-5) (5x-3)=0

II. 15y² +(10+12)y + 8 = 0


15y² + 10y + 12y + 8 = 0
5y (3y+2) +4 (3y+2)=0
(3y+2) (5y+4)=0

68). D)
69). E) I. x² + (7+5)x + 35 = 0
x² + 7x + 5x + 35 =0
x (x+7) +5 (x+7)=0
(x+7) (x+5) = 0
x= -7, x=-5
II. y² + (15+3) +45=0
y² + 15y + 3y + 45=0
y (y+15)+3 (y+15)=0
(y+15) (y+3) = 0
y= -15, y=-3

70). E)
I. 14x² - (35+6)x +15=0
14x² -35x -6x+15=056
7x (2x-5)-3 (2x-5)=0
(2x-5) (7x-3)=0

II. 56y²- (40+14)y +10=0


56y² - 40y- 14y +10=0
8y(7y-5)-2(7y-50=0
(7y-5) (8y-2) =0

www.ibpsguide.com | estore.ibpsguide.com | www.sscexamguide.com 24


500 Quadratic Equations for IBPS RRB/PO/Clerk

Directions(71-85): In each question two equations are given, find x and y and give the answer:
a) x> y
b) x< y
c) x ≥ y
d) x ≤ y
e) x = y or relation can not be established.

71)a) 3x² - 22x + 7 = 0


b) y² - 15y + 56 = 0

72)a) 2x² - 17x + 36 = 0


b) 2y² - 19y + 44 = 0

73) x - √169 = 0
b) y² - 169 = 0

74) a) 3x² + 20x + 25 = 0


b) 3y² + 14y + 8 = 0

75) a) 3x² + 5x + 2 = 0
b) 3y² + 18y + 24 = 0

76)a) 6x² + 31x + 35 = 0


b) 2y² + 3y + 1 = 0

77) a) 2x² + 9x + 10 = 0
b) 4y² + 28y + 45 = 0

78) a) 15x² - 11x - 12 = 0


b) 20y² - 49y + 30 = 0

79) a) 2x² - 15 = 7x
b) 17y = -7 - 6y²

80) a) 3x² - 19x - 14 = 0


b) 2y² +15y + 13 = 0

81) a) (x³ -13x + 12)/(x-1) = 0


b) (y³ + 5y² - 2y - 24)/(y-2) = 0

82)a) y = 2x + 1
b) 2y = 3x - 1

83) a) 9x² - 29x + 22 = 0


b) y² - 7y + 12 = 0
84) a) 3x² - 4x - 32 = 0

www.ibpsguide.com | estore.ibpsguide.com | www.sscexamguide.com 25


500 Quadratic Equations for IBPS RRB/PO/Clerk

b) 12y² - 109y + 247 = 0

85) a) 4x + 7y = 42
b) 3x - 11y = -1

71)D
a) 3x² - 21x - x + 7 = 0
3x(x-7) -1(x-7) = 0
(3x-1)(x-7) = 0
x = 7, 1/3.
b) y² -7y - 8y + 56 = 0
y(y-7) - 8(y-7) = 0
(y-8)(y-7) = 0
y = 8, 7.
Thus, y≥x.

72) E
a) 2x² - 8x - 9x + 36 = 0
2x(x-4) - 9(x-4) = 0
(2x-9)(x-4) = 0
x = 9/2, 4.
b) 2y² - 8y - 11y + 44 = 0
2y(y-4) - 11(y-4) = 0
(2y-11)(y-4) = 0
y = 11/2, 4.
Thus, no relation.

73) C
a) x = 13
b) y = ±√169 = ±13.
Thus, x≥y.

74) E
a) 3x² + 15x + 5x + 25 = 0
3x(x+5) + 5(x+5) = 0
(3x+5)(x+5) = 0
x = -5/3, -5.
b) 3y² + 12y + 2y + 8 = 0
3y(y+4) + 2(y+4) = 0
(3y+2)(y+4) = 0
y = -2/3, -4.
Thus, no relation.

75) A
a) 3x² + 3x + 2x + 2 = 0
3x(x+1) + 2(x+1) = 0

www.ibpsguide.com | estore.ibpsguide.com | www.sscexamguide.com 26


500 Quadratic Equations for IBPS RRB/PO/Clerk

(3x+2)(x+1) = 0
x = -2/3, -1.
b) 3(y² + 6y + 8) = 0
y² + 6y + 8 = 0
y² + 4y + 2y + 8 = 0
y(y+4) + 2(y+4) = 0
y = -2, -4.
Thus, x>y.

76) B
a) 6x² + 21x + 10x + 35 = 0
3x(2x+7) + 5(2x+7) = 0
(3x+5)(2x+7) = 0
x = -5/3, -7/2.
b) 2y² + 2y + y + 1 = 0
2y(y+1) + 1(y+1) = 0
(2y+1)(y+1) = 0
y = -1/2, -1.
Thus, x<y.

77) C
a) 2x² + 4x + 5x + 10 = 0
2x(x+2) + 5(x+2) = 0
(2x+5)(x+2) = 0
x = -5/2, -2.
b) 4y² +18y + 10y + 45 = 0
2y(2y+9) + 5(2y+9) = 0
(2y+5)(2y+9) = 0
y = -5/2, -9/2.
Thus, x≥y.

78) E
a) 15x² - 20x + 9x - 12 = 0
5x(3x-4) +3(3x-4) = 0
(5x+3)(3x-4) = 0
x = -3/5, 4/3.
b) 20y² -25y - 24y + 30 = 0
5y(4y-5) -6(4y-5) = 0
(5y-6)(4y-5) = 0
y = 6/5, 5/4.
Thus, no relation.

79) E
a) 2x² - 10x + 3x - 15 = 0
2x(x-5) +3(x-5) = 0

www.ibpsguide.com | estore.ibpsguide.com | www.sscexamguide.com 27


500 Quadratic Equations for IBPS RRB/PO/Clerk

(2x+3)(x-5) = 0
x = -3/2, 5.
b) 6y² + 3y + 14y + 7 = 0
3y(2y+1) +7(2y+1) = 0
(3y+7)(2y+1) = 0
y = -7/3, -1/2.
Thus, no relation.

80) A
a) 3x² - 21x - 2x - 14 = 0
3x(x-7) -2(x-7) = 0
(3x-2)(x-7)
x = 2/3, 7.
b) 2y² +2y +13y + 13 = 0
2y(y+1) +13(y+1) = 0
(2y+13)(y+1) = 0
y = -13/2, -1.
Thus, x>y.
81) C
a) (x³ - x - 12x + 12)/(x-1) = 0
[x(x²-1) -12(x-1)]/(x-1) = 0
[(x-1)(x+1)(x-12)]/(x-1) = 0
x² + x - 12 = 0
Solving, x = 3, -4
b) y³ + 5y² - 2y - 24/(y-2) = 0
y³ + 7y² - 2y² -14y + 12y - 24/(y-2) = 0
(y-2)(y² + 7y + 12)/(y-2) = 0
Solving, y = -4, -3.
Thus, x≥y.

82) A
Sol. Solving the two using substitution method,
x = -3
y = -5
Thus, x>y.

83) B
a) 9x² - 18x - 11x + 22 = 0
9x(x-2) - 11(x-2) = 0
(9x-11)(x-2) = 0
x = 11/9, 2.
b) y² - 4y - 3y + 12 = 0
y(y-4) - 3(y-4) = 0
(y-4)(y-3) = 0
y = 3, 4.

www.ibpsguide.com | estore.ibpsguide.com | www.sscexamguide.com 28


500 Quadratic Equations for IBPS RRB/PO/Clerk

Thus, x<y.

84) B
a) 3x² - 12x + 8x - 32 = 0
3x(x-4) + 8(x-4) = 0
(3x+8)(x-4) = 0
x = -8/3, 4.
b) 12y² - 52y - 57y + 247 = 0
4y(3y - 13) - 19(y - 13) = 0
(4y-19)(3y-13) = 0
y = 19/4, 13/3.
Thus, x<y.

85) A
Sol. Solving the two equations using the substitution method,
x=7
y=2
Thus, x>y.

86). I. x2 – 4x + 3 = 0
II. y2 – 13y + 40 = 0
a. x > y
b. x ≥ y
c. x < y
d. x ≤ y
e. x = y or Cannot be determined.

87).I. 6x2 – 7x + 2 = 0
II. 6y2 + y – 2 = 0
a. x > y
b. x ≥ y
c. x < y
d. x ≤ y
e. x = y or Cannot be determined.

88). I. 4x4 – 5x2 + 1 = 0


II. 2y2 + 11y + 14 = 0
a. x > y
b. x ≥ y
c. x < y
d. x ≤ y
e. x = y or Cannot be determined.

89). I. 6x2 + 7x + 2 = 0
II. y2 – y – 2 = 0
a. x > y
b. x ≥ y
c. x < y
d. x ≤ y

www.ibpsguide.com | estore.ibpsguide.com | www.sscexamguide.com 29


500 Quadratic Equations for IBPS RRB/PO/Clerk

e. x = y or Cannot be determined.

90). I. x2 – 3x + 2 = 0
II. y2 -5y + 6 = 0
a. x > y
b. x ≥ y
c. x < y
d. x ≤ y
e. x = y or Cannot be determined.
86. (c)
x2 – 4x + 3 = 0
(x – 1)(x – 3) = 0
x = 1 or 3
y2 – 13y + 40 = 0
(y – 5)(y – 8) = 0
y = 5 or 8
Largest value of x = 3 < 5 = Least value of y
So, x < y

87. (b)
6x2 – 7x + 2 = 0
(2x – 1)(3x – 2) = 0
x = 1/2 or 2/3
6y2 + y – 2 = 0
(2y – 1)(3y + 2) = 0
y = 1/2 or -2/3
Largest value of y = 1/2 = Least value of x
So, x ≥ y

88. (a)
4x4 – 5x2 + 1 = 0
(x2 – 1)(42 – 1) = 0
x = +1, -1, 1/2 or -1/2
2y2 + 11y + 14 = 0
(y + 2)(2y + 7) = 0
y = -2, -7/2
Least value of x = -1 > -2 = Largest value of y
So, x > y

89. (e)
6x2 + 7x + 2 = 0
(3x + 2)(2x + 1) = 0
x = -2/3 or -1/2
y2 – y – 2 = 0
(y – 2)(y + 1) = 0
y = -1 or + 2

www.ibpsguide.com | estore.ibpsguide.com | www.sscexamguide.com 30


500 Quadratic Equations for IBPS RRB/PO/Clerk

If x = -1/2, y = -1, then x > y


If x = -1/2, y = +2, then x < y
So, we cant determine the relationship.

90. (d)
x2 – 3x + 2 = 0
(x – 1)(x – 2) = 0
x = 1 or 2
y2 -5y + 6 = 0
(y – 2)(y – 3) = 0
y = 2 or 3
Largest value of x = 2 = Least value of y
So x ≤ y

Direction (Q. 91 - 95): In Each of the following questions, two equations are given. You have to solve
these questions and find out the values of x and y .
91. 16x² + 20x + 6 = 0 and 10y² + 38y + 24 = 0
A. x > y
B. x ≤ y
C. Relation can not be established or x = y
D. x ≥ y
E. x < y

92. 18x² + 18x + 4 = 0 and 12y² + 29y + 14 = 0


A. x ≥ y
B. x > y
C. x < y
D. x ≤ y
E. Relation can not be established or x = y

93. 8x² + 6x = 5 and 12y² - 22y + 8 = 0


A. x < y
B. Relation can not be established or x = y
C. x ≥ y
D. x > y
E. x ≤ y

94. 17x² + 48x - 9 = 0 and 13y² = 32y - 21


A. Relation can not be established or x = y
B. x > y
C. x < y
D. x ≥ y
E. x ≤ y

95. 821x² - 757x² = 256 and √196 y³ - 12y³ = 16


A. x ≤ y
B. x > y
C. Relation can not be established or x = y
D. x < y
E. x ≥ y

www.ibpsguide.com | estore.ibpsguide.com | www.sscexamguide.com 31


500 Quadratic Equations for IBPS RRB/PO/Clerk

Directions ( Q. 96 - 100 ) In the following question, two equations are given. You have to solve both the
equations and choose the correct answer out of five alternatives.

96. 4x + 3y = √1600 and 6x - 5y = √484


A. x < y
B. x ≥ y
C. x > y
D. x ≤ y
E. Relation can not be extablished or x = y

97. 2x² - ( 4 - √13 )x + 2√13 = 0and 10y² - ( 18 + 5√13 )y + 9√13 = 0


A. x ≤ y
B. Relation can not be Established or x = y
C. x > y
D. x < y
E. x ≥ y

98. (6x² + 17 ) - ( 3x² + 20 ) = 0 and (5y² - 12 ) - ( 9y² - 16 ) = 0


A. x > y
B. x ≤ y
C. x < y
D. Relation can not be extablished or x = y
E. x ≥ y

99. √(169) x + √289 = 134 and √(361) y² - 270 = 1629


A. Relation can not be established or x = y
B. x ≥ y
C. x < y
D. x ≤ y
E. x > y

100. 63x - 194√x + 143 = 0 and 99y - 255√y + 150 = 0


A. Relation can not be established or x = y
B. x < y
C. x ≤ y
D. x > y
E. x ≥ y
91.(A)
16x² + 20x + 6 = 0
or, 8x² + 10x + 3 = 0
or, (4x + 3 ) ( 2x + 1) = 0
Therefore, x = -3/4 = -0.75 or x = - 1/2 = - 0.5
AND
10y² + 38y + 24 = 0

www.ibpsguide.com | estore.ibpsguide.com | www.sscexamguide.com 32


500 Quadratic Equations for IBPS RRB/PO/Clerk

or, 5y² + 19y + 12 = 0


or, ( y + 3) (5y + 4 ) = 0
Therefore, y = - 3 or y = - 4/5 = -0.8
Hence x > y

92.(A)
18x² + 18x + 4 = 0
or, 9x² + 9x + 2 = 0
or, ( 3x + 2 ) ( 3x + 1) = 0
Therefore, x = - 2/3 = -0.67 or x = - 1/3 = -0.33
AND
12y² + 29y + 14 = 0
or, ( 3y + 2 ) ( 4y + 7 ) = 0
Therefore, y = - 2/3 = -0.67 or y = -7/4 = -1.75
Hence x ≥ y

93.(E)
8x² + 6x - 5 = 0
or, ( 4x + 5 ) ( 2x - 1) = 0
Therefore, x = - 5/4 = -1.25 or x = 1/2 = 0.5
AND
12y² - 22y + 8 = 0
or, 6y² -11y + 4 = 0
or, (2y -1 ) ( 3y - 4 ) = 0
Therefore, y = 1/2 = 0.5 or y = 4/3 = 1.33
Hence x ≤ y

94.(C)
17x² + 48x - 9 = 0
or, (x + 3) ( 17x - 3 ) = 0
Therefore, x = -3 or x = 3/17 = 0.18
AND
13y² - 32y + 12 = 0
or, ( y -2 ) ( 13y - 6) = 0
Therefore, y = 2 or y = 6/13 = 0.46
Hence x < y

95.(A)
64x² = 256
or, x² = 4
or, x = ±2
AND
14y³ - 12y³ = 16
or, 2y³ = 16
or, y³ = 8

www.ibpsguide.com | estore.ibpsguide.com | www.sscexamguide.com 33


500 Quadratic Equations for IBPS RRB/PO/Clerk

or, y = 2
Hence x ≤ y

96.(C)
4x + 3y = 40 ............................ (i)
6x - 5y = 22 ..............................(ii)
Solving the two equations we have,
x=7
y = 4 Hence x > y

97.(E)
2x² - 4x - √13 x + 2√13 = 0
or, 2x ( x - 2 ) - √13 ( x - 2 ) = 0
or, (x - 2 ) ( 2x - √13 ) = 0
Therefore, x = 2 or x = √13 /2 = 3.6 /2 = 1.8
AND
10y² - 18y - 5√13y + 9√13 = 0
or, 2y ( 5y - 9 ) - √13 (5y - 9 ) = 0
or, ( 5y - 9 ) ( 2y - √13 ) = 0
Therefore, y = 9/5 = 1.8 or y = √13 / 2 = 1.8
Hence x ≥ y

98.(D)
6x² + 17 - 3x² - 20 = 0
or, 3x² = 3
Therefore, x = ± 1
AND
5y² - 12 - 9y² + 16 = 0
or, 4y² = 4
Therefore, y = ± 1
Hence x = y

99.(B)
13x + 17 = 134
or, x = 117/13 = 9
AND
√(361) y² - 270 = 1629
or, 19y² = 1629 + 270 = 1539
or, y² = 1539/19 = 81
or, y = ± 9
Hence x ≥ y

100.(A)
63x - 194√x + 143 = 0
or, 63(√x)² - 117√x - 77√x + 143 = 0

www.ibpsguide.com | estore.ibpsguide.com | www.sscexamguide.com 34


500 Quadratic Equations for IBPS RRB/PO/Clerk

or, ( 7√x - 13 ) ( 9√x - 11 ) = 0


Therefore, x = 169/49 = 3.45 or x = 121/81 = 1.49
AND
99y - 225√y + 150 = 0
or, 99(√y)² - 90√y - 165√y + 150
or, ( 11√y - 10 ) ( 9√y - 15 ) = 0
Therefore, y = 100/ 121 = 0.83 or y = 225/81 = 2.8
Hence, Relation can not be established ( Since it is not possible to get any conclusion)

101). x² – 11x + 28 = 0
y² – 14y + 48 = 0
A. X > Y
B. X < Y
C. X ≥ Y
D. X ≤ Y
E. X = Y or relation cannot be established

102). x² – 31x + 228 = 0


y² – 21y + 108 = 0
A. X > Y
B. X < Y
C. X ≥ Y
D. X ≤ Y
E. X = Y or relation cannot be established

103). x² + 31x + 234 = 0


y² + 21y + 104 = 0
A. X > Y
B. X < Y
C. X ≥ Y
D. X ≤ Y
E. X = Y or relation cannot be established

104). x² – 20x + 84 = 0
y² – 24y + 135 = 0
A. X > Y
B. X < Y
C. X ≥ Y
D. X ≤ Y
E. X = Y or relation cannot be established

105). (x – 14)² = 0
y² = 196
A. X > Y
B. X < Y
C. X ≥ Y
D. X ≤ Y
E. X = Y or relation cannot be established

www.ibpsguide.com | estore.ibpsguide.com | www.sscexamguide.com 35


500 Quadratic Equations for IBPS RRB/PO/Clerk

106). x² – 43x + 450 = 0


y² – 32y + 255 = 0
A. X > Y
B. X < Y
C. X ≥ Y
D. X ≤ Y
E. X = Y or relation cannot be established

107). x² – 27x + 182 = 0


y² – 36y + 323 = 0
A. X > Y
B. X < Y
C. X ≥ Y
D. X ≤ Y
E. X = Y or relation cannot be established

108). x² – 37x + 322 = 0


y² – 22y + 120 = 0
A. X > Y
B. X < Y
C. X ≥ Y
D. X ≤ Y
E. X = Y or relation cannot be established

109). x² = 81
y² – 30y + 225 = 0
A. X > Y
B. X < Y
C. X ≥ Y
D. X ≤ Y
E. X = Y or relation cannot be established

110). x² – 30x + 221 = 0


y² – 33y + 270 = 0
A. X > Y
B. X < Y
C. X ≥ Y
D. X ≤ Y
E. X = Y or relation cannot be established
Solutions
101).E.
X = Y or relation cannot be established
Explanation:
x² – 11x + 28 = 0
x = 7, 4
y² – 14y + 48 = 0
y = 6, 8

102) C.
X≥Y

www.ibpsguide.com | estore.ibpsguide.com | www.sscexamguide.com 36


500 Quadratic Equations for IBPS RRB/PO/Clerk

Explanation:
x² – 31x + 228 = 0
x = 12, 19
y² – 21y + 108 = 0
y = 12, 9

103). D.
X≤Y
Explanation:
x² + 31x + 234 = 0
x = -13, -18
y² + 21y + 104 = 0
y = -13, -8

104). E.
X = Y or relation cannot be established
Explanation:
x² – 20x + 84 = 0
x = 14, 6
y² – 24y + 135 = 0
y = 15, 9

105). C.
X≥Y
Explanation:
x² – 28x + 196 = 0
x = 14, 14
y² = 196
y = ±14

106). A.
X>Y
Explanation:
x² – 43x + 450 = 0
x = 25, 18
y² – 32y + 255 = 0
y = 17, 15

107). B.
X<Y
Explanation:
x² – 27x + 182 = 0
x = 14, 13
y² – 36y + 323 = 0
y = 17, 19

www.ibpsguide.com | estore.ibpsguide.com | www.sscexamguide.com 37


500 Quadratic Equations for IBPS RRB/PO/Clerk

108). A.
X>Y
Explanation:
x² – 37x + 322 = 0
x = 23, 14
y² – 22y + 120 = 0
y = 10, 12

109). B.
X<Y
Explanation:
x² = 81
x = 9, – 9
y² – 30y + 225 = 0
y = 15, 15

110). E.
X = Y or relation cannot be established
Explanation:
x² – 30x + 221 = 0
x = 13, 17
y² – 33y + 270 = 0
y = 15, 18

Solutions

www.ibpsguide.com | estore.ibpsguide.com | www.sscexamguide.com 38


500 Quadratic Equations for IBPS RRB/PO/Clerk

www.ibpsguide.com | estore.ibpsguide.com | www.sscexamguide.com 39


500 Quadratic Equations for IBPS RRB/PO/Clerk

Directions (Q. 121 – 130): In each of these questions, two equations (I) and (II) are given. You have to
solve both the equations and give answer
1) if x > y
2) if x ≥ y
3) if x < y
4) if x ≤ y
5) if x = y or relationship between x and y cannot be established
121) I. 7 x² – 9 x + 2 = 0
II. y² – 4 y + 3 = 0

122) I. x² = 64
II. 2 y² + 25 y + 72 = 0

123) I. x² + x – 20 = 0
II. 2 y² – 19 y + 45 = 0

124) I. 7 x + 3 y = 26
II. 2 x + 17 y = – 41

125) I. 3 x² – 20 x + 33 = 0

www.ibpsguide.com | estore.ibpsguide.com | www.sscexamguide.com 40


500 Quadratic Equations for IBPS RRB/PO/Clerk

II. 2 y² – 11 y + 15 = 0

126) I. 4 x² – 43 x + 105 = 0
II. 7 y² – 29 y + 30 = 0

127) I. x² + 13 x + 40 = 0
II. y² + 7 y + 10 = 0

128) I. x = ³√2197
II. 2 y² – 54 y + 364 = 0

129) I. 5 x² – 27 x + 36 = 0
II. y² – 3 y + 2 – 0

130) I. 13 x – 8 y + 81 = 0
II. 15 x + 5 y + 65 = 0

121). D) ;
I. 7x2 – 7x – 2x + 2 = 0
or, 7x(x – 1) –2(x – 1) = 0
(7x – 2) (x – 1) = 0
or, x = 2/7 , 1
II. y 2 – y – 3y + 3 = 0
or, y(y – 1) –3(y – 1) = 0
or, (y – 3) (y – 1) = 0
y = 1, 3
x≤y

122). E);
I. x2= 64
x = ±8
2y2 + 9y + 16y + 72 = 0
or, y(2y + 9) + 8(2y + 9) = 0
or, (y + 8) (2y + 9) = 0
y = –8,-9/2
if x = y or relationship between x and y cannot be established

123). C);
I. x2 + x – 20 = 0
or, x2+ 5x – 4x – 20 = 0
or, x(x + 5) –4(x + 5) = 0
or, (x – 4) (x + 5) = 0
x = 4, – 5
II. 2y2 – 10y – 9y + 45 = 0
or, 2y(y – 5) –9(y – 5) = 0
or, (y – 5) (2y – 9) = 0
y = 5, 9/2
x<y

www.ibpsguide.com | estore.ibpsguide.com | www.sscexamguide.com 41


500 Quadratic Equations for IBPS RRB/PO/Clerk

124). A);
Eqn (I) × 2 14 x+ 6y= 52
Eqn (II) × 7  14 x +119 y =287
y = –3 and x = 5, ie x > y

125). B);
I. 3x2 – 9x – 11x + 33 = 0
or, 3x(x – 3) – 11(x – 3) = 0
or, (3x – 11) (x – 3) = 0
x =11/ 3, 3
II. 2y2 – 6y – 5y + 15 = 0
or, 2y(y – 3) –5(y – 3) = 0
or, (y – 3) (2y – 5) = 0
y = 3, 5/2
x≥y

126). A);
I. 4x2– 28x – 15x + 105 = 0
or, 4x(x – 7) –15(x – 7) = 0
or, (x – 7) (4x – 15) = 0
x = 7, 15/4
II. 7y2 – 14y – 15y + 30 = 0
or, 7y(y – 2) – 15(y – 2) = 0
or, (y – 2) (7y – 15) = 0
y = 2, 15/7
x>y

127). D)
I. x2 + 8x + 5x + 40 = 0
or, x(x + 8) + 5(x + 8) = 0
or, (x + 5) (x + 8) = 0
x = –5, – 8
II. y2 + 2y + 5y + 10 = 0
or, y(y + 2) +5(y + 2) = 0
or, (y + 2) (y + 5) = 0
y = –2, –5
x≤y

128). D);
I. x=3√ 2197
x = 13
II. 2y2 – 28y – 26y + 364 = 0
or, 2y(y – 14) –26(y – 14) = 0

www.ibpsguide.com | estore.ibpsguide.com | www.sscexamguide.com 42


500 Quadratic Equations for IBPS RRB/PO/Clerk

or, (2y – 26) (y – 14) = 0


y = 14, 13
x≤y

129). A);
I. 5x2 – 15x – 12x + 36 = 0
or, 5x(x – 3) – 12(x – 3) = 0
or, (5x – 12) (x – 3) = 0
x = 12/5 , 3
II. y2 – y – 2y + 2 = 0
or, y(y – 1) –2(y – 1) = 0
or, (y – 1) (y – 2) = 0
y = 1, 2
x>y

130). C);
eqn (I) × 5  (65x- 40y +405= 0)+ eqn (II) × 8(120x +40y+ 520= 0)
= 185x = -925
x = -5
y = (13x + 81)/8  (-65+81)/8  16/8 = 2
x<y

Directions (131-135): In each of these questions, two equations numbered I and II with variables x and y
are given. You have to solve both the equations to find the value of x and y.
131) I. 12x2 – 59x + 72 = 0
II. 8y2 – 58y + 99 = 0
(a) if x > y
(b) if x >= y
(c) if x < y
(d) if x <= y
(e) if x = y or relationship between x and y cannot be determined.

132) I. x2 + x – 20 = 0
II. y2 + 13y + 40 = 0
(a) if x > y
(b) if x >= y
(c) if x < y
(d) if x <= y
(e) if x = y or relationship between x and y cannot be determined.

133) I. x2 + 12x + 36 = 0
II. 5y2 – 2 √15y + 3 = 0
(a) if x > y
(b) if x >= y
(c) if x < y
(d) if x <= y
(e) if x = y or relationship between x and y cannot be determined.

www.ibpsguide.com | estore.ibpsguide.com | www.sscexamguide.com 43


500 Quadratic Equations for IBPS RRB/PO/Clerk

134) I. 10x2 – 29x – 21 = 0


II. y2 + 13y – 68 = 0
(a) if x > y
(b) if x >= y
(c) if x < y
(d) if x <= y
(e) if x = y or relationship between x and y cannot be determined.

135) I. 72x2 – 163x + 88 = 0


II. 56y2 – 187y + 156 = 0
(a) if x > y
(b) if x >= y
(c) if x < y
(d) if x <= y
(e) if x = y or relationship between x and y cannot be determined.

Directions (Q. 136-140): Two equations (I) and (II) are given in each question. On the basis of these
equations you have to decide the relation between ‘x’and ‘y’and give answer.
(1) if x > y
(2) if x < y
(3) if x≥ y
(4) if x ≤y
(5) if x = y or no relation can be established between ‘x’and ‘y’.
136). I. 6x2 - 19x + 15 = 0
II. 10y2 - 29y + 21 = 0

137). I. 12x2 + 11x - 56 = 0


II. 4y2 - 15y + 14 = 0

138). I. 3x2+ 13x + 12 = 0


II. y2 + 9y + 20 = 0

139). I. 8x2 - 15x + 7 = 0


II. 2y2 - 7y + 6 = 0

140). I. 7x - 3y = 13
II. 5x + 4y = 40

131) Ans.(c)
Sol. I. 12x2 – 32x – 27x + 72 = 0
(3x – 8) (4x – 9) = 0
x = 8/3, 9/4
II. 8y2 – 22y – 36y + 99 = 0
(2y – 9) (4y – 11) = 0
y = 9/2, 11/4
x<y

132) Ans.(b)
Sol. I. x2 + 5x – 4x – 20 = 0

www.ibpsguide.com | estore.ibpsguide.com | www.sscexamguide.com 44


500 Quadratic Equations for IBPS RRB/PO/Clerk

(x – 4) (x + 5) = 0
x = 4, –5
II. y2 + 5y + 8y + 40 = 0
(y + 8) (y + 5) = 0
y = –8, – 5
x >= y

133) Ans.(c)
Sol. I. (x + 6)2 = 0
x = –6
II. 2
(√ 5y – √ 3)2 = 0
y = √ 3/ √ 5
x<y

134) Ans.(e)
Sol. I. 10x2 – 35x + 6x – 21 = 0
(5x + 3) (2x – 7) = 0
x = –3/5, 7/2
II. y2 + 17y – 4y – 68 = 0
(y – 4) (y + 17) = 0
y = 4, – 17
No relationship between x and y exists.

135) Ans.(c)
Sol. I. 72x2 – 64x – 99x + 88 = 0
(8x – 11) (9x – 8) = 0
x = 11/8, 8/9
II. 56y2 – 91y – 96y + 156 = 0
(7y – 12) (8y – 13) = 0
y = 12/7, 13/8
x<y

136). C);
I. 6x2- 9x - 10x + 15 = 0
or, 3x(2x - 3) - 5(2x - 3) = 0
or, (3x - 5) (2x - 3) = 0
x =5/3, 3/2
II. 10y2- 15y - 14y + 21 = 0
or, 5y(2y - 3) - 7(2y - 3) = 0
or, (5y - 7) (2y - 3) = 0
y =7/5, 3/2
x≥y

137). D);

www.ibpsguide.com | estore.ibpsguide.com | www.sscexamguide.com 45


500 Quadratic Equations for IBPS RRB/PO/Clerk

I. 12x2+ 32x - 21x - 56 = 0


or, 4x(3x + 8) - 7(3x + 8) = 0
or, (4x - 7) (3x + 8) = 0
x = (7/4), (-8/3)
II. 4y2- 8y - 7y + 14 = 0
or, 4y(y - 2) - 7(y - 2) = 0
or, (4y - 7) (y - 2) = 0
y = 2, 7/4
x ≤y

138). A);
I. 3x2+ 9x + 4x + 12 = 0
or, 3x(x + 3) + 4(x + 3) = 0
or, (3x + 4) (x + 3) = 0
x = -4/3, -3
II. y2+ 5y + 4y + 20 = 0
or, y(y + 5) + 4(y + 5) = 0
or, (y + 4) (y + 5) = 0
y = - 4, - 5
x>y

139). B);
I. 8x2- 8x - 7x + 7 = 0
or, 8x(x - 1) -7(x - 1) = 0
or, (8x - 7) (x - 1) = 0
x = 7/8, 1
II. 2y2- 4y - 3y + 6 = 0
or, 2y(y - 2) -3(y - 2) = 0
or, (y - 2) (2y - 3) = 0
y = 2, 3/2

140). B);
Eqn (I) × 4 + Eqn (II) × 3
28x - 12y = 52
15x + 12y = 120
43x = 172
x = 4 and y = 5
x<y

Directions (Q. 141-145): In the following questions, two equations numbered I and II are given. You
have to solve both the equations and give answer
(1) if x > y
(2) if x < y
(3) if x≥ y
(4) if x ≤y

www.ibpsguide.com | estore.ibpsguide.com | www.sscexamguide.com 46


500 Quadratic Equations for IBPS RRB/PO/Clerk

(5) if x = y or no relation can be established between ‘x’and ‘y’.


141). I. 2x2 - 11x + 15 = 0
II. 21y2 - 23y + 6 = 0

142). I. 5x2 - 16x + 11= 0


II. 5y2 - 3y - 2 = 0

143). I. x2 + 11x + 28 = 0
II. 2y2 + 13y + 20 = 0

144). I. 6x2 + 29x + 35 = 0


II. 3y2 + 19y + 30 = 0

145). I. 2x + 5y = 6
II. 5x + 11y = 9

Directions (Q. Nos. 146-150) In the following questions two equations numbered I and II are given. You
have to solve both the equations and—Give answer
(1) if x > y
(2) if x < y
(3) if x≥ y
(4) if x ≤y
(5) if x = y or no relation can be established between ‘x’and ‘y’.
146. I. √1225x +√4900 = 0
II. (81)^1/4 y + (343)^1/3 = 0

147. I. 18/x2 +6/x – 12/x2 = 8/x2


II. y^2 + 9.68 + 5.64 = 16.95

148. I.[(2)^5 + (11)^3] / 6 =x^3


II. 4y3 = - (589÷4) + 5y3

149. I. 12x2 + 11x + 12 = 10x2+22x


II. 13y2 - 18y + 3 = 9y2 - 10y

150. I. (x^(7/5) ÷ 9) = 169 ÷ x^(3/5)


II. y^(1/4) × y^(1/4) × 7 = 273 ÷ y^(1/2)

141). A);
I. 2x2 - 6x - 5x + 15 = 0
or,2x(x - 3) - 5(x - 3) = 0
or, (2x - 5) (x - 3) = 0
x = 3, 5/2
II. 21y2- 14y - 9y + 6 = 0
or,7y(3y - 2) - 3 (3y - 2) = 0
or,(7y - 3)(3y - 2) = 0
y = 3/7, 2/3
x>y

www.ibpsguide.com | estore.ibpsguide.com | www.sscexamguide.com 47


500 Quadratic Equations for IBPS RRB/PO/Clerk

142). C);
I. 5x2 - 5x - 11x + 11 = 0
or,5x(x - 1) - 11(x - 1) = 0
or,(x - 1) (5x - 11) = 0
x = 1,11/5
II. 5y2- 5y + 2y - 2 = 0
or, 5y (y - 1) + 2(y - 1) = 0
or, (5y + 2)(y - 1) = 0
y = 1, -2/5
x≥y

143). D);
I. x2+ 4x + 7x + 28 = 0
or, x(x + 4) +7(x + 7) = 0
or, (x + 4) (x + 7) = 0
x = - 4, - 7
II. 2y2+ 8y + 5y + 20 = 0
or, 2y(y + 4) + 5(y + 4) = 0
or, (y + 4) (2y + 5) = 0
y = -4,-5/2
x≤y

144). A);
I. 6x2+ 15x + 14x + 35 = 0
or, 3x(2x + 5) + 7(2x + 5) = 0
or,(3x + 7) (2x + 5) = 0
x=-7/3, -5/2
II. 3y2+ 9y + 10y + 30 = 0
or, 3y(y + 3) +10(y + 3) = 0
or,(3y + 10) (y + 3) = 0
y = -3, -10/3
x>y

145). B);
eqn(I) × 5 - eqn(II) × 2
(10x + 25y = 30) – (10x ± 22y = 18) =
3y = 12
y = 4 and x = -7
y>x

146). A);
I. √1225x + √4900 = 0
or, 35x + 70 = 0 or, x =-70/35 = -2

www.ibpsguide.com | estore.ibpsguide.com | www.sscexamguide.com 48


500 Quadratic Equations for IBPS RRB/PO/Clerk

II. 3y + 7 = 0 or y = (-7/3)
:. x > y

147). E);
I. (18 +6x- 12)/x^2 = 8/x^2
or, x =1/3 = 0.33
II. y2 = 16.95 - 9.68 - 5.64 = 1.63
y = ±1.277
148). A); I. x^3 = (32 + 1331)/6
II. 5y3 - 4y3 = 589/4
or y3 = 589/4
x>y

149). C);
I. 2x2 - 11x + 12 = 0
or, x = 4,3/2
II. 4y2- 8y + 3 = 0
Y=3/2, 1/2
x>y

150). D);
I.x^(7/5)÷9 = 169÷x^(3/5)
x^(7/5) ×x^(3/5) = 169×9
x^2 = 1521
x = ± 39
II.y^(1/4)×y^(1/4)×y^(1/2) = 273/7
or, y^[(1/4)+^(1/4)+1/2)] = 39
or, y = 39
x≤y

151). 5x + 2y = 31
3x + 7y = 36
A. X > Y
B. X < Y
C. X ≥ Y
D. X ≤ Y
E. X = Y or relation cannot be established

152). x2 – x – √3x + √3= 0


y2 – 3y + 2 = 0
A. X > Y
B. X < Y
C. X ≥ Y
D. X ≤ Y

www.ibpsguide.com | estore.ibpsguide.com | www.sscexamguide.com 49


500 Quadratic Equations for IBPS RRB/PO/Clerk

E. X = Y or relation cannot be established

153). [48 / x4/7] – [12 / x4/7] = x10/7


y³ + 783 = 999
A. X > Y
B. X < Y
C. X ≥ Y
D. X ≤ Y
E. X = Y or relation cannot be established

154). 172 + 144 ÷ 18 = x


262 – 18 * 21 = y
A. X > Y
B. X < Y
C. X ≥ Y
D. X ≤ Y
E. X = Y or relation cannot be established

155). 5/7 – 5/21 = √x/42


√y/4 + √y/16 = 250/√y
A. X > Y
B. X < Y
C. X ≥ Y
D. X ≤ Y
E. X = Y or relation cannot be established

156). 9/√x + 19/√x = √x


y5 – [(28)11/2 /√y] = 0
A. X > Y
B. X < Y
C. X ≥ Y
D. X ≤ Y
E. X = Y or relation cannot be established

157). 12/√x – 23/√x = 5√x


√y/12 – 5√y/12 = 1/√y
A. X > Y
B. X < Y
C. X ≥ Y
D. X ≤ Y
E. X = Y or relation cannot be established

158). 7x + 6y + 4z = 122
4x + 5y + 3z = 88
9x + 2y + z = 78
A. X < Y = Z
B. X ≤ Y < Z
C. X < Y > Z
D. X = Y > Z
E. X = Y = Z or relation cannot be established

www.ibpsguide.com | estore.ibpsguide.com | www.sscexamguide.com 50


500 Quadratic Equations for IBPS RRB/PO/Clerk

159). (x+y)³ = 1331


x–y+z=0
xy = 28
A. X < Y = Z
B. X ≤ Y < Z
C. X < Y > Z
D. X = Y > Z
E. X = Y = Z or relation cannot be established

160). 7x + 6y = 110
4x + 3y = 59
x + z = 15
A. X < Y = Z
B. X ≤ Y < Z
C. X < Y > Z
D. X = Y > Z
E. X = Y = Z or relation cannot be established
151). A.
X>Y
Explanation:
5x + 2y = 31 —-(1)
3x + 7y = 36 —-(2)
By solving eqn(1) and (2)
x=5;y=3

152) E.
X = Y or relation cannot be established
Explanation:
x2 – x – √3x + √3= 0
x (x-1) – √3(x -1) = 0
(x-1) (x-√3) = 0
x = 1, 1.732
y2 – 3y + 2 = 0
y2 – y – 2y + 2 = 0
y = 1, 2
Put on number line
1, 1, 1.732, 2

153). D.
X≤Y
Explanation:
(48 – 12) / x4/7 = x10/7
36 = x(10/7 + 4/7)
36 = x2
x= ±6
y3 + 783 = 999
y3 = 999 – 783
y3 = 216

www.ibpsguide.com | estore.ibpsguide.com | www.sscexamguide.com 51


500 Quadratic Equations for IBPS RRB/PO/Clerk

y=6
Put on number line
-6, 6, 6

154). B.
X<Y
Explanation:
172 + 144 ÷ 18 = x
x = 297
262 – 18 * 21 = y
y = 676 – 378 = 29

155). B.
X<Y
Explanation:
5/7 – 5/21 = √x/42
10/21 = √x/42
√x = 20
x = 400
√y/4 + √y/16 = 250/√y
5√y/16 = 250/√y
y = 800

156). E.
X = Y or relation cannot be established
Explanation:
9/√x + 19/√x = √x
x = 28
y5 – [(28)11/2 /√y] = 0
y11/2 = (28)11/2
y = 28

157). A.
X>Y
Explanation:
12/√x – 23/√x = 5√x
-11 = 5x
x = -2.2
√y/12 – 5√y/12 = 1/√y
√y[1/12 – 5/12]= 1/√y
y = -3

158). A.
X<Y=Z
Explanation:

www.ibpsguide.com | estore.ibpsguide.com | www.sscexamguide.com 52


500 Quadratic Equations for IBPS RRB/PO/Clerk

7x + 6y + 4z = 122 —(1)
4x + 5y + 3z = 88 —(2)
9x + 2y + z = 78 —(3)
From (1) and (2) => 5x – 2y =4 —(a)
From (2) and (3) => 23x + y = 146 —(b)
From (a) and (b) => x = 6, y = 8. Put values in eqn (3) => z = 8

159). E.
X = Y or relation cannot be established
Explanation:
(x + y)³ = 1331
x + y = 11 —(a)
(x + y)2 = 121
(a + b)2 – (a – b)2 = 4ab
(x – y)2 + 4xy = 121
x – y = 3 —(b)
From eqn (a) and (b)
x = 7; y = 4 Put values in eqn (2) => z = -3

160). C.
X<Y>Z
Explanation:
7x + 6y = 110 —(1)
4x + 3y = 59 —(2)
x + z = 15 —(3)
From eqn(1) and (2)
x = 8; y = 9 Put values in eqn (3) => z = 7

Directions (Q. 161-170):

161)

162).

163).

www.ibpsguide.com | estore.ibpsguide.com | www.sscexamguide.com 53


500 Quadratic Equations for IBPS RRB/PO/Clerk

164).

165).

166).

167).

168).

169).

170).

161). A)

162). D)

www.ibpsguide.com | estore.ibpsguide.com | www.sscexamguide.com 54


500 Quadratic Equations for IBPS RRB/PO/Clerk

163). D)

164). E)

165). D)

166). C)

www.ibpsguide.com | estore.ibpsguide.com | www.sscexamguide.com 55


500 Quadratic Equations for IBPS RRB/PO/Clerk

167). A)

168). E)

169). B)

170). B)

Directions (Q. 171 - 180): You have to solve equation I and II ,Give answer
1)If X>Y
2)If X<Y

www.ibpsguide.com | estore.ibpsguide.com | www.sscexamguide.com 56


500 Quadratic Equations for IBPS RRB/PO/Clerk

3)If X≥ Y
4)If X≤ Y
5)If X=Y or cannot be established

171). I. 4X2 – 19X + 12= 0


II. 3Y2 + 8Y + 4 = 0

172). I. X2 = 729
II. Y – √324 = √81

173). I. X2 + X – 56= 0
II. Y2 – 17Y + 72 = 0

174). I. 20X2 – 17X + 3= 0


II. 20Y2 – 9Y + 1 = 0

175). I. 3X2 – 10X + 8= 0


II. Y2 + 3Y – 4 = 0

176). I. X2 + 5X + 6= 0
II. Y2 + 3Y + 2 = 0

177). I. 2X2 + 5X + 2= 0
II. 4Y2 – 1 = 0

178). I. X2 + 14X -72= 0


II. 3Y2 + 14Y + 15 = 0

179). I. X2 + 8X + 15= 0
II. Y2 – Y + 12 = 0

180). I. 4X2 – 13X – 12= 0


II. Y2 – 7Y – 60 = 0

171). I. 4X2 – 19X + 12= 0


II. 3Y2 + 8Y + 4 = 0
Answer –1)If X>Y
Explanation :
(x-4)(4x-3) = 0
X=4,3/4
(y+2)(3y+2) = 0
y=-2, -2/3
4,3/4,-2/3,-2
x>y
172). I. X2 = 729
II. Y – √324 = √81
Answer –4)If X≤ Y
Explanation :
X= ±27

www.ibpsguide.com | estore.ibpsguide.com | www.sscexamguide.com 57


500 Quadratic Equations for IBPS RRB/PO/Clerk

Y = 9+18 = 27
x≤y
173). I. X2 + X – 56= 0
II. Y2 – 17Y + 72 = 0
Answer –2)If X<Y
Explanation :
(x+8)(x-7) = 0
X=7,-8
(y-8)(y-9) = 0
Y=8, 9
9,8,7,-8
Yyxx = x<y
174). I. 20X2 – 17X + 3= 0
II. 20Y2 – 9Y + 1 = 0
Answer –3)If X≥ Y
Explanation :
(4x-1)(5x-3) = 0
X=1/4, 3/5
(4y-1)(5y-1) =0
Y = 1/4, 1/5
3/5, ¼,1/4, 1/5
X≥ Y
175). I. 3X2 – 10X + 8= 0
II. Y2 + 3Y – 4 = 0
Answer –1)If X>Y
Explanation :
(x-2)(3x-4) = 0
X=2, 4/3
(y-1)(y+4) = 0
Y = 1, -4
2,4/3,1,-4
x>y
176). I. X2 + 5X + 6= 0
II. Y2 + 3Y + 2 = 0
Answer –4)If X≤ Y
Explanation :
(x+2)(x+3) = 0
X=-2,-3
(y+1)(y+2) = 0
Y=-1,-2
-1,-2,-2,-3
x≤y
177). I. 2X2 + 5X + 2= 0
II. 4Y2 – 1 = 0

www.ibpsguide.com | estore.ibpsguide.com | www.sscexamguide.com 58


500 Quadratic Equations for IBPS RRB/PO/Clerk

Answer –4)If X≤ Y
Explanation :
(2x+1)(x+2) = 0
X= – 1/2, -2
Y = ½, -1/2
x≤y
178). I. X2 + 14X -72= 0
II. 3Y2 + 14Y + 15 = 0
Answer – 5)If X=Y or cannot be established
Explanation :
(x-4)(x+18) = 0
X=4,-18
(y+3)(3y+5) = 0
Y= -3, -5/3

179). I. X2 + 8X + 15= 0
II. Y2 – Y + 12 = 0
Answer – 4)If X≤ Y
Explanation :
(x+5)(x+3) = 0
X=-5,-3
(y-4)(y+3) = 0
Y=4,-3
x≤y
180). I. 4X2 – 13X – 12= 0
II. Y2 – 7Y – 60 = 0
Answer – 5)If X=Y or cannot be established
Explanation :
(x-4)(4x+3) = 0
X=4, -3/4
(x+5)(x-12) =0
Y = 12, -5

www.ibpsguide.com | estore.ibpsguide.com | www.sscexamguide.com 59


500 Quadratic Equations for IBPS RRB/PO/Clerk

www.ibpsguide.com | estore.ibpsguide.com | www.sscexamguide.com 60


500 Quadratic Equations for IBPS RRB/PO/Clerk

www.ibpsguide.com | estore.ibpsguide.com | www.sscexamguide.com 61


500 Quadratic Equations for IBPS RRB/PO/Clerk

www.ibpsguide.com | estore.ibpsguide.com | www.sscexamguide.com 62


500 Quadratic Equations for IBPS RRB/PO/Clerk

Directions (Q. 191-200): in each question two equations numbered I and II are given. You have to solve
both the equations and mark the answer.
a) If x > y
b) If x ≥ y
c) If x < y
d) If x ≤ y
e) If x = y or no relation can be established between x and y.
191).
I. x2 + 18x + 72 = 0
II. y2 + 6y + 8 =0

192).
I. 8x2 – 22x + 15 = 0
II. 3y2 – 13y + 14 = 0

193).
I. 9x2 – 26x + 16 = 0
II. 3y2 – 16y + 20 = 0

194).
I. 10x2 – 17x + 7 = 0
II. 15y2 – 19y + 6 = 0

195).
I. 12x2 + 19x + 5 = 0
II. 5y2 + 16y + 3 = 0

196).
I. x2 – 8x + 15 = 0
II. 2y2 – 11y + 14 = 0
a) x> y
b) x = y or relationship can’t be established
c) x ≤ y
d) x ≥ y

www.ibpsguide.com | estore.ibpsguide.com | www.sscexamguide.com 63


500 Quadratic Equations for IBPS RRB/PO/Clerk

e) x< y

197).
I. x = √2916
II. y2 = 2916
a) x< y
b) x> y
c) x ≤ y
d) x ≥ y
e) x = y or relationship can’t be established

198).
I. 3x2 + 35x + 88 = 0
II. y2 + 787 = 1316
a) x> y
b) x ≤ y
c) x = y or relationship can’t be established
d) x ≥ y
e) x< y

199).
I. x2 – 14x + 45 = 0
II. y2 – 9y + 20 = 0
a) x< y
b) x> y
c) x ≤ y
d) x ≥ y
e) x = y or relationship can’t be established

200).
I. x2 – 54 = 3x
II. y2 = 36
a) x> y
b) x ≤ y
c) x ≥ y
d) x> y
e) x = y or relationship can’t be established

191). C)
I. x2 + 18x + 72 = 0
or, x2 + 12x + 6x + 72 =0
or, x(x + 12) + 6(x + 12) = 0
x = -9, -12
II. y2 + 6y + 8 =0
or, y2 + 4y + 2y + 8 =0

www.ibpsguide.com | estore.ibpsguide.com | www.sscexamguide.com 64


500 Quadratic Equations for IBPS RRB/PO/Clerk

or, y(y + 4) + 2(y + 4) = 0


y = -2, -4
Hence x < y

192). C)
I. 8x2 – 22x + 15 = 0
or, 8x2 - 12x - 10x + 15 = 0
or, 4x(2x - 3) - 5(2x - 3) = 0
or, (4x - 5) (2x - 3) = 0
x = 5/4, 3/2
II. 3y2 – 13y + 14 = 0
or, 3y2 - 6y - 7y + 14 =0
or, 3y(y - 2) - 7(y - 2) = 0
or, (3y - 7) (y - 2) = 0
y = 7/3, 2
Hence x < y

193). D)
I. 9x2 – 26x + 16 = 0
or, 9x2 - 18x - 8x + 16 = 0
or, 9x(x - 2) - 8(x - 2) = 0
or, (9x - 8) (x - 2) = 0
x = 8/9, 2
II. 3y2 – 16y + 20 = 0
or, 3y2 - 6y - 10y + 20 =0
or, 3y(y - 2) - 10(y - 2) = 0
or, (3y - 10) (y - 2) = 0
y = 2, 10/3
Hence x ≤ y

194). A)
I. 10x2 – 17x + 7 = 0
or, 10x2 - 10x - 7x + 7 = 0
or, 10x(x - 1) - 7(x - 1) = 0
or, (10x - 7) (x - 1) = 0
x = 7/10, 1
II. 15y2 – 19y + 6 = 0
or, 15y2 - 10y - 9y + 6 =0
or, 5y(3y - 2) – (3y - 2) = 0
or, (3y - 2) (5y - 3) = 0
y = 3/5, 2/3
Hence x > y

www.ibpsguide.com | estore.ibpsguide.com | www.sscexamguide.com 65


500 Quadratic Equations for IBPS RRB/PO/Clerk

195). E)
I. 12x2 + 19x + 5 = 0
or, 12x2 + 4x + 15x + 5 =0
or, 4x(3x + 1) + 5(3x + 1) = 0
or, (4x + 5) (3x + 1) = 0
x = -5/4, -1/3
II. 5y2 + 16y + 3 = 0
or,5y2 + 15y + y + 3 =0
or, 5y(y + 3) + 1(y + 3) = 0
or, (5y + 1) (y + 3) = 0
y = -1/5, -3
Hence no relationship can be established.

196). B)
I. x2 – 8x + 15 = 0
or, x2 - 5x - 3x + 15 =0
or, x(x - 5) - 3(x - 5) = 0
or, (x - 5) (x - 3) = 0
x = 3, 5
II. 2y2 – 11y + 14 = 0
or, 2y2 – 7y – 4y + 14 =0
or, 2y(y - 2) - 7(y - 2) = 0
or, (2y - 7) (y - 2) = 0
y = 7/2, 2
Hence no relationship can be established.

197). D)
I. x = √2916 = 54
II. y2 = 2916
y = ± 54
hence x ≥ y

198). C)
I. 3x2 + 35x + 88 = 0
or, 3x2 + 24x + 11x + 88 =0
or, 3x(x + 8) +11(x + 8) = 0
or, (3x + 11) (x + 8) = 0
x = -11/3, -8
II. y2 + 787 = 1316
y2 = 1316 – 787 = 529
y = √529 = ±23

www.ibpsguide.com | estore.ibpsguide.com | www.sscexamguide.com 66


500 Quadratic Equations for IBPS RRB/PO/Clerk

Hence no relationship can be established.

199). D)
I. x2 – 14x + 45 = 0
or, x2 - 9x - 5x + 45 =0
or, x(x - 9) -5(x - 9) = 0
or, (x - 5) (x - 9) = 0
x = 5, 9
II. y2 – 9y + 20 = 0
or, y(y - 5) – 4 (y - 5) = 0
y = 5, 4
Hence x ≥ y

200). E)
I. x2 – 54 = 3x
Or, x2 – 9x + 6x – 54 = 0
Or, x (x - 9) +6 (x - 9) = 0
x = 9, -6
II. y2 = 36
y=±6
Hence no relationship can be established

Directions(201-210): In the following questions, two equations numbered are given in variables x and y.
You have to solve both the equations and find out the relationship between x and y. Then give answer
accordingly-
A) If x > y
B) If x < y
C) If x ≥ y
D) If x ≤ y
E) If x = y or relation cannot be established

201) I. 5x2 – 18x + 9 =0


II. 3y2 + 5y – 2 = 0

202) I. √x – √6 / √x = 0
II. y3 – 63/2 = 0

203) I. (625)1/4x + √1225 = 155


II. √196y + 13 = 279

204) I. 3x2 – 17x + 24 =0


II. 4y2 – 15y + 14 = 0

www.ibpsguide.com | estore.ibpsguide.com | www.sscexamguide.com 67


500 Quadratic Equations for IBPS RRB/PO/Clerk

205) I. x2 – 2x – √5 x + 2√5 = 0
II. y2 – √3 y – √2 y + √6 =0

206) I. 5x2 – 23x + 12 = 0,


II. 5y2 – 28y + 15 = 0

207) I. 6x2 + 5x – 6 = 0,
II. 3y2 – 11y + 6 = 0

208) I. 3x2 – 5x – 12 = 0,
II. 2y2 – 17y + 36 = 0

209) I. 8x2 – (4 + 4√3)x + 2√3 = 0,


II. 3y2 – (4 + 3√3)y + 4√3 = 0

210) I. x2 + (4 + 2√2)x + 8√2 = 0


II. 3y2 – (3 + √3)y + √3 = 0

201) A
5x2 – 18x + 9 = 0
=> 5x2 – 15x – 3x + 9 =0
=> (5x – 3 )(x-3 )= 0
=> x= 3/ 5 or x= 3
3y2 + 5y – 2 = 0
=> 3y2 + 6y – y -2 = 0
=> (3y-1)(y + 2) = 0
=> y = 1/3 or -2

202) E
√x – √6 / √x = 0
x – √6 = 0
x = √6
y3 – 6 (3/2 ) = 0
=>y3 = (√6)3
=> y = √6

203) A
5x + 35 = 155
=> 5x = 155 – 35
=> x = 120/ 5 = 24√196 y + 13 = 279
=> 14y = 279 -13
=> y = 266/14 =19

204) A
3x2 -17x + 24 =0
=> 3x2 – 9x – 8x +24 = 0
=> (3x- 8)(x-3) = 0

www.ibpsguide.com | estore.ibpsguide.com | www.sscexamguide.com 68


500 Quadratic Equations for IBPS RRB/PO/Clerk

=> x= 8/3 or 3
4y2 – 15y + 14 = 0
=> 4y2 – 8y -7y +14 = 0
=> (4y -7 )(y-2)=0
=> y = 7 /4 or 2

205) A
x2 – 2x- √5 x + 2√5 = 0
=> x(x-2 ) – √5 (x-2 )= 0
=> (x-2)(x-√5)=0
=> x= 2 0r √5
y2 – √3 y – √2 y + √6 =0
=> y(y-√3) – √2(y – √3) = 0
=> (y – √2)(y- √3) =0
=> y = √2 or √3

206) E
x = 3/5, 4
y = 3/5, 5

207) D
x = -3/2, 2/3
y = 2/3, 3

208) B
x = -4/3, 3
y = 4, 9/2

209) B
8x2 – (4 + 4√3)x + 2√3 = 0
(8x2 – 4x) – (4√3x – 2√3) = 0
4x (2x- 1) – 2√3 (2x – 1) = 0,
So x = 1/2 (0.5), 2√3/4 (0.87)
3y2 – (4 + 3√3)y + 4√3 = 0
(3y2 – 4y) – (3√3y – 4√3) = 0
y (3y – 4) – √3 (3y – 4) = 0
So, y = √3 (1.732), 4/3
put on number line
0.5………..0.87……..4/3………1.732

210) B
x2 + (4 + 2√2)x + 8√2 = 0
(x2 + 4x) + (2√2x + 8√2) = 0
x (x + 4) + 2√2 (x + 4) = 0

www.ibpsguide.com | estore.ibpsguide.com | www.sscexamguide.com 69


500 Quadratic Equations for IBPS RRB/PO/Clerk

So x = -4, -2√2 (-2.8)


3y2 – (3 + √3)y + √3 = 0
(3y2 – 3y) – (√3y – √3) = 0
3y (y – 1) – √3 (y – 1) = 0
So y = 1, √3/3 (0.57)

Directions(211-220): In the following questions, two equations numbered are given in variables x and y.
You have to solve both the equations and find out the relationship between x and y. Then give answer
accordingly-
A) If x > y
B) If x < y
C) If x ≥ y
D) If x ≤ y
E)If x = y or relation cannot be established

211) I. 7x2 – 9x + 2 = 0
II. y2 – 4y + 3 = 0

212) I. x2+ x – 20 = 0
II. 2y2 – 19y + 45 = 0

213) I. 7x + 3y = 26
II. 2x + 17y = -41

214) I. 3x2 – 20x + 33 = 0


II. 2y2 – 11y + 15 = 0

215) I. 5x2 + 2x – 3 = 0
II. 2y2+ 7y + 6 = 0

216) I. 35x2 – 53x + 20 = 0


II. 56y2 -97y + 42 = 0

217) I. x = 3√4913
II. 13y + 3x = 246

218) I. x2 – 5x – 14 = 0
II. y2+ 7y + 10 = 0

219) I. x2= 64
II. 2y2 + 25y + 72 = 0

220) I. x2 – 3481 = 0
II. 3y2 = 3√216000y

211) D
I. 7x2– 7x – 2x + 2 = 0
or, 7x(x – 1) – 2(x – 1) = 0
(7x – 2) (x – 1) = 0
or, x =2/7 , 1

www.ibpsguide.com | estore.ibpsguide.com | www.sscexamguide.com 70


500 Quadratic Equations for IBPS RRB/PO/Clerk

II. y2– y – 3y + 3 = 0
or, y(y – 1) – 3(y – 1) = 0
or, (y – 3) (y – 1) = 0
y = 1, 3
x <= y

212) B
I. x2+ x – 20 = 0
or, x2+ 5x – 4x – 20 = 0
or, x(x + 5) – 4(x + 5) = 0
or, (x – 4) (x + 5) = 0
x = 4, – 5
II. 2y2– 10y – 9y + 45 = 0
or, 2y(y – 5) – 9(y – 5) = 0
or, (y – 5) (2y – 9) = 0
y = 5 , 9/2

213) A
Eqn (I) × 2
Eqn (II) × 7
14x + 6y = 52
14x + 119y = – 287
– 113y = 339
y = – 3 and x = 5, ie x > y

214) C
I. 3x2– 9x – 11x + 33 = 0
or, 3x(x – 3) – 11(x – 3) = 0
or, (3x – 11) (x – 3) = 0
x = 3, 11/3
II. 2y2– 6y – 5y + 15 = 0
or, 2y(y – 3) – 5(y – 3) = 0
or, (y – 3) (2y – 5) = 0
y = 3 , 5/2

215) A
5x2+ 5x – 3x – 3 = 0
or 5x (x + 1) – 3(x + 1) = 0
or (5x – 3) (x + 1) = 0
x= 3/5, -1 II. 2y2+ 4x + 3y + 6 = 0
or 2y(y + 2) + 3(y + 2) =0
or (2y + 3) (y + 2) = 0
y = -3/2 , -2

216) B

www.ibpsguide.com | estore.ibpsguide.com | www.sscexamguide.com 71


500 Quadratic Equations for IBPS RRB/PO/Clerk

I. 35x2 – 28x – 25x + 20 = 0


or 7x(5x – 4) – 5(5x – 4) = 0
or (7x – 5) (5x – 4) = 0
x 5/7 , 4/5 II. 56y2 – 48y – 49y + 42 = 0
or 8y(7y – 6) – 7(7y – 6) = 0
or (8y – 7) (7y – 6) = 0
y = 7/8 , 6/7

217) A
I. x = 3√4913
x = 17 II. 13y = 246 – 3x
or 13y = 246 – 51 = 195
y = 15
x>y

218) C
I. x2– 7x + 2x – 14 = 0
or x(x – 7) + 2(x – 7) = 0
(x + 2) (x – 7) = 0
x = -2, 7 II. y2+ 5y + 2y + 10 = 0
or y(y + 5) + 2(y + 5) = 0
or (y + 2) (y + 5) = 0
y = -2, -5
x≥ y

219) E
I. x2= 64
x = ±8 II. 2y2+ 9y + 16y + 72 = 0
or, y(2y + 9) + 8(2y + 9) = 0
or, (y + 8) (2y + 9) = 0
y = -8 , -9/2
no relation between x and y.

220) E
I. x2= 3481
x = ± 59 II. 3y2= 3 √216000
3y2= 60y
y = ± 20
No relation

Directions: In the following questions, two equations numbered are given in variables x and y. You have
to solve both the equations and find out the relationship between x and y. Then give answer accordingly-
A) If x > y
B) If x < y
C) If x ≥ y

www.ibpsguide.com | estore.ibpsguide.com | www.sscexamguide.com 72


500 Quadratic Equations for IBPS RRB/PO/Clerk

D) If x ≤ y
E) If x = y or relation cannot be established

221) I. 3x2 + 10x – 8 = 0,


II. 3y2 – 20y + 12 = 0

222) I. 4x2 – 23x + 15 = 0,


II. 4y2 + 9y – 9 = 0

223) I. 5x2 – 13x – 6 = 0,


II. 5y2 – 18y – 8 = 0

224) I. 2x2 + 7x – 4 = 0,
II. 3y2 – 19y + 20 = 0

225) I. 2x2 – 3x – 9 = 0,
II. 3y2 + 13y + 14 = 0

226) I. 3x2 – x – 10 = 0,
II. 3y2 – 11y + 6 = 0

227) I. 3x2 – (3 – 2√2)x – 2√2 = 0


II. 3y2 – (1 + 3√3)y + √3 = 0

228) I. x2 + (4 + √2)x + 4√2 = 0


II. 5y2 + (2 + 5√2)y + 2√2 = 0

229) I. 6x2 – (3 + 4√3)x + 2√3 = 0,


II. 3y2 – (6 + 2√3)y + 4√3 = 0

230) I. 8x2 + (4 + 2√2)x + √2 = 0


II. y2 – (3 + √3)y + 3√3 = 0

221) D
3x2 + 10x – 8 = 0
3x2 + 12x – 2x + 8 = 0
Gives x = -4, 2/3
3y2 – 20y + 12 = 0
3y2 – 18y – 2y + 12 = 0
Gives y = 2/3, 6

222) C
4x2 – 23x + 15 = 0
4x2 – 20x -3x + 15 = 0
Gives x = 3/4, 5
4y2 + 9y – 9 = 0
4y2 + 12y – 3y – 9 = 0
Gives y = -3, 3/4

www.ibpsguide.com | estore.ibpsguide.com | www.sscexamguide.com 73


500 Quadratic Equations for IBPS RRB/PO/Clerk

223) E
x = -2/5, 3
y = -2/5, 4

224) C
2x2 + 7x – 4 = 0
Gives x = -4, 1/2
3y2 – 19y + 20 = 0
Gives y= 4/3, 5

225) A
2x2 – 3x – 9 = 0
Gives x = -3/2, 3
3y2 + 13y + 14 = 0
Gives y= -7/3, -2

226) E
3x2 – x – 10 = 0
Gives x = -5/3, 2
3y2 – 11y + 6 = 0
Gives y = 2/3, 3

227) E
3x2 – (3 – 2√2)x – 2√2 = 0
(3x2 – 3x) + (2√2x – 2√2) = 0
3x (x – 1) + 2√2 (x – 1) = 0
So x = 1, -2√2/3 (-0.9)
3y2 – (1 + 3√3)y + √3 = 0
(3y2 – y) – (3√3y – √3) = 0
y (3y – 1) – √3 (3y – 1) = 0
So, y = 1/3, √3 (1.7)

228) D
x2 + (4 + √2)x + 4√2 = 0
(x2 + 4x) + (√2x + 4√2) = 0
x (x + 4) + √2 (x + 4) = 0
So x = -4, -√2 (-1.4)
5y2 + (2 + 5√2)y + 2√2 = 0
(5y2 + 2y) + (5√2y + 2√2) = 0
y (5y + 2) + √2 (5y + 2) = 0
So, y = -2/5 (-0.4), -√2 (-1.4)

229) D
6x2 – (3 + 4√3)x + 2√3 = 0
(6x2 – 3x) – (4√3x – 2√3) = 0

www.ibpsguide.com | estore.ibpsguide.com | www.sscexamguide.com 74


500 Quadratic Equations for IBPS RRB/PO/Clerk

3x (2x- 1) – 2√3 (2x – 1) = 0,


So x = 1/2, 2√3/3 (1.15)
3y2 – (6 + 2√3)y + 4√3 = 0
(3y2 – 6y) – (2√3y – 4√3) = 0
3y (y – 2) – 2√3 (y – 2) = 0
So, y = 2, 2√3/3

230) B
8x2 + (4 + 2√2)x + √2 = 0
(8x2 + 4x) + (2√2x + √2) = 0
4x (2x + 1) + √2 (2x + 1) = 0
So x = -1/2 (-0.5), -√2/4 (-0.35)
y2 – (3 + √3)y + 3√3 = 0
(y2 – 3y) – (√3y – 3√3) = 0
y (y – 3) – √3 (y – 3) = 0
So y = 3, √3 (1.73)

Directions (231-240): In the following questions, two equations numbered are given in variables x and y.
You have to solve both the equations and find out the relationship between x and y. Then give answer
accordingly-
A) If x > y
B) If x < y
C) If x ≥ y
D) If x ≤ y
E) If x = y or relation cannot be established

231) I. 3x2 – 4x – 15 = 0,
II. 3y2 + 11y + 10 = 0

232) I. 3x2 + 17x + 24 = 0,


II. 3y2 – 4y – 32 = 0

233) I. 3x2 – 2x – 16 = 0,
II. 3y2 – 20y + 32 = 0

234)I. 2x2 + 5x – 12 = 0,
II. 4y2 – 19y – 30 = 0

235) I. 2x2 – 17x + 36 = 0,


II. 3y2 – 2y – 8 = 0

236) I. 3x2 + 19x + 30 = 0,


II. 2y2 + 3y – 2 = 0

237) I. 5x2 + 11x – 12 = 0,


II. 3y2 + 7y – 6 = 0

238) I. 3x2 + x – 10 = 0,
II. 2y2 + 11y + 14 = 0

www.ibpsguide.com | estore.ibpsguide.com | www.sscexamguide.com 75


500 Quadratic Equations for IBPS RRB/PO/Clerk

239) I. 3x2 + 19x + 30 = 0,


II. 2y2 – 3y – 20 = 0

240) I. 2x2 – x – 36 = 0,
II. 2y2 – 21y + 54 = 0

231) C
3x2 – 4x – 15 = 0
3x2 – 9x + 5x – 15 = 0
Gives x = -5/3, 3
3y2 + 11y + 10 = 0
3y2 + 6y + 5y + 10 = 0
Gives y = -2. -5/3

232) D
3x2 + 17x + 24 = 0
3x2 + 17x + 24 = 0
Gives x = -3, -8/3
3y2 – 4y – 32 = 0
3y2 – 12y + 8y – 32 = 0
Gives y = -8/3, 4

233) D
3x2 – 2x – 16 = 0
3x2 – 2x – 16 = 0
Gives x = -2, 8/3
3y2 – 20y + 32 = 0
3y2 – 20y + 32 = 0
Gives y = 8/3, 4

234) C
2x2 + 5x – 12 = 0
2x2 + 8x – 3x – 12 = 0
Gives x = -4, 3/2
4y2 – 19y – 30 = 0
4y2 – 24y + 5y – 30 = 0
Gives y= -5/4, 6

235) A
2x2 – 17x + 36 = 0
2x2 – 8x – 9x + 36 = 0
Gives x = 4, 9/2
3y2 – 2y – 8 = 0
3y2 – 6y + 4y – 8 = 0
Gives y= -4/3, 2

www.ibpsguide.com | estore.ibpsguide.com | www.sscexamguide.com 76


500 Quadratic Equations for IBPS RRB/PO/Clerk

236) B
3x2 + 19x + 30 = 0
3x2 + 9x + 10x + 30 = 0
Gives x = -10/3, -3
2y2 + 3y – 2 = 0
2y2 + 4y – y – 2 = 0
Gives y = -2, ½

237) E
5x2 + 11x – 12 = 0
5x2 + 15x – 4x – 12 = 0
Gives x = -3, 4/5
3y2 + 7y – 6 = 0
3y2 + 9y – 2y – 6 = 0
So y = -3, 2/3

238) C
3x2 + x – 10 = 0
3x2 + 6x – 5x – 10 = 0
Gives x = -2, 5/3
2y2 + 11y + 14 = 0
2y2 + 4y + 7y + 14 = 0
So y = -7/2, -2

239) B
3x2 + 19x + 30 = 0
3x2 + 9x + 10x + 30 = 0
Gives x = -3, -10/3
2y2 – 3y – 20 = 0
2y2 – 8y + 5y – 20 = 0
So y = -5/2, 4

240) D
2x2 – x – 36 = 0
2x2 + 8x – 9x – 36 = 0
Gives x = -4, 9/2
2y2 – 21y + 54 = 0
2y2 – 12y – 9y + 54 = 0
So y = 9/2, 6

Directions(241-250): In the following questions, two equations numbered are given in variables x and y.
You have to solve both the equations and find out the relationship between x and y. Then give answer
accordingly-
A) x> y
B) x < y
C) x ≥ y

www.ibpsguide.com | estore.ibpsguide.com | www.sscexamguide.com 77


500 Quadratic Equations for IBPS RRB/PO/Clerk

D) x ≤ y
E) x = y or relationship cannot be determined

241) I. 4x2 – 9x – 9 = 0,
II. 4y2 + 11y + 6 = 0

242) I. 3x2 – 2x – 21 = 0,
II. 6y2 + 17y + 7 = 0

243) I. 3x2 – 13x + 14 = 0,


II. 3y2 – 20y + 32 = 0

244) I. 3x2 – 2x – 16 = 0,
II. 3y2 – 20y + 32 = 0

245)I. 5x2 – 16x – 16 = 0,


II. 3y2 – 14y + 8 = 0

246) I. 4x2 – 17x + 18 = 0,


II. 3y2 – 2y – 8 = 0

247) I. 2x2 – 5x – 12 = 0,
II. 3y2 – 17y + 10 = 0

248) I. 3x2 + 2x – 16 = 0,
II. y2 + 11y + 24 = 0

249) I. 3x2 + 10x + 8 = 0,


II. 3y2 – 11y – 20 = 0

250) I. 4x2 + 23x + 15 = 0,


II. 3y2 – 19y – 14 = 0

241) C
4x2 – 9x – 9 = 0
4x2 – 12x + 3x – 9 = 0
Gives x = -3/4, 3
4y2 + 11y + 6 = 0
4y2 + 8y + 3y + 6 = 0
Gives y = -2, -3/4
Put all values on number line and analyze the relationship
-2…. -3/4….3

242) E
3x2 – 2x – 21 = 0
3x2 – 9x + 7x – 21 = 0
Gives x = -7/3, 3
6y2 + 17y + 7 = 0

www.ibpsguide.com | estore.ibpsguide.com | www.sscexamguide.com 78


500 Quadratic Equations for IBPS RRB/PO/Clerk

6y2 + 3y + 14y + 7 = 0
Gives y = -7/3, -1/2

243) B
3x2 – 13x + 14 = 0
3x2 – 6x – 7x + 14 = 0
Gives x = 2, 7/3
3y2 – 20y + 32 = 0
3y2 – 12y – 8y + 32 = 0
Gives y = 8/3, 4

244) D
3x2 – 2x – 16 = 0
3x2 – + 6x – 8x – 16 = 0
Gives x = -2, 8/3
3y2 – 20y + 32 = 0
3y2 – 12y – 8y + 32 = 0
Gives y = 8/3, 4

245) E
5x2 – 16x – 16 = 0
5x2 – 20x + 4x – 16 = 0
Gives x = -4/5, 4
3y2 – 14y + 8 = 0
3y2 – 12y – 2y + 8 = 0
Gives y= 2/3, 4

246) C
4x2 – 17x + 18 = 0
4x2 – 8x – 9x + 18 = 0
Gives x = 2, 9/4
3y2 – 2y – 8 = 0
3y2 – 6y + 4y – 8 = 0
Gives y = -4/3, 2

247) E
2x2 – 5x – 12 = 0
2x2 – 8x + 3x – 12 = 0
Gives x = -3/2, 4
3y2 – 17y + 10 = 0
3y2 – 15y – 2y + 10 = 0
So y = 2/3, 5

248) A

www.ibpsguide.com | estore.ibpsguide.com | www.sscexamguide.com 79


500 Quadratic Equations for IBPS RRB/PO/Clerk

3x2 + 2x – 16 = 0
3x2 – 6x + 8x – 16 = 0
Gives x = -8/3, 2
y2 + 11y + 24 = 0
y2 + 8y + 3y + 24 = 0
So y = -8, -3

249) D
3x2 + 10x + 8 = 0
3x2 + 6x + 4x + 8 = 0
Gives x = -2, -4/3
3y2 – 11y – 20 = 0
3y2 – 15y + 4y – 20 = 0
So y = -4/3, 5

250) B
4x2 + 23x + 15 = 0
4x2 + 20x + 3x + 15 = 0
Gives x = -5, -3/4
3y2 – 19y – 14 = 0
3y2 – 21y + 2y – 14 = 0
So y = -2/3, 7

Directions(251-260): In the following questions, two equations numbered are given in variables x and y.
You have to solve both the equations and find out the relationship between x and y. Then give answer
accordingly-
A) If x > y
B) If x < y
C) If x ≥ y
D) If x ≤ y
E) If x = y or relation cannot be established

251) I. 3x2 + 8x + 4 = 0,
II. 2y2 – 7y – 4 = 0

252) I. 2x2 – 13x + 20 = 0,


II. 3y2 + 4y – 20 = 0

253) I. 3x2 + x – 14 = 0,
II. 3y2 – 5y – 12 = 0

254) I. 3x2 – 2x – 21 = 0,
II. 3y2 + 19y + 28 = 0

255) I. 4x2 + 23x + 28 = 0,


II. 4y2 – y – 14 = 0

www.ibpsguide.com | estore.ibpsguide.com | www.sscexamguide.com 80


500 Quadratic Equations for IBPS RRB/PO/Clerk

256) I. 4x2 + x – 18 = 0,
II. 4y2 – 3y – 27 = 0

257) I. 3x2 – 16x + 21 = 0,


II. 2y2 – y – 6 = 0

258) I. 3x2 + x – 2 = 0,
II. 4y2 + 13y + 10 = 0

259) I. 4x2 + 29x + 45 = 0,


II. 4y2 – 3y – 27 = 0

260) I. 3x2 – 22x + 35 = 0,


II. 3y2 – 16y + 21 = 0

251) B
3x2 + 8x + 4 = 0
3x2 + 6x + 2x + 4 = 0
Gives x = -2/3, -2
2y2 – 7y – 4 = 0
2y2 + y – 8y – 4 = 0
Gives y = 4, -1/2
Put all values on number line and analyze the relationship
-2…. -2/3….-1/2….. 4

252) A
2x2 – 13x + 20 = 0
2x2 – 8x – 5x + 20 = 0
Gives x = 5/2, 4
3y2 + 4y – 20 = 0
3y2 – 6y + 10y – 20 = 0
Gives y = 2, -10/3

253) E
3x2 + x – 14 = 0
3x2 – 6x + 7x – 14 = 0
Gives x = 2, -7/3
3y2 – 5y – 12 = 0
3y2 – 9y + 4y – 12 = 0
Gives y = -4/3, 3

254) C
3x2 – 2x – 21 = 0
3x2 – 9x + 7x – 21 = 0
Gives x = -7/3, 3
3y2 + 19y + 28 = 0

www.ibpsguide.com | estore.ibpsguide.com | www.sscexamguide.com 81


500 Quadratic Equations for IBPS RRB/PO/Clerk

3y2 + 12y + 7y + 28 = 0
Gives y= -7/3, -4

255) D
4x2 + 23x + 28 = 0
4x2 + 16x + 7x + 28 = 0
Gives x = -4, -7/4
4y2 – y – 14 = 0
4y2 – 8y + 7y – 14 = 0
Gives y= -7/4, 2

256) E
4x2 + x – 18 = 0
4x2 – 8x + 9x – 18 = 0
Gives x = -9/4, 2
4y2 – 3y – 27 = 0
4y2 – 12y + 9y – 27 = 0
Gives y = -9/4, 3

257) A
3x2 – 16x + 21 = 0
3x2 – 9x – 7x + 21 = 0
Gives x = 3, 7/3
2y2 – y – 6 = 0
2y2 – 4y + 3y – 6 = 0
So y = -3/2, 2

258) A
3x2 + x – 2 = 0
3x2 + 3x – 2x – 2 = 0
Gives x = -1/3, 2
4y2 + 13y + 10 = 0
4y2 + 8y + 5y + 10 = 0
So y = -2, -5/4

259) D
4x2 + 29x + 45 = 0
4x2 + 20x + 9x + 45 = 0
Gives x = -5, -9/4
4y2 – 3y – 27 = 0
4y2 – 12y + 9y – 27 = 0
So y = -9/4, 3

260) E

www.ibpsguide.com | estore.ibpsguide.com | www.sscexamguide.com 82


500 Quadratic Equations for IBPS RRB/PO/Clerk

3x2 – 22x + 35 = 0
3x2 – 15x – 7x + 35 = 0
Gives x = 7/3, 5
3y2 – 16y + 21 = 0
3y2 – 9y – 7y + 21 = 0
So y = 7/3, 3

Directions(261-270): In the following questions, two equations numbered are given in variables x and y.
You have to solve both the equations and find out the relationship between x and y. Then give answer
accordingly-
A) If x > y
B) If x < y
C) If x ≥ y
D) If x ≤ y
E) If x = y or relation cannot be established

261) I. 3x2 – 13x – 10 = 0,


II. 2y2 + 11y + 12 = 0

262) I. 3x2 – 8x – 16 = 0,
II. 3y2 – 26y + 56 = 0

263) I. 3x2 + 10x – 8 = 0,


II. 3y2 + 10y + 8 = 0

264) I. 3x2 – 25x + 52 = 0,


II. 2y2 – 17y + 36 = 0

265) I. 3x2 + 7x – 6 = 0,
II. 4y2 – 11y + 6 = 0

266) I. 2x2 – 3x – 14 = 0,
II. 3y2 + 16y + 20 = 0

267) I.7x2 + 19x – 6 = 0,


II. 3y2 – 8y – 16 = 0

268) I. 8x2 + (4 + 2√2)x + √2 = 0


II. 3y2 – (6 + 2√3)y + 4√3 = 0

269) I. 3x2 – (3 – 2√2)x – 2√2 = 0


II. 5y2 + (2 + 5√2)y + 2√2 = 0

270) I. 6x2 – (3 + 4√3)x + 2√3 = 0,


II. y2 – (3 + √3)y + 3√3 = 0

261) A
3x2 – 13x – 10 = 0
3x2 – 15x + 2x – 10 = 0
Gives x = -2/3, 5

www.ibpsguide.com | estore.ibpsguide.com | www.sscexamguide.com 83


500 Quadratic Equations for IBPS RRB/PO/Clerk

2y2 + 11y + 12 = 0
2y2 + 8y + 3y + 12 = 0
Gives y = -4, -3/2
Put all values on number line and analyze the relationship
-4…. -3/2….-2/3….. 5

262) D
3x2 – 8x – 16 = 0
3x2 – 12x + 4x – 16 = 0
Gives x = -4/3, 4
3y2 – 26y + 56 = 0
3y2 – 26y + 56 = 0
Gives y = 4, 14/3

263) E
3x2 + 10x – 8 = 0
3x2 + 12x – 2x – 8 = 0
Gives x = -2, 2/3
3y2 + 10y + 8 = 0
3y2 + 6y + 4y + 8 = 0
Gives y = -2, -4/3

264) E
3x2 – 25x + 52 = 0
3x2 – 12x – 13x + 52 = 0
Gives x = 4, 13/3
2y2 – 17y + 36 = 0
2y2 – 8y – 9y + 36 = 0
Gives y= 4, 9/2

265) B
3x2 + 7x – 6 = 0
3x2 + 9x – 2x – 6 = 0
Gives x = -3, 2/3
4y2 – 11y + 6 = 0
4y2 – 8y – 3y + 6 = 0
Gives y= 3/4, 2

266) C
2x2 – 3x – 14 = 0
2x2 + 4x – 7x – 14 = 0
Gives x = -2, 7/2
3y2 + 16y + 20 = 0
3y2 + 6y + 10y + 20 = 0
Gives y = -10/3, -2

www.ibpsguide.com | estore.ibpsguide.com | www.sscexamguide.com 84


500 Quadratic Equations for IBPS RRB/PO/Clerk

267) E
7x2 + 19x – 6 = 0
7x2 + 21x – 2x – 6 = 0
Gives x = -3, 2/7
3y2 – 8y – 16 = 0
3y2 – 12y + 4y – 16 = 0
So y = -4/3, 4

268) B
8x2 + (4 + 2√2)x + √2 = 0
(8x2 + 4x) + (2√2x + √2) = 0
4x (2x + 1) + √2 (2x + 1) = 0
So x = -1/2 (-0.5), -√2/4 (-0.35)
3y2 – (6 + 2√3)y + 4√3 = 0
(3y2 – 6y) – (2√3y – 4√3) = 0
3y (y – 2) – 2√3 (y – 2) = 0
So, y = 2, 2√3/3

269) E
3x2 – (3 – 2√2)x – 3√2 = 0
(3x2 – 3x) + (2√2x – 2√2) = 0
3x (x – 1) + 2√2 (x – 1) = 0
So x = 1, -2√2/3 (-0.9)
5y2 + (2 + 5√2)y + 2√2 = 0
(5y2 + 2y) + (5√2y + 2√2) = 0
y (5y + 2) + √2 (5y + 2) = 0
So, y = -2/5 (-0.4), -√2 (-1.4)

270) B
6x2 – (3 + 4√3)x + 2√3 = 0
(6x2 – 3x) – (4√3x – 2√3) = 0
3x (2x- 1) – 2√3 (x – 2) = 0,
So x = 1/2, 2√3/3 (1.15)
y2 – (3 + √3)y + 3√3 = 0
(y2 – 3y) – (√3y – 3√3) = 0
y (y – 3) – √3 (y – 3) = 0
So x = 3, √3 (1.73)

Directions: In the following questions, two equations numbered are given in variables x and y. You have
to solve both the equations and find out the relationship between x and y. Then give answer accordingly-
A) If x > y
B) If x < y
C) If x ≥ y
D) If x ≤ y
E) If x = y or relation cannot be established

www.ibpsguide.com | estore.ibpsguide.com | www.sscexamguide.com 85


500 Quadratic Equations for IBPS RRB/PO/Clerk

271) I. 3x2 – 25x + 52 = 0,


II. 3y2 – 16y + 16 = 0

272) I. 3x2 – 8x – 16 = 0,
II. 2y2 – 5y – 12 = 0

273) I. 3x2 + 22x + 24 = 0,


II. 2y2 + 7y – 4 = 0

274) I. 2x2 – 9x + 4 = 0,
II. 2y2 – 17y + 36 = 0

275) I. 3x2 + 7x – 6 = 0,
II. 3y2 – 19y + 20 = 0

276) I. 3x2 – 4x – 4 = 0,
II. 3y2 + 16y + 20 = 0

277) I. 7x2 + 19x – 6 = 0,


II. 2y2 + 13y + 21 = 0

278) I. x2 + (4 + √2)x + 4√2 = 0


II. 3y2 – (1 + 3√3)y + √3 = 0

279)I. 3x2 + (3 + 2√2)x + 2√2 = 0


II. 5y2 + (2 + 5√2)y + 2√2 = 0

280) I. 4x2 – 12x + 5 = 0,


II. 2y2 + 3y – 20 = 0
271) C
3x2 – 25x + 52 = 0
3x2 – 12x – 13x + 52 = 0
Gives x = 4, 13/3
3y2 – 16y + 16 = 0
3y2 – 14y – 4y + 16 = 0
Gives y = 4, 4/3

272) E
3x2 – 8x – 16 = 0
3x2 – 12x + 4x – 16 = 0
Gives x = -4/3, 4
2y2 – 5y – 12 = 0
2y2 – 8y + 3y – 12 = 0
Gives y = -3/2, 4

273) E
3x2 + 22 x + 24 = 0
3x2 + 18x + 4x + 24 = 0

www.ibpsguide.com | estore.ibpsguide.com | www.sscexamguide.com 86


500 Quadratic Equations for IBPS RRB/PO/Clerk

So x = -4/3, -6
2y2 + 7y – 4 = 0
2y2 + 8y – y – 4 = 0
Gives y = -4, ½

274) D
2x2 – 9x + 4 = 0
2x2 – 8x – x + 4 = 0
So x = 4 , 1/2
2y2 – 17y + 36 = 0
2y2 – 8y – 9y + 36 = 0
Gives y= 4, 9/2

275) B
3x2 + 7x – 6 = 0
3x2 + 9x – 2x – 6 = 0
Gives x = -3, 2/3
3y2 – 19y + 20 = 0
3y2 – 15y – 4y + 20 = 0
Gives y = 4/3, 5

276) A
3x2 – 4x – 4 = 0
3x2 – 6x + 2x – 4 = 0
Gives x = -2/3, 2
3y2 + 16y + 20 = 0
3y2 + 6y + 10y + 20 = 0
Gives y = -10/3, -2

277) C
7x2 + 19x – 6 = 0
7x2 + 21x – 2x – 6 = 0
Gives x = -3, 2/7
2y2 + 13y + 21 = 0
2y2 + 6y + 7y + 21 = 0
So y = -7/2, -3

278) B
x2 + (4 + √2)x + 4√2 = 0
(x2 + 4x) + (√2x + 4√2) = 0
x (x + 4) + √2 (x + 4) = 0
So x = -4, -√2 (-1.4)
3y2 – (1 + 3√3)y + √3 = 0
(3y2 – y) – (3√3y – √3) = 0

www.ibpsguide.com | estore.ibpsguide.com | www.sscexamguide.com 87


500 Quadratic Equations for IBPS RRB/PO/Clerk

y (3y – 1) – √3 (3y – 1) = 0
So, y = 1/3, √3 (1.7)

279) E
3x2 + (3 + 2√2)x + 3√2 = 0
(3x2 + 3x) + (2√2x + 2√2) = 0
3x (x + 1) + 2√2 (x + 1) = 0
So x = -1, -2√2/3
5y2 + (2 + 5√2)y + 2√2 = 0
(5y2 + 2y) + (5√2y + 2√2) = 0
y (5y + 2) + √2 (5y + 2) = 0
So, y = -2/5, -√2

280) E
4x2 – 12x + 5 = 0
4x2 – 2x – 10x + 5 = 0
x = 1/2, 5/2
2y2 + 3y – 20 = 0
2y2 + 8y – 5y – 20 = 0
So y = -3, 5/2

Directions(281-290): In the following questions, two equations numbered are given in variables x and y.
You have to solve both the equations and find out the relationship between x and y. Then give answer
accordingly-
A) If x > y
B) If x < y
C) If x ≥ y
D) If x ≤ y
E) If x = y or relation cannot be established

281) I. 4x2 + 5x – 6 = 0,
II. 2y2 + 11y + 12 = 0

282) I. 12x2 – 49x + 30 = 0,


II. 6y2 – 35y + 50 = 0

283) I. 4x2 + 13x + 10 = 0,


II. 4y2 – 7y – 15 = 0

284) I. 12x2 – 5x – 3 = 0,
II. 6y2 + y – 2 = 0

285) I. 3x2 + 7x – 6 = 0,
II. 3y2 – 11y + 6 = 0

286) I. 5x2 – 36x – 32 = 0,


II. 3y2 + 16y + 20 = 0

www.ibpsguide.com | estore.ibpsguide.com | www.sscexamguide.com 88


500 Quadratic Equations for IBPS RRB/PO/Clerk

287) I. 3x2 – (6 + 2√3)x + 4√3 = 0,


II. 3y2 – (2 + 3√3)y + 2√3 = 0

288) I. 2x2 + (4 + √2)x + 2√2 = 0


II. y2 – (1 + 3√3)y + 3√3 = 0

289) I. x2 + (3 + 2√2)x + 6√2 = 0


II. 5y2 – (1 + 5√2)y + √2 = 0

290)I. 2x2 + (4 + 2√6)x + 4√6 = 0


II. 5y2 + (10 + √6)y + 2√6 = 0

281) E
4x2 + 5x – 6 = 0
4x2 + 8x – 3x – 6 = 0
Gives x = -2, 3/4
2y2 + 11y + 12 = 0
2y2 + 8y + 3y + 12 = 0
Gives y = -4, -3/2

282) E
12x2 – 49x + 30 = 0
12x2 – 9x – 40x + 30 = 0
Gives x = 3/4, 10/3
6y2 – 35y + 50 = 0
6y2 – 15y – 20y + 50 = 0
Gives y = 5/2, 10/3

283) D
4x2 + 13x + 10 = 0
4x2 + 8x + 5x + 10 = 0
Gives x = -2, -5/4
4y2 – 7y – 15 = 0
4y2 – 12y + 5y – 15 = 0
Gives y = -5/4, 3

284) E
12x2 – 5x – 3 = 0
12x2 + 4x – 9x – 3 = 0
Gives x = -1/3, 3/4
6y2 + y – 2 = 0
6y2 – 3y + 4y – 2 = 0
Gives y= -2/3, ½

285) D
3x2 + 7x – 6 = 0
3x2 + 9x – 2x – 6 = 0

www.ibpsguide.com | estore.ibpsguide.com | www.sscexamguide.com 89


500 Quadratic Equations for IBPS RRB/PO/Clerk

Gives x = -3, 2/3


3y2 – 11y + 6 = 0
3y2 – 9y – 2y + 6 = 0
Gives y = 2/3, 3

286) A
5x2 – 36x – 32 = 0
5x2 + 4x – 40x – 32 = 0
Gives x = -4/5, 8
3y2 + 16y + 20 = 0
3y2 + 6y + 10y + 20 = 0
Gives y = -10/3, -2

287) E
3x2 – (6 + 2√3)x + 4√3 = 0
(3x2 – 6x) – (2√3x – 4√3) = 0
3x (x- 2) – 2√3 (x – 2) = 0,
So x = 2, 2√3/3 (1.15)
3y2 – (2 + 3√3)y + 2√3 = 0
(3y2 – 2y) – (3√3y – 2√3) = 0
y (3y – 2) – √3 (3y – 2) = 0
So x = 2/3, √3 (1.73)

288) B
2x2 + (4 + √2)x + 2√2 = 0
(2x2 + 4x) + (√2x + 2√2) = 0
2x (x + 2) + √2 (x + 2) = 0
So x = -2, -√2/2 (-0.7)
y2 – (1 + 3√3)y + 3√3 = 0
(y2 – y) – (3√3y – 3√3) = 0
y (y – 1) – 3√3 (y – 1) = 0
So, y = 1, 3√3 (5.2)

289) B
x2 + (3 + 2√2)x + 6√2 = 0
(x2 + 3x) + (2√2x + 6√2) = 0
x (x + 3) + 2√2 (x + 3) = 0
So x = -3, -2√2
5y2 – (1 + 5√2)y + √2 = 0
(5y2 – y) – (5√2y – √2) = 0
y (5y – 1) – 3√2 (5y – 1) = 0
So, y = 1/5, 3√2

290) D

www.ibpsguide.com | estore.ibpsguide.com | www.sscexamguide.com 90


500 Quadratic Equations for IBPS RRB/PO/Clerk

2x2 + (4 + 2√6)x + 4√6 = 0


(2x2 + 4x) + (2√6x + 4√6) = 0
2x (x + 2) + 2√6 (x + 2) = 0
So x = -2, -√6
5y2 + (10 + √6)y + 2√6 = 0
(5y2 + 10y) + (√6y + 2√6) = 0
5y (y + 2) + √6 (y + 2) = 0
So, y = -2, -√6/5

Directions(291-300): In the following questions, two equations numbered are given in variables x and y.
You have to solve both the equations and find out the relationship between x and y. Then give answer
accordingly-
A) If x > y
B) If x < y
C) If x ≥ y
D) If x ≤ y
E) If x = y or relation cannot be established

291) I. 3x2 + 10x – 8 = 0,


II. 2y2 – 13y + 6 = 0

292) I. 16x2 + 8x – 15 = 0,
II. 4y2 + 29y + 30 = 0

293) I. 3x2 – 25x + 52 = 0,


II. 15y2 – 38y – 40 = 0

294) I. 12x2 – 5x – 3 = 0,
II. 4y2 – 11y + 6 = 0

295) I. 3x2 + 7x – 6 = 0,
II. 6y2 – y – 2 = 0

296) I. 4x2 + 15x + 9 = 0,


II. 4y2 – 13y – 12 = 0

297) I. 2x2 – (6 + √3)x + 3√3 = 0,


II. 3y2 – (9 + √3)y + 3√3 = 0

298) I. 2x2 – (2 + 2√5)x + 2√5 = 0


II. 4y2 – (6 + 2√2)y + 3√2 = 0

299) I. 2x2 – 15√3x + 84 = 0,


II. 3y2 – 2y – 8 = 0

300) I. 16x2 + 20x + 6 = 0


II. 10y2 + 38y + 24 = 0

291) E

www.ibpsguide.com | estore.ibpsguide.com | www.sscexamguide.com 91


500 Quadratic Equations for IBPS RRB/PO/Clerk

3x2 + 10x – 8 = 0
3x2 + 12x – 2x – 8 = 0
Gives x = -2, 2/3
2y2 – 13y + 6 = 0
2y2 – 12y – y + 6 = 0
Gives y = 1/2, 6

292) C
16x2 + 8x – 15 = 0
16x2 + 20x – 12x – 15 = 0
Gives x = -5/4, 3/4
4y2 + 29y + 30 = 0
4y2 + 24y + 5y + 30 = 0
Gives y = -6, -5/4

293) A
3x2 – 25x + 52 = 0
3x2 – 12x – 13x + 52 = 0
Gives x = 4, 13/3
15y2 – 38y – 40 = 0
15y2 + 12y – 50y – 40 = 0
Gives y = -4/5, 10/3

294) D
12x2 – 5x – 3 = 0
12x2 + 4x – 9x – 3 = 0
Gives x = -1/3, 3/4
4y2 – 11y + 6 = 0
4y2 – 8y – 3y + 6 = 0
Gives y= 3/4, 2

295) E
3x2 + 7x – 6 = 0
3x2 + 9x – 2x – 6 = 0
Gives x = -3, 2/3
6y2 – y – 2 = 0
6y2 + 3y – 4y – 2 = 0
Gives y = -1/2, 2/3
Put on number line
-3… -1/2 … 2/3

296) D
4x2 + 15x + 9 = 0
4x2 + 12x + 3x + 9 = 0
Gives x = -3, -3/4

www.ibpsguide.com | estore.ibpsguide.com | www.sscexamguide.com 92


500 Quadratic Equations for IBPS RRB/PO/Clerk

4y2 – 13y – 12 = 0
4y2 – 16y + 3y – 12 = 0
Gives y = -3/4, 4

297) E
2x2 – 6x – √3x + 3√3 = 0
2x (x- 3) – √3 (x – 3) = 0,
So x = 3, √3/2 (0.7)
3y2 – 9y – √3y + 3√3 = 0
3y (y – 3) – √3 (y – 3) = 0
So x = 3, √3/3 (0.6)

298) E
2x2 – 2x – 2√5x + 2√5 = 0
2x (x – 1) – 2√5 (x – 1) = 0
So x = 1, √5 (2.2)
4y2 – 6y – 2√2y + 3√2 = 0
2y (2y – 3) – √2 (2y – 3) = 0
So, y = 3/2, 1/√2 (0.7)

299) A
2x2 – 15√3x + 84 = 0
Now multiply 2 and 84 = 168
we have √3 in equation, so divide, 168/3 = 56
Now make factors so as by multiply you get 56, and by addition or subtraction you get –15
we have factors (-8) and (-7)
So 2x2 – 15√3x + 84 = 0
gives
2x2 – 8√3x – 7√3x + 84 = 0
2x (x – 4√3) – 7√3 (x – 4√3x) = 0
So x = 3.5√3, 4√3
3y2 – 2y – 8 = 0
3y2 – 6y + 4y – 8 = 0
So y = -4/3, 1
Plot on number line
-4/3……1….. 3.5√3….. 4√3

300) A
Divide both equations by 2
8x2 + 10x + 3 = 0
8x2 + 4x + 6x + 3 = 0
Gives x = -1/2, -3/4
5y2 + 19y + 12 = 0
5y2 + 15y + 4y + 12 = 0
Gives y = -4, -4/5

www.ibpsguide.com | estore.ibpsguide.com | www.sscexamguide.com 93


500 Quadratic Equations for IBPS RRB/PO/Clerk

Directions(301-310): In the following questions, two equations numbered are given in variables x and y.
You have to solve both the equations and find out the relationship between x and y. Then give answer
accordingly-
A) If x > y
B) If x < y
C) If x ≥ y
D) If x ≤ y
E) If x = y or relation cannot be established

301) I. 3x2 + 22x + 24 = 0,


II. 3y2 – 8y – 16 = 0

302) I. 5x2 – 18x – 8 = 0,


II. 2y2 + 11y + 12 = 0

303) I. x2 – 652 = 504,


II. y = √1156

304) I. 9/√x + 8/(√x +1) = 5,


II. 12/√y – 4/√y = 2

305) I. 3x2 – 6x – √3x + 2√3 = 0,


II. 2y2 – 3y – 2√2y + 3√2 = 0,

306) I. x2 – 2x – √5x + 2√5 = 0


II. y2 – 3y – √6y + 3√6 = 0

307) I. 8x2 + 6x + 1 = 0,
II. 5y2 + 8y – 4 = 0

308) I. 4x2 – 23x + 30 = 0,


II. 4y2 – 3y – 45 = 0

309) I. 5x2 – 7x – 6 = 0,
II. 3y2 – 2y – 8 = 0

310) I. 3x2 + 2x – 21 = 0,
II. 3y2 – 19y + 28 = 0

301) D
3x2 + 22 x + 24 = 0
3x2 + 18x + 4x + 24 = 0
Gives x = -4/3, -6
3y2 – 8y – 16 = 0
3y2 – 12y + 4y – 16 = 0
So y = -4/3, 4
Plot on number line
-6…. -4/3……. 4

www.ibpsguide.com | estore.ibpsguide.com | www.sscexamguide.com 94


500 Quadratic Equations for IBPS RRB/PO/Clerk

302) A
5x2 – 18x – 8 = 0
5x2 – 20x + 2x – 8 = 0
So x = -2/5, 4
2y2 + 11y + 12 = 0
2y2 + 8y + 3y + 12 = 0
Gives y = -4, -3/2
Plot on number line
-4… -3/2…. -2/5….. 4

303) D
x2 – 652 = 504
x2 = 1156
So x = 34, -34
y = √1156 = 34
Plot on number line
-34… 34

304) B
9/√x + 8/(√x +1) = 5
[9(√x +1) + 8√x]/[√x * (√x +1)] = 5
17√x + 9 = 5 (x + √x)
5x – 12√x – 9 = 0
5x – 15√x + 3√x – 9 = 0
5√x (√x – 3) + 3 (√x – 3) = 0
√x cannot be -3/3
So √x = 3, so x = 9
12/√y – 4/√y = 2
8/√y = 2
So √y = 4 or y = 16
So y > x

305) E
3x2 – 6x – √3x + 2√3 = 0
3x (x- 2) – √3 (x – 2) = 0,
So x = 2, √3/3
2y2 – 3y – 2√2y + 3√2 = 0
y (2y – 3) – √2 (2y – 3) = 0
So y = 3/2, √2 (1.44)
plot on number line
√3/3(0.57)…….√2…..(3/2)……2

306) B
x2 – 2x – √5x + 2√5 = 0
x (x – 2) – √5 (x – 2) = 0

www.ibpsguide.com | estore.ibpsguide.com | www.sscexamguide.com 95


500 Quadratic Equations for IBPS RRB/PO/Clerk

So x = 2, √5 (2.23)
y2 – 3y – √6y + 3√6 = 0
y (y – 3) – √6 (y – 3) = 0
So y = 3, √6 (2.44)
Plot on number line
2…2.23……2.44…….3

307) E
8x2 + 6x + 1 = 0
8x2 + 4x + 2x + 1 = 0
So x = -1/4, -1/2
5y2 + 8y – 4 = 0
5y2 + 10y – 2y – 4 = 0
So y = -2, 2/5

308) E
4x2 – 23x + 30 = 0
4x2 – 15x – 8x + 30 = 0
So x = 15/4, 2
4y2 – 3y – 45 = 0
4y2 + 12y – 15y – 45 = 0
So y = -3, 15/4
Put on number line
-3…. 2…. 15/4

309) E
5x2 – 7x – 6 = 0
5x2 – 10x + 3x – 6 = 0
So x = -3/5, 2
3y2 – 2y – 8 = 0
3y2 – 6y + 4y – 8 = 0
So y = -4/3, 1
Plot on number line
-4/3……-3/5….. 1….. 2

310) D
3x2 + 2x – 21 = 0
3x2 + 9x – 7x – 21 = 0
Gives x = -3, 7/3
3y2 – 19y + 28 = 0
3y2 – 12y – 7y + 28 = 0
So y = 7/3, 4
Put on number line
-3……7/3……4

www.ibpsguide.com | estore.ibpsguide.com | www.sscexamguide.com 96


500 Quadratic Equations for IBPS RRB/PO/Clerk

Directions(311-320): In the following questions, two equations numbered are given in variables x and y.
You have to solve both the equations and find out the relationship between x and y. Then give answer
accordingly –
A) If x > y
B) If x < y
C) If x ≥ y
D) If x ≤ y
E) If x = y or relation cannot be established

311) I. 20x2 – 31x + 12 = 0,


II. 3y2 – 5y + 2 = 0

312) I. 3x2 – 19x + 30 = 0,


II. 3y2 – 10y + 3 = 0

313) I. 3x2 – 25x + 52 = 0,


II. 5y2 – 18y + 9 = 0

314)I. 4x2 – 5x – 6 = 0,
II. 5y2 – 7y – 6 = 0

315) I. 3x2 – 10x + 8 = 0,


II. 3y2 – 14y + 16 = 0

316) I. 2x2 + 17x + 30 = 0,


II. 4y2 – 7y – 15 = 0

317) I. 3x2 + 16x + 20 = 0,


II. 5y2 + 8y – 4 = 0

318) I. 2x2 + 17x + 21 = 0,


II. 2y2 + 13y + 15 = 0

319) I. 5x2 – 7x – 6 = 0,
II. 3y2 – 19y + 28 = 0

320) I. 8x2 + 6x – 5 = 0,
II. 2y2 + 7y – 4 = 0

311) E
20x2 – 31x + 12 = 0
20x2 – 16x – 15x + 12 = 0
So x = 3/4, 4/5
3y2 – 5y + 2 = 0
3y2 – 3y – 2y + 2 = 0
So y = 1, 2/3

312) C

www.ibpsguide.com | estore.ibpsguide.com | www.sscexamguide.com 97


500 Quadratic Equations for IBPS RRB/PO/Clerk

3x2 – 19x + 30 = 0
3x2 – 9x – 10x + 30 = 0
So x = 3, 10/3
3y2 – 10y + 3 = 0
3y2 – 9y – y + 3 = 0
So y = 1/3, 3

313) A
3x2 – 25x + 52 = 0
3x2 – 12x – 13x + 52 = 0
So x = 4, 13/3
5y2 – 18y + 9 = 0
5y2 – 15y – 3y + 9 = 0
So y = 3/5, 3
Put on number line
-1/2 …. 3/5 ….2/3 …. 3

314) E
4x2 – 5x – 6 = 0
4x2 – 8x + 3x – 6 = 0
So x = -3/4, 2
5y2 – 7y – 6 = 0
5y2 – 10y + 3y – 6 = 0
So y = -3/5, 2

315) D
3x2 – 10x + 8 = 0
3x2 – 6x – 4x + 8 = 0
So x = 2, 4/3
3y2 – 14y + 16 = 0
3y2 – 6y – 8y + 16 = 0
So y = 2, 8/3

316) B
2x2 + 17x + 30 = 0
2x2 + 12x + 5x + 30 = 0
So x = -6, -5/2
4y2 – 7y – 15 = 0
4y2 – 12y + 5y – 15 = 0
So y = -5/4, 3

317) D
3x2 + 16x + 20 = 0
3x2 + 6x + 10x + 20 = 0
So x = -10/3, -2

www.ibpsguide.com | estore.ibpsguide.com | www.sscexamguide.com 98


500 Quadratic Equations for IBPS RRB/PO/Clerk

5y2 + 8y – 4 = 0
5y2 + 10y – 2y – 4 = 0
So y = -2, 2/5

318) E
2x2 + 17x + 21 = 0
2x2 + 14x + 3x + 21 = 0
So x = -7, -3/2
2y2 + 13y + 15 = 0
2y2 + 10y + 3y + 15 = 0
So y = -5, -3/2

319) B
5x2 – 7x – 6 = 0
5x2 – 10x + 3x – 6 = 0
So x = -3/5, 2
3y2 – 19y + 28 = 0
3y2 – 12y – 7y + 28 = 0
So y = 7/3, 4

320) E
8x2 + 6x – 5 = 0
8x2 – 4x + 10x – 5 = 0
So x = -5/4, 1/2
2y2 + 7y – 4 = 0
2y2 + 8y – y – 4 = 0
So y = -4, 1/2

Directions(321-330): In the following questions, two equations numbered are given in variables x and y.
You have to solve both the equations and find out the relationship between x and y. Then give answer
accordingly-
A) x> y
B) x < y
C) x ≥ y
D) x ≤ y
E) x = y or relation cannot be established

321) I. 20x2 – 31x + 12 = 0


II.3y2 – 16y + 16 = 0

322) I. 3x2 + 22 x + 24 = 0
II. 2y2 – 5y – 12 = 0

323) I. 2x2 – 9x + 4 = 0
II. 4y2 – 13y – 12 = 0

www.ibpsguide.com | estore.ibpsguide.com | www.sscexamguide.com 99


500 Quadratic Equations for IBPS RRB/PO/Clerk

324) I. 5x2 + 23x + 12 = 0


II. 5y2 – 7y – 6 = 0

325) I. 7x2 + 19x – 6 = 0,


II. 2y2 – 7y + 3 = 0

326) I. 4x2 – 12x + 5 = 0,


II. 2y2 – 19y + 35 = 0

327) I. 2x2 + 5x ¬– 12 = 0,
II. 4y2 + 13y – 12 = 0

328) I. 3x2 + 22x + 24 = 0,


II. 4y2 – 9y – 9 = 0

329) I. 20x2 – 31x + 12 = 0,


II. 4y2 + 9y – 9 = 0

330) I. 6x2 – 7x – 3 = 0,
II. 4y2 + 5y – 6 = 0

321) B
20x2 – 31x + 12 = 0
20x2 – 16x – 15x + 12 = 0
So x = 3/4, 4/5
3y2 – 16y + 16 = 0
3y2 – 12y – 4y + 16 = 0
Gives y = 4, 4/3

322) E
3x2 + 22 x + 24 = 0
3x2 + 18x + 4x + 24 = 0
So x = -4/3, -6
2y2 – 5y – 12 = 0
2y2 – 8y + 3y – 12 = 0
Gives y = -3/2, 4

323) E
2x2 – 9x + 4 = 0
2x2 – 8x – x + 4 = 0
So x = 4 , 1/2
4y2 – 13y – 12 = 0
4y2 – 16y + 3y – 12 = 0
Gives y = -3/4, 4

324) D
5x2 + 23x + 12 = 0
5x2 + 20x + 3x + 12 = 0

www.ibpsguide.com | estore.ibpsguide.com | www.sscexamguide.com 100


500 Quadratic Equations for IBPS RRB/PO/Clerk

So x = -4, -3/5
5y2 – 7y – 6 = 0
5y2 – 10y + 3y – 6 = 0
So y = -3/5, 2
Put all values on number line and analyze the relationship
-4….. -3/5…. 2

325) B
7x2 + 19x – 6 = 0
7x2 + 21x – 2x – 6 = 0
Gives x = -3, 2/7
2y2 – 7y + 3 = 0
2y2 – 6y – y + 3 = 0
So y = 1/2, 3

326) D
4x2 – 12x + 5 = 0
4x2 – 2x – 10x + 5 = 0
x = 1/2, 5/2
2y2 – 19y + 35 = 0
2y2 – 14y – 5y + 35 = 0
So y = 5/2, 7

327) E
2x2 + 5x ¬12 = 0
2x2 + 8x ¬ 3x – 12 = 0
So x = -4 , 3/2
4y2 + 13y – 12 = 0
4y2 + 16y – 3y – 12 = 0
y = -4, ¾

328) B
3x2 + 22 x + 24 = 0
3x2 + 18x + 4x + 24 = 0
Gives x = -4/3, -6
4y2 – 9y – 9 = 0
4y2 – 12y + 3y – 9 = 0
y = -3/4, 3

329) C
20x2 – 31x + 12 = 0
20x2 – 16x – 15x + 12 = 0
Gives x = 3/4, 4/5
4y2 + 9y – 9 = 0

www.ibpsguide.com | estore.ibpsguide.com | www.sscexamguide.com 101


500 Quadratic Equations for IBPS RRB/PO/Clerk

4y2 + 12y – 3y – 9 = 0
y = 3/4, -3

330) E
6x2 – 7x – 3 = 0
6x2 + 2x – 9x – 3 = 0
Gives x = -1/3, 3/2
4y2 + 5y – 6 = 0
4y2 + 8y – 3y – 6 = 0
Gives y = -2, 3/4

Directions (Q. 331 - 340): You have to solve equation I and II ,Give answer
A) If X > Y
B) If X < Y
C) If X ≥ Y
D) If X ≤ Y
E) If X = Y or cannot be established

331). I. 2X2 + 11X + 12= 0


II.5Y2 + 27Y + 10 = 0

332). I. 25X2 + 25X + 6= 0


II.5Y2 + 20Y + 20 = 0

333). I. 3X2 + 7X + 4= 0
II.3Y2 + 10Y + 8 = 0

334). I.(x+y)2 = 900


II.Y + 1689 = 170

335). I.8X2 + 3X – 38= 0


II.6Y2 – 29Y + 34 = 0

336). 9x – 3x = 64.55 + 19.45


√(y *155) = 7 + 6

337). I.2X2 + 19X + 44= 0


II.2Y2 + 3Y – 20 = 0

338). I.P2 + 9P = 2P – 12
II.4Q2 + 8Q = 4Q + 8

339). I.2X2 – 7X + 6= 0
II.4Y2 = 9

340).I. X2 = 1296
II.Y = √1296

331) E

www.ibpsguide.com | estore.ibpsguide.com | www.sscexamguide.com 102


500 Quadratic Equations for IBPS RRB/PO/Clerk

3x2 + 22 x + 24 = 0
3x2 + 18x + 4x + 24 = 0
Gives x = -4/3, -6
2y2 + 11y + 12 = 0
2y2 + 8y + 3y + 12 = 0
Gives y = -4, -3/2
Put all values on number line and analyze the relationship
-6…..-4…. -3/2….-4/3

332) B
3x2 + 7x – 6 = 0
3x2 + 9x – 2x – 6 = 0
Gives x = -3, 2/3
6y2 – 35y + 50 = 0
6y2 – 15y – 20y + 50 = 0
Gives y = 5/2, 10/3
Put all values on number line and analyze the relationship
-3… 2/3… 5/2…. 10/3

333) D
4x2 + 13x + 10 = 0
4x2 + 8x + 5x + 10 = 0
Gives x = -2, -5/4
4y2 – 7y – 15 = 0
4y2 – 12y + 5y – 15 = 0
Gives y = -5/4, 3
Put all values on number line and analyze the relationship
-2… -5/4…. 3

334) B
3x2 + 23x + 30 = 0
3x2 + 18x + 5x + 30 = 0
Gives x = -5/3, -6
3y2 – 4y – 4 = 0
3y2 – 6y + 2y – 4 = 0
Gives y = 2, -2/3
Put all values on number line and analyze the relationship
-6…. -5/3….. -2/3…… 2

335) B
6x2 + 5x – 1 = 0
6x2 + 6x – x – 1 = 0
Gives x = -1, 1/6
3y2 – 11y + 6 = 0
3y2 – 9y – 2y + 6 = 0

www.ibpsguide.com | estore.ibpsguide.com | www.sscexamguide.com 103


500 Quadratic Equations for IBPS RRB/PO/Clerk

Gives y = 2/3, 3
Put on number line
-1… 1/6… 2/3… 3

336) E
3x2 + 4x – 4 = 0
3x2 + 6x – 2x – 4 = 0
Gives x = -2, 2/3
4y2 + 5y – 6 = 0
4y2 + 5y – 6 = 0
Gives y = -2, 3/4
Put on number line
-2…. 2/3… 3/4
When x=2/3, x>y(= -2) and x<y(= 3/4)
So cant be determined

337) E
5x2 – 36x – 32 = 0
5x2 + 4x – 40x – 32 = 0
Gives x = -4/5, 8
3y2 – 17y – 6 = 0
3y2 + y – 18y – 6 = 0
Gives y= -1/3, 6
Put on number line
-4/5…. -1/3… 6… 8

338) A
3x2 – 25x + 52 = 0
3x2 – 12x – 13x + 52 = 0
Gives x = 4, 13/3
15y2 – 38y – 40 = 0
15y2 + 12y – 50y – 40 = 0
Gives y = -4/5, 10/3
Put on number line
-4/5… 10/3… 4… 13/3

339) A
6x2 + x – 2 = 0
6x2 + 4x – 3x – 2 = 0
Gives x = -2/3, 1/2
2y2 + 11y + 14 = 0
2y2 + 4y + 7y + 14 = 0
Gives y = -7/2, -2

340) D

www.ibpsguide.com | estore.ibpsguide.com | www.sscexamguide.com 104


500 Quadratic Equations for IBPS RRB/PO/Clerk

3x2 + 14x – 5 = 0
3x2 + 15x – x – 5 = 0
Gives x = -5, 1/3
3y2 – 19y + 6 = 0
3y2 – 18y – y + 6 = 0
Gives y = 1/3, 6
Put on number line
-5…. 1/3… 6

Directions(341-350): In the following questions, two equations numbered are given in variables x and y.
You have to solve both the equations and find out the relationship between x and y. Then give answer
accordingly-
341) I. 3x2 + 22 x + 24 = 0
II. 2y2 + 11y + 12 = 0

342) I. 3x2 + 7x – 6 = 0
II. 6y2 – 35y + 50 = 0

343) I. 4x2 + 13x + 10 = 0


II. 4y2 – 7y – 15 = 0

344) I. 3x2 + 23x + 30 = 0


II. 3y2 – 4y – 4 = 0

345) I. 6x2 + 5x – 1 = 0,
II. 3y2 – 11y + 6 = 0

346) I. 3x2 + 4x – 4 = 0,
II. 4y2 + 5y – 6 = 0

347) I. 5x2 – 36x – 32 = 0,


II. 3y2 – 17y – 6 = 0

348)I. 3x2 – 25x + 52 = 0,


II. 15y2 – 38y – 40 = 0

349) I. 6x2 + x – 2 = 0,
II. 2y2 + 11y + 14 = 0

350) I. 3x2 + 14x – 5 = 0,


II. 3y2 – 19y + 6 = 0

341) E
3x2 + 22 x + 24 = 0
3x2 + 18x + 4x + 24 = 0
Gives x = -4/3, -6
2y2 + 11y + 12 = 0
2y2 + 8y + 3y + 12 = 0
Gives y = -4, -3/2

www.ibpsguide.com | estore.ibpsguide.com | www.sscexamguide.com 105


500 Quadratic Equations for IBPS RRB/PO/Clerk

Put all values on number line and analyze the relationship


-6…..-4…. -3/2….-4/3

342) B
3x2 + 7x – 6 = 0
3x2 + 9x – 2x – 6 = 0
Gives x = -3, 2/3
6y2 – 35y + 50 = 0
6y2 – 15y – 20y + 50 = 0
Gives y = 5/2, 10/3
Put all values on number line and analyze the relationship
-3… 2/3… 5/2…. 10/3

343) D
4x2 + 13x + 10 = 0
4x2 + 8x + 5x + 10 = 0
Gives x = -2, -5/4
4y2 – 7y – 15 = 0
4y2 – 12y + 5y – 15 = 0
Gives y = -5/4, 3
Put all values on number line and analyze the relationship
-2… -5/4…. 3

344) B
3x2 + 23x + 30 = 0
3x2 + 18x + 5x + 30 = 0
Gives x = -5/3, -6
3y2 – 4y – 4 = 0
3y2 – 6y + 2y – 4 = 0
Gives y = 2, -2/3
Put all values on number line and analyze the relationship
-6…. -5/3….. -2/3…… 2

345) B
6x2 + 5x – 1 = 0
6x2 + 6x – x – 1 = 0
Gives x = -1, 1/6
3y2 – 11y + 6 = 0
3y2 – 9y – 2y + 6 = 0
Gives y = 2/3, 3
Put on number line
-1… 1/6… 2/3… 3

346) E

www.ibpsguide.com | estore.ibpsguide.com | www.sscexamguide.com 106


500 Quadratic Equations for IBPS RRB/PO/Clerk

3x2 + 4x – 4 = 0
3x2 + 6x – 2x – 4 = 0
Gives x = -2, 2/3
4y2 + 5y – 6 = 0
4y2 + 5y – 6 = 0
Gives y = -2, 3/4
Put on number line
-2…. 2/3… 3/4
When x=2/3, x>y(= -2) and x<y(= 3/4)
So cant be determined

347) E
5x2 – 36x – 32 = 0
5x2 + 4x – 40x – 32 = 0
Gives x = -4/5, 8
3y2 – 17y – 6 = 0
3y2 + y – 18y – 6 = 0
Gives y= -1/3, 6
Put on number line
-4/5…. -1/3… 6… 8

348) A
3x2 – 25x + 52 = 0
3x2 – 12x – 13x + 52 = 0
Gives x = 4, 13/3
15y2 – 38y – 40 = 0
15y2 + 12y – 50y – 40 = 0
Gives y = -4/5, 10/3
Put on number line
-4/5… 10/3… 4… 13/3

349) A
6x2 + x – 2 = 0
6x2 + 4x – 3x – 2 = 0
Gives x = -2/3, 1/2
2y2 + 11y + 14 = 0
2y2 + 4y + 7y + 14 = 0
Gives y = -7/2, -2

350) D
3x2 + 14x – 5 = 0
3x2 + 15x – x – 5 = 0
Gives x = -5, 1/3
3y2 – 19y + 6 = 0
3y2 – 18y – y + 6 = 0

www.ibpsguide.com | estore.ibpsguide.com | www.sscexamguide.com 107


500 Quadratic Equations for IBPS RRB/PO/Clerk

Gives y = 1/3, 6
Put on number line
-5…. 1/3… 6

Directions(351-360): In the following questions, two equations numbered are given in variables x and y.
You have to solve both the equations and find out the relationship between x and y. Then give answer
accordingly-
A) x> y
B) x < y
C) x ≥ y
D) x ≤ y
E) x = y or relation cannot be established

351) I. 8/√x + 9/(√x +1) = 7,


II. 9/√y – 3/√y = 2

352) I. 9/√x + 3/√x = √x + 1,


II. 4y2 + 5y – 6 = 0

353) I. 6x2 + 13x + 6 = 0,


II. 6y2 – y – 2 = 0

354) I. 3x2 + 14x – 5 = 0,


II. 3y2 – 11y + 6 = 0

355) I. 6x2 + 5x – 1 = 0,
II. 3y2 – 10y + 3 = 0

356) I. 12x2 – 5x – 3 = 0,
II. 3y2 – 11y + 6 = 0

357) I. 6x2 + 7x + 2 = 0,
II. 15y2 – 38y – 40 = 0

358) I. 3x2 – 25x + 52 = 0,


II. 2y2 – 7y + 3 = 0

359) I. x2 = 1156,
II. y = √1156

360) I. x2 – √3969 = √6561,


II. y2 – √1296 = √4096

351) B
8/√x + 9/(√x +1) = 7
[8(√x +1) + 9√x]/[√x * (√x +1)] = 7
17√x + 8 = 7 (x + √x)
7x – 10√x – 8 = 0
7x – 14√x + 4√x – 8 = 0
7√x (√x – 2) + 4 (√x – 2) = 0

www.ibpsguide.com | estore.ibpsguide.com | www.sscexamguide.com 108


500 Quadratic Equations for IBPS RRB/PO/Clerk

√x cannot be -4/7
So √x = 2, so x = 4
9/√y – 3/√y = 2
(9 – 3)/√y = 2
Gives √y = 3, so y = 9

352) A
9/√x + 3/√x = √x + 1
12/√x = √x + 1
x + √x – 12 = 0
x + 4√x – 3√x – 12 = 0
√x(√x + 4) – 3 (√x + 4) = 0
√x cannot be -4, So √x = 3 => x = 9
4y2 + 5y – 6 = 0
4y2 + 5y – 6 = 0
Gives y = -2, 3/4
Put all values on number line and analyze the relationship
-2… 3/4… 9

353) B
6x2 + 13x + 6 = 0
6x2 + 9x + 4x + 6 = 0
Gives x = -2/3, -3/2
6y2 – y – 2 = 0
6y2 + 3y – 4y – 2 = 0
Gives y = -1/2, 2/3
Put all values on number line and analyze the relationship
-3/2… -2/3… -1/2… 2/3

354) B
3x2 + 14x – 5 = 0
3x2 + 15x – x – 5 = 0
Gives x = -5, 1/3
3y2 – 11y + 6 = 0
3y2 – 9y – 2y + 6 = 0
Gives y = 2/3, 3
Put all values on number line and analyze the relationship
-5… 1/3… 2/3… 3

355) B
6x2 + 5x – 1 = 0
6x2 + 6x – x – 1 = 0
Gives x = -1, 1/6
3y2 – 10y + 3 = 0
3y2 – 9y – y + 3 = 0

www.ibpsguide.com | estore.ibpsguide.com | www.sscexamguide.com 109


500 Quadratic Equations for IBPS RRB/PO/Clerk

Gives y = 1/3, 3
Put all values on number line and analyze the relationship
-1… 1/6… 1/3… 3

356) E
12x2 – 5x – 3 = 0
12x2 + 4x – 9x – 3 = 0
Gives x = -1/3, 3/4
3y2 – 11y + 6 = 0
3y2 – 9y – 2y + 6 = 0
Gives y = 2/3, 3
Put all values on number line and analyze the relationship
-1/3… 2/3… 3/4… 3

357) E
6x2 + 7x + 2 = 0
6x2 + 4x + 3x + 2 = 0
Gives x = -2/3, -1/2
15y2 – 38y – 40 = 0
15y2 + 12y – 50y – 40 = 0
Gives y = -4/5, 10/3
Put all values on number line and analyze the relationship
-4/5… -2/3… -1/2… 10/3

358) A
3x2 – 25x + 52 = 0
3x2 – 12x – 13x + 52 = 0
Gives x = 4, 13/3
2y2 – 7y + 3 = 0
2y2 – 6y – y + 3 = 0
So y = 1/2, 3
Put all values on number line and analyze the relationship
1/2… 3… 4… 13/3

359) D
x2 = 1156,
So x = -34, 34
y = √1156
So y = 34
Put all values on number line and analyze the relationship
-34… 34

360) E
x2 – √3969 = √6561
x2 – 63 = 81

www.ibpsguide.com | estore.ibpsguide.com | www.sscexamguide.com 110


500 Quadratic Equations for IBPS RRB/PO/Clerk

x2 = 144
So x = -12, 12
y2 – √1296 = √4096
y2 – 36 = 64
y2 = 100
So y = -10, 10
Put all values on number line and analyze the relationship
-12… -10….10…. 12

361) I. 2x2 – 15√3x + 84 = 0


II. 3y2 – 10√3y + 9 = 0
A) x > y
B) x < y
C) x ≥ y
D) x ≤ y
E) x = y or relation cannot be established

362) I. x2 + √5x – 10 = 0
II. 2y2 + 9√5y + 50 = 0
A) x > y
B) x < y
C) x ≥ y
D) x ≤ y
E) x = y or relation cannot be established

363) I. 2x2 – (8+√3)x + 4√3 = 0


II. 3y2 – (6+2√3)y + 4√3 = 0
A) x > y
B) x < y
C) x ≥ y
D) x ≤ y
E) x = y or relation cannot be established

364) I. x2 – (2+√5)x + 2√5 = 0


II. 2y2 – (6+3√5)y + 9√5 = 0
A) x > y
B) x < y
C) x ≥ y
D) x ≤ y
E) x = y or relation cannot be established

365) I. 3x2 + 5√2x – 24 = 0


II. y2 – 6√2y + 16 = 0
A) x > y
B) x < y
C) x ≥ y
D) x ≤ y
E) x = y or relation cannot be established

366) I. 3x2 – 23x + 40 = 0


II. 3y2 – 8y + 4 = 0

www.ibpsguide.com | estore.ibpsguide.com | www.sscexamguide.com 111


500 Quadratic Equations for IBPS RRB/PO/Clerk

A) x > y
B) x < y
C) x ≥ y
D) x ≤ y
E) x = y or relation cannot be established

367) I. 5x2 – 17x + 6 = 0


II. 4y2 – 16y + 7 = 0
A) x > y
B) x < y
C) x ≥ y
D) x ≤ y
E) x = y or relation cannot be established

368) I. 3x2 – 14x + 8 = 0


II. 3y2 – 20y + 12 = 0
A) x > y
B) x < y
C) x ≥ y
D) x ≤ y
E) x = y or relation cannot be established

369) I. 12x2 + 25x + 12 = 0


II. 3y2 + 22y + 24 = 0
A) x > y
B) x < y
C) x ≥ y
D) x ≤ y
E) x = y or relation cannot be established

370) I. 6x2 + x – 2 = 0
II. 3y2 – 22y + 40 = 0
A) x > y
B) x < y
C) x ≥ y
D) x ≤ y
E) x = y or relation cannot be established

361) A
2x2 – 15√3x + 84 = 0
Now multiply 2 and 84 = 168
we have √3 in equation, so divide, 168/3 = 56
Now make factors so as by multiply you get 56, and by addition or subtraction you get –15
we have factors (-8) and (-7)
So 2x2 – 15√3x + 84 = 0
gives
2x2 – 8√3x – 7√3x + 84 = 0
2x (x – 4√3) – 7√3 (x – 4√3x) = 0
So x = 7√3/2, 4√3
Similarly for

www.ibpsguide.com | estore.ibpsguide.com | www.sscexamguide.com 112


500 Quadratic Equations for IBPS RRB/PO/Clerk

3y2 – 10√3y + 9 = 0
Multiply 3 and 9 = 27
we have √3 in equation, so divide, 27/3 = 9
Now make factors so as by multiply you get 9, and by addition or subtraction you get –10
we have factors (-9) and (-1)
So 3y2 – 10√3y + 9 = 0
gives
3y2 – 9√3y – √3y + 9 = 0
3x (x – 3√3) – √3 (x – 3√3x) = 0
Put all values on number line and analyze the relationship
√3/3 …. 3√3 ….. 7√3/2 …… 4√3

362) C
x2 + √5x – 10 = 0
x2 + 2√5x – √5x – 10 = 0
Gives x = -2√5, √5
2y2 + 9√5y + 50 = 0
2y2 + 4√5y + 5√5y + 50 = 0
Gives y = -2√5, -5√5/2
Put all values on number line and analyze the relationship
-5√5/2….. -2√5….. √5

363) E
2x2 – (8+√3)x + 4√3 = 0
By multiplying we have to 2*4√3 = 8√3 and by adding/subtracting we have to get – (8+√3)
So factors are -8 and -√3
So 2x2 – (8+√3)x + 4√3 = 0
Gives
2x2 – 8x – √3x + 4√3 = 0
2x(x- 4) – √3(x – 4) = 0
So x = 4, √3/2
NEXT
3y2 – (6+2√3)y + 4√3 = 0
By multiplying we have to 3*4√3 = 12√3 and by adding/subtracting we have to get –(6+2√3)
So factors are -6 and -2√3
So 3y2 – (6+2√3)y + 4√3 = 0
Gives
3y2 – 6y – 2√3y + 4√3 = 0
3y(y- 2) – 2√3(y – 2) = 0
So x = 2, 2√3/3
Put all values on number line and analyze the relationship
√3/2…… 2√3/3…… 2… 4

364) B

www.ibpsguide.com | estore.ibpsguide.com | www.sscexamguide.com 113


500 Quadratic Equations for IBPS RRB/PO/Clerk

x2 – (2+√5)x + 2√5 = 0
By multiplying we have to 2√5 and by adding/subtracting we have to get – (2+√5)
So factors are -2 and -√5
So x2 – (2+√5)x + 2√5 = 0
Gives
x2 – 2x – √5x + 2√5 = 0
x(x- 2) – √5(x – 2) = 0
So x = 2, √5
NEXT
2y2 – (6+3√5)y + 9√5 = 0
By multiplying we have to 2*9√5 = 18√5 and by adding/subtracting we have to get –(6+3√5)
So factors are -6 and -3√5
So 2y2 – (6+3√5)y + 9√5 = 0
Gives
2y2 – 6y – 3√5y + 9√5 = 0
2y(y- 3) – 3√5(y – 3) = 0
So x = 3, 3√5/2
Put all values on number line and analyze the relationship
2…… √5…… 3… 3√5/2

365) B
3x2 + 5√2x – 24 = 0
3x2 + 9√2x – 4√2x – 24 = 0
Gives x = -3√2, 4√2/3
y2 – 6√2y + 16 = 0
y2 – 2√2y – 4√2y + 16 = 0
Gives y = 2√2, 4√2
Put all values on number line and analyze the relationship
3√2……. 4√2/3…… 2√2….. 4√2

366) A
3x2 – 23x + 40 = 0
3x2 – 15x – 8x + 40 = 0
Gives x = 5, 8/3
3y2 – 8y + 4 = 0
3y2 – 6y – 2y + 4 = 0
Gives y = 2/3, 2
Put all values on number line and analyze the relationship
2/3….. 2….. 8/3….. 5

367) E
5x2 – 17x + 6 = 0
5x2 – 15x – 2x + 6 = 0
Gives x = 2/5, 3
4y2 – 16y + 7 = 0

www.ibpsguide.com | estore.ibpsguide.com | www.sscexamguide.com 114


500 Quadratic Equations for IBPS RRB/PO/Clerk

4y2 – 2y – 14y + 7 = 0
Gives y = 1/2, 7/2
Put all values on number line and analyze the relationship
2/5….. 1/2….. 3…. 7/2

368) E
3x2 – 14x + 8 = 0
3x2 – 12x – 2x + 8 = 0
Gives x = 4, 2/3
3y2 – 20y + 12 = 0
3y2 – 18y – 2y + 12 = 0
Gives y = 2/3, 6
Put all values on number line and analyze the relationship
2/3…….. 4….. 6

369) C
12x2 + 25x + 12 = 0
12x2 + 16x + 9x + 12 = 0
Gives x = -4/3, -3/4
3y2 + 22y + 24 = 0
3y2 + 18y + 4y + 24 = 0
Gives y = -4/3, -6
Put all values on number line and analyze the relationship
-6…… -4/3…… -3/4

370) B
6x2 + x – 2 = 0
6x2 + 4x – 3x – 2 = 0
Gives x = 1/2, -2/3
3y2 – 22y + 40 = 0
3y2 – 12y – 10y + 40 = 0
Gives y = 10/3, 4
Put all values on number line and analyze the relationship
-2/3…… 1/2…… 10/3….. 4

Directions(371-780): In the following questions, two equations numbered are given in variables x and y.
You have to solve both the equations and find out the relationship between x and y. Then give answer
accordingly.
A) x> y
B) x < y
C) x ≥ y
D) x ≤ y
E) x = y or relation cannot be established

371) I. 3x2 – 17x + 10 = 0


II. 3y2 + 4y – 4 = 0

www.ibpsguide.com | estore.ibpsguide.com | www.sscexamguide.com 115


500 Quadratic Equations for IBPS RRB/PO/Clerk

372) I. 3x2 – 14x + 8 = 0


II. 3y2 – 20y + 12 = 0

373) I. 3x2 – 19x + 28 = 0


II. 4y2 – 5y – 6 = 0

374) I. 6x2 + 23x + 21 = 0


II. 3y2 – 14y – 5 = 0

375) I. 2x2 – 7x + 3 = 0
II. 2y2 + 11y + 12 = 0

376) I. 3x2 + 22x + 35 = 0


II. 6y2 + 11y – 7 = 0

377) I. 2x2 – 3x – 9 = 0
II. 3y2 + 11y + 6 = 0

378) I. x2 + 14x + 45 = 0
II. 3y2 – y – 10 = 0

379) I. 4x2 + 17x + 15 = 0


II. 4y2 – 3y – 10 = 0

380) I. 2x2 – 17x + 36 = 0


II. 3y2 – 14y + 8 = 0

371) C
3x2 – 17x + 10 = 0
3x2 – 15x – 2x + 10 = 0
Gives x = 2/3, 5
3y2 + 4y – 4 = 0
3y2 + 6y – 2y – 4 = 0
Gives y = -2, 2/3
Put all values on number line and analyze the relationship
-2…. 2/3…… 5

372) E
3x2 – 14x +8 = 0
3x2 – 12x – 2x + 8 = 0
Gives x = 2/3, 4
3y2 – 20y + 12 = 0
3y2 – 18y – 2y + 12 = 0
Gives y = 6, 2/3
Put all values on number line and analyze the relationship
2/3….. 4….. 6

373) A

www.ibpsguide.com | estore.ibpsguide.com | www.sscexamguide.com 116


500 Quadratic Equations for IBPS RRB/PO/Clerk

4x2 – 19x + 28 = 0
4x2 – 12x – 7x + 28 = 0
Gives x = 7/3, 4
4y2 – 5y – 6 = 0
4y2 – 8y + 3y – 6 = 0
Gives y = -3/4, 2
Put all values on number line and analyze the relationship
-3/4…… 2…… 7/3…. 4

374) B
6x2 + 23x + 21 = 0
6x2 + 9x + 14x + 21 = 0
Gives x = -7/3, -3/2
3y2 – 14y – 5 = 0
3y2 – 15y + y – 5 = 0
Gives y = -1/3, 5
Put all values on number line and analyze the relationship
-7/3…… -3/2….. -1/3…… 5

375) A
2x2 – 7x + 3 = 0
2x2 – 6x – x + 3 = 0
Gives x = 3, 1/2
2y2 + 11y + 12 = 0
2y2 + 8y + 3y + 12 = 0
Gives y = -3/2, -4
Put all values on number line and analyze the relationship
-4……. -3/2…… 1/2….. 3

376) D
3x2 + 22x + 35 = 0
3x2 + 15x + 7x + 35 = 0
Gives x = -5, -7/3
6y2 + 11y – 7 = 0
6y2 – 3y + 14y – 7 = 0
Gives y = 1/2, -7/3
Put all values on number line and analyze the relationship
-5….. -7/3….. ½

377) E
2x2 – 3x – 9 = 0
2x2 – 6x + 3x – 9 = 0
Gives x = -3/2, 3
3y2 + 11y + 6 = 0
3y2 + 9y + 2y + 6 = 0

www.ibpsguide.com | estore.ibpsguide.com | www.sscexamguide.com 117


500 Quadratic Equations for IBPS RRB/PO/Clerk

Gives y = -3, -2/3


Put all values on number line and analyze the relationship
-3….. -3/2….. -2/3…. 3

378) B
x2 + 14x + 45 = 0
x2 + 9x + 5x + 45 = 0
Gives x = -9, -5
3y2 – y – 10 = 0
3y2 – 6y + 5y – 10 = 0
Gives y = -5/3, 2
Put all values on number line and analyze the relationship
-9…….. -5….. -5/3……. 2

379) D
4x2 + 17x + 15 = 0
4x2 + 12x + 5x + 15 = 0
Gives x = -3, -5/4
4y2 – 3y – 10 = 0
4y2 – 8y + 5y – 10 = 0
Gives y = -5/4, 2
Put all values on number line and analyze the relationship
-3…… -5/4…… 2

380) C
2x2 – 17x + 36 = 0
2x2 – 8x – 9x + 36 = 0
Gives x = 4, 9/2
3y2 – 14y + 8 = 0
3y2 – 12y + 2y + 8 = 0
Gives y = 2/3, 4
Put all values on number line and analyze the relationship
2/3…… 4…… 9/2

Directions (381-385): In each of these questions, two equations (I) and (II) are given. You have to solve
both the equations and give answer
a) if x > y
b) if x < y
c) if x ≥ y
d) if x ≤ y
e) if x = y or relation cannot be established between 'x’and 'y'.

381) I. 8x+y=10
II. 4x+2y=13

382) I. (x+3) (y+2)=12


II. 2xy+4x+5y=11

www.ibpsguide.com | estore.ibpsguide.com | www.sscexamguide.com 118


500 Quadratic Equations for IBPS RRB/PO/Clerk

383) I. (3x-2)/y = (3x+6)/(y+16)


II.(x+2)/(y+4) = (x+5)/(Y+10)

384) I. x^2+20x+4=50-25x
II. y^2-10y+24=0

385) I. (x^2-10x+16)/(x^2 -12x +24) = 2/3


II. y^2-y-20=0

Directions (386-390): In each of these questions, two equations (I) and (II) are given. You have to solve
both the equations and give answer
a) if x < y
b) if x > y
c) if x ≤ y
d) if x≥ y
e) if x = y, or relationship between x and y can't be established.

386) I. 6x^2-49x+99=0
II. 5y^2+17y+14=0

387) I. 5x^2=19x-12
II. 5y^2+11y=12

388) I. x^3=(1331)
II. 2y^2-21y+55=0

389) I. 5x=7y+21
II. 11x+4y+109=0

390) I. 2x^2-11x+12=0
II. 2y^2-17y+36=0

381) B
from both equation
x=7/12, y=16/3
y>x

382) E
xy+3y+2x +6 =12
2xy+6y+4x=12----(i)
2xy+5y+4x=11 ---- (ii)
From eq. (i) --- (ii)
Y=1
From eq. (i)
x=1
x=y

383) B
(3x-2)/y = (3x+6)/(y+16)

www.ibpsguide.com | estore.ibpsguide.com | www.sscexamguide.com 119


500 Quadratic Equations for IBPS RRB/PO/Clerk

48x-8y = 32 ---- (i)


(x+2)/(y+4) = (x+5)/(y+10)
y = 2x ---- (i)
From Equation (i) & (ii)
x=1, y=2
y>x

384) B
From the given Equation
x=1, -46
& y=-2,/2
x<y

385) E
From 1st equation
x^2-6x=0
x=0,6
From 2nd equation
(y+4) (y-5)
y=-4,5
x#y

386) B
6x^2-49x+99=0
(3x-11) (2x-9)=0
x=11/3, 9/2
5^2+17y+14=>
(5y+7) (y+2)=>
y=-2,-7/5
x>y

387) D
5x^2-19x+12=0
x=3,4/5
5y^2+11y=12
y = 4/5 ,-3

388) B
x=11
2y^2-21y+55= 0
(2y-11) (y-5)= 0
y=5,11/2
x>y

389) B

www.ibpsguide.com | estore.ibpsguide.com | www.sscexamguide.com 120


500 Quadratic Equations for IBPS RRB/PO/Clerk

From given equation


x=-7
y=-8
x>Y

390) C
2x^2-11x+12=>
x=3/2,4
2y^2-17y+36=>
y=4,9/2

391) x² – 10x + 24 = 0
y² – 14y + 48 = 0
A. X > Y
B. X < Y
C. X ≥ Y
D. X ≤ Y
E. X = Y or relation cannot be established

392) x² – 30x + 216 = 0


y² – 23y + 132 = 0
A. X > Y
B. X < Y
C. X ≥ Y
D. X ≤ Y
E. X = Y or relation cannot be established

393) x² + 32x + 247 = 0


y² + 20y + 91 = 0
A. X > Y
B. X < Y
C. X ≥ Y
D. X ≤ Y
E. X = Y or relation cannot be established

394) x² – 21x + 98 = 0
y² – 23y + 120 = 0
A. X > Y
B. X < Y
C. X ≥ Y
D. X ≤ Y
E. X = Y or relation cannot be established

395) (x – 16)² = 0
y² = 256
A. X > Y
B. X < Y
C. X ≥ Y
D. X ≤ Y
E. X = Y or relation cannot be established

www.ibpsguide.com | estore.ibpsguide.com | www.sscexamguide.com 121


500 Quadratic Equations for IBPS RRB/PO/Clerk

396) x² – 43x + 450 = 0


y² – 33y + 272 = 0
A. X > Y
B. X < Y
C. X ≥ Y
D. X ≤ Y
E. X = Y or relation cannot be established

397) x² – 28x + 195 = 0


y² – 35y + 306 = 0
A. X > Y
B. X < Y
C. X ≥ Y
D. X ≤ Y
E. X = Y or relation cannot be established

398) x² – 38x + 345 = 0


y² – 23y + 130 = 0
A. X > Y
B. X < Y
C. X ≥ Y
D. X ≤ Y
E. X = Y or relation cannot be established

399)x² = 64
y² – 30y + 225 = 0
A. X > Y
B. X < Y
C. X ≥ Y
D. X ≤ Y
E. X = Y or relation cannot be established

400) x² – 30x + 221 = 0


y² – 31y + 240 = 0
A. X > Y
B. X < Y
C. X ≥ Y
D. X ≤ Y
E. X = Y or relation cannot be established

391) D
x² – 10x + 24 = 0
x = 4, 6
y² – 14y + 48 = 0
y = 6, 8

392) C
x² – 30x + 216 = 0
x = 12, 18

www.ibpsguide.com | estore.ibpsguide.com | www.sscexamguide.com 122


500 Quadratic Equations for IBPS RRB/PO/Clerk

y² – 23y + 132 = 0
y = 12, 11

393) D
x² + 32x + 247 = 0
x = -13, -19
y² + 20y + 91 = 0
y = -13, -7

394) E
x² – 21x + 98 = 0
x = 14, 7
y² – 23y + 120 = 0
y = 15, 8

395) C
x² – 32x + 256 = 0
x = 16, 16
y² = 256
y = ±16

396) A
x² – 43x + 450 = 0
x = 25, 18
y² – 33y + 272 = 0
y = 17, 16

397) B
x² – 28x + 195 = 0
x = 13, 15
y² – 35y + 306 = 0
y = 17, 18

398) A
x² – 38x + 345 = 0
x = 23, 15
y² – 23y + 130 = 0
y = 10, 13

399) B
x² = 64
x = 8, – 8
y² – 30y + 225 = 0
y = 15, 15

www.ibpsguide.com | estore.ibpsguide.com | www.sscexamguide.com 123


500 Quadratic Equations for IBPS RRB/PO/Clerk

400) E
x² – 30x + 221 = 0
x = 13, 17
y² – 31y + 240 = 0
y = 15, 16

Directions (401-410): In each of these questions, two equations (I) and (II) are given. You have to solve
both the equations and give answer
A. X > Y
B. X < Y
C. X ≥ Y
D. X ≤ Y
E. X = Y or relation cannot be established

401. 4x² + 8x + 3 = 0
4y² – 29y + 45 = 0

402. 2x² – 23x + 21 = 0


y² + 42y + 272 = 0

403. 5x² – 26x + 21 = 0


2y² – 17y + 21 = 0

404. x² – 21x + 104 = 0


y² – 33y + 260 = 0

405. x² – 31x + 240 = 0


y² – 28y + 195 = 0

406. x² – 37x + 300 = 0


y² – 43y + 372 = 0

407. x² – 32x + 255 = 0


y² – 28y + 195 = 0

408. x² + 39x + 368 = 0


y² – 15y – 250 = 0

409. 4x² + 25x + 21 = 0


3y² + 29y + 56 = 0

410. x² – 17x + 72 = 0
6y² – 31y + 35 = 0

401). B. X < Y
4x² + 8x + 3 = 0
x = -0.5, -3.5
4y² – 29y + 45 = 0
y = 2.25, 5

www.ibpsguide.com | estore.ibpsguide.com | www.sscexamguide.com 124


500 Quadratic Equations for IBPS RRB/PO/Clerk

402). A. X > Y
2x² – 23x + 21 = 0
x = 10.5, 2
y² + 42y + 272 = 0
y = -16, -17

403). E. X = Y or relation cannot be established


5x² – 26x + 21 = 0
x = 4.2, 1
2y² – 17y + 21 = 0
y = 7, 1.5

404). D. X ≤ Y
x² – 21x + 104 = 0
x = 13, 8
y² – 33y + 260 = 0
y = 13, 20

405). C. X ≥ Y
x² – 31x + 240 = 0
x = 15, 16
y² – 28y + 195 = 0
y = 13, 15

406). E. X = Y or relation cannot be established


x² – 37x + 300 = 0
x = 25, 12
y² – 43y + 372 = 0
y = 31, 12

407). C. X ≥ Y
x² – 32x + 255 = 0
x = 15, 17
y² – 28y + 195 = 0
y = 15, 13

408). B. X < Y
x² + 39x + 368 = 0
x = -23, -16
y² – 15y – 250 = 0
y = 25, – 10

409). E. X = Y or relation cannot be established


4x² + 25x + 21 = 0
x = -1, – 5.25

www.ibpsguide.com | estore.ibpsguide.com | www.sscexamguide.com 125


500 Quadratic Equations for IBPS RRB/PO/Clerk

3y² + 29y + 56 = 0
y = -7, -2.6

410). A. X > Y
x² – 17x + 72 = 0
x = 8, 9
6y² – 31y + 35 = 0
y = 1.6, 3.5

Directions(411-420): In the following questions, two equations numbered are given in variables x and y.
You have to solve both the equations and find out the relationship between x and y. Then give answer
accordingly-
411). I. 4x2 + 5x – 6 = 0,
II. 2y2 + 11y + 12 = 0
A) If x > y
B) If x < y
C) If x ≥ y
D) If x ≤ y
E) If x = y or relation cannot be established

412). I. 12x2 – 49x + 30 = 0,


II. 6y2 – 35y + 50 = 0
A) If x > y
B) If x < y
C) If x ≥ y
D) If x ≤ y
E) If x = y or relation cannot be established

413). I. 4x2 + 13x + 10 = 0,


II. 4y2 – 7y – 15 = 0
A) If x > y
B) If x < y
C) If x ≥ y
D) If x ≤ y
E) If x = y or relation cannot be established

414). I. 12x2 – 5x – 3 = 0,
II. 6y2 + y – 2 = 0
A) If x > y
B) If x < y
C) If x ≥ y
D) If x ≤ y
E) If x = y or relation cannot be established

415). I. 3x2 + 7x – 6 = 0,
II. 3y2 – 11y + 6 = 0
A) If x > y
B) If x < y
C) If x ≥ y
D) If x ≤ y
E) If x = y or relation cannot be established

www.ibpsguide.com | estore.ibpsguide.com | www.sscexamguide.com 126


500 Quadratic Equations for IBPS RRB/PO/Clerk

416). I. 5x2 – 36x – 32 = 0,


II. 3y2 + 16y + 20 = 0
A) If x > y
B) If x < y
C) If x ≥ y
D) If x ≤ y
E) If x = y or relation cannot be established

417). I. 2x2 + (4 + √2)x + 2√2 = 0


II. y2 – (1 + 3√3)y + 3√3 = 0
A) If x > y
B) If x < y
C) If x ≥ y
D) If x ≤ y
E) If x = y or relation cannot be established

418). I. x2 + (3 + 2√2)x + 6√2 = 0


II. 5y2 – (1 + 5√2)y + √2 = 0
A) If x > y
B) If x < y
C) If x ≥ y
D) If x ≤ y
E) If x = y or relation cannot be established

419). I. 2x2 + (4 + 2√6)x + 4√6 = 0


II. 5y2 + (10 + √6)y + 2√6 = 0
A) If x > y
B) If x < y
C) If x ≥ y
D) If x ≤ y
E) If x = y or relation cannot be established

420). I. 9a2 + 18a + 5 = 0,


II. 2b2 + 13b + 20 = 0 to solve both the equations to find the values of a and b?
If a > b
B. If a ≥ b
C. If a < b
D. If a ≤ b
E. If a = b or the relationship between a and b cannot be established.

411). E)
4x2 + 5x – 6 = 0
4x2 + 8x – 3x – 6 = 0
Gives x = -2, 3/4
2y2 + 11y + 12 = 0
2y2 + 8y + 3y + 12 = 0
Gives y = -4, -3/2

412). E)
12x2 – 49x + 30 = 0

www.ibpsguide.com | estore.ibpsguide.com | www.sscexamguide.com 127


500 Quadratic Equations for IBPS RRB/PO/Clerk

12x2 – 9x – 40x + 30 = 0
Gives x = 3/4, 10/3
6y2 – 35y + 50 = 0
6y2 – 15y – 20y + 50 = 0
Gives y = 5/2, 10/3

413). D)
4x2 + 13x + 10 = 0
4x2 + 8x + 5x + 10 = 0
Gives x = -2, -5/4
4y2 – 7y – 15 = 0
4y2 – 12y + 5y – 15 = 0
Gives y = -5/4, 3

414. E)
12x2 – 5x – 3 = 0
12x2 + 4x – 9x – 3 = 0
Gives x = -1/3, 3/4
6y2 + y – 2 = 0
6y2 – 3y + 4y – 2 = 0
Gives y= -2/3, 1/2

415). D)
3x2 + 7x – 6 = 0
3x2 + 9x – 2x – 6 = 0
Gives x = -3, 2/3
3y2 – 11y + 6 = 0
3y2 – 9y – 2y + 6 = 0
Gives y = 2/3, 3

416). A)
5x2 – 36x – 32 = 0
5x2 + 4x – 40x – 32 = 0
Gives x = -4/5, 8
3y2 + 16y + 20 = 0
3y2 + 6y + 10y + 20 = 0
Gives y = -10/3, -2

417). B)
2x2 + (4 + √2)x + 2√2 = 0
(2x2 + 4x) + (√2x + 2√2) = 0
2x (x + 2) + √2 (x + 2) = 0
So x = -2, -√2/2 (-0.7)
y2 – (1 + 3√3)y + 3√3 = 0
(y2 – y) – (3√3y – 3√3) = 0

www.ibpsguide.com | estore.ibpsguide.com | www.sscexamguide.com 128


500 Quadratic Equations for IBPS RRB/PO/Clerk

y (y – 1) – 3√3 (y – 1) = 0
So, y = 1, 3√3 (5.2)

418). B)
x2 + (3 + 2√2)x + 6√2 = 0
(x2 + 3x) + (2√2x + 6√2) = 0
x (x + 3) + 2√2 (x + 3) = 0
So x = -3, -2√2
5y2 – (1 + 5√2)y + √2 = 0
(5y2 – y) – (5√2y – √2) = 0
y (5y – 1) – 3√2 (5y – 1) = 0
So, y = 1/5, 3√2

419). D)
2x2 + (4 + 2√6)x + 4√6 = 0
(2x2 + 4x) + (2√6x + 4√6) = 0
2x (x + 2) + 2√6 (x + 2) = 0
So x = -2, -√6
5y2 + (10 + √6)y + 2√6 = 0
(5y2 + 10y) + (√6y + 2√6) = 0
5y (y + 2) + √6 (y + 2) = 0
So, y = -2, -√6/5

420). A)
I. 9a2 + 3a + 15a + 5 = 0
=>(3a + 5)(3a + 1) = 0 => a = -5/3, -1/3
II. 2b2 + 8b + 5b + 20 = 0
=>(2b + 5)(b + 4) = 0 => b = -5/2, -4
a is always more than b.
a > b.

421). In the following question two equations (A) and (B) given. You have to solve both:
a. 21a² - 20a - 9 = 0
b. 7b² - 23b + 18 = 0
i. If a > b
ii. If a < b
iii.If a = b or C.N.E
iv. If a > b
v. If a < b

422). In the following question two equations (A) and (B) given. You have to solve both:
a. 48a² + 2a -1 = 0
b. 6b² - 11b - 2 = 0
i. If a < b
ii. If a > b
iii.If a > b
iv. If a < b
v. If a = b or C.N.E

www.ibpsguide.com | estore.ibpsguide.com | www.sscexamguide.com 129


500 Quadratic Equations for IBPS RRB/PO/Clerk

423). In the following question two equations (A) and (B) given. You have to solve both:
a. 9a² - 24a + 143 = 0
b. 9b² + 50b - 91=0
i. If a > b
ii. If a < b
iii. If a > b
iv. If a = b or C.N.E
v. If a < b

424). In the following question two equations (A) and (B) given. You have to solve both:
a. 36a² - 19a - 7 = 0
b. 12b² - 5b - 2 = 0
i. If a < b
ii. If a = b or C.N.E
iii. If a < b
iv. If a > b
v. If a > b

425). In the following question two equations (A) and (B) given. You have to solve both:
i. If a < b
ii. If a > b
iii.If a = b or C.N.E
iv. If a < b
v. If a > b

Directions (426-430): Two equations (I) and (II) are given in each questions. On the basis of these
questions, you have to decide the relation between x and y and give answer
a) if x> y
b) if x< y
c) if x ≥ y
d) if x ≤ y
e) if x =y, or no relation can be established between x and y.
426). I.5x2 – 87x + 378 = 0
II.3y2 – 49y + 200 = 0

427). I.10x2 – x – 24 = 0
II.y2 – 2y = 0

428). I.x2 – 5x + 6 = 0
II.2y2 – 15y + 27 = 0

429). I.3x + 2y = 301


II.7x – 5y = 74

430). I.14x2 – 37x + 24 = 0


II.28y2 – 53y + 24 = 0

421). E)

422). E)

www.ibpsguide.com | estore.ibpsguide.com | www.sscexamguide.com 130


500 Quadratic Equations for IBPS RRB/PO/Clerk

423). C)

424). B)

425). C)

426). I. 5x2– 45x – 42x + 378 = 0


or,5x(x – 9) – 42(x- 9) = 0
or. (5x – 42)(x -9) = 0
x = 9, 42/5
II. 3y2– 24y – 25y + 200=0
or, 3y(y – 8) -25(y – 8) = 0 or, (y – 8)(3y-25)=0
y = 8, 25/3
Hence, x>y
Answer: a)

427). I. 10x2-16x + 15x – 24 = 0


or, 2x(5x -8) + 3(5x 8)=0
or,(2x + 3)(5x +8) = 0
x = -3/2, 8/5
II. y2 – 2y = 0
or, y(y -2) = 0
y = 0, 2
ie no relationship exists between x and y.
Answer: e)

428).x2 – 2x – 3x + 6 =0
or, x(x -2) – 3(x -2) = 0
or, (x -2) (x -3) = 0
x =2, 3
2y2 – 6y – 9y + 27 = 0
or, 2y(y – 3)-9(y -3) = 0
or, (y- 3) (2y -9) = 0
y = 3, 9/2
hence, x ≤ y
Answer: d)

429).I. eqn (I) × 5 + eqn (II) × 2


[15x + 10y = 1505] + [14x – 10y = 148] = 29x = 1653
x = (1653/29) = 57
and y = 65
hence, x< y
Answer: b)

www.ibpsguide.com | estore.ibpsguide.com | www.sscexamguide.com 131


500 Quadratic Equations for IBPS RRB/PO/Clerk

430).14x2 – 37x + 24 = 0
or, I4x2– 21x- 16x + 24 = 0
or, 7x(2x -3) -8(2x -3) = 0
or, (2x – 3)(7x – 8) = 0
x = (3/2), (8/7)
II. 28y2– 53y + 24 = 0
or, 28y2 -21y – 32y + 24 =0
or, 7y(4y – 3) -8(4y – 3) = 0
or, (7y – 8) (4y – 3) = 0
y = 8/7, 3/4
x≥y
Answer: c)

Directions (431-435) : In each of these questions, two equations (I) and (II) are given. You have to solve
both the equations and give answer
431) I. 3x2 – 29x + 56 = 0
II. 3y2 – 5y – 8 = 0
(a) x> y
(b) x ≥ y
(c) x< y
(d) x ≤ y
(e) x = y or the relationship between ‘x’and ‘y’cannot be established.

432) I. 5x2 + 26x – 24 = 0


II. 5y2 – 34y + 24 = 0
(a) x> y
(b) x ≥ y
(c) x< y
(d) x ≤ y
(e) x = y or the relationship between ‘x’and ‘y’cannot be established.

433) I. x2– 7x = 0
II. 2y2 + 5y + 3 = 0
(a) x> y
(b) x ≥ y
(c) x< y
(d) x ≤ y
(e) x = y or the relationship between ‘x’and ‘y’cannot be established.

434) I. 7x – 4y = 40
II. 8x + 8y = 8
(a) x> y
(b) x ≥ y
(c) x< y
(d) x ≤ y
(e) x = y or the relationship between ‘x’and ‘y’cannot be established.

435) I. 15x2 – 41x + 14 = 0


II. 2y2 – 13y + 20 = 0
(a) x> y

www.ibpsguide.com | estore.ibpsguide.com | www.sscexamguide.com 132


500 Quadratic Equations for IBPS RRB/PO/Clerk

(b) x ≥ y
(c) x< y
(d) x ≤ y
(e) x = y or the relationship between ‘x’and ‘y’cannot be established.

(436-440) : In each of these questions, two equations (I) and (II) are given. You have to solve both the
equations and give answer.
436) I. x2-8√3x+45=0
II. y2-√2y-24=0
(a) if x > y
(b) if x ≥ y
(c) if x < y
(d) if x ≤ y
(e) if x = y or no relation can be established between x and y.

437) x-7√2x+24=0
II. y-5√2y+12=0
(a) if x > y
(b) if x ≥ y
(c) if x < y
(d) if x ≤ y
(e) if x = y or no relation can be established between x and y.

438). I. 12x2 – 17x + 6 = 0


II. 20y2 – 31y + 12 = 0
(a) if x > y
(b) if x ≥ y
(c) if x < y
(d) if x ≤ y
(e) if x = y or no relation can be established between x and y.

439). I. 3x2 – 8x + 4 = 0
II. 4y2 – 15y + 9 = 0
(a) if x > y
(b) if x ≥ y
(c) if x < y
(d) if x ≤ y
(e) if x = y or no relation can be established between x and y.

440). I. x2 -16x + 63 = 0
II. y2 – 2y – 35 = 0
(a) if x > y
(b) if x ≥ y
(c) if x < y
(d) if x ≤ y
(e) if x = y or no relation can be established between x and y.

431). (b)
3x2 – 29x + 56 = 0
or 3x2 – 21x – 8x + 56 = 0
or 3x(x – 7) – 8(x – 7) = 0

www.ibpsguide.com | estore.ibpsguide.com | www.sscexamguide.com 133


500 Quadratic Equations for IBPS RRB/PO/Clerk

or (3x – 8) (x – 7) = 0
x=,7
II. 3y2 – 5y – 8 = 0
or 3y2 + 3y – 8y – 8 = 0
or 3y(y + 1) – 8(y + 1) = 0
or (3y – 8) (y + 1) = 0
or (3y – 8) (y + 1) = 0
y = -1,
x≥ y

432). (d)
5x2 + 26x – 24 = 0
or 5x2 + 30x – 4x – 24 = 0
or 5x(x + 6) – 4(x + 6) = 0
or (5x – 4) (x + 6) = 0
x = 4/5, 6
II. 5y2 – 30y – 4y + 24 = 0
or 5y(y – 6) – 4(y – 6) = 0
or (5y – 4) (y – 6) = 0 4
y =4/5 6
x≤y

433). (a)
x2 – 7x = 0
or x (x – 7) = 0
x = 0, 7
II. 2y2 + 5y + 3 = 0
or 2y2 + 2y + 3y + 3 = 0
or 2y(y + 1) + 3(y + 1) = 0
or (2y + 3) (y + 1) = 0
y = -1, -3/2
x>y

434). (a)
7x – 4y = 40 …(i)
and 8x + 8y = 8
or x+y= 1 …(ii)
Solving (i) and (ii), we have
x = 4, y = -3
x>y

435). (c)
15x2 – 4x + 14 = 0
or 15x2 – 6x – 35x + 14 = 0
or 3x(5x – 2) – 7(5x – 2) = 0

www.ibpsguide.com | estore.ibpsguide.com | www.sscexamguide.com 134


500 Quadratic Equations for IBPS RRB/PO/Clerk

or (3x – 7)(5x – 2) = 0
x = 7/3, 2/5
II. 2y2 – 13y + 20 = 0
or 2y2 – 8y – 5y + 20 = 0
or 2y(y – 4) – 5(y – 4) = 0
or (2y – 5) (y – 4) = 0
y = 4, 5/2
x<y

436). (e)
I.x2 -8√3x + 45= 0
or x2 – 5√3x + 3√3 (x – 5√3) = 0
or, (x + 3√3) (x – 5√3) = 0
X = 3√3, 5√3
II. y2 – √2y – 24 = 0
Or y2 – 4√2y + 3 √2y – 24 = 0
Or (y-4√2y) (y + 2√2)
y = -3 √2, 4√2
Hence relation cannot be established between x and y.

437). B)
x – 7 √2x + 24 = 0
Or x – 4√2x – 3 √2x + 24 = 0
Or √x (√x – 4√2) – 3√2 (√x – 4√2) = 0
Or (√x – 3√2) (√x – 4√2) = 0
Now, if √x -3√2 = 0
then √x = 3√2
x = 9 × 2 = 18
If √x – 4√2 = 0
then √x = 4√2
x = 16 × 2 = 32
II. y – 5√2y + 12 = 0
y -3√2y – 2√2y + 12 = 0
Or √y (√y – 3√2) -2√2y + 12 = 0
Or (√y – 2√2) – (√y -3√2) = 0
If (√y -2√2) = 0
Then √y = 2√2
y = 4 2 = 18
If √y -3√2 = 0
Then, √y -√2
y = 9 × 2 = 18
x≥y

438). (d)
12x2 – 17x + 6 = 0

www.ibpsguide.com | estore.ibpsguide.com | www.sscexamguide.com 135


500 Quadratic Equations for IBPS RRB/PO/Clerk

or 12x2 – 9x – 8x + 6 = 0
or 3x(4x – 3) – 2(4x – 3) = 0
or (3x – 2) (4x – 3) = 0
If 3x – 2 = 0
then 3x = 2
x= 2/3
If 4x – 3 = 0
then x = 3/4
II. 20y2 – 31y + 12 = 0
or 20y2 – 16y – 15y + 12 = 0
or 4y (5y – 4) – 3 (5y – 4) = 0
or (4y – 3) (5y – 4) = 0
y=3/4,4/5
Hence x ≤ y

439). (e)
3x2 – 8x + 4 = 0
or 3x2 – 6x – 2x + 4 = 0
or (3x – 2) (x – 2) = 0
x=2,2/3
II. 4y2 – 15y + 9 = 0
or 4y2 – 12y – 3y + 9 = 0
or 4y(y – 3) – 3(y – 3) = 0
or (4y – 3) (y – 3) = 0
y = 3/4, 3
Relation cannot be established between x and y.

440). (a)
I. x2 – 16x + 63 = 0
or x2 – 9x – 7x + 63 = 0
or x(x – 9) – 7(x – 9) = 0
or (x – 7) (x – 9) = 0
x = 7, 9
II. y2 – 2y – 35 = 0
or y2 – 17y + 5y – 35 = 0
or y(y – 7) + 5(y – 7) = 0
or (y + 5) (y – 7) = 0
y = -5, 7
Hence, x ≥ y

441). x² + 6x + 9 = 0
y² + 2y – 35 = 0
A. X > Y
B. X < Y
C. X ≥ Y
D. X ≤ Y

www.ibpsguide.com | estore.ibpsguide.com | www.sscexamguide.com 136


500 Quadratic Equations for IBPS RRB/PO/Clerk

E. X = Y or relation cannot be established

442). 2x² – 9x + 10 = 0
y² – 18y + 72 = 0
A. X > Y
B. X < Y
C. X ≥ Y
D. X ≤ Y
E. X = Y or relation cannot be established

443). 2x² + 11x + 12 = 0


y² – 14y + 48 = 0
A. X > Y
B. X < Y
C. X ≥ Y
D. X ≤ Y
E. X = Y or relation cannot be established

444). x² – 11x + 30 = 0
y² – 4y – 12 = 0
A. X > Y
B. X < Y
C. X ≥ Y
D. X ≤ Y
E. X = Y or relation cannot be established

445). 13x² – 32x – 21 = 0


8y² + 6y – 5 = 0
A. X > Y
B. X < Y
C. X ≥ Y
D. X ≤ Y
E. X = Y or relation cannot be established

446). 17x² + 48x – 9 = 0


y² – 2y – 15 = 0
A. X > Y
B. X < Y
C. X ≥ Y
D. X ≤ Y
E. X = Y or relation cannot be established

447). 18x² + 18x + 4 = 0


12y² – 22y + 8 = 0
A. X > Y
B. X < Y
C. X ≥ Y
D. X ≤ Y
E. X = Y or relation cannot be established

448). 16x² + 20x + 6 = 0


y² + 15y + 56 = 0

www.ibpsguide.com | estore.ibpsguide.com | www.sscexamguide.com 137


500 Quadratic Equations for IBPS RRB/PO/Clerk

A. X > Y
B. X < Y
C. X ≥ Y
D. X ≤ Y
E. X = Y or relation cannot be established

449). 12x² + 29x + 14 = 0


y² + 9y + 18 = 0
A. X > Y
B. X < Y
C. X ≥ Y
D. X ≤ Y
E. X = Y or relation cannot be established

450). x² – 8x + 15 = 0
y² – 13y + 40 = 0
A. X > Y
B. X < Y
C. X ≥ Y
D. X ≤ Y
E. X = Y or relation cannot be established

441). E.
X = Y or relation cannot be established
(x + 3) (x + 3) = 0
x = – 3, – 3
y² + 2y – 35 = 0
y = -7, 5

442). B.
X<Y
2x² – 9x + 10 = 0
x = 2.5, 2
y² – 18y + 72 = 0
y = 12, 6

443). B.
X<Y
2x² + 11x + 12 = 0
x = – 4, – 1.5
y² – 14y + 48 = 0
y = 6, 8

444). E.
X = Y or relation cannot be established
x² – 11x + 30 = 0
x = 5, 6

www.ibpsguide.com | estore.ibpsguide.com | www.sscexamguide.com 138


500 Quadratic Equations for IBPS RRB/PO/Clerk

y² – 4y – 12 = 0
y = 6, -2

445). E.
X = Y or relation cannot be established
13x² – 32x – 21 = 0
x = 3, -0.5
8y² + 6y – 5 = 0
y = -1.25, 0.5

446). E.
X = Y or relation cannot be established
17x² + 48x – 9 = 0
x = -3, 0.1
y² – 2y – 15 = 0
y = 5, -3

447). B.
X<Y
18x² + 18x + 4 = 0
x = -0.6, -0.3
12y² – 22y + 8 = 0
y = 0.13, 0.5

448). A.
X>Y
16x² + 20x + 6 = 0
x = – 0.25, – 1.5
y² + 15y + 56 = 0
y = -8, -7

449). A.
X>Y
12x² + 29x + 14 = 0
x = – 1.75, – 0.6
y² + 9y + 18 = 0
y = -6, -3

450). D.
X≤Y
x² – 8x + 15 = 0
x = 5, 3
y² – 13y + 40 = 0
y = 8, 5

www.ibpsguide.com | estore.ibpsguide.com | www.sscexamguide.com 139


500 Quadratic Equations for IBPS RRB/PO/Clerk

Direction(451-460)Find The Relationship Between X And Y By Solving Given Equations:


a) X>Y
b) X ≥ Y
c) X<Y
d) X ≤ Y
e) X=Y RELATIONSHIP CANNOT BE ESTABLISHED

451) X² – 51x-630=0
Y²+ 52y-640=0

452). 2) X²+33x+540=0
Y²+36y+320=0

453). 3)X²-456=1569
Y²-91y+2070=0

454) X²-259=1037
Y²-359=1241

455). X³-1650=7611
Y³-2013=2900

456). X³=³√29791
Y³=³√91125

457). X²-11√6+180=0
Y²-15√6+324=0

458). X²-2√5 -175=0


Y²-17√5+360

459). 2x²-3x-20=0
3y²+11y+10=0

460). X²+7√7x+84=0
Y²+4√8y-96=0

451). A)
X² – 51x-630=0
(X-30)(X-21)=0
X=30,21
Y²+ 52y-640=0
(Y+32)(Y+20)=0
Y=-32,-20

452). E)
X²+33x+540=0
(X-12)(X+45)=0
X=12,-45
Y²+36y+320=0

www.ibpsguide.com | estore.ibpsguide.com | www.sscexamguide.com 140


500 Quadratic Equations for IBPS RRB/PO/Clerk

(Y+16)(Y+20)=0
Y=-16,-20

453). D)
X² - 456=1569
X²=2025
X=±45
Y²-91y+2070=0
(Y-45)(Y-46)=0
Y=45,46

454). E)
X²-259=1037
X²=1296
X=±36
Y²-359=1241
Y²=1600
Y=±40

455). A)
X³-1650=7611
X³=9261
X=21
Y³-2013=2900
Y³=4913
Y=17

456). C)
X³=³√29791
X=31
Y³=³√91125
Y=45
X>Y

457). D)
X²-11√6+180=0
(X-5√6)(X-6√6)=0
X=5√6,6√6
Y²-15√6+324=0
(Y-6√6)(Y-9√6)=0
Y=6√6,9√6
X ≤Y

458). C)
X²-2√5 -175=0

www.ibpsguide.com | estore.ibpsguide.com | www.sscexamguide.com 141


500 Quadratic Equations for IBPS RRB/PO/Clerk

(X+5√5)(X-7√5)=0
X=-5√5,7√5
Y²-17√5+360=0
(Y-8√5)(Y-9√5)=0
Y=8√5,9√5

459). E)
2x²-3x-20=0
(X-4)(X+2.5)=0
X=4,-2.5
3y²+11y+10=0
(Y+2)(Y+1.6)=0
Y=-2,-1.6

460). E)
X²+7√7x+84=0
(X+3√7)(X+4√7)=0
X=-3√7,-4√7
Y²+4√8y-96=0
(Y+6√8)(Y-2√8)=0
Y=-6√8,2√8

Directions(461-465): In each of these questions two equations numbered I and II are given. You have
to solve both the equations and give answer.
461) I. x2-32x+256=0
II. y2-33y+272=0
A. x < y
B. x ≤ y
C. x > y
D. x ≥ y
E. If either x=y or the relationship can’t be
established

462) I. 3x-4y+9=0
I.7x-7y-7=0
A. x < y
B. x ≤ y
C. x > y
D. x ≥ y
E. If either x=y or the relationship can’t be
established

463) I. x2-2x-15=0
II. y2-9y+14=0
A. x < y
B. x ≤ y
C. x > y
D. x ≥ y

www.ibpsguide.com | estore.ibpsguide.com | www.sscexamguide.com 142


500 Quadratic Equations for IBPS RRB/PO/Clerk

E. If either x=y or the relationship can’t be


established

464) I. 4x2-8x+3=0
II. 4y2+8y+3=0
A. x < y
B. x ≤ y
C. x > y
D. x ≥ y
E. If either x=y or the relationship can’t be
established

465) I. 2x2-3x+1=0
II. 2y2-4y+2=0
A. x < y
B. x ≤ y
C. x > y
D. x ≥ y
E. If either x=y or the relationship can’t be
established

Directions(466-470): In each of these questions two equations numbered I and II are given. You have
to solve both the equations and give answer.
406) I. 3x2+12x-180=0
II. 2y2+4y-96=0
A. x < y
B. x ≤ y
C. If either x=y or the relationship can’t be
established
D. x > y
E. x ≥ y

467) I. 36x2+30x+6=0
II.45y2+24y+3=0
A. x < y
B. x ≤ y
C. If either x=y or the relationship can’t be
established
D. x > y
E. x ≥ y

468) I. 2x2-9x+9=0
I. y2-11y+24=0
A. x < y
B. x ≤ y
C. If either x=y or the relationship can’t be
established
D. x > y
E. x ≥ y

469) I. x2-13x+40=0
II. y2+9y+18=0

www.ibpsguide.com | estore.ibpsguide.com | www.sscexamguide.com 143


500 Quadratic Equations for IBPS RRB/PO/Clerk

A. x < y
B. x ≤ y
C. If either x=y or the relationship can’t be
established
D. x > y
E. x ≥ y

470) I. 42x2-162x-24=0
II. 12y2+24y-288=0
A. x < y
B. x ≤ y
C. If either x=y or the relationship can’t be
established
D. x > y
E. x ≥ y

461) B
I. x2-32x +256 =0
=> (x-16)2 =0
=> x = 16,16
and II. y2– 33y + 272 =0
=> (y-16)(y-17)=0
=> y =16,17
So, x ≤ y
Hence, option B

462) C
Solving I and II, we get
x = 13 and y =12
So, x > y. Hence, option C

463) E
I. x2-2x -15 =0
=>x2-5x +3x -15 =0
=> (x-5)(x+3) =0
=> x= 5, -3
II. y2 – 9y +14 =0
=>y2 – 2y -7y +14 =0
=>(y -2)(y-7) =0
=>y = 2,7
So, relation between x and y is not determined.
Hence, option e.

464) C

www.ibpsguide.com | estore.ibpsguide.com | www.sscexamguide.com 144


500 Quadratic Equations for IBPS RRB/PO/Clerk

465) B

466) C
From I,
x2 + 4x – 60 = 0
x2– 6x + 10x – 60 = 0

www.ibpsguide.com | estore.ibpsguide.com | www.sscexamguide.com 145


500 Quadratic Equations for IBPS RRB/PO/Clerk

x(x – 6)+10(x – 6) = 0
(x – 6)(x + 10) = 0
x = 6 or -10
From II,
y2 + 2y – 48 = 0
y2– 6y + 8y – 48 = 0
y(y – 6)+8(y – 6) = 0
(y – 6)(y + 8) = 0
y = -8 or 6
So, no relationship can be established between x
and y. Hence, option c.

467) B
From I,
6x2 + 5x + 1 = 0
6x2 + 3x + 2x + 1 = 0
3x(2x + 1) + 1(2x + 1) = 0
(3x + 1)(2x + 1) = 0
x = -1/3 or -1/2
From II,
15y2 + 8y + 1 = 0
15y2 + 5y + 3y + 1 = 0
5y(3y + 1) + 1(3y + 1) = 0
(5y + 1)(3y + 1) = 0
y = -1/5 or -1/3
So, y≥x. Hence, option b.

468) B
From I,
2x2– 6x – 3x + 9 = 0
2x(x – 3)-3(x – 3) = 0
(2x – 3)(x – 3) = 0
x = 3/2 or 3
From II,
y2– 3y – 8y + 24 = 0
y(y – 3)-8(y – 3) = 0
(y – 3)(y – 8) = 0
y = 3 or 8
So, y ≥ x. Hence, option b

469) D
From I,
x2– 5x – 8x + 40 = 0
x(x – 5)-8(x – 5) = 0
(x – 5)(x – 8) = 0

www.ibpsguide.com | estore.ibpsguide.com | www.sscexamguide.com 146


500 Quadratic Equations for IBPS RRB/PO/Clerk

x = 5 or 8
From II,
y2 + 3y + 6y + 18 = 0
y(y + 3)+6(y + 3) = 0
(y + 3)(y + 6) = 0
y = -3 or -6
So, x > y. Hence, option d.

470) C
From I,
7x2– 27x – 4 = 0
7x2– 28x + x – 4 = 0
7x(x – 4) + 1(x – 4) = 0
(x – 4)(7x + 1) = 0
x = -1/7 or 4
From II,
y2 + 2y – 24 = 0
y2– 4y + 6y – 24 = 0
y(y – 4)+6(y – 4) = 0
(y – 4)(y + 6) = 0
y = 4 or -6
So, no relationship can be established between x
and y. Hence, option c.

Directions(471-480): In the following questions, two equations numbered are given in variables x and y.
You have to solve both the equations and find out the relationship between x and y. Then give answer
accordingly-
471) I. 20x2 – 31x + 12 = 0,
II. 6y2 – 7y + 2 = 0
A) If x > y
B) If x < y
C) If x ≥ y
D) If x ≤ y
E) If x = y or relation cannot be established

472) I. 3x2 + 22 x + 24 = 0,
II. 3y2 – 10y + 3 = 0
A) If x > y
B) If x < y
C) If x ≥ y
D) If x ≤ y
E) If x = y or relation cannot be established

473) I. 6x2 – x – 2 = 0,
II. 5y2 – 18y + 9 = 0
A) If x > y
B) If x < y
C) If x ≥ y

www.ibpsguide.com | estore.ibpsguide.com | www.sscexamguide.com 147


500 Quadratic Equations for IBPS RRB/PO/Clerk

D) If x ≤ y
E) If x = y or relation cannot be established

474) I. x2 – x – 6 = 0,
II. 5y2 – 7y – 6 = 0
A) If x > y
B) If x < y
C) If x ≥ y
D) If x ≤ y
E) If x = y or relation cannot be established

475) I. 3x2 – 10x + 8 = 0,


II. 3y2 + 8y – 16 = 0
A) If x > y
B) If x < y
C) If x ≥ y
D) If x ≤ y
E) If x = y or relation cannot be established

476) I. 2x2 + 17x + 30 = 0,


II. 2y2 + 13y + 18 = 0
A) If x > y
B) If x < y
C) If x ≥ y
D) If x ≤ y
E) If x = y or relation cannot be established

477) I. 3x2 + 16x + 20 = 0,


II. 3y2 + 8y + 4 = 0
A) x > y
B) x < y
C) x ≥ y
D) x ≤ y
E) x = y or relationship cannot be determined

478) I. x2 + x – 20 = 0,
II. 2y2 + 13y + 15 = 0
A) If x > y
B) If x < y
C) If x ≥ y
D) If x ≤ y
E) If x = y or relation cannot be established

479)I. 5x2 – 7x – 6 = 0,
II. 5y2 + 23y + 12 = 0
A) If x > y
B) If x < y
C) If x ≥ y
D) If x ≤ y
E) If x = y or relation cannot be established

www.ibpsguide.com | estore.ibpsguide.com | www.sscexamguide.com 148


500 Quadratic Equations for IBPS RRB/PO/Clerk

480) I. 2x2 – 9x + 4 = 0,
II. 2y2 + 7y – 4 = 0
A) x > y
B) x < y
C) x ≥ y
D) x ≤ y
E) x = y or relationship cannot be determined

471) A) If x > y
Solution:
20x2 – 31x + 12 = 0
20x2 – 16x – 15x + 12 = 0
So x = 3/4, 4/5
6y2 – 7y + 2 = 0
6y2 – 3y – 4y + 2 = 0
So y = 1/2, 2/3
Put on number line
1/2… 2/3… 3/4… 4/5

472) B) If x < y
Solution:
3x2 + 22 x + 24 = 0
3x2 + 18x + 4x + 24 = 0
So x = -4/3, -6
3y2 – 10y + 3 = 0
3y2 – 9y – y + 3 = 0
So y = 1/3, 3
Put on number line
-6… -4/3… 1/3… 3

473) E) If x = y or relation cannot be established


Solution:
6x2 – x – 2 = 0
6x2 + 3x – 4x – 2 = 0
So x = -1/2, 2/3
5y2 – 18y + 9 = 0
5y2 – 15y – 3y + 9 = 0
So y = 3/5, 3
Put on number line
-1/2 …. 3/5 ….2/3 …. 3

474) E) If x = y or relation cannot be established


Solution:
x2 – x – 6 = 0
x2 – 2x + 3x – 6 = 0
So x = -3, 2

www.ibpsguide.com | estore.ibpsguide.com | www.sscexamguide.com 149


500 Quadratic Equations for IBPS RRB/PO/Clerk

5y2 – 7y – 6 = 0
5y2 – 10y + 3y – 6 = 0
So y = -3/5, 2
Put on number line
-3 …. -3/5….. 2

475) C) If x ≥ y
Solution:
3x2 – 10x + 8 = 0
3x2 – 6x – 4x + 8 = 0
So x = 2, 4/3
3y2 + 8y – 16 = 0
3y2 + 12y – 4y – 16 = 0
So y = -4, 4/3
Put on number line
-4 …. 4/3…. 2

476) E) If x = y or cannot be established


Solution:
2x2 + 17x + 30 = 0
2x2 + 12x + 5x + 30 = 0
So x = -6, -5/2
2y2 + 13y + 18 = 0
2y2 + 4y + 9y + 18 = 0
So y = -9/2, -2
Put on number line
-6 … -9/2 …. -5/2 …. -2

477) D) If x ≤ y
Solution:
3x2 + 16x + 20 = 0
3x2 + 6x + 10x + 20 = 0
So x = -10/3, -2
3y2 + 8y + 4 = 0
3y2 + 6y + 2y + 4 = 0
So y = -2, -2/3
put on number line
-10/3…. -2…. -2/3

478) E) If x = y or relation cannot be established


Solution:
x2 + x – 20 = 0
(x+5)(x-4) = 0
So x = -5, 4
2y2 + 13y + 15 = 0

www.ibpsguide.com | estore.ibpsguide.com | www.sscexamguide.com 150


500 Quadratic Equations for IBPS RRB/PO/Clerk

2y2 + 10y + 3y + 15 = 0
So y = -5, -3/2
Put on number line
-5…. -3/2…. 4

479) C) If x ≥ y
Solution:
5x2 – 7x – 6 = 0
5x2 – 10x + 3x – 6 = 0
So x = -3/5, 2
5y2 + 23y + 12 = 0
5y2 + 20y + 3y + 12 = 0
So y = -4, -3/5
Put on number line
-4….. -3/5…. 2

480) C) If x ≥ y
Solution:
2x2 – 9x + 4 = 0
2x2 – 8x – x + 4 = 0
So x = 4 , 1/2
2y2 + 7y – 4 = 0
2y2 + 8y – y – 4 = 0
So y = -4, 1/2
Put on number line
-4……. 1/2…… 4

Directions(481-490): In the following questions, two equations numbered are given in variables x and y.
You have to solve both the equations and find out the relationship between x and y. Then give answer
accordingly-
481) I. 4x2 + 27x + 18 = 0,
II. 2y2 – 7y + 3 = 0
A) x > y
B) x < y
C) x ≥ y
D) x ≤ y
E) x = y or relation cannot be established

482) I. 3x2 – 2x – 8 = 0,
II. 6y2 – 17y + 10 = 0
A) x > y
B) x < y
C) x ≥ y
D) x ≤ y
E) x = y or relation cannot be established

483) I. 32 + 11x + 6 = 0,
II. 5y2 + 16y + 3 = 0

www.ibpsguide.com | estore.ibpsguide.com | www.sscexamguide.com 151


500 Quadratic Equations for IBPS RRB/PO/Clerk

A) x > y
B) x < y
C) x ≥ y
D) x ≤ y
E) x = y or relation cannot be established

484) I. 4x2 – 11x + 6 = 0,


II. 6y2 – 29y + 28 = 0
A) x > y
B) x < y
C) x ≥ y
D) x ≤ y
E) x = y or relation cannot be established

485) I. 3x2 – 25x + 52 = 0,


II. 3y2 – 8y – 16 = 0
A) x > y
B) x < y
C) x ≥ y
D) x ≤ y
E) x = y or relation cannot be established

486) I. 8x2 + 10x + 3 = 0,


II. 3y2 + 70y + 40 = 0
A) x > y
B) x < y
C) x ≥ y
D) x ≤ y
E) x = y or relation cannot be established

487) I. 50x2 - 95x + 42 = 0,


II. 50y2 – 65y + 21 = 0
A) x > y
B) x < y
C) x ≥ y
D) x ≤ y
E) x = y or relationship cannot be determined

488) I. 5x2 – 13x + 6 = 0,


II. 3y2 – 22y – 35 = 0
A) x > y
B) x < y
C) x ≥ y
D) x ≤ y
E) x = y or relation cannot be established

489)I. 3x2 – 4x – 15 = 0,
II. 5y2 – 11y – 18 = 0
A) x > y
B) x < y
C) x ≥ y
D) x ≤ y

www.ibpsguide.com | estore.ibpsguide.com | www.sscexamguide.com 152


500 Quadratic Equations for IBPS RRB/PO/Clerk

E) x = y or relation cannot be established

490) I. 2x2 + 5x – 12 = 0,
II. 2y2 – 19y + 35 = 0
A) x > y
B) x < y
C) x ≥ y
D) x ≤ y
E) x = y or relationship cannot be determined

481) Option B
Solution:
4x2 + 27x + 18 = 0
4x2 + 24x + 3x + 18 = 0
So x = -3/4, -6
2y2 – 7y + 3 = 0
2y2 – 6y – y + 3 = 0
So y = 1/2, 3
Put all values on number line and analyze the relationship
-6… -3/4… 1/2… 3

482) Option D
Solution:
3x2 – 2x – 8 = 0
3x2 – 6x + 4x – 8 = 0
So x = -4/3, 2
6y2 – 17y + 10 = 0
6y2 – 12y – 5y + 10 = 0
So y = 5/6, 2
Put all values on number line and analyze the relationship
-4/3… 2… 5/6

483) Option E
Solution:
32 + 11x + 6 = 0
32 + 9x + 2x + 6 = 0
So x = -3, -2/3
5y2 + 16y + 3 = 0
5y2 + 15y + y + 3 = 0
So y = -1/5, -3
Put all values on number line and analyze the relationship
-3 …. -2/3 ….-1/5
Since the common value (-3) is not in between other 2 values, there is no relationship between x and y.

484) Option E
Solution:
4x2 – 11x + 6 = 0

www.ibpsguide.com | estore.ibpsguide.com | www.sscexamguide.com 153


500 Quadratic Equations for IBPS RRB/PO/Clerk

4x2 – 8x – 3x + 6 = 0
So x = 3/4, 2
6y2 – 29y + 28 = 0
6y2 – 8y – 21y + 28 = 0
So y = 4/3, 7/2
Put all values on number line and analyze the relationship
3/4 …. 4/3….. 2…. 7/2

485) Option C
Solution:
3x2 – 25x + 52 = 0
3x2 – 12x – 13x + 52 = 0
So x = 4, 13/3
3y2 – 8y – 16 = 0
3y2 – 12y + 4y – 16 = 0
So y = 4, -4/3
Put all values on number line and analyze the relationship
-4/3 …. 4…. 13/3

486) Option A
Solution:
8x2 + 10x + 3 = 0
8x2 + 4x + 6x + 3 = 0
So x = -3/4, -1/2
3y2 + 70y + 40 = 0
3y2 + 30y + 40y + 40 = 0
So y = -10, -4/3
Put all values on number line and analyze the relationship
-10 … -4/3 …. -3/4 …. -1/2

487) Option C
Solution:
50x2 - 95x + 42 = 0
50x2 - 60x – 35x + 42 = 0
So x = 7/10, 6/5
50y2 – 65y + 21 = 0
50y2 – 65y + 21 = 0
So y = 3/5, 7/10
Put all values on number line and analyze the relationship
3/5…. 7/10…. 6/5

488) Option B
Solution:
5x2 – 13x + 6 = 0
5x2 – 10x – 3x + 6 = 0

www.ibpsguide.com | estore.ibpsguide.com | www.sscexamguide.com 154


500 Quadratic Equations for IBPS RRB/PO/Clerk

So x = 3/5, 2
3y2 – 22y – 35 = 0
3y2 – 15y – 7y – 35 = 0
So y = 7/3, 5
Put all values on number line and analyze the relationship
3/5…. 2…. 7/3… 5

489) Option E
Solution:
3x2 – 4x – 15 = 0
3x2 – 9x + 5x – 15 = 0
So x = -5/3, 3
5y2 – 11y – 18 = 0
5y2 – 15y + 6y – 18 = 0
So y = -6/5, 3
Put all values on number line and analyze the relationship
-5/3….. -6/5…. 3
Since the common value (3) is not in between other 2 values, there is no relationship between x and y.

490) Option B
Solution:
2x2 + 5x ¬– 12 = 0
2x2 + 8x ¬– 3x – 12 = 0
So x = -4 , 3/2
2y2 – 19y + 35 = 0
2y2 – 14y – 5y + 35 = 0
So y = 5/2, 7
Put all values on number line and analyze the relationship
-4……. 3/2…… 5/2…

Directions(491-500): In the following questions, two equations numbered are given in variables x and y.
You have to solve both the equations and find out the relationship between x and y. Then give answer
accordingly-
A) x> y
B) x < y
C) x ≥ y
D) x ≤ y
E) x = y or relationship cannot be determined

491) I. 3x2 + 20x + 32 = 0,


II. 3y2 – 4y – 4 = 0

492) I. 4x2 – 12x + 5 = 0,


II. 6y2 – 13y + 6 = 0

493) I. 32 – 14x + 16 = 0,
II. 4y2 – 5y – 6 = 0

www.ibpsguide.com | estore.ibpsguide.com | www.sscexamguide.com 155


500 Quadratic Equations for IBPS RRB/PO/Clerk

494) I. 5x2 – 8x – 4 = 0,
II. 5y2 – 23y – 10 = 0

495) I. 3x2 + 13x + 14 = 0,


II. 4y2 + 9y + 2 = 0

496) I. 3x2 + 8x + 5 = 0,
II. 5y2 – 7y – 6 = 0

497) I. 3x2 ¬¬+ 16x + 20 = 0,


II. 3y2 + 14y + 16 = 0

498) I. 4x2 – 9x + 2 = 0,
II. 3y2 – 16y + 21 = 0

499) I. 3x2 + 5x + 2 = 0,
II. 3y2 + 11y + 10 = 0

500) I. 4x2 – 9x + 2 = 0,
II. 2y2 – 19y + 35 = 0

491) Option B
Solution:
3x2 + 20x + 32 = 0
3x2 + 12x + 8x + 32 = 0
So x = -4, -8/3
3y2 – 4y – 4 = 0
3y2 – 6y + 2y – 4 = 0
So y = -2/3, 2
Put all values on number line and analyze the relationship
-4… -8/3… -2/3… 2

492) Option E
Solution:
4x2 – 12x + 5 = 0
4x2 – 2x – 10x + 5 = 0
So x = ½, 5/2
6y2 – 13y + 6 = 0
6y2 – 4y – 9y + 6 = 0
So y = 2/3, 3/2
Put all values on number line and analyze the relationship
1/2… 2/3… 3/2…. 5/2

493) Option C
Solution:
32 – 14x + 16 = 0
32 – 6x – 8x + 16 = 0
So x = 8/3, 2

www.ibpsguide.com | estore.ibpsguide.com | www.sscexamguide.com 156


500 Quadratic Equations for IBPS RRB/PO/Clerk

4y2 – 5y – 6 = 0
4y2 – 8y + 3y – 6 = 0
So y = -3/4, 2
Put all values on number line and analyze the relationship
-3/4 …. 2 ….8/3

494) Option E
Solution:
5x2 – 8x – 4 = 0
5x2 – 10x + 2x – 4 = 0
So x = -2/5, 2
5y2 – 23y – 10 = 0
5y2 – 25y + 2y – 10 = 0
So y = -2/5, 5
Put all values on number line and analyze the relationship
-2/5 …. 2….. 5

495) Option D
Solution:
3x2 + 13x + 14 = 0
3x2 + 6x + 7x + 14 = 0
So x = -7/3, -2
4y2 + 9y + 2 = 0
4y2 + 8y + y + 2 = 0
So y = -2, -1/4
Put all values on number line and analyze the relationship
-7/3 …. -2…. -1/4

496) Option B
Solution:
3x2 + 8x + 5 = 0
3x2 + 3x + 5x + 5 = 0
So x = -5/3, -1
5y2 – 7y – 6 = 0
5y2 – 7y – 6 = 0
So y = -3/5, 2
Put all values on number line and analyze the relationship
-5/3…. -1…. -3/5…. 2

497) Option E
Solution:
3x2 ¬¬+ 16x + 20 = 0
3x2 ¬¬+ 6x + 10x + 20 = 0
So x = -10/3, -2
3y2 + 14y + 16 = 0

www.ibpsguide.com | estore.ibpsguide.com | www.sscexamguide.com 157


500 Quadratic Equations for IBPS RRB/PO/Clerk

3y2 + 6y + 8y + 16 = 0
So y = -8/3, -2
Put all values on number line and analyze the relationship
-10/3…. -8/3…. -2

498) Option B
Solution:
4x2 – 9x + 2 = 0
4x2 – 8x – x + 2 = 0
So x = 1/4, 2
3y2 – 16y + 21 = 0
3y2 – 9y – 7y + 21 = 0
So y = 7/3, 3
Put all values on number line and analyze the relationship
1/4…. 2…. 7/3… 3

499) Option A
Solution:
3x2 + 5x + 2 = 0
3x2 + 3x + 2x + 2 = 0
So x = -1, -2/3
3y2 + 11y + 10 = 0
3y2 + 6y + 5y + 10 = 0
So y = -2, -5/3
Put all values on number line and analyze the relationship
-2….. -5/3…. -1….. -2/3

500) Option B
Solution:
4x2 – 9x + 2 = 0
4x2 – 8x – x + 2 = 0
So x = 1/4, 2
2y2 – 19y + 35 = 0
2y2 – 14y – 5y + 35 = 0
So y = 5/2, 7
Put all values on number line and analyze the relationship
1/4……. 2…… 5/2…7

www.ibpsguide.com | estore.ibpsguide.com | www.sscexamguide.com 158


500 Most Important Number Series Questions

500 Most Important Number Series (Missing Number)

Directions (Q. 1-10): find the missing number in the following number series?
1). 125, 152, 161, 169, 173, ?, 175
A) 180
B) 174
C) 176
D) 184
E) 173

2). 27, 54, 162, 648, ?, 19440


A) 3274
B) 3280
C) 3240
D) 3365
E) 3384

3). 161, 320, 636, ?, 2512, 4992


A) 1329
B) 1340
C) 1264
D) 1380
E) 1348

4). 4032, 4290, 4556, 4830, 5112, ?


A) 5329
B) 5402

www.ibpsguide.com | estore.ibpsguide.com| www.sscexamguide.com


1
For Free Online Mock Test for IBPS PO/Clerk 2017 – Visit: estore.ibpsguide.com
500 Most Important Number Series Questions

C) 5302
D) 5529
E) 5482

5). 16, ?, 296, 1192, 4776, 19112


A) 90
B) 80
C) 100
D) 75
E) 72

6). 584, 559, 608, 527, ?, 479


A) 627
B) 648
C) 623
D) 608
E) 671

7). 36, 90, ?, 1417.5, 7796.25, 50675.625


A) 275
B) 517
C) 364
D) 315
E) 385

8). 28, 29, ?, 195, 796, 4005


A) 62
B) 68
C) 81
D) 83
E) 74

9). 15, 40, 70, 150, ?, 590


A) 310

www.ibpsguide.com | estore.ibpsguide.com| www.sscexamguide.com


2
For Free Online Mock Test for IBPS PO/Clerk 2017 – Visit: estore.ibpsguide.com
500 Most Important Number Series Questions

B) 280
C) 290
D) 300
E) 305

10). 205, 989, 364, 848, 487, ?


A) 748
B) 745
C) 643
D) 648
E) 743

Solution With Answer Key:


1). B)
The series is +33 , +32, +23, +22, +13, +12, …..
125 + (3)3 = 152 ; 152 + (3)2 = 161 ; 161 + (2)3 = 169; 169 + (2)2 = 173
173 + (1)3 = 174; 174 + (1)2 = 175

2). C)
The series is x2, x3, x4, x5, x6,….
27 × 2 = 54; 54×3 = 162; 162 × 4 = 648; 648 × 5 = 3240
3240 × 6 = 19440

3). C)
The series is x2 — 21, x2 — 22, x2 - 23, x2 - 24, ...
ie 161 x 2 — 2 = 320,
320 x 2 — 22 = 636,
636 x 2 — 23 = 1264,
1264 x 2 — 24 = 2512,
2512 x 2 — 25 = 4992,

4). B)
The series is 632 + 63 = 4032,
652 + 65 = 4290, 672 + 67 = 4556,

www.ibpsguide.com | estore.ibpsguide.com| www.sscexamguide.com


3
For Free Online Mock Test for IBPS PO/Clerk 2017 – Visit: estore.ibpsguide.com
500 Most Important Number Series Questions

692 + 69 = 4830, 712 + 71 = 5112,


732 + 73 = 5402, ...

5). E)
The series is
16 x 4 + 8 = 72
72 x 4 + 8 = 296
296 x 4 + 8 = 1192
1192 x 4 + 8 = 4776
4776 x 4 + 8 19112

6). B)
The series is —(5)2, +(7)2 - (9)2, +(11)2, —(13)2….
ie 584 — 52 = 559,
559 + 72 = 608,
608 - 92 = 527,
527 + 112 = 648,
648 — 132 = 479,

7). D)
The series is
36 x 2.5 = 90
90 x3.5 = 315
315 x4.5 = 1417.5
1417.5 x5.5 = 7796.25
7796.25 x6.5 = 5.675.625

8). A)
The series is x1 + 12, x2 + 22, x3 + 32, x4 + 42,
je 28 x 1 + 12= 29,
29 x 2 + 22 = 62,
62 x 3 + 32 = 195,
195 x 4 + 42 = 796,
796 x 5 + 52 = 4005,

www.ibpsguide.com | estore.ibpsguide.com| www.sscexamguide.com


4
For Free Online Mock Test for IBPS PO/Clerk 2017 – Visit: estore.ibpsguide.com
500 Most Important Number Series Questions

9). C)
The series is x2 + 10, x2 — 10, x2 + 10, x2 - 10, ...(repeated alternately)
ie 15 x 2 + 10 = 40,
40 x 2 — 10 = 70,
70 x 2 + 10 = 150,
150 x 2 — 10 = 290,
290 x 2 + 10 = 590,

10). E)
The series is 205 + (28)2 = 989,
989 - (25)2 = 364,
364 +(22)2 = 848.
848 — (19)2 = 487,
487 + (16)2 = 743,

Directions (Q. 11-20): What will come in place of question mark (?) in the given number
series?
11). 3, 72, 129, 173, 203, ?
A) 248
B) 227
C) 254
D) 252
E) 218

12). 4, 3, 5, 14, 55, ?


A) 285
B) 274
C) 319
D) 315
E) 265

13). 3, 3, 12, 108, ?, 43200


A) 2700

www.ibpsguide.com | estore.ibpsguide.com| www.sscexamguide.com


5
For Free Online Mock Test for IBPS PO/Clerk 2017 – Visit: estore.ibpsguide.com
500 Most Important Number Series Questions

B) 1728
C) 972
D) 432
E) 650

14). 24, 12, 12, 18, 36, ?


A) 92
B) 90
C) 94.5
D) 81.5
E) 108

15). 126, 64, 34, 20, ?, 12


A) 18
B) 12
C) 14
D) 16
E) 15

16). 455, 212, 131, 104, 95, ?


A) 84
B) 92
C) 45
D) 61
E) 49

17). 2, 3, 8, 27, 112, ?


A) 565
B) 650
C) 316
D) 290
E) 430

18). 45, 57, 67, 75, 81, ?

www.ibpsguide.com | estore.ibpsguide.com| www.sscexamguide.com


6
For Free Online Mock Test for IBPS PO/Clerk 2017 – Visit: estore.ibpsguide.com
500 Most Important Number Series Questions

A) 89
B) 85
C) 105
D) 91
E) 78

19). 36, 37, 33, 42, 26, ?


A) 51
B) 41
C) 61
D) 45
E) 49

20). 5, 7.5, 15, 37.5, ?, 393.75


A) 80
B) 112.5
C) 160
D) 48
E) 72

Solution:
11). E)
The series is
3 + 69=72 ; 72+57=129; 129+44=173; 173+30=203; 203+15=218
69 – 57 = 12; 57 – 44 = 13; 44-30 = 14; 30-15 = 15

12). B)
The series is
4×1–1=3
3×2–1=5
5 × 3 – 1 = 14
14 × 4 – 1 = 55
55 × 5 – 1 = 274

www.ibpsguide.com | estore.ibpsguide.com| www.sscexamguide.com


7
For Free Online Mock Test for IBPS PO/Clerk 2017 – Visit: estore.ibpsguide.com
500 Most Important Number Series Questions

13). B)
The series is
3 × (1)2 = 3; 3×(2)2 = 12; 12×(3)2 = 108; 108×(4)2 = 1728; 1728×(5)2 = 43200

14). B)
The series is
24 ×0.5 = 12; 12×1 = 12; 12×1.5 = 18, 18×2=36; 36×2.5=90

15). C)
The series is
126 × (1/2)+1 =64
64 × (1/2) + 2 = 34
34 × (1/2) + 3 = 20
20 × (1/2) + 4 = 14
14 × (1/2) + 5 = 12

16). B)
The series is
455 – 212 = 243; 212 – 131 = 81; 131 – 104 = 9; 95 – 92 = 3
243 ÷ 3 = 81; 81 ÷ 3 = 27; 27÷3 = 9; 9÷3 = 3

17). A)
The series is
2×1+1=3
3×2+2=8
8 × 3 + 3 = 27
27 × 4 + 4 = 112
112 × 5 + 5 = 565

18). B)
The series is
45 + 12 = 57; 57 + 10 = 67; 67 + 8 = 75; 75 + 6 = 81; 81 + 4 = 85

19). A)

www.ibpsguide.com | estore.ibpsguide.com| www.sscexamguide.com


8
For Free Online Mock Test for IBPS PO/Clerk 2017 – Visit: estore.ibpsguide.com
500 Most Important Number Series Questions

The Series
36 + (1)2 = 37
37 - (2)2 = 33
33 + (3)2 = 42
42 – (4)2 = 26
26 + (5)2 = 51

20). B)
The series is
5 × 1.5 = 7.5; 7.5 × 2 = 15; 15 × 2.5 = 37.5; 37.5 × 3 = 112.5; 112.5×3.5 = 393.75

Directions (Q. 21-30): What will come in place of question mark (?) in the given number
series?
21). 10, 11, 15, 24, 40, ?
A) 90
B) 87
C) 114
D) 95
E) 65

22). 25, 35, 49, 67, 89, ?


A) 24
B) 81
C) 115
D) 107
E) 93

23). 11, 9, 15, 41, 159, ?


A) 740
B) 607
C) 751
D) 789
E) 785

www.ibpsguide.com | estore.ibpsguide.com| www.sscexamguide.com


9
For Free Online Mock Test for IBPS PO/Clerk 2017 – Visit: estore.ibpsguide.com
500 Most Important Number Series Questions

24). 8.5, 10.5, 16.5, 28.5, 48.5, ?


A) 57.5
B) 78.5
C) 93.5
D) 64.5
E) 85.5

25). 18, 9, 9, 18, 72, ?


A) 460
B) 372
C) 576
D) 484
E) 380

26). 68, ?, 77, 104, 168, 293


A) 69
B) 70
C) 68
D) 74
E) None of these

27). 18, 19.7, 16.3, 23.1, 9.5, ?


A) 36.5
B) 36.8
C) 36.7
D) 36.9
E) None of these

28). 2, 5, 12, 27, 58, ?


A) 122
B) 121
C) 123
D) 120
E) None of these

www.ibpsguide.com | estore.ibpsguide.com| www.sscexamguide.com


10
For Free Online Mock Test for IBPS PO/Clerk 2017 – Visit: estore.ibpsguide.com
500 Most Important Number Series Questions

29). 16, 8, 12, 30, ?, 472.5


A) 104
B) 103
C) 106
D) 105
E) None of these

30). 334, ?, 226, 217, 214, 213


A) 253
B) 251
C) 252
D) 254
E) None of these

Explanation With Answer Key:


21).

Answer: E)

22).

Answer: C)

23).

Answer: D)

www.ibpsguide.com | estore.ibpsguide.com| www.sscexamguide.com


11
For Free Online Mock Test for IBPS PO/Clerk 2017 – Visit: estore.ibpsguide.com
500 Most Important Number Series Questions

24).

Answer: B)

25).

Answer: C)

26).

Answer: A)

27).

Answer: C)

28).

Answer: B)

29).

www.ibpsguide.com | estore.ibpsguide.com| www.sscexamguide.com


12
For Free Online Mock Test for IBPS PO/Clerk 2017 – Visit: estore.ibpsguide.com
500 Most Important Number Series Questions

Answer: D)

30).

Answer: A)
Directions (Q. 31-40): What will some in place of question mark (?) in the given number
series?
31). 5, 9, 25, 89, ?, 1369
A) 343
B) 355
C) 349
D) 341
E) 345

32). 6, 17, 50, 149, ?, 1337


A) 454
B) 446
C) 442
D) 452
E) 432

33). 61, 62, 54, 81, 17, ?


A) 142
B) 145
C) 138
D) 144
E) 140

34). 6, 11, 18, 29, 46, ?


A) 60
B) 75

www.ibpsguide.com | estore.ibpsguide.com| www.sscexamguide.com


13
For Free Online Mock Test for IBPS PO/Clerk 2017 – Visit: estore.ibpsguide.com
500 Most Important Number Series Questions

C) 69
D) 71
E) 79

35). 26, 13, 13, 19.5, ?, 97.5


A) 39
B) 45
C) 48
D) 41
E) 35

36). 82, 93, ?, 148, 192, 247


A) 124
B) 122
C) 117
D) 115
E) 120

37). 163, ?, 188, 216, 253, 298


A) 175
B) 168
C) 171
D) 170
E) 169

38). 224, 228, 219, 235, ?, 246


A) 210
B) 209
C) 220
D) 225
E) 215

39). 52, 102, 303, 1208, ?, 36204


A) 6240

www.ibpsguide.com | estore.ibpsguide.com| www.sscexamguide.com


14
For Free Online Mock Test for IBPS PO/Clerk 2017 – Visit: estore.ibpsguide.com
500 Most Important Number Series Questions

B) 6040
C) 6035
D) 6020
E) 6340

40). 121, 128, 144, 168, 199, ?


A) 237
B) 236
C) 226
D) 216
E) 246

Explanation With Answer Key:


31). E)
The series is +4, +16, +64, +256, +1042,…..
Ie 5 + 4 = 9; 9 + 16 = 25, 25 + 64 = 89
89 + 256 = 345, 345+1024 = 1369

32). B)
The series is ×3 – 1, (repeateD)
Ie 6 × 3 – 1 = 17, 17 × 3 – 1 = 50, 50 × 3 – 1 = 149 , 149 × 3 – 1 = 446, 446 × 3 – 1 = 1337

33). A)
The Series +(1)3 , -(2)3, +(3)3, …..
Ie 61 + 13 = 62, 62 – 23 = 54, 54 + 33 = 81
81 – 43 = 17, 17 + 53 = 142

34). D)
6 + 5 = 11, 11 +7 = 18, 18 + 11 = 29, 29 + 17 = 46, 46 + 25 = 71

35). A)
The series is ×0.5, ×1, ×1.5, ×2, ×2.5,…
Ie 26 × 0.5 = 13, 13 × 1 = 13, 13 × 1.5 = 19.5, 19.5 × 2 = 39, 39 × 2.5 = 97.5, …..

www.ibpsguide.com | estore.ibpsguide.com| www.sscexamguide.com


15
For Free Online Mock Test for IBPS PO/Clerk 2017 – Visit: estore.ibpsguide.com
500 Most Important Number Series Questions

36). D)
The series is
82 + 11 = 93
93 + 22 = 115
115 + 33 = 148
148 + 44 = 192
192 + 55 = 247

37). D)
The Series is
163 + 7 = 170 ; 170 + 18 = 188 ; 188 + 28 = 216; 216 + 37 = 253; 253 + 45 = 298
7 + 11 = 18; 18 + 10 = 28; 28 + 9 = 37; 37 + 8 = 45

38). A)
The series is
224 + 22 = 228
228 – 32 = 219
219 + 42 = 235
235 – 52 = 210
210 + 62 = 246

39). C)
The series is 52 × 2 – 2 = 102
102 × 3 – 3 = 303
303 × 4 – 4 = 1208
1208 × 5 – 5 = 6035
6035 × 6 – 6 = 36204

40). B)
The series is
121 + 7 = 128; 128 + 16 = 144
144 + 24 = 168 ; 168 + 31 = 199; 199 + 37 = 236
7 + 9 = 16; 16 + 8 = 24; 24 + 7 = 31; 31 + 6 = 37

www.ibpsguide.com | estore.ibpsguide.com| www.sscexamguide.com


16
For Free Online Mock Test for IBPS PO/Clerk 2017 – Visit: estore.ibpsguide.com
500 Most Important Number Series Questions

Directions (Q. 41-50): What should come in place of question mark in the following number
series?
41). 5, ?, 41, 191, 941, 4691
A) 12
B) 15
C) 18
D) 21
E) 11

42). 5000, 2508, 1262, 639, ?, 171.75


A) 329.6
B) 339
C) 327.5
D) 326
E) 335.5

43). 9, 5, ?, 10.5, 23, 60


A) 8
B) 7.5
C) 9
D) 6
E) 5.5

44). 551, 600, 636, 661, 677 ?


A) 686
B) 676
C) 689
D) 645
E) 690

45). 5, 2.75, ?, 7.87, 19.75, 55.625


A) 7.5
B) 7.875

www.ibpsguide.com | estore.ibpsguide.com| www.sscexamguide.com


17
For Free Online Mock Test for IBPS PO/Clerk 2017 – Visit: estore.ibpsguide.com
500 Most Important Number Series Questions

C) 3.75
D) 7.1
E) 8.5

46). 16, 18, 27, 55, 120, ?


A) 232
B) 246
C) 254
D) 242
E) 248

47). 13, 10.8, 5.4, 8.2, 17.8, ?


A) 5.8
B) 5.6
C) 6.2
D) 7.2
E) 6.8

48). 4, 19, 64, 139, ?, 379


A) 254
B) 233
C) 236
D) 244
E) 262

49). 7, 17, 37, 77, ?, 317


A) 167
B) 160
C) 157
D) 147
E) 151

50). 12, 8, 10, 17, 36, ?


A) 92

www.ibpsguide.com | estore.ibpsguide.com| www.sscexamguide.com


18
For Free Online Mock Test for IBPS PO/Clerk 2017 – Visit: estore.ibpsguide.com
500 Most Important Number Series Questions

B) 84
C) 76
D) 98
E) 104

Explanation with Answers Key:


41). E)
The series is
5 + [(6/5)×5] = 11, 11 + (6×5) = 41, 41 + (30×5) = 191
191 + (150×5) = 941, 941 + (750×5) = 4691

42). C)
The series is 5000 ÷ 2 + 8 = 2508, 2508 ÷ 2 + 8 = 1262, 1262 ÷ 2 + 8 = 639, 639 ÷ 2 + 8 =
327.5, 327.5 ÷ 2 + 8 = 171.75,….

43). D)
The series is 9 x 0.5 + 0.5 = 5, 5 × 1 + 1 = 6, 6 x 1.5 + 1.5 = 10.5, 10.5 × 2 + 2 = 23, 23 x 2.5 +
2.5 = 60

44). A)
The series is +72, +62, +52, ... .
551 +49 = 600, 6
00 + 36= 636, 636 + 25 = 661, 661+16 = 677, 677+9 = 686

45). C)
The series is
5 x 0.5 + 0.25 x 12 = 2.75,
2.75 x 1 + 0.25 x 22 = 3.75,
3.75 x 1.5 + 0.25 x 32 = 7.875,
7.875 x 2 + 0.25 x 42 = 19.75,
19.75 x 2.5 + 0.25 x 52 = 55.625, ...

46). B)

www.ibpsguide.com | estore.ibpsguide.com| www.sscexamguide.com


19
For Free Online Mock Test for IBPS PO/Clerk 2017 – Visit: estore.ibpsguide.com
500 Most Important Number Series Questions

The series is +13 + 1, +23 + 1, +33 + 1, +43 + 1, +53 + 1, ...


ie 18 + 23 + 1 = 27,
27 + 33 + 1 = 55,
55 + 43 + 1 = 120,
120 + 53 + 1 = 246, ...

47). B)
The series is a combination of two series

48). D)
The series is +15 x 1, +15 x 3, +15 x 5, +15 x 7, +15 x 9, ...
ie 4 + 15 x 1 = 19,
19 + 15 x 3 = 64,
64 + 15 x 5 = 139,
139 + 15 x 7 = 244,
244 + 15 x = 379, ...

49). C)
The series is +10, +20, +40, +80, +160,
ie 7 + 10 = 17,
17 + 20 = 37,
37 + 40 = 77,
77 + 80 = 157,
157 160 = 317, ...

50). A)
The series is x0.5 + 2, x 1 + 2, x1.5 + 2, x2 + 2, ...

www.ibpsguide.com | estore.ibpsguide.com| www.sscexamguide.com


20
For Free Online Mock Test for IBPS PO/Clerk 2017 – Visit: estore.ibpsguide.com
500 Most Important Number Series Questions

ie 12 x 0.5 + 2 = 8,
8 x 1 + 2 = 10,
10 x 1.5 + 2 = 17,
17 x 2 + 2 = 36,

36 x 2.5 +2 = 92, ...

Directions (Q. 51-60): Find out the missing number in place of question mark (?) in the
following number series.

51). 3, 7, 29, 143, 779, ?


A) 4997
B) 4649
C) 4847
D) 4799
E) 4999

52). 4, 5, ?, 88, 620, 5585


A) 16
B) 18
C) 17
D) 20
E) 22

53). 11, 20, 42, ?, 166, 330


A) 72
B) 66
C) 86
D) 80
E) 82

54). 128, 64, 96, 240, ?, 3780


A) 740
B) 840

www.ibpsguide.com | estore.ibpsguide.com| www.sscexamguide.com


21
For Free Online Mock Test for IBPS PO/Clerk 2017 – Visit: estore.ibpsguide.com
500 Most Important Number Series Questions

C) 850
D) 845
E) 745

55). 8484, ?, 8486, 8495, 8488, 8493


A) 8485
B) 8584
C) 8684
D) 8497
E) 8496

56). 68, 132, 268, ?, 1068, 2132


A) 394
B) 464
C) 325
D) 532
E) 484

57). 23, 12, 13, 21, ?, 112.5


A) 33
B) 44
C) 55
D) 66
E) 65

58). 8, 10, 18, 44, 124, ?


A) 366
B) 263
C) 194
D) 276
E) 286

59). 4, 4, 21, 68, ?, 2154


A) 435

www.ibpsguide.com | estore.ibpsguide.com| www.sscexamguide.com


22
For Free Online Mock Test for IBPS PO/Clerk 2017 – Visit: estore.ibpsguide.com
500 Most Important Number Series Questions

B) 365
C) 462
D) 572
E) 425

60). 7, 20, 59, 176, ?, 1580


A) 527
B) 526
C) 627
D) 562
E) 270

Explanation:
51). D)
The series is x2 + 13, x3 + 23, x4 + 33, x5 + 43, ...
ie 3 x 2 +13 = 7,
7 x 3 + 23 = 29,
29 x 4 + 33 = 143,
143 x 5 +43 = 779,
779 x 6 + 53 = 4799,

52). C)
The series is x1 + 1, x3 + 2, ×5 + 3, x7 + 4, x9 + 5, x11 + 6,
Ie 4 x 1+1 = 5,
5 x3+2=17,
17 x 5 + 3 = 88,
88 x 7 + 4 = 620,
620 x 9 + 5 = 5585,

53). E)
The series is x2 - 2, x2 + 2, (Alternately repeateD)

www.ibpsguide.com | estore.ibpsguide.com| www.sscexamguide.com


23
For Free Online Mock Test for IBPS PO/Clerk 2017 – Visit: estore.ibpsguide.com
500 Most Important Number Series Questions

54). B)
The series is x0.5, x1.5, x2.5, x3.5, ...

55). D)
The series is +13, —11, +9, —7, +5, ... +13

56). D)
The series is x2 - 4, x2 + 4, (repeateD)

57). B)
The series is ×1/2 + 0.5, ×1+1, ×1.5+1.5…..

58). A)
The series is

59). B)

www.ibpsguide.com | estore.ibpsguide.com| www.sscexamguide.com


24
For Free Online Mock Test for IBPS PO/Clerk 2017 – Visit: estore.ibpsguide.com
500 Most Important Number Series Questions

The Series is

60). A)
The series is ×3 – 1, ×3 – 1 (repeateD)
7 × 3 = 21 – 1 = 20
20 × 3 = 60 – 1 = 59
59 × 3 = 177 – 1 = 176
176 × 3 = 528 – 1 = 527

527 × 3 = 1581 – 1 = 1580


Directions (Q. 61-70): what should come in the place of question mark (?) in the given
number series?
61). 14, 8, 7, 11.5, 22, ?
A) 54
B) 64
C) 62
D) 58
E) 56

62). 8, 14, 25, 46, 82, ?


A) 132
B) 130
C) 138
D) 128
E) 142

63). 13, 14, 30, 93, ?, 1885


A) 358
B) 336
C) 364
D) 376

www.ibpsguide.com | estore.ibpsguide.com| www.sscexamguide.com


25
For Free Online Mock Test for IBPS PO/Clerk 2017 – Visit: estore.ibpsguide.com
500 Most Important Number Series Questions

E) 380

64). 65, 70, 63, 74, 61, ?


A) 78
B) 58
C) 72
D) 46
E) 68

65). 9, 11, 16, 33, 98, ?


A) 350
B) 355
C) 360
D) 365
E) 370

66). 4, 11, 26, 57, 120, ?


A) 247
B) 251
C) 187
D) 236
E) 215

67). 50, 81, 121, 172, 236, ?


A) 392
B) 309
C) 361
D) 379
E) 315

68). 84, 82.4, 85.6, 80.8, ?, 79.2


A) 72.8
B) 86.4
C) 88.8

www.ibpsguide.com | estore.ibpsguide.com| www.sscexamguide.com


26
For Free Online Mock Test for IBPS PO/Clerk 2017 – Visit: estore.ibpsguide.com
500 Most Important Number Series Questions

D) 87.2
E) 85.7

69). 900, 448, 220, 104, 44, ?


A) 8
B) 16
C) 24
D) 42
E) 12

70). 60, ?, 120, 80, 40, 16


A) 90
B) 110
C) 120
D) 70
E) 100

Explanation:
61). E)

62). C)

63). D)

64). A)

www.ibpsguide.com | estore.ibpsguide.com| www.sscexamguide.com


27
For Free Online Mock Test for IBPS PO/Clerk 2017 – Visit: estore.ibpsguide.com
500 Most Important Number Series Questions

65). B)

66). A)

67). E)

68). D)

69). E)

70). C)

www.ibpsguide.com | estore.ibpsguide.com| www.sscexamguide.com


28
For Free Online Mock Test for IBPS PO/Clerk 2017 – Visit: estore.ibpsguide.com
500 Most Important Number Series Questions

Directions (Q. 71-80): what should come in place of question mark (?) in the following
questions?
71). 2, 2, 4, 16, ?, 2048
A) 138
B) 128
C) 256
D) 124
E) 40

72). 2, ?, 27, 113, 561, 3369


A) 11
B) 13
C) 9
D) 15
E) 18

73). 4, 14, ?, 149, 295, 293


A) 40
B) 45
C) 50
D) 51
E) 48

74). 5, 12, 38, ?, 772, 4634


A) 150
B) 158
C) 160
D) 162
E) 154

75). 37, ?, 222, 666, 1332, 3996


A) 120
B) 110
C) 111

www.ibpsguide.com | estore.ibpsguide.com| www.sscexamguide.com


29
For Free Online Mock Test for IBPS PO/Clerk 2017 – Visit: estore.ibpsguide.com
500 Most Important Number Series Questions

D) 119
E) 118

76). 14, 12.4, 15.6, ?, 17.2, 9.2


A) 10.8
B) 12
C) 12.8
D) 14.6
E) 18.6

77). 28, ?, 168, 672, 3360, 20160


A) 60
B) 84
C) 72
D) 56
E) 40

78). ?, 256, 3375, 196, 2197, 144


A) 4319
B) 4923
C) 4913
D) 4139
E) 4096

79). 987, ?, 945, 1071, 903, 1113


A) 1040
B) 1039
C) 1030
D) 1029
E) 1027

80). 480, 240, 160, ?, 96, 80


A) 100
B) 120

www.ibpsguide.com | estore.ibpsguide.com| www.sscexamguide.com


30
For Free Online Mock Test for IBPS PO/Clerk 2017 – Visit: estore.ibpsguide.com
500 Most Important Number Series Questions

C) 136
D) 140
E) 135

Solution:
71). B) The series is

72). A) The series is 2 x 2 + 7 = 11,


11 x 3 - 6 = 27, 27 x 4 + 5 = 113, 113 x 5 - 4 = 561, 561 x 6 + 3 = 3369,

73). D) The series is 4 x 5 - 6 = 14,


14 x 4 – 5 = 51, 51 x 3 - 4 = 149, 149 x 2 — 3 = 295, 295 x 1 — 2 = 293, ...

74). E) The series is 5 x 2 + 2 = 12,


12 x 3 + 2 = 38, 38 x 4 + 2 = 154, 154 x 5 + 2 = 772, 772 x 6 + 2 = 4634, ...

75). C) The series is

76). A) The series is 14 +1.6 x – 1 = 12.4, 12.4 + 1.6 × 2 = 15.6, 15.6 + 1.6 x -3 = 10.8, 10.8 +
1.6 x 4 = 17.2, 17.2 + 1.6 × -5 = 9.2

77). D) The series is 28 x 2 = 56, 56 x3 =168, 168 x 4 = 672, 672 x 5 = 3360,


3360 x 6 = 20160,

78). C) The series is (17)3, (16)2, (15)3, (14)2, (13)3, (12)2,…..


ie 4913, 256, 3375, 196, 2197, 144

79). D) The series is

www.ibpsguide.com | estore.ibpsguide.com| www.sscexamguide.com


31
For Free Online Mock Test for IBPS PO/Clerk 2017 – Visit: estore.ibpsguide.com
500 Most Important Number Series Questions

80). B) The series is

Directions (Q. 81-85): What will come in place of question mark (?) in the given number
series?
81). 28 39 63 102 158 ?
A) 232
B) 242
C) 233
D) 244
E) None of these

82). 7 16 141 190 919 ?


A) 1029
B) 1019
C) 1020
D) 1030
E) None of these

83). 12 17 32 57 92 ?
A) 198
B) 195
C) 137
D) 205
E) None of these

84). 19 25 45 87 159 ?
A) 254
B) 279
C) 284

www.ibpsguide.com | estore.ibpsguide.com| www.sscexamguide.com


32
For Free Online Mock Test for IBPS PO/Clerk 2017 – Visit: estore.ibpsguide.com
500 Most Important Number Series Questions

D) 269
E) None of these

85). 83 124 206 370 698 ?


A) 1344
B) 1324
C) 1364
D) 1334
E) None of these

Directions (Q. 86-90): In each of these questions a number series is given. In each series
only one number is missing. Find out the number.
86). 78 421 -91 ? -362 969
A) 645
B) 635
C) 640
D) 648
E) 638

87). 584 662 1246 1908 ? 5062


A) 3155
B) 3164
C) 3154
D) 3162
E) 4654

88). 51 74 143 350 971 ?


A) 2818
B) 2318
C) 2384
D) 2834
E) 2438

89). 740 ? 181 86.5 38.25 13.125

www.ibpsguide.com | estore.ibpsguide.com| www.sscexamguide.com


33
For Free Online Mock Test for IBPS PO/Clerk 2017 – Visit: estore.ibpsguide.com
500 Most Important Number Series Questions

A) 364
B) 368
C) 366
D) 378
E) 374

90). 4 25 ? 511 1537 3079


A) 125
B) 129
C) 137
D) 127
E) 139

Solution:
81). C) The series is +(10 x 1 + 12), +(10 x 2 + 22), +(10 x 3 + 32), +(10 x 4 + 42),
+(10 x 5 + 52),…..
ie 28 + 10 x 1 + 12 = 39,
39 + (10 x 2 + 22) = 63,
63 + (10 x 3 + 32) = 102,
102 + (10 x 4 + 42) = 158,
158 + (10 x 5 + 52) = 233, ...
82). E) The series +32, +53, +72, +93, +112, +133, ...
ie 7 + 32= 16, 16 + 53= 141, 141 + 72= 190, 190 + 93 = 919, 919 + 112 = 1040, ...
83). C) The series is +(5 x 1), +(5 x 3), +(5 x 5), +(5 x 7), +(5 x 9),….
ie 12 + (5 x 1) = 17, 17 + (5 x 3) = 32, 32 + (5 x 5) = 57, 57 + (5 x 7) = 92, 92 + (5 x 9) = 137,
84). D) The series is +22 + 2, +42 + 4, +62 + 6, +82 + 8, +102 + 10, ...
ie 19 + 22 + 2 = 25, 25 + 42 + 4 = 45, 45 + 62 + 6 = 87, 87 + 82 + 8 = 159, 159 + 102 + 10 = 269,
...
85). E) The series is +41, +82, +164, +328, +656,….
ie 83 + 41 = 124, 124 + 82 = 206, 206 + 164 = 370, 370 + 328 = 698, 698 + 656 =1354,…..
86). E) The series is 78 + 73 = 421
421 – 83 = -91, -91 + 93 = 638, 638 -103 = -362, -362 + 113 = 969,
87). C) The series is

www.ibpsguide.com | estore.ibpsguide.com| www.sscexamguide.com


34
For Free Online Mock Test for IBPS PO/Clerk 2017 – Visit: estore.ibpsguide.com
500 Most Important Number Series Questions

88). D) The series is

89). B) The series is 740 x (1/2) - 2 =368,


368 x (1/2) -3 =181, 181x (1/2) - 4 =-86.5, 86.5 x (1/2) = 38.25, 38.25 x (1/2) – 6= 13.125
90). D) The series is x6 + 1, x5 + 2, x4 + 3, ×3 + 4, x2 + 5,... ...
? = 25 x 5 +2 = 127

Directions (Q. 91-100): What should come in place of question mark (?) in the following
number series?
91). 6 -3 24 -57 186 ?
A) -582
B) 475
C) 627
D) 545
E) -543

92). 4 8 14 23 36.5 ?
A) 124
B) 91.75
C) 108.5
D) 84.5
E) 56.75

93). 9 5 6 10.5 23 ?
A) 60
B) 16
C) 18.5

www.ibpsguide.com | estore.ibpsguide.com| www.sscexamguide.com


35
For Free Online Mock Test for IBPS PO/Clerk 2017 – Visit: estore.ibpsguide.com
500 Most Important Number Series Questions

D) 42.5
E) 32.5

94).1 19 55 112 193 ?


A) 301
B) 336
C) 330
D) 252
E) 636

95). 3 7 15 31 63 ?
A) 95
B) 125
C) 128
D) 127
E) 98

96). 13 27 56 ? 234 473


A) 110
B) 112
C) 115
D) 120
E) 80

97). 48 144 288 864 1728 ?


A) 5164
B) 4185
C) 5284
D) 5184
E) 5418

98). 46656 1156 32768 ? 21952 676


A) 861
B) 900

www.ibpsguide.com | estore.ibpsguide.com| www.sscexamguide.com


36
For Free Online Mock Test for IBPS PO/Clerk 2017 – Visit: estore.ibpsguide.com
500 Most Important Number Series Questions

C) 2700
D) 961
E) 841

99). 4 14 32 60 ? 154
A) 90
B) 80
C) 95
D) 100
E) 108

100). 56 ? 80 102 128 161


A) 66
B) 67
C) 77
D) 75
E) 87

Solution:
91). The series is x-3 +15, x-3 +15, (repeateD)
ie 6 x -3 + 15 = -3, -3 x -3 + 15 24,
24 x -3 + 15 = -57, -57 x -3 + 15 = 186, 186 x -3 + 15 = -543,
Answer: E)

92). The series is x1.5 + 2 (repeateD)


ie 4 x 1.5 +2 = 8, 8 x 1.5 + 2 = 14, 14x 1.5 + 2 = 23, 23 x 1.5 + 2 = 36.5, 36.5 x 1.5 + 2 = 56.75,
Answer: E)

93). The series is x0.5 + 0.5, x1 + 1, ×1.5+ 1.5, x2 + 2, x2.5 + 2.5,


ie 9 x 0.5 + 0.5 = 5, 5 x 1 + 1 = 6, 6 x 1.5 + 1.5 = 10.5, 10.5 x 2 + 2 = 23, 23 x 2.5 + 2.5 = 60.
Answer: A)

94). The series is

www.ibpsguide.com | estore.ibpsguide.com| www.sscexamguide.com


37
For Free Online Mock Test for IBPS PO/Clerk 2017 – Visit: estore.ibpsguide.com
500 Most Important Number Series Questions

Answer: A)

95). The series is x2 + 1 (repeateD)


ie 3 × 2 + 1 = 7, 7 x 2 + 1 =15, 15 × 2 + 1 = 31, 31 x 2 + 1= 63, 63 x 2 +1 = 127,…
Answer: D)

96). The series is 13 x 2 + 1 = 27, 27 x 2 + 2 = 56, 56 x 2 + 3 = 115, 115 x 2 + 4 = 234, 234 x 2 +


5 = 473, ...
Answer: C)

97). The series is x3, x2 (alternatively)

Answer: D)

98). The series is (36)3, (34)2, (32)3, (30)2, (28)3, (26)2,


ie (36)3 = 46656, (34)2 = 1156, (32)3 = 32768, (30)2 = 900, (28)3 = 21952, (26)2 = 676,…..
Answer: B)

99). The series is +32 + 1, +42 + 2, +52 + 3, +62 + 4, +72 + 5,


le 4 + 32 + 1 = 14, 14 + 42+ 2 = 32, 32 + 52 + 3 = 60, 60 + 62 + 4 = 100, 100 + 72 +5 = 154,
Answer: D)

100). The series is +11, +13, +22, +26, +33, +39, ... ie 56 + 11 = 67, 67 + 13 = 80, 80 + 22 = 102,
102 + 26 = 128, 128 + 33 = 161, 161 + 39 = 200,
Answer: B)

www.ibpsguide.com | estore.ibpsguide.com| www.sscexamguide.com


38
For Free Online Mock Test for IBPS PO/Clerk 2017 – Visit: estore.ibpsguide.com
500 Most Important Number Series Questions

Direction (Q. 101-105): What value should come in the place of question mark (?) in the
following number series:
101).3, 11, 54, 339, 2732,?
A) 21650
B) 27345
C) 22852
D) 27356
E) 27375

102). 358, 320, 284, 250,?


A) 236
B) 258
C) 264
D) 218
E) 278

103). 8, 22, 80, 364, ?


A) 4805
B) 2010
C) 2007
D) 2560
E) 2700

104). 7, 22, 118, 1107, ?


A) 24334
B) 17756
C) 17779
D) 17772
E) 28986

105). 8, 37, 104, 233, 454, ?


A) 338
B) 837
C) 823

www.ibpsguide.com | estore.ibpsguide.com| www.sscexamguide.com


39
For Free Online Mock Test for IBPS PO/Clerk 2017 – Visit: estore.ibpsguide.com
500 Most Important Number Series Questions

D) 934
E) 803

Direction (Q. 106-110): What will come in place of the question mark :
106). 15, 16, 10, 33, ?, 66.25
A) 14.50
B) 12.50
C) 12.25
D) 22.25
E) None of these

107). 2, 9, 82, ?, 32801


A) 13130
B) 13456
C) 1313
D) 2675
E) None of these

108). 32, 321, 682, 1123, ?, 2277


A) 1702
B) 1652
C) 1834
D) 1567
E) None of these

109). 84, ?, 78, 109, 72, 113, 70


A) 109
B) 124
C) 144
D) 107
E) None of these

110). 11, 15, ?, 71, 333


A) 27

www.ibpsguide.com | estore.ibpsguide.com| www.sscexamguide.com


40
For Free Online Mock Test for IBPS PO/Clerk 2017 – Visit: estore.ibpsguide.com
500 Most Important Number Series Questions

B) 36
C) 29
D) 25
E) None of these

Solution:
Directions (Q. 101-105):
101). B) Series is x2+5, x4+10, x6+15, x8+20, x10+25
102). D) Series is 19^2-3,18^2-4, 17^2-5, 16^2-6, 15^2-7
103). C) Series is x2.5+2, x3.5+3, x4.5+4, x5.5+5
104). D) Series is x1^2+15, x2^2+30, x3^2+45, x4^2+60
105). E) Series is Difference between no. is, 3^3+2, 4^3+3, 5^3+4, 6^3+5, 7^3+6

Directions(Q. 106-110)
106). C) x1+1, ÷2+2, x3+3, ÷4+4, x5+5
107). C) x2^2-2+3, x3^2-3+4, x4^2-4+5, x5^2-5+6
108). B) +17^2, +19^2, +21^2, +23^2, +25^2
109). D) +23, -29, +31, -37, +41, -43
110).D) Diff of diff is multiples of 6

Direction (Q. 111-115): What value should come in the place of question mark (?) in the
following number series :
111). 6, 11, 32, 111, 464, ?
A) 2165
B) 2205
C) 2285
D) 2345
E) 2375

112). 3, 14, 39, 84, 155, ?


A) 236
B) 258
C) 264
D) 272

www.ibpsguide.com | estore.ibpsguide.com| www.sscexamguide.com


41
For Free Online Mock Test for IBPS PO/Clerk 2017 – Visit: estore.ibpsguide.com
500 Most Important Number Series Questions

E) 278

113). 8, 12, 24, 60, 180, ?


A) 480
B) 510
C) 630
D) 720
E) 780

114). 168, 171, 178, 191, 212, ?


A) 243
B) 247
C) 251
D) 254
E) 257

115). 4, 7, 10, 11, 22, 19, 46, ?


A) 33
B) 35
C) 37
D) 39
E) 41

Direction (Q. 116-120): What will come in place of the question mark :
116). 124, 228, 436, ?, 1684, 3348
A) 944
B) 852
C) 872
D) 444
E) None of these

117). 1, 3, 24, 360, 8640, ?, 14515200


A) 282400
B) 292400

www.ibpsguide.com | estore.ibpsguide.com| www.sscexamguide.com


42
For Free Online Mock Test for IBPS PO/Clerk 2017 – Visit: estore.ibpsguide.com
500 Most Important Number Series Questions

C) 302400
D) 25426
E) None of these

118). 10, 17, 48, 165, ?, 3475, 20892


A) 688
B) 712
C) 848
D) 918
E) None of these

119). 15, 17, 38, 120, 488, ?, 14712


A) 2450
B) 2650
C) 2850
D) 2950
E) None of these

120). 1, 6, 19, ?, 85, 146, 231


A) 46
B) 44
C) 48
D) 65
E) None of these

Solution:
111). D) Series is ×1+5, × 2 + 10, ×3 + 15, × 4 + 20
112). B) Series is 1 + 12 + 13 , 2 + 22 +23 , 3 + 32 + 33
113). C) Series is × 0.5 + 8, ×1.0 + 12, ×1.5 + 24
114). A) Series is + 12 + 2, + 22 + 3, + 32 + 4......
115). B) Series 1 , 4+6 = 10, 10 + 12 = 22, 22 + 24 = 26
Series 2 7, 7 + 4 = 11, 11 + 8= 19, 19 + 16 = 35

116). B) × 2 - 20, ×2 - 20, × 2 - 20, ------------------

www.ibpsguide.com | estore.ibpsguide.com| www.sscexamguide.com


43
For Free Online Mock Test for IBPS PO/Clerk 2017 – Visit: estore.ibpsguide.com
500 Most Important Number Series Questions

117). C) ×(22 - 1), × (32 - 1), (42 - 1), ×(52 -1), ×(62 -1) ----------
118). A) ×1 + 7, ×2 + 14, × 3 + 21, ×4 + 28 ----------------
119). A) × 1 + 2, ×2+ 4 , ×3 + 6, × 4 + 8, × 5 + 10 ------------
120). B)

Directions (121 – 130): What should come in place of the question mark (?) in the following
number series?

121). 7 5 7 17 63 ?
A) 308
B) 302
C) 309
D) 409
E) 390
122). 50 ? 61 89 154 280
A) 52
B) 51
C) 60
D) 62
E) 60
123). 17 19 25 37 ? 87
A) 47
B) 37
C) 57
D) 67
E) 75
124). 11 14 19 28 43 ?
A) 55
B) 44

www.ibpsguide.com | estore.ibpsguide.com| www.sscexamguide.com


44
For Free Online Mock Test for IBPS PO/Clerk 2017 – Visit: estore.ibpsguide.com
500 Most Important Number Series Questions

C) 77
D) 88
E) 66
125). 26 144 590 1164 ?
A) 1296
B) 1182
C) 2059
D) 1182
E) 1181
126). 2916 972 ? 108 36 12
A) 324
B) 234
C) 248
D) 391
E) None of these
127). 8, 27, 125, ?, 1331
A) 512
B) 216
C) 81
D) 343
E) 169
128). 2 11 30 97 ? 1975
A) 325
B) 360
C) 34
D) 394
E) 376
129). 8 47 234 935 ? 5607
A) 2804
B) 2802
C) 2808
D) 2801
E) 2800
130). 6 9 15 27 51 ?

www.ibpsguide.com | estore.ibpsguide.com| www.sscexamguide.com


45
For Free Online Mock Test for IBPS PO/Clerk 2017 – Visit: estore.ibpsguide.com
500 Most Important Number Series Questions

A) 84
B) 75
C) 99
D) 123
E) None of the above
Answer Key with Solutions:
121. C) 309
7x1–2=5
5x2–3=7
7 x 3 – 4 = 17
17 x 4 – 5 = 63
63 x 5 – 6 = 309
122. A) 52
50..50+(1^3+1) = 52
52+(2^3+1)=61.
61+(3^3+1)=89
89+(4^3+1)=154
154 + (5^3+1) = 280
123. C) 57
17 + 1 x 2 = 19
19 + 2 x 3 = 25
25 + 3 x 4 = 37
37 + 4 x 5 = 57
57 + 5 x 6 = 87
124. E) 66
3….5…9…15…23
…2….4….6….8…
Answer 43+23= 66
125. B) 1182
26 x 6 – 12 = 144
144 x 4 + 14 = 590
590 x 2 – 16 = 1164
1164 x 1 + 18 = 1182
126. A) 324

www.ibpsguide.com | estore.ibpsguide.com| www.sscexamguide.com


46
For Free Online Mock Test for IBPS PO/Clerk 2017 – Visit: estore.ibpsguide.com
500 Most Important Number Series Questions

2916/3 = 972
972/3 = 324
324/3 = 108
108/3 = 36
36/3 = 12
127. D) 343
Cube prime series 23, 33, 53, 73, 113
128. D)
2 x 1 + 9 = 11 x 2 + 8 = 30 x 3 + 7 = 97 x 4 + 6 = 394
129. A)
x 6 – 1, x 5 – 1, x 4 – 1, x 3 – 1 = 935 x 3 – 1 = 2804
130. C)
+3, +6, +12, +24, +48 = 51 + 48 = 99

Directions (Q.131-140) What should come in place of question mark (?) in the following number
series?
131). 15, 21, 39, 77, 143, ?
A) 243
B) 240
C) 253
D) 245
E) None of these

132). 33, 39, 57, 87, 129, ?


A) 183
B) 177
C) 189
D) 199
E) None of these

133). 15, 19, 83, 119, 631, ?


A) 731
B) 693
C) 712

www.ibpsguide.com | estore.ibpsguide.com| www.sscexamguide.com


47
For Free Online Mock Test for IBPS PO/Clerk 2017 – Visit: estore.ibpsguide.com
500 Most Important Number Series Questions

D) 683
E) None of these

134). 19, 26, 40, 68, 124, ?


A) 246
B) 238
C) 236
D) 256
E) None of these

135). 43, 69, 58, 84, 73,?


A) 62
B) 98
C) 109
D) 63
E) None of these

136). 1, 7, 49, 343,?


A) 16807
B) 1227
C) 2058
D) 2401
E) None of these

137). 13, 20, 39, 78, 145,?


A) 234
B) 244
C) 236
D) 248
E) None of these

138). 12, 35, 81, 173, 357, ?


A) 725
B) 715

www.ibpsguide.com | estore.ibpsguide.com| www.sscexamguide.com


48
For Free Online Mock Test for IBPS PO/Clerk 2017 – Visit: estore.ibpsguide.com
500 Most Important Number Series Questions

C) 726
D) 736
E) None of these

139). 3, 100, 297, 594, 991, ?


A) 1489
B) 1479
C) 1478
D) 1498
E) None of these

140). 112, 119, 140, 175, 224, ?


A) 277
B) 276
C) 287
D) 266
E) None of these

Solution:

131).

Answer: D)

132).

www.ibpsguide.com | estore.ibpsguide.com| www.sscexamguide.com


49
For Free Online Mock Test for IBPS PO/Clerk 2017 – Visit: estore.ibpsguide.com
500 Most Important Number Series Questions

Answer: A)

133).

Answer: A)

134).

Answer: C)

135).

www.ibpsguide.com | estore.ibpsguide.com| www.sscexamguide.com


50
For Free Online Mock Test for IBPS PO/Clerk 2017 – Visit: estore.ibpsguide.com
500 Most Important Number Series Questions

Answer: E)

136).

Answer: D)

137).

Answer: D)

138).

Answer: A)

139). 3 + 97 = 100 + 100↓


100 + 197 = 297 + 100↓
297 + 297 = 594 + 100↓
594 + 397 = 991 + 100↓
991 + 497 = 1488
Answer: E)

www.ibpsguide.com | estore.ibpsguide.com| www.sscexamguide.com


51
For Free Online Mock Test for IBPS PO/Clerk 2017 – Visit: estore.ibpsguide.com
500 Most Important Number Series Questions

140).

Answer: C)

141). 1, 4, 9, 16, 25, ?


A) 49
B) 60
C) 30
D) 36
E) 48

142). 2, 5, 11, 23, ?


A) 46
B) 52
C) 47
D) 57
E) 48

143). 198, 194, 185, 169, ?


A) 92
B) 136
C) 144
D) 112
E) None of these

144). 101, 100, ?, 87, 71, 46


A) 92
B) 88
C) 89
D) 96
E) None of these

www.ibpsguide.com | estore.ibpsguide.com| www.sscexamguide.com


52
For Free Online Mock Test for IBPS PO/Clerk 2017 – Visit: estore.ibpsguide.com
500 Most Important Number Series Questions

145). 100, 50, 52, 26, 28, ?, 16, 8


A) 30
B) 36
C) 14
D) 32
E) None of these

146). 462, 420, 380, ?, 306


A) 322
B) 332
C) 342
D) 352
E) None of these

147). 0, 6, 24, 60, 120, 210, ?


A) 290
B) 240
C) 336
D) 504
E) None of these

148). 3, 15, 35, 63, ?, 143


A) 120
B) 110
C) 99
D) 91
E) None of these

149). 4, 9, 19, 39, 79, ?


A) 159
B) 119
C) 139
D) 169

www.ibpsguide.com | estore.ibpsguide.com| www.sscexamguide.com


53
For Free Online Mock Test for IBPS PO/Clerk 2017 – Visit: estore.ibpsguide.com
500 Most Important Number Series Questions

E) None of these

150). 5, 10, 8, 12, 11, 14, ?, 16


A) 17
B) 13
C) 20
D) 14
E) None of these
Solution:

141). The pattern is as follows

Answer: D)

142). The pattern is as follows

Answer: C)

143). The pattern is as follows

Answer: C)

144). The pattern is as follows

www.ibpsguide.com | estore.ibpsguide.com| www.sscexamguide.com


54
For Free Online Mock Test for IBPS PO/Clerk 2017 – Visit: estore.ibpsguide.com
500 Most Important Number Series Questions

Missing number = 100 – (2)2 = 100 – 4 = 96


Answer: D)

145). The pattern is as follows

Missing number = 28 / 2 = 14
Answer: C)

146). The pattern is as follows

Answer: C)

147). The pattern is as follows


03 – 0 = 0
23 – 2 = 6
33 – 3 = 24
43 – 4 = 60
53 – 5 = 120
63 – 6 = 210
73 – 7 = 336
? = 336
Answer: C)

148). The pattern is as follows


3 = 1 × 3 ; 15 = 3 × 5
35 = 5 × 7 ; 63 = 7 × 9
99 = 9 × 11; 143 = 11 × 13

www.ibpsguide.com | estore.ibpsguide.com| www.sscexamguide.com


55
For Free Online Mock Test for IBPS PO/Clerk 2017 – Visit: estore.ibpsguide.com
500 Most Important Number Series Questions

? = 99
Answer: C)

149). The pattern is as follows

? = 342
Answer: A)

150). There are two alternate series

Answer: D)
151). 3, 732, 1244, 1587, 1803, 1928, ?
A) 2144
B) 1999
C) 1995
D) 2053
E) None of these

152). 16, 24, ?, 210, 945, 5197.5, 33783.76


A) 40
B) 36
C) 58
D) 60
E) None of these

153). 5, 12, 36, 123, ?, 2555, 15342


A) 508

www.ibpsguide.com | estore.ibpsguide.com| www.sscexamguide.com


56
For Free Online Mock Test for IBPS PO/Clerk 2017 – Visit: estore.ibpsguide.com
500 Most Important Number Series Questions

B) 381
C) 504
D) 635
E) None of these

154). 8, 11, 17, ?, 65, 165.5, 498.5


A) 27.5
B) 32
C) 28
D) 30.5
E) None of these

155). 9, 15, 27, 51, 99, ?


A) 165
B) 195
C) 180
D) 190
E) None of these

156). 13, 21, 36, 58, 87, ?


A) 122
B) 128
C) 133
D) 123
E) None of these

157). 7, 9, 19, 45, 95, ?


A) 150
B) 160
C) 145
D) 177
E) None of these

158). 14, 15, 23, 32, 96, ?

www.ibpsguide.com | estore.ibpsguide.com| www.sscexamguide.com


57
For Free Online Mock Test for IBPS PO/Clerk 2017 – Visit: estore.ibpsguide.com
500 Most Important Number Series Questions

A) 121
B) 124
C) 152
D) 111
E) None of these

159). 20, 24, 36, 56, 84, ?


A) 116
B) 124
C) 120
D) 128
E) None of these

160). 117, 389, 525, 593, 627, ?


A) 654
B) 640
C) 6
D) 630
E) None of these
Solution:

151).

Answer: E)

152).

Answer: D)

www.ibpsguide.com | estore.ibpsguide.com| www.sscexamguide.com


58
For Free Online Mock Test for IBPS PO/Clerk 2017 – Visit: estore.ibpsguide.com
500 Most Important Number Series Questions

153). 5 × 1 + 1 × 7 = 12
12 × 2 + 2 × 6 = 36
36 × 3 + 3 × 5 = 123
123 × 4 + 4 × 4 = 508
508 × 5 + 5 × 3 = 2555
2555 × 6 + 6 × 2 = 15342
Answer: A)

154).8 ×0.5 + 7 = 11
11 × 1 + 6 = 17
17 × 1.5 + 5 = 30.5
30.5 × 2 + 4 = 65
65 × 2.5 + 3 = 165.5
165.5 × 3 + 2 = 498.5
Answer: D)

155).

Answer: B)

156).

Answer: D)

157). 7 + (1)2 + 1 = 9
9 + (3)2 + 1 = 19

www.ibpsguide.com | estore.ibpsguide.com| www.sscexamguide.com


59
For Free Online Mock Test for IBPS PO/Clerk 2017 – Visit: estore.ibpsguide.com
500 Most Important Number Series Questions

19 + (5)2 + 1 = 45
45 + (7)2 + 1 = 95
95 + (9)2 + 1 = 177
Answer: D)

158).

Answer: A)

159).

Answer: C)

160).

Answer: E)
Directions (Q.161-165): What will come in place of question mark(?)in the given number series?

161). 264 262 271 243 308 ?


A) 216
B) 163
C) 194

www.ibpsguide.com | estore.ibpsguide.com| www.sscexamguide.com


60
For Free Online Mock Test for IBPS PO/Clerk 2017 – Visit: estore.ibpsguide.com
500 Most Important Number Series Questions

D) 205
E) 182

162).1.5 2.5 7 24 100 ?


A) 345
B) 460
C) 525
D) 380
E) 505

163).71 78 92 120 ? 288


A) 160
B) 176
C) 199
D) 208
E) 164

164). 17 9 10 16.5 35 ?
A) 192
B) 80
C) 114
D) 90
E) 76

165).79 39 19 9 4 ?
A) 0.2
B) 1.5
C) 0.5
D) 2
E) 1

Directions (Q. 166-170): What will come in place of question mark (?)in the following number
series?
166). 13 13 19 43 103 ?

www.ibpsguide.com | estore.ibpsguide.com| www.sscexamguide.com


61
For Free Online Mock Test for IBPS PO/Clerk 2017 – Visit: estore.ibpsguide.com
500 Most Important Number Series Questions

A) 221
B) 227
C) 223
D) 217
E) 239

167). 27 13 12 16.5 ? 75
A) 31
B) 29
C) 37
D) 33
E) 35

168). 17 19 42 132 ? 2690


A) 532
B) 544
C) 528
D) 536
E) 512

169). 25 29 67 217 ? 4501


A) 927
B) 877
C) 885
D) 911
E) 893

170). 21 38 59 84 113 ?
A) 138
B) 152
C) 134
D) 146
E) 148

www.ibpsguide.com | estore.ibpsguide.com| www.sscexamguide.com


62
For Free Online Mock Test for IBPS PO/Clerk 2017 – Visit: estore.ibpsguide.com
500 Most Important Number Series Questions

Solution:

161). The series is -(13 + 1), +(23 + 1), -(33 + 1), +(43 + 1), -(53 + 1), …...
ie 264 – (13 + 1) = 262, 264 + (23 + 1) = 271
271 – (33 + 1) = 243, 243 + (43 + 1) = 308
308 – (53 + 1) = 182
Answer: E)

162). The series is ×1 + 1, ×2 + 2, ×3 + 3, ×4 + 4, ×5 + 5, ….


ie 1.5 ×1 + 1 = 2.5, 2.5 × 2 + 2 = 7,
7 × 3 + 3 = 24, 24 × 4 + 4 = 100,
100 × 5 + 5 = 505
Answer: E)

163). The series is +7, +14, +28, +56, +112,


ie 71 + 7 = 78, 78 + 14 = 92, 92 + 28 = 120,
120 + 56 = 176, 176 + 112 = 288,
Answer: B)

164). The series is ×0.5 + 0.5, ×1 + 1, ×1.5 + 1, ×2 + 2, ×2.5 + 2.5, …


ie 17 × 0.5 + 0.5 = 9, 9 × 1 + 1 = 10,
10 × 1.5 + 1.5 = 16.5, 16.5 × 2 + 2 = 35,
35 × 2.5 + 2.5 =90, …
Answer: D)

165). The series is /2 – 0.5, /2 -0.5, (repeateD)


ie 79 / 2 – 0.5 = 39, 39 / 2 – 0.5 = 19,
19 / 2 – 0.5 = 9, 9 / 2 – 0.5 = 4, 4 / 2 - 0.5 = 1.5, …
Answer: B)

166). The series is

www.ibpsguide.com | estore.ibpsguide.com| www.sscexamguide.com


63
For Free Online Mock Test for IBPS PO/Clerk 2017 – Visit: estore.ibpsguide.com
500 Most Important Number Series Questions

Answer: C)

167). The series is ×0.5 – 0.5, × 1 – 1,


× 1.5 – 1.5, × 2 – 2, × 2.5 – 2.5, …
ie 27 × 0.5 – 0.5 = 13, 13 × 1 – 1 = 12, 12 × 1.5 – 1.5 = 16 . 5,
16.5 × 2 – 2 = 31, 31 × 2.5 – 2.5 = 75,
Answer: A)

168). The series is × 1 + 2, ×2 + 4, ×3 + 6, × 4 + 8, × 5 + 10,


ie 17 × 1 + 2 = 19, 19 × 2 + 4 = 42, 42 × 3 + 6 = 132,
132 × 4 + 8 = 536, 536 ×5 + 10 = 2690, …
Answer: D)

169). The series is ×1 + 22, ×2 + 32, ×3 + 42, ×4 + 52, ×5 + 62, …


ie 25 × 1 + 22 = 29, 29 × 2 + 32 = 67, 67 × 3 + 42 = 217, 217 × 4 + 52 = 893,
893 × 5 + 62 = 4501,
Answer: E)

170). The series is+17, +21, +25, +29, +33, +37, ..


21+17 = 38, 38 + 21 = 59, 59 + 25 = 84, 84 + 29 = 113,
113 + 33 = 146, 146 + 37 = 183, …
Answer: D)
Directions (Questions 171 to 180): Find the missing number in the series given below.
171). 16, 22, 34, 58, 106, ?, 394
A) 178

www.ibpsguide.com | estore.ibpsguide.com| www.sscexamguide.com


64
For Free Online Mock Test for IBPS PO/Clerk 2017 – Visit: estore.ibpsguide.com
500 Most Important Number Series Questions

B) 175
C) 288
D) 202
E) 206

172). 10, 33, 102, 309, ?


A) 1030
B) 1050
C) 928
D) 930
E) 935

173). 121, 112, ?, 97, 91, 86


A) 102
B) 108
C) 99
D) 104
E) 106

174). 975, 864, 753, 642, ?


A) 431
B) 314
C) 531
D) 532
E) 542

175). 3, 4, 7, 11, 18, 29, ?


A) 31
B) 39
C) 43
D) 47
E) 32

176). 26, 37, 50, 65, ?, 101

www.ibpsguide.com | estore.ibpsguide.com| www.sscexamguide.com


65
For Free Online Mock Test for IBPS PO/Clerk 2017 – Visit: estore.ibpsguide.com
500 Most Important Number Series Questions

A) 77
B) 80
C) 81
D) 82
E) 83

177). 758, 753, 748, 744, 740, 736, ?


A) 732
B) 733
C) 734
D) 735
E) 736

178). 15, 17, 20, 22, 27, 29, ?, ?


A) 31, 38
B) 36, 38
C) 36, 43
D) 38, 45
E) 39, 46

179). 90, 61, 52, 63, 94, ?, 18


A) 72
B) 46
C) 54
D) 81
E) 82

180). 8, 24, 12, ?, 18, 54


A) 28
B) 36
C) 46
D) 38
E) 42

www.ibpsguide.com | estore.ibpsguide.com| www.sscexamguide.com


66
For Free Online Mock Test for IBPS PO/Clerk 2017 – Visit: estore.ibpsguide.com
500 Most Important Number Series Questions

Solutions:
171). Here, series moves with the addition of 6, 12, 24, 48, 96 and 192, respectively. Hence the
missing number is 202.
Answer is: D)

172). The pattern is as follows,

Therefore, ? = 930
Answer is: D)

173). Here, series is written in descending order with a difference of 9, 8, 7, 6 and 5, respectively.
Hence, the missing number will be 104.
Answer is: D)

174). The pattern is as follows

Therefore, ? = 531
Answer is: C)

175). Every third element is the sum of its previous two elements
3+4=7
4 + 7 + 11
7 + 11 = 18
11 + 18 = 29
18 + 29 = 47
Therefore, ? = 47
Answer is: D)

176). The pattern is as follows

www.ibpsguide.com | estore.ibpsguide.com| www.sscexamguide.com


67
For Free Online Mock Test for IBPS PO/Clerk 2017 – Visit: estore.ibpsguide.com
500 Most Important Number Series Questions

Therefore, ? = 82
Answer is: D)

177). Debuct the middle digit each time to obtain the next number
758 – 5 = 753
753 – 5 = 748
748 – 4 = 744
744 – 4 = 740
740 – 4 = 736
736 – 3 = 733
Therefore, ? = 733
Answer is: B)

178). The pattern is as follows

First ? = 36 and second ? = 38


Required answer = 36, 38
Answer is: B)

179). When the digit of the numbers are reversed these are the perfect squares 09, 16, 25, 36, 49,
64, 81 consecutive
Therefore, ? = Reverse of 64 = 46
Answer is: B)

180). The pattern is as follows

Therefore, ? = 36
Answer is: B)
181). 3, 6 18, 72, ?
A) 144
B) 216

www.ibpsguide.com | estore.ibpsguide.com| www.sscexamguide.com


68
For Free Online Mock Test for IBPS PO/Clerk 2017 – Visit: estore.ibpsguide.com
500 Most Important Number Series Questions

C) 288
D) 360
E) 152
182). 24, 60, 120, 210, ?
A) 300
B) 336
C) 420
D) 525
E) 250
183). 5, 16, 51, 158, ?
A) 1452
B) 483
C) 481
D) 1454
E) 1500
184). 4, ?, 144, 400, 900, 1764
A) 25
B) 36
C) 49
D) 100
E) 120
185). 8, 3, 11, 14, 25
A) 50
B) 39
C) 29
D) 11
E) 12
186). 980, 392, 156.8, ?, 25.088, 10.0352
A) 66.04
B) 61.28
C) 63.72
D) 64.85
E) None of these
187). 77, 59, 55, 35, 25, ?

www.ibpsguide.com | estore.ibpsguide.com| www.sscexamguide.com


69
For Free Online Mock Test for IBPS PO/Clerk 2017 – Visit: estore.ibpsguide.com
500 Most Important Number Series Questions

A) 24
B) 28
C) 20
D) 27
E) 29
188). 99, 82, 18, 11, ?
A) 5
B) 10
C) 2
D) 9
E) 8
189). 17, 36, 74, 150, ?, 606
A) 250
B) 303
C) 300
D) 302
E) 305
190). 51975, 9450, 2100, 600, 240, 160, ?
A) 80
B) 120
C) 320
D) 240
E) None of these

181).

182).

www.ibpsguide.com | estore.ibpsguide.com| www.sscexamguide.com


70
For Free Online Mock Test for IBPS PO/Clerk 2017 – Visit: estore.ibpsguide.com
500 Most Important Number Series Questions

183).

184).

185).

186). E)
Divided by 2.5 continuesly………
Answer: 62.72

187).

188).

www.ibpsguide.com | estore.ibpsguide.com| www.sscexamguide.com


71
For Free Online Mock Test for IBPS PO/Clerk 2017 – Visit: estore.ibpsguide.com
500 Most Important Number Series Questions

189).

190).

Directions (Q. 191-195): Find the missing number (P) in the following series
191). 126 158 174 P 186 188
A) 180
B) 182
C) 184
D) 178
E) None of these

192). 2.7 P 3.6 2.4 4.8 3.2


A) 1.5
B) 1.8
C) 2.1
D) 2.4
E) 2.5

www.ibpsguide.com | estore.ibpsguide.com| www.sscexamguide.com


72
For Free Online Mock Test for IBPS PO/Clerk 2017 – Visit: estore.ibpsguide.com
500 Most Important Number Series Questions

193). 15(2/3) 13(1/3) P 8(2/3) 6(1/3) 4


A) 8
B) 9
C) 10
D) 11
E) None of these

194). P 58 84 114 146 182 220


A) 28
B) 30
C) 32
D) 34
E) 36

195). 127 P 85 67 51 37 25
A) 105
B) 100
C) 95
D) 90
E) None of these

Directions (Q. 196-200): What should come in place question mark (?) in the following
number series?
196). 262 234 206 178 150 122 ?
A) 76
B) 78
C) 84
D) 89
E) 94
197). 4762 4627 4494 4363 4234 ?
A) 4147
B) 4137
C) 4127

www.ibpsguide.com | estore.ibpsguide.com| www.sscexamguide.com


73
For Free Online Mock Test for IBPS PO/Clerk 2017 – Visit: estore.ibpsguide.com
500 Most Important Number Series Questions

D) 4117
E) 4107

198). 672 560 448 336 224 ?


A) 172
B) 142
C) 132
D) 112
E) 102

199). 18 97 396 1197 2404 ?


A) 2816
B) 3215
C) 3612
D) 2415
E) 3600

200). 26 144 590 1164 ?


A) 1864
B) 1732
C) 1460
D) 1296
E) 1182

Solutions:

191. The logic of the sequence is:


+25, +24, +23, 22, +21
Answer: B)

192. The logic of the sequence is: ÷1.5, × 2 alternately


Answer: B)

193. The logic of the sequence is: -2(1/3)

www.ibpsguide.com | estore.ibpsguide.com| www.sscexamguide.com


74
For Free Online Mock Test for IBPS PO/Clerk 2017 – Visit: estore.ibpsguide.com
500 Most Important Number Series Questions

Answer: D)

194. The logic of the sequence is: +24, +26, +30, +32, +36, +38
Answer: D)

195. The logic of the sequence is -22, -20, -18, -16, -14, -12
Answer: A)

196. The series is


(29×9)+1=262
(29×8)+2=234
(29×7)+3=206
(29×6)+4=178
(29×5)+5=50
(29×4)+6=122
(29×3)+7=94
Answer: E)

197. The series is


(69)2+=4762
(68)2+3=4627
(67)2+5=4494
(66)2+7=4363
(65)2+9=4234
(64)2+11=4107
Answer: E)

198. The series is


666+6=672
555+5=560
444+4=448
333+3=336
222+2=224
111+1=112

www.ibpsguide.com | estore.ibpsguide.com| www.sscexamguide.com


75
For Free Online Mock Test for IBPS PO/Clerk 2017 – Visit: estore.ibpsguide.com
500 Most Important Number Series Questions

Answer: D)

199. The series is


2×6+6=18
18×5+7=97
97×4+8=396
396×3+9=1197
1197×2+10=2404
2404×1+11=2415
Answer: D)

200. The series is


2×8+10=26
26×6-12=144
144×4+14=590
590×2-16=1164
1164×1+18=1182
Answer: E)

Directions (Questions 201 to 210): What should come in place of question mark (?) in the
following number series?
201). 1 2 6 15 31 56 ?
A) 96
B) 94
C) 98
D) 92
E) 99

202). 41 39 43 37 45 ?
A) 55
B) 36
C) 33
D) 35
E) 49

www.ibpsguide.com | estore.ibpsguide.com| www.sscexamguide.com


76
For Free Online Mock Test for IBPS PO/Clerk 2017 – Visit: estore.ibpsguide.com
500 Most Important Number Series Questions

203). 3 8 23 68 203 ?
A) 607
B) 608
C) 604
D) 606
E) 610

204). 12 6 6 9 18 ?
A) 45
B) 54
C) 63
D) 36
E) 32

205). 9 15 27 45 69 99 ?
A) 137
B) 133
C) 135
D) 139
E) 149

206). 2 51 87 112 ? 137


A) 148
B) 121
C) 128
D) 118
E) None of these

207). 2 3 8 27 ? 565 3396


A) 34
B) 108
C) 110
D) 112

www.ibpsguide.com | estore.ibpsguide.com| www.sscexamguide.com


77
For Free Online Mock Test for IBPS PO/Clerk 2017 – Visit: estore.ibpsguide.com
500 Most Important Number Series Questions

E) 120

208). 8 5 ? 26.75 97.625 444.3125


A) 10.5
B) 9.5
C) 7.5
D) 12.5
E) 11.25

209). 27 125 ? 729 1331 2197


A) 216
B) 343
C) 512
D) 289
E) 256

210). 4 10 30 102 ? 2110


A) 610
B) 420
C) 121
D) 120
E) 510

Solutions:
201). The pattern of the number series is as given below:
1 + (1)2 = 2, 2 +(2)2= 6, 6 + (3)2 = 15, 15 + (4)2 = 31, 31 + (5)2 = 56, 56 + (6)2 = 56
+ 36 = 92
Answer is: D)

202). There are two alternating series of +2 and -2 pattern respectively.

www.ibpsguide.com | estore.ibpsguide.com| www.sscexamguide.com


78
For Free Online Mock Test for IBPS PO/Clerk 2017 – Visit: estore.ibpsguide.com
500 Most Important Number Series Questions

Answer is: D)
203). The difference gets multiplied by 3 successively.
The series is 3 + 5 = 8, 8 + 15 = 23,
23 + 45 = 68, 68 + 135 = 203, 203 + 405 = 608, ….
Answer is: B)
204). The series is 12 × 0.5 = 6, 6 × 1 = 6, 6 × 1.5 = 9, 9 × 2 = 18, 18 × 2.5 = 45, ….
Answer is: A)
205). The series is 9 + (1 × 6) = 15, 15 + (2 × 6) = 15 + 12 = 27, 27 + (3 × 6) = 27 + 18 = 45,
45 + (4 × 6) = 45 + 24 = 69, 69 + (5 × 6) = 69 + 30 = 99, 99 + (6 × 6) = 99 × 36 = 135, ….
Answer is: C)
206). The series is
+72, +62, +52, +42, +32, …..
Answer is: C)
207). The series is 2 × 1 + 1 = 3, 3 × 2 + 2 = 8, 8 × 3 + 3 = 27, 27 × 4 + 4 = 112, 112 × 5 + 5
= 565, 565 × 6 + 6 = 3396, ….
Answer is: D)
208). The series is 8 × 0.5 + 1 = 5, 5 × 1.5 + 2 = 9.5, 9.5 × 2.5 + 3 = 26.75, 26.75 × 3.5 + 4 =
97.625, 97.625 × 4.5 + 5 = 444.3125, ….
Answer is: B)
209). The series is (3)3, (5)3, (7)3, (9)3, (11)3, (13)3, ….
Answer is: B)
210). The series is 4 × 1 + 1 × 6 = 10
10 × 2 + 2 × 5 = 30
30 × 3 + 3 × 4 = 102
102 × 4 + 4 × 3 = 420
420 × 5 + 5 × 2 = 2110
610 × 6 + 6 × 1 = 3660 + 6 = 3666
Answer is: B)

www.ibpsguide.com | estore.ibpsguide.com| www.sscexamguide.com


79
For Free Online Mock Test for IBPS PO/Clerk 2017 – Visit: estore.ibpsguide.com
500 Most Important Number Series Questions

Directions (Q 211-220): What should come in place of questions mark (?) in the following
number series?

211). 7, 35, 105, 525, 1575, 7875, ?


A) 39375
B) 23625
C) 11815
D) 15750
E) None of these

212). 0, 5, 24, 75, 152, ?


A) 153
B) 304
C) 308
D) 312
E) None of these

213). 17, 19, 21, 22, 25, 25, 29, 28, ?


A) 31
B) 32
C) 33
D) 34
E) None of these

214). 16, 48, 24, 72, 36, 108, ?


A) 148
B) 196
C) 216
D) 224
E) None of these

215). 2/31, 5/17, 12.5/23, 31.25/19, 78.125/15, ?/11.


A) 156.25

www.ibpsguide.com | estore.ibpsguide.com| www.sscexamguide.com


80
For Free Online Mock Test for IBPS PO/Clerk 2017 – Visit: estore.ibpsguide.com
500 Most Important Number Series Questions

B) 156.3125
C) 234.375
D) 312.5
E) None of these

216). 11, 13, 17, 19, 23, 29, 31, 37, ?


A) 47
B) 43
C) 41
D) 39
E) None of these

217). 165, 195, 255, 285, 345, ?


A) 375
B) 420
C) 435
D) 390
E) None of these

218). 7, 26, 63, 124, 215, 342, ?


A) 481
B) 511
C) 391
D) 421
E) 451

219). 2, 4, 12, 48, 240, ?


A) 1920
B) 960
C) 1080
D) 1440
E) None of these

220). 8, 7, 11, 12, 14, 17, 17, 22, ?

www.ibpsguide.com | estore.ibpsguide.com| www.sscexamguide.com


81
For Free Online Mock Test for IBPS PO/Clerk 2017 – Visit: estore.ibpsguide.com
500 Most Important Number Series Questions

A) 20
B) 22
C) 24
D) 27
E) None of these

Solutions:
211).

212).

213).

214).

www.ibpsguide.com | estore.ibpsguide.com| www.sscexamguide.com


82
For Free Online Mock Test for IBPS PO/Clerk 2017 – Visit: estore.ibpsguide.com
500 Most Important Number Series Questions

215).

216). All are Prime Numbers


Answer: c

217).

www.ibpsguide.com | estore.ibpsguide.com| www.sscexamguide.com


83
For Free Online Mock Test for IBPS PO/Clerk 2017 – Visit: estore.ibpsguide.com
500 Most Important Number Series Questions

218).

219).

220). There are two series


8, 11, 14, 17, 20 and 7, 12, 17, 22
There for Next number is 20
Answer: a

Directions (221-230): What will come in place of question mark (?) in the following number
series?

www.ibpsguide.com | estore.ibpsguide.com| www.sscexamguide.com


84
For Free Online Mock Test for IBPS PO/Clerk 2017 – Visit: estore.ibpsguide.com
500 Most Important Number Series Questions

221). 13, 16, 22, 33, 51, ?


A) 89
B) 78
C) 102
D) 69
E) None of these

222). 39, 52, 78, 117, 169, ?


A) 246
B) 182
C) 234
D) 256
E) None of these

223). 656, 432, 320, 264, 236, ?


A) 222
B) 229
C) 232
D) 223
E) None of these

224). 62, 87, 187, 412, 812, ?


A) 1012
B) 1437
C) 1337
D) 1457
E) None of these

225). 7, 8, 24, 105, 361, ?


A) 986
B) 617
C) 486
D) 1657
E) None of these

www.ibpsguide.com | estore.ibpsguide.com| www.sscexamguide.com


85
For Free Online Mock Test for IBPS PO/Clerk 2017 – Visit: estore.ibpsguide.com
500 Most Important Number Series Questions

226). 9, 62, ?, 1854, 7415, 22244


A) 433
B) 309
C) 406
D) 371
E) None of these

227). 4, 8, 24, 60, ?, 224


A) 178
B) 96
C) 109
D) 141
E) None of these

228). 8000, 1600, 320, 64, 12.8, ?


A) 2.56
B) 3.5
C) 3.2
D) 2.98
E) None of these

229). 6, 9, 15, 27, 51, ?


A) 84
B) 99
C) 123
D) 75
E) None of these

230). 7, 8, 18, ?, 232, 1165


A) 84
B) 42
C) 57
D) 36

www.ibpsguide.com | estore.ibpsguide.com| www.sscexamguide.com


86
For Free Online Mock Test for IBPS PO/Clerk 2017 – Visit: estore.ibpsguide.com
500 Most Important Number Series Questions

E) None of these
Solutions:

221).

222).

223).

224).

225).

226).

www.ibpsguide.com | estore.ibpsguide.com| www.sscexamguide.com


87
For Free Online Mock Test for IBPS PO/Clerk 2017 – Visit: estore.ibpsguide.com
500 Most Important Number Series Questions

227).

228).

229).

230).

Direction (Q. 231–240): Find out the number in place of question mark (?) in the following
number series.

231). 529 841 961 1369 1681 1849 ?


A) 2809
B) 3249
C) 2208
D) 6424
E) 2209

232). 1108 1117 1142 1191 ? 1481


A) 1312

www.ibpsguide.com | estore.ibpsguide.com| www.sscexamguide.com


88
For Free Online Mock Test for IBPS PO/Clerk 2017 – Visit: estore.ibpsguide.com
500 Most Important Number Series Questions

B) 1272
C) 1300
D) 1204
E) None of these

233). 841 961 1089 1225 1369 1521 ?


A) 1785
B) 1581
C) 1681
D) 1881
E) 1781

234). 12 14 32 102 416 2090 ?


A) 15522
B) 12552
C) 13525
D) 17552
E) None of these

235). 384 381 372 345 264 ?


A) 25
B) 27
C) 44
D) 49
E) None of these

236). 5 41 321 1921 7681 15361 ?


A) 21381
B) 23710
C) 22173
D) 23121
E) 1

237). 1 3 7 13 22 34 51 71 96 124 ?

www.ibpsguide.com | estore.ibpsguide.com| www.sscexamguide.com


89
For Free Online Mock Test for IBPS PO/Clerk 2017 – Visit: estore.ibpsguide.com
500 Most Important Number Series Questions

A) 137
B) 147
C) 157
D) 167
E) 177

238). 664 334 85.5 16.25 4.03125 ?


A) 2.73012
B) 3.17203
C) 2.403125
D) 3.220175
E) 2.220175

239). 33 110 453 2282 13711 ?


A) 95673
B) 96747
C) 96312
D) 951321
E) 96000

240). 0.8 3.8 12.6 44.8 188.2 ?


A) 758.6
B) 868.8
C) 952.00
D) 1012.2
E) 1112.2

Solution:
231). The series is 232, 292, 312, 372, ….
Answer: E)

232). The series is +32, +52, +72, +112, +132,….


Answer: A)

www.ibpsguide.com | estore.ibpsguide.com| www.sscexamguide.com


90
For Free Online Mock Test for IBPS PO/Clerk 2017 – Visit: estore.ibpsguide.com
500 Most Important Number Series Questions

233). The series is +120, +128, +136, +…. +160, ….


Answer: C)

234). The series is ×1 + 2, ×2 + 4, ×3 + 6, ×4 + 8, ×5 + 10, ×6 + 12,…….


Answer: B)

235). The series is -3, -9, -27 ,-81 -243, ….


Answer: E)

236). The series is :


5 × 10 – 9 = 41
41 × 8 – 7 = 321
321 × 6 – 5 = 1921
1921 × 4 – 3 = 7681
7681 × 2 – 1 = 15361
15361 × 0 – (-1) = 1
Answer: E)

237). The series is :


1+1+1=3
3+2+2=7
7 + 3 + 3 = 13
13 + 4 + 5 = 22
22 + 5 + 7 = 34
34 + 6 + 11 = 51
51 + 7 + 13 = 71
71 + 8 + 17 = 96
96 + 9 + 19 = 124
124 + 10 + 23 = 157
First is number (1, 2, 3, 4…) etc and second is prime numbers (1, 2, 3, 5, 7, 11, 13…)
Answer: C)

238). The series is :


664 + 4 ÷ 2 = 334

www.ibpsguide.com | estore.ibpsguide.com| www.sscexamguide.com


91
For Free Online Mock Test for IBPS PO/Clerk 2017 – Visit: estore.ibpsguide.com
500 Most Important Number Series Questions

334 + 8 ÷ 4 = 85.5
85.5 + 12 ÷ 6 = 16.25
16.25 + 16 ÷ 8 = 4.03125
4.03125 + 20 ÷ 10 = 2.403125
Answer: C)

239). The series is :


13 × 2 + 7 = 33
33 × 3 + 11 = 110
110 × 4 + 13 = 453
453 × 5 + 17 = 2282
2282 × 6 + 19 = 13711
13711 × 7 + 23 = 96000
All are prime number ≥ 7.
Answer: E)

240). The series is :


0.8 × 1 + 3 = 3.8
3.8 × 2 + 5 = 12.6
12.6 × 3 + 7 = 44.8
44.8 × 4 + 9 = 188.2
188.2 × 5 + 11 = 952.00
Answer: C)

Directions (Q. 241-245): What should come in place of question mark (?) in the following
number series?

241). 19 42 88 180 364 ? 1468


A) 1046
B) 732
C) 472
D) 630
E) 595

www.ibpsguide.com | estore.ibpsguide.com| www.sscexamguide.com


92
For Free Online Mock Test for IBPS PO/Clerk 2017 – Visit: estore.ibpsguide.com
500 Most Important Number Series Questions

242). 51 57 102 324 ? 6390 38304


A) 1590
B) 1296
C) 1680
D) 1250
E) 1272

243). 1953.125 781.25 312.5 125 50 ?


A) 20
B) 25
C) 45
D) 15
E) 30

244). 4 11 32 74 144 ? 396


A) 289
B) 236
C) 205
D) 249
E) 196

245). 8 28 116 544 ? 13300


A) 3589
B) 5482
C) 2672
D) 7864
E) 9378

Directions (Q. 246-250): In each of the following questions, one number is missing in the
series. You have to understand the pattern of the series and then insert the missing
number.

246). 35 31 40 24 49 13 62 ?
A) 11

www.ibpsguide.com | estore.ibpsguide.com| www.sscexamguide.com


93
For Free Online Mock Test for IBPS PO/Clerk 2017 – Visit: estore.ibpsguide.com
500 Most Important Number Series Questions

B) -7
C) 18
D) -2
E) 5

247). 25 27.5 22.5 30 20 32.5 ?


A) 17.5
B) 7.5
C) 13.5
D) 22.5
E) 18.5

248). 11 12 26 81 328 ? 9876


A) 986
B) 1132
C) 1645
D) 1286
E) 1542

249). 1284 644 324 164 84 44 24 ?


A) 10
B) 19
C) 15
D) 9
E) 14

250). 4 5 18 81 388 ? 12606


A) 2933
B) 2468
C) 2065
D) 1979
E) 1732

Answers:

www.ibpsguide.com | estore.ibpsguide.com| www.sscexamguide.com


94
For Free Online Mock Test for IBPS PO/Clerk 2017 – Visit: estore.ibpsguide.com
500 Most Important Number Series Questions

1). B) 2).E) 3). A) 4). D) 5). C) 6). D) 7). A) 8). C) 9). E) 10). C)

Solution:

241) The series is +23, +46, +92, +184, +368, +736, …

Answer: b

242. The series is ×1 +6, ×2 -12, ×3 +18, ×4 -24, ×5 + 30, ×6 -36, ….


ie 51 × 1 + 6 = 57, 57×2-12 = 102, 102×3+18 = 324, 324×4-24 = 1272, 1272×5+30 = 6390,
6390×6-36 = 38304, …..
Answer: e

243. The series is ÷2.5, ÷2.5 (repeateD)

Answer: a

244. The series is +7 ×1, +7 ×3, +7 ×6, +7 ×10, +7 ×15, +7 ×21,


ie 4+7×1 = 11, 11+7×3 = 32
32+7×6 = 74, 74+7×10 =144,
144+7×15 = 249, 249+7× 21= 396, ….
Answer: d

245. The series is 8×5 – 12 = 28,


28 × 5 -24 = 116, 116 × 5 - 36 = 544,
544 × 5 - 48 = 2672, 2672 × 5 – 60 = 13300
Answer: c

www.ibpsguide.com | estore.ibpsguide.com| www.sscexamguide.com


95
For Free Online Mock Test for IBPS PO/Clerk 2017 – Visit: estore.ibpsguide.com
500 Most Important Number Series Questions

246. The series is –(2)2, + (3)², -(4)², +(5)², …

Answer: d

247. The series is +(5/2), -(10/2), (15/2), -(20/2) …

Answer: a

248. The series is ×1 + 1, ×2 +2, ×3 + 3, ×4 + 4, ….

Answer: c

249. The series is ÷2 + 2, ÷2 + 2, (repeateD)

Answer: e

250. The series is ×1 +1³, ×2 + 2³, ×3 + 3³, …

Answer: c

www.ibpsguide.com | estore.ibpsguide.com| www.sscexamguide.com


96
For Free Online Mock Test for IBPS PO/Clerk 2017 – Visit: estore.ibpsguide.com
500 Most Important Number Series Questions

Directions (251-26): What should come in place question mark (?) in the following number
series?

251).18 99 682 5443 ?


A) 48972
B) 48963
C) 48957
D) 48948
E) 48682

252). 1324 1721 2190 2737 3368 ?


A) 3986
B) 4089
C) 4136
D) 4290
E) 4068

253). 363 234 141 78 39 18 ?


A) 12
B) 11
C) 10
D) 9
E) 8

254). 665 463 307 191 109 ?


A) 79
B) 69
C) 55
D) 51
E) 47

255). 210 336 504 720 990 1320 ?


A) 1651
B) 1688

www.ibpsguide.com | estore.ibpsguide.com| www.sscexamguide.com


97
For Free Online Mock Test for IBPS PO/Clerk 2017 – Visit: estore.ibpsguide.com
500 Most Important Number Series Questions

C) 1716
D) 1794
E) 1720

256). 3 81 ? 1029 2187 3993


A) 375
B) 648
C) 192
D) 575
E) 243

257). 30 45 75 105 165 ?


A) 185
B) 205
C) 215
D) 195
E) 230

258). 8 24 12 36 18 54 ?
A) 64
B) 79
C) 34
D) 37
E) 27

259). 4320 720 144 ? 12 6 6


A) 56
B) 60
C) 26
D) 36
E) 16

260). 26 63 124 215 342 ?


A) 511

www.ibpsguide.com | estore.ibpsguide.com| www.sscexamguide.com


98
For Free Online Mock Test for IBPS PO/Clerk 2017 – Visit: estore.ibpsguide.com
500 Most Important Number Series Questions

B) 509
C) 504
D) 515
E) 525

Answers:
1). A) 2).b ) 3).d ) 4).c ) 5). C) 6). A) 7).d ) 8).e ) 9). D) 10). A)

Solution:

251). The series is:


18 × 6 – 9 = 99
99 × 7 – 11 = 682
682 × 8 – 13 = 5443
5443 × 9 – 15 = 48972
Answer: A)

252). The series is:


(11)3 – 7 = 1324
(12)3 – 7 = 1721
(13)3 – 7 = 2190
(14)3 – 7 = 2737
(15)3 – 7 = 3368
(16)3 – 7 = 4089
Answer: B)

253). The series is:


13 + 7 + 73 = 363
12 + 6 + 63 = 234
11 + 5 + 53 = 141
10 + 4 + 43 = 78
9 + 3 + 33 = 39
8 + 2 + 23 = 18
7 + 1 + 13 = 9

www.ibpsguide.com | estore.ibpsguide.com| www.sscexamguide.com


99
For Free Online Mock Test for IBPS PO/Clerk 2017 – Visit: estore.ibpsguide.com
500 Most Important Number Series Questions

Answer: D)

254). The series is:


93 – 82 = 665
83 – 72 = 463
73 – 62 = 307
63 – 52 = 191
53 – 42 = 109
43 – 32 = 55
Answer: C)

255). The series is:


5 × 6 × 7 = 210
6 × 7 × 8 = 336
7 × 8 × 9 = 504
8 × 9 × 10 = 720
9 × 10 × 11 = 990
10 × 11 × 12 = 1320
11 × 12 × 13 = 1716
Answer: C)

256). The series is 13 × 3, 33 × 3, 53 × 3, 73 × 3, ….


Answer: A)

257). Each number is a prime number multiplied by 15.


Thus, the series is 15 × 2, 15 × 3, 15 × 5, 15 × 7, 15 × 11, ….
Answer: D)

258). The series is ×3, ÷2, ×3, ÷2,…..


Answer: E)

259). The series is ÷6, ÷5, ÷4, ÷3 ….


Answer: D)

www.ibpsguide.com | estore.ibpsguide.com| www.sscexamguide.com


100
For Free Online Mock Test for IBPS PO/Clerk 2017 – Visit: estore.ibpsguide.com
500 Most Important Number Series Questions

260). The series is (33 – 1), (43 – 1), (53 – 1), (63 – 1), (73 – 1)
Answer: A)
Directions (261-270): What value should come in place of question mark (?) in the following
number series?

261). 796 199 50 13 4 ?


A) 4
B) 1
C) 5
D) 2
E) 3

262). 445 534 631 736 849 ?


A) 961
B) 965
C) 970
D) 958
E) 968

263). 7 9.21 16.05 29.94 53.3 ?


A) 88.55
B) 90.58
C) 92.48
D) 91.68
E) 94.78

264). 6 10 28 68 138 ?
A) 246
B) 236
C) 238
D) 232
E) 242

265). 16 9 6 5 ? 5.5

www.ibpsguide.com | estore.ibpsguide.com| www.sscexamguide.com


101
For Free Online Mock Test for IBPS PO/Clerk 2017 – Visit: estore.ibpsguide.com
500 Most Important Number Series Questions

A) 5
B) 4
C) 3
D) 1
E) 2

266). 24 12 ? 18 36 90
A) 15
B) 12
C) 10.5
D) 11
E) 18

267). 7 20 59 ? 215 332


A) 120
B) 155
C) 165
D) 124
E) None of these

268). ? 17 29 59 115 205


A) 14
B) 16
C) 15
D) 13
E) 11

269). 10 30 68 130 ? 350


A) 225
B) 250
C) 242
D) 222
E) None of these

www.ibpsguide.com | estore.ibpsguide.com| www.sscexamguide.com


102
For Free Online Mock Test for IBPS PO/Clerk 2017 – Visit: estore.ibpsguide.com
500 Most Important Number Series Questions

270). 170 196 248 ? 560 976


A) 350
B) 352
C) 452
D) 380
E) None of these

Answer:
1) D) 2) C) 3) A) 4) A) 5) A) 6) B) 7) D) 8) C) 9) D) 10) B)

Detailed Explanation:

261). The series is ÷ 4, + 1 ÷4, + 2 ÷ 4, + 3 ÷4,…

:. ? = (4 + 4) ÷ 4 = 2
Answer: D)

262). The series is (21)2 + 4, (23)2 + 5, (25)2 + 6, (27)2 + 7,….


:. ? = (31)2 + 9 = 961 + 9 =970
Answer: C)

263). The series is +1 + (1.1)2, + 2 + (2.2)2, + 3 + (3.3)2,…

:. ? = 53.3 + 5 + (5.5)2 = 88.55


Answer: A)

264). The series is + 32 – 5, + 52 – 7, + 72 – 9, + 92 – 11, ….

www.ibpsguide.com | estore.ibpsguide.com| www.sscexamguide.com


103
For Free Online Mock Test for IBPS PO/Clerk 2017 – Visit: estore.ibpsguide.com
500 Most Important Number Series Questions

:. ? = 138 + 121 – 13 =246


Answer: A)

265). The series is + 2 ÷ 2, + 3 ÷ 2, + 4 ÷ 2, + 5 ÷ 2,…..


:. ? = (5 + 5) * (1 / 2) = 5
Answer: A)

266). The series is × 0.5, ×1, × 1.5, × 2, × 2.5, ….


Answer: B)

267). The series is + (1 × 13), + (3 × 13), + (5 × 13), + (7 × 13),….


Answer: D)

268). The series is 15 + 1 × 2 = 17, 17 + 3 × 4 =29, 29 + 5 × 6 = 59, 59 + 7 × 8 = 115, 115 + 9


× 10 = 205, ….
Answer: C)

269). The series is 23 + 2, 33 + 3 , 43 + 4 , 53 + 5, 63 + 6 , 73 + 7,…..


Answer: D)

270). The series is + 26, + 52, + 104, + 208, + 416,…


Answer: B)
Directions (271-275): In each of the following questions, one number is missing in the
series. You have to understand the pattern of the series and then insert the missing number
in place of question mark (?).

271). 5 10 30 105 ? 2225


A) 860
B) 440
C) 1250
D) 320
E) 1575

272). 2 3 5 15 65 255 1265 ?

www.ibpsguide.com | estore.ibpsguide.com| www.sscexamguide.com


104
For Free Online Mock Test for IBPS PO/Clerk 2017 – Visit: estore.ibpsguide.com
500 Most Important Number Series Questions

A) 4295
B) 3405
C) 2560
D) 3185
E) 5055

273). 523 507 471 407 307 163 ?


A) 105
B) 29
C) -33
D) -17
E) None of these

274). 276 140 68 36 16 ? 3


A) 10
B) 15
C) 5
D) 7
E) 11

275). 105 185 247 293 ? 345 355


A) 225
B) 286
C) 315
D) 325
E) 338

Directions ( 276-280): What should come in place of question mark in the following
questions ?

276). 7 10 26 87 360 1815 ?


A) 13256
B) 12864
C) 10908

www.ibpsguide.com | estore.ibpsguide.com| www.sscexamguide.com


105
For Free Online Mock Test for IBPS PO/Clerk 2017 – Visit: estore.ibpsguide.com
500 Most Important Number Series Questions

D) 15069
E) 20185

277). 11 5 -2 -24 -120 ? -3816


A) -630
B) -600
C) -675
D) -720
E) -913

278). 4 6 18 81 486 ? 32805


A) 2562
B) 3218
C) 3985
D) 3645
E) 4286

279). 4 10 33 136 ? 4116 28819


A) 829
B) 972
C) 730
D) 523
E) 685

280). 175 174 173 169 161 152 ? 109


A) 108
B) 119
C) 125
D) 128
E) 135

Detailed Explanation:

271). The series is × 1 + 5, × 2 + 10, × 3 + 15, × 4 + 20,……

www.ibpsguide.com | estore.ibpsguide.com| www.sscexamguide.com


106
For Free Online Mock Test for IBPS PO/Clerk 2017 – Visit: estore.ibpsguide.com
500 Most Important Number Series Questions

Answer: B)

272). The series is × 4 – 5, × 5 – 10, × 4 – 5, × 5 – 10,

Answer: E)

273). The series is – (4)2, - (6)2, - (8)2,…

Answer: C)

274). The series is ÷ 2 + 2, ÷ 2 – 2, ÷ 2 + 2, ÷ 2 – 2,

Answer: A)

275). The series is + 92 - 1, + 82 – 2, + 72 – 3,

Answer: D)

276). The series is × 1 + 3, × 2 + 6, × 3 + 9, × 4 + 12, × 5 + 15,…

Answer: C)

277). The series is × 1 – 6, × 2 – 12, × 3 – 18,

www.ibpsguide.com | estore.ibpsguide.com| www.sscexamguide.com


107
For Free Online Mock Test for IBPS PO/Clerk 2017 – Visit: estore.ibpsguide.com
500 Most Important Number Series Questions

Answer: A)

278). The series is × 1.5, × 3 , × 4.5,..

Answer: D)

279). The series is × 2 + 2, × 3 + 3, × 4 + 4, × 5 + 5, × 6 + 6,….


Ie 4 × 2 + 2 =10, 10 × 3 + 3 =33,
33 × 4 + 4 = 136, 136 × 5 + 5= 685,
685 × 6 + 6 = 4116, 4116 × 7 + 7 = 28819
Answer: E)

280). The series is – (1)2 , - (1)3, - (2)2, -(2)3, -(3)2, -(3)3,….

Answer: C)
Directions (281-290): What will come in place of question mark (?) in the given number
series?

281) 8 11 20 47 128 ?
A) 372
B) 371
C) 308
D) 416
E) 364

282).13 27 44 ? 87 113
A) 64
B) 63
C) 62
D) 81
E) 69

www.ibpsguide.com | estore.ibpsguide.com| www.sscexamguide.com


108
For Free Online Mock Test for IBPS PO/Clerk 2017 – Visit: estore.ibpsguide.com
500 Most Important Number Series Questions

283). 2 11 38 119 362 ?


A) 1052
B) 1091
C) 1152
D) 1061
E) 1291

284).17 19 41 127 ? 2571


A) 508
B) 506
C) 513
D) 416
E) 516

285).108 113 103 ? 98 123


A) 116
B) 109
C) 115
D) 118
E) 123

286). 511 733 1177 ? 2731 3841


A) 1840
B) 1843
C) 1748
D) 1943
E) 2040

287). 3.5 8 36 225 1816 ?


A) 12736
B) 18165
C) 12737
D) 14737
E) 18185

www.ibpsguide.com | estore.ibpsguide.com| www.sscexamguide.com


109
For Free Online Mock Test for IBPS PO/Clerk 2017 – Visit: estore.ibpsguide.com
500 Most Important Number Series Questions

288). 7 21 105 735 ? 72765


A) 6315
B) 6625
C) 6605
D) 6615
E) 6251

289). 63 64 132 405 ? 8205


A) 1620
B) 1646
C) 1656
D) 1426
E) 1636

290). 79 88 106 142 214 ?


A) 258
B) 352
C) 358
D) 458
E) 268

Solution Hint:

281).The series is +3, +9, +27, +81, +243, ….


Answer: B)

282).The series is +14, +17, +20, +23, +26, +29, …


Answer: A)

283).The series is +32, +33, +34 , +35, +36, …


Answer: B)

284).The series is ×1 +2, ×2+3, ×3+4, ×4+5, ×5+6, …

www.ibpsguide.com | estore.ibpsguide.com| www.sscexamguide.com


110
For Free Online Mock Test for IBPS PO/Clerk 2017 – Visit: estore.ibpsguide.com
500 Most Important Number Series Questions

Answer: C)

285).The series is +5, -10, +15, -20, +25, …


Answer: D)

286).The series is +222×1, +222×2, +222×3, +222×4, …


∴?= 1177+666=1843
Answer: B)

287). The series is ×2+1², ×4+2², ×6+3², ×8+4², ×10+5², …


Answer: E)

288).The series is ×3, ×5, ×7, ×9, ×11, …


Answer: D)

289).The series is ×1+1², ×2+2², ×3+3², ×4+4², ×5+5², …


Answer: E)

290). The series is +9, +18, +36, +72, +144, …


Answer: C)
Directions (291-300): What value should come in place of question mark (?) in the following
questions?

291). 5 3 3 5 15 ?
A) 55
B) 5
C) 69
D) 21
E) 3

292). 18 19 42 135 556 ?


A) 2910
B) 2805
C) 2530

www.ibpsguide.com | estore.ibpsguide.com| www.sscexamguide.com


111
For Free Online Mock Test for IBPS PO/Clerk 2017 – Visit: estore.ibpsguide.com
500 Most Important Number Series Questions

D) 2790
E) 2525

293). 2 4 ? 18 64 330
A) 12
B) 20
C) 28
D) 10
E) 4

294). 7 24 38 86 162 ?
A) 259
B) 269
C) 324
D) 334
E) 314

295). 12 24 96 576 4608 ?


A) 19536
B) 46080
C) 40680
D) 27648
E) 34560

296). 9 17 65 385 3073 ?


A) 40704
B) 73251
C) 65506
D) 38521
E) 30721

297). 25 241 584 1096 1825 ?


A) 2625
B) 2525

www.ibpsguide.com | estore.ibpsguide.com| www.sscexamguide.com


112
For Free Online Mock Test for IBPS PO/Clerk 2017 – Visit: estore.ibpsguide.com
500 Most Important Number Series Questions

C) 2725
D) 2825
E) 2025

298). 50 25 37.5 93.75 328.125 ?


A) 1656.6525
B) 1476.5625
C) 1576.6225
D) 2025.5625
E) None of these

299). 468 516 984 1500 2484 ?


A) 3984
B) 4884
C) 2784
D) 2824
E) 3874

300). 11 30 87 258 771 ?


A) 2610
B) 2450
C) 2310
D) 2730
E) 2510

Solutions:

291). The series is ×1-2, ×2-3, ×3-4, ×4-5, ×5-6, …


Answer: C)

292). The series is ×1+1², ×2+2², ×3+3², ×4+4², ×5+5², …


Answer: B)

293). The series is ×1+2, ×2-4, ×3+6, ×4-8, ×5+10, …

www.ibpsguide.com | estore.ibpsguide.com| www.sscexamguide.com


113
For Free Online Mock Test for IBPS PO/Clerk 2017 – Visit: estore.ibpsguide.com
500 Most Important Number Series Questions

Answer: E)

294). ×2+10, ×2-10, ×2+10, ×2-10, …. Repeated


Answer: D)

295).12×2=24, 24×4=96, 96×6=576, 576×8=4608, 4608×10=46080


Answer: B)

296). The series is ×2-1, ×4-3, ×6-5, ×8-7, ×10-9, …


Answer: E)

297).The series is +63, +73, +83, +93, +103, …


Answer: D)

298). The series is


50×0.5=25,
25×1.5=37.5,
37.5×2.5=93.75,
93.75×3.5=328.125,
328.125×4.5=1476.5625
Answer: B)

299). 468+516=984;
516+984=1500;
984+1500=2484;
1500+2484=3984
Answer: A)

300). The series is


11×3-3=30,
30×3-3=87,
87×3-3=258,
258×3-3=771,
771×3-3=2310

www.ibpsguide.com | estore.ibpsguide.com| www.sscexamguide.com


114
For Free Online Mock Test for IBPS PO/Clerk 2017 – Visit: estore.ibpsguide.com
500 Most Important Number Series Questions

Answer: C)
Directions (301-310): What should come in the place of question mark in the following
number series?

301). 1224 ? 3022 4221 5620 7219


A) 2165
B) 1986
C) 1976
D) 1875
E) 2023

302). ? 208 501 986 1711 2724


A) 59
B) 63
C) 78
D) 54
E) 67

303). ? 4 17 126 1297 16808


A) 1
B) 2
C) 0
D) -3
E) 3

304). 11 29 ? 89 131 181


A) 59
B) 55
C) 76
D) 64
E) 68

305). 974 1366 1754 2146 2534 ?

www.ibpsguide.com | estore.ibpsguide.com| www.sscexamguide.com


115
For Free Online Mock Test for IBPS PO/Clerk 2017 – Visit: estore.ibpsguide.com
500 Most Important Number Series Questions

A) 2926
B) 2929
C) 2976
D) 2956
E) 2946

306). 8835 9023 9213 9405 ? 9795


A) 9899
B) 9599
C) 9539
D) 9509
E) None of these

307). 5477 5626 5777 ? 6085 6242 6401


A) 5830
B) 5960
C) 5840
D) 5950
E) 5930

308). 1716 2184 2730 3360 4080 ? 5814


A) 4896
B) 4876
C) 4796
D) 4696
E) 5096

309). 137 247 411 548 ? 822 959


A) 785
B) 635
C) 735
D) 685
E) 695

www.ibpsguide.com | estore.ibpsguide.com| www.sscexamguide.com


116
For Free Online Mock Test for IBPS PO/Clerk 2017 – Visit: estore.ibpsguide.com
500 Most Important Number Series Questions

310). 8 12 18 27 40.5 60.75 ?


A) 81.125
B) 92.125
C) 91.125
D) 94.125
E) 87.275

Solutions:
301). The series is
(35)2 -1, (45)2 -2, (55)2 -3, (65)2 -4, (75)2 -5, …
So the series is 1224, 2023, 3022, 4221, 5620, 7219, ...
Answer: E)

302). The series is 3*4*5-1, 5*6*7-2, 7*8*9-3, 9*10*11-4, 11*12*13-5, 13*14*15-6, …..
Answer: A)

303). The series is (2)0+1, (3)1+1, (4)2+1, (5)3+1, (6)4+1, (7)5+1, …


Answer: B)

304). The series is 3*5-4, 5*7-6, 7*9-8, 9*11-10, 11*13-12, 13*15-14, …


Answer: B)

305). The series is 195*5-1, 195*7+1, 195*9-1, 195*11+1, 195*13-1, 195*15+1, …


Answer: A)

306). The series is (94)2 -1, (95)2 -2, (96)2 -3, (97)2 -4, (98)2 -5, (99)2 -6,
ie, 8835, 9023, 9213, 9405, 9599, 9795
Answer: B)

307). The series is +149, +151, +153, +155, +157, +159, +161, …
Answer: E)

308). The series is (12)3 -12, (13)3-13, (14)3-14, (15)3-15, (16)3-16, (17)3-17, (18)3-18, (19)3-19, …
Answer: A)

www.ibpsguide.com | estore.ibpsguide.com| www.sscexamguide.com


117
For Free Online Mock Test for IBPS PO/Clerk 2017 – Visit: estore.ibpsguide.com
500 Most Important Number Series Questions

309). The series is +137, +137, repeated.

Answer: D)

310).c; The series is ×1.5, ×1.5, ×1.5……..


Answer: C)
311). 2 12 36 80 150 ?
A) 194
B) 210
C) 252
D) 258

312). 1, 8, 9, 64, 25, 216, ?, ?


A) 49, 64
B) 343, 64
C) 49, 512
D) 343, 512

313). 3, 13, 53, 213, ?


A) 553
B) 653
C) 753
D) 853

314). 4, 16, 36, ?, 100, 144


A) 72
B) 68
C) 81
D) 64

315). 8, 15, 28, 53, ...?


A) 98
B) 106

www.ibpsguide.com | estore.ibpsguide.com| www.sscexamguide.com


118
For Free Online Mock Test for IBPS PO/Clerk 2017 – Visit: estore.ibpsguide.com
500 Most Important Number Series Questions

C) 100
D) 102

316). 6, 12, 21, ?, 48


A) 38
B) 40
C) 45
D) 33

317). 1, 4, 27, 16, ?, 36, 343


A) 87
B) 120
C) 25
D) 125

318). 28, 33, 31, 36, ?, 39


A) 32
B) 34
C) 38
D) 40

319). 165, 195, 255, 285, 345, ?


A) 375
B) 390
C) 420
D) 435

320). 1, 5, 14, 30, 55, 91, ?


A) 130
B) 140
C) 150
D) 160

Solutions for the above aptitude Questions:

www.ibpsguide.com | estore.ibpsguide.com| www.sscexamguide.com


119
For Free Online Mock Test for IBPS PO/Clerk 2017 – Visit: estore.ibpsguide.com
500 Most Important Number Series Questions

311). 13 + 12 = 2, 23 + 22 = 12, 33 + 32 = 36 and so on 63 + 62 = 252


Answer: C)
312). The odd terms are squares of the numbers 1, 3, 5, ....while the even terms are cubes of the
numbers 2, 4, 6, ....
Answer: C)
313). The difference in consecutive nos. follows the sequence, 10, 40, 160.
So the next difference = 160 × 4 = 640
Therefore number = 213 + 640 = 853.
Answer: D)
314). The numbers in the series are square of 2,4,6 ...,10,12 .
Therefore, the missing number is square of 8 i.e. 82=64.
Answer: D)
315). 8 × 2 – 1 = 15, 15 × 2 – 2 = 28, 28 × 2 – 3 = 53, 53 × 2 – 4 =102
Answer: D)
316). The common differences between the consecutive terms of the series are 6, 9, 12, 15, etc.
Answer: D)
317). The series is 13, 22, 33, 42, 53, 62, 73.
Answer: D)
318). The pattern is +5, –2, +5, –2, .....
So, missing term = 36 –2 = 34.
Answer: B)
319). Each number of the series is 15 multiplied by a prime number
i.e. 15 × 11, 15 × 13, 15 × 17, 15 × 19, 15 × 23, .....
So, missing term = 15 × 29 = 435.
Answer: D)
320). The pattern is + 4, +9, + 16, + 25, + 36, ..... i.e. + 2 2, + 32, + 42, + 52, + 62, .....
So missing term = 91 + 72 = 91 + 49 = 140.
Answer: B)

Q(321-6-325) Find the missing number in the given seriese.

321) 2, 7, 10, 22, 18, 37, 26,?


A. 42

www.ibpsguide.com | estore.ibpsguide.com| www.sscexamguide.com


120
For Free Online Mock Test for IBPS PO/Clerk 2017 – Visit: estore.ibpsguide.com
500 Most Important Number Series Questions

B. 52
C. 46
D. 62
E.None of these

322) 279936, 46656, 7776, 1296, 216, ?


A. 60
B. 46
C. 36
D. 66
E.None of these

323)12, 38, 116, 350, 1052, ?


A. 1800
B. 2200
C. 2800
D. 3158
E.None of these

324) 46080, 3840, 384, 48, 8, 2, ?


A. 1
B. 1/64
C. 1/8
D.2
E. None of these

325)5,28,57,88,125
A.156
B.147
C.166
D.186
E. None of these

www.ibpsguide.com | estore.ibpsguide.com| www.sscexamguide.com


121
For Free Online Mock Test for IBPS PO/Clerk 2017 – Visit: estore.ibpsguide.com
500 Most Important Number Series Questions

321.B
There are two series here
2, 10, 18, 26, ... (Increase by 8)
7, 22, 37, ... (Increase by 15)
Hence, next term is 37+15 = 52

322.C
Go on dividing by 6 to the next number

323.D
12
12 × 3 + 2 = 38
38 × 3 + 2 = 116
116 × 3 + 2 = 350
350 × 3 + 2 = 1052
1052 × 3 + 2 = 3158

324.A
46080 /12 = 3840
3840 /10 = 384
384 /8 = 48
48 /6 = 8
8/4 = 2
2 /2 = 1

325.C
28=23+5
57=29+28
88=31+57
125=37+88
166=41+125

326) 42, 40, 38, 35, 33, 31, 28, ?, ?


A. 25, 22

www.ibpsguide.com | estore.ibpsguide.com| www.sscexamguide.com


122
For Free Online Mock Test for IBPS PO/Clerk 2017 – Visit: estore.ibpsguide.com
500 Most Important Number Series Questions

B. 26, 23
C. 26, 24
D. 25, 23
E. 26, 22

327) 8, 12, 9, 13, 10, 14, 11, ?, ?


A. 14, 11
B. 15, 12
C. 8, 15
D. 15, 19
E. 8, 5

328) 36, 31, 29, 24, 22, 17, 15, ?, ?


A. 13, 11
B. 10, 5
C. 13, 8
D. 12, 7
E. 10, 8

329) 3, 5, 35, 10, 12, 35, 17, ?, ?


A. 22, 35
B. 35, 19
C. 19, 35
D. 19, 24
E. 22, 24

330) 13, 29, 15, 26, 17, 23, 19, ?, ?


A. 21, 23
B. 20, 21
C. 20, 17
D. 25, 27
E. 22, 20

331) 14, 14, 26, 26, 38, 38, 50, ?, ?

www.ibpsguide.com | estore.ibpsguide.com| www.sscexamguide.com


123
For Free Online Mock Test for IBPS PO/Clerk 2017 – Visit: estore.ibpsguide.com
500 Most Important Number Series Questions

A. 60, 72
B. 50, 62
C. 50, 72
D. 62, 62
E. 62, 80

332) 32, 31, 32, 29, 32, 27, 32, ?, ?


A. 25, 32
B. 31, 32
C. 29, 32
D. 25, 30
E. 29, 30

333) 83, 73, 93, 63, __, 93, 43,


A. 33
B. 53
C. 73
D. 93

334) 15, __, 27, 27, 39, 39


A. 51
B. 39
C. 23
D. 15

335) 72, 76, 73, 77, 74, __, 75


A. 70
B. 71
C. 75
D. 78

336) F2, __, D8, C16, B32


A. A16
B. G4

www.ibpsguide.com | estore.ibpsguide.com| www.sscexamguide.com


124
For Free Online Mock Test for IBPS PO/Clerk 2017 – Visit: estore.ibpsguide.com
500 Most Important Number Series Questions

C. E4
D. E3

337) 664, 332, 340, 170, ____, 89


A. 85
B. 97
C. 109
D. 178

338) 70, 71, 76, __, 81, 86, 70, 91


A. 70
B. 71
C. 80
D. 96

339) 8, 43, 11, 41, __, 39, 17


A. 8
B. 14
C. 43
D. 44

340) 10, 6, 12, 35, ?, 591.75


(A) 130
(B) 129.5
(C) 127.25
(D) 133
(E) None of these

326) Option C
Explanation: This is an alternating subtraction series in which 2 is subtracted twice, then 3 is
subtracted once, then 2 is subtracted twice, and so on.

327) Option B
Explanation: This is an alternating addition and subtraction series, in which the addition of 4 is

www.ibpsguide.com | estore.ibpsguide.com| www.sscexamguide.com


125
For Free Online Mock Test for IBPS PO/Clerk 2017 – Visit: estore.ibpsguide.com
500 Most Important Number Series Questions

alternated with the subtraction of 3.

328) Option E
Explanation: This is an alternating subtraction series, which subtracts 5, then 2, then 5, and so
on.

329) Option C
Explanation: This is an alternating addition series, with a random number, 35, interpolated as
every third number. The pattern of addition is to add 2, add 5, add 2, and so on. The number 35
comes after each "add 2" step.

330) Option B
Explanation: Here, there are two alternating patterns, with every other number following a
different pattern. The first pattern begins with 13 and adds 2 to each number to arrive at the
next; the alternating pattern begins with 29 and subtracts 3 each time.

331) Option B
Explanation: In this simple addition with repetition series, each number in the series repeats
itself, and then increases by 12 to arrive at the next number.

332)OptionA
Explanation: This is an alternating repetition series. The number 32 alternates with a series in
which each number decreases by 2.

333) Option B
Explanation: This is a simple subtraction series in which a random number, 93, is interpolated as
every third number. In the subtraction series, 10 is subtracted from each number to arrive at the
next.

334) Option D
Explanation: In this simple addition with repetition series, each number in the series repeats
itself, and then increases by 12 to arrive at the next number.

335) Option D

www.ibpsguide.com | estore.ibpsguide.com| www.sscexamguide.com


126
For Free Online Mock Test for IBPS PO/Clerk 2017 – Visit: estore.ibpsguide.com
500 Most Important Number Series Questions

Explanation: This series alternates the addition of 4 with the subtraction of 3.

336) Option C
Explanation: The letters decrease by 1; the numbers are multiplied by 2.

337) Option D
Explanation: This is an alternating division and addition series: First, divide by 2, and then add 8.

338) Option A
Explanation: In this series, 5 is added to the previous number; the number 70 is inserted as every
third number.

339) Option B
Explanation: This is a simple alternating addition and subtraction series. The first series begins
with 8 and adds 3; the second begins with 43 and subtracts 2.

340)Option B

Directions (1-10): What will come in place of the question marks (?) in the following Number
series?
341. 0, 6, 24, 60, 120, 210, ?
A. 336
B. 349
C. 312
D. 337
E. None of these

342. 11, 14, 19, 22, 27, 30, ?


A. 39
B. 34
C. 36
D. 35
E. None of these

www.ibpsguide.com | estore.ibpsguide.com| www.sscexamguide.com


127
For Free Online Mock Test for IBPS PO/Clerk 2017 – Visit: estore.ibpsguide.com
500 Most Important Number Series Questions

343. 6, 12, 21, ? , 48


A. 33
B. 39
C. 36
D. 31
E. None of these

344. 18, 22, 30, ?,78, 142


A. 44
B. 35
C. 46
D. 48
E. None of these

345. 73205, 6655, 605, 55, ?


A. 9
B. 5
C. 13
D. 11
E. None of these

346. 25, 100, ?, 1600, 6400


A. 400
B. 300
C. 360
D. 420
E. None of these

347. 125, ?, 343, 512, 729, 1000


A. 216
B. 215
C. 256
D. 225

www.ibpsguide.com | estore.ibpsguide.com| www.sscexamguide.com


128
For Free Online Mock Test for IBPS PO/Clerk 2017 – Visit: estore.ibpsguide.com
500 Most Important Number Series Questions

E. None of these

348. 1 , 9 , 125 , 343 , ? , 1331


A. 730
B. 729
C. 512
D. 772
E. None of these

349. 121, 144, 169, ?, 225


A. 180
B. 172
C. 186
D. 196
E. None of these

350. ?, 2116, 2209, 2304, 2401, 2500


A. 2124
B. 1972
C. 1521
D. 2025
E. None of these

341. (A)
The given series is : 13 – 1, 23 – 2, 33 – 3, 43 – 4, 53 – 5, 63 – 6,
So the missing term = 73 – 7 = 343 – 7 = 336 .

342. (D)
The pattern is + 3, + 5, + 3, + 5, …………
So the missing term is = 30 + 5 = 35 .

343. (A)
The pattern is + 6, + 9, + 12, +15 ………..
So the missing term is = 21 + 12 = 33 .

www.ibpsguide.com | estore.ibpsguide.com| www.sscexamguide.com


129
For Free Online Mock Test for IBPS PO/Clerk 2017 – Visit: estore.ibpsguide.com
500 Most Important Number Series Questions

344. (C)
The pattern is +4, +8, +16, +32, +64
So the missing term is = 30 + 16 = 46 .

345. (B)
5 x 11 = 55, 55 x 11 = 605, 605 x 11 = 6655, 6655 x 11 = 73205

346. (A)
25 x 4 = 100, 100 x 4 = 400, 400 x 4 = 1600, 1600 x 4 = 6400.

347. (A)
125 = 53 , 216 = 63, 343 = 73, 512 = 83, 729 = 93, 1000 = 103.

348. (B)
13 , 33 , 53 , 73 , 93 , 113

349. (D)
121 = 112, 144 = 122, 169 = 132, 196 = 142, 225 = 152.

350. (D)
2025 = 452, 2116 = 462, 2304 = 482, 2401 = 492, 2500 = 502

(351-360) What will come in place of the question mark (?) in the following number series.
351) 100, 100, 119, ?, 299, 562
A) 170
B) 176
C) 174
D) 184
E) 164
352) 3, 4, 12, 45, ?, 1005
A) 196
B) 109
C) 99

www.ibpsguide.com | estore.ibpsguide.com| www.sscexamguide.com


130
For Free Online Mock Test for IBPS PO/Clerk 2017 – Visit: estore.ibpsguide.com
500 Most Important Number Series Questions

D) 110
E) 182
353) 1, 3, 9, 31, ?, 651
A) 127
B) 129
C) 131
D) 120
E) 130
354) 5, 12, 26, 54, ?, 222
A) 109
B) 110
C) 112
D) 111
E) None of these

355) 8, 11, 20, ?, 128, 371


A) 55
B) 51
C) 49
D) 48
E) 47
356) 3, 10, 20, 39, 85, ?
A) 105
B) 121
C) 212
D) 127
E) 129
357) 4, 31, 156, 499, 1228, ?
A) 1525
B) 2559
C) 2232
D) 2560
E) 2533
358) 152, 156, 140, 176, 112, ?

www.ibpsguide.com | estore.ibpsguide.com| www.sscexamguide.com


131
For Free Online Mock Test for IBPS PO/Clerk 2017 – Visit: estore.ibpsguide.com
500 Most Important Number Series Questions

A) 212
B) 112
C) 152
D) 198
E) 232
359) 7, 9, 21, 67, 273, ?
A) 1171
B) 1272
C) 1225
D) 1371
E) 1470
360) 6, 4, 5, 11,?, 361
A) 55
B) 51
C) 49
D) 48
E) 45

351) C)
The number series pattern is *2-102, *2 – 92, *2 -82…….,
? = 119 *2 – 82 = 174

352) A)
*1 + 12, *2 + 22, *3+32, *4+42……
? = 45*4+16 = 196

353) B)
*1+2, *2+3, *3+4,…..*5+6
? = 31*4+5 = 129

354) B)
+7, +14, +28, +56……
? = 54+56 = 110

www.ibpsguide.com | estore.ibpsguide.com| www.sscexamguide.com


132
For Free Online Mock Test for IBPS PO/Clerk 2017 – Visit: estore.ibpsguide.com
500 Most Important Number Series Questions

355) E)
+3, +9, +27, +81, 243
? = 47

356) C)
First difference of numbers i.e.

357) B)
+33 , +53, +73, +93, 113
? = 1228 + 1331 = 2559

358) A)
+22, +42, +62, +82…..
? = 112 + 100 = 212

359) D)
*1+2, *2+3, *3+4…….
? = 273 * 5+6 = 1371

360) E)
*0.5+1, *1+1, *2+1, *4+1….
? = 45 * 8+1 = 361

Directions (361-370): Find the missing number:

361) 11, 13, 17, 19, 23, 29, 31, 37, 41, (…)
(A) 43
(B) 47
(C) 53

www.ibpsguide.com | estore.ibpsguide.com| www.sscexamguide.com


133
For Free Online Mock Test for IBPS PO/Clerk 2017 – Visit: estore.ibpsguide.com
500 Most Important Number Series Questions

(D) 51
(E) None of these

362) 15, 31, 63, 127, 255, (…)


(A) 513
(B) 511
(C) 517
(D) 523
(E) None of these

363) 8, 24, 12, 36, 18, 54, (…)


(A) 27
(B) 108
(C) 68
(D) 72
(E) None of these

364) 7, 26, 63, 124, 215, 342, (…)


(A) 481
(B) 511
(C) 391
(D) 421
(E) None of these

365) 2, 4, 12, 48, 240, (…)


(A) 960
(B) 1440
(C) 1080
(D) 1920
(E) None of these

366) 100 100 119 ? 299 562


(A) 184
(B) 220

www.ibpsguide.com | estore.ibpsguide.com| www.sscexamguide.com


134
For Free Online Mock Test for IBPS PO/Clerk 2017 – Visit: estore.ibpsguide.com
500 Most Important Number Series Questions

(C) 240
(D) 174
(E) None of these

367) 3 4 12 45 ? 1005
(A) 152
(B) 198
(C) 144
(D) 192
(E) None of these

368) 1 3 9 31 ? 651
(A) 97
(B) 127
(C) 129
(D) 109
(E) None of these

369) 5 12 36 123 ? 2555 15342


(A) 508
(B) 381
(C) 504
(D) 635
(E) None of these

370) 8 11 17 ? 65 165.5 498.5


(A) 27.5
(B) 32
(C) 28
(D) 30.5
(E) None of these

361) A
Sol. Numbers are all primes. The next prime is 43.

www.ibpsguide.com | estore.ibpsguide.com| www.sscexamguide.com


135
For Free Online Mock Test for IBPS PO/Clerk 2017 – Visit: estore.ibpsguide.com
500 Most Important Number Series Questions

362) B
Sol. Each number is double the preceding one plus 1.
So, the next number is (255 × 2) + 1 = 511.

363) A
Sol. Numbers are alternately multiplied by 3 and divided by 2.
So, the next number = 54 ÷ 2 = 27.

364) B

365) B
Sol. Go on multiplying the given numbers by 2, 3, 4, 5, 6. So, the correct next number is 1440.

366) D
Sol. The Series is:
× 2 - 102, × 2 - 92, × 2 - 82, × 2 - 72, × 2 - 62

367) E
Sol. The Series is:
× 1 + 12, × 2 + 22, × 3 + 32……

368) C
Sol. The Series is:
× 1 + 2, × 2 + 3, × 3 + 4, × 4 + 5

369) A
Sol. The pattern of the Series is:
5x1 + 1x7 = 12
12x2 + 2x6 = 36
36x3 + 3x5 = 123
123x4 + 4x4 = 508

www.ibpsguide.com | estore.ibpsguide.com| www.sscexamguide.com


136
For Free Online Mock Test for IBPS PO/Clerk 2017 – Visit: estore.ibpsguide.com
500 Most Important Number Series Questions

370) D
Sol. The pattern of the Series is:
8x0.5 + 7 = 11
11x1 + 6 = 17
17x1.5 + 5 = 30.5
30.5x2 + 4 = 65

Direction (371-380): What will be the come in place of question mark (?) in the following
number series?
371)5 7.5 15 37.5 112.5 ?
(A) 340
(B) 395.5
(C) 393.75
(D) 397.25
(E) 339

372)66 35 72 38 78 ?
(A) 39
(B) 158
(C) 37
(D) 41
(E) 40

373)9 5 6 10.5 23 ?
(A) 30
(B) 48
(C) 60
(D) 69
(E) 65

374)2 3 6 18 108 ?
(A) 2000
(B) 1953
(C) 1928

www.ibpsguide.com | estore.ibpsguide.com| www.sscexamguide.com


137
For Free Online Mock Test for IBPS PO/Clerk 2017 – Visit: estore.ibpsguide.com
500 Most Important Number Series Questions

(D) 1944
(E) 1900

375)6 20 83 419 2519 ?


(A) 18924
(B) 19230
(C) 16510
(D) 17892
(E) 17639

376) 680 655 704 623 744 ?


A) 575
B) 675
C) 600
D) 913
E) None of these

377) 10 10 20 ? 110 300 930


A) 40
B) 35
C) 45
D) 50
E) 60

378)1 ? 22 188 2052 28748


A) 2
B) 3
C) 6
D) 16
E) 10

379) 50327 7169 1215 223 76 ?


A) 1
B) 2

www.ibpsguide.com | estore.ibpsguide.com| www.sscexamguide.com


138
For Free Online Mock Test for IBPS PO/Clerk 2017 – Visit: estore.ibpsguide.com
500 Most Important Number Series Questions

C) 3
D) 4
E) 8

380) 14 34 62 ? 142 194


A) 94
B) 98
C) 108
D) 112
E) None of these

371) Ans.(C)
Sol. The series is:
5×1.5, 7.5×2, 15×2.5, 37.5×3, 112.5×3.5

372) Ans.(D)
Sol. The series is:
66, 72, 78….. & 35, 38, 41

373) Ans.(C)
Sol. The series is:
9× ½ + ½ , 5×1 + 1, 6×1.5 + 1.5, 10.5×2 + 2, 23×2.5 + 2.5

374) Ans.(D)
Sol. The series is:
2×3, 3×6, 18×6, 108×18 = 1944

375) Ans.(E)
Sol. The series is:
6×3 + 2, 20×4 + 3, 83×5 + 4, 419×6 + 5, 2519×7 + 6

376) A
680 - 5^2 = 655
655 + 7^2 = 704

www.ibpsguide.com | estore.ibpsguide.com| www.sscexamguide.com


139
For Free Online Mock Test for IBPS PO/Clerk 2017 – Visit: estore.ibpsguide.com
500 Most Important Number Series Questions

704 - 9^2 = 623


623 + 11^2 = 744
744 - 13^2 = 575

377) C
10 × 0.5 + 5 = 10
10 × 1 + 10 = 20
20 × 1.5 + 15 = 45
45 × 2 + 20 = 110
110 × 2.5 + 25 = 300
300 × 3 + 30 = 930

378) C
1×2+4=6
6 × 5 - 8 = 22
22 × 8 + 12 = 188
188 × 11 - 16 = 2052
2052 × 14 + 20 = 28748

379) D
(50327 – 144) ÷ 7 = 7169
(7169 + 121) ÷ 6 = 1215
(1215 – 100) ÷ 5 = 223
(223 + 81) ÷ 4 = 76
(76 – 64) ÷ 3 = 4

380) B
2 + 3 × 4 = 14
4 + 5 × 6 = 34
6 + 7 × 8 = 62
8 + 9 × 10 = 98
10 + 11 × 12 = 142
12 + 13 × 14 = 194

www.ibpsguide.com | estore.ibpsguide.com| www.sscexamguide.com


140
For Free Online Mock Test for IBPS PO/Clerk 2017 – Visit: estore.ibpsguide.com
500 Most Important Number Series Questions

Direction(381-390): What would come in place of the question mark (?) in the following
number series?
381) 7, 28, 63, ?, 215, 344
a. 121
b. 163
c. 126
d. 134
e. 156

382) 441, 429, 454, 442, 467, ?


a. 421
b. 455
c. 445
d. 492
e. 437

383) 38, 50, 65, 81, 96, 108 ?


a. 38
b. 50
c. 82
d. 96
e. 108

384) 18, 41, 112, 327, 974, ?


a. 2917
b. 1260
c. 1458
d. 1267
e. 1563

385) 0, 1, 5, 41, ?, 3263441


a. 1841
b. 1830
c. 1820

www.ibpsguide.com | estore.ibpsguide.com| www.sscexamguide.com


141
For Free Online Mock Test for IBPS PO/Clerk 2017 – Visit: estore.ibpsguide.com
500 Most Important Number Series Questions

d. 1876
e. 1805

386) 7, 9, 21, 67, ?, 1371, 8233


a. 273
b. 268
c. 337
d. 341
e. 409

387) 6, 16, 57 , 244, 1245, ?, 52591


a. 7515
b. 7506
c. 7445
d. 7816
e. 7726

388) 24, 36, 90, 315, ?, 7796.25


a. 1415
b. 1517
c. 1525.5
d. 1335.5
e. 1417.5

389) 2, 3, 8, 30, 144, ?


a. 720
b. 640
c. 240
d. 840
e. 480

390) 408, 120, 312, ?, 72, 120


a. 148
b. 184

www.ibpsguide.com | estore.ibpsguide.com| www.sscexamguide.com


142
For Free Online Mock Test for IBPS PO/Clerk 2017 – Visit: estore.ibpsguide.com
500 Most Important Number Series Questions

c. 168
d. 124
e. None of these

381) 23 – 1 = 7
33 + 1 = 28
43– 1 = 63
53 + 1 = 126
63– 1 = 215
73 + 1 = 344
Hence, option c.

382) 441-12 = 429


429 +25= 454
454-12=442
442+25 =467
467-12= 455
Hence, option b

383) 31+7*1 = 38
38+6*2 = 50
50+5*3 = 65
65+4*4 = 81
81+3*5 = 96
96+2*6 = 108
Hence, option c.

384) 6+ 6*2= 18
5 + 18*2 = 41
4 + 54*2 = 112
3 + 162*2 =327
2 + 486*2 = 974
1 + 1458*2 = 2917
Hence, option a.

www.ibpsguide.com | estore.ibpsguide.com| www.sscexamguide.com


143
For Free Online Mock Test for IBPS PO/Clerk 2017 – Visit: estore.ibpsguide.com
500 Most Important Number Series Questions

385) 0 +12= 1
1+22 = 5
5+62= 41
41+ 422 = 1805
1805 + 18062 = 3263441
Hence, option e.

386) 7 × 1 + 2 = 9, 9 × 2 + 3 = 21
21 × 3 + 4 = 67, 67 × 4 + 5 = 273
273 × 5 + 6 = 1371, 1371 × 6 + 7 = 8233
Hence, option a

387) 6 × 2 + 22 = 16, 16 × 3 + 32 = 57
57 × 4 + 42 = 244, 244 × 5 + 52 = 1245
1245 × 6 + 62 = 7506, 7506 × 7 + 72 = 52591
Hence, option b

388) 24 × 1.5 = 36, 36 × 2.5 = 90


90 × 3.5 = 315, 315 × 4.5 = 1417.5
1417.5 × 5.5 = 7796.25
Hence, option e.

389) 0! + 1! = 2, 2! + 1! = 3
3! + 2! = 8, 4! + 3! = 30
5! + 4! = 144, 6! + 5! = 840
Hence, option d.

390) 372 – 312 = 408, 312 – 292 = 120


292 – 232 = 312, 232 – 192 = 168
192 – 172 = 72, 172 – 132 = 120
Hence, option c.

www.ibpsguide.com | estore.ibpsguide.com| www.sscexamguide.com


144
For Free Online Mock Test for IBPS PO/Clerk 2017 – Visit: estore.ibpsguide.com
500 Most Important Number Series Questions

Direction(391-400). In the given number series, what would come in place of the (?)
question mark?
391) 28, 41, ?, 696, 5816
a. 120
b. 146
c. 129
d. 132
e. 210

392) 195, 167, 141, 117, 95, ?, 57


a. 80
b. 75
c. 82
d. 81
e. 79

393) 114, 39, -18, ?, -41


a. 32
b. 35
c. 38
d. -35
e. -38

394) 343, 441, 512, 576, 729, 729, ?


a. 841
b. 824
c. 1000
d. 1331
e. 900

395) -20, -16, -7, 18, 67, ? , 357


a. 126
b. 144
c. 188

www.ibpsguide.com | estore.ibpsguide.com| www.sscexamguide.com


145
For Free Online Mock Test for IBPS PO/Clerk 2017 – Visit: estore.ibpsguide.com
500 Most Important Number Series Questions

d. 208
e. 257

396) 0, 10, 24, 68, ?


a. 90
b. 105
c. 120
d. 130
e. 145

397) 2500, 2275, 2106, ?, 1904


a. 1975
b. 1985
c. 1990
d. 1995
e. None of these

398) 11, 42, 71,?,113,130


a. 93
b. 96
c. 95
d. 94
e. None of these

399) 33, 47, 63, ?, 101, 123


a. 78
b. 76
c. 81
d. 83
e. 87

400) 47, 112, 118,184, ?, 330


a. 265
b. 227

www.ibpsguide.com | estore.ibpsguide.com| www.sscexamguide.com


146
For Free Online Mock Test for IBPS PO/Clerk 2017 – Visit: estore.ibpsguide.com
500 Most Important Number Series Questions

c. 189
d. 295
e. None of these

391) The series follows the pattern


28+13*(1)2= 41, 41+11*(2)3 = 129, 129 + 7*(3)4 = 696, 696 + 5*(4)5 = 5816
Hence, option c

392) The series follows the pattern


142 – 1 = 195, 132 – 2 = 167, 122 – 3 = 141, 112 – 4 = 117, 102 – 5 = 95, 92 – 6 = 75
82 – 7 = 57
Hence, option b

393) The series follows the pattern


5! – (1+2+3) = 114, 4! + (4+5+6) = 39, 3! – (7+8+9) = -18, 2! + (10+11+12) = 35, 1! –
(13+14+15) = -41
Hence, option b.

394) The series follows the pattern


343 = 73, 441 = (7*3)2, 512 = 83, 576 = (8*3)2, 729 = 93, 729 = (9*3)2, 1000 = 103
Hence, option c.

395) The series follows the pattern


-20 + 22 = -16, -16 + 32 = -7, -7 + 52 =18, 18 + 72 = 18 + 49 = 67, 67 + 112 = 67 + 121 = 188, 188
+132 = 188 +169 = 357
Hence, option c

396) 0 = 13-1,
10 = 23+2,
24 = 33– 3
and so on
So missing no. will be 53-5 = 120
Hence, option c

www.ibpsguide.com | estore.ibpsguide.com| www.sscexamguide.com


147
For Free Online Mock Test for IBPS PO/Clerk 2017 – Visit: estore.ibpsguide.com
500 Most Important Number Series Questions

397) The series follows the pattern:


2500-152 = 2275, 2275-132 = 2106, 2106-112 = 1985, 1985-92 = 1904
Hence, option b

398) The series follows the pattern


11+31 = 42, 42+29 = 71, 71+23 = 94, 94+19 = 113, 113+17 = 130
Hence, option d

399)The series follows the pattern:


33 + 14 = 47, 47 + 16 = 63, 63 + 18 = 81, 81 + 20 = 101, 101 + 22 = 123
Hence, option c.

400) The series follows the pattern:


47 + 42 + 72 = 112, 112 + 12 + 12 + 22 = 118, 118 + 12 + 12 + 82 = 184, 184 + 12 + 82 + 42 = 265,
265 +22 + 62 + 52 = 330
Hence, option a

What would come in place of the question mark (?) in the following number series?
401) 13, 64, 248, 717, 1370, ?
a. 1394
b. 1379
c. 1245
d. 1296
e. 1398

402) 9, 15, 26, 46, 85, ?


a. 151
b. 155
c. 163
d. 174
e. 180

403) 5, 1, 5, 25, 157, ?


a. 1516

www.ibpsguide.com | estore.ibpsguide.com| www.sscexamguide.com


148
For Free Online Mock Test for IBPS PO/Clerk 2017 – Visit: estore.ibpsguide.com
500 Most Important Number Series Questions

b. 1558
c. 1432
d. 1265
e. 1213

404) 42, 78, 303, 339, ?, 789


a. 564
b. 432
c. 386
d. 271
e. 621

405) 68, 69, 73, 100, ?


a. 156
b. 107
c. 132
d. 116
e. 124

406) 111, 138, 238, 302, 423, ?


a. 532
b. 523
c. 548
d. 623
e. 638

407) 31, 29, 54, 154, 600, ?


a. 1696
b. 2024
c. 2025
d. 2116
e. 2968

www.ibpsguide.com | estore.ibpsguide.com| www.sscexamguide.com


149
For Free Online Mock Test for IBPS PO/Clerk 2017 – Visit: estore.ibpsguide.com
500 Most Important Number Series Questions

408) 4, 13, 49, 130, ?


a. 245
b. 264
c. 274
d. 215
e. 166

409) 16, 48, -3, 69, ?


a. -5
b. 12
c. -14
d. -26
e. None of these

410) 23, 26, 44, 64, ?


a. 92
b. 84
c. 79
d. 95
e. 67

401) The series follows the pattern


13*5 -13 = 64, 64*4 -23 = 248, 248*3 -33 = 717, 717*2 – 43 = 1370, hence 1370*1 -53 = 1245
Hence, option c.

402) The next term is obtained by adding the immediate prime number smaller than the given
number and subtracting consecutive natural numbers from them.
9+7-1=15, 15+13-2=26, 26+23-3=46, 46+43-5=85, hence 85+83-5= 163
Hence, option c.

403) The series follows the pattern


5*0 + 1 = 1, 1*2 + 3 = 5, 5*4 + 5 = 25, 25*6 + 7 = 157, 157*8 + 9 = 1265
Hence, option d.

www.ibpsguide.com | estore.ibpsguide.com| www.sscexamguide.com


150
For Free Online Mock Test for IBPS PO/Clerk 2017 – Visit: estore.ibpsguide.com
500 Most Important Number Series Questions

404) The series follows the pattern


42 + (4 + 2)2 = 78, 78 + (7 + 8)2 = 303, 303 + (3 + 0 + 3)2 = 339, 339 + (3 + 3 + 9)2 = 564, 564 +
(5 + 6 + 4)2 = 789
Hence, option a

405) The series follows the pattern


68 + 13 = 69, 69 + 22 = 73, 73 + 33 = 100, 100 + 42 = 116
Hence, option d.

406) The series follows the pattern


111 + 33 = 138, 138 + 102 = 238, 238 + 43 = 302, 302 + 112 = 423, 423 + 53 = 548
Hence, option c.

407) The series follows the pattern


(31*1) – 2 = 29, (29*2) – 4 = 54, (54*3) – 8 = 154, (154*4) – 16 = 600, (600*5) – 32 = 2968
Hence, option e.

408) The series follows the pattern


4 + 32 = 13, 13 + 62 = 49, 49 + 92 = 130, 130 + 122 = 274
Hence, option c.

409) The series follows the pattern


16 + 16*2 = 48, 48 – 17*3 = -3, -3 + 18*4 = 69, 69 – 19*5 = -26
Hence, option d
410) The last digit of the previous term is multiplied by consecutive odd numbers to get the next
term
23 + 3 * 1 = 26, 26 + 6 * 3 = 44, 44 + 4 * 5 = 64, 64 + 4 * 7 = 92
Hence, option a.
Direction (411 – 42): What will come in place of (?) in the following number series.
411) 3, 11, 54, 339, 2732,?
a) 27356
b) 22852
c) 21650
d) 27375

www.ibpsguide.com | estore.ibpsguide.com| www.sscexamguide.com


151
For Free Online Mock Test for IBPS PO/Clerk 2017 – Visit: estore.ibpsguide.com
500 Most Important Number Series Questions

e) 27345
412) 358, 320, 284, 250,?
a) 218
b) 258
c) 236
d) 264
e) 278
413) 8, 22, 80, 364, ?
a) 4805
b) 2010
c) 2007
d) 2560
e) 2700
414) 7, 22, 118, 1107, ?
a) 17756
b) 24334
c) 17779
d) 17772
e) 28986
415) 8, 37, 104, 233, 454, ?
a) 338
b) 823
c) 837
d) 803
e) 934
416) 12, 19, 35, 59, 90, ?
a) 132
b) 126
c) 127
d) None of these
e) 146
417). 4, 10, 27, 112, 555, ?
a) 3786
b) 4756

www.ibpsguide.com | estore.ibpsguide.com| www.sscexamguide.com


152
For Free Online Mock Test for IBPS PO/Clerk 2017 – Visit: estore.ibpsguide.com
500 Most Important Number Series Questions

c) None of these
d) 3336
e) 3686
418) 6,14,45,184,925 ?
a) 4596
b) 5556
c) 6756
d) 6556
e) None of these
419) 11,20,38,74,?
a) 164
b) 146
c) 182
d) 150
e) None of these
420) 15,21,38,65,101, ?
a) 150
b) 130
c) 155
d) 145
e) None of these
411). The Series is x2+5, x4+10, x6+15, x8+20, x10+25
412).The Series is 19^2-3,18^2-4, 17^2-5, 16^2-6, 15^2-7
413). Series is x2.5+2, x3.5+3, x4.5+4, x5.5+5
414).Series is x1^2+15, x2^2+30, x3^2+45, x4^2+60
415).Difference between no. is, 3^3+2, 4^3+3, 5^3+4, 6^3+5, 7^3+6
146
416). The difference of difference of the series is, 9, 8, 7,….(i.e. 16 – 7 = 9; 24 – 16 = 8…..)
19 – 12 = 7
35 – 19 = 16
59 – 35 = 24…
417).
The series is,
4 * 2 + 2 = 10

www.ibpsguide.com | estore.ibpsguide.com| www.sscexamguide.com


153
For Free Online Mock Test for IBPS PO/Clerk 2017 – Visit: estore.ibpsguide.com
500 Most Important Number Series Questions

10 * 3 – 3 = 27
27 * 4 + 4 = 112…
418).
The series is,
6 * 2 + 2 = 14
14 * 3 + 3 = 45
45 * 4 + 4 = 184…..
419)The series is
20 – 11 = 9
38 – 20 = 18
74 – 38 = 36….
420) The difference of difference of the series is, 11, 10, 9,….(i.e. 17-6 = 11; 27-17 = 10….)
21 – 15 = 6
38 – 21 = 17
65 – 38 = 27…

Directions(421-430): Find the missing number in the given series:


421) 13, 7, 6, 14, 52, _
A. 482
B. 476
C. 466
D. 424
E. 460

422)139, 142, 133, 160, 79, _


A. 326
B. 322
C. 331
D. 340
E. 355

423) 10, 16, 27, 48, 84, _


A. 138
B. 153

www.ibpsguide.com | estore.ibpsguide.com| www.sscexamguide.com


154
For Free Online Mock Test for IBPS PO/Clerk 2017 – Visit: estore.ibpsguide.com
500 Most Important Number Series Questions

C. 166
D. 140
E. 135

424) 66, 77, 63, 73, 60, _


A. 66
B. 69
C. 61
D. 64
E. 63

425) 16, 9, 8, 13, 25, _


A. 64.5
B. 59.5
C. 63.5
D. 68.5
E. 61.5

426) 11, 12, 22, 69, 272, _


A. 1365
B. 1368
C. 1324
D. 1325
E. 1329

427) 11, 13, 16, 33, 96, _


A. 358
B. 354
C. 386
D. 366
E. 353

428) 4, 21, 120, 595, 2376, _


A. 7125

www.ibpsguide.com | estore.ibpsguide.com| www.sscexamguide.com


155
For Free Online Mock Test for IBPS PO/Clerk 2017 – Visit: estore.ibpsguide.com
500 Most Important Number Series Questions

B. 7035
C. 7085
D. 7055
E. 7115

429) 64, 66, 75, 103, 168, _


A. 282
B. 294
C. 286
D. 295
E. 288

430) 13, 25, 39, 57, 83, _


A. 122
B. 124
C. 126
D. 120
E. 125

421) Sol. D.424


13 * 0.5 + 0.5 = 7
7*1–1=6
6 * 2 + 2 = 14

422) Sol. B.322


139 + 3 = 142
142 – 9 = 133
133 + 27 = 160

423) Sol. D
16 – 10 = 6
27 – 16 = 11
48 – 27 = 21; 11 – 6 = 5, 21 – 11 = 10

www.ibpsguide.com | estore.ibpsguide.com| www.sscexamguide.com


156
For Free Online Mock Test for IBPS PO/Clerk 2017 – Visit: estore.ibpsguide.com
500 Most Important Number Series Questions

424) Sol. B
66 – 63 = 3
77 – 73 = 4
63 – 60 = 3

425) Sol. C
16 * 0.5 + 1 = 9
9*1–1=8
8 * 1.5 + 1 = 13

426) Sol. A
11 * 1 + 1 = 12
12 * 2 – 2 = 22
22 * 3 + 3 = 69

427) Sol. E
11 + 1² + 1= 13
13 + 2² – 1= 16
16 + 4² + 1= 33

428) Sol. A
4*7 -7 = 21
21*6 -6 = 120
120*5 -5 = 595

429) Sol. B
64 + (1³ + 1) = 66
66 + (2³ + 1) = 75
75 + (3³ + 1) = 103

430) Sol. E
13*2 – 1 = 25
25*2 – 11 = 39
39*2 – 21 = 57

www.ibpsguide.com | estore.ibpsguide.com| www.sscexamguide.com


157
For Free Online Mock Test for IBPS PO/Clerk 2017 – Visit: estore.ibpsguide.com
500 Most Important Number Series Questions

Directions (431-440) : What should come in place of question mark (?) in the following
number series ?
431)13 14 30 93 376 1885 ?
(A) 10818
(B) 10316
(C) 11316
(D) 11318
(E) None of these

432)4 6 9 13.5 20.25 30.375 ?


(A) 40.25
(B) 45.5625
(C) 42.7525
(D) 48.5625
(E) None of these

433)400 240 144 86.4 51.84 31.104 ?


(A) 19.2466
(B) 17.2244
(C) 16.8824
(D) 18.6624
(E) None of these

434)9 4.5 4.5 6.75 13.5 33.75 ?


(A) 101.25
(B) 103.75
(C) 99.75
(D) 105.50
(E) None of these

435)705 728 774 843 935 1050 ?


(A) 1190
(B) 1180

www.ibpsguide.com | estore.ibpsguide.com| www.sscexamguide.com


158
For Free Online Mock Test for IBPS PO/Clerk 2017 – Visit: estore.ibpsguide.com
500 Most Important Number Series Questions

(C) 1185
(D) 1187
(E) None of these
436) 2916, 972, ?,108, 36, 12
a) 324
b) 248
c) 234
d) 391
e) None of these
437). 8, 27, 125, ?, 1331
a) 81
b) 216
c) 512
d) 343
e) 169
438) 3, 22, 7, 45, 15,?, 31
a) 91
b) 90
c) 151
d) 121
e) 5
439) . 3, 15, ?, 51, 53, 159, 161
a) 27
b) 35
c) 17
d) 24
e) None of these
440). 5963, ?, 5938, 5913, 5877, 5828
a) 5932
b) 5940
c) 5951
d) 5954
e) None of these

www.ibpsguide.com | estore.ibpsguide.com| www.sscexamguide.com


159
For Free Online Mock Test for IBPS PO/Clerk 2017 – Visit: estore.ibpsguide.com
500 Most Important Number Series Questions

SOLUTION:
431) Ans.(C)
Sol. The given number series is based on the following pattern :
13 × 1 + 1 = 14
14 × 2 + 2 = 30
30 × 3 + 3 = 93
93 × 4 + 4 = 376
376 × 5 + 5 = 1885
? = 1885 × 6 + 6 = 11316
Hence, number 11316 will replace the question mark.

432) Ans.(B) )
Sol. The given series is based on the following pattern :×1.5,×1.5,……..

433) Ans.(D)
Sol. The given series is based on the following pattern : ×0.6,×0.6……..

434) Ans.(A)
Sol. The given series is based on the following pattern : ×0.5,×1,×1.5,×2,×2.5…….

435) Ans.(E)
Sol.
705 + 1 × 23 = 728
728 + 2 × 23 = 774
774 + 3 × 23 = 843
843 + 4 × 23 = 935
935 + 5 × 23 = 1050
? = 1050 + 6 × 23 = 1050 + 138 = 1188

436) Explanation:
2916 ÷ 3 = 972
972 ÷ 3 = 324
324 ÷ 3 = 108
108 ÷ 3 = 36

www.ibpsguide.com | estore.ibpsguide.com| www.sscexamguide.com


160
For Free Online Mock Test for IBPS PO/Clerk 2017 – Visit: estore.ibpsguide.com
500 Most Important Number Series Questions

36 ÷ 3 = 12

437) Explanation:
The pattern of the given series is:
⇒ Cube of prime numbers: ⇒
2^3 = 8
3^3 = 27
5^3 = 125
7^3 = 343
11^3 = 1331

438)Here, two alternate series are going together with the pattern given below
7=3×2+1
45 = 22 × 2 + 1
15 = 7 × 2 + 1
31 = 15 × 2 + 1
∴The number in the place of question mark = 45 × 2 + 1 = 91

439)We have the terms of the given series formed as


15 = 5 × 3
? = 15 + 2 = 17
51 = 17 × 3
53 = 51 + 2
159 = 53 × 3
161 = 159 + 2

440)
Explanation:
5963 – 3^2 = 5954
5954 – 4^2 = 5938
5938 – 5^2 = 5913
5913 – 6^2 = 5877
5877 – 7^2 = 5828

www.ibpsguide.com | estore.ibpsguide.com| www.sscexamguide.com


161
For Free Online Mock Test for IBPS PO/Clerk 2017 – Visit: estore.ibpsguide.com
500 Most Important Number Series Questions

Directions (441-450) What will come in place of question mark (?) in the given number
series ?
441)18 96 161 213 252 ?
A) 278
B) 291
C) 382
D) 362
E) 560

442)512 128 32 8 2 ?
A) 1/32
B) 1/16
C) 1/4
D) 1/2
E) 1/8

443)178 537.5 ? 18858.5 169736 1867107.5


A) 2694
B) 2693
C) 3892
D) 2574
E) 2486

444)68921 79507 91125 103823 117649 ?


A) 267752
B) 267254
C) 267755
D) 132651
E) 267782

445) 5 12.5 54.5 333.5 2676.5 ? ?


A) 26775
B) 26725.5
C) 26775.5

www.ibpsguide.com | estore.ibpsguide.com| www.sscexamguide.com


162
For Free Online Mock Test for IBPS PO/Clerk 2017 – Visit: estore.ibpsguide.com
500 Most Important Number Series Questions

D) 26785.5
E) 26778

446)19 22 28 ? 49 64
A) 42
B) 37
C) 36
D) 45
E) 40

447)48 47 92 273 ? 5435


A) 1020
B) 1050
C) 1100
D) 1088
E) 1540

448)576 599 ? 645 484 691


A) 530
B) 315
C) 360
D) 560
E) 358

449)64 25 216 49 512 ?


A) 100
B) 81
C) 136
D) 140
E) 135

450)776 ? 772 785 768 787


A) 760
B) 750

www.ibpsguide.com | estore.ibpsguide.com| www.sscexamguide.com


163
For Free Online Mock Test for IBPS PO/Clerk 2017 – Visit: estore.ibpsguide.com
500 Most Important Number Series Questions

C) 985
D) 783
E) 776

441) A-278)
The series is +78/+65/+52…..

442)D-1/2)
The pattern is divided by 4

443) B-2693)
The series is *3+3.5/*5+5.5/…..

444) D-132651)
The series is cube of 41/43/45/47

445) C-26775.5)
The pattern is *2+2.5/*4+4.5/*6+6.5/……

446) B-37)
The series is +3/+6/+9/……

447) D-1088)
The series is *1-1/*2-2/*3-3/…….

448) A-530)
The series is +23*1/-23*3/+23*5/

449) B-81)
The series is cube & square 4/5/6 /…….

450) D-783)
The series is +7/-11/+13/-17/+19

www.ibpsguide.com | estore.ibpsguide.com| www.sscexamguide.com


164
For Free Online Mock Test for IBPS PO/Clerk 2017 – Visit: estore.ibpsguide.com
500 Most Important Number Series Questions

Directions (451-460) What will come in place of question mark (?) in the given number
series ?
451) 6 3 4.5 11.25 39.375 ?
A) 589
B) 534
C) 634
D) 530
E) None of these

452) 45 5678 ? 155 210


A) 110
B) 112
C) 90
D) 111
E) 115

453)292 144 74 35 19.5 ? 5.875


A) 11.25
B) 9.5
C) 7.75
D) 8.25
E) 6.135

454)740 ? 181 86.5 38.25 13.125


A) 364
B) 368
C) 366
D) 378
E) 374

455)13 10.8 15.4 8.2 17.8 ?


A) 5.8
B) 5.6
C) 6.2

www.ibpsguide.com | estore.ibpsguide.com| www.sscexamguide.com


165
For Free Online Mock Test for IBPS PO/Clerk 2017 – Visit: estore.ibpsguide.com
500 Most Important Number Series Questions

D) 7.2
E) 6.8

456)14 12.4 15.6 ? 17.2 9.2


A) 10.8
B) 12
C) 12.8
D) 14.6
E) 18.6

457) 7 10.5 21 ? 157.5 551.25


A) 42.5
B) 52
C) 63
D) 52.5
E) 62.5

458)93 95.8 99.8 105 ? 119


A) 110.4
B) 111.4
C) 110.8
D) 112.4
E) 111.8

459) 2 3 7.5 26.25 118.125 ?


A) 648.6875
B) 649.6875
C) 650.6875
D) 675.6875
E) None of these

460) 5 7.25 13.5 25.75 46 ?


A) 70.25
B) 71.25

www.ibpsguide.com | estore.ibpsguide.com| www.sscexamguide.com


166
For Free Online Mock Test for IBPS PO/Clerk 2017 – Visit: estore.ibpsguide.com
500 Most Important Number Series Questions

C) 73.25
D) 75.25
E) 76.25

451) E- 117.1875)
The series is * .5/*1.5/*2.5/……..

452) D-111.9)
The pattern is +11/+22/+33/+44/…..

453) C-7.75)
The series is -4÷2/+4÷2/……

454) B-368)
The series is (740*.5)-2=368/(368 *.5)-3=181

455) B-5.6)
The pattern is combination of two series 13 + 2.4/10.8-2.6/

456) A-10.8)
The series is 14 +1.6*-1=12.4/12.4 +1.6 *2=15.6/15.6 +1.6*-3=10.8/

457) D-52.5)
The series is *1.5 /*2/*2.5/*3/…..

458) B-111.4)
The series is 93 +2.8=95.8/95.8 +4=99.8/99.8 +5.2=105/…….

459) B-649.6875)
The series is (2 *1.5)=3/(3*2.5=7.5/……

460) E-76.25)
The series is +square of 1.5/+square of 2.5/+ square of 3.5/…..

www.ibpsguide.com | estore.ibpsguide.com| www.sscexamguide.com


167
For Free Online Mock Test for IBPS PO/Clerk 2017 – Visit: estore.ibpsguide.com
500 Most Important Number Series Questions

Directions (461-470) What will come in place of question mark (?) in the given number
series ?
461)8835 9023 9213 9405 ? 9795
A) 9278
B) 9599
C) 9382
D) 9362
E) 9560

462) 1716 2184 2730 3360 ? 4896


A) 4080
B) 4216
C) 4400
D) 4378
E) 4234

463) 2197 3375 4913 6859 ? 12167


A) 9361
B) 10648
C) 9216
D) 9261
E) None of these

464) 68921 79507 91125 103823 117649 ?


A) 267752
B) 267254
C) 267755
D) 132651
E) 267782

465)3249 3969 4225 4761 5625 ?


A) 5929
B) 5939
C) 5949

www.ibpsguide.com | estore.ibpsguide.com| www.sscexamguide.com


168
For Free Online Mock Test for IBPS PO/Clerk 2017 – Visit: estore.ibpsguide.com
500 Most Important Number Series Questions

D) 5959
E) 4929

466) 5477 5626 5777 ? 6085 6242


A) 5842
B) 5937
C) 5736
D) 5645
E) 5930

467) 46656 1156 32768 ? 21952 676


A) 861
B) 900
C) 2700
D) 961
E) 841

468)4032 4290 4556 4830 5112 ?


A) 5329
B) 5402
C) 5302
D) 5529
E) 5482

469)1546 1731 1924 2125 2334 ?


A) 2521
B) 2531
C) 2541
D) 2551
E) 2561

470)1543 1440 1337 ? 1131


A) 1233
B) 1234

www.ibpsguide.com | estore.ibpsguide.com| www.sscexamguide.com


169
For Free Online Mock Test for IBPS PO/Clerk 2017 – Visit: estore.ibpsguide.com
500 Most Important Number Series Questions

C) 1235
D) 1238
E) 1239

461) B-9599)
The series is Sq of 94 -1/sq of 95-2/……..
462) A-4080)
The pattern is cube of 12-12/cube of 13-13/…..

463) D-9261)
The series is cube of 13/cube of 15/cube of 17/……

464) D-132651)
The series is cube of 41/43/45/47

465) A-5929)
The pattern is Square of consecutive odd numbers but not prime numbers
Sq of 57/sq of 63/sq of 65/sq of 69/sq of 75/sq of 77

466) E-5930)
The series is +149/+151/+153/…….

467) B-900)
The series is mixed .square &cube .cube of 36/sq of 34/cube of 32/sq of 30/

468) B-5402)
The series is Sq of 63 + 63/sq of 65 + 65/sq of 67 + 67/…..

469) D-2551)
The series is sq of 39 +25/sq of 41 +50/sq of 43 +75/…..

470) B-1234)
The series is subtract 103 from all the numbers

www.ibpsguide.com | estore.ibpsguide.com| www.sscexamguide.com


170
For Free Online Mock Test for IBPS PO/Clerk 2017 – Visit: estore.ibpsguide.com
500 Most Important Number Series Questions

Directions (471-780) What will come in place of question mark (?) in the given number
series ?
471) 3 5 13 49 241 ?
A) 1430
B) 1348
C) 1228
D) 1441
E) 1334

472) 7 13 31 85 247 ?
A) 365
B) 335
C) 733
D) 285
E) 295

473) 5 7 17 47 115 ?
A) 285
B) 245
C) 195
D) 299
E) 88

474) 508 256 130 67 35.5 ?


A) 18.72
B) 19.75
C) 16.66
D) 15.54
E) 14.84

475) 17 9 15 40 143.5 ?
A) 435
B) 700
C) 650.25

www.ibpsguide.com | estore.ibpsguide.com| www.sscexamguide.com


171
For Free Online Mock Test for IBPS PO/Clerk 2017 – Visit: estore.ibpsguide.com
500 Most Important Number Series Questions

D) 578
E) 678.5

476) 5120 1280 320 80 ?


A) 25
B) 20
C) 28
D) 40
E) 57

477) 8 17 42 91 ?
A) 122
B) 167
C) 142
D) 172
E) 187

478) 8 7 12 33 128 ?
A) 635
B) 528
C) 372
D) 402
E) 692

479) 982 977 952 827 822 ?


A) 632
B) 698
C) 625
D) 538
E) 797

480)11 23 47 95 191 ?
A) 351
B) 329

www.ibpsguide.com | estore.ibpsguide.com| www.sscexamguide.com


172
For Free Online Mock Test for IBPS PO/Clerk 2017 – Visit: estore.ibpsguide.com
500 Most Important Number Series Questions

C) 213
D) 383
E) 221

471).D)
The series is * 2-1/&3-2/…..

472).C)
The series is *3-8 /*3-8/ ….

473).B)
The series is difference is -Cube of any number + same number
7-5=2=cube of 1 + 1/17-7=10=cube of 2+2/……

474).B)
The series is ÷2 +2/ …….

475).C)
The series is *.5 + .5/*1.5 + 1.5/*2.5 +2.5/ ……….

476).B)
The series is ÷ 4/÷4/….

477).D)
The series is + square of number + square of 3/+ square of 5/…..

478).A)
The series is*1-1/*2-2/*3-3/….

479).E)
The series is -5/-25/-125/-5/-25/…..

480).D)
The series is *2+1/*2+1/*2+1/……..

www.ibpsguide.com | estore.ibpsguide.com| www.sscexamguide.com


173
For Free Online Mock Test for IBPS PO/Clerk 2017 – Visit: estore.ibpsguide.com
500 Most Important Number Series Questions

Directions (481-490) What will come in place of question mark (?) in the given number
series ?
481) 2 4 10 ? 82 244
A) 30
B) 48
C) 28
D) 46
E) 34

482) 33 16.5 ? 24.75 49.5 123.75


A) 16.5
B) 13.5
C) 22.5
D) 20.5
E) 12.5

483) 20 23 30 43 64 ?
A) 85
B) 92
C) 95
D) 99
E) 88

484) 44 ? 99 148.5 222.75 334.125


A) 72
B) 77
C) 66
D) 54
E) 84

485). 7 8 4 13 -3 22 ?
A) -7
B) -10

www.ibpsguide.com | estore.ibpsguide.com| www.sscexamguide.com


174
For Free Online Mock Test for IBPS PO/Clerk 2017 – Visit: estore.ibpsguide.com
500 Most Important Number Series Questions

C) -12
D) -14
E) -9

486). 124 215 342 511 ? 999


A) 695
B) 625
C) 728
D) 806
E) 573

487). 516 256 126 61 28.5 ?


A) 12.25
B) 16.75
C) 14.25
D) 20.20
E) 18.75

488). 7 21 50 109 ? 467


A) 198
B) 228
C) 372
D) 402
E) 275

489). 18 82 118 134 138 ?


A) 132
B) 198
C) 125
D) 138
E) 92

490). 5 22 103 406 ? 2422


A) 1516

www.ibpsguide.com | estore.ibpsguide.com| www.sscexamguide.com


175
For Free Online Mock Test for IBPS PO/Clerk 2017 – Visit: estore.ibpsguide.com
500 Most Important Number Series Questions

B) 1298
C) 1136
D) 1602
E) 1213

481). C)
The series is *3-2 , *3-2 ,……..

482). A)
The series is *5 , *1 , *1.5 , ….

483). C)
+ 3 , + 7, +13 , +21, + 31
+4 +6 +8 +10

484). C)
The series is *1.5 , *1.5 , …….

485). D)
The series is + square of 1 , – square of 2 , ……….

486). C)
The series is cube of number -1
Cube of 5 – 1 = 124 , cube of 6-1= 215 , …..

487). A)
The series is 516-4 ÷2=256 , 256-4 ÷2=126 , …….

488). B)
The series is*2 + 7 , *2 +8 ,…….

489). D)

www.ibpsguide.com | estore.ibpsguide.com| www.sscexamguide.com


176
For Free Online Mock Test for IBPS PO/Clerk 2017 – Visit: estore.ibpsguide.com
500 Most Important Number Series Questions

The series is + square of number


+ square of 8 , + square of 6 , …………..

490). E)
The series is
*6-8/*5-7/*4-6/……………..

491). 150 102 70 46 26 ?


A) 7
B) 13
C) 8
E) 2
E) 12

492). 10 14 24 52 134
A) 351
B) 302
C) 368
D) 341
E) 378

493). 24 11 10 14 27 ?
A) 67.5
B) 60.5
C) 66.5
D) 61.5
E) 62.55

494). 4500 900 90 6 ? 0.012


A) 0.3
B) 0.09
C) 0.9
D) 0.015
E) 0.03

www.ibpsguide.com | estore.ibpsguide.com| www.sscexamguide.com


177
For Free Online Mock Test for IBPS PO/Clerk 2017 – Visit: estore.ibpsguide.com
500 Most Important Number Series Questions

495). 8 7 12 33 128 ?
A) 672
B) 684
C) 635
D) 620
E) 692

496). 16 17 21 30 46 ?
A) 82
B) 104
C) 71
D) 92
E) 84

497). 2 2 4 12 48 ?
A) 180
B) 220
C) 240
D) 160
E) 210

498). 10 14 23 37 56 ?
A) 74
B) 80
C) 118
D) 120
E) 94

499). 4 7 13 25 49 ?
A) 118
B) 136
C) 86

www.ibpsguide.com | estore.ibpsguide.com| www.sscexamguide.com


178
For Free Online Mock Test for IBPS PO/Clerk 2017 – Visit: estore.ibpsguide.com
500 Most Important Number Series Questions

D) 97
E) 124

500). 50 59 41 68 32 ?
a) 77
b) 45
c) 20
d) 85
e) 60

491) C
150-102=48
102-70=32
70-46=24
46-26=20
26-8=18

492) E
10+(4)=14
14 + (10)=24
24 +(28)=52
52+(82)=134
134 + (244)=378

493) C
24*(1/2)-1=11
11*1-1=10
10*(3/2)-1=14
So on

494) A
4500/5=900
900/10=90
90/15=6

www.ibpsguide.com | estore.ibpsguide.com| www.sscexamguide.com


179
For Free Online Mock Test for IBPS PO/Clerk 2017 – Visit: estore.ibpsguide.com
500 Most Important Number Series Questions

6/20=.3
.3/.25=.012

495) C
8*1-1=7
7*2-2=12
So on

496) C
16 + (12)=17
17 + (22)=21
So on

497) C
2 *1=2
3*2=6
4*3=12
So on

498) B
10 + (4)=14
14+(9)=23
23+(14)=37
So on

499) D
4 *2-1=7
7*2-1=13
So on

500) A
50 + (9)=59
59 -(18)=41
41+ (27)=68

www.ibpsguide.com | estore.ibpsguide.com| www.sscexamguide.com


180
For Free Online Mock Test for IBPS PO/Clerk 2017 – Visit: estore.ibpsguide.com
500 Most Important Number Series Questions

68-(36)=32
So on.

www.ibpsguide.com | estore.ibpsguide.com| www.sscexamguide.com


181
For Free Online Mock Test for IBPS PO/Clerk 2017 – Visit: estore.ibpsguide.com
Ibpsguide.com (Paid) 1000 IMPORTANT SIMPLIFICATION AND APPROXIMATION Q and A by Harisan

-
Ibpsguide.com (Paid) 1000 IMPORTANT SIMPLIFICATION AND APPROXIMATION Q and A by Harisan
Ibpsguide.com (Paid) 1000 IMPORTANT SIMPLIFICATION AND APPROXIMATION Q and A by Harisan
Ibpsguide.com (Paid) 1000 IMPORTANT SIMPLIFICATION AND APPROXIMATION Q and A by Harisan
Ibpsguide.com (Paid) 1000 IMPORTANT SIMPLIFICATION AND APPROXIMATION Q and A by Harisan
Ibpsguide.com (Paid) 1000 IMPORTANT SIMPLIFICATION AND APPROXIMATION Q and A by Harisan
Ibpsguide.com (Paid) 1000 IMPORTANT SIMPLIFICATION AND APPROXIMATION Q and A by Harisan
Ibpsguide.com (Paid) 1000 IMPORTANT SIMPLIFICATION AND APPROXIMATION Q and A by Harisan
Ibpsguide.com (Paid) 1000 IMPORTANT SIMPLIFICATION AND APPROXIMATION Q and A by Harisan
Ibpsguide.com (Paid) 1000 IMPORTANT SIMPLIFICATION AND APPROXIMATION Q and A by Harisan
Ibpsguide.com (Paid) 1000 IMPORTANT SIMPLIFICATION AND APPROXIMATION Q and A by Harisan
Ibpsguide.com (Paid) 1000 IMPORTANT SIMPLIFICATION AND APPROXIMATION Q and A by Harisan
Ibpsguide.com (Paid) 1000 IMPORTANT SIMPLIFICATION AND APPROXIMATION Q and A by Harisan
Ibpsguide.com (Paid) 1000 IMPORTANT SIMPLIFICATION AND APPROXIMATION Q and A by Harisan
Ibpsguide.com (Paid) 1000 IMPORTANT SIMPLIFICATION AND APPROXIMATION Q and A by Harisan
Ibpsguide.com (Paid) 1000 IMPORTANT SIMPLIFICATION AND APPROXIMATION Q and A by Harisan
Ibpsguide.com (Paid) 1000 IMPORTANT SIMPLIFICATION AND APPROXIMATION Q and A by Harisan
Ibpsguide.com (Paid) 1000 IMPORTANT SIMPLIFICATION AND APPROXIMATION Q and A by Harisan
Ibpsguide.com (Paid) 1000 IMPORTANT SIMPLIFICATION AND APPROXIMATION Q and A by Harisan
Ibpsguide.com (Paid) 1000 IMPORTANT SIMPLIFICATION AND APPROXIMATION Q and A by Harisan
Ibpsguide.com (Paid) 1000 IMPORTANT SIMPLIFICATION AND APPROXIMATION Q and A by Harisan
Ibpsguide.com (Paid) 1000 IMPORTANT SIMPLIFICATION AND APPROXIMATION Q and A by Harisan
Ibpsguide.com (Paid) 1000 IMPORTANT SIMPLIFICATION AND APPROXIMATION Q and A by Harisan
Ibpsguide.com (Paid) 1000 IMPORTANT SIMPLIFICATION AND APPROXIMATION Q and A by Harisan
Ibpsguide.com (Paid) 1000 IMPORTANT SIMPLIFICATION AND APPROXIMATION Q and A by Harisan
Ibpsguide.com (Paid) 1000 IMPORTANT SIMPLIFICATION AND APPROXIMATION Q and A by Harisan
Ibpsguide.com (Paid) 1000 IMPORTANT SIMPLIFICATION AND APPROXIMATION Q and A by Harisan
Ibpsguide.com (Paid) 1000 IMPORTANT SIMPLIFICATION AND APPROXIMATION Q and A by Harisan
Ibpsguide.com (Paid) 1000 IMPORTANT SIMPLIFICATION AND APPROXIMATION Q and A by Harisan
Ibpsguide.com (Paid) 1000 IMPORTANT SIMPLIFICATION AND APPROXIMATION Q and A by Harisan
Ibpsguide.com (Paid) 1000 IMPORTANT SIMPLIFICATION AND APPROXIMATION Q and A by Harisan
Ibpsguide.com (Paid) 1000 IMPORTANT SIMPLIFICATION AND APPROXIMATION Q and A by Harisan
Ibpsguide.com (Paid) 1000 IMPORTANT SIMPLIFICATION AND APPROXIMATION Q and A by Harisan
Ibpsguide.com (Paid) 1000 IMPORTANT SIMPLIFICATION AND APPROXIMATION Q and A by Harisan
Ibpsguide.com (Paid) 1000 IMPORTANT SIMPLIFICATION AND APPROXIMATION Q and A by Harisan
Ibpsguide.com (Paid) 1000 IMPORTANT SIMPLIFICATION AND APPROXIMATION Q and A by Harisan
Ibpsguide.com (Paid) 1000 IMPORTANT SIMPLIFICATION AND APPROXIMATION Q and A by Harisan
Ibpsguide.com (Paid) 1000 IMPORTANT SIMPLIFICATION AND APPROXIMATION Q and A by Harisan
Ibpsguide.com (Paid) 1000 IMPORTANT SIMPLIFICATION AND APPROXIMATION Q and A by Harisan
Ibpsguide.com (Paid) 1000 IMPORTANT SIMPLIFICATION AND APPROXIMATION Q and A by Harisan
Ibpsguide.com (Paid) 1000 IMPORTANT SIMPLIFICATION AND APPROXIMATION Q and A by Harisan
Ibpsguide.com (Paid) 1000 IMPORTANT SIMPLIFICATION AND APPROXIMATION Q and A by Harisan
Ibpsguide.com (Paid) 1000 IMPORTANT SIMPLIFICATION AND APPROXIMATION Q and A by Harisan
Ibpsguide.com (Paid) 1000 IMPORTANT SIMPLIFICATION AND APPROXIMATION Q and A by Harisan
Ibpsguide.com (Paid) 1000 IMPORTANT SIMPLIFICATION AND APPROXIMATION Q and A by Harisan
Ibpsguide.com (Paid) 1000 IMPORTANT SIMPLIFICATION AND APPROXIMATION Q and A by Harisan
Ibpsguide.com (Paid) 1000 IMPORTANT SIMPLIFICATION AND APPROXIMATION Q and A by Harisan
Ibpsguide.com (Paid) 1000 IMPORTANT SIMPLIFICATION AND APPROXIMATION Q and A by Harisan
Ibpsguide.com (Paid) 1000 IMPORTANT SIMPLIFICATION AND APPROXIMATION Q and A by Harisan
Ibpsguide.com (Paid) 1000 IMPORTANT SIMPLIFICATION AND APPROXIMATION Q and A by Harisan
Ibpsguide.com (Paid) 1000 IMPORTANT SIMPLIFICATION AND APPROXIMATION Q and A by Harisan
Ibpsguide.com (Paid) 1000 IMPORTANT SIMPLIFICATION AND APPROXIMATION Q and A by Harisan
Ibpsguide.com (Paid) 1000 IMPORTANT SIMPLIFICATION AND APPROXIMATION Q and A by Harisan
Ibpsguide.com (Paid) 1000 IMPORTANT SIMPLIFICATION AND APPROXIMATION Q and A by Harisan
Ibpsguide.com (Paid) 1000 IMPORTANT SIMPLIFICATION AND APPROXIMATION Q and A by Harisan
Ibpsguide.com (Paid) 1000 IMPORTANT SIMPLIFICATION AND APPROXIMATION Q and A by Harisan
Ibpsguide.com (Paid) 1000 IMPORTANT SIMPLIFICATION AND APPROXIMATION Q and A by Harisan
Ibpsguide.com (Paid) 1000 IMPORTANT SIMPLIFICATION AND APPROXIMATION Q and A by Harisan
Ibpsguide.com (Paid) 1000 IMPORTANT SIMPLIFICATION AND APPROXIMATION Q and A by Harisan
Ibpsguide.com (Paid) 1000 IMPORTANT SIMPLIFICATION AND APPROXIMATION Q and A by Harisan
Ibpsguide.com (Paid) 1000 IMPORTANT SIMPLIFICATION AND APPROXIMATION Q and A by Harisan
Ibpsguide.com (Paid) 1000 IMPORTANT SIMPLIFICATION AND APPROXIMATION Q and A by Harisan
Ibpsguide.com (Paid) 1000 IMPORTANT SIMPLIFICATION AND APPROXIMATION Q and A by Harisan
Ibpsguide.com (Paid) 1000 IMPORTANT SIMPLIFICATION AND APPROXIMATION Q and A by Harisan
Ibpsguide.com (Paid) 1000 IMPORTANT SIMPLIFICATION AND APPROXIMATION Q and A by Harisan
Ibpsguide.com (Paid) 1000 IMPORTANT SIMPLIFICATION AND APPROXIMATION Q and A by Harisan
Ibpsguide.com (Paid) 1000 IMPORTANT SIMPLIFICATION AND APPROXIMATION Q and A by Harisan
Ibpsguide.com (Paid) 1000 IMPORTANT SIMPLIFICATION AND APPROXIMATION Q and A by Harisan
Ibpsguide.com (Paid) 1000 IMPORTANT SIMPLIFICATION AND APPROXIMATION Q and A by Harisan
Ibpsguide.com (Paid) 1000 IMPORTANT SIMPLIFICATION AND APPROXIMATION Q and A by Harisan
Ibpsguide.com (Paid) 1000 IMPORTANT SIMPLIFICATION AND APPROXIMATION Q and A by Harisan
Ibpsguide.com (Paid) 1000 IMPORTANT SIMPLIFICATION AND APPROXIMATION Q and A by Harisan
Ibpsguide.com (Paid) 1000 IMPORTANT SIMPLIFICATION AND APPROXIMATION Q and A by Harisan
Ibpsguide.com (Paid) 1000 IMPORTANT SIMPLIFICATION AND APPROXIMATION Q and A by Harisan
Ibpsguide.com (Paid) 1000 IMPORTANT SIMPLIFICATION AND APPROXIMATION Q and A by Harisan
Ibpsguide.com (Paid) 1000 IMPORTANT SIMPLIFICATION AND APPROXIMATION Q and A by Harisan
Ibpsguide.com (Paid) 1000 IMPORTANT SIMPLIFICATION AND APPROXIMATION Q and A by Harisan
Ibpsguide.com (Paid) 1000 IMPORTANT SIMPLIFICATION AND APPROXIMATION Q and A by Harisan
Ibpsguide.com (Paid) 1000 IMPORTANT SIMPLIFICATION AND APPROXIMATION Q and A by Harisan
Ibpsguide.com (Paid) 1000 IMPORTANT SIMPLIFICATION AND APPROXIMATION Q and A by Harisan
Ibpsguide.com (Paid) 1000 IMPORTANT SIMPLIFICATION AND APPROXIMATION Q and A by Harisan
Ibpsguide.com (Paid) 1000 IMPORTANT SIMPLIFICATION AND APPROXIMATION Q and A by Harisan
Ibpsguide.com (Paid) 1000 IMPORTANT SIMPLIFICATION AND APPROXIMATION Q and A by Harisan
Ibpsguide.com (Paid) 1000 IMPORTANT SIMPLIFICATION AND APPROXIMATION Q and A by Harisan
Ibpsguide.com (Paid) 1000 IMPORTANT SIMPLIFICATION AND APPROXIMATION Q and A by Harisan
Ibpsguide.com (Paid) 1000 IMPORTANT SIMPLIFICATION AND APPROXIMATION Q and A by Harisan
Ibpsguide.com (Paid) 1000 IMPORTANT SIMPLIFICATION AND APPROXIMATION Q and A by Harisan
Ibpsguide.com (Paid) 1000 IMPORTANT SIMPLIFICATION AND APPROXIMATION Q and A by Harisan
Ibpsguide.com (Paid) 1000 IMPORTANT SIMPLIFICATION AND APPROXIMATION Q and A by Harisan
Ibpsguide.com (Paid) 1000 IMPORTANT SIMPLIFICATION AND APPROXIMATION Q and A by Harisan
Ibpsguide.com (Paid) 1000 IMPORTANT SIMPLIFICATION AND APPROXIMATION Q and A by Harisan
Ibpsguide.com (Paid) 1000 IMPORTANT SIMPLIFICATION AND APPROXIMATION Q and A by Harisan
Ibpsguide.com (Paid) 1000 IMPORTANT SIMPLIFICATION AND APPROXIMATION Q and A by Harisan
Ibpsguide.com (Paid) 1000 IMPORTANT SIMPLIFICATION AND APPROXIMATION Q and A by Harisan
Ibpsguide.com (Paid) 1000 IMPORTANT SIMPLIFICATION AND APPROXIMATION Q and A by Harisan
Ibpsguide.com (Paid) 1000 IMPORTANT SIMPLIFICATION AND APPROXIMATION Q and A by Harisan
Ibpsguide.com (Paid) 1000 IMPORTANT SIMPLIFICATION AND APPROXIMATION Q and A by Harisan
Ibpsguide.com (Paid) 1000 IMPORTANT SIMPLIFICATION AND APPROXIMATION Q and A by Harisan
Ibpsguide.com (Paid) 1000 IMPORTANT SIMPLIFICATION AND APPROXIMATION Q and A by Harisan
Ibpsguide.com (Paid) 1000 IMPORTANT SIMPLIFICATION AND APPROXIMATION Q and A by Harisan
Ibpsguide.com (Paid) 1000 IMPORTANT SIMPLIFICATION AND APPROXIMATION Q and A by Harisan
Ibpsguide.com (Paid) 1000 IMPORTANT SIMPLIFICATION AND APPROXIMATION Q and A by Harisan
Ibpsguide.com (Paid) 1000 IMPORTANT SIMPLIFICATION AND APPROXIMATION Q and A by Harisan
Ibpsguide.com (Paid) 1000 IMPORTANT SIMPLIFICATION AND APPROXIMATION Q and A by Harisan
Ibpsguide.com (Paid) 1000 IMPORTANT SIMPLIFICATION AND APPROXIMATION Q and A by Harisan
Ibpsguide.com (Paid) 1000 IMPORTANT SIMPLIFICATION AND APPROXIMATION Q and A by Harisan
Ibpsguide.com (Paid) 1000 IMPORTANT SIMPLIFICATION AND APPROXIMATION Q and A by Harisan
Ibpsguide.com (Paid) 1000 IMPORTANT SIMPLIFICATION AND APPROXIMATION Q and A by Harisan
Ibpsguide.com (Paid) 1000 IMPORTANT SIMPLIFICATION AND APPROXIMATION Q and A by Harisan
Ibpsguide.com (Paid) 1000 IMPORTANT SIMPLIFICATION AND APPROXIMATION Q and A by Harisan
Ibpsguide.com (Paid) 1000 IMPORTANT SIMPLIFICATION AND APPROXIMATION Q and A by Harisan
Ibpsguide.com (Paid) 1000 IMPORTANT SIMPLIFICATION AND APPROXIMATION Q and A by Harisan
Ibpsguide.com (Paid) 1000 IMPORTANT SIMPLIFICATION AND APPROXIMATION Q and A by Harisan
Ibpsguide.com (Paid) 1000 IMPORTANT SIMPLIFICATION AND APPROXIMATION Q and A by Harisan
Ibpsguide.com (Paid) 1000 IMPORTANT SIMPLIFICATION AND APPROXIMATION Q and A by Harisan
Ibpsguide.com (Paid) 1000 IMPORTANT SIMPLIFICATION AND APPROXIMATION Q and A by Harisan
Ibpsguide.com (Paid) 1000 IMPORTANT SIMPLIFICATION AND APPROXIMATION Q and A by Harisan
Ibpsguide.com (Paid) 1000 IMPORTANT SIMPLIFICATION AND APPROXIMATION Q and A by Harisan
Ibpsguide.com (Paid) 1000 IMPORTANT SIMPLIFICATION AND APPROXIMATION Q and A by Harisan
Ibpsguide.com (Paid) 1000 IMPORTANT SIMPLIFICATION AND APPROXIMATION Q and A by Harisan
Ibpsguide.com (Paid) 1000 IMPORTANT SIMPLIFICATION AND APPROXIMATION Q and A by Harisan
Ibpsguide.com (Paid) 1000 IMPORTANT SIMPLIFICATION AND APPROXIMATION Q and A by Harisan
Ibpsguide.com (Paid) 1000 IMPORTANT SIMPLIFICATION AND APPROXIMATION Q and A by Harisan
Ibpsguide.com (Paid) 1000 IMPORTANT SIMPLIFICATION AND APPROXIMATION Q and A by Harisan
Ibpsguide.com (Paid) 1000 IMPORTANT SIMPLIFICATION AND APPROXIMATION Q and A by Harisan
Ibpsguide.com (Paid) 1000 IMPORTANT SIMPLIFICATION AND APPROXIMATION Q and A by Harisan
Ibpsguide.com (Paid) 1000 IMPORTANT SIMPLIFICATION AND APPROXIMATION Q and A by Harisan
Ibpsguide.com (Paid) 1000 IMPORTANT SIMPLIFICATION AND APPROXIMATION Q and A by Harisan
Ibpsguide.com (Paid) 1000 IMPORTANT SIMPLIFICATION AND APPROXIMATION Q and A by Harisan
Ibpsguide.com (Paid) 1000 IMPORTANT SIMPLIFICATION AND APPROXIMATION Q and A by Harisan
Ibpsguide.com (Paid) 1000 IMPORTANT SIMPLIFICATION AND APPROXIMATION Q and A by Harisan
Ibpsguide.com (Paid) 1000 IMPORTANT SIMPLIFICATION AND APPROXIMATION Q and A by Harisan
Ibpsguide.com (Paid) 1000 IMPORTANT SIMPLIFICATION AND APPROXIMATION Q and A by Harisan
Ibpsguide.com (Paid) 1000 IMPORTANT SIMPLIFICATION AND APPROXIMATION Q and A by Harisan
Ibpsguide.com (Paid) 1000 IMPORTANT SIMPLIFICATION AND APPROXIMATION Q and A by Harisan
Ibpsguide.com (Paid) 1000 IMPORTANT SIMPLIFICATION AND APPROXIMATION Q and A by Harisan
Ibpsguide.com (Paid) 1000 IMPORTANT SIMPLIFICATION AND APPROXIMATION Q and A by Harisan
Ibpsguide.com (Paid) 1000 IMPORTANT SIMPLIFICATION AND APPROXIMATION Q and A by Harisan
Ibpsguide.com (Paid) 1000 IMPORTANT SIMPLIFICATION AND APPROXIMATION Q and A by Harisan
Ibpsguide.com (Paid) 1000 IMPORTANT SIMPLIFICATION AND APPROXIMATION Q and A by Harisan
Ibpsguide.com (Paid) 1000 IMPORTANT SIMPLIFICATION AND APPROXIMATION Q and A by Harisan
Ibpsguide.com (Paid) 1000 IMPORTANT SIMPLIFICATION AND APPROXIMATION Q and A by Harisan
Ibpsguide.com (Paid) 1000 IMPORTANT SIMPLIFICATION AND APPROXIMATION Q and A by Harisan
Ibpsguide.com (Paid) 1000 IMPORTANT SIMPLIFICATION AND APPROXIMATION Q and A by Harisan
Ibpsguide.com (Paid) 1000 IMPORTANT SIMPLIFICATION AND APPROXIMATION Q and A by Harisan
Ibpsguide.com (Paid) 1000 IMPORTANT SIMPLIFICATION AND APPROXIMATION Q and A by Harisan
Ibpsguide.com (Paid) 1000 IMPORTANT SIMPLIFICATION AND APPROXIMATION Q and A by Harisan
Ibpsguide.com (Paid) 1000 IMPORTANT SIMPLIFICATION AND APPROXIMATION Q and A by Harisan
Ibpsguide.com (Paid) 1000 IMPORTANT SIMPLIFICATION AND APPROXIMATION Q and A by Harisan
Ibpsguide.com (Paid) 1000 IMPORTANT SIMPLIFICATION AND APPROXIMATION Q and A by Harisan
Ibpsguide.com (Paid) 1000 IMPORTANT SIMPLIFICATION AND APPROXIMATION Q and A by Harisan
Ibpsguide.com (Paid) 1000 IMPORTANT SIMPLIFICATION AND APPROXIMATION Q and A by Harisan
Ibpsguide.com (Paid) 1000 IMPORTANT SIMPLIFICATION AND APPROXIMATION Q and A by Harisan
Ibpsguide.com (Paid) 1000 IMPORTANT SIMPLIFICATION AND APPROXIMATION Q and A by Harisan
Ibpsguide.com (Paid) 1000 IMPORTANT SIMPLIFICATION AND APPROXIMATION Q and A by Harisan
Ibpsguide.com (Paid) 1000 IMPORTANT SIMPLIFICATION AND APPROXIMATION Q and A by Harisan
Ibpsguide.com (Paid) 1000 IMPORTANT SIMPLIFICATION AND APPROXIMATION Q and A by Harisan
Ibpsguide.com (Paid) 1000 IMPORTANT SIMPLIFICATION AND APPROXIMATION Q and A by Harisan
Ibpsguide.com (Paid) 1000 IMPORTANT SIMPLIFICATION AND APPROXIMATION Q and A by Harisan
Ibpsguide.com (Paid) 1000 IMPORTANT SIMPLIFICATION AND APPROXIMATION Q and A by Harisan
Scanned by CamScanner
Scanned by CamScanner
Scanned by CamScanner
Scanned by CamScanner
Scanned by CamScanner
Scanned by CamScanner
Scanned by CamScanner
Scanned by CamScanner
Scanned by CamScanner
Scanned by CamScanner
Scanned by CamScanner
Scanned by CamScanner
Scanned by CamScanner
Scanned by CamScanner
Scanned by CamScanner
Scanned by CamScanner
Scanned by CamScanner
Scanned by CamScanner
Scanned by CamScanner
Scanned by CamScanner
Scanned by CamScanner
Scanned by CamScanner
Scanned by CamScanner
Scanned by CamScanner
Scanned by CamScanner
Scanned by CamScanner
Scanned by CamScanner
Scanned by CamScanner
Scanned by CamScanner
Scanned by CamScanner
Scanned by CamScanner
Scanned by CamScanner
Scanned by CamScanner
Scanned by CamScanner
Scanned by CamScanner
Scanned by CamScanner
Scanned by CamScanner
Scanned by CamScanner
Scanned by CamScanner
Scanned by CamScanner
Scanned by CamScanner
Scanned by CamScanner
Scanned by CamScanner
Scanned by CamScanner
Scanned by CamScanner
Scanned by CamScanner
Scanned by CamScanner
Scanned by CamScanner
Scanned by CamScanner
Scanned by CamScanner
Scanned by CamScanner
Exclusively on New Pattern – Reasoning Ability eBook

Exclusively on New Pattern – Reasoning Ability eBook


S.No Topics Total Questions Page No
1 Puzzle / Seating Arrangement 100 2
2 Reverse Syllogism 100 44
3 Input Output 50 134
4 Coding Decoding 50 162

www.ibpsguide.com | estore.ibpsguide.com | www.sscexamguide.com


1
Exclusively on New Pattern – Reasoning Ability eBook

Puzzle and Seating Arrangement

Directions (Q. 1-5): Study the following information carefully and answer the given questions.
In an inauguration function of Mumbai metro rail station, 10 central ministers started the train and travel in the
metro rail,metro rail moving on east direction. Ten ministers are sitting in two parallel sides containing five
ministers each. In side 1, Arun Jaitley, Rajnath Singh, Suresh Prabhu, Nitin Gadkari, Ravi Shankar Prasad are
sitting all of them are facing right side. In side 2, Prakash Javadekar, Piyush Goyal, Harsh Vardhan, Ananth
Kumar, Jagat Prakash Nadda are sitting all of them facing left side. Each member seated in a side faces another
member of the other side. Moreover, each of them selected to Lok Sabha from different States viz, Uttar Pradesh,
Maharashtra, Haryana, Punjab, Tripura, Rajasthan, West Bengal, Assam, Jharkhand, and Madya Pradesh but not
necessarily in the same order.
 There are only two ministers sitting between the minister selected from Uttar Pradesh, Who sits at an extreme
end, and Ravi Shankar Prasad.
 Prakash Javadekar, who sits in the middle of the row, is not an immediate neighbour of Piyush Goyal, who is
not selected fromMadya Pradesh.
 Jagat Prakash Nadda is sitting at an extreme end.
 Ravi Shankar Prasad was selected from Tripura sits on the immediate right of the minister selected from
Maharashtra and faces the immediate neighbour of Piyush Goyal.
 Rajnath Singh is not sitting at an extreme left end.
 Prakash Javadekar is not selected from Jharkhand.
 There is only one person between Suresh Prabhu and Nitin Gadkari, Who is selected from Uttar Pradesh.
 Ananth Kumarselected from Rajasthan is an immediate neighbour of the minister who selected from Madya
Pradesh and does not face the minister selected from Haryana.
 Harsh Vardhan, who is selected from Punjab, is an immediate neighbour of the minister who selected from
Jharkhand, who in turn faces the immediate neighbour of the Tripura.
 There are two ministers between the minister selected fromWest Bengal and the minister selected from
Haryana.
 Rajnath Singh is not selected from West Bengal.
 Jagat Prakash Nadda is not selected from Assam.

1. Which of the following minister selected from Madya Pradesh?


a) Harsh Vardhan
b) Rajnath Singh
c) Ananth Kumar
d) Suresh Prabhu
e) Jagat Prakash Nadda
2.Which of the following minister sits third to the left of the minister selected from Rajasthan?
a) Minister selected from Punjab

www.ibpsguide.com | estore.ibpsguide.com | www.sscexamguide.com


2
Exclusively on New Pattern – Reasoning Ability eBook

b) Minister selected from Assam


c) Minister selected from Tripura
d) Minister selected from Jharkhand
e) Minister selected from Madya Pradesh

3.Four among the following form a group in a certain way. Which of the following does not belong to Group?
a) West Bengal
b) Haryana
c) Assam
d) Tripura
e) Maharashtra

4.Arun Jaitley selected from which of the following state?


a) Rajasthan
b) West Bengal
c) Punjab
d) Tripura
e) Uttar Pradesh

5.Which of the followingstatement is true?


a) There are two ministers sitbetween Ravi Shankar Prasad andNitinGadkari
b) Harsh Vardhan faces the minister who is selected from Maharashtra
c) Prakash Javadekar and Rajnath Singh are sits at extreme ends
d) Suresh Prabhu and the minister selected from Uttar Pradesh are immediate neighbours
e) All the statements are true
Answers:
1. Answer: e)
2. Answer: d)
3. Answer: c)
4. Answer: b)
5. Answer: a)
Ravi Shankar
Rajnath Singh Suresh Prabhu Arun Jaitley Nitin Gadkari
Facing Right Prasad
(Haryana) (Maharashtra) (West Bengal) (Uttar Pradesh)
(Tripura)
Prakash JagatPrakash
Piyush Goyal Harsh Vardhan Ananth Kumar
Facing Left Javadekar Nadda
(Jharkhand) (Punjab) (Rajasthan)
(Assam) (Madya Pradesh)

www.ibpsguide.com | estore.ibpsguide.com | www.sscexamguide.com


3
Exclusively on New Pattern – Reasoning Ability eBook

Directions (Q. 6-10): Study the following information carefully and answer the given questions.
Twelve friends are sitting in two parallel rows of chairs containing six people each, in such a way that there is
equal distance between adjacent players. In row 1: Shaun Marsh, Ross Taylor, Imran Tahir, Rohit Sharma,Lasith
Malinga and Chris Gayle are seated and all of them are facing south. In row 2: Azhar Ali, Brendan Taylor,
Mushfiqur Rahim,Ben Stokes, Mohammad Nabi and Kevin O’Brien are seated and all of them are facing north.
Each of them plays for different cricket club teams such as, Somerset County Cricket Club, Lancashire County
Cricket Club, Worcestershire County Cricket Club, Essex County Cricket Club, Warwickshire County Cricket Club,
Nottinghamshire County Cricket Club, Surrey County Cricket Club, Kent County Cricket Club, Yorkshire County
Cricket Club, Glamorgan County Cricket Club, Durham County Cricket Club and Hampshire County Cricket Club
but not necessarily in the same order. Each of them belongs to different countries: England, Pakistan,
Bangladesh, South Africa, Australia, Sri Lanka, India, New Zealand, West Indies, Afghanistan, Zimbabwe and
Ireland. In the given seating arrangement, each member seated in a row faces another member of the other row.
Lasith Malinga, whobelongs toSri Lanka sits third to the left of the player who plays for Lancashire County Cricket
Club and not sitting at the extreme ends, Ross Taylor, who plays forEssex County Cricket Club, does not sit at an
extreme end of the row. If Lasith Malinga sits at an extreme end then Ross Taylor does not sit to the immediate
right of the one who plays forLancashire County Cricket Club. Mushfiqur Rahim and Azhar Ali face the players
who belong to Sri Lanka and Australia respectively. There are two players sitting on the right side of Brendan
Taylor, who sits second to the right of the player, whose favourite player is Afghanistan. Brendan Taylor, who
plays forWarwickshire County Cricket Club, faces the immediate neighbour of Shaun Marsh. Shaun Marshplays
forWorcestershire County Cricket Club. Azhar Ali sits with the players who play for Warwickshire County Cricket
Club and Hampshire County Cricket Club. Chris Gayle faces the player who plays forDurham County Cricket Club
and Ireland player but Chris Gayle does not play for Lancashire County Cricket Club. The one who plays
forHampshire County Cricket Clubdoes not sit with the one who plays forDurham County Cricket Club. The player,
who plays forGlamorgan County Cricket Club, sits second to the right of the player who plays forNottinghamshire
County Cricket Club. Ben Stokes does not play for Durham County Cricket Club and faces the player, who plays
forSomerset County Cricket Club. The player who belongs to West Indies is an immediate neighbour of the player
who plays forKent County Cricket Club. Imran Tahir belongs to South Africa and he does not sit with Shaun
Marsh. Mushfiqur Rahimnot belongs to Pakistan. The player, who belongs to England, faces the player, who sits
second to the right of the player, who belongs to Australia. The playerwho belongs to New Zealand does not sit in
row 2 and faces the player who belongs to Zimbabwe. Mohammad Nabi faces the player who belongs to India.
Imran Tahir does not face the player who plays for Nottinghamshire County Cricket Club. The one who plays
forSurrey County Cricket Club sits in Row 2.

6. Which of the following players sits at the extreme ends of one row?
a) Ben Stokes and Azhar Ali
b) Mohammad Nabi and Mushfiqur Rahim
c) Rohit Sharma and Chris Gayle
d) Shaun Marsh and Lasith Malinga

www.ibpsguide.com | estore.ibpsguide.com | www.sscexamguide.com


4
Exclusively on New Pattern – Reasoning Ability eBook

e) Ben Stokes and Kevin O’Brien

7. Azhar Ali plays for which of the following team?


a) Durham County Cricket Club
b) Lancashire County Cricket Club
c) Worcestershire County Cricket Club
d) Nottinghamshire County Cricket Club
e) Kent County Cricket Club

8. Which of the following player belongs to Afghanistan?


a) Mohammad Nabi
b) Azhar Ali
c) Kevin O’Brien
d) Mushfiqur Rahim
e) Ben Stokes

9. The one who belongs to India sits third to the right of ______________
a) Shaun Marsh
b) Ross Taylor
c) Lasith Malinga
d) Chris Gayle
e) Imran Tahir

10. Which of the following combinations is true?


a) Chris Gayle – Somerset County Cricket Club - Bangladesh
b) Lasith Malinga – Nottinghamshire County Cricket Club – Sri Lanka
c) Mushfiqur Rahim – Glamorgan County Cricket Club – Australia
d) Ross Taylor – Essex County Cricket Club - New Zealand
e) None of these

Answers:
6. Answer: e)
7. Answer: d)
8. Answer: a)
9. Answer: c)
10. Answer: d)

www.ibpsguide.com | estore.ibpsguide.com | www.sscexamguide.com


5
Exclusively on New Pattern – Reasoning Ability eBook

Facing South Imran Tahir Rohit Shaun Marsh Ross Taylor Lasith Malinga Chris Gayle
(Somerset Sharma (Worcestershir (Essex (Kent County (Yorkshire
County (Lancashire e County County Cricket Club) County
Cricket Club) County Cricket Club) Cricket Club) (Sri Lanka) Cricket Club)
(South Africa) Cricket Club) (Australia) (New (West Indies)
(India) Zealand)
Facing North Ben Stokes Mohammad Azhar Ali Brendan Mushfiqur Kevin O’Brien
(Surrey Nabi (Nottinghamsh Taylor Rahim (Durham
County (Hampshire ire County (Warwickshire (Glamorgan County
Cricket Club) County Cricket Club) County County Cricket Cricket Club)
(England) Cricket Club) (Pakistan) Cricket Club) Club) (Ireland)
(Afghanistan (Zimbabwe) (Bangladesh)
)

Directions (Q. 11-15): Study the following information carefully and answer the given questions.
Aarthi, Suchitra, Gayathri, Haritha, Yazhini, Ritaj, Priya, Ilakiya, Shalini, Durga, Jyothi and Lakshmi are twelve
persons sitting in two rows, among them Aarthi, Gayathri, Haritha, Ritaj, Shalini and Jyothi are facing north while
the remaining are facing south. Each person faces exactly one person in the other row. Each one of them produce
different vegetables in home among Cabbage, Carrot, Brinjal, Onion, Beetroot, Capsicum, Cauliflower, Cucumber,
Beans, Peas, Pumpkin and Garlic (not necessarily in the same order) and each of them belongs to different cities
among Mumbai, Bangalore, Hyderabad, Kolkata, Jaipur, Chennai, Delhi, Kochi, Patna, Lucknow, Surat and
Shimla (again not necessarily in the same order).
 The one who producesBeetroot is from Chennai and is facing Ritaj
 None of Suchitra, Lakshmi and YazhiniproducesBeetroot or Cabbage
 The persons from Hyderabad and Shimla are the immediate neighbours of Durga
 Suchitra is second to the right of Priya, who is from Delhi
 Suchitraproduces Cauliflower and faces the person from Lucknow, who is third to the right of Haritha
 Neither Aarthi nor HarithaproducesCarrot or from Bangalore
 Jyothi, produces the Brinjal, sits second to the right of the person who producesPeas, who faces the person
from Shimla, None of them sits at an extreme end
 The Carrotproduce by the person from Surat, sits as far as possible from Haritha
 Lakshmi, produces the Capsicum, sits opposite the person who produces the Pumpkin, who is an immediate
neighbour of the person sitting at an extreme end
 The Cauliflower and Onionproducers are from Hyderabad and Mumbai respectively
 The person who producesCapsicum and Onion face the same direction
 Ritaj, produce the Cucumber and from Kolkata, sits equidistant in the same row who produces the Beans and
the person from Bangalore

www.ibpsguide.com | estore.ibpsguide.com | www.sscexamguide.com


6
Exclusively on New Pattern – Reasoning Ability eBook

 The person from Patna is opposite to the person from Mumbai and second to the left of the person from
Kolkata
 Aarthi, who does not produce the Pumpkin, is third to the left of Shalini; and neither of them produces Carrot
 Ilakiyaproduces the Garlic and is third to the right of person who producesCapsicum from Jaipur

11. The one who belongs to Jaipur sits third to the left of ______________
a) The one who belongs to Hyderabad
b) The one who belongs to Chennai
c) The one who produces Cabbage
d) The one who produces Cauliflower
e) The one who produces Onion

12. Who sits second to the left of Durga?


a) Yazhini
b) Suchitra
c) Ilakiya
d) Priya
e) Lakshmi

13. Which of the following person belongs to Lucknow?


a) Jyothi
b) Shalini
c) Durga
d) Aarthi
e) Lakshmi

14. Which of the following persons sits at the extreme ends of one row?
a) Yazhini and Suchitra
b) Aarthi and Gayathri
c) Haritha and Gayathri
d) Priya and Ilakiya
e) Ritaj and Haritha

15. Which of the following combinations is true?


a) Ritaj - Cucumber – Hyderabad
b) Ilakiya - Garlic - Shimla
c) Shalini - Peas - Bangalore
d) Lakshmi – Cauliflower - Jaipur

www.ibpsguide.com | estore.ibpsguide.com | www.sscexamguide.com


7
Exclusively on New Pattern – Reasoning Ability eBook

e) None of these

11. Answer: d)
12. Answer: e)
13. Answer: a)
14. Answer: c)
15. Answer: b)

Facing South Yazhini Ilakiya Durga Suchitra Lakshmi Priya


(Onion) (Garlic) (Beetroot) (Cauliflower) (Capsicum) (Cabbage)
(Mumbai) (Shimla) (Chennai) (Hyderabad) (Jaipur) (Delhi)
Facing North Haritha Aarthi Ritaj Jyothi Shalini Gayathri
(Beans) (Peas) (Cucumber) (Brinjal) (Pumpkin) (Carrot)
(Patna) (Kochi) (Kolkata) (Lucknow) (Bangalore) (Surat)

Directions (Q. 16-20): Study the following information carefully and answer the questions given below.
There are ten friends are sitting in twelve seats in two parallel rows containing five students on each, in such a
way that there is an equal distance between adjacent students. In row 1, Prakash, Vijay, Rajesh, Akram and
David are seated and all of them are facing south, and in row 2, Ganesh,Lenin, Wahid, Sharma and Bala are
sitting and all of them are facing north. One seat is vacant in each row. Therefore, in the given seating
arrangement each member seated in a row faces another member of the other row. All of them working in
different Arab countries i.e. Egypt, Iraq, Kuwait, Morocco, Oman, Palestine, Qatar, Saudi Arabia, UAE and Yemen
 Prakash sits second to left of one, whoworking inEgypt
 Either Prakash or the one, whoworking inEgypt, sits adjacent to the extreme end position
 Bala sits one of the extreme ends of the row
 There are three friends sit between Bala and Sharma, whoworking inIraq
 Immediate neighbour of Bala faces Vijay
 One of immediate neighbour of Vijay faces Wahid, whoworking inKuwait.
 There are no vacant seats adjacent to Wahid.
 Rajeshworking inMorocco and sits second to left of vacant seat
 One of immediate neighbour of Wahid is Lenin.
 Leninworking inOman
 One of immediate neighbour of Lenin faces Akram.
 Akramworking inPalestine
 The one whoworking inQatar sits immediate left of the one whoworking inSaudi Arabia
 The one whoworking inUAE sits third to left of one whoworking inYemen

www.ibpsguide.com | estore.ibpsguide.com | www.sscexamguide.com


8
Exclusively on New Pattern – Reasoning Ability eBook

16. Bala working in which of the following country?


a) UAE
b) Saudi Arabia
c) Egypt
d) Yemen
e) Palestine

17.How many people will sit between Rajesh and Akram?


a) One
b) Two
c) Three
d) Four
e) More than four

18.Which of the following person working in Qatar?


a) Vijay
b) Ganesh
c) David
d) Sharma
e) Akram

19.Which of the following statement is true?


a) Rajesh faces the person who is the immediate neighbour of Bala
b) There are two persons between Sharma and Bala
c) Prakash working in Egypt
d) Ganesh is adjacent to the vacant seat
e) The persons working in Palestine and Qatar are immediate neighbours

20. Which of the following statement is false?


a) Akram working in Palestine
b) David faces the person working in Iraq
c) The person working in Kuwait faces Vijay
d) There are two persons between vacant seat and Wahid
e) All the options are true

Answers:
16. Answer: d)
17. Answer: b)

www.ibpsguide.com | estore.ibpsguide.com | www.sscexamguide.com


9
Exclusively on New Pattern – Reasoning Ability eBook

18. Answer: a)
19. Answer: e)
20. Answer: c)

Facing South Vacant David (Egypt) Rajesh Prakash (Saudi Vijay (Qatar) Akram
(Morocco) Arabia) (Palestine)
Facing North Vacant Sharma (Iraq) Ganesh (UAE) Wahid (Kuwait) Lenin (Oman) Bala (Yemen)

Directions (Q. 21-25): Study the following information carefully and answer the given questions.
Ten Friends are sitting in two parallel rows of six seats each. One seat is vacant in each row. Arun,Victor, Vaasu,
Jerome and Prathap are sitting in row-1 facing south. Naresh, Krishnan, Sanjay, Mohan and Karthik are facing
north. Each of them have a different model bikes i.e. Bajaj Dominar 400, Royal Enfield Classic 350, Bajaj Pulsar
150, Suzuki Gixxer, Yamaha FZ S FI, TVS Apache RTR 200 4V, Honda CB Hornet 160R, Kawasaki Vulcan S,
KTM 1050 Adventure and TVS Akula 310. Mohan sits third to the right of Sanjay and haveSuzuki Gixxer. Only two
people sit between Krishnan and the vacant seat. Krishnan does not haveYamaha FZ S FI or Bajaj Pulsar 150.
Prathap is not an immediate neighbour of Vaasu. VictorhaveTVS Akula 310. The one who haveBajaj Pulsar
150faces the one who haveKawasaki Vulcan S. The one who haveBajaj Pulsar 150 sits opposite to the one who
sits third right of the person who sits opposite to Mohan. Vaasu is not an immediate neighbour of Jerome. Karthik,
who have neither Yamaha FZ S FInor TVS Apache RTR 200 4V, does not face the vacant seat. Neither Mohan
nor Sanjay sits at any of the extreme ends of the row. Jerome faces Sanjay. Vacant seats are not opposite to
each other. Two seats are there between Vaasu and Victor, who sits third right of the one who haveHonda CB
Hornet 160R. The one who haveKTM 1050 Adventure faces the one who haveSuzuki Gixxer. The friends who
have the Bajaj Dominar 400and Kawasaki Vulcan S are adjacent to each other. Vacant seat of row – 1 is not an
immediate neighbour of Jerome.Krishnan sits at one of the extreme ends of the row. Sanjay does not haveBajaj
Dominar 400and Kawasaki Vulcan S. Vacant seat of row-1 does not face Mohan.
21. Which of the following model bike Karthikhave?
a) TVS Akula 310
b) Suzuki Gixxer
c) Honda CB Hornet 160R
d) Royal Enfield Classic 350
e) KTM 1050 Adventure

22.Who among the following person haveTVS Apache RTR 200 4V bike?
a) Karthik
b) Mohan
c) Krishnan
www.ibpsguide.com | estore.ibpsguide.com | www.sscexamguide.com
10
Exclusively on New Pattern – Reasoning Ability eBook

d) Jerome
e) None of these

23.How many people will sit between Naresh and Krishnan?


a) One
b) Two
c) Three
d) Four
e) More than four

24.Four among the following form a group in a certain way. Which of the following does not belong to Group?
a) Prathap, TVS Akula 310
b) Naresh, Bajaj Pulsar 150
c) Krishnan, Yamaha FZ S FI
d) Victor, KTM 1050 Adventure
e) Karthik, Suzuki Gixxer

25.Which of the following is correctly matched?


a) Jerome – TVS Akula 310
b) Naresh – Yamaha FZ S FI
c) Karthik – Honda CB Hornet 160R
d) Krishnan – KTM 1050 Adventure
e) Sanjay – Suzuki Gixxer

Answers:
21. Answer: d)
22. Answer: c)
23. Answer: a)
24. Answer: e)
25. Answer: b)

Prathap Jerome Victor Arun Vaasu


Facing South (Bajaj (Kawasaki (TVS Akula - (KTM 1050 (Honda CB
Dominar 400 ) Vulcan S) 310) Adventure) Hornet 160R)
Karthik Sanjay Naresh Mohan Krishnan
Facing North (Royal Enfield (Bajaj Pulsar - (Yamaha FZ (Suzuki (TVS Apache
Classic 350) 150) S FI ) Gixxer) RTR 200 4V)

www.ibpsguide.com | estore.ibpsguide.com | www.sscexamguide.com


11
Exclusively on New Pattern – Reasoning Ability eBook

Directions (Q. 26-30): Study the following information carefully to answer the given questions:
There are seven Grand Prix playersNicoRosberg, Lewis Hamilton, Sebastian Vettel, Daniel Ricciardo,
ValtteriBottas, Max Verstappen and Fernando Alonso. They drive for different teams, won different grand prix and
were born in different countries and months. They all were born in the same year. One of the team is Renault
Sport. One of the Countries is USA. One of the grand prix is Australian Grand Prix. One of the persons was born
in the month of August. The following information is known about them.
 Max Verstappen was born in Belgium or in UK
 The one who was born in the month of April won Chinese Grand Prix and drive for Scuderia Toro Rossoteam
and is either Sebastian Vettel or Fernando Alonso
 Daniel Ricciardo is not the one who won Spanish Grand Prix
 The one who won Monaco Grand Prixdrive for Mobile Bullteam
 The one who was born in UK is the youngest one and is either Lewis Hamilton or Fernando Alonso
 The one who won Bahrain Grand Prix was born in Spain but is not Lewis Hamilton
 ValtteriBottas was born in the month of January
 Daniel Ricciardo was born in Australia in the month of June and drive for Mercedes AMG Petronasteam
 The one who won Russian Grand Prix was born inNovember but is not the youngest person
 Lewis Hamiltondrive for Sahara Force Indiateam
 The one who won Monaco Grand Prix was born in Finland but that person is not Sebastian Vettel who drive
for Mclaren Honda team
 The one who was born in Germanydrive for Scuderia Ferrariteam but it is not ValtteriBottas
 Max Verstappen is the second youngest person
 The one who won Canadian Grand Prix is one month younger than ValtteriBottas

26. Who was born in USA?


a) ValtteriBottas
b) NicoRosberg
c) Lewis Hamilton
d) Fernando Alonso
e) Sebastian Vettel

27. Who drive for Renault Sport team?


a) Max Verstappen
b) Fernando Alonso
c) NicoRosberg
d) Lewis Hamilton
e) Sebastian Vettel

www.ibpsguide.com | estore.ibpsguide.com | www.sscexamguide.com


12
Exclusively on New Pattern – Reasoning Ability eBook

28. Daniel Ricciardowon which of the following grand prix?


a) Russian Grand Prix
b) Australian Grand Prix
c) Chinese Grand Prix
d) Canadian Grand Prix
e) Bahrain Grand Prix

29. Which month Sebastian Vettel was born?


a) February
b) April
c) August
d) November
e) Cannot be determined

30. Which of the following statements is correct?


a) ValtteriBottaswon Australian Grand Prix Bank
b) NicoRosbergdrive for Scuderia Toro Rossoteam
c) Fernando Alonso was born in the month of August
d) Lewis Hamilton was born in Belgium
e) None of these

Answers:
26. Answer: d)
27. Answer: a)
28. Answer: b)
29. Answer: c)
30. Answer: e)

Persons Grand Prix Month Team Countries


NicoRosberg Canadian Grand Prix February Scuderia Ferrari Germany
Lewis Hamilton Spanish Grand Prix December Sahara Force India UK
Sebastian Vettel Bahrain Grand Prix August Mclaren Honda Spain
Daniel Ricciardo Australian Grand Prix June Mercedes AMG Petronas Australia
ValtteriBottas Monaco Grand Prix January Mobile Bull Finland
Max Verstappen Russian Grand Prix November Renault Sport Belgium
Fernando Alonso Chinese Grand Prix April Scuderia Toro Rosso USA

www.ibpsguide.com | estore.ibpsguide.com | www.sscexamguide.com


13
Exclusively on New Pattern – Reasoning Ability eBook

Directions (Q. 31-35): Study the following information carefully to answer the given questions:
There are seven peoples – Hanif, Vikram, Praveen, Bharat, Vishal, Devanand Sunil –who purchased in different e
commerce sites they are- Amazon, Flipkart, Snapdeal, Jabong, Shopclues, Ebay and Paytm but not necessarily in
the same order. They ordered the product on different days of the week starting from Monday. Also, they order
different products, viz- Mobile, Watch, Shirt, Books, Laptop, Memory Card and Pen drive but not necessarily in the
same order.
 Vikram, who orders Memory Card, he ordered on fourth day of the week but he purchased on neither Amazon
nor Paytm site
 Two peoplesordered the products between the days on which Vikramand Devanordered the products and
neither of them orders 1st day of the week
 There is only one peopleorder the product between Hanifand Praveen, But Praveen does not order either on
1st or 3rd day of the week.
 Praveen order Watchand he purchased inAmazon
 The one who purchased in Snapdealorders on the last day of the week and he ordered Shirt
 Bharat does not order on the day either immediately before or immediately after the day on which Hanif order
the product, Hanif does not order either Laptop or Mobile or Books
 Sunil orders the product immediately after Praveen and he ordered Mobile
 Vishaldoes not order the Laptopand hepurchased in Ebay
 Hanif’sdoes not purchased in neither Shopclues nor Jabong
 The one who purchased in Flipkartorder the product immediately after the day who purchased the product in
Amazon

31.The person who orders Mobile ordered the mobile on which of the following days?
a) Wednesday
b) Saturday
c) Thursday
d) Monday
e) Sunday

32. Which of the following product does Vishal order?


a) Books
b) Pen drive
c) Mobile
d) Shirt
e) Watch

33. Which of the following person ordered the product on Monday?


a) Praveen

www.ibpsguide.com | estore.ibpsguide.com | www.sscexamguide.com


14
Exclusively on New Pattern – Reasoning Ability eBook

b) Devan
c) Hanif
d) Sunil
e) Bharat

34. If ‘Laptop’ is related to ‘Paytm’, ‘Pen drive‘ is related to ‘Amazon’, then which following is ‘Watch’ related to?
a) Flipkart
b) Jabong
c) Snapdeal
d) Ebay
e) None of these

35. Which of the following combinations is true?


a)Sunday – Sunil - Mobile - Flipkart
b) Friday – Praveen – Memory Card - Amazon
c) Sunday – Vikram – Shirt - Snapdeal
d) Wednesday – Hanif - Pen drive - Paytm
e)Monday – Vishal – Books - Ebay

Answers:
31. Answer: b)
32. Answer: a)
33. Answer: e)
34. Answer: c)
35. Answer: d)

Days People Product E commerce site


Monday Bharat Laptop Jabong / Shopclues
Tuesday Vishal Books Ebay
Wednesday Hanif Pen drive Paytm
Thursday Vikram Memory Card Jabong / Shopclues
Friday Praveen Watch Amazon
Saturday Sunil Mobile Flipkart
Sunday Devan Shirt Snapdeal

www.ibpsguide.com | estore.ibpsguide.com | www.sscexamguide.com


15
Exclusively on New Pattern – Reasoning Ability eBook

Directions (Q. 36-40): Study the following information carefully to answer the given questions:
In Year 2016, seven personsA, B, C, D, E, F and Geach person saved some money on different months of the
year starting from January and ending on November. Some of them are females and some of them are male.
Each person saves money on each month only. Their contributions,(in rupees) are – 5000, 500, 40, 1600, 80, 40
and 2500.
 Fsaved money in the month of June.
 Two persons saved money between savings of F and A
 On the first month of the year, a male saved 5000 rupees.
 D did not saved money on a month after the saving of B.
 None of the given person saved his money in a month which has 28 days.
 Two successive months which has 31 days,in one month the savingis double that ofanother month.
 Thesaving of Rs 500 was made in November month.C saved money in the month which has least number of
days.
 The sum of savings of G and B is equal to that of D.
 Amount saved by a female is always preceded and succeeded by the savings of a male.
 E is the only female who cannot be succeeded by any one.
 A is the only male which is followed by another male.
 Only one person saved the money between the savings ofE and D.
 The savings of the male who saved his money on June is the perfect square of the saving made on August.
 The savings of E is not made on the month immediately before or immediately after the month when Fsaved
money (considering only those month in which the donations were made).
 Csaved money immediately before G.
 Only three persons saved their money in a month which has 31 days.

36. Which of the following person saved 2500 rupees?


a) B
b) C
c) E
d) A
e) D

37.The person who saved 80 rupees, saved on which of the following month?
a) November
b) January
c) July
d) August
e) June

www.ibpsguide.com | estore.ibpsguide.com | www.sscexamguide.com


16
Exclusively on New Pattern – Reasoning Ability eBook

38. The person who saved money on August, Saved ________


a) 40
b) 80
c) 500
d) 2500
e) 1600

39. Four among the following form a group in a certain way. Which of the following does not belong to Group?
a) A
b) F
c) C
d) B
e) D

40. Which of the following combinations is true?


a)November – E – Male - 500
b) April – G – Female - 1600
c) August – B – Male - 2500
d) July – D - Female - 80
e) January – A - Female - 5000

Answer:
36. Answer: b)
37. Answer: c)
38. Answer: a)
39. Answer: e)
40. Answer: d)

Months Person Money Saved


January A (Male) 5000
February C (Male) 2500
April G (Female) 40
June F (Male) 1600
July D (Female) 80
August B (Male) 40
November E (Female) 500

www.ibpsguide.com | estore.ibpsguide.com | www.sscexamguide.com


17
Exclusively on New Pattern – Reasoning Ability eBook

Directions (Q. 41-45): Study the following information carefully to answer the given questions:
There are seven public sector Banksnamely Andra Bank, Canara Bank, Dena Bank, Syndicate Bank, Vijaya
Bank, Punjab National Bank and Bank of Barodalocated in different statessuch as Kerala, Gujarat, Karnataka,
Jharkhand, Maharashtra, Bihar andRajasthan, not necessarily in the same order. Each bank have different
number of ATM’s such as, 500, 700, 900 and 1000in different state. Depends on the number of customers in bank
it was listed from 1 to 7 positions, from top to bottom. The following information is known about them.
 Not more than threeBanks has samenumber of ATM’s.
 There's only one Bank has 1000 ATM’s which belongs to Maharashtra and listed in bottom 3 positions and is
not Dena BankBank
 The bank which belongs to Kerala listed just ahead of Canara Bank, which does not belong to Maharashtra.
 The Bank in which has700 ATM’s presented is listed 1st.
 The Banks which listed in 3rd and 4th have same number of ATM’s and it is not 500.
 The Bank located in Gujarat listed in last and Bank located in Bihar listed as 4th and none of them are has
700 ATM’s.
 Punjab National BankBankhas500ATM’s and is not in bottom 3.
 Syndicate Bank belongs to Jharkhand and Bank of BarodaBank belongs to Kerala, both are not having
500ATM’s and in the list there are two Banks between them.
 The Banks from Rajasthan and Gujarat are havingsame number of ATM’s.
 Vijaya Bank not located in Karnataka, Gujarat or Maharashtra.

41.Vijaya bank have how many ATM’s?


a) 500
b) 900
c) 1000
d) Either 700 or 500
e) Either 700 or 900

42. Which of the following Bank listed on 3rd place from the top?
a) Andra Bank
b) Dena Bank
c) Syndicate Bank
d) Canara Bank
e) Bank of Baroda

43. Punjab National Bank located in______________


a) Rajasthan
b) Gujarat

www.ibpsguide.com | estore.ibpsguide.com | www.sscexamguide.com


18
Exclusively on New Pattern – Reasoning Ability eBook

c) Maharashtra
d) Bihar
e) Karnataka

44. If ‘Punjab National Bank’ is related to ‘Canara Bank’, ‘Syndicate Bank‘ is related to ‘Vijaya Bank’, then which
following Bankis ‘Dena Bank’ related to?
a) Vijaya Bank
b) Canara Bank
c) Andra Bank
d) Bank of Baroda
e) Punjab National Bank

45. Which of the following combinations is true?


a)3 – Syndicate Bank – Jharkhand - 500
b) 1 – Punjab National Bank – Karnataka -700
c) 2 - Andra Bank – Maharashtra - 1000
d) 7 – Canara Bank – Gujarat - 700
e) 4 – Vijaya Bank – Bihar - 900

Answers:
41. Answer: b)
42. Answer: c)
43. Answer: a)
44. Answer: d)
45. Answer: e)

Position Banks Name States Number of ATM’s


1 Dena Bank Karnataka 700
2 Punjab National Bank Rajasthan 500
3 Syndicate Bank Jharkhand 900
4 Vijaya Bank Bihar 900
5 Andra Bank Maharashtra 1000
6 Bank of Baroda Kerala 700 or 900
7 Canara Bank Gujarat 500

www.ibpsguide.com | estore.ibpsguide.com | www.sscexamguide.com


19
Exclusively on New Pattern – Reasoning Ability eBook

Directions (Q. 46-50): Study the following information carefully to answer the given questions:
There are twelve students – Harish, Naren, Maran, Dinesh, Chandran, Jadhav, Yokesh, Sarath, Gowtham,
Arafath, Vigneshand Gopi. They have exams in different months – January, March, April, June, August, and
October. The exams are on dates 9th or 23rd of a given month. Each student got different number of marks in
exams. If the month contains odd number of days then the marks got by the student is odd. Like exam in March
(31 days) the student got odd number of marks and exam in April (30 days) the student got even number of
marks.
 Harish had exam on June 9. Sarathexam was before Harish.
 Sarath got 55 marks in exam and he had exam on 23rd of a month.
 There was one exam between Sarathand Arafathexam.
 The mark got by the Arafath was 6 less than the Sarath’sexam marks.
 There were threeexams between Dinesh’sexam and the one who got 26 marks in exam.
 Dineshhad examon one of the months before Harish exam
 Dineshexamwas not in January.
 There were twoexams between Vigneshexam and theone who scored 26 marks in exam.
 The marks scored by Vignesh is higher than Arafath and less than Sarath
 The mark scored by Vignesh in exam was a multiple of 3.
 Chandranand Yokeshhad exam in same month but not April.
 There was one exam between the exams of Chandranand Gopi.
 There were twoexams between Jadhavexam and the one who scored 16 marks in exam
 Jadhavhad exam before the one who scored 16 marks in exam.
 The total number of marks scored by Gopiand Sarathwas twice the number of marks scored by Harish and
Naren.
 Exam of Maranand the one who scored 18 marks was in same month.
 Total marks scored by Chandran and Yokesh in exam were 20.
 The marks scored by Chandran in exam was greater than the marks scored by Jadhav and Yokesh together
 The marks scored by Vignesh was greater than the marks scored by Naren by 24
 Maran scored 38 marks in exam
 The one who had exam in March scored more mark than Naren but less than 30
 There were more than threeexams between the exams of Gowthamand Arafath

46.Which of the following students wrote the exam on June month?


a) Chandran and Vignesh
b) Dinesh and Sarath
c) Naren and Gowtham
d) Harish and Gowtham
e) Gopi and Yokesh

www.ibpsguide.com | estore.ibpsguide.com | www.sscexamguide.com


20
Exclusively on New Pattern – Reasoning Ability eBook

47. Which of the following month Jadhav and Maran wrote the exam?
a) October
b) April
c) March
d) June
e) August
48. Which of the following student not wrote the exam on the day 9th?
a) Gowtham
b) Dinesh
c) Gopi
d) Harish
e) Naren
49. Which of the following statement is false?
a) Two students attend exam between Dinesh and Maran
b) Harish scored 26 marks in the exam
c) Chandran and Yokesh wrote the exam on different months and same dates
d) The sum of marks scored by Harish and Sarath is a perfect square
e) All the statements are false
50. Which of the following combinations is true?
a)Dinesh and Sarath wrote the exam on January
b) Maran exam is next to Harish exam
c) Two students attend the exams between Gowtham and Vignesh
d) Yokesh and Naren have exam on 23rd of different months
e) Total marks scored by Gopi and Arafath is 100
Answers:
46. Answer: d)
47. Answer: b)
48. Answer: a)
49. Answer: c)
50. Answer: e)
Month 9th 23rd
January Naren(27) Arafath(49)
March Dinesh(29) Sarath(55)
April Jadhav(18) Maran(38)
June Harish (26) Gowtham(16)
August Gopi(51) Vignesh(51)
October Chandran(19) Yokesh(1)

www.ibpsguide.com | estore.ibpsguide.com | www.sscexamguide.com


21
Exclusively on New Pattern – Reasoning Ability eBook

Directions (Q. 51-55): Study the following information carefully to answer the given questions:
Nine people Madhan, Roshan, Balram, Murali, Gandhi, Krishna, Jairam, Praveen and Nitish stay in a building.
The building has nine floors and only one person stays on one floor. Each of them working in different banks
namely- Bank of India, Canara Bank, Allahabad Bank,Bank of Baroda, Corporation Bank, Indian Bank, Union
Bank of India, Punjab National Bank, and Central Bank of India. Each person belongs to different states, i.e.
Andra Pradesh, Tamil Nadu, Kerala, Maharashtra, Karnataka, Rajasthan, Haryana, Punjab and Gujarat, but not
necessarily in the same order. The ground floor is numbered 1, the floor above it is numbered 2, and so on, and
the topmost floor is numbered 9.
 The one who belongs to Punjab stays on the fourth floor.
 Madhan does not belong to Rajasthan and does not working in Bank of India and Allahabad Bank.
 There are three floors between the floors on which Balram and Jairam stay.
 Murali stays on a floor immediately above the Nitish’s floor.
 The one who working inCanara Bank stays on an even numbered floor
 Krishna does not belong to Gujarat
 The one who belongs to Andra Pradesh stays on the topmost floor.
 The one who working inUnion Bank of India stays immediate below to the one who working inBank of Baroda
 Krishnaworking inBank of India and does not stay on the ground floor.
 Praveen belongs to Maharashtra and stays on an even-numbered floor and he working inUnion Bank of India.
 Gandhi stays on the second floor and belongs to Kerala.
 There are three people between the one who working inCorporation Bank and the one who working
inAllahabad Bank.
 The one who working inCorporation Bank stays below the person who working inAllahabad Bank
 The one who belongs to Gujarat stays on the third floor
 The one who working inPunjab National Bank does not stay on sixth floor
 There are two floors between the floors on which the people who are from Haryana and Maharashtra stay
 The person who working inAllahabad Bank is from Haryana
 Balram belongs to Tamil Nadu
 The one who working inCentral Bank of India stays immediate above to Balram
 There is one floor between the floors on which Krishna and Jairam stay
 There is one floor between the floors in which the one who working inIndian Bank and the one who working
inCentral Bank of India stay
 Madhan stays on an even-numbered floor below the floor on which Praveen stays.
51. Who among the following exactly sitting between Krishna and Roshan?
a) The one who working in Union Bank of India
b) Praveen
c) The one who lives in 8th floor
d) The one who is from Maharashtra
e) All of the above

www.ibpsguide.com | estore.ibpsguide.com | www.sscexamguide.com


22
Exclusively on New Pattern – Reasoning Ability eBook

52. How many people live between the one who working inPunjab National Bank and the one who working
inUnion Bank of India?
a) Two
b) Three
c) Four
d) Five
e) Six

53. Which of the following person working in Allahabad Bank?


a) Jairam
b) Madhan
c) Nitish
d) Gandhi
e) Roshan

54. If ‘Bank of Baroda’ is related to ‘Rajasthan’, ‘Canara Bank’ is related to ‘Punjab’, then which following is
‘Punjab National Bank’ related to?
a) Karnataka
b) Tamil Nadu
c) Kerala
d) Gujarat
e) Haryana

55. Which of the following combinations is true?


a)5 – Maharashtra – Praveen - Union Bank of India
b) 4 – Punjab – Krishna - Indian Bank
c) 2 – Kerala – Balram - Central Bank of India
d) 6 – Karnataka – Madhan - Canara Bank
e) None of these

Answers:
51. Answer: e)
52. Answer: c)
53. Answer: a)
54. Answer: b)
55. Answer: d)

www.ibpsguide.com | estore.ibpsguide.com | www.sscexamguide.com


23
Exclusively on New Pattern – Reasoning Ability eBook

Floor Person State Banks


9 Roshan Andra Pradesh Bank of Baroda
8 Praveen Maharashtra Union Bank of India
7 Krishna Rajasthan Bank of India
6 Madhan Karnataka Canara Bank
5 Jairam Haryana Allahabad Bank
4 Murali Punjab Indian Bank
3 Nitish Gujarat Punjab National Bank
2 Gandhi Kerala Central Bank of India
1 Balram Tamil Nadu Corporation Bank

Directions (Q. 56-60): Study the following information carefully to answer the given questions:
Eight persons stayed in a building (The
groundfloorisnumbered1,theflooraboveitisnumbered2,andsoon,andthetopmostfloorisnumbered8)from different IT
companies such as TCS, Infosys, Wipro, HCL, Tech Mahindra, Oracle, Mindtree and Mphasiseach of them have
training on different cities such as Mumbai, Hyderabad, Bangalore, Kochi, Chennai, New Delhi ,Kolkata and
Bhubaneswar . And each of them going to training by different mode of transports such asBus, Train, Car and
Flight
 The person who is from Infosyshave training inBhubaneswar
 The person whohave training inKolkatausingFlightas a transport
 The third floor stayed person have training inHyderabad and usingBus as a transport
 The person who have training in Bangalore using Train as a transport
 Minimum of three persons stayed below the person who is from Mphasis. And the person who belongs to HCL
not stayed in the odd numbered floor
 The persons using Train and Flight as transport have same number of users
 The person from Wipro stayed two floor below from the person who is belongs to Infosys
 The top floor is not occupied by the person from Mindtree
 The person using Car as a transport have training inMumbai
 The persons from HCL and Infosysusing same mode of transports and both of them not usingTrainas a
transport and also three persons stayed in between them
 The persons who have training inKochi and Hyderabad are using same mode of transport with the one
whohave training inChennai
 The person using Caras a transport stayed in fourth floor
 There are four persons stayed in between of person from Mindtree and person from HCL
 The lowest floor is occupied by the person from Tech Mahindra
 Only three persons using one samemode of transport and the person from Mindtree whohave training
inChennai is one of them

www.ibpsguide.com | estore.ibpsguide.com | www.sscexamguide.com


24
Exclusively on New Pattern – Reasoning Ability eBook

 The person from Oracle and the person who have training inChennai are not stayed in the even numbered
floor
 Only one person usingCaras a transport
 The person from TCShave training in New Delhi stayed in even numbered floor and is using same mode of
transport with the person, who is have training inBangalore also stayed in first floor
 Flighttransport is used by the person who is from HCL
 The person who have training inBhubaneswar stayed above the floor of the person who have training
inKolkata
 The person from Oracle who have training inHyderabadusing same mode of transport with the person from
Mphasis

56. Who lives in the sixth floor?


a) The one who is from Infosys
b) The one who is from HCL
c) The one who is from Mindtree
d) The one who is from TCS
e) The one who is from Mphasis

57. How many persons using Bus as a transport?


a) One
b) Two
c) Three
d) Four
e) Five

58.Which of the following company person have training in Chennai?


a) Wipro
b) Infosys
c) HCL
d) TCS
e) Mindtree

59. Which of the following person sits exactly between who have training inMumbai and Bhubaneswar?
a) The one who have training in Hyderabad
b) The one who have training in Kolkata
c) The one who have training in New Delhi
d) The one who have training in Kochi
e) The one who have training in Bangalore

www.ibpsguide.com | estore.ibpsguide.com | www.sscexamguide.com


25
Exclusively on New Pattern – Reasoning Ability eBook

60. Which of the following combinations is true?


a) 2 – Bangalore – Flight - Tech Mahindra
b) 4 - Mumbai - Car - Wipro
c) 8 – Mumbai – Train - Tech Mahindra
d) 7 - Hyderabad – Bus - TCS
e) None of these

Answers:
56. Answer: a)
57. Answer: c)
58. Answer: e)
59. Answer: d)
60. Answer: b)

Floors Company Cities Transport Mode


8 TCS New Delhi Train
7 Mindtree Chennai Bus
6 Infosys Bhubaneswar Flight
5 Mphasis Kochi Bus
4 Wipro Mumbai Car
3 Oracle Hyderabad Bus
2 HCL Kolkata Flight
1 Tech Mahindra Bangalore Train

Directions (Q. 61-65): Study the following information carefully to answer the given questions:
Nine people Ganesh, Sadiq, Raman, Arun, Dharish, Madhavan, Vinoth, Thomas and Akshay stay in a building.
The building has nine floors and only one person stays on one floor. Each of them likes different sports namely-
Cricket, Football, Hockey, Tennis, Badminton, Golf, Rugby, Table Tennis and Basketball. Each person belongs to
different countries, i.e. Australia, China, Denmark, Japan, USA, Russia, India, Spain and Germany but not
necessarily in the same order. The ground floor is numbered 1, the floor above it is numbered 2, and so on, and
the topmost floor is numbered 9.
The one who belongs to Spain stays on the 4th floor. There are 4 floors between the floors on which Arun&Vinoth
stay with Arun living above Vinoth. Raman stays on the floor immediately above Thomas's floor. The one who
likes Cricket stays on an even numbered floor and is a neighbour of the one who likes Football. Akshay does not

www.ibpsguide.com | estore.ibpsguide.com | www.sscexamguide.com


26
Exclusively on New Pattern – Reasoning Ability eBook

belong to China. The one who belongs to Australia stays on the topmost floor. The one who likes Rugby stays
immediately below the one who likes Badminton. Dharish likes Table Tennis. Madhavan belongs to USA and
stays on 8th floor. There are 3 persons between the one who likes Tennis and the one who like Table Tennis.
There are two floors between the floors on which people from Germany and Spain stay. Dharish stays
immediately below the floor on which the person from Japan lives. The person who likes Golf belongs to China
and stays on an odd numbered floor. Akshay stays on the bottommost floor. The person who likes Hockey is from
Japan and stays on 6th floor. The person from India stays on 2nd floor. The persons from Russia and China are
not the neighbours of the person from Japan. Arun likes Football and Vinoth does not like Cricket. Sadiq does not
live below the floor of Ganesh.

61. Which of the following persons sits exactly between Akshay and the one who likes Basketball?
a) Thomas and Vinoth
b) Raman and Dharish
c) Vinoth and Arun
d) Ganesh and Thomas
e) Sadiq and Dharish

62. Who lives in the Third floor?


a) Dharish
b) The one who likes Basketball
c) The one who is from China
d) Madhavan
e) None of these

63. Hockey like by which of the following person?


a) Arun
b) Ganesh
c) Vinoth
d) Akshay
e) Raman

64. How many persons sit between Vinoth and the one who likes Tennis?
a) Five
b) Three
c) Four
d) Two
e) Six

www.ibpsguide.com | estore.ibpsguide.com | www.sscexamguide.com


27
Exclusively on New Pattern – Reasoning Ability eBook

65. Which of the following combinations is true?


a) 4 - Spain - Raman - Tennis
b) 3 –China – Vinoth - Badminton
c) 8 - Australia – Sadiq - Hockey
d) 7 - Germany - Arun - Football
e) None of these

Answers:
61. Answer: a)
62. Answer: c)
63. Answer: b)
64. Answer: e)
65. Answer: d)

Floor Names Country Sports


9 Sadiq Australia Tennis
8 Madhavan USA Cricket
7 Arun Germany Football
6 Ganesh Japan Hockey
5 Dharish Denmark Table Tennis
4 Raman Spain Basketball
3 Thomas China Golf
2 Vinoth India Badminton
1 Akshay Russia Rugby

Directions (Q. 66-70): Study the following information carefully to answer the given questions:
There are twelve students – Rohit, Mahesh, Mohan, Sanjay, Lakshman, Yusuf, Ajay, Baskar, Saravanan, Hemant,
Gandhi and Raman who live in a building with 5 floors (lowest floornumbered 1 to topmost floor 5). There are 3
flats on each floor from left to right numbered 1 to 3. Immediate right of flat 2 means flat 3. Flat above flat means
same flat number of above floors. There are 3 flats empty in this building. There can be only 1 flat empty on any
floor. Also same flat number cannot be empty on any of the floors i.e. if Flat number 1 is empty on floor number 1,
than flat 1 of any other floor cannot be empty. They all bought magazines for different price in a shop. They all buy
different magazines – India Today, Businessworld, Frontline, Outlook, Dataquest, Business Today, The Week,
PCQuest, Autocar, Digit, Sportstar, and Wisdom.
 Gandhi lives on flat number 1 of floor number 3.
 One who buyFrontlinemagazine lives somewhere in the flat which is above Gandhi’s flat

www.ibpsguide.com | estore.ibpsguide.com | www.sscexamguide.com


28
Exclusively on New Pattern – Reasoning Ability eBook

 One person lives between the one who buyFrontlinemagazine and Ajay who has bought magazinefor 96
rupees
 Rohit buyOutlookmagazine who lives just below Ajay’s flat
 One of the flats on which the floor Rohitlives is empty but that is not immediate left of Rohit’s flat
 The one who buyDigitmagazine lives on the flat 2nd to right of Gandhi
 There are 2 floors between Baskar who bought magazinefor 72 rupees and the one who bought magazine for
85 rupees
 Baskar lives above the one who bought magazinefor 85 rupees. Baskar does not buy Frontlinemagazine
 Hemant buySportstarmagazine and lives immediate right of the one who has bought magazine for 85 rupees.
Raman and Hemant live on same floor
 Raman buyBusinessworldmagazine and has bought magazine for 9 rupees more than Yusuf
 There are 2 flats between Raman and an empty flat
 Yusuf and the one who buyDataquestmagazine lives on the same floor which is either floor number 1 or 2.
One flat is empty on that floor
 There is one empty flat between the one who bought magazinefor 61 rupees and the one who buyIndia
Todaymagazine
 The one who bought magazinefor 61 rupees is above the one who buyIndia Todaymagazine
 The one who has bought magazinefor 55 rupees lives on floor number 1
 Yusuf has bought magazinefor 25 rupees less than Sanjay who does not live on top floor. Mohan and empty
flat are on same floor
 Saravanan lives on one of the flats on the floor on which Ajay lives
 The one who buyWisdommagazine lives immediate right of Saravanan
 The one who buyPCQuestmagazine lives just above the one who buyAutocarmagazine
 The one who buyThe Weekmagazine lives below the one who buyAutocarmagazine such that no one lives
between them
 Lakshman and Yusuf lives on same floor
 Lakshman has bought magazinefor 15 rupees more than Rohit
 Mohan lives on a floor on which one flat is empty
 One of the students on same floor as one buying Autocarmagazine has bought magazinefor 50% of price
obtained by Ajay
 The one who bought magazinefor 82 rupees lives on one of the floors above the floor of one who bought
magazinefor 90 rupees

66. Gandhi buys magazine for how much rupees?


a) 63
b) 61
c) 76
d) 96

www.ibpsguide.com | estore.ibpsguide.com | www.sscexamguide.com


29
Exclusively on New Pattern – Reasoning Ability eBook

e) 55

67. Sanjay buys which of the following magazines?


a) Wisdom
b) India Today
c) Digit
d) Business Today
e) Business world

68. How many persons live between Ajay and Hemant in the third flat?
a) One
b) Two
c) Three
d) None
e) Cannot be determined

69. Which of the following statement is false?


a) Raman buys Business world magazine for 69 rupees
b) Saravanan buys magazine for 82 rupees
c) Sportstar magazine is bought by Ajay
d) Rohit buys Outlook magazine
e) All the statements are true

70. Which of the following combinations is true?


a)Yusuf buys Dataquest magazine
b) In floor number 2 all three flats are occupied
c) Person who buy India Today magazine buy for 55 rupees
d) Baskar living in first flat and buy Autocar magazine
e) Mahesh buy magazine for 61 rupees

Answers:
66. Answer: a)
67. Answer: d)
68. Answer: b)
69. Answer: c)
70. Answer: e)

www.ibpsguide.com | estore.ibpsguide.com | www.sscexamguide.com


30
Exclusively on New Pattern – Reasoning Ability eBook

Floors First Second Third


5 Mahesh Saravanan Ajay
4 - Baskar Rohit
3 Gandhi - Mohan
2 Lakshman Yusuf -
1 Raman Sanjay Hemant

Persons Magazines Magazine Price


Rohit Outlook 48
Mahesh Frontline 61
Mohan Digit 76
Sanjay Business Today 85
Lakshman Dataquest 90
Yusuf The Week 60
Ajay Wisdom 96
Baskar Autocar 72
Saravanan PCQuest 82
Hemant Sportstar 55
Gandhi India Today 63
Raman Businessworld 69

Directions (Q. 71-75): Study the following information carefully to answer the given questions:
Nine people Gayathri, Nivetha, Nithya, Pavithra, Reena, Latha, Anjana, Mahima and Suvetha stay in a building
(floors numbered 1 to 9).
Thegroundfloorisnumbered1,theflooraboveitisnumbered2,andsoon, and the topmostfloorisnumbered9.They are
using different versions of Android– Lollipop version and Kitkat version. Four of them are usingKitkat version and
remaining persons are usingLollipop version. All of them have different mobiles viz – Samsung, HTC, Apple,
Microsoft, Sony, Motorola, Vivo, Redmi and Oppo, but not necessarily in the same order. Each of them also uses
different sims– Airtel, Idea, Reliance, Aircel, Docomo, BSNL, Vodafone, MTS and Reliance Jio, again but not
necessarily in the same order.

 There is one floor between the floors on which Latha, who usesIdeasim and Anjana stay

www.ibpsguide.com | estore.ibpsguide.com | www.sscexamguide.com


31
Exclusively on New Pattern – Reasoning Ability eBook

 Pavithra, who usesBSNL sim, stays on a floor immediately above Suvetha’s floor, who usesVodafonesim
 Neither Gayathri nor Nithyahave Sony mobile.
 Nithya does not have Samsungmobile and usesMTS sim
 The one who have Vivomobile stays on the fourth floor but is not usingLollipop version
 There are two floors between the floors on which the people who have Motorolamobile and Microsoft mobile
stay
 Reena stays on the second floor and have Apple mobile
 There are three floors between the floor on which Nithya, who is usingKitkat version and Anjana stay, who
does not have Oppomobile and usesReliance sim
 The person who have Sony mobile and Samsungmobile are using same android version.
 Gayathri is usingLollipop version and lives on an even numbered floor which is below the floor on which
Mahima stays, who usesAircel sim
 The one who have Oppomobile stays on the third floor and is not usingKitkat version
 The one who have Samsung mobile stays on the topmost floor and usesAirtel sim
 The person have Motorola mobile and Microsoftmobile are using same android version
 Mahimahave Microsoftmobile and is usingLollipop version and lives on an even numbered floor
 The person hasRedmimobile does not live below the person who have HTC mobile.
 Gayathri does not useReliance Jio sim.

71. Which of the following mobile does Pavithra have?


a) Samsung
b) Oppo
c) Motorola
d) Sony
e) Vivo
72. Which of the following person have Redmi mobile?
a) Anjana
b) Pavithra
c) Nithya
d) Gayathri
e) Reena
73. Mahima lives on which of the following floor?
a) 8th floor
b) 6th floor
c) 4th floor
d) 2nd floor
e) 3rd floor
74. Four among the following form a group in a certain way. Which of the following does not belong to Group?

www.ibpsguide.com | estore.ibpsguide.com | www.sscexamguide.com


32
Exclusively on New Pattern – Reasoning Ability eBook

a) Nivetha
b) Gayathri
c) Pavithra
d) Nithya
e) Latha

75. Which of the following combinations is true?


a)3 - Suvetha - Oppo – Idea - Lollipop version
b) 9 - Nivetha - Samsung - Airtel – Lollipop version
c) 7 - Latha – Sony - Idea - Kitkat version
d) 5 – Gayathri - Motorola - Reliance - Lollipop version
e) 2 - Reena – Sony - Reliance Jio - Lollipop version

Answer:
71. Answer: e)
72. Answer: d)
73. Answer: a)
74. Answer: b)
75. Answer: c)

Floor Person Mobiles Sim Android Version


9 Nivetha Samsung Airtel Kitkat version
8 Mahima Microsoft Aircel Lollipop version
7 Latha Sony Idea Kitkat version
6 Gayathri Redmi Docomo Lollipop version
5 Anjana Motorola Reliance Lollipop version
4 Pavithra Vivo BSNL Kitkat version
3 Suvetha Oppo Vodafone Lollipop version
2 Reena Apple Reliance Jio Lollipop version
1 Nithya HTC MTS Kitkat version

Directions (Q. 76-80): Study the following information carefully to answer the given questions:
Ten persons Maalini, Haasini, Geetha, Neelam, Abinaya, Janani, Nisha, Saritha, Bhavanaand Divyaare sitting
around a rectangular table and likes a different foods i.e. MughlaiBiryani, LucknowiBiryani, HyderabadiBiryani,
AmburBiryani, Kolkata Biryani, KacchiBiryani, Bombay Biryani, Malabar Biryani, Afghani Biryani and
BhatkaliBiryani but not necessarily in the same order. All of them are sitting at a rectangular table in such a way
that four of them sit at the corners, two each on the longer sides and one each on the smaller sides, but not

www.ibpsguide.com | estore.ibpsguide.com | www.sscexamguide.com


33
Exclusively on New Pattern – Reasoning Ability eBook

necessarily in the same order. Some of them are facing the centre while the rest are not facing the centre. Not
more than two friends sitting together face the same direction.
Abinayasits on the immediate left of Neelamand is not an immediate neighbour of Geetha. Maaliniand
Abinayaface in the same direction. The one who likes MughlaiBiryani sits immediate left of the one who likes
KacchiBiryani. Neelamand Nishasit diagonally and face the opposite directions. Five of them face in the same
directions. Sarithadoes not like KacchiBiryani and Afghani Biryani. The one who likes Malabar Biryani sits
immediate right of Bhavana, who likes Afghani Biryani. The one who likes KacchiBiryani sits second to the left of
Geetha. Only two among four sitting on the corners face outward. Sarithaand Neelamare sitting on the immediate
left and third to the left of Divyarespectively. Maalinisits on one of the smaller sides and third to the right of Janani.
Neelamlikes Bombay Biryani and sits third to the left of the one who likes Kolkata Biryani. Divyasits immediate
right of the one who likes BhatkaliBiryani. Abinayalikes LucknowiBiryani and sits second to the left of the one who
likes AmburBiryani. Bhavanais not an immediate neighbour of Maalini, Haasinior Janani, but sits on the immediate
right of Geetha, who is not facing the centre. One of the four friends sitting on the corner is Bhavana.
Bhavanafaces to the centre.

76. Divya likes which of the following Biryani?


a) AmburBiryani
b) Afghani Biryani
c) Bombay Biryani
d) KacchiBiryani
e) Kolkata Biryani

77. Which of the following person diagonally opposite to Maalini?


a) Janani
b) Neelam
c) Abinaya
d) Haasini
e) None of these

78. Nisha sits third to the right of ______________


a) Haasini
b) Bhavana
c) Divya
d) Saritha
e) None of these

79. Four among the following form a group in a certain way. Which of the following does not belong to Group?
a) Nisha

www.ibpsguide.com | estore.ibpsguide.com | www.sscexamguide.com


34
Exclusively on New Pattern – Reasoning Ability eBook

b) Janani
c) Neelam
d) Divya
e) Geetha

80. Which of the following statement is true?


a) Maalini likes Lucknowibriyani
b) Abinaya sits second to the right of Janani
c) Saritha faces outside and likes Hyderabad biryani
d) Geetha likes Malabar biryani and sits second to the left of Nisha
e) None of these

Answers:
76. Answer: e)
77. Answer: c)
78. Answer: b)
79. Answer: a)
80. Answer: d)
Q (76-80)

www.ibpsguide.com | estore.ibpsguide.com | www.sscexamguide.com


35
Exclusively on New Pattern – Reasoning Ability eBook

Directions (Q. 81-85): Study the following information carefully to answer the given questions:
Eight people Rahaman, Kavin, Niranjan, Ramesh, Arvind, Karan, Venkat and Shankarsit around a circular table
facing the centre. They likes different juices Mango juice, Apple juice, Grapes juice, Orange juice, Papaya juice,
Pomegranate juice, Pineapple juice and Banana juice. They belong to different ages youngest being 17 and
oldest being 64 years old.
 One who likes Banana juice is as much older than Niranjan, who likes Orange juice, as the age of Rahaman
 Difference of ages of Ramesh and Kavin is 41
 One who likes Pomegranate Juice sits second to the right to one who likes Mango juice
 One who likes Orange juice sits second to the left to the one who likes Banana juice
 Age of Arvind is 36 years. Rahaman is 3rd youngest among all the people
 Two people sit between Karan and the one whose age is 35
 One who likes Pineapple juice and the one who likes Banana juice sit adjacent to each other and Ramesh is
not likes Pineapple juice
 Shankar is the youngest of them and Ramesh is not the oldest
 One who likes Grapes juice is opposite to one whose age is 47
 One person sits between Kavin, who likes Papaya juice, and Arvind
 Karan sits third to the left of the person who likes Pomegranate juice who is the oldest person among them
 Rahaman is sitting third to the left of the one who likes Grapes juice
 Sum of ages of Kavin and Shankar is equal to the age of Niranjan
 One whose age is 35 sits 3rd right of the one whose age is 18
 The age of Karan is average of ages of Niranjan and Ramesh
 One who likes Papaya juice and Pomegranate juice are sitting adjacent to each other
 One person sits between Karan and the one who likes Pineapple juice

81. Shankar likes which of the following juice?


a) Apple juice
b) Pomegranate juice
c) Orange juice
d) Mango juice
e) Pineapple juice

82. Which of the following persons likes Grapes juice?


a) Arvind
b) Venkat
c) Kavin
d) Niranjan
e) Karan

www.ibpsguide.com | estore.ibpsguide.com | www.sscexamguide.com


36
Exclusively on New Pattern – Reasoning Ability eBook

83. Who sits third to the left of the one who likes Orange juice?
a) Rahaman
b) Kavin
c) Venkat
d) Arvind
e) Karan

84. What is the age of Karan?


a) 18
b) 36
c) 47
d) 24
e) 35

85. Which of the following combinations is true?


a) Ramesh – Pineapple juice - 59
b) Niranjan – Orange juice - 36
c) Rahaman – Apple juice - 24
d) Venkat – Pomegranate juice - 64
e) None of these

Answers:
81. Answer: e)
82. Answer: a)
83. Answer: b)
84. Answer: c)
85. Answer: d)
Q (81-85)

www.ibpsguide.com | estore.ibpsguide.com | www.sscexamguide.com


37
Exclusively on New Pattern – Reasoning Ability eBook

Directions (Q. 86-90): Study the following information carefully to answer the given questions:
Kumar, Dinesh, Saran, Vinay, Udaya, Siva, Mukeshand Ashok are eight friends sitting around a circular table.
Four of them are facing away from the centre and four of them are facing towards the centre. Each of them like
different watches -Titan, Citizen, Timex, Casio, Guess, Speed, Fastrack and Fossil. All of them are having a
different companymobile viz. Samsung, Sony, Nokia, Motorola, Vivo, Redmi, Oppo and Apple but not necessarily
in the same order.
Udayafaces towards the centre and havingSamsungmobile. Both the immediate neighbours of Udayaface away
from the centre and are havingeither Nokia orMotorolamobile. Vinayfaces away from the centre and his
favouritewatch is Titan. Both the immediate neighbours of Vinaydo not face away from the centre. Udayasits third
to the right of Siva, who having a Vivomobile and faces away from the centre. Saran sits third to the left of Siva.
The one who is having Nokiamobile sits opposite to Siva. The one who ishaving Sonymobile is not the immediate
neighbour of Siva and faces away from the centre. Kumar sits second to the left of Saran and he having neither
Oppo nor Applemobile. The one who ishaving aOppomobile sits between Ashok and Siva. Dineshfaces away
from the centre, likes Citizenwatchand does not have a Sonymobile. Udaya’sfavourite watch is Fastrack. The
person who likes Casiowatch sits opposite to Vinay. The person having Redmimobile likes Timex watch. The
person who likes Fossil watch is not near to MukeshnorUdaya. Saran faces the person who likes Speed watch.

86. Who sits third to the left of the one who have Sony mobile?
a) The one who have Redmi mobile
b) The one who have Oppo mobile
c) The one who likes Citizen watch
d) The one who have Samsung mobile
e) The one who likes Casio watch

87. The one who likes Fossil watch likes which of the following mobile?
a) Sony
www.ibpsguide.com | estore.ibpsguide.com | www.sscexamguide.com
38
Exclusively on New Pattern – Reasoning Ability eBook

b) Vivo
c) Apple
d) Oppo
e) Samsung

88. Which of the following persons likes Speed watch?


a) Mukesh
b) Kumar
c) Vinay
d) Dinesh
e) Ashok

89. Saran likes which of the followingwatch?


a) Titan
b)Timex
c) Casio
d) Fastrack
e)Fossil

90. Which of the following statement is true?


a) Udayahave Nokia mobile
b) Kumar sits third to the left of the person who have Motorola mobile
c) Dinesh sits second to the left of Ashok
d) Siva likes Citizen watch
e) None of these

Answers:
86. Answer: d)
87. Answer: c)
88. Answer: a)
89. Answer: e)
90. Answer: b)
Q (86-90)

www.ibpsguide.com | estore.ibpsguide.com | www.sscexamguide.com


39
Exclusively on New Pattern – Reasoning Ability eBook

Directions (Q. 91-95): Study the following information carefully to answer the given questions:
M, N, O, P, Q, R, S and T are eight family members sitting around a circular table in a restaurant. All of them are
facing the centre. Three of the family members are selected in SSC exam and others are selected in IBPS exam.
All the eight family members are Government employees. They are working in different states Kerala,
Maharashtra, Haryana, Punjab, Rajasthan, Karnataka, Gujarat and Odisha, but not necessarily in the same order.
There are two married couples, three brothers, two daughters, one granddaughter and one grandson in the group.
Two of the female members in the group M and Tare selected in same exam. They selected in different exams,
they are RRB Clerk, RRB PO, IBPS PO, SSC CGL, IBPS SO, SSC CHSL and IBPS Clerk.
S, works in Kerala, is the head of the family, and is immediate neighbour of T and P. The granddaughter R is
sitting third to the left of S. The grandmother of R is sitting on the immediate left of her. R was selected in IBPS
exam. Q is sitting second to the right of his father S. He works in Rajasthan, and selected in IBPS Clerk exam.
Only S’s daughter is sitting exactly between M and Q. She selected in IBPS SO exam and she works in
Karnataka. T works in Haryana and N works in Punjab. The son of T is sitting on the immediate left of his father.
His father works in Maharashtra. O works in Karnataka and is sitting on the immediate left of her mother. She
selected in IBPS SO exam. P, the husband of the one who works in Haryana, selected in SSC CHSLexam and
his mother selected in SSC CGL exam. The one who works in Punjab sits second to the right of M, who is an
immediate neighbour of the one who works in Karnataka and the Gujarat. The one who works in Gujarat does not
selected in RRB Clerk exam and the one who works in Kerala does not selected IBPS PO or RRB PO exam.
91. The person who selected in SSC CGL exam is working in which of the following state?
a) Odisha
b) Punjab
c) Haryana
d) Both Odisha and Haryana
e) Gujarat
92. The only person who is sitting between R and O is working on which of the following state?
www.ibpsguide.com | estore.ibpsguide.com | www.sscexamguide.com
40
Exclusively on New Pattern – Reasoning Ability eBook

a) Maharashtra
b) Punjab
c) Kerala
d) Haryana
e) Odisha
93. Who among the following working in Gujarat?
a) R
b) P
c) O
d) M
e) None of these
94. Who among the following is selected in RRB Clerk exam?
a) R
b) S
c) M
d) N
e) None of these
95. Which of the following is correctly matched?
a) N – Punjab – IBPS PO
b) Q – Odisha – IBPS Clerk
c) P – Maharashtra - SSC CHSL
d) O – Karnataka – SSC CHSL
e) None of these
Answer:
91. Answer: d)
92. Answer: e)
93. Answer: a)
94. Answer: b)
95. Answer: c)
Q (91-95)

www.ibpsguide.com | estore.ibpsguide.com | www.sscexamguide.com


41
Exclusively on New Pattern – Reasoning Ability eBook

Directions (Q. 96-100): Study the following information carefully to answer the given questions:
There are sixteen persons
Deepak,Basha,Lalit,Prasad,Eshwar,Charan,Zakir,Wahab,Abdul,Gopal,Irfan,Subash,Rana,Yadav,Nirmal and
Vishnu parking their vehicles in a square shed. Inside a Square shed, another square shaped shed for bikeis
developed. The persons who are parking inside the shed facing outside.The persons who are parking outside the
shed facing inside the centre and have different cars namely viz., – Scorpio, Swift, Alto, Verna, Wagon R, Bolero,
Dzire and Tiago. So all the persons parking in the inner square faces the persons parking in the outer Square and
the persons parking inside the shed have bikes namely viz., – Splendor bike, Yamaha bike, Pulsar bike, Duke
bike, Hornet bike, Glamour bike, Shine bike and Apache bike.
 Charan faces the centre and Vishnu faces Charan
 Lalit parks second to the right of Charan
 There are four persons parks between Charanand Prasad
 Lalitis not an immediate neighbour of Prasad
 There are three persons parking vehicles between Wahab and Prasad
 There are two persons parking vehicles between Wahab and Deepak
 Deepak parks vehicle exactly between the Prasad and Eshwar
 Eshwarparks vehicle to the immediate left of Charan
 There are two persons parking vehicles between Vishnu and Yadav
 Yadavfaces Zakir. Ranafaces outside
 There are two persons parking vehicles between Ranaand Gopal
 Ranafaces Basha. Gopalparks vehicle to the immediate left of Vishnu
 Irfan, the one who faces Deepak parks vehicle exactly between the persons Abdul and Nirmal
 Abdul faces Prasad. The one who sits in the corner of the square likes Scorpio.
 The one who have Scorpioparks between the persons who have Alto and Swift
 The one who have Swiftparks second to the right of the person who have Bolero
 Three persons park their vehicle between one who haveAlto and one who haveBolero

www.ibpsguide.com | estore.ibpsguide.com | www.sscexamguide.com


42
Exclusively on New Pattern – Reasoning Ability eBook

 Two persons park the vehicles between one who haveAlto and one who haveWagon R
 Two persons sit between one who haveWagon R and one who haveTiago
 Wahab andEshwar do not have Dzire and Wagon R respectively
 The one who haveScorpio faces Abdul
 The immediate neighbours of Abdul are the one who have Splendor bike and the one who haveDuke bike
 The one who have Splendor bike faces the one who have Alto
 Three persons parktheir vehicle between the one who have Splendor bike and the one who haveShine bike
 The immediate neighbours of the person who haveYamaha bike are the one who haveSplendor bike and the
one who have Glamour bike
 The one who have Hornet bikeparks exactly behind to the one who have Yamaha bike
 The one who have Apache bikeparks exactly behind to the one who have Pulsar bike
 The one who have Apache bike faces Prasad
96. Vishnu have which of the following bike?
a) Apache bike
b) Glamour bike
c) Hornet bike
d) Pulsar bike
e) None of these
97.Wahabhave which of the following car?
a) Alto
b) Wagon R
c) Scorpio
d) Swift
e) Verna
98. Subash faces which of the following person?
a) The one who have Bolero car
b) Basha
c) The one who have Tiago car
d) Eshwar
e) None of these
99. Which of the following statement is false?
a) Prasad have Scorpio car and parks in the corner
b) Vishnu park the bike on the immediate left of Nirmal
c) Irfan have Apache bike
d) Lalit parks his vehicle between Zakir and Wahab
e) None of these
100. Which of the following statement is true?
a) Deepak have Swift car

www.ibpsguide.com | estore.ibpsguide.com | www.sscexamguide.com


43
Exclusively on New Pattern – Reasoning Ability eBook

b) Gopal sits second to the left of Yadav


c) Rana faces the person who have Wagon R car
d) Eshwar faces the person who have Yamaha bike
e) None of these
Answers:
96. Answer: b)
97. Answer: e)
98. Answer: a)
99. Answer: c)
100. Answer: d)
Q (96-100)

Syllogism

Direction (101-200): In the following questions, only one Conclusion is given and five statements are given as a),
b), c), d) and e). From this you have to take the statements to be true even if they seem to be at variance with
commonly known facts and then decide which of the given statement logically follows.

101. Conclusions: Some cyclones are thunders.


Some thunders are clouds is a possibility Some thunders are lightning.
Some storms are thunders b) Statements:
Some lightning are storms is a possibility All clouds are storms.
Some lightning are clouds is a possibility Some storms are cyclones.
a) Statements: Some cyclones are thunders.
All clouds are storms. All thunders are lightning.
Some storms are cyclones. c) Statements:

www.ibpsguide.com | estore.ibpsguide.com | www.sscexamguide.com


44
Exclusively on New Pattern – Reasoning Ability eBook

All clouds are storms. Some locks are handles


Some storms are cyclones. Some locks are keys.
All cyclones are thunders. e) Statements:
Some thunders are lightning. All pins are needles.
d) Statements: All needles are handles.
All clouds are storms. No handles is locks.
All storms are cyclones. All locks are keys.
Some cyclones are thunders.
Some thunders are lightning. 103. Conclusions:
e) Statements: Some dams are lakes
All clouds are storms. Some lakes are mountains is a possibility
No storms are cyclones. Some dams are hills
No cyclones are thunders. Some rivers are hills is a possibility
Some thunders are lightning. a) Statements:
All hills are mountains.
102. Conclusions: All mountains are dams.
Some keys are pins is a possibility Some dams are rivers.
Some locks are handles Some rivers are lakes.
Some needles are locks is a possibility b) Statements:
Some needles are pins All hills are mountains.
a) Statements: Some mountains are dams.
Some pins are needles. Some dams are rivers.
Some needles are handles. All rivers are lakes.
Some handles are locks. c) Statements:
Some locks are keys. All hills are mountains.
b) Statements: All mountains are dams.
Some pins are needles. All dams are rivers.
All needles are handles. Some rivers are lakes.
Some handles are not locks. d) Statements:
All locks are keys. All hills are mountains.
c) Statements: All mountains are dams.
No pins are needles. Some dams are rivers.
Some needles are handles. All rivers are lakes.
Some handles are locks. e) Statements:
Some locks are keys. All hills are mountains.
d) Statements: No mountains are dams.
Some pins are needles. Some dams are rivers.
Some needles are handles. All rivers are lakes.

www.ibpsguide.com | estore.ibpsguide.com | www.sscexamguide.com


45
Exclusively on New Pattern – Reasoning Ability eBook

Some bottles are jars.


104. Conclusions: Some jars are containers.
Some papers are files. Some containers are lids.
Some files are receipts is a possibility Some lids are caps.
Some books are drafts is a possibility b) Statements:
Some books are papers All bottles are jars.
a) Statements: Some jars are containers.
Some receipts are drafts. All containers are lids.
Alldrafts are papers. All lids are caps.
All papers are books. c) Statements:
Some books are files. All bottles are jars.
b) Statements: All jars are containers.
Some receipts are drafts. All containers are lids.
Some drafts are papers. No lids are caps.
Some papers are books. d) Statements:
All books are files. All bottles are jars.
c) Statements: No jars are containers.
Some receipts are drafts. All containers are lids.
Some drafts are papers. All lids are caps.
All papers are books. e) Statements:
No books are files. All bottles are jars.
d) Statements: All jars are containers.
Some receipts are drafts. All containers are lids.
Some drafts are papers. All lids are caps.
No papers are books.
All books are files. 106. Conclusions:
e) Statements: Some stems are fruit is a possibility
Some receipts are drafts. Some branches are not flower
No drafts are papers. Some leaves are not fruit
No papers are books. Some stems are leaves is a possibility
All books are files. a) Statements:
Some leaves are flowers.
105. Conclusions: Some flower is fruit.
All bottles are lids. Some fruits are branches.
Some containers are bottles Some branches are stems.
Some caps are lids b) Statements:
Some lids are jars. Some leaves are flowers.
a) Statements: No flower is fruit.

www.ibpsguide.com | estore.ibpsguide.com | www.sscexamguide.com


46
Exclusively on New Pattern – Reasoning Ability eBook

Some fruits are branches. d) Statements:


Some branches are stems. All lions are tigers.
c) Statements: All tigers are leopards.
Some leaves are flowers. All leopards are wolves.
No flower is fruit. No wolf is elephant.
Some fruits are branches. e) Statements:
All branches are stems. Some lions are tigers.
d) Statements: Some tigers are leopards.
Some leaves are flowers. Some leopards are wolves.
All flowersis fruit. Some wolf is elephant.
All fruits are branches.
All branches are stems. 108. Conclusions:
e) Statements: Some trousers are umbrellas
No leaves are flowers. Some trousers are caps is a possibility
No flower is fruit. All raincoats are jackets
No fruits are branches. Some jackets are umbrellas
No branches are stems. a) Statements:
Some caps are umbrellas.
107. Conclusions: Some umbrellas are raincoats.
Some leopards are lions Some raincoats are trousers.
Some elephants are lion is a possibility All trousers are jackets.
Some leopards are not elephants b) Statements:
Some wolves are tiger is a possibility Some caps are umbrellas.
a) Statements: Some umbrellas are raincoats.
All lions are tigers. All raincoats are trousers.
Some tigers are leopards. Some trousers are jackets.
Some leopards are wolves. c) Statements:
All wolfs is elephant. Some caps are umbrellas.
b) Statements: All umbrellas are raincoats.
All lions are tigers. All raincoats are trousers.
Some tigers are leopards. Some trousers are jackets.
Some leopards are wolves. d) Statements:
No wolf is elephant. All caps are umbrellas.
c) Statements: All umbrellas are raincoats.
All lions are tigers. Some raincoats are trousers.
All tigers are leopards. All trousers are jackets.
Some leopards are wolves. e) Statements:
No wolf is elephant. Some caps are umbrellas.

www.ibpsguide.com | estore.ibpsguide.com | www.sscexamguide.com


47
Exclusively on New Pattern – Reasoning Ability eBook

Some umbrellas are raincoats. Some erasers are keys is a possibility


All raincoats are trousers. Some staplers are keys
All trousers are jackets. a) Statements:
All keys are staplers.
109. Conclusions: Some staplers are blades.
Some televisions are machines Some blades are erasers.
Some machines are fans is a possibility Some erasers are sharpeners.
Some computers are machines b) Statements:
Some televisions are coolers is a possibility No keys are staplers.
a) Statements: All staplers are blades.
Some fans are coolers. Some blades are erasers.
Some coolers are machines. Some erasers are sharpeners.
Some machines are computers. c) Statements:
All computers are televisions. Some keys are staplers.
b) Statements: All staplers are blades.
Some fans are coolers. All blades are erasers.
All coolers are machines. Some erasers are sharpeners.
Some computers are machines. d) Statements:
All computers are televisions. All keys are staplers.
c) Statements: All staplers are blades.
Some fans are coolers. Some blades are erasers.
Some coolers are machines. Some erasers are sharpeners.
No machines are computers. e) Statements:
All computers are televisions. All keys are staplers.
d) Statements: Some staplers are blades.
Some fans are coolers. Some blades are erasers.
Some coolers are machines. All erasers are sharpeners.
Some machines are computers.
Some computers are televisions. 111.Conclusions:
e) Statements: Some oranges are grapes
Some fans are coolers. Some apples are grapes is a possibility
All coolers are machines. All plums are guavas is a possibility
Some machines are not computers. Some guavas are oranges is a possibility
All computers are televisions. a) Statements:
All grapes are plums.
110.Conclusions: Some plums are oranges.
Some sharpeners are blades is a possibility Some oranges are apples.
All keys are blades Some apples are guavas.

www.ibpsguide.com | estore.ibpsguide.com | www.sscexamguide.com


48
Exclusively on New Pattern – Reasoning Ability eBook

b) Statements: No dog is cat.


All grapes are plums. All dogs are rats.
Some plums are not oranges. No rat is goat.
Some oranges are apples. d) Statements:
All apples are guavas. Some pets are dogs.
c) Statements: Some dogs are cats.
All grapes are plums. Some dogs are rats.
All plums are oranges. No rat is goat.
Some oranges are apples. e) Statements:
Some apples are guavas. Some pets are dogs.
d) Statements: No dogs are cats.
No grapes are plums. No dogs are rats.
All plums are oranges. No rat is goat.
All oranges are apples.
All apples are guavas. 113.Conclusions:
e) Statements: All lilies are sunflowers.
Some grapes are plums. Some jasmines are marigolds.
Some plums are oranges. Some marigolds are roses is a possibility
Some oranges are apples. Some jasmines are sunflowers.
Some apples are guavas. a) Statements:
Some roses are jasmines.
112.Conclusions: Some jasmines are lilies.
Some dogs are not goats All lilies are marigolds.
Some cats are pets is a possibility Some marigolds are sunflowers.
Some rats are cats is a possibility b) Statements:
Some rats are pets is a possibility Some roses are jasmines.
a) Statements: Some jasmines are lilies.
Some pets are dogs. Some lilies are marigolds.
Some dogs are cats. All marigolds are sunflowers.
Some dogs are rats. c) Statements:
Some rat is goat. All roses are jasmines.
b) Statements: All jasmines are lilies.
Some pets are dogs. Some lilies are marigolds.
All dogs are cats. All marigolds are sunflowers.
No dogs israt. d) Statements:
No rat is goat. Some roses are jasmines.
c) Statements: Some jasmines are lilies.
Some pets are dogs. Some lilies are marigolds.

www.ibpsguide.com | estore.ibpsguide.com | www.sscexamguide.com


49
Exclusively on New Pattern – Reasoning Ability eBook

Some marigolds are sunflowers. 115.Conclusions:


e) Statements: Some desserts are carrots is a possibility
Some roses are jasmines. Some sweets are not chilly
Some jasmines are lilies. Some sweets are lemons is a possibility
All lilies are marigolds. Some lemons are not carrot
All marigolds are sunflowers. a) Statements:
Some lemons are chillies.
114.Conclusions: No chilly is carrot.
Some restaurants are hotels No carrotis sweet.
Some hotels are flats is a possibility All sweets are desserts.
Some houses are not hotels b) Statements:
Some restaurants are not bungalow Some lemons are chillies.
a) Statements: No chilly is carrot.
Some flats are houses. All carrots are sweets.
Some houses are bungalows. Some sweets are desserts.
No bungalow is hotel. c) Statements:
All hotels are restaurants. Some lemons are chillies.
b) Statements: Some chilly is carrot.
Some flats are houses. All carrots are sweets.
Some houses are bungalows. Some sweets are desserts.
No bungalow is hotel. d) Statements:
No hotels are restaurants. Some lemons are chillies.
c) Statements: No chilly is carrot.
Some flats are houses. No carrots are sweets.
Some houses are bungalows. Some sweets are desserts.
Some bungalow is hotel. e) Statements:
All hotels are restaurants. Some lemons are chillies.
d) Statements: No chilly is carrot.
All flats are houses. No carrots are sweets.
All houses are bungalows. No sweets are desserts.
All bungalowsis hotel.
All hotels are restaurants. 116.Conclusions:
e) Statements: Some sticks are scales.
Some flats are houses. Some sticks are not weight
Nohouses are bungalows. Some canes are pens is a possibility
No bungalow is hotel. No cane is weight.
No hotels are restaurants. a) Statements:
Some pens are sticks.

www.ibpsguide.com | estore.ibpsguide.com | www.sscexamguide.com


50
Exclusively on New Pattern – Reasoning Ability eBook

Some sticks are canes. All bags are sacks.


Some canes are scales. c) Statements:
No scale is weight. Some folders are boxes.
b) Statements: No boxes are bags.
Some pens are sticks. All bags are containers.
Some sticks are canes. Some bags are sacks.
All canes are scales. d) Statements:
Some scale is weight. Some folders are boxes.
c) Statements: Some boxes are bags.
Some pens are sticks. All bags are containers.
Some sticks are canes. Some bags are sacks.
All canes are scales. e) Statements:
All scale is weight. No folders are boxes.
d) Statements: Some boxes are bags.
Some pens are sticks. All bags are containers.
No stick is cane. Some bags are sacks.
All canes are scales.
No scale is weight. 118.Conclusions:
e) Statements: Some insects are birds.
Some pens are sticks. Some animals are not birds
Some sticks are canes. No pests are amphibians.
All canes are scales. Some insects are not amphibian
No scale is weight.
a) Statements:
117.Conclusions: Some insects are pests.
Some boxes are containers. Some pests are birds.
Some containers are folders is a possibility No bird is amphibian.
Some sacks are containers. All amphibians are animals.
Some boxes are folders b) Statements:
a) Statements: Some insects are pests.
Some folders are boxes. All pests are birds.
Some boxes are bags. All bird is amphibian.
Some bags are containers. All amphibians are animals.
Some bags are sacks. c) Statements:
b) Statements: Some insects are pests.
Some boxes are folders. All pests are birds.
All boxes are bags. No bird is amphibian.
All bags are containers. All amphibians are animals.

www.ibpsguide.com | estore.ibpsguide.com | www.sscexamguide.com


51
Exclusively on New Pattern – Reasoning Ability eBook

d) Statements: All colours are solutions.


Some insects are pests. All solutions are liquids.
All pests are birds. No liquid is solid.
Some bird is amphibian.
All amphibians are animals. 120.Conclusions:
e) Statements: Some floors are locks is a possibility
Some insects are pests. Some windows are keys is a possibility
Some pests are birds. Some doors are locks
Some bird is amphibian. Some floors are doors is a possibility
Someamphibians are animals.
a) Statements:
119.Conclusions: All locks are keys.
Some solutions are paints All keys are doors.
Some liquids are paints is a possibility Some doors are windows.
Some solids are solutions Some windows are floors.
Some solids are colours is a possibility b) Statements:
a) Statements: All locks are keys.
Some paints are colours. Some keys are doors.
Some colours are solutions. All doors are windows.
Some solutions are liquids. All windows are floors.
All liquids are solids. c) Statements:
b) Statements: All locks are keys.
Some paints are colours. Some keys are doors.
All colours are solutions. All doors are windows.
Some solutions are liquids. Some windows are floors.
All liquids are solids. d) Statements:
c) Statements: All locks are keys.
Some paints are colours. No keys are doors.
All colours are solutions. Some doors are windows.
Some solutions are liquids. Some windows are floors.
Some liquids are solids. e) Statements:
d) Statements: No lock is key.
Some paints are not colours. All keys are doors.
All colours are solutions. Some doors are windows.
All solutions are liquids. All windows are floors.
All liquids are solids.
e) Statements: 121.Conclusions:
Some paints are colours. Some poems are not movies

www.ibpsguide.com | estore.ibpsguide.com | www.sscexamguide.com


52
Exclusively on New Pattern – Reasoning Ability eBook

Some stories are novels is a possibility All suits are trousers.


Some novels are books Some trousers are shirts.
Some poems are books is a possibility All shirts are coats.
b) Statements:
a) Statements: Some bags are suits.
All books are novels. All suits are trousers.
Some novels are poems. Some trousers are shirts.
Some poems are stories. Some shirts are coats.
Some story is a movie. c) Statements:
b) Statements: Some bags are suits.
Some books are not novels. Some suits are trousers.
Some novels are poems. Some trousers are shirts.
All poems are stories. All shirts are coats.
No story is a movie. d) Statements:
c) Statements: All bags are suits.
All books are novels. All suits are trousers.
All novels are poems. All trousers are shirts.
No poem is story. Some shirts are coats.
No story is a movie. e) Statements:
d) Statements: Some bags are not suits.
All books are novels. All suits are trousers.
Some novels are poems. All trousers are shirts.
Some poems are stories. All shirts are coats.
No story is a movie.
e) Statements: 123. Conclusions:
No books are novels. Some juices are vitamins.
Some novels are poems. Some proteins are not fruits
Some poems are stories. Some vitamins are flowers is a possibility
No story is a movie. No flower is juice.

122.Conclusions: a) Statements:
Some trousers are coats. All flowers are fruits.
Some shirts are bags is a possibility No fruit is juice.
Some bags are trousers. Some juices are proteins.
Some coats are suits is a possibility Some proteins are vitamins.
b) Statements:
a) Statements: All flowers are fruits.
Some bags are suits. No fruit is juice.

www.ibpsguide.com | estore.ibpsguide.com | www.sscexamguide.com


53
Exclusively on New Pattern – Reasoning Ability eBook

Some juices are proteins. No flat is house.


All proteins are vitamins. d) Statements:
c) Statements: Some towers are pillars.
All flowers are fruits. Some pillars are buildings.
All fruit is juice. Some buildings are flats.
All juices are proteins. No flat is house.
All proteins are vitamins. e) Statements:
d) Statements: Some towers are pillars.
No flowers are fruits. No pillar is building.
No fruit is juice. All buildings are flats.
Some juices are proteins. No flat is house.
All proteins are vitamins.
e) Statements: 125.Conclusions:
All flowers are fruits. Some shoes are not blankets
No fruit is juice. No towel is blanket.
No juices are proteins. Some shoes are towels.
All proteins are vitamins. Some shoes are bed sheets.

124.Conclusions: a) Statements:
No building is house. Some shoes are socks.
Some flats are towers is a possibility All socks are towels.
Some pillars are flats All towels are bedsheets.
Some pillars are not houses All bedsheetsis blanket.
b) Statements:
a) Statements: Some shoes are socks.
Some towers are pillars. All socks are towels.
Some pillars are buildings. No towels are bed sheets.
All buildings are flats. No bed sheet is blanket.
All flat is house. c) Statements:
b) Statements: Some shoes are socks.
Some towers are pillars. Some socks are towels.
Some pillars are buildings. All towels are bed sheets.
No buildings are flats. No bed sheet is blanket.
No flat is house. d) Statements:
c) Statements: Some shoes are socks.
Some towers are pillars. Some socks are towels.
Some pillars are buildings. Some towels are bed sheets.
All buildings are flats. Some bed sheet is blanket.

www.ibpsguide.com | estore.ibpsguide.com | www.sscexamguide.com


54
Exclusively on New Pattern – Reasoning Ability eBook

e) Statements: Allguitar is a flute.


Some shoes are socks. e) Statements:
All socks are towels. Some pianos are violins.
All towels are bed sheets. Some violins are drums.
No bed sheet is blanket. No drums are guitars.
No guitar is a flute.
Direction: In the following questions, only one
Conclusion is given and five statements are given as 127.Conclusions:
a), b), c), d) and e). From this you have to take the Some hills are boulders is a possibility
statements to be true even if they seem to be at Some mountains are rocks is a possibility
variance with commonly known facts and then Some hills are rocks is a possibility
decide which of the given statement logically follows. Some boulders are stones.

126.Conclusions: a) Statements:
No drum is a flute Some stones are rocks.
Some guitars are violins is a possibility Some rocks are boulders.
Some drums are pianos is a possibility Some boulders are mountains.
Some flute are pianos is a possibility All hills are mountains.
b) Statements:
a) Statements: All stones are rocks.
Some pianos are violins. Some rocks are boulders.
Some violins are drums. Some boulders are mountains.
No drums are guitars. All hills are mountains.
No guitar is a flute. c) Statements:
b) Statements: Some stones are rocks.
Some pianos are violins. No rocks are boulders.
No violins are drums. Some boulders are mountains.
All drums are guitars. All hills are mountains.
All guitarsare flute. d) Statements:
c) Statements: No stones are rocks.
Some pianos are violins. All rocks are boulders.
Some violins are drums. Some boulders are mountains.
All drums are guitars. All hills are mountains.
No guitar is a flute. e) Statements:
d) Statements: Some stones are rocks.
Some pianos are violins. All rocks are boulders.
Some violins are drums. Some boulders are mountains.
Some drums are guitars. All hills are mountains.

www.ibpsguide.com | estore.ibpsguide.com | www.sscexamguide.com


55
Exclusively on New Pattern – Reasoning Ability eBook

128. Conclusions: a) Statements:


Some woods are plastics is a possibility Some airplanes are helicopters.
Some metals are glasses is apossibility All helicopters are gliders.
Some cloths are woods Some gliders are kites.
Some clothes are glasses is a possibility No kites are balloons.
b) Statements:
a) Statements: Some airplanes are helicopters.
All plastics are glasses. Some helicopters are gliders.
Some glasses are wood. All gliders are kites.
Some woods are metals. All kites are balloons.
All metals are cloths. c) Statements:
b) Statements: Some airplanes are helicopters.
All plastics are glasses. All helicopters are gliders.
Some glasses are wood. All gliders are kites.
Some woods are metals. Some kites are balloons.
Some metals are cloths. d) Statements:
c) Statements: Some airplanes are helicopters.
All plastics are glasses. All helicopters are gliders.
All glasses are wood. All gliders are kites.
No woods are metals. All kites are balloons.
All metals are cloths. e) Statements:
d) Statements: Some airplanes are helicopters.
All plastics are glasses. Some helicopters are gliders.
No glasses are wood. Some gliders are kites.
Some woods are metals. Some kites are balloons.
All metals are cloths.
e) Statements: 130. Conclusions:
All plastics are glasses. Some warriors are sentries.
Some glasses are wood. Some soldiers are kings is a possibility.
Some woods are metals. Some warriors are soldiers
No metals are cloths. Some sentries are soldiers is a possibility

129. Conclusions: a) Statements:


All helicopters are balloons. Some kings are warriors.
Some balloons are gliders. Some soldiers are warriors.
All helicopters are kites. All sentries are warriors.
Some balloons are airplanes Some sentries are soldiers.

www.ibpsguide.com | estore.ibpsguide.com | www.sscexamguide.com


56
Exclusively on New Pattern – Reasoning Ability eBook

b) Statements: All jungles are bins.


All kings are warriors. All bins are petals.
All soldiers are warriors. Some petal is root.
All sentries are warriors. All roots are flowers.
Some sentries are soldiers. d) Statements:
c) Statements: All jungles are bins.
All kings are warriors. All bins are petals.
No soldiers are warriors. No petal is root.
All sentries are warriors. No roots are flowers.
Some sentries are soldiers. e) Statements:
d) Statements: All jungles are bins.
All kings are warriors. All bins are petals.
Some soldiers are warriors. No petal is root.
Some sentries are warriors. All roots are flowers.
Some sentries are soldiers.
e) Statements: 132. Conclusions:
Some kings are warriors. Some stands are poles is apossibility
Some soldiers are warriors. Some boxes are fans is a possibility
Some sentries are warriors. Some pens are fans is possibility
Some sentries are soldiers. Some stands are fans

131. Conclusions: a) Statements:


No jungle is root. All poles are fans.
All jungles are petals. No fans are stands.
No bins is a root Some stands are pens.
Some flowers are not petals Some pens are boxes.
b) Statements:
a) Statements: All poles are fans.
All jungles are bins. All fans are stands.
No bins are petals. All stands are pens.
No petal is root. No pens are boxes.
All roots are flowers. c) Statements:
b) Statements: All poles are fans.
Some jungles are bins. All fans are stands.
All bins are petals. No stands are pens.
No petal is root. Some pens are boxes.
All roots are flowers. d) Statements:
c) Statements: All poles are fans.

www.ibpsguide.com | estore.ibpsguide.com | www.sscexamguide.com


57
Exclusively on New Pattern – Reasoning Ability eBook

All fans are stands. Some metals are rings.


Some stands are pens. Some rings are bands.
Some pens are boxes.
e) Statements: 134. Conclusions:
All poles are fans. Some tubes are houses is apossibility
All fans are stands. Some rains are tubes
No stands are pens. Some rains are beads is a possibility
No pens are boxes. Some cycles are beads

133. Conclusions: a) Statements:


Some rings are weights is a possibility Some houses are beads.
Some metals are scales All beads are cycles.
Some bands are weights is a possibility All cycles are tubes.
Some bands are metals No tubes are rains.
b) Statements:
a) Statements: Some houses are beads.
Some scales are weights. Some beads are cycles.
Some weights are metals. Some cycles are tubes.
Some metals are rings. Some tubes are rains.
All rings are bands. c) Statements:
b) Statements: Some houses are beads.
Some scales are weights. Nobeads are cycles.
All weights are metals. Some cycles are tubes.
Some metals are rings. Some tubes are rains.
Some rings are bands. d) Statements:
c) Statements: All houses are beads.
Some scales are weights. All beads are cycles.
All weights are metals. No cycles are tubes.
Some metals are rings. Some tubes are rains.
All rings are bands. e) Statements:
d) Statements: Some houses are beads.
Some scales are weights. All beads are cycles.
All weights are metals. All cycles are tubes.
No metals are rings. No tubes are rains.
All rings are bands.
e) Statements: 135. Conclusions:
Some scales are weights. Some boxes are stone is a possibility
Some weights are metals. Some shells are not container

www.ibpsguide.com | estore.ibpsguide.com | www.sscexamguide.com


58
Exclusively on New Pattern – Reasoning Ability eBook

Some boxes are pearls is a possibility Allhostel is office.


Some pearls are stones All offices are institutes.
b) Statements:
a) Statements: Some schools are colleges.
All stones are pearls. Some colleges are hostels.
Some pearls are shells. No hostel is office.
Some shells are boxes. No offices are institutes.
No box is container. c) Statements:
b) Statements: Some schools are colleges.
All stones are pearls. No colleges are hostels.
Some pearls are shells. No hostel is office.
Some shells are boxes. All offices are institutes.
Allbox is container. d) Statements:
c) Statements: Some schools are colleges.
All stones are pearls. Some colleges are hostels.
All pearls are shells. No hostel is office.
No shells are boxes. All offices are institutes.
No box is container. e) Statements:
d) Statements: Some schools are colleges.
No stones are pearls. All colleges are hostels.
Some pearls are shells. No hostel is office.
Some shells are boxes. No offices are institutes.
No box is container.
e) Statements: 137.Conclusions:
Some stones are pearls. No raincoats are blazer
Some pearls are shells. Some suits are not shirts
Some shells are boxes. Some shirts are umbrellas
Somebox is container. Some suits are not raincoats

136. Conclusions: a) Statements:


Some institutes are not hostels Some umbrellas are raincoats.
Some institutes are colleges is a possibility All raincoats are shirts.
Some colleges are not offices Some shirt is a blazer.
Some hostels are schools is a possibility Some blazers are suits.
b) Statements:
a) Statements: Some umbrellas are raincoats.
Some schools are colleges. Some raincoats are shirts.
Some colleges are hostels. No shirt is a blazer.

www.ibpsguide.com | estore.ibpsguide.com | www.sscexamguide.com


59
Exclusively on New Pattern – Reasoning Ability eBook

Some blazers are suits. Some tables are desks.


c) Statements: d) Statements:
Some umbrellas are raincoats. Some chairs are rooms.
All raincoats are shirts. No room is sofa.
No shirt is a blazer. No sofas are tables.
Some blazers are suits. Some tables are desks.
d) Statements: e) Statements:
Some umbrellas are raincoats. Some chairs are rooms.
Some raincoats are shirts. No room is sofa.
Some shirt is a blazer. All sofas are tables.
Some blazers are suits. Some tables are desks.
e) Statements:
Some umbrellas are raincoats. 139.Conclusions:
All raincoats are shirts. No chalk is tubelight
No shirt is a blazer. Some boards are bulbs
No blazers are suits. Some bulbs are computers is a possibility
Some boards are not tubelights

138.Conclusions: a) Statements:
Some tables are not rooms All computers are boards.
Some desks are sofas is a possibility All boards are chalks.
Some tables are sofas No chalks are bulbs.
Some chairs are not sofa No bulb is tubelight.
b) Statements:
a) Statements: Some computers are boards.
Some chairs are rooms. Some boards are chalks.
Some room is sofa. No chalks are bulbs.
All sofas are tables. No bulb is tubelight.
Some tables are desks. c) Statements:
b) Statements: Some computers are boards.
Some chairs are rooms. Some boards are chalks.
All room is sofa. Some chalks are bulbs.
All sofas are tables. No bulb is tubelight.
Some tables are desks. d) Statements:
c) Statements: Some computers are boards.
Some chairs are rooms. Some boards are chalks.
Some room is sofa. Some chalks are bulbs.
Some sofas are tables. Some bulb is tubelight.

www.ibpsguide.com | estore.ibpsguide.com | www.sscexamguide.com


60
Exclusively on New Pattern – Reasoning Ability eBook

e) Statements: 141. Conclusions:


Some computers are boards. Some cooks are teachers.
Some boards are chalks. Some merchants are engineers is a possibility
All chalks are bulbs. Some poets are engineers is a possibility
No bulb is tubelight. Some cooks are poets.

140. Conclusions: a) Statements:


Some buses are jeeps All teachers are engineers.
Some buses are trains Some engineers are cooks.
Some boats are trains is a possibility Some cooks are merchants.
Some scooters are trains is a possibility All merchants are poets.
b) Statements:
a) Statements: All teachers are engineers.
Some jeeps are trains. All engineers are cooks.
Some trains are buses. Some cooks are merchants.
Some boats are jeeps. All merchants are poets.
Some scooters are buses. c) Statements:
b) Statements: All teachers are engineers.
Some jeeps are trains. All engineers are cooks.
No trains are buses. Some cooks are merchants.
Some boats are jeeps. Some merchants are poets.
Some scooters are buses. d) Statements:
c) Statements: Some teachers are engineers.
Some jeeps are trains. Some engineers are cooks.
All trains are buses. Some cooks are merchants.
Some boats are jeeps. Some merchants are poets.
Some scooters are buses. e) Statements:
d) Statements: All teachers are engineers.
Some jeeps are trains. All engineers are cooks.
No trains are buses. No cooks are merchants.
No boats are jeeps. All merchants are poets.
No scooters are buses.
e) Statements: 142. Conclusions:
All jeeps are trains. Some nails are tools is a possibility
Some trains are buses. Some nuts are nails
Some boats are jeeps. Some screws are hammers
Some scooters are buses. Some nuts are hammers

www.ibpsguide.com | estore.ibpsguide.com | www.sscexamguide.com


61
Exclusively on New Pattern – Reasoning Ability eBook

a) Statements: No pens are bags.


Some tools are hammers. All bags are glasses.
Some hammers are nails. No glass is a spoon.
All nails are screws. All spoons are books.
Some screws are nuts. c) Statements:
b) Statements: All pens are bags.
Some tools are hammers. All bags are glasses.
Some hammers are nails. Some glass is a spoon.
Some nails are screws. All spoons are books.
All screws are nuts. d) Statements:
c) Statements: All pens are bags.
Some tools are hammers. Some bags are glasses.
Some hammers are nails. No glass is a spoon.
All nails are screws. All spoons are books.
All screws are nuts. e) Statements:
d) Statements: All pens are bags.
Some tools are hammers. All bags are glasses.
Some hammers are nails. No glass is a spoon.
Some nails are screws. All spoons are books.
Some screws are nuts.
e) Statements: 144. Conclusions:
Some tools are hammers. Some tiles are floors
All hammers are nails. Some paints are floors
Some nails are screws. Some stones are floors is a possibility
All screws are nuts. Some paints are doors is a possibility

143. Conclusions: a) Statements:


Some glasses are pens. All doors are floors.
No bag is a spoon Some floors are tiles.
No pen is aspoon. Some tiles are paints.
Some books are not glasses Some paints are stones.
b) Statements:
a) Statements: All doors are floors.
All pens are bags. Some floors are tiles.
No bags are glasses. All tiles are paints.
No glass is a spoon. Some paints are stones.
All spoons are books. c) Statements:
b) Statements: Some doors are floors.

www.ibpsguide.com | estore.ibpsguide.com | www.sscexamguide.com


62
Exclusively on New Pattern – Reasoning Ability eBook

Some floors are tiles. All rabbits are birds.


Some tiles are paints. Some birds are animals.
Some paints are stones. e) Statements:
d) Statements: Some snakes are eagles.
All doors are floors. Some eagles are rabbits.
All floors are tiles. Some rabbits are birds.
All tiles are paints. Some birds are animals.
No paints are stones.
e) Statements: 146. Conclusions:
All doors are floors. Some diaries are cameras is a possibility
All floors are tiles. Some calculators are computers.
No tiles are paints. All notebooks are computers.
Some paints are stones. Some computers are diaries

145. Conclusions: a) Statements:


Some birds are eagles. All cameras are calculators.
Some animals are rabbit is a possibility No calculators are diaries.
Some birds are snakes is a possibility All notebooks are diaries.
Some birds are rabbits All diaries are computers.
b) Statements:
a) Statements: Some cameras are calculators.
All snakes are eagles. Some calculators are diaries.
Some eagles are rabbits. All notebooks are diaries.
Some rabbits are birds. No diaries are computers.
Some birds are animals. c) Statements:
b) Statements: Some cameras are calculators.
All snakes are eagles. Some calculators are diaries.
Some eagles are rabbits. All notebooks are diaries.
All rabbits are birds. All diaries are computers.
No birds are animals. d) Statements:
c) Statements: Some cameras are calculators.
All snakes are eagles. Some calculators are diaries.
All eagles are rabbits. All notebooks are diaries.
No rabbits are birds. Some diaries are computers.
Some birds are animals. e) Statements:
d) Statements: Some cameras are calculators.
All snakes are eagles. Some calculators are diaries.
Some eagles are rabbits. Some notebooks are diaries.

www.ibpsguide.com | estore.ibpsguide.com | www.sscexamguide.com


63
Exclusively on New Pattern – Reasoning Ability eBook

Some diaries are computers. Some glass are toys is a possibility

147.Conclusions: a) Statements:
All planets are asteroids Some bats are toys.
Some rocks are asteroids is a possibility Some toys are plastics.
Some moons are planets Some plastics are mirrors.
Some asteroids are stars Some mirror is glass.
b) Statements:
a) Statements: Some bats are toys.
All planets are stars. All toys are plastics.
Some stars are asteroids. No plastics are mirrors.
All asteroids are moons. No mirror is glass.
Some moons are rocks. c) Statements:
b) Statements: Some bats are toys.
All planets are stars. No toys are plastics.
All stars are asteroids. Some plastics are mirrors.
All asteroids are moons. No mirror is glass.
Some moons are rocks. d) Statements:
c) Statements: Some bats are toys.
Some planets are stars. All toys are plastics.
Some stars are asteroids. Some plastics are not mirrors.
Some asteroids are moons. No mirror is glass.
Some moons are rocks. e) Statements:
d) Statements: Some bats are toys.
All planets are stars. Some toys are plastics.
No stars are asteroids. Some plastics are mirrors.
All asteroids are moons. No mirror is glass.
Some moons are rocks.
e) Statements: 149. Conclusions:
All planets are stars. Some lawyers are advocates
All stars are asteroids. Some doctors are lawyers
All asteroids are moons. Some lawyers are graduates is a possibility
No moons are rocks. Some advocates are graduates

148.Conclusions: a) Statements:
Some plastic are not glass All graduates are advocates.
Some mirrors are toys is a possibility Some advocates are judges.
Some plastics are toys Some judges are lawyers.

www.ibpsguide.com | estore.ibpsguide.com | www.sscexamguide.com


64
Exclusively on New Pattern – Reasoning Ability eBook

Some lawyers are doctors. c) Statements:


b) Statements: Some roses are flowers.
No graduates are advocates. Some flowers are buds.
Some advocates are judges. All buds are leaves.
All judges are lawyers. Some leaves are plants.
Some lawyers are doctors. d) Statements:
c) Statements: Some roses are flowers.
All graduates are advocates. Some flowers are buds.
All advocates are judges. Some buds are leaves.
No judges are lawyers. Some leaves are plants.
Some lawyers are doctors. e) Statements:
d) Statements: All roses are flowers.
All graduates are advocates. No flowers are buds.
Some advocates are judges. All buds are leaves.
All judges are lawyers. All leaves are plants.
Some lawyers are doctors.
e) Statements:
All graduates are advocates. Direction (Q.151-200):In each of the questions below
Some advocates are judges. there are given four conclusions followed by
All judges are lawyers. fivestatements numbered I, II, III, IV & IV. You have
No lawyers are doctors. to take the given statements to be true even if they
seem to be at variance with commonly known facts.
150. Conclusions: Read all the statements and then decide which of the
Some plants are flowers. given statement logically follows from the given
Some leaves are flowers conclusion disregarding commonly known facts.
Some buds are roses is a possibility
Some plants are buds 151.Conclusions:
Some papers are books.
a) Statements: Some papers are boards.
Some roses are flowers. Some boards are journals is a possibility
Some flowers are buds. Some cards are books is a possibility
All buds are leaves.
All leaves are plants. (I) Statements:
b) Statements: Some books are journals.
Some roses are flowers. All journals are papers.
Some flowers are buds. Some papers are cards.
Some buds are leaves. All cards are boards.
All leaves are plants. (II) Statements:

www.ibpsguide.com | estore.ibpsguide.com | www.sscexamguide.com


65
Exclusively on New Pattern – Reasoning Ability eBook

Some books are journals.


Some journals are papers. (II) Statements:
Some papers are cards. Some grapes are apples.
All cards are boards. Some apples are bananas.
(III) Statements: All bananas are guavas.
All books are journals. No guava is pomegranate.
All journals are papers.
Some papers are cards. (III) Statements:
All cards are boards. Some grapes are apples.
(IV) Statements: Some apples are bananas.
Some books are journals. No bananas are guavas.
Some journals are papers. No guava is pomegranate.
Some papers are cards.
Some cards are boards. (IV) Statements:
(V) Statements: Some grapes are apples.
All books are journals. No apples are bananas.
All journals are papers. All bananas are guavas.
All papers are cards. No guava is pomegranate.
All cards are boards.
(V) Statements:
Some grapes are apples.
a)Only IV follows Some apples are bananas.
b) Only I and II follow All bananas are guavas.
c) Only II and III follow All guavas are pomegranate.
d) Only I, III and V follow
e) Only I follow a) Only III follow
b) Only II follow
152. Conclusions: c) Only II and III follow
Some guavas are apples. d) Only II, III and IV follow
No bananas are pomegranates. e) Only IV follows
Some apples are not pomegranate
Some guavas are bananas 153. Conclusions:
All walls are rooms.
(I) Statements: Some rooms are doors.
Some grapes are apples. Some rooms are walls.
Some apples are bananas. Some floors are doors.
Some bananas are guavas.
No guava is pomegranate. (I) Statements:

www.ibpsguide.com | estore.ibpsguide.com | www.sscexamguide.com


66
Exclusively on New Pattern – Reasoning Ability eBook

Some doors are walls. Some bowls are forks


All walls are floors. Some plates are bowls
Some floors are rooms.
Some rooms are windows. (I) Statements:
Some spoons are forks.
(II) Statements: Some forks are bowls.
Some doors are walls. Some bowls are plates.
All walls are floors. Some plates are utensils.
All floors are rooms.
Some rooms are windows. (II) Statements:
Some spoons are forks.
(III) Statements: All forks are bowls.
Some doors are walls. All bowls are plates.
Some walls are floors. No plates are utensils.
All floors are rooms.
Some rooms are windows. (III) Statements:
Some spoons are forks.
(IV) Statements: No forks are bowls.
Some doors are walls. All bowls are plates.
Some walls are floors. Some plates are utensils.
Some floors are rooms.
Some rooms are windows. (IV) Statements:
Some spoons are forks.
(V) Statements: Some forks are bowls.
All doors are walls. No bowls are plates.
All walls are floors. Some plates are utensils.
All floors are rooms.
All rooms are windows. (V) Statements:
Some spoons are forks.
a)Only I and IV follow Some forks are bowls.
b) Only I and II follow All bowls are plates.
c) Only II and III follow Some plates are utensils.
d) Only II, III and IV follow
e) Only II and V follow a) Only V follows
b) Only II follows
154. Conclusions: c) Only I and III follow
Some plates are forks. d) Only II and IV follow
Some utensils are forks is a possibility e) Only I follow

www.ibpsguide.com | estore.ibpsguide.com | www.sscexamguide.com


67
Exclusively on New Pattern – Reasoning Ability eBook

b) Only I and II follow


155. Conclusions: c) Only II and III follow
Some sofas are benches is a possibility d) Only III follows
Some desks are tables e)Only IV follows
Some benches are tables is a possibility
Some desks are chairs 156. Conclusions:
Some diets are chocolates is a possibility
(I) Statements: Some mints are chocolates
All chairs are tables. Some foods are sweets is a possibility
No tables are desks. Some diets are food
Some desks are benches.
Some desks are sofas. (I) Statements:
Some sweets are chocolates.
(II) Statements: No chocolates are mints.
All chairs are tables. Some mints are food.
Some tables are desks. Some food is diet.
Some desks are benches.
Some desks are sofas. (II) Statements:
Some sweets are chocolates.
(III) Statements: Some chocolates are mints.
All chairs are tables. Some mints are food.
All tables are desks. Some food is diet.
Some desks are benches.
Some desks are sofas. (III) Statements:
Some sweets are chocolates.
(IV) Statements: All chocolates are mints.
Some chairs are tables. All mints are food.
Some tables are desks. All food is diet.
Some desks are benches.
Some desks are sofas. (IV) Statements:
All sweets are chocolates.
(V) Statements: All chocolates are mints.
All chairs are tables. All mints are food.
All tables are desks. Nofood is diet.
No desks are benches.
Some desks are sofas. (V) Statements:
All sweets are chocolates.
a)Only II follow All chocolates are mints.

www.ibpsguide.com | estore.ibpsguide.com | www.sscexamguide.com


68
Exclusively on New Pattern – Reasoning Ability eBook

All mints are food.


All food is diet. (V) Statements:
Some doctors are lawyers.
a)Only II and V follow All teachers are lawyers.
b)Only II, III and V follow Some engineers are lawyers.
c) Only III and IV follow No engineers are businessmen.
d) Only II and III follow
e) None of these a)Only II follows
b) Only III follows
157. Conclusions: c) Only I follow
Some businessmen are lawyers. d) Only II and IV follow
Some lawyers are teachers. e) None of these
Some teachers are doctors is a possibility
Some engineers are teachers is a possibility 158. Conclusions:
Some liquids are sponges
(I) Statements: Some sponges are plastics is a possibility
Some doctors are lawyers. Some clothes are glasses
All teachers are lawyers. Some liquids are glasses
Some engineers are lawyers.
All engineers are businessmen. (I) Statements:
All plastics are glasses.
(II) Statements: Some sponges are glasses.
Some doctors are lawyers. All sponges are clothes.
All teachers are lawyers. Some clothes are liquids.
Some engineers are lawyers.
Someengineers are businessmen. (II) Statements:
All plastics are glasses.
(III) Statements: Some sponges are glasses.
Some doctors are lawyers. Some sponges are clothes.
Some teachers are lawyers. All clothes are liquids.
Some engineers are lawyers.
Someengineers are businessmen. (III) Statements:
All plastics are glasses.
(IV) Statements: Some sponges are glasses.
Some doctors are lawyers. Some sponges are clothes.
No teachers are lawyers. Some clothes are liquids.
Some engineers are lawyers.
All engineers are businessmen. (IV) Statements:

www.ibpsguide.com | estore.ibpsguide.com | www.sscexamguide.com


69
Exclusively on New Pattern – Reasoning Ability eBook

Some plastics are glasses. Some beaches are trees.


Some sponges are glasses. All trees are hotels.
Some sponges are clothes.
Some clothes are liquids. (IV) Statements:
All sands are beaches.
(V) Statements: All shores are beaches.
All plastics are glasses. No beaches are trees.
Some sponges are glasses. Notrees are hotels.
All sponges are clothes.
All clothes are liquids. (V) Statements:
All sands are beaches.
a)Only II follows All shores are beaches.
b) Only II and IV follow Some beaches are trees.
c) Only IV follows Some trees are hotels.
d) Only V follows
e)Only II and III follow a)Only III follow
b) Only II follows
159. Conclusions: c) Only I and III follows
Some beaches are shores d) Only IV follows
Some shores aresands. e) None of these
Some beaches are hotels.
Some sands are trees is a possibility 160. Conclusions:
Some eagles are crows.
(I) Statements: Some crows are parrots is a possibility
All sands are beaches. Some eagles are sparrows
All shores are beaches. Some sparrows are pigeon is a possibility
No beaches are trees.
All trees are hotels. (I) Statements:
All parrots are pigeons.
(II) Statements: Some crows are pigeons.
Some sands are beaches. Some sparrows are crows.
Some shores are beaches. Some sparrows are eagles.
Some beaches are trees.
All trees are hotels. (II) Statements:
All parrots are pigeons.
(III) Statements: Some crows are pigeons.
All sands are beaches. Some sparrows are crows.
All shores are sands. All sparrows are eagles.

www.ibpsguide.com | estore.ibpsguide.com | www.sscexamguide.com


70
Exclusively on New Pattern – Reasoning Ability eBook

All chairs are tables.


(III) Statements: All tables are cushions.
All parrots are pigeons. All cushions are trolleys.
Some crows are pigeons. All trolleys are lamps.
Some sparrows are crows.
No sparrows are eagles. (III) Statements:
All chairs are tables.
(IV) Statements: All tables are cushions.
All parrots are pigeons. Some cushions are trolleys.
No crows are pigeons. Some trolleys are lamps.
Some sparrows are crows.
All sparrows are eagles. (IV) Statements:
All chairs are tables.
(V) Statements: Some tables are cushions.
All parrots are pigeons. Some cushions are trolleys.
No crows are pigeons. No trolleys are lamps.
All sparrows are crows.
All sparrows are eagles. (V) Statements:
Some chairs are tables.
a) Only I follow Some tables are not cushions.
b) Only III follows Some cushions are trolleys.
c) Only I and IV follow All trolleys are lamps.
d) Only I and III follows
e)Only II follow a) Only I follow
b) Only I and II follow
161. Conclusions: c) Only I and III follow
Some cushions are lamps. d) All follow
Some trolleys are tables is a possibility e)Only II follows
All chairs are cushions.
Some cushions are tables 162. Conclusions:
Some sticks are gems.
(I) Statements: Some toys are dolls.
All chairs are tables. Some boxes are toys is a possibility
All tables are cushions. Some sticks are boxes
Some cushions are trolleys.
All trolleys are lamps. (I) Statements:
All dolls are toys.
(II) Statements: Some toys are gems.

www.ibpsguide.com | estore.ibpsguide.com | www.sscexamguide.com


71
Exclusively on New Pattern – Reasoning Ability eBook

Some gems are boxes.


No boxes are sticks. (I) Statements:
All vegetables are plants.
(II) Statements: All plant is flower.
All dolls are toys. Some flowers are jungles.
All toys are gems. All jungles are trees.
No gems are boxes.
All boxes are sticks. (II) Statements:
No vegetables are plants.
(III) Statements: No plant is flower.
Some dolls are toys. Some flowers are jungles.
Some toys are gems. All jungles are trees.
Some gems are boxes.
Some boxes are sticks. (III) Statements:
All vegetables are plants.
(IV) Statements: No plant is flower.
All dolls are toys. No flowers are jungles.
Some toys are gems. All jungles are trees.
Some gems are boxes.
All boxes are sticks. (IV) Statements:
All vegetables are plants.
(V) Statements: No plant is flower.
All dolls are toys. Some flowers are jungles.
Some toys are gems. Some jungles are trees.
Some gems are boxes.
Some boxes are sticks. (V) Statements:
All vegetables are plants.
a) Only IV follows No plant is flower.
b) Only II follows Some flowers are jungles.
c) Only III and IV follow All jungles are trees.
d) Only I and IV follow
e)Only V follows a) Only I follow
b) Only II follows
163. Conclusions: c) Only III and IV follow
Some jungle is not plant d) Only I and IV follow
Some trees are flowers e)Only V follows
No vegetable is a flower
Some jungles are flowers 164. Conclusions:

www.ibpsguide.com | estore.ibpsguide.com | www.sscexamguide.com


72
Exclusively on New Pattern – Reasoning Ability eBook

Some poles are knives d) Only I and IV follow


Some sticks are hammers e) Only V follows
Some pencils are poles is a possibility
Some sticks are knives 165. Conclusions:
Some tables are desks is a possibility
(I) Statements: Some tables are chairs
Some knives are hammers. Some chairs are pens is a possibility
All hammers are poles. Some pens are books
All poles are sticks.
Some sticks are pencils. (I) Statements:
All books are pens.
(II) Statements: Some pens are desks.
Some knives are hammers. All desks are chairs.
All hammers are poles. All chairs are tables.
Some poles are sticks.
Some sticks are pencils. (II) Statements:
All chairs are pens.
(III) Statements: Some pens are desks.
Some knives are hammers. Some desks are books.
Some hammers are poles. Some chairs are tables.
All poles are sticks.
Some sticks are pencils. (III) Statements:
All books are pens.
(IV) Statements: Some pens are desks.
Some knives are hammers. Some desks are chairs.
Some hammers are poles. Some chairs are tables.
Some poles are sticks.
Some sticks are pencils. (IV) Statements:
All books are pens.
(V) Statements: All pens are desks.
All knives are hammers. All desks are chairs.
All hammers are poles. All chairs are tables.
All poles are sticks.
All sticks are pencils. (V) Statements:
All books are pens.
a) Only I follow Some pens are desks.
b) Only II follows Some desks are chairs.
c) Only I and V follow Some chairs are not tables.

www.ibpsguide.com | estore.ibpsguide.com | www.sscexamguide.com


73
Exclusively on New Pattern – Reasoning Ability eBook

All pins are magnets.


a) Only I follows All magnets are scales.
b) Only II follows All scales are trucks.
c) Only III and IV follow All trucks are buses.
d) Only I, III and IV follow
e) Only II, III and IV follow a) Only II and V follow
b) Only II follows
166. Conclusions: c) Only III and IV follow
Some buses are scales d) Only I and IV follow
Some trucks are pins is a possibility e)Only III and IV follow
Some scales are magnets
Some buses are trucks 167. Conclusions:
All trees are houses.
(I) Statements: Some buildings are houses
Some pins are magnets. Some buildings are roads.
Some magnets are scales. Some roads are trees
Some scales are trucks.
Some trucks are buses. (I) Statements:
All jungles are trees.
(II) Statements: All trees are roads.
Some pins are magnets. All roads are houses.
Some magnets are scales. Some houses are buildings.
Some scales are trucks.
All trucks are buses. (II) Statements:
All jungles are trees.
(III) Statements: All trees are roads.
Some pins are magnets. Some roads are houses.
Some magnets are scales. All houses are buildings.
Some scales are trucks.
No trucks are buses. (III) Statements:
All jungles are trees.
(IV) Statements: All trees are roads.
Some pins are magnets. All roads are houses.
No magnets are scales. All houses are buildings.
Some scales are trucks.
All trucks are buses. (IV) Statements:
All jungles are trees.
(V) Statements: Some trees are roads.

www.ibpsguide.com | estore.ibpsguide.com | www.sscexamguide.com


74
Exclusively on New Pattern – Reasoning Ability eBook

Some roads are houses.


All houses are buildings. (IV) Statements:
Some tablets are packets.
(V) Statements: Some packet is bag.
Some jungles are trees. Some bags are toys.
Some trees are roads. Some toys are puppets.
Some roads are houses.
Some houses are buildings. (V) Statements:
All tablets are packets.
a) Only I follow No packet is bag.
b) Only II follows Some bags are toys.
c) Only IV follows All toys are puppets.
d) Only I and IV follow
e)Only III follows a) Only I follow
b) Only II follows
168. Conclusions: c) Only V follows
Some puppets are tablets is a possibility d) Only IV follows
Some puppets are bags. e) Only III follow
No tablet is a bag
Some toys are not packets 169.Conclusions:
Some benches are desks is a possibility
(I) Statements: Some chairs are tables
All tablets are packets. Some cots are tables is a possibility
No packet is bag. Some benches are chairs
No bags are toys.
All toys are puppets. (I) Statements:
Some desks are tables.
(II) Statements: Some tables are chairs.
All tablets are packets. Some chairs are benches.
No packet is bag. Some benches are cots.
Some bags are toys.
Some toys are puppets. (II) Statements:
Some desks are tables.
(III) Statements: No tables are chairs.
Some tablets are packets. Some chairs are benches.
No packet is bag. Some benches are cots.
Some bags are toys.
Some toys are puppets. (III) Statements:

www.ibpsguide.com | estore.ibpsguide.com | www.sscexamguide.com


75
Exclusively on New Pattern – Reasoning Ability eBook

All desks are tables. Some bracelets are jewels.


All tables are chairs. Some jewels are stones.
All chairs are benches.
All benches are cots. (III) Statements:
All bangles are rings.
(IV) Statements: All rings are bracelets.
Some desks are tables. All bracelets are jewels.
All tables are chairs. All jewels are stones.
All chairs are benches.
No benches are cots. (IV) Statements:
All bangles are rings.
(V) Statements: All rings are bracelets.
All desks are tables. Some bracelets are jewels.
All tables are chairs. No jewels are stones.
No chairs are benches.
Some benches are cots. (V) Statements:
All bangles are rings.
a) Only I follow All rings are bracelets.
b) Only II follows All bracelets are jewels.
c) Only III and IV follow No jewels are stones.
d) Only I and III follow
e) Only II and III follow a) Only I follow
b) Only II and III follow
170.Conclusions: c) Only III and IV follow
Some stones are jewels d) Only I and IV follow
Some stones are bangles is a possibility e) Only I and II follow
Some bracelets are rings
Some jewels are rings is a possibility 171.Conclusions:
Some stars are planets
(I) Statements: Some gases are suns is a possibility
All bangles are rings. Some stars are moons
No rings are bracelets. All planets are sun is a possibility
Some bracelets are jewels.
Some jewels are stones. (I) Statements:
No sun are stars
(II) Statements: No moons are stars
All bangles are rings. Some planets are sun
All rings are bracelets. Some stars are gases

www.ibpsguide.com | estore.ibpsguide.com | www.sscexamguide.com


76
Exclusively on New Pattern – Reasoning Ability eBook

Some dolls are diaries


(II) Statements: Some diaries are pinks
All sunare stars Some pinks are drawers
All moons are stars No drawers are chairs
No planets are sun
No stars are gases (II) Statements:
All dolls are diaries
(III) Statements: No diaries are drawers
No sun are stars Some pinks are drawers
All moons are stars All drawers are chairs
No planets are sun
Some stars are gases (III) Statements:
All dolls are diaries
(IV) Statements: Some diaries are pinks
All sunare stars Some pinks are drawers
All moons are stars All drawers are chairs
Some planets are suns
Some stars are gases (IV) Statements:
No dolls are diaries
(V) Statements: Some diaries are pinks
All sun are stars Some pinks are drawers
No moons are stars No drawers are chairs
No planets are sun
Some stars are gases (V) Statements:
All dolls are diaries
a) Only I follow All diaries are pinks
b) Only II follows All pinks are drawers
c) Only III follow All drawers are chairs
d) Only IV follows
e)Only V follows a) Only I follow
b) Only II and III follow
172.Conclusions: c) Only III and V follow
Some chairs are pinks d) Only I and IV follow
Some diaries are dolls e)Only V follows
Some drawers are dolls is a possibility
Some chairs are drawers 173.Conclusions:
Some buildings are trucks
(I) Statements: Some rivers are buildings

www.ibpsguide.com | estore.ibpsguide.com | www.sscexamguide.com


77
Exclusively on New Pattern – Reasoning Ability eBook

Some trucks are roads


Some buildings are roads 174.Conclusions:
Some bills are conical
(I) Statements: Some bills are rivers
Some buildings are rivers Some conical are rivers
Some mountains are both buildings and rivers Some rounds are tables
Some roads are buildings
All roads are trucks (I) Statements:
No tables are round
(II) Statements: Some bills are round
No buildings are rivers Some rivers are bills
No mountains are both buildings and rivers All rivers are conical
Some roads are buildings
All roads are trucks (II) Statements:
All tables are round
(III) Statements: No bills are round
All buildings are rivers Some rivers are bills
Some mountains are both buildings and rivers All rivers are conical
No roads are buildings
All roads are trucks (III) Statements:
All tables are round
(IV) Statements: No bills are round
Some buildings are rivers No rivers are bills
No mountains are both buildings and rivers All rivers are conical
Some roads are buildings
No roads are trucks (IV) Statements:
All tables are round
(V) Statements: Some bills are round
Some buildings are rivers Some rivers are bills
Some mountains are both buildings and rivers No rivers are conical
Some roads are buildings
No roads are trucks (V) Statements:
All tables are round
a) Only I follow Some bills are round
b) Only II follows Some rivers are bills
c) Only III follow All rivers are conical
d) Only IV follows
e)Only V follows a) Only V follows

www.ibpsguide.com | estore.ibpsguide.com | www.sscexamguide.com


78
Exclusively on New Pattern – Reasoning Ability eBook

b) Only II follows Some waters are desks


c) Only III and IV follow Some desks are tables
d) Only I and IV follow
e)Only II and V follow a) Only III follows
b) Only II follows
175. Conclusions: c) Only III and IV follow
Some waters are tins d) Only I and IV follow
All tins are waters e)Only IV follows
Some waters are books
Some tables are desks 176. Conclusions:
All buses are trains is a possibility
(I) Statements: Some boats are trucks
No tins are books All trucks are buses is a possibility
All books are waters Some cars are boats
Some waters are desks
No desks are tables (I) Statements:
No trains are buses
(II) Statements: Some buses are trucks
All tins are books No trucks are boats
All books are waters Some boats are cars
Some waters are desks
Some desks are tables (II) Statements:
Some trains are buses
(III) Statements: Some buses are trucks
All tins are books No trucks are boats
No books are waters Some boats are cars
Some waters are desks
Some desks are tables (III) Statements:
Some trains are buses
(IV) Statements: Some buses are trucks
All tins are books Some trucks are boats
No books are waters Some boats are cars
Some waters are desks
No desks are tables (IV) Statements:
Some trains are buses
(V) Statements: No buses are trucks
Some tins are books No trucks are boats
Some books are waters Some boats are cars

www.ibpsguide.com | estore.ibpsguide.com | www.sscexamguide.com


79
Exclusively on New Pattern – Reasoning Ability eBook

Some chains are bangles


(V) Statements: Some bangles are rings
All trains are buses All rings are tyres.
All buses are trucks All tyres are wheels
All trucks are boats
All boats are cars (V) Statements:
Some chains are bangles
a) Only I and III follow Some bangles are rings
b) Only II and IV follow No rings are tyres.
c) Only III and V follow No tyres are wheels
d) Only I and IV follow
e) Only II and III follow a) Only I follow
b) Only II follows
177.Conclusions: c) Only II and IV follow
Some wheels are bangles d) Only III and IV follow
Some tyres are bangles e) Only I and IV follow
Some tyres are rings
Some chains are wheels is a possibility 178.Conclusions:
Some seas are stones
(I) Statements: Some jungles are hills
Some chains are bangles Some stones are hills
Some bangles are rings All stones are seas
No rings are tyres.
All tyres are wheels (I) Statements:
All hills are roads
(II) Statements: All roads are stones
Some chains are bangles All stones are jungles
No bangles are rings All jungles are sea
No rings are tyres.
All tyres are wheels (II) Statements:
No hills are roads
(III) Statements: All roads are stones
All chains are bangles All stones are jungles
All bangles are rings All jungles are sea
All rings are tyres.
All tyres are wheels (III) Statements:
Some hills are roads
(IV) Statements: All roads are stones

www.ibpsguide.com | estore.ibpsguide.com | www.sscexamguide.com


80
Exclusively on New Pattern – Reasoning Ability eBook

All stones are jungles


All jungles are sea (III) Statements:
No needles are threads
(IV) Statements: No threads are clothes
Some hills are roads No cloth is room
All roads are stones All rooms are houses
All stones are jungles
Some jungles are sea (IV) Statements:
Some needles are threads
(V) Statements: Some threads are clothes
All hills are roads Some cloth is room
Some roads are stones All rooms are houses
All stones are jungles
Some jungles are sea (V) Statements:
No needles are threads
a) Only I and III follow No threads are clothes
b) Only II and IV follow No cloth is room
c) Only III and IV follow No rooms are houses
d) Only I and IV follow
e) Only II and V follow a) Only I follow
b) Only II follows
179. Conclusions: c) Only III follow
Some threads are not rooms d) Only IV follows
Some houses are not cloth e)Only V follows
Some threads are needles
Some clothes are threads 180. Conclusions:
Some roofs are curtains
(I) Statements: Some curtains are bags
All needles are threads Some curtains are toys
All threads are clothes Some roofs are walls
All cloth is room
All rooms are houses (I) Statements:
Some bags are toys
(II) Statements: No toys are curtains
All needles are threads Some curtains are walls
Some threads are clothes All walls are roofs
No cloth is room
All rooms are houses (II) Statements:

www.ibpsguide.com | estore.ibpsguide.com | www.sscexamguide.com


81
Exclusively on New Pattern – Reasoning Ability eBook

No bags are toys Some bottles are trucks.


No toys are curtains Some trucks are lanes
Some curtains are walls
All walls are roofs (II) Statements:
All bikes are buildings.
(III) Statements: All buildings are bottles.
Some bags are toys All bottles are trucks.
All toys are curtains All trucks are lanes
No curtains are walls
No walls are roofs (III) Statements:
Some bikes are buildings.
(IV) Statements: All buildings are bottles.
All bags are toys All bottles are trucks.
All toys are curtains No trucks are lanes
All curtains are walls
All walls are roofs (IV) Statements:
All bikes are buildings.
(V) Statements: All buildings are bottles.
Some bags are toys Some bottles are trucks.
All toys are curtains No trucks are lanes
Some curtains are walls
All walls are roofs (V) Statements:
Some bikes are buildings.
a) Only I and II follows All buildings are bottles.
b) Only II and III follow Some bottles are trucks.
c) Only III and IV follow Some trucks are lanes
d) Only I and IV follow
e)Only IV and V follow a) Only II and III follow
b) Only III and V follow
181. Conclusions: c) Only III and IV follow
All bottles are bikes is a possibility d) Only II and V follow
Some bottles are buildings e)Only I and III follow
Some lanes are bikes is a possibility
Some lanes are bottles is a possibility 182.Conclusions:
Some lines are not copies
(I) Statements: Some telephones are copies
Some bikes are buildings. No copy is a radio
No buildings are bottles. Some radios are lines is a possibility

www.ibpsguide.com | estore.ibpsguide.com | www.sscexamguide.com


82
Exclusively on New Pattern – Reasoning Ability eBook

Some dogs are bags


(I) Statements: All cats are bags is a possibility
All cards are lines. No cat is a cycle
All copies are cards. Some bulls are bags is a possibility
All cards are telephones.
No radios are telephones. (I) Statements:
Some bags are cats.
(II) Statements: All cats are dogs.
No cards are lines. No dog is a cycle.
All copies are cards. All cycles are bulls.
All cards are telephones.
No radios are telephones. (II) Statements:
Some bags are cats.
(III) Statements: Some cats are dogs.
No cards are lines. Some dog is a cycle.
Some copies are cards. All cycles are bulls.
Some cards are telephones.
Some radios are telephones. (III) Statements:
Some bags are cats.
(IV) Statements: No cats are dogs.
No cards are lines. No dog is a cycle.
All copies are cards. No cycles are bulls.
All cards are telephones.
Some radios are telephones. (IV) Statements:
Some bags are cats.
(V) Statements: All cats are dogs.
No cards are lines. All dog is a cycle.
No copies are cards. All cycles are bulls.
All cards are telephones.
No radios are telephones. (V) Statements:
All bags are cats.
a) Only I follow All cats are dogs.
b) Only II follow No dog is a cycle.
c) Only III follow All cycles are bulls.
d) Only IV follows
e)Only V follows a) Only I and V follow
b) Only II and IV follow
183. Conclusions: c) Only III and IV follow

www.ibpsguide.com | estore.ibpsguide.com | www.sscexamguide.com


83
Exclusively on New Pattern – Reasoning Ability eBook

d) Only I and IV follow


e)Only I and II follow a) Only I and III follow
b) Only II and III follow
184. Conclusions: c) Only III and IV follow
Some guns are bats d) Only I and IV follow
Some umbrellas are balls is a possibility e) None of these
Some guns are balls
All bats are balls is a possibility 185. Conclusions:
All dictionaries are cards is a possibility
(I) Statements: Some envelopes are dictionaries
All umbrellas are sticks. Some envelopes are cards is a possibility
All sticks are balls. All files are dictionaries is a possibility
All balls are bats.
All bats are guns. (I) Statements:
All cards are notebooks.
(II) Statements: All notebooks are dictionaries.
Some umbrellas are sticks. Some dictionaries are files.
Some sticks are balls. No files are envelopes.
Some balls are bats.
Some bats are guns. (II) Statements:
Some cards are notebooks.
(III) Statements: Some notebooks are dictionaries.
All umbrellas are sticks. Some dictionaries are files.
No sticks are balls. Some files are envelopes.
All balls are bats.
All bats are guns. (III) Statements:
Some cards are notebooks.
(IV) Statements: Some notebooks are dictionaries.
Some umbrellas are sticks. Some dictionaries are files.
Some sticks are balls. All files are envelopes.
Some balls are bats.
All bats are guns. (IV) Statements:
All cards are notebooks.
(V) Statements: All notebooks are dictionaries.
Some umbrellas are sticks. All dictionaries are files.
All sticks are balls. All files are envelopes.
Some balls are bats.
No bats are guns. (V) Statements:

www.ibpsguide.com | estore.ibpsguide.com | www.sscexamguide.com


84
Exclusively on New Pattern – Reasoning Ability eBook

No cards are notebooks. All roads are codes


Some notebooks are dictionaries. All codes are dresses
No dictionaries are files.
All files are envelopes. (V) Statements:
Some buses are cars
a) Only I and III follow All cars are roads
b) Only II and IV follow All roads are codes
c) Only III and IV follow Some codes are dresses
d) Only I and IV follow
e) None of these a) Only I and IV follow
b) Only II and V follow
186. Conclusions: c) Only III and IV follow
Some codes are buses d) Only I and V follow
Some dresses are cars is a possibility e)Only IV and V follow
Some roads are buses
Some buses are dresses is a possibility 187. Conclusions:
Some mountains are desks
(I) Statements: Some notes are colours is a possibility
Some buses are cars Some notes are kites
Some cars are roads Some colours are desks is a possibility
All roads are codes
Some codes are dresses (I) Statements:
Some colours are kites
(II) Statements: Some kites are desks
All buses are cars Some desks are notes
Some cars are roads Some notes are mountains
All roads are codes
Some codes are dresses (II) Statements:
Some colours are kites
(III) Statements: All kites are desks
Some buses are cars Some desks are notes
No cars are roads Somenotes are mountains
All roads are codes
All codes are dresses (III) Statements:
Some colours are kites
(IV) Statements: Some kites are desks
All buses are cars All desks are notes
All cars are roads All notes are mountains

www.ibpsguide.com | estore.ibpsguide.com | www.sscexamguide.com


85
Exclusively on New Pattern – Reasoning Ability eBook

All papers are boxes


(IV) Statements: All boxes are boards
Some colours are kites Some boards are blacks
All kites are desks Some blacks are roads
No desks are notes
All notes are mountains (IV) Statements:
All papers are boxes
(V) Statements: Some boxes are boards
All colours are kites Some boards are blacks
All kites are desks All blacks are roads
All desks are notes
All notes are mountains (V) Statements:
Some papers are boxes
a) Only I and IV follow Some boxes are boards
b) Only II and V follow Some boards are blacks
c) Only III and V follow Some blacks are roads
d) Only I and IV follow
e)Only II and V follow a) Only I and V follow
b) Only II and V follow
188. Conclusions: c) Only III and IV follow
Some roads are boards d) Only II and IV follow
Some blacks are papers is a possibility e) None of these
Some boards are papers is a possibility
Some roads are boxes is a possibility 189. Conclusions:
Some tyres are pens is a possibility
(I) Statements: Some tyres are buses is a possibility
All papers are boxes Some walls are pens
Some boxes are boards Some buses are walls is a possibility
Some boards are blacks
Some blacks are roads (I) Statements:
All pens are walls
(II) Statements: Some walls are tyres
All papers are boxes Some tyres are rounds
All boxes are boards Some rounds are buses
All boards are blacks
All blacks are roads (II) Statements:
No pens are walls
(III) Statements: All walls are tyres

www.ibpsguide.com | estore.ibpsguide.com | www.sscexamguide.com


86
Exclusively on New Pattern – Reasoning Ability eBook

All tyres are rounds


Some rounds are buses (II) Statements:
All stones are Sheets
(III) Statements: No sheets is ring
All pens are walls Some rings are woods
Some walls are tyres All woods are windows
All tyres are rounds
No rounds are buses (III) Statements:
No stones are Sheets
(IV) Statements: No sheets is ring
All pens are walls Some rings are woods
All walls are tyres All woods are windows
All tyres are rounds
All rounds are buses (IV) Statements:
All stones are Sheets
(V) Statements: No sheets is ring
No pens are walls Some rings are woods
All walls are tyres Some woods are windows
Some tyres are rounds
Some rounds are buses (V) Statements:
All stones are Sheets
a) Only I and IV follow All sheets is ring
b) Only II and V follow All rings are woods
c) Only III and IV follow All woods are windows
d) Only I and V follow
e)Only II and III follow a) Only I follow
b) Only II follow
190.Conclusions: c) Only III follow
Some windows are rings d) Only IV follows
Some woods are not sheets e)Only V follows
No stone is a ring
Some windows are stone is a possibility 191. Conclusions:
Some buildings are sticks
(I) Statements: All buildings are tables is a possibility
All stones are Sheets Some houses are tables
All sheets is ring Some houses are sticks
Some rings are woods
All woods are windows (I) Statements:

www.ibpsguide.com | estore.ibpsguide.com | www.sscexamguide.com


87
Exclusively on New Pattern – Reasoning Ability eBook

Some tables are sticks Some tents are not chairs


All sticks are houses Some tents are roads
Some houses are buildings
Some buildings are homes (I) Statements:
Some boxes are chairs
(II) Statements: No chairs is roads
Some tables are sticks No roads are tents
Some sticks are houses Some roads are darks
All houses are buildings
Some buildings are homes (II) Statements:
Some boxes are chairs
(III) Statements: All chairs is roads
Some tables are sticks All roads are tents
All sticks are houses Some roads are darks
All houses are buildings
Some buildings are homes (III) Statements:
Some boxes are chairs
(IV) Statements: Some chairs is roads
Some tables are sticks All roads are tents
Some sticks are houses Some roads are darks
Some houses are buildings
Some buildings are homes (IV) Statements:
No boxes are chairs
(V) Statements: No chairs is roads
Some tables are sticks No roads are tents
No sticks are houses Some roads are darks
All houses are buildings
Some buildings are homes (V) Statements:
Some boxes are chairs
a) Only I follow No chairs is roads
b) Only II follow All roads are tents
c) Only III follow Some roads are darks
d) Only IV follows
e) Only V follows a) Only I follow
b) Only II follow
192. Conclusions: c) Only III follow
Some boxes are not roads d) Only IV follows
Some tents are boxes is a possibility e) Only V follows

www.ibpsguide.com | estore.ibpsguide.com | www.sscexamguide.com


88
Exclusively on New Pattern – Reasoning Ability eBook

b) Only II follow
193. Conclusions: c) Only III follow
Some glasses are cups is a possibility d) Only V follows
Some tables are not jugs e) Only I follow
Some stars are not glasses
Some tables are stars is a possibility 194. Conclusions:
Some pots are chairs
(I) Statements: Some buses are bottles is a possibility
All cups are stars Some rats are bottles
Some stars are jugs Some pots are buses is a possibility
Some jug is glass
Some glasses are tables (I) Statements:
Some chairs are bottles
(II) Statements: All bottles are pots
All cups are stars All pots are rats
Some stars are jugs Some rats are buses
No jug is glass (II) Statements:
Some glasses are tables Some chairs are bottles
Some bottles are pots
(III) Statements: All pots are rats
All cups are stars Some rats are buses
Some stars are jugs (III) Statements:
No jug is glass Some chairs are bottles
No glasses are tables All bottles are pots
Some pots are rats
(IV) Statements: Some rats are buses
All cups are stars (IV) Statements:
All stars are jugs Allchairs are bottles
All jugs is glass All bottles are pots
Some glasses are tables All pots are rats
All rats are buses
(V) Statements: (V) Statements:
All cups are stars Some chairs are bottles
Some stars are jugs All bottles are pots
All jugs is glass Some pots are rats
Some glasses are tables All rats are buses

a) Only IV follows a) Only I and IV follow

www.ibpsguide.com | estore.ibpsguide.com | www.sscexamguide.com


89
Exclusively on New Pattern – Reasoning Ability eBook

b) Only II and V follow b) Only II and IV follow


c) Only III and IV follow c) Only III and V follow
d) Only II and IV follow d) Only I and IV follow
e)Only II and III follow e) None of these

195. Conclusions: 196. Conclusions:


Some monkeys are birds is a possibility Some houses are benches
All handles are real is a possibility Some roads are horses is a possibility
Some tigers are monkeys is a possibility Some roads are houses
Some tigers are birds Some lions are benches is a possibility

(I) Statements: (I) Statements:


All birds are handles Some benches are horses
No handles are tigers Some horses are houses
All tigers are real Some houses are lions
Some real are monkeys All lions are roads
(II) Statements: (II) Statements:
All birds are handles Some benches are horses
Some handles are tigers All horses are houses
Some tigers are real Some houses are lions
Some real are monkeys Some lions are roads
(III) Statements: (III) Statements:
All birds are handles All benches are horses
All handles are tigers All horses are houses
Some tigers are real All houses are lions
Some real are monkeys All lions are roads
(IV) Statements: (IV) Statements:
Some birds are handles Some benches are horses
Some handles are tigers All horses are houses
Some tigers are real Some houses are lions
Some real are monkeys All lions are roads
(V) Statements: (V) Statements:
All birds are handles All benches are horses
All handles are tigers All horses are houses
All tigers are real Some houses are lions
All real are monkeys Some lions are roads

a) Only I and V follow a) Only I and V follow

www.ibpsguide.com | estore.ibpsguide.com | www.sscexamguide.com


90
Exclusively on New Pattern – Reasoning Ability eBook

b) Only II and V follow b) Only II follows


c) Only III and V follow c) Only III follow
d) Only III and IV follow d) Only IV follows
e) None of these e)Only V follows

197. Conclusions: 198. Conclusions:


Some dresses are sticks is a possibility Some fats are sofas
Some shirts are lamps Some beds are tablets
Some flowers are shirts is a possibility Some fats are tablets
Some sticks are flowers is a possibility All fats are sofas is a possibility
(I) Statements:
(I) Statements: All mobiles are tablets
Some sticks are lamps All tablets are sofas
Some flowers are lamps Some sofas are beds
All lamps are dresses All beds are fats
Some dresses are shirts (II) Statements:
(II) Statements: All mobiles are tablets
Some sticks are lamps All tablets are sofas
Some flowers are lamps All sofas are beds
Some lamps are dresses Some beds are fats
Some dresses are shirts (III) Statements:
(III) Statements: All mobiles are tablets
All sticks are lamps Some tablets are sofas
All flowers are lamps Some sofas are beds
Some lamps are dresses Some beds are fats
Some dresses are shirts (IV) Statements:
(IV) Statements: All mobiles are tablets
Some sticks are lamps All tablets are sofas
Some flowers are lamps All sofas are beds
Some lamps are dresses All beds are fats
No dresses are shirts (V) Statements:
(V) Statements: All mobiles are tablets
Some sticks are lamps All tablets are sofas
Some flowers are lamps No sofas are beds
Some lamps are dresses All beds are fats
All dresses are shirts a) Only I follow
b) Only II follows
a) Only I follow c) Only III follow

www.ibpsguide.com | estore.ibpsguide.com | www.sscexamguide.com


91
Exclusively on New Pattern – Reasoning Ability eBook

d) Only IV follows e) Only V follows


e) Only V follows
200. Conclusions:
199. Conclusions: Some discs are keys
Some plants are bowls Some boards are books is a possibility
Some slides are spoons is a possibility Some discs are books
All spoons are bowls Some keys are files is a possibility
Some forks are plants (I) Statements:
(I) Statements: Some books are files
Some plants are spoons Some files are discs
All spoons are forks Some discs are boards
All forks are bowls All boards are keys
Some bowls are slides (II) Statements:
(II) Statements: Some books are files
Some plants are spoons All files are discs
Some spoons are forks Some discs are boards
All forks are bowls All boards are keys
Some bowls are slides (III) Statements:
(III) Statements: Some books are files
Some plants are spoons All files are discs
Some spoons are forks Some discs are boards
All forks are bowls Some boards are keys
All bowls are slides (IV) Statements:
(IV) Statements: All books are files
Some plants are spoons All files are discs
All spoons are forks All discs are boards
Some forks are bowls All boards are keys
Some bowls are slides (V) Statements:
(V) Statements: Some books are files
Some plants are spoons Some files are discs
Some spoons are forks Some discs are boards
Some forks are bowls Some boards are keys
Some bowls are slides
a) Only I and IV follow
a) Only I follow b) Only II and IV follow
b) Only IV follows c) Only III and IV follow
c) Only II follow d) Only I and V follow
d) Only III follows e) Only III and V follow

www.ibpsguide.com | estore.ibpsguide.com | www.sscexamguide.com


92
Exclusively on New Pattern – Reasoning Ability eBook

101. Answer: c)

Storms
Thunders

Clouds Cyclones Lightning

102. Answer: a)

Pins Needles Handles Locks Keys

103. Answer: d)

Dams
Mountains

Lakes
Rivers
Hills

www.ibpsguide.com | estore.ibpsguide.com | www.sscexamguide.com


93
Exclusively on New Pattern – Reasoning Ability eBook

104. Answer: b)

Files
Receipts Papers
Drafts
Books

105. Answer: e)

Caps

Lids
Containers
Jars
Bottles

106. Answer: b)

Leaves Flowers Fruit


Branches
Stems

www.ibpsguide.com | estore.ibpsguide.com | www.sscexamguide.com


94
Exclusively on New Pattern – Reasoning Ability eBook

107. Answer: c)

Leopards
Tigers
Lions
Wolf
Elephant

108. Answer: e)

Jackets

Caps Umbrellas Trousers


Raincoats

109. Answer: a)

Televisions
Fans Machines
Coolers
Computers

www.ibpsguide.com | estore.ibpsguide.com | www.sscexamguide.com


95
Exclusively on New Pattern – Reasoning Ability eBook

110.Answer: d)

Blades

Staplers

Erasers Sharpeners
Keys

111.Answer: c)

Oranges

Plums

Apples
Guava
Grapes

112. Answer: d)

Cats

Pets Dogs
Goat

Rats

www.ibpsguide.com | estore.ibpsguide.com | www.sscexamguide.com


96
Exclusively on New Pattern – Reasoning Ability eBook

113. Answer: e)

Sunflowers
Marigolds
Roses Jasmines
Lilies

114. Answer: a)

Restaurants
Flats Houses Bungalows

Hotel

115.Answer: b)

Sweets
Lemon Chilly
Desserts
Carrot

www.ibpsguide.com | estore.ibpsguide.com | www.sscexamguide.com


97
Exclusively on New Pattern – Reasoning Ability eBook

116. Answer: e)

Pens Sticks
Scales
Canes Weight

117. Answer: d)

Bags
Folders Boxes Containers

Sacks

118. Answer: c)

Animals
Insects Birds
Amphibian
Pest

119. Answer: b)

Solutions

Paints
Solids
Colours Liquids

www.ibpsguide.com | estore.ibpsguide.com | www.sscexamguide.com


98
Exclusively on New Pattern – Reasoning Ability eBook

120. Answer: a)

Doors
Keys
Locks

Windows Floors

121. Answer: d)

Novels

Books Poems
Stories
Movie

122. Answer: a)

Trousers

Bags
Suits Coats
Shirts

www.ibpsguide.com | estore.ibpsguide.com | www.sscexamguide.com


99
Exclusively on New Pattern – Reasoning Ability eBook

123. Answer: b)

Fruits

Flowers Juice
Proteins vitaminvitamins
s

124. Answer: c)

Towers
Pillars
Flats
Buildings House

125. Answer: e)

Towels
Shoes Bedsheet

Socks ssheesheets Blanket

www.ibpsguide.com | estore.ibpsguide.com | www.sscexamguide.com


100
Exclusively on New Pattern – Reasoning Ability eBook

126. Answer: c)

Flute
Pianos Violins Guitars
Drums

127. Answer: e)

Boulders
Rocks Mountains
Stones Hills

128. Answer: a)

Glasses
Clothes
Plastics
Wood
Metals

129. Answer: d)

Balloons
Kites
Gliders
Airplanes Helicopters

www.ibpsguide.com | estore.ibpsguide.com | www.sscexamguide.com


101
Exclusively on New Pattern – Reasoning Ability eBook

130. Answer: b)

Warriors

Kings

Soldiers Sentries

131. Answer: e)

Petals

Bins Flowers
Jungles

Root

132. Answer: d)

Stands
Fans

Poles Pens Boxes

133. Answer: c)

Bands
Scales Rings
Metals
Weights

www.ibpsguide.com | estore.ibpsguide.com | www.sscexamguide.com


102
Exclusively on New Pattern – Reasoning Ability eBook

134. Answer: b)

House Beads
Cycles Tubes Rains

135. Answer: a)

Pearls

Stones

Shells Boxes Container

136. Answer: d)

Institutes
Schools Colleges Hostels
Office

137. Answer: c)

Umbrellas Shirts
Raincoats Blazer Suits

www.ibpsguide.com | estore.ibpsguide.com | www.sscexamguide.com


103
Exclusively on New Pattern – Reasoning Ability eBook

138. Answer: e)

Tables

Chairs Rooms
Sofa Desks

139. Answer: e)

Computers Boards Bulbs


Chalks Tubelight

140. Answer: c)

Jeeps Buses
Boats Trains Scooters

141. Answer: b)

Cooks
Poets
Engineers
Merchants
Teachers

www.ibpsguide.com | estore.ibpsguide.com | www.sscexamguide.com


104
Exclusively on New Pattern – Reasoning Ability eBook

142. Answer: c)

Nuts
Screws
Nails
Tools Hammers

143. Answer: e)

Books
Glassesg Bags glasses
Pens Spoons

144. Answer: b)

Floors

Paints
Tiles
Doors Stones

145. Answer: d)

Birds
Eagles
Snakes
Rabbits Animals

www.ibpsguide.com | estore.ibpsguide.com | www.sscexamguide.com


105
Exclusively on New Pattern – Reasoning Ability eBook

146. Answer: c)

Computers
Diaries
Camera Calculator

Notebooks

147. Answer: b)

Moons
Asteroids
asteroid Stars

Rocks
Planets

148. Answer: e)

Bats Toys
Plastics Mirrors

Glass

www.ibpsguide.com | estore.ibpsguide.com | www.sscexamguide.com


106
Exclusively on New Pattern – Reasoning Ability eBook

149. Answer: d)

Lawyers
Advocates

Graduates Judges
Doctors

150. Answer: a)

Roses Flowers Plants


Leaves
Buds

151. Answer: c)
Statement (I)

Papers
Books
Journals Cards Boards

Statement (III)

Journals
Papers
Boards
Cards
Books

www.ibpsguide.com | estore.ibpsguide.com | www.sscexamguide.com


107
Exclusively on New Pattern – Reasoning Ability eBook

Statement (V)

Boards

Books
Cards
Journals
Papers

152. Answer: b)

Statement (II)

Grapes Apples Guavas


Bananas Pomegranate

153. Answer: e)

Statement (II)

Floors
Walls Rooms
Doors
Windows

www.ibpsguide.com | estore.ibpsguide.com | www.sscexamguide.com


108
Exclusively on New Pattern – Reasoning Ability eBook

Statement (V)

Windows

Walls
Doors Floors
Rooms

154. Answer: a)

Statement (V)

Spoons Forks Plates


Bowls Utensils

155. Answer: d)

Statement (III)

Desks Benches
Tables

Chairs

Sofas

www.ibpsguide.com | estore.ibpsguide.com | www.sscexamguide.com


109
Exclusively on New Pattern – Reasoning Ability eBook

156. Answer: b)

Statement (II)

Sweets Chocolates Mints Food Diet

Statement (III)

Foods
Chocolates Diet
Sweets
Mints

Statement (V)

Diet
Food

Mints

Chocolates

Sweets

www.ibpsguide.com | estore.ibpsguide.com | www.sscexamguide.com


110
Exclusively on New Pattern – Reasoning Ability eBook

157. Answer: c)

Statement (I)

Lawyers
Engineers
Doctors
Businessmen

Teachers

158. Answer: d)

Statement (V)

Glasses
Plastics Clothes
Sponges
Liquids

159. Answer: a)

Statement (III)

Beaches

Sands Shores Trees


Hotels

160. Answer: e)

Statement (II)

Parrots Eagles
Pigeons Sparrows
Crows

www.ibpsguide.com | estore.ibpsguide.com | www.sscexamguide.com


111
Exclusively on New Pattern – Reasoning Ability eBook

161. Answer: b)

Statement (I)

Lamps
Chairs Trolleys

Tables
Cushions

Statement (II)

Lamps Trolleys
Cushions

Tables Chairs

162. Answer: a)

Statement (IV)

Toys
Dolls Sticks
Boxes
Gems

www.ibpsguide.com | estore.ibpsguide.com | www.sscexamguide.com


112
Exclusively on New Pattern – Reasoning Ability eBook

163. Answer: e)

Statement (V)

Vegetables
Plants
Jungles
Flower Trees

164. Answer: c)

Statement (I)

Hammers Sticks
Knives Poles
Pencils

Statement (V)

Pencils
Sticks
Poles

Hammers

Knives

www.ibpsguide.com | estore.ibpsguide.com | www.sscexamguide.com


113
Exclusively on New Pattern – Reasoning Ability eBook

165. Answer: d)

Statement (I)

Pens
Books

Tables
Desks Chairs

Statement (III)

Books Pens
Desks Chairs Tables

Statement (IV)

Tables
Chairs
Desks

Pens

Books

www.ibpsguide.com | estore.ibpsguide.com | www.sscexamguide.com


114
Exclusively on New Pattern – Reasoning Ability eBook

166. Answer: a)

Statement (II)

Pins Magnets Scales Buses


Trucks

Statement (V)

Buses
Trucks
Scales

Magnets

Pins

167. Answer: e)

Statement (III)

Buildings

Houses
Roads
Trees
Jungles

www.ibpsguide.com | estore.ibpsguide.com | www.sscexamguide.com


115
Exclusively on New Pattern – Reasoning Ability eBook

168. Answer: c)
Statement (V)

Tablets
Packets Toys
Bag
Puppets

169. Answer: d)

Statement (I)

Desks Tables Chairs Benches Cots

Statement (III)

Cots
Benches
Chairs

Tables

Desks

www.ibpsguide.com | estore.ibpsguide.com | www.sscexamguide.com


116
Exclusively on New Pattern – Reasoning Ability eBook

170. Answer: b)

Statement (II)

Rings
Bangles Bracelets

Jewels Stones

Statement (III)

Stones
Jewels
Bracelets

Rings Bangles

171. Answer: d)
Statement (IV)

Stars

Planets Sun Gases


Moons

www.ibpsguide.com | estore.ibpsguide.com | www.sscexamguide.com


117
Exclusively on New Pattern – Reasoning Ability eBook

172. Answer: c)

Statement (III)

Diaries

Pinks Chairs
Drawers
Dolls

Statement (V)

Chairs
Drawers

Pinks

Diaries

Dolls

173. Answer: a)Statement (I)

Buildings Rivers
Trucks
Roads
Mountains

www.ibpsguide.com | estore.ibpsguide.com | www.sscexamguide.com


118
Exclusively on New Pattern – Reasoning Ability eBook

174.Answer: e)
Statement (II)

Rounds

Rivers
Tables Bills Conical

Statement (V)

Round
Rivers
Bills Conical
Tables

175. Answer: b)
Statement (II)

Waters
Desks Tables
Books

Tins

176. Answer: c)
Statement (III)

Trains Buses Trucks Boats Cars

www.ibpsguide.com | estore.ibpsguide.com | www.sscexamguide.com


119
Exclusively on New Pattern – Reasoning Ability eBook

Statement (V)

Cars
Boats

Trucks
Buses
Trains

177. Answer: d)

Statement (III)

Wheels

Tyres

Bangles
Rings
Chains

Statement (IV)

Rings
Chains Bangles Wheels
Tyres

www.ibpsguide.com | estore.ibpsguide.com | www.sscexamguide.com


120
Exclusively on New Pattern – Reasoning Ability eBook

178. Answer: a)
Statement (I)

Sea Jungles

Stones
Roads
Hills

Statement (III)

Stones Sea
Hills Roads
Jungles

179. Answer: b)

Statement (II)

Threads Houses

Clothes

Needles
Rooms

www.ibpsguide.com | estore.ibpsguide.com | www.sscexamguide.com


121
Exclusively on New Pattern – Reasoning Ability eBook

180. Answer: e)

Statement (IV)

Roofs
Walls

Curtains
Toys

Bags

Statement (V)

Bags Curtains Walls


Toys Roofs

181. Answer: d)

Statement (II)

Lanes
Trucks
Bottles

Buildings
Bikes

www.ibpsguide.com | estore.ibpsguide.com | www.sscexamguide.com


122
Exclusively on New Pattern – Reasoning Ability eBook

Statement (V)

Bikes Bottles
Buildings Trucks Lanes

182. Answer: b)

Statement (II)

Lines
Tele
Cards

Phones
Copies

Radio

183. Answer: a)

Statement (I)

Bulls
Bags Dogs
Cats

Cycle

www.ibpsguide.com | estore.ibpsguide.com | www.sscexamguide.com


123
Exclusively on New Pattern – Reasoning Ability eBook

Statement (V)

Dogs Bulls

Cats Cycle
Bags

184. Answer: d)Statement (I)

Guns
Bats
Balls

Sticks

Umbrellas

Statement (IV)

Guns
Umbrella Stick Bats
Balls

www.ibpsguide.com | estore.ibpsguide.com | www.sscexamguide.com


124
Exclusively on New Pattern – Reasoning Ability eBook

185. Answer: c)Statement (III)

Cards Notebooks Dictionaries Envelopes


Files

Statement (IV)

Envelopes
Files
Dictionaries

Notebooks

Cards

186.Answer: e)
Statement (IV)

Dresses

Codes
Roads
Cars

Bus

www.ibpsguide.com | estore.ibpsguide.com | www.sscexamguide.com


125
Exclusively on New Pattern – Reasoning Ability eBook

Statement (V)

Codes

Buses Roads
Cars Dresses

187.Answer: c)
Statement (III)

Colours Kites Desks


Mountains
Notes

Statement (V)

Mountains
Notes
Desks

Kites

Colours

www.ibpsguide.com | estore.ibpsguide.com | www.sscexamguide.com


126
Exclusively on New Pattern – Reasoning Ability eBook

188. Answer: d)

Statement (II)

Roads
Blacks
Boards

Boxes

Papers

Statement (IV)

Boxes

Boards Blacks Roads

Papers

189. Answer: a)

Statement (I)

Walls

Tyres Rounds Buses

Pens

www.ibpsguide.com | estore.ibpsguide.com | www.sscexamguide.com


127
Exclusively on New Pattern – Reasoning Ability eBook

Statement (IV)

Buses
Rounds
Tyres

Walls

Pens

190. Answer: b)

Statement (II)

Sheets

Ring Woods Windows


Stones

191. Answer: c)

Statement (III)

Sticks Houses Buildings


Tables Homes

www.ibpsguide.com | estore.ibpsguide.com | www.sscexamguide.com


128
Exclusively on New Pattern – Reasoning Ability eBook

192. Answer: e)

Statement (V)

Tents

Boxes Chairs

Roads Darks

193. Answer: b)

Statement (II)

Stars

Jugs
Glass Tables
Cups

194. Answer: a)

Statement (I)

Rats

Pots Buses
Chairs Bottles

www.ibpsguide.com | estore.ibpsguide.com | www.sscexamguide.com


129
Exclusively on New Pattern – Reasoning Ability eBook

Statement (IV)

Buses
Rats
Pots

Bottles

Chairs

195. Answer: c)

Statement (III)

Tigers
Handles
Real Monkeys
Birds

www.ibpsguide.com | estore.ibpsguide.com | www.sscexamguide.com


130
Exclusively on New Pattern – Reasoning Ability eBook

Statement (V)

Monkeys
Real

Tigers

Handles Birds

196. Answer: d)

Statement (III)

Roads
Lions
Houses

Horses Benches

www.ibpsguide.com | estore.ibpsguide.com | www.sscexamguide.com


131
Exclusively on New Pattern – Reasoning Ability eBook

Statement (IV)

Houses
Benchers Roads
Horses Lions

197. Answer: e)

Statement (V)

Flowers

Sticks Lamps

Dresses
Shirts

198. Answer: d)

Statement (IV)

Fats
Beds
Sofas
Tablets

Mobiles

www.ibpsguide.com | estore.ibpsguide.com | www.sscexamguide.com


132
Exclusively on New Pattern – Reasoning Ability eBook

199. Answer: a)

Statement (I)

Bowls
Forks
Plants Spoons Slides

200. Answer: b)

Statement (II)

Discs

Books Boards
Files Keys

Statement (IV)

Keys
Boards
Discs
Files

Books

www.ibpsguide.com | estore.ibpsguide.com | www.sscexamguide.com


133
Exclusively on New Pattern – Reasoning Ability eBook

INPUT OUTPUT
Directions (201-205): Study the following information carefully and answer the given questions.

A number arrangement machine arranges two digit numbers into a typical manner. Each step gives output taking
input from the previous step. The following is an illustration of Input and rearrangement. Using the illustration

answer the question given below.


Input:

3 4 5 3 2 4 2 1 3 1 2 4

Step I
3 8 5 9 8 8

Step II
2 0 2 1

Step III
8 9

Step IV
17

Input:

7 3 4 5 8 3 2 1 1 2 3 1

201.Which of the following combination represent the first digit of the third number and second digit of the first
number in step I of the given input?
a) 6, 7
b) 8, 6
c) 9, 8
d) 8, 7
e) 5, 8

www.ibpsguide.com | estore.ibpsguide.com | www.sscexamguide.com


134
Exclusively on New Pattern – Reasoning Ability eBook

202. Which of the following represent the value, the second digit of the third number in step I divided by the
second digit of the second number in step II?
a) 4
b) 1
c) 6
d) 3
e) 2

203.Which of the following represent the sum of the first digit of the second number and second digit of the first
number in step II?
a) 3
b) 5
c) 2
d) 9
e) 7

204.If the value ‘6^2’ is subtracted from the final output, then what will be the resultant value?
a) 7
b) 18
c) 13
d) 22
e) 9

205.What is the addition of two numbers obtained in step II?


a) 36
b) 33
c) 23
d) 46
e) 43

Input:

7 3 4 5 8 3 2 1 1 2 3 1

Step I
7 6 8 5 8 9

www.ibpsguide.com | estore.ibpsguide.com | www.sscexamguide.com


135
Exclusively on New Pattern – Reasoning Ability eBook

Step II
2 0 2 3

Step III
8 35

Step IV
43

Step I – First digit multiply with the second digit and second digit multiply with first digit

Step II –
First number –Add first digit of the first number with second digit of the second number and first digit of the third
number
Second number –Add second digit of the first number with first digit of the second number and second digit of the
third number

Step III – Sum of cube of both digits


First number – 2^3 + 0^3 = 8
Second number – 2^3 + 3^3 = 35

Step IV –Add both the numbers

201. Answer: b)
202. Answer: d)
203. Answer: c)
204. Answer: a)
205. Answer: e)

Directions (206-210): Study the following information carefully and answer the givenquestions.
A number arrangement machine arranges two digit numbers into a typical manner. Each step gives output taking
input from the previous step. Thefollowing is an illustration of Input and rearrangement. Using the
illustrationanswer the question given below.

www.ibpsguide.com | estore.ibpsguide.com | www.sscexamguide.com


136
Exclusively on New Pattern – Reasoning Ability eBook

Input:

3 5 8 3 4 6 4 5 2 1 2 4

Step I
8 9 9 5 8 8

Step II
5 0 4 4

Step III
1 -4

Step IV
-3

Input:

1 5 2 5 8 5 2 3 4 6 3 1

206.What is the multiplication of two numbers obtained in step III?


a) 6
b) 10
c) 12
d) 5
e) 0

207. Which of the following combination represent the first digit of the third number and second digit of the second
number in step I of the given input?
a) 9, 9
b) 9, 8
c) 9, 4
d) 7, 9

www.ibpsguide.com | estore.ibpsguide.com | www.sscexamguide.com


137
Exclusively on New Pattern – Reasoning Ability eBook

e) 6, 5

208. What is the difference of both numbers obtained in step II?


a) 5
b) 10
c) 8
d) 6
e) 9

209.If the value ‘-8’ is multiply with the final output, then what will be the resultant value?
a) 56
b) 48
c) 64
d) 72
e) 40

210.Which of the following represent the difference between the first digit of the second number and second digit
of the first number in step II?
a) 6
b) 4
c) 2
d) 1
e) 3

Input:

1 5 2 5 8 5 2 3 4 6 3 1

Step I
4 7 8 9 9 8

Step II
4 2 4 8

www.ibpsguide.com | estore.ibpsguide.com | www.sscexamguide.com


138
Exclusively on New Pattern – Reasoning Ability eBook

Step III
0 -6

Step IV
-6

Step I – First digit add with the second digit and second digit add with first digit

Step II –
First number – Sum of first digit multiply with 2
Second number – Sum of the second digit multiply with 2

Step III –
First number – Subtract first digit of the first number with the first digit of the second number
Second number – Subtract second digit of the first number with the second digit of the second number

Step IV – Add both the numbers

206. Answer: e)
207. Answer: a)
208. Answer: d)
209. Answer: b)
210. Answer: c)

Directions (211-215): Study the following information carefully and answer the givenquestions.
A number arrangement machine arranges two digit numbers into a typical manner. Each step gives output taking
input from the previous step. Thefollowing is an illustration of Input and rearrangement. Using the
illustrationanswer the question given below.
Input:

8 7 5 3 9 6 1 4 2 3 3 4

Step I
4 6 2 1 5 3

Step II
1 0 1 1

www.ibpsguide.com | estore.ibpsguide.com | www.sscexamguide.com


139
Exclusively on New Pattern – Reasoning Ability eBook

Step III
1 2

Step IV
2

Input:

3 6 8 9 5 7 2 1 6 2 2 1

211.Which of the following combination represent the second digit of the third number and second digit of the first
number in step I of the given input?
a) 4, 6
b) 4, 5
c) 6, 4
d) 3, 2
e) 5, 4

212. If both numbers in the step II is divided by 2, then what will be the sum of both numbers?
a) 24
b) 12
c) 10
d) 36
e) 48

213.If the value ‘4.5’ is divided from the final output, then what will be the resultant value?
a) 2
b) 10
c) 6
d) 8
e) 4

214.Which of the following represent the difference between the first digit of the second number in step II and
second digit of the second number in step I?
a) 3
b) 4
c) 2

www.ibpsguide.com | estore.ibpsguide.com | www.sscexamguide.com


140
Exclusively on New Pattern – Reasoning Ability eBook

d) 5
e) 6

215.What is the multiplication of two numbers obtained in step II?


a) 125
b) 148
c) 196
d) 144
e) 169

Input:

3 6 8 9 5 7 2 1 6 2 2 1

Step I
2 4 6 3 4 5

Step II
1 2 1 2

Step III
3 3

Step IV
9

Step I – First digit subtract with the second digit and second digit subtract with the first digit

Step II –
First number – Sum of second digit of three numbers
Second number – Sum of first digit of three numbers

Step III – Add both digits of the number


Step IV – Multiply both numbers

www.ibpsguide.com | estore.ibpsguide.com | www.sscexamguide.com


141
Exclusively on New Pattern – Reasoning Ability eBook

211. Answer: e)
212. Answer: b)
213. Answer: a)
214. Answer: c)
215. Answer: d)

Directions (216-220): Study the following information carefully and answer the given questions.
A number arrangement machine arranges two digit numbers into a typical manner. Each step gives output taking
input from the previous step. The following is an illustration of Input and rearrangement. Using the illustration
answer the question given below.

Input:

1 6 8 4 3 1 8 9 2 2 3 2

Step I
1 2 4 2 4 3

Step II
1 6 1 2

Step III
32 24

Step IV
8

Input:

9 8 4 6 2 3 1 5 1 2 5 1

www.ibpsguide.com | estore.ibpsguide.com | www.sscexamguide.com


142
Exclusively on New Pattern – Reasoning Ability eBook

216.What is the addition of two numbers obtained in step III?


a) 50
b) 32
c) 74
d) 60
e) 66

217.Which of the following represent the difference between the first digit of the second number and second digit
of the first number in step II?
a) 2
b) 4
c) 0
d) 6
e) 5

218. If each digit in the step I is multiply with 2 and then added, then what will be the sum of all numbers in step I?
a) 28
b) 42
c) 34
d) 52
e) 20

219.If the value ‘3’ is multiply with the final output, then what will be the resultant value?
a) 60
b) -150
c) -75
d) 90
e) -108

220.Which of the following combination represent the first digit of the first number in step II and second digit of the
third number in step I of the given input?
a) 0, 1
b) 6, 5
c) 2, 1
d) 6, 1
e) 4, 2

www.ibpsguide.com | estore.ibpsguide.com | www.sscexamguide.com


143
Exclusively on New Pattern – Reasoning Ability eBook

Input:

9 8 4 6 2 3 1 5 1 2 5 1

Step I
3 4 2 6 1 1

Step II
0 6 2 4

Step III
12 48

Step IV

-36

Step I – First digit divided with the second digit and the second digit divided by the first digit

Step II –
First number – Multiply all the first digit of three numbers
Second number – Multiply all the second digit of three numbers

Step III –
First number – Multiply the first number value × 2 = 06 × 2 = 12
Second number – Multiply the second number value × 2 = 24 × 2 = 48

Step IV – Subtract the first number with second number

216. Answer: d)
217. Answer: b)
218. Answer: c)
219. Answer: e)
220. Answer: a)

www.ibpsguide.com | estore.ibpsguide.com | www.sscexamguide.com


144
Exclusively on New Pattern – Reasoning Ability eBook

Directions (221-225): Study the following information carefully and answer the given questions.
A number arrangement machine arranges two digit numbers into a typical manner. Each step gives output taking
input from the previous step. The following is an illustration of Input and rearrangement. Using the illustration
answer the question given below.

Input:

3 5 7 3 1 2 2 1 3 1 9 4

Step I
6 5 7 9 8 9

Step II
2 3 2 1

Step III
13 5

Step IV
18

Input:

2 3 7 3 1 2 9 5 2 1 1 1

221. Which of the following value to add with final input to make the value perfect square?
a) 17
b) 18
c) 13
d) 12
e) 11

222. If both the numbers in the step II is multiply with 2 and then added, then what will be the final sum?

www.ibpsguide.com | estore.ibpsguide.com | www.sscexamguide.com


145
Exclusively on New Pattern – Reasoning Ability eBook

a) 98
b) 72
c) 64
d) 68
e) 48

223.What is the multiplication of two numbers obtained in step II?


a) 195
b) 255
c) 165
d) 345
e) 285

224.Which of the following combination represent the first digit of the third number and second digit of the first
number in step I of the given input?
a) 5, 6
b) 9, 3
c) 9, 4
d) 7, 3
e) 6, 4

225.Which of the following represent the difference between the first digit of the second number and second digit
of the first number in step II?
a) 8
b) 5
c) 4
d) 6
e) 3

Input:

2 3 7 3 1 2 9 5 2 1 1 1

Step I
4 3 7 6 9 5

www.ibpsguide.com | estore.ibpsguide.com | www.sscexamguide.com


146
Exclusively on New Pattern – Reasoning Ability eBook

Step II
1 9 1 5

Step III
82 26

Step IV
108

Step I – First digit multiply with the second digit and second digit multiply with the first digit

Step II –
First number –Add first digit of the first number with second digit of the second number and first digit of the third
number
Second number –Add second digit of the first number with first digit of the second number and second digit of the
third number

Step III – sum of squares of both digits


First number – 1^2 + 9^2 = 1 + 81 = 82
Second number – 1^2 + 5^2 = 1 + 25 = 26

Step IV –Add both numbers


221. Answer: c)
222. Answer: d)
223. Answer: e)
224. Answer: b)
225. Answer: a)

Directions (226-230): Study the following information carefully and answer the given questions.
A number arrangement machine arranges two digit numbers into a typical manner. Each step gives output taking
input from the previous step. The following is an illustration of Input and rearrangement. Using the illustration
answer the question given below.

www.ibpsguide.com | estore.ibpsguide.com | www.sscexamguide.com


147
Exclusively on New Pattern – Reasoning Ability eBook

Input:

5 6 2 3 6 2 1 3 1 7 2 4

Step I

1 5 6 1 4 3 2 4 4

Step II
1 3 1 3 0 4

Step III
1 0 2 6

Step IV
1 6

Step V
7

Input:

3 4 8 1 6 2 2 4 5 9 3 7

226.What is the result of first number subtracted from the second number in step III?
a) 53
b) 61
c) 47
d) 39
e) 26

227. If each digit in the step I is added with 2, then what will be the second number in same step?
a) 856
www.ibpsguide.com | estore.ibpsguide.com | www.sscexamguide.com
148
Exclusively on New Pattern – Reasoning Ability eBook

b) 748
c) 368
d) 582
e) 947

228.Which of the following represent the difference between the first digit of the second number in step III and
second digit of the second number in step II?
a) 3
b) 5
c) 4
d) 2
e) 1

229.If the value ‘20’ is multiply with the final output, then what will be the resultant value?
a) 20
b) 40
c) 60
d) 80
e) 100

230.Which of the following combination represent the first digit of the second number and second digit of the third
number in step I of the given input?
a) 7, 2
b) 5, 4
c) 6, 8
d) 2, 5
e) 4, 7

Input:

3 4 8 1 6 2 2 4 5 9 3 7

Step I

1 2 8 7 2 5 4 2 6

www.ibpsguide.com | estore.ibpsguide.com | www.sscexamguide.com


149
Exclusively on New Pattern – Reasoning Ability eBook

Step II
1 9 0 6 1 2

Step III
3 8 8 5

Step IV
5 -3

Step V
2

Step I – First digit multiply with the second digit and second digit multiply with first digit

Step II –
First number - Sum of the third digit of all three numbers
Second number – Sum of the second digit of all three numbers
Third number – Sum of the first digit of all three numbers

Step III – Sum of the squares of the number


First number – squares of first digit of the first number, second digit of the second number and first digit of the
third number
= 1^2 + 6^2 + 1^2 = 38
Second number = squares of second digit of the first number, first digit of the second number and second digit of
the third number
= 9^2 + 0^2 +2^2 = 85

Step IV –
First number - Subtract the first digit of the first number from the first digit of the second number
Second number – Subtract the second digit of the first number from the second digit of the second number

Step V - Add both the numbers

226. Answer: c)
227. Answer: e)

www.ibpsguide.com | estore.ibpsguide.com | www.sscexamguide.com


150
Exclusively on New Pattern – Reasoning Ability eBook

228. Answer: d)
229. Answer: b)
230. Answer: a)

Directions (31-35): Study the following information carefully and answer the givenquestions.
A number arrangement machine arranges two digit numbers into a typical manner. Each step gives output taking
input from the previous step. Thefollowing is an illustration of Input and rearrangement. Using the
illustrationanswer the question given below.

Input:

8 7 6 3 5 4 1 8 3 9 4 7

Step I

1 6 8 1 5 6 1 2 8

Step II
8 4 7 8 6 4

Step III
1 6 2 1

Step IV
7 3

Step V
21

Input:

5 6 9 4 7 3 2 5 0 9 3 5

231.What is the addition of two numbers obtained in step III?


a) 29
b) 16
www.ibpsguide.com | estore.ibpsguide.com | www.sscexamguide.com
151
Exclusively on New Pattern – Reasoning Ability eBook

c) 35
d) 18
e) 30

232. If each digit in the step II is subtracted with 2 then added, then what will be the final sum?
a) 25
b) 19
c) 17
d) 20
e) 10

233.If the value ‘9’ is subtracted from the final output, then what will be the resultant value?
a) 5
b) 12
c) 10
d) 15
e) 9

234.Which of the following combination represent the first digit of the third number and third digit of the first
number in step I of the given input?
a) 2, 0
b) 1, 8
c) 6, 1
d) 4, 8
e) 1, 1

235.Which of the following represent the difference between the first digit of the second number and second digit
of the third number in step II?
a) 9
b) 5
c) 5
d) 6
e) 3

www.ibpsguide.com | estore.ibpsguide.com | www.sscexamguide.com


152
Exclusively on New Pattern – Reasoning Ability eBook

Input:

5 6 9 4 7 3 2 5 0 9 3 5

Step I

1 0 8 1 8 4 1 2 6

Step II
5 4 9 2 6 3

Step III
0 9 2 0

Step IV
9 2

Step V
18

Step I – First digit add with the second digit and second digit add with the first digit
Step II –
First number –first number divided by 2
Second number – second number divided by 2
Third number – third number divided by 2

Step III – Addition of numbers


First number – add second digits of all numbers
Second number – add first digit of all numbers

Step IV –
First number –Add the first digit with the second digit
Second number –Add the second digit with the first digit

www.ibpsguide.com | estore.ibpsguide.com | www.sscexamguide.com


153
Exclusively on New Pattern – Reasoning Ability eBook

Step V –Multiply both numbers

231. Answer: a)
232. Answer: c)
233. Answer: e)
234. Answer: b)
235. Answer: d)

Directions (236-240): Study the following information carefully and answer the givenquestions.
A number arrangement machine arranges two digit numbers into a typical manner. Each step gives output taking
input from the previous step. Thefollowing is an illustration of Input and rearrangement. Using the
illustrationanswer the question given below.

Input:

6 5 7 3 5 1 1 6 3 9 7 5

Step I

3 6 5 6 3 9 2 5 7

Step II
1 1 1 4 2 1

Step III
0 2 0 4

Step IV
2 4

Step V
2

Input:

9 3 7 1 8 4 3 9 6 9 2 6

www.ibpsguide.com | estore.ibpsguide.com | www.sscexamguide.com


154
Exclusively on New Pattern – Reasoning Ability eBook

236.Which of the following represent the difference between the first digit of the third number and second digit of
the first number in step II?
a) 3
b) 7
c) 4
d) 6
e) 5

237.What is the multiplication value of two numbers obtained in step III?


a) 96
b) 90
c) 80
d) 72
e) 54

238. If all the three numbers in the step II is added, then what will be the final sum?
a) 45
b) 39
c) 63
d) 58
e) 53

239.If the value ‘5^2’ is added to the final output, then what will be the resultant value?
a) 27
b) 29
c) 33
d) 36
e) 30

240.Which of the following combination represent the first digit of the third number and third digit of the first
number in step I of the given input?
a) 4, 6
b) 6, 8
c) 4, 9
d) 1, 3
e) 4, 3

www.ibpsguide.com | estore.ibpsguide.com | www.sscexamguide.com


155
Exclusively on New Pattern – Reasoning Ability eBook

Input:

9 3 7 1 8 4 3 9 6 9 2 6

Step I

8 1 9 6 3 6 4 8 8

Step II
1 8 1 2 2 3

Step III
0 2 4 8

Step IV
2 4

Step V
2

Step I – First digit multiply with the second digit and second digit multiply with the first digit
Step II –
First number - Sum of the first digit of all three numbers
Second number – Sum of the second digit of all three numbers
Third number – Sum of the third digit of all three numbers
Step III – multiply the numbers
First number - Multiply all the first digit numbers = 1 × 1 × 2 = 02
Second number -Multiply all the second digit numbers = 8 × 2 × 3 = 04
Step IV – Difference between both digits of the number
Step V –Subtract the first digit from the second digit
236. Answer: d)
237. Answer: a)
238. Answer: e)
239. Answer: a)
240. Answer: c)

www.ibpsguide.com | estore.ibpsguide.com | www.sscexamguide.com


156
Exclusively on New Pattern – Reasoning Ability eBook

Directions (241-245): Study the following information carefully and answer the givenquestions.
A number arrangement machine arranges two digit numbers into a typical manner. Each step gives output taking
input from the previous step. Thefollowing is an illustration of Input and rearrangement. Using the
illustrationanswer the question given below.

Input:

2 3 9 4 2 9 7 6 4 7 3 8

Step I

1 1 5 1 6 8 1 5 9

Step II
0 5 4 8 4 5

Step III
0 8 1 8

Step IV
1 0

Step V
1

Input:

7 5 8 3 2 6 9 5 2 4 1 8

241. If the value ‘3’ is subtracted from the final output, then what will be the resultant value?
a) -9
b) -13
c) -7
d) 8
e) 5
242. If each digit in the step II is added with 3, then what will be the final sum of all digits in step II?
a) 28
b) 30
c) 36
d) 22
e) 18
www.ibpsguide.com | estore.ibpsguide.com | www.sscexamguide.com
157
Exclusively on New Pattern – Reasoning Ability eBook

243.Which of the following represent the difference between the third digit of the third number in step I and
second digit of the first number in step III?
a) 7
b) 5
c) 2
d) 1
e) 0
244.Which of the following value represent the third number in step II is divided by the first number in the same
step?
a) 3
b) 2
c) 1.5
d) 1
e) 4.5
245.What is the addition of two numbers obtained in step III?
a) 05
b) 07
c) 12
d) 08
e) 10

Input:

7 5 8 3 2 6 9 5 2 4 1 8

Step I

1 3 7 1 2 5 1 7 6

Step II
2 1 1 0 4 2
Step III
0 7 0 3

Step IV
0 -4

Step V
-4
Step I – First digit add with the second digit and second digit add with the first digit

www.ibpsguide.com | estore.ibpsguide.com | www.sscexamguide.com


158
Exclusively on New Pattern – Reasoning Ability eBook

Step II – Multiply the digits within the number


Step III – Addition of the numbers
First number -Sum of the first digit of all three numbers
Second number - Sum of the second digit of all three numbers
Step IV –
First number - Subtract the first digit of first number from the first digit of the second number
Second number – Subtract the second digit of first number from the second digit of second number
Step V - Add both the numbers
241. Answer: c)
242. Answer: a)
243. Answer: d)
244. Answer: b)
245. Answer: e)

Directions (246-250): Study the following information carefully and answer the given questions.
A number arrangement machine arranges two digit numbers into a typical manner. Each step gives output taking
input from the previous step. Thefollowing is an illustration of Input and rearrangement. Using the illustration
answer the question given below.
Input:

7 5 4 6 1 7 9 3 1 8 3 9

Step I

4 9 5 3 2 6 8 1 9

Step II
3 6 2 2 3 6

Step III
0 8 1 4

Step IV
8 5
Step V
40
Input:

9 4 5 3 2 9 8 3 3 5 2 8

www.ibpsguide.com | estore.ibpsguide.com | www.sscexamguide.com


159
Exclusively on New Pattern – Reasoning Ability eBook

246.What is the multiplication of two numbers obtained in step III?


a) 72
b) 48
c) 56
d) 32
e) 24
247.Which of the following combination represent the first digit of the second number in step III and second digit
of the third number in step I of the given input?
a) 0, 7
b) 0, 9
c) 0, 6
d) 0, 5
e) 0, 4
248. What is the sum of all three numbers obtained in step II?
a) 96
b) 100
c) 98
d) 106
e) 88
249.If the value ‘9^2’ is subtracted from the final output, then what will be the resultant value?
a) 45
b) 9
c) 19
d) 26
e) 33
250.Which of the following represent the difference between the first digit of the second number and second digit
of the third number in step II?
a) 3
b) 0
c) 4
d) 1
e) 2

www.ibpsguide.com | estore.ibpsguide.com | www.sscexamguide.com


160
Exclusively on New Pattern – Reasoning Ability eBook

Input:

9 4 5 3 2 9 8 3 3 5 2 8

Step I

8 1 8 2 5 9 6 4 6

Step II
3 4 3 2 3 2

Step III
0 9 0 8

Step IV
9 8

Step V
72

Step I – First digit multiply with the second digit and second digit multiply with the first digit
Step II – Sum of the digits within the number multiply with 2
First number –8+8+1 = 17 × 2 = 34
Second number – 2+5+9 = 16 × 2 = 32
Third number – 6+4+6 = 16 × 2 = 32
Step III – Sum of the number
First number – Sum of the first digit of all numbers
Second number - Sum of the second digit of all numbers
Step IV – Add the digits within the number
Step V –Multiply both numbers
246. Answer: a)
247. Answer: e)
248. Answer: c)
249. Answer: b)
250. Answer: d)

www.ibpsguide.com | estore.ibpsguide.com | www.sscexamguide.com


161
Exclusively on New Pattern – Reasoning Ability eBook

CODING DECODING

Directions (Q. 251-255): Study the following information carefully and answer the given questions
In a certain code language
"Verb and calculator magnetic" is written as "20*RA, 16&UG, 09!ZW, 08#KD.
"Truthness satisfy kids be" is written as " 21!QB, 08#XL , 04&TC, 27*JT.
"Dancer have reaccept cap" is written as "16+BQ, 12!UT, 08&KL, 09*NQ.
" Baby plays as interesting" is written as "08!IR, 33&LP, 15+VY, 04*MW.

251. What may be the possible code for ‘so calculator’ in the given code language?
a) 04&TC, 20*RA
b) 21!QB, 16&UG
c) 08#XL, 08#KD
d) 21!QB, 09!ZW
e) 27*JT, 09!ZW

252. What is the code for ‘Truthness’ in the given code language?
a) 21!QB
b) 08!IR
c)27*JT
d) 04&TC
e) 08#XL

253. What is the code for ‘Dancer magnetic’ in the given code language?
a) 16+BQ, MW04
b)12!UT, 09!ZW
c) 12!UT, 20*RA
d) 08&KL, 16&UG
e) 12!UT, 16&UG

254. What may be the possible code for ‘have interesting Truthness’ in the given code language?
a) 08!IR, 08&KL, 27*JT
b) 33&LP, 08&KL, 16+BQ
c) 09!ZW, 15+VY, 27*JT
d) 33&LP, 08&KL, 27*JT
e) 09!ZW, 08&KL, 27*JT

255. What is the code for ‘plays Verb’ in the given code language?

www.ibpsguide.com | estore.ibpsguide.com | www.sscexamguide.com


162
Exclusively on New Pattern – Reasoning Ability eBook

a) 16&UG, 33&LP
b) 08#KD, 15+VY
c) 09!ZW, 08!IR
d) 20*RA, 04*MW
e) 16&UG, 33&LP

The logic for all the above code is:


There is count of total number of letter in the word
If it is odd number multiply with 3 (cap – 3 ×3 =09)
If it is even number multiply with 2 (kids – 4 × 2 = 08)
And rest of the code include two letter and one symbol that can be anything.

251. Answer: a)
252. Answer: c)
253. Answer: e)
254. Answer: d)
255. Answer: b)

Directions (Q. 256-160): Study the following information carefully and answer the given questions
In a certain code language
"Crow must welled tonner" is written as "14#R, 12!H, 15&U, 19%X .
"Miss good fault once" is written as "03*Q , 21$D, 15#R, 19!L.
" The Sorry Language repeat " is written as "18%R, 05&K, 16#H , 14!D.
"This answer pad coupon " is written as "04$D , 19*Q, 21%R, 09@K .

256. What is the code for ‘good language’ in the given code language?
a) 15#R, 14!D
b)05&K, 15#R
c) 18%R, 03*Q
d) 19!L, 14!D
e) 16#H , 01$X

257. What is the code for ‘welled’ in the given code language?
a) 19%X
b) 15&U
c)14#R
d) 19*Q
e) 12!H

www.ibpsguide.com | estore.ibpsguide.com | www.sscexamguide.com


163
Exclusively on New Pattern – Reasoning Ability eBook

258. What may be the possible word for ‘03*Q, 16#H,14#R’ in the given code language?
a) repeat welled good
b) fault crow repeat
c) once tonner repeat
d) the good tonner
e) None of these

259. What may be the possible code for ‘answer hiring’ in the given code language?
a) 21%R, 12#O
b) 19*Q, %L18
c) 09@K, *N09
d) 04$D, @K17
e) 21%R, $J21

260. What is the code for ‘crow sorry’ in the given code language?
a) 14#R, 18%R
b) 16#H, 19%X
c) 14!D, 15&U
d)15&U, 18%R
e) 05&K, 12!H

The logic for all the above code is:


The letter denotes second letter of each word + 3
Like “Crow” – Second letter “R” + 3 = U
The number denotes the alphabet number of Third letter (1-26)
Like “The” – “E”- 05
And rest of the one symbol and can be anything.

256. Answer: a)
257. Answer: e)
258. Answer: c)
259. Answer: b)
260. Answer: d)

Directions (Q. 261-265): Study the following information carefully and answer the given questions

www.ibpsguide.com | estore.ibpsguide.com | www.sscexamguide.com


164
Exclusively on New Pattern – Reasoning Ability eBook

In a certain code language


"The college coach is" is written as "56%F, 06&U, 12#F , 30!J.
"Previewed book for" is written as "12$T , 12*F, 20%M, 42@F.
"Cousins named their societies" is written as "90#U, 30!T, 56&U, 30%F .
" To repayment counters rush" is written as "72*U , 20$J, 90#V, 06!P.

261. What may be the possible word for “30!T, 20%M, 72*U” in the given code language?
a) pre named rush
b) to book societies
c) repayment for cousins
d) book their counters
e) None of these

262. What is the code for ‘viewed’ in the given code language?
a) 20%M
b) 12*F
c) 42@F
d) 12$T
e) None of these

263. What may be the possible code for ‘coach pre’ in the given code language?
a) 12$T,12#F
b) 12*F, 30!J
c) 20%M,06&U
d) 42@F,12#F
e) 20%M,56%F

264. What is the code for ‘rush college’ in the given code language?
a) 90#V, 06&U
b) 06!P, 12#F
c) 90#V, 56%F
d)72*U, 06&U
e) 20$J, 56%F

265. What is the possible word for “56&U, 12$T”in the given code language?
a) for cousins
b) named pre
c) book cousins

www.ibpsguide.com | estore.ibpsguide.com | www.sscexamguide.com


165
Exclusively on New Pattern – Reasoning Ability eBook

d) societies viewed
e) None of these

The logic for all the above code is:


The letter denotes
If the last letter of the word is vowel +1
Like “the” – “e + 1” = F
If the last letter of the word is constant +2
Like “for” – “r+2” = T
The number denotes the total number of letters multiply with next number
Like “Coach” – Number of letters is 5
= 5 × 6 = 30
Like “Repayment” – Number of letters is 9
= 9 × 10 = 90
And rest of the one symbol can be anything.

261. Answer: d)
262. Answer: c)
263. Answer: b)
264. Answer: e)
265. Answer: a)

Directions (Q. 266-270): Study the following information carefully and answer the given questions
In a certain code language
"Modem took wrong program" is written as "20%U, 13&N, 23#X , 16!Q.
"Maps catch difficult batches" is written as "04$E, 03*D, 13%N, 02@C .
"Like your jewel entry" is written as "05#F, 10!K, 12&M, 25%Z .
"Late reaction on traffic" is written as "18*S, 15@P, 12#M, 20!U.

266. What may be the possible word for ‘02@C, 23#X, 20!U ’ in the given code language?
a) Modem catch jewel
b) Like took traffic
c) Late program difficult
d) wrong batches traffic
e) None of these

267. What is the code for ‘difficult’ in the given code language?
a) 03*D

www.ibpsguide.com | estore.ibpsguide.com | www.sscexamguide.com


166
Exclusively on New Pattern – Reasoning Ability eBook

b) 13%N
c) 04$E
d) 02@C
e) None of these

268. What is the possible for ‘03*D, 10!K’ in the given code language?
a) catch jewel
b) Maps entry
c) difficult Like
d) entry batches
e) None of these

269. What may be the possible code for ‘Maps reaction’ in the given code language?
a) 23#X, 15@P
b) 16!Q, 15@P
c) 23#X, 12#M
d) 20%U, 20!U
e) 13%N, 18*S

260. What is the code for ‘Sole trader power’ in the given code language?
a) #R18, &P22, %S20
b) #Q16, &U20, %T19
c) #S14, &T18, %P21
d) #M10, &O15, %T17
e) None of these

The logic for all the above code is:


The letter denotes the next letter of the First letter of each word
Like “Catch” – First letter “C” – Next letter- D
The Two digit number denotes the alphabet number of First letter (1 - 26)
Like “Maps” – First letter - M – 13
And rest of the one symbol and can be anything.
266. Answer: d)
267. Answer: c)
268. Answer: a)
269. Answer: e)
270. Answer: b)
Directions (Q. 271-275): Study the following information carefully and answer the given questions

www.ibpsguide.com | estore.ibpsguide.com | www.sscexamguide.com


167
Exclusively on New Pattern – Reasoning Ability eBook

In a certain code language


"Spontaneously serve to megaproject" is written as "11@E, 23*K, 05$B, 27%M” .
"Childrens dancing magazine priced" is written as "19%I , 13#F, 15&G, 17@H”.
"Pages abbreviations for backbenchers" is written as "07#C, 11%E, 25$L, 27@M”.
"That headphones readjusts objectively" is written as "21&J, 19*I, 09!D, 23%K”.

271. What is the code for ‘That’ in the given code language?
a) 19*I
b) 09!D
c) 21&J
d) 23%K
e) None of these

272. What is the code for ‘dancing back benchers’ in the given code language?
a) 15&G,25$L
b) 19%I, 27@M
c) 13#F, 07#C
d) 17@H, 27@M
e) 19%I, 11%E

273. What may be the possible word for “27%M, 17@H, 27@M” in the given code language?
a) Childrens mega project abbreviations
b) Megaproject backbenchers priced
c) to magazine Pages
d) Spontaneously magazine abbreviations
e) None of these

274. What may be the possible code for ‘readjusts priced’ in the given code language?
a) 09!D, 13#F
b) 23%K, 15&G
c) 21&J, 17@H
d) 23%K, 19%I
e) 19*I, 13#F

275. What is the possible word for “11%E,23%K”in the given code language?
a) abbreviations readjusts
b) backbenchers objectively
c) Pages objectively

www.ibpsguide.com | estore.ibpsguide.com | www.sscexamguide.com


168
Exclusively on New Pattern – Reasoning Ability eBook

d) That Pages
e) None of these

The logic for all the above code is:


The letter denotes the total number of letters in alphabetical order (1-26)
Like “Headphones” – Total number of letters is 10 - J
The number denotes the total number of letters add with next number
Like “advancement” – Number of letters is 11 + 12 = 23
And rest of the one symbol can be anything.

271. Answer: b)
272. Answer: a)
273. Answer: d)
274. Answer: e)
275. Answer: c)

Directions (Q. 276-280): Study the following information carefully and answer the given questions
In a certain code language
"Auto roundup the lorry" is written as "22+B, 12#L, 02*V , 11&M”.
"Animals cartoon house coach" is written as "19*E , 08&P, 22*B, 13&K”.
"Windmills hold huge layer" is written as "23#A, 09*O, 08!P, 22#B” .
"Succeeded people gain Lightning" is written as "22$B,20!D, 23!A, 13#K”.

276. What is the code for ‘lorry’ in the given code language?
a) 22+B
b) 12#L
c) 02*V
d) 11&M
e) None of these

277. What is the possible word for ‘12#L, 20!D’ in the given code language?
a) Auto Lightning
b) hold layer
c) layer Auto
d) roundup people
e) None of these

278. What may be the possible word for ‘22*B, 23#A,23!A” in the given code language?

www.ibpsguide.com | estore.ibpsguide.com | www.sscexamguide.com


169
Exclusively on New Pattern – Reasoning Ability eBook

a) coach huge layer


b) house layer people
c) cartoon hold succeeded
d) house hold succeeded
e) None of these

279. What may be the possible code for ‘house Lightning’ in the given code language?
a) 22*B, 23!A
b) 08&P, 13#K
c) 19*E, 22$B
d) 13&K, 23!A
e) 22*B, 20!D

280. What is the code for ‘Animals people’ in the given code language?
a) 13&K, 20!D
b) 08&P, 22$B
c) 22*B, 22$B
d) 19*E, 23!A
e) 08&P,13#K

The logic for all the above code is:


The letter denotes last letter of each word (Minus) - 3
Like “Hold” – Last letter “D” - 3 = A
The Two digit number denotes the reverse alphabet number of Last letter (26 - 1)
Like “The” – “E”- 22
And rest of the one symbol is
For three letter word - +
For Four letter word - #
For Five letter word - *
For Six letter word - $
For Seven letter word - &
For Nine letter word – !

276. Answer: c)
277. Answer: a)
278. Answer: d)
279. Answer: e)
280. Answer: b)

www.ibpsguide.com | estore.ibpsguide.com | www.sscexamguide.com


170
Exclusively on New Pattern – Reasoning Ability eBook

Directions (Q. 281-285): Study the following information carefully and answer the given questions
In a certain code language
"Sukumar worked in sales" is written as "02#Y, 06%T, 04+V, 04+U”.
"Plans are renovating downtown" is written as "04+S , 04+X, 02#V, 08$Q”.
"The reliance supermarket started" is written as "08$S, 04+T, 02#X, 08$P” .
"Devika planned arranging mountains" is written as " 04+T, 08$R, 06%R, 06%U”.

281. What is the code for ‘started’ in the given code language?
a) 08$S
b) 02#X
c) 08$P
d) 04+T
e) None of these

282. What may be the possible code for ‘arranging supermarket’ in the given code language?
a) 08$R,04+T
b) 06%R, 08$P
c) 06%U, 08$S
d) 04+T,02#X
e) 04+T,08$S

283. What is the possible word for “06%T,04+X”in the given code language?
a) renovating plans
b) plans sales
c) downtown in
d) renovating worked
e) Sukumar are

284. What is the code for ‘downtown mountains’ in the given code language?
a) 04+S, 08$R
b) 02#V, 04+T
c) 04+X, 04+T
d) 08$Q, 06%R
e) 02#V, 06%U

285. What may be the possible word for “02#Y,06%U,08$S” in the given code language?

www.ibpsguide.com | estore.ibpsguide.com | www.sscexamguide.com


171
Exclusively on New Pattern – Reasoning Ability eBook

a) supermarket arranging worked


b) sales planned started
c) devika in reliance
d) reliance in mountains
e) None of these

The logic for all the above code is:


The letter denotes the total number of letters in reverse alphabetical order (26-1)
Like “plans” –Total number of letters is 5 = V
The two digit number denotes the total number of Vowels in the word multiply with 2
Like “Sukumar” – Number of vowels = 3 × 2 = 06
And rest of the one symbol is
Based on the number of vowels in each word
One vowel word - #
Two vowel word - +
Three vowel word - %
Four vowel word - $

281. Answer: d)
282. Answer: b)
283. Answer: e)
284. Answer: a)
285. Answer: c)

Directions (Q. 286-290): Study the following information carefully and answer the given questions
In a certain code language
" Ishwarya have cute friends " is written as "12@G, 12@B, 42#E , 56%H”.
"You must study hard " is written as "12#G, 20#R, 12#L, 06@X” .
"The warehouse is big " is written as "06@S, 02!H, 72@V, 06#A” .
" Teachers motivate him well " is written as " 56@L , 56#S, 06#G, 12#V”.

286. What is the code for ‘study’ in the given code language?
a) 20#R
b) 12#G
c) 06@X
d) 12#L
e) None of these

www.ibpsguide.com | estore.ibpsguide.com | www.sscexamguide.com


172
Exclusively on New Pattern – Reasoning Ability eBook

287. What is the code for ‘06#A,42#E ’ in the given code language?
a) big warehouse
b) warehouse have
c) Ishwarya is
d) cute warehouse
e) big friends

288. What may be the possible word for ‘42#E ,56@L,06#G ’ in the given code language?
a) Teachers cute friends
b) friends motivate him
c) have motivate teachers
d) you motivate him
e) None of these

289. What may be the possible code for ‘Teachers have’ in the given code language?
a) 56@L, 12@B
b) 06#G, 56%H
c) 56#S, 12@G
d) 12#V, 12@B
e) None of these

280. What is the code for ‘Ishwarya must well’ in the given code language?
a) 12@B, 12#G,56@L
b) 42#E, 12#L, 56#S
c) 12@G, 06#A, 06#G
d) 56%H, 12#L, 12#V
e) None of these

The logic for all the above code is:


The letter denotes the previous letter of the first letter of each word
Like “must” – First letter “M” – Previous letter of M is L
The number denotes the number of letters multiply with previous number
Like “warehouse” – Number of letters9 = 9 × 8 = 72
And rest of the one symbol should be,if
First and last letter of the word is vowel – %
First and last letter of the word is consonant - #
First letter is consonant and last letter is vowel - @
First letter is vowel and last letter is consonant - !

www.ibpsguide.com | estore.ibpsguide.com | www.sscexamguide.com


173
Exclusively on New Pattern – Reasoning Ability eBook

286. Answer: a)
287. Answer: e)
288. Answer: b)
289. Answer: c)
290. Answer: d)

Directions (Q. 291-295): Study the following information carefully and answer the given questions
In a certain code language
" Heart remember unimportant number " is written as "11$Q, 21!X, 09*K , 15@U”.
" Hack above recover tabs " is written as "13&U, 09*D, 07#W, 07#K” .
"Hit Unable impaired eggs " is written as "05+K, 15@L, 11$X, 07#H” .
"Army fires upto embassy " is written as " 07#X , 13&H, 09*I, 07#D”.

291. What is the code for ‘Unable’ in the given code language?
a) 07#H
b) 15@L
c) 11$X
d)05+K
e) None of these

292. What is the word for ‘07#K, 13&H’ in the given code language?
a) Above upto
b) Army hack
c) recover fires
d) embassy tabs
e) hack embassy

293. What may be the possible word for ‘15@L, "13&U, 11$Q’ in the given code language?
a) Unable tabs refer
b) refer recover eggs
c) Hit remember above
d) impaired recover number
e) None of these

294. What may be the possible code for ‘upto remember’ in the given code language?
a) 07#D, 11$Q
b) 07#X, 15@U

www.ibpsguide.com | estore.ibpsguide.com | www.sscexamguide.com


174
Exclusively on New Pattern – Reasoning Ability eBook

c) 13&H,21!X
d) 09*I, 09*K
e) None of these

295. What is the code for ‘hack unimportant eggs’ in the given code language?
a) 07#K , 21!X, 07#H
b) 09*D, 09*K, 05+K
c) 07#W, 15@U, 15@L
d) 13&U, 11$Q, 11$X
e) None of these

The logic for all the above code is:


The letter denotes first letter of each word + 3
Like “number” – first letter – “N” + 3 = Q
The Two digit number denotes ( total number of each letter)^2-(previous number)^2
Like “Recover” – Total letters– 7
= 7^2 – 6^2 =49 – 36 = 13
And rest of the one symbol is
For three letter word - +
For Four letter word - #
For Five letter word - *
For Six letter word - $
For Seven letter word - &
For Eight letter word - @
For Eleven letter word – !

291. Answer: c)
292. Answer: e)
293. Answer: d)
294. Answer: b)
295. Answer: a)

Directions (Q. 296-300): Study the following information carefully and answer the given questions
In a certain code language
" Union Government has inaugurated" is written as “04%I, 04#V, 25%J , 49%H”.
" United Nations Children Fund" is written as “16#O , 09#V, 09%G, 36%D”.
"Chief Minister of Assam" is written as “09%B, 01#P, 25%N, 09%D” .
" Provide emergency ambulance service" is written as “16#Q , 36%F, 25%B, 16#T”.

www.ibpsguide.com | estore.ibpsguide.com | www.sscexamguide.com


175
Exclusively on New Pattern – Reasoning Ability eBook

296. What is the code for ‘Children’ in the given code language?
a) 09%G
b) 09#V
c) 16#O
d) 36%D
e) None of these

297. What may be the possible code for ‘Union Minister’ in the given code language?
a) 25%J, 01#P
b) 49%H, 09%D
c) 04%I, 09%B
d) 04#V, 01#P
e) 04#V, 25%N

298. What is the possible word for “25%B,09%D”in the given code language?
a) Assam emergency
b) ambulance chief
c) Minister service
d) provide service
e) emergency minister

299. What is the code for ‘provide fund’ in the given code language?
a) 16#Q, 09%G
b) 25%B, 09#V
c) 16#Q, 36%D
d) 36%F, 16#O
e) None of these

300. What may be the possible word for “49%H, 16#O,16#T ” in the given code language?
a) United nations provide
b) Government fund service
c) Government nations service
d) Union nations fund
e) None of these

The logic for all the above code is:

www.ibpsguide.com | estore.ibpsguide.com | www.sscexamguide.com


176
Exclusively on New Pattern – Reasoning Ability eBook

The letter denotes the next letter of the first letter in the word
Like “government” – G – next letter = H
The two digit number denotes square of the total number of consonants in that words
Like “Nations” – Number of consonants = 5^2 = 25
And rest of the one symbol should be,
If first letter of the word in alphabet number
A-M = %
N-Z = #

296. Answer: d)
297. Answer: e)
298. Answer: b)
299. Answer: a)
300. Answer: c)

www.ibpsguide.com | estore.ibpsguide.com | www.sscexamguide.com


177
Insurance Awareness Capsule

INSURANCE AWARENESS CAPSULE

Sno Contents Page No.


1 History of Insurance in India 02
2 Important Point to Know about The New India Assurance Co. Ltd 04
Important Point to Know about National Insurance Company Limited
3 05
(NICL)
4 History of LIC 06
5 List of Major Insurance Plans of LIC and its Features 09
6 Important Insurances Companies and their Head with Taglines 11
7 Insurance Policies 12
8 Important Insurance terms 14
10 All About IRDA 16
11 General Insurance Council 18
12 Government Sponsored Socially Oriented Insurance Schemes 18
13 Important Insurance Schemes and its Functions: 23
14 Functions of Ombudsman 58
15 Important Points to remember 60

www.ibpsguide.com | estore.ibpsguide.com | www.sscexamguide.com


Insurance Awareness Capsule

History of Insurance in India

In India, insurance has a deep-rooted history. It finds mention in the writings of Manu ( Manusmrithi ),
Yagnavalkya ( Dharmasastra ) and Kautilya ( Arthasastra ). The writings talk in terms of pooling of resources
that could be re-distributed in times of calamities such as fire, floods, epidemics and famine. This was
probably a pre-cursor to modern day insurance. Ancient Indian history has preserved the earliest traces of
insurance in the form of marine trade loans and carriers’ contracts. Insurance in India has evolved over time
heavily drawing from other countries, England in particular.

1818 saw the advent of life insurance business in India with the establishment of the Oriental Life
Insurance Company in Calcutta. This Company however failed in 1834. In 1829, the Madras Equitable had
begun transacting life insurance business in the Madras Presidency. 1870 saw the enactment of the British
Insurance Act and in the last three decades of the nineteenth century, the Bombay Mutual (1871), Oriental
(1874) and Empire of India (1897) were started in the Bombay Residency. This era, however, was dominated
by foreign insurance offices which did good business in India, namely Albert Life Assurance, Royal Insurance,
Liverpool and London Globe Insurance and the Indian offices were up for hard competition from the foreign
companies.

In 1914, the Government of India started publishing returns of Insurance Companies in India. The
Indian Life Assurance Companies Act, 1912 was the first statutory measure to regulate life business. In 1928,
the Indian Insurance Companies Act was enacted to enable the Government to collect statistical information
about both life and non-life business transacted in India by Indian and foreign insurers including provident
insurance societies. In 1938, with a view to protecting the interest of the Insurance public, the earlier
legislation was consolidated and amended by the Insurance Act, 1938 with comprehensive provisions for
effective control over the activities of insurers.

The Insurance Amendment Act of 1950 abolished Principal Agencies. However, there were a large
number of insurance companies and the level of competition was high. There were also allegations of unfair
trade practices. The Government of India, therefore, decided to nationalize insurance business.
An Ordinance was issued on 19th January, 1956 nationalising the Life Insurance sector and Life
Insurance Corporation came into existence in the same year. The LIC absorbed 154 Indian, 16 non-Indian
insurers as also 75 provident societies—245 Indian and foreign insurers in all. The LIC had monopoly till the
late 90s when the Insurance sector was reopened to the private sector.

The history of general insurance dates back to the Industrial Revolution in the west and the
consequent growth of sea-faring trade and commerce in the 17th century. It came to India as a legacy of
British occupation. General Insurance in India has its roots in the establishment of Triton Insurance Company
Ltd., in the year 1850 in Calcutta by the British. In 1907, the Indian Mercantile Insurance Ltd, was set up. This
was the first company to transact all classes of general insurance business. 1957 saw the formation of the

www.ibpsguide.com | estore.ibpsguide.com | www.sscexamguide.com


Insurance Awareness Capsule

General Insurance Council, a wing of the Insurance Association of India. The General Insurance Council
framed a code of conduct for ensuring fair conduct and sound business practices.
In 1968, the Insurance Act was amended to regulate investments and set minimum solvency margins.
The Tariff Advisory Committee was also set up then.
In 1972 with the passing of the General Insurance Business (Nationalisation) Act, general insurance
business was nationalized with effect from 1st January, 1973. 107 insurers were amalgamated and grouped
into four companies, namely National Insurance Company Ltd., the New India Assurance Company Ltd., the
Oriental Insurance Company Ltd and the United India Insurance Company Ltd. The General Insurance
Corporation of India was incorporated as a company in 1971 and it commence business on January 1sst
1973.
This millennium has seen insurance come a full circle in a journey extending to nearly 200 years. The
process of re-opening of the sector had begun in the early 1990s and the last decade and more has seen it
been opened up substantially. In 1993, the Government set up a committee under the chairmanship of RN
Malhotra, former Governor of RBI, to propose recommendations for reforms in the insurance sector. The
objective was to complement the reforms initiated in the financial sector. The committee submitted its report
in 1994 wherein, among other things, it recommended that the private sector be permitted to enter the
insurance industry. They stated that foreign companies be allowed to enter by floating Indian companies,
preferably a joint venture with Indian partners.
Following the recommendations of the Malhotra Committee report, in 1999, the Insurance Regulatory
and Development Authority (IRDA) was constituted as an autonomous body to regulate and develop the
insurance industry. The IRDA was incorporated as a statutory body in April, 2000. The key objectives of the
IRDA include promotion of competition so as to enhance customer satisfaction through increased consumer
choice and lower premiums, while ensuring the financial security of the insurance market.
The IRDA opened up the market in August 2000 with the invitation for application for registrations.
Foreign companies were allowed ownership of up to 26%. The Authority has the power to frame regulations
under Section 114A of the Insurance Act, 1938 and has from 2000 onwards framed various regulations
ranging from registration of companies for carrying on insurance business to protection of policyholders’
interests.
In December, 2000, the subsidiaries of the General Insurance Corporation of India were restructured
as independent companies and at the same time GIC was converted into a national re-insurer. Parliament
passed a bill de-linking the four subsidiaries from GIC in July, 2002.
there are 28 general insurance companies including the ECGC and Agriculture Insurance Corporation
of India and 24 life insurance companies operating in the country.
The insurance sector is a colossal one and is growing at a speedy rate of 15-20%. Together with
banking services, insurance services add about 7% to the country’s GDP. A well-developed and evolved
insurance sector is a boon for economic development as it provides long- term funds for infrastructure
development at the same time strengthening the risk taking ability of the country.

www.ibpsguide.com | estore.ibpsguide.com | www.sscexamguide.com


Insurance Awareness Capsule

The New India Assurance Co. Ltd


NIACL Pioneers:
• First company to set up an Aviation Insurance Department in 1946.
• First company to handle the Hull Insurance requirements of the Indian Shipping Fleet.
• First company to establish its own Training School.
• First company to introduce the concept of 'Model Office Training'.
• First company to create department in Engineering insurance.
• Pioneer in Satellite insurance.

NIACL Vision:
• To be the most respected, trusted and preferred Non-life Insurer in the Global markets we operate.

NIACL Mission:
• To develop General Insurance Business in the best interest of the community.
• To provide Financial Security to Individuals, Trade, Commerce & all other segments of the Society by
offering Insurance products & Services of High Quality at affordable Cost.

NIACL Values:
• Highest priority to Customers needs
• High standards of Public Conduct
• Transparency in operations

NIACL Culture:
• Courtesy and Caring
• Initiatives and Innovation
• Integrity, Trustworthiness and Reliability

NIACL Commitments:
• Act courteously, fairly and reasonably in all our dealings with the customers.
• Make sure all our policy documents and claim procedures are clear and complete information is given
about our products and services.
• Deal quickly and sympathetically with the grievances of the customer and resolve efficiently through
nominated customer service officers in all operating offices. We will educate the client about
grievance redressal mechanism including the system of grievance redressal through Ombudsman.
• Respond to all commercially viable General Insurance requirements of all categories including
products for weaker section of the society at affordable price within 3 months from the date of such
requirement.
• Continue to develop a dedicated, sensitized and professional workforce for efficient execution of roles
assigned to them.

www.ibpsguide.com | estore.ibpsguide.com | www.sscexamguide.com


Insurance Awareness Capsule

• Have a regular monitoring and consultative process with all our service providers and set up
monitoring mechanism for delivery of promised services to our customers.
National Insurance Company
History:
After nationalisation in 1972, NICL operated as a subsidiary of General Insurance Corporation of India (GIC).
National Insurance Company Limited was spun off as a distinct company under the General Insurance
Business (Nationalisation) Amendment Act in 2002. In April 2004, NIC signed an agreement with Nainital
Bank for distribution of its general insurance products through the bank's branches in Uttarakhand, Haryana
and New Delhi.

Profile:
National Insurance Company Limited was incorporated on 6 December 1906 with its registered office in
Kolkata. Consequent to passing of the General Insurance Business Nationalisation Act in 1972, 21 Foreign
and 11 Indian Companies were merged with it and National became a subsidiary of General Insurance
Corporation of India (GIC) which is fully owned by the Government of India. After the notification of the
General Insurance Business and its India's largest General Insurance Company(Nationalisation) Amendment
Act, on 7 August 2002, National has been de-linked from its holding company GIC and presently operates as
an independent insurance company wholly owned by Govt of India. National Insurance Company Ltd (NIC) is
one of the leading public sector insurance companies of India. It transacts non life insurance business.
Headquartered in Kolkata, NIC's network of about 2000 offices, manned by more than 16,000 skilled
personnels, is spread over the length and breadth of the country covering remote rural areas, townships and
metropolitan cities. NIC's foreign operations are carried out from its branch offices in Nepal. Befittingly, the
product ranges, of more than 200 policies offered by NIC cater to the diverse insurance requirements of its
14 million policyholders. Innovative and customised policies ensure that even specialised insurance
requirements are fully taken care of. The paid-up share capital of National is ₹1 billion. Starting off with a
premium base of ₹50 crores in 1974, NIC's gross direct premium income has steadily grown to about ₹90
billion rupees in the financial year 2012–13. National transacts general insurance business of Fire, Marine
and Miscellaneous insurance. The Company offers protection against a wide range of risks to its customers.
The Company is privileged to cater its services to almost every sector or industry in the Indian Economy viz.
Banking, Telecom, Aviation, Shipping, Information Technology, Power, Oil & Energy, Agronomy, Plantations,
Foreign Trade, Healthcare, Tea, Automobile, Education, Environment, Space Research etc. As of 2010, NICL
has a AAA rating from Indian rating agency, CRISIL, a subsidiary of Standard and Poor's Company. The gross
premiums from underwriting by the company grew by 32.22% to over ₹61 billion during the Financial Year
2010–2011. And Gross Premium grew up to 100 billion during the financial year 2013–2014. With this, the
company was ranked second among general insurance companies operating in India, behind New India
Assurance, at the end of the 2014 Financial Year. With about 1000 offices and 16,000 employees and agents,
the company operates in all of India, and neighbouring Nepal. In 2008, the company signed a deal with HCL
Technologies worth almost ₹4 billion to outsource the company's information technology requirements over 7
years.

www.ibpsguide.com | estore.ibpsguide.com | www.sscexamguide.com


Insurance Awareness Capsule

Products and services:


NICL has a range of coverage policies targeting different sectors:
• Personal Insurance policies include medical insurance, accident, property and auto insurance
coverage
• Rural Insurance policies provide protection against natural and climatic disasters for agriculture and
rural businesses
• Industrial Insurance policies provide coverage for project, construction, contracts, fire, equipment
loss, theft, etc.
• Commercial Insurance policies provide protection against loss and damage of property during
transportation, transactions, etc.

History of Life Insurance Corporation

The study of insurance is probably as old as the story of mankind. The same instinct
that prompts modern businessmen to secure themselves against loss and disaster existed in primitive men
also. They too sought to avert the evil consequences of fire and fold and loss of life and were willing to make
some sort of sacrifice in order to achieve security.
Though the concept of insurance is largely a development of the recent past, particularly after the industrial
era – past few centuries – yet its beginnings date back almost 6000 years.
Life Insurance in its modern form came to India from England in the year 1818. Oriental Life Insurance
Company started by Europeans in Calcutta was the first life insurance company
on India Soil. All the insurance companies established during that period were brought up with the purpose of
looking after the needs of European community and Indian natives were not being insured by these
companies. However, later with the efforts of eminent people like Babu Mutttylal Seal, the foreign life
insurance companies started insuring Indian lives. But Indian lives were being treated as sub – standard lives
and heavy extra premiums were being charged on them. Bombay Mutual Life Assurance Society heralded the
birth of first Indian life insurance company in the years 1870, and covered Indian lives at normal rates,
Starting as Indian enterprise with highly patriotic motives; insurance companies came into existence to carry
the message of insurance and social security through insurance to various sectors of society. Bharat
Insurance Company (1896) was also one of such companies inspired by nationalism. The Swadeshi
movement of 1905 – 1907 gave rise to more insurance companies.
The United India in Madras, National Indian and National Insurance in Calcutta and the Cooperative
Assurance at Lahore were established in 1906. In 1907, Hindustan Co-operative Insurance Company took its
birth in one of the rooms of the Jorasanko, house of the great poet Rabindranath Tagore, in Calcutta.
The Indian Mercantile, General Assurance and Swadeshi Life (later Bombay Life) were some of the
companies established during the same period. Prior to 1912 India has no legislation to regulate insurance
business. In the year 1912, the Life Insurance Companies Act, and the Provident Fund Act were passed. The
Life Insurance Companies Act, 1912 made it necessary that the premium rate tables and periodical

www.ibpsguide.com | estore.ibpsguide.com | www.sscexamguide.com


Insurance Awareness Capsule

valuations of companies should be certified by an actuary. But the Act discriminated between foreign and
Indian companies on many accounts, putting the Indian companies at a disadvantage.
The first two decades of the twentieth century saw lot of growth in insurance business. From 44 companies
with total business – in – force as Rs. 22.44 crore, it rose to 176 companies with total business – in – force as
Rs. 298 crore in 1938. During the mushrooming of insurance companies many financially unsound concerns
were also floated which failed miserably. The demand form nationalization of life insurance industry was
made repeatedly in the past but it gathered momentum in 1944 when a bill to amend the Life Insurance Act
1938 was introduced in the Legislative Assembly. However, it was much later on the 19th of January, 1956,
that life insurance in India was nationalized. About 154 Indian Insurance companies, 16 non-Indian
companies and 75 provident were operating in India at the time of nationalization.
Nationalization was accomplished in two stages; initially the management of the companies was taken over
by means of an Ordinance, and later, the ownership too by means of a comprehensive bill. The Parliament of
India passed the Life Insurance Corporation Acct on the 19th of June 1956, and the Life Insurance
Corporation of India was created on 1st September,
1956, with the objective of spreading life insurance much more widely and in particular to the rural areas
with a view to reach all insurable persons in the country, providing them adequate financial cover at a
reasonable cost.
LIC had 5 zonal offices, 33 divisional offices and 212 branch offices, apart from its corporate office in the
year 1956. Since life insurance contracts are long term contracts and during the currency of the policy it
requires a variety of service need was felt in the later years to expand the operations and place a branch
office at each district headquarter. Reorganization of LIC took place and large numbers of new branch offices
were opened. As a result of re-organisation servicing functions were transferred to the branches, and
branches were made accounting units. It worked wonders with the performance of the corporation. It may be
seen that from abut Rs. 200 crores of New Business in 1957 the corporation crossed Rs. 1000 crores only in
the ear 1969-70. And it took another 10 years for LIC to cross Rs. 2000 crore mark of new business. But with
re-organisation happening in the early eighties, by 1985- 86 LIC had already crossed Rs. 7000 crore Sum
Assured on new policies.
LIC functions with 2048 fully computerized branch offices, 109 divisional offices, 8 zonal offices, 992 satellite
offices and the corporate office. LIC’s Wide Area Network covers 109 divisional offices and connects all the
branches through a Metro Area Network. LIC has tied up with some Bank and Service providers to offer on-
line premium collection facility in selected cities. LIC’s ECS and ATM premium payment facility is an addition
to customer convenience. Apart from on-line Kiosks and IVRS, Info Centers have been commissioned at
Mumbai, Ahmadabad, Bangalore, Chennai, Hyderabad, Kolkata, New Delhi, Pune and many other cities. With
a vision of providing easy access to its policyholders, LIC has launched its SATELLITE SAMPARK offices. The
satellite offices are smaller, leaner and to closer to the customer. The digitalized record of the satellite
offices will facilitate anywhere servicing and many other conveniences in the future.
LIC continues to be the dominant life insurer even in the liberalized scenario of Indian insurance and is
moving fast on a new growth trajectory surpassing its own past records. LIC has issued over one crore

www.ibpsguide.com | estore.ibpsguide.com | www.sscexamguide.com


Insurance Awareness Capsule

politics during the current year. It has crossed the milestone of issuing 1,01,32,955 new policies by 15th Oct,
2005, posting a healthy growth rate of 16.67% over the corresponding period of the previous year.
From then to now, LIC has crossed many milestones and has set unprecedented performance records in
various aspects of life insurance business. The same motives which inspired our forefathers to bring
insurance into existence in this country inspire us at LIC to take this message of protection to light the lamps
of security in as many homes as possible and to help the people in providing security to their families.

Objectives of Life Insurance Corporation of India (LIC)

Spread life insurance widely and in particular to the rural areas and to the socially and economically
backward classes with a view to reaching all insurable persons in the country and providing them adequate
financial cover against death at a reasonable cost.
• Maximize mobilization of people’s savings by making insurance linked savings adequately attractive.
• Bear in mind, in the investment of funds, the primary obligation to its policy holders, whose money it
holds in trust, without losing sight of the interest of the community as a whole; the funds to be
deployed to the community as a whole, keeping in view national priorities and obligations attractive
return.
• Conduct business with utmost economy and with the full realization that the moneys belong to the
policyholders.
• Act as trustees of the insured public in their individual and collective capacities.
• Meet the various life insurance need of the community that would arise in the changing social and
economic environment.
• Involve all people working in the Corporation to the best of their capabilities furthering the interests of
the insured public by providing efficient service with courtesy.
• Promote amongst all agents and employees of the Corporation a sense of participation, pride job
satisfaction through discharge of their duties with dedication towards achievement of corporate
objective.

Roles and Functions of LIC- Life Insurance Corporation of India

The life insurance business was nationalized on 19th January, 1956 and the Life Insurance Corporation of
India came into being on 1st September, 1956 to carry on life business in India with capital of Rs.5 crores
contributed by the Central Government. The Corporation is a body corporate having perpetual succession
with a common seal with powers to acquire, hold and dispose of property and may by its name sue and be
sued. The functions of the Corporation shall be to carry on and develop life insurance business to the best
advantage of the community.
The Corporation shall have power –

www.ibpsguide.com | estore.ibpsguide.com | www.sscexamguide.com


Insurance Awareness Capsule

• To carry on capital redemption business, annuity certain business or reinsurance business in so far as
such reinsurance business relating to life insurance business;
• To invest the funds of the Corporation in such manner as the Corporation may think fit and to take all
such steps as may be necessary or expedient for the protection or realisation of any investment;
including the taking over of and administering any property offered as security for the investment until
a suitable opportunity arises for its disposal;
• To acquire, hold and dispose of any property for the purpose of its business;
• To transfer the whole or any part of the life insurance business carried on outside India to any other
person or persons, if in the interest of the Corporation it is expedient so to do;
• To advance or lend money upon the security of any movable or immovable property or otherwise;
• To borrow or raise any money in such manner and upon such security as the Corporation may think
fit;
• To carry on either by itself or through any subsidiary any other business in any case where such other
business was being carried on by a subsidiary of an insurer whose controlled business has been
transferred to and vested in the Corporation by this act;
• To carry on any other business which may seem to the Corporation to be capable of being
conveniently carried on in connection with its business and calculated directly or indirectly to render
profitable the business of the Corporation; and
• To do all such things as may be incidental or conducive to the proper exercise of any of the powers of
the Corporation.
• In the discharge of any of its functions the Corporation shall act so far as may be on business
principles.

List of Major Insurance Plans of LIC and its Features

Life Insurance Corporation of India (LIC) offers a basket of schemes to meet the various needs of an
Individual and his family:

List of Various Insurance Plans of LIC and its Feature were given below

1.) ENDOWMENT ASSURANCE PLANS:


a) LIC’s Single Premium Endowment Plan No. 817):
A single Premium With-Profits plan where the sum assured is payable along with the accrued bonuses on
maturity or on earlier death of the life assured. On death before the commencement of risk, the single
premium paid is returned without interest.
b) LIC’s New Endowment Plan (Plan No. 814):
A Regular premium With-Profit plan which provides for applicable sum assured along with accrued
bonuses on maturity or an earlier death of life assured.
c) LIC’s New Jeevan Anand (Plan No. 815)

www.ibpsguide.com | estore.ibpsguide.com | www.sscexamguide.com


Insurance Awareness Capsule

Is a unique With-Profits plan wherein on death during the policy term or on survival to the end of the
policy term, the applicable sum assured is payable along with the accrued bonuses. On death after the expiry
of the policy term, basic sum assured is payable.

2.) TERM ASSURANCE PLANS:


a) LIC’s Ammol Jeevan – II (Plan No. 822):
A pure term assurance plan where one can choose any sum assured from 6 lakhs to 24 lakhs
b) LIC’s Amulya Jeevan – II (Plan No. 823):
A pure team assurance plan with a minimum sum assured of 25 lakhs
c) LIC’s e-Term Plan (Plan No. 825):
A pure term assurance plan which is available for online sale only
Under this

3.) PENSION PLANS:


a) LIC’s Jeevan Akshay – VI (Plan No. 189):
An immediate annuity plan with a number of options.
b) LIC’s New Jeevan Nidhi (Plan No. 818):
A with-Profits, deferred pension plan which provides death cover during the deferment period.

4.) MICRO INSURANCE PLANS:


LIC’S New Jeevan Mangal (Plan No. 819):
A Micro Insurance cum Term Insurance plan wherein the Sum assured varies from 10,000/- to 50,000/- In
this plan, total premiums paid is repaid on maturity.

5.) HEALTH INSURANCE PLAN


LIC’s Jeevan Arogya (Plan No. 904):
A non-linked health insurance plan which provides for fixed benefits for hospitalization and almost all
types of surgical procedures irrespective of actual cost incurred.

6.) OTHER PLANS


a) LIC’s New Money Bank Plans (Plan Nos. 820, 821):
Besides providing life cover during the term (20 & 25 years) of the policy, survival benefits linked to the
sum assured during the term of the policy will be available.
b) LIC’s New Bima Bachat (Plan No. 816):
A Single Premium money back plan with policy term of 9, 12 and 15 years. Survival benefits during the
term of policy are available.

Important Insurances Companies and their Head with Taglines

www.ibpsguide.com | estore.ibpsguide.com | www.sscexamguide.com


Insurance Awareness Capsule

S.no Company Tagline Heads


Life Insurance Corporation of Vijay Kumar Sharma
1. Yogakshemam Vahamyaham
India (LIC) ( Chairman)
Rajesh Sud ( Executive
Max Life Insurance Company
2. Aapke Sachhe Advisor Vice Chairman &
Limited
Managing Director)
New India Assurance Company India’s premier general insurance
3. G. Srinivasan (CMD)
Limited company
SBI Life Insurance Company
4. With Us, You’re Sure Arijit Basu ( CEO)
Limited
United India Insurance Company
5. Rest Assured with Us Milind A Kharat (CEO)
Limited
Oriental Insurance Company Prithvi, Agni, Jal, Akash, Sabki AK Saxena ( Chairman
6.
Limited Suraksha Hamare Paas and Managing Director)
G. Murlidhar (
Kotak Mahindra Old Mutual Life
7. Faidey ka Insurance Managing Director and
Insurance Limited
Executive Director)
CEO-Anthony Jacob and
8. Apollo Munich Health Insurance We know Healthcare Chairman - Prathap C
Reddy)
G N Bajpai and
Managing Director &
9. Future General Life Insurance Ek Shagun Zindagi Ke Naam
CEO - Munish Sharda

CEO-T R
10. Aviva India Life Insurance Kal par Control
Ramachandaran
Naveen Tahilyani
Tata AIA Life Insurance
11. You click, we cover (Chief Executive Officer
Company Limited
& Managing Director)
ICICI Prudential Life Insurance
12. Zimmedari ka humsafar CEO-Sandeep Bakshi
Company Limited
MD & CEO-Amitabh
Chaudhary, Executive
HDFC Standard life Insurance
13. Sar utha Ke Jiyo Director and Chief
Company Limited
Financial Officer-Vibha
Padalkar
14. Bajaj Allianz Life Insurance Jiyo Befiqar CEO- Tarun Chugh

www.ibpsguide.com | estore.ibpsguide.com | www.sscexamguide.com


Insurance Awareness Capsule

Company Limited
Birla Sun Life Insurance MD & Chief Executive
15. Muskurate Raho
Company Limited Officer- Pankaj Razdan
16. Max Bupa Health Insurance Your Health First CEO - Ashish Mehrotra
Exide Life Insurance India
Company (Earlier Known as ING Managing Director &
17. Adding Life to Insurance
Vysya Life Insurance India CEO - Kshitij Jain
Company)
Principal Officer & CEO
PNB MetLife India Insurance
18. Have you met life today? - Ashish Kumar
Company Limited
Srivastava

Insurance Policies
(A) LIFE INSURANCE:
• Term Life Insurance
• Permanent Life Insurance
(B) GENERAL INSURANCE:
• Fire Insurance
• Marine Insurance
• Accident Insurance

(A)Life Insurance
Life Insurance is a contract providing for payment of a sum of money to the person assured or, following him
to the person entitled to receive the same, on the happening of a certain event. It is a good method to
protect your family financially, in case of death, by providing funds for the loss of income.

A1. TERM LIFE INSURANCE: Under a Term Life contract, the insurance company pays a specific lump
sum to the designated beneficiary in case of the death of the insured. These policies are usually for 5, 10,
15, 20 or 30 years.
Term life insurance are the most popular in advance countries but were not so popular in India. However,
after the entry of the private operators and aggressive marketing by few players this kind of policies are
becoming popular. The premium on such type of policies is comparatively quite low when compared with
other types of life insurance policies, mainly due to the fact that these policies do not carry cash value.

PLUS OF TERM LIFE INSURANCE MINUSES OF TERM LIFE INSURANCE

www.ibpsguide.com | estore.ibpsguide.com | www.sscexamguide.com


Insurance Awareness Capsule

- The premium payable on these policies is - If one survives the period of the policy, he /
low as they do not carry any cash value. she does not get any money at the end of the
- One can afford for quite high value insurance policy.
policies The premium on such policies keeps on
increasing with age mainly because the risk of
death of older people is more. Over the page
of 60, these policies become difficult to afford.

A2. PERMANENT LIFE INSURANCE:


In a Permanent Life contract, a portion of the money paid as premiums is invested in a fund that earns
interest on a tax-deferred basis. Thus, over a period of time, this policy will accumulate certain "cash value"
which you will be able to get back either during the period of the policy or at the end of the policy.
Your need for life insurance can change over a lifetime. At any age, you should consider your individual
circumstances and the standard of living you wish to maintain for your dependents. In most cases, you need
life insurance only if someone depends on you for support. Your life insurance premium is based on the type
of insurance you buy, the amount you buy and your chance of death while the policy is in effect. This type of
policy not only provides protection for your dependents by paying a death benefit to your designated
beneficiary upon your death, but it also allows you to use some part of the money while you are alive or at
the end of the policy. Some examples of such policies are :- Whole Life, Universal Life and Variable-
Universal Life.

ENDOWMENT POLICIES
These policies provide for period payment of premiums and a lump sum amount either in the event of death
of the insured or on the date of expiry of the policy, whichever occurs earlier.

MONEY BACK POLICIES


These policies provide for periodic payments of partial survival benefits during the term of the policy itself. A
unique feature associated with this type of policies is that in the event of death of the insured during the
policy term, the designated beneficiary will get the full sum assured without deducting any of the survival
benefit amounts, which have already been paid as money-back components. Moreover, the bonus on such
policies is also calculated on the full sum assured.

ANNUITY / PENSION POLICIES / FUNDS


This policies / funds require the insured to pay the premium as a single lump sum or through installments
paid over a certain number of years. The insured in return will receive back a specific sum periodically from
a specified date onwards (the returns can can be monthly, half yearly or annually), either for life or for a
fixed number of years. In case of the death of the insured, or after the fixed annuity period expires for annuity

www.ibpsguide.com | estore.ibpsguide.com | www.sscexamguide.com


Insurance Awareness Capsule

payments, the invested annuity fund is refunded, usually with some additional amounts as per the terms of
the policy.
Annuities / Pension funds are different from all other forms of life insurance as an annuity policy / fund
does not provide any life insurance cover but merely offers a guaranteed income either for life or a certain
period. Therefore, this type of insurance is taken so as to get income after the retirement.

Important Insurance terms

• Actuary: A specialist in the mathematics of insurance who calculates rates, reserves,


dividends and other statistics

• Annuity: An agreement by an insurer to make periodic payments which continue during the
survival of the annuitant(s) or for a specified period.

• Claim: A demand made by the insured, or the insured’s beneficiary, for payment of the benefits as
provided by the policy.

• Coverage: It is the range of protection that you are provided under an insurance policy.

• Death Benefit: The limit of insurance or the amount of benefit that will be paid in the event of the
death of a covered person.

• Deductible: The amount you have to pay out of pocket for expenses before the insurance company
will cover the remaining costs.

• Endowment Insurance: Endowment insurance pays the sum assured upon the death of the life
insured during the policy terms or on survival to the end of the policy term.

• Exclusive: Things that are not covered under the policy

• Indemnity: Restoration to the victim of a loss by payment, repair or replacement.

• Insurable Interest: Interest in property such that loss or destruction of the property could cause a
financial loss.

• Maturity value: It is the amount the insurance company has to pay you when the policy matures. This
would include the sum assured and the bonuses.

www.ibpsguide.com | estore.ibpsguide.com | www.sscexamguide.com


Insurance Awareness Capsule

• Paid-up value: If you stop paying the premiums, but do not withdraw the money from your policy, the
policy is referred to as paid-up.

• Peril: It is the causes of the possible loss or damage.

• Policy: Policy is the legal document issued by the insurance company that outlines the general terms
and conditions of the insurance.

• Premium: It is the amount you need to pay to the insurance company.

• Reinsurance: In effect, insurance that an insurance company buys for its own protection. The risk of
loss is spread so a disproportionately large loss under a single policy doesn’t fall on one company.
Reinsurance enables an insurance company to expand its expanding volume; secure catastrophe
protection against shock losses; and withdraw a specified time period.

• Renewal: The automatic re-establishment of in-force status effected by the payment of another
premium.

• Rider: It is an optional feature that can be added to a policy. You will have to pay an additional
premium to avail this benefit.

• Subrogation: It is the right for an insurer to pursue a third party that caused an insurance loss to the
insured. This is done as a means of recovering the amount of the claim paid to the insured for the
loss.

• Sum assured: It is the amount of money an insurance policy guarantees to pay before any bonuses
are added. In other words, sum assured is the guaranteed amount you will receive. Halfway through
the policy, you might want to discontinue it and take whatever money is due to you.

• Surrender value: This is the amount payable at the end of specified duration. These amounts are
fixed and predetermined.

• Insurance Settlement: A payment on an insurance claim. That is, when a valid insurance claim is
made, the insurer makes a payment to the policyholder. This is called as the insurance settlement.

• Lapse: Termination of a policy due to failure to pay the required renewal premium.

www.ibpsguide.com | estore.ibpsguide.com | www.sscexamguide.com


Insurance Awareness Capsule

• Term insurance: It is the most traditional life insurance policy wherein the insured gets death benefit
if any contingency happens within the policy term. The insured is, however, not entitled to receive any
survival benefit if he outlives the policy term.

• Underwriting: The process of selection risks for insurance and determining in what amounts and on
what terms the insurance company will accept. Unlike a term insurance cover, if you live, an amount
will be paid to you on maturity of the plan.

All about IRDA

• The Insurance Regulatory and Development Authority (IRDA) is a national agency of the Government of
India and is based in Hyderabad (Andhra Pradesh). It was formed by an Act of Indian Parliament known
as IRDA Act 1999, which was amended in 2002 to incorporate some emerging requirements.

• Mission of IRDA as stated in the act is "to protect the interests of the policyholders, to regulate, promote
and ensure orderly growth of the insurance industry and for matters connected therewith or incidental
thereto."

• The Insurance Regulatory and Development Authority(IRDAI) was constituted to regulate and develop
insurance business in India. As a key part of its role, it is responsible to protect the rights of
policyholders.

• In order to create awareness about IRDAI, it's role, duties and responsibilities are stated here under:

• IRDAI provides a certificate of registration to a life insurance company.

• IRDAI is responsible for the renewal, modification, withdrawal, suspension or cancellation of this
certificate of registration.

• IRDAI frames regulations on protection of policyholders' interests. It offers policyholders the right to voice
their complaints against insurers or insurance companies.

• The IRDAI has set up the grievance redressal cell to take up the complaints of the policyholder.

• It specifies the requisite qualifications, code of conduct and practical training for intermediaries or
insurance intermediaries and agents. It specifies the code of conduct for surveyors and loss assessors;

• It promotes efficiency in the conduct of insurance businesses;

www.ibpsguide.com | estore.ibpsguide.com | www.sscexamguide.com


Insurance Awareness Capsule

• It promotes and regulates activities of professional organisations connected with life insurance;

• It levies fees and other charges to carry out the purposes of the IRDAI Act;

• It can call for information from, undertake the inspection of, conduct enquiries and investigations
including the auditing of insurers, intermediaries, insurance intermediaries and other organizations
connected with the business of life insurance;

• It specifies the form and manner in which books of account should be maintained and statements of
accounts should be rendered by insurers and other insurance intermediaries;

• It regulates the investment of funds by insurance companies;

• It regulates the maintenance of margins of solvency;

• It adjudicates disputes between insurers and intermediaries or insurance intermediaries;

• It specifies the percentage of premium income of the insurer to finance schemes for the promotion and
regulation of certain specified professional organisations;

• It specifies the percentage of life insurance business to be undertaken by an insurer in the rural or social
sector; and It exercises any other powers as may be prescribed

MISSION STATEMENT OF THE AUTHORITY:


• To protect the interest of and secure fair treatment to policyholders;

• To bring about speedy and orderly growth of the insurance industry (including annuity and
superannuation payments), for the benefit of the common man, and to provide long term funds for
accelerating growth of the economy;

• To set, promote, monitor and enforce high standards of integrity, financial soundness, fair dealing and
competence of those it regulates;

• To ensure speedy settlement of genuine claims, to prevent insurance frauds and other malpractices and
put in place effective grievance redressal machinery;

• To promote fairness, transparency and orderly conduct in financial markets dealing with insurance and
build a reliable management information system to enforce high standards of financial soundness
amongst market players;

www.ibpsguide.com | estore.ibpsguide.com | www.sscexamguide.com


Insurance Awareness Capsule

• To take action where such standards are inadequate or ineffectively enforced;

• To bring about optimum amount of self-regulation in day-to-day working of the industry consistent with
the requirements of prudential regulation.

General Insurance Council

History: The General Insurance Council is a constitutional body under the Indian Insurance Act 1938.
As per the Section 64 C of Part II-A of the Indian Insurance Act 1938, there shall be two Councils of the
Insurance Association of India: The Life Insurance Council and associate members of the Association, and
The General Insurance Council consisting of all the members and associate members of the Association. At
present, there are 22 members of the General Insurance Council of India.

General overview: GI Council membership is extended by invitation to all insurance companies licensed to
underwrite non-life insurance business of any class in India. The General Insurance Council stands for the
combined interests of the Non-life Insurance companies in India. The Council takes care of the issues of
common interest; helps to enlighten and contribute in discussions associated with policy formation; and work
as an advocate for high standards of customer service in the insurance industry.

Government Sponsored Socially Oriented Insurance Schemes

• Aam Aadmi Bima Yojana(AABY)


• Janashree Bima Yojana
• Shiksha Sahayog Yojana (SSY)
• Micro-Insurance Products
• Varishtha Pension Bima Yojana (VPBY)
• Universal Health Insurance Scheme (UHIS)
• National Agricultural Insurance Scheme (NAIS)
• Pilot Modified National Agricultural Insurance Scheme (MNAIS)
• Pilot Weather based Crop Insurance Scheme (WBCIS)

Aam Aadmi Bima Yojana(AABY)


• AAM ADMI BIMA YOJANA, a Social Security Scheme for rural landless household was launched on
2nd October, 2007 at the hands of the then Hon'ble Finance Minister at Shimla. The head of the family or one
earning member in the family of such a household is covered under the scheme. The premium of Rs.200/-

www.ibpsguide.com | estore.ibpsguide.com | www.sscexamguide.com


Insurance Awareness Capsule

per person per annum is shared equally by the Central Government and the State Government. The member
to be covered should be aged between 18 and 59 years.
Benefits

On natural death Rs. 30000/-


On Death due to accident/on permanent total disability Rs. 75000/-
due to accident (loss of 2 eyes or 2 limbs)
On partial permanent disability due to accident (loss of one eye or one Rs. 37500/-
limb)

A separate fund called "Aam Admi Bima Yojana Premium Fund" has been set up by Central Govt. to pay the
Govt. contribution. Fund is maintained by LIC. A free add-on benefit in the form of scholarship to children is
also available under the Scheme.

Janashree Bima Yojana


Janashree Bima Yojana (JBY) was launched on 10th August 2000. The Scheme has replaced Social Security
Group Insurance Scheme (SSGIS) and Rural Group Life Insurance Scheme (RGLIS). 45 occupational groups
have been covered under this scheme

No. Occupation No. Occupation


1 Beedi workers 24 Hilly area woman
2 Brick kiln workers 25 Food stuffs like khandsari/Sugar
3 Carpenters 26 Textile
4 Cobblers 27 Manufacture of food products
5 Fishermen 28 Manufacture of paper products
6 Hamals 29 Manufacture of leather products
7 Handicraft Artisans 30 Printing
8 Handloom Weavers 31 Rubber and coal products
9 Handloom & khadi weavers 32 Chemical products like candle manufacture
10 Lady tailors 33 Mineral products like earthern toys
manufacture
11 Leather Tannery workers 34 Agriculturists
12 Papad workers attached to SEWA 35 Transport drivers association
13 Physically handicapped self employed persons 36 Transport karmacharis
14 Primary milk producers 37 Rural poor
15 Rickshaw pullers/Auto Drivers 38 Construction workers
16 Safai karmacharis 39 Fire crackers workers
17 Salt growers 40 Coconut processors
18 Tenduleaf collectors 41 Aanganwadi Workers/Helpers

www.ibpsguide.com | estore.ibpsguide.com | www.sscexamguide.com


Insurance Awareness Capsule

19 Scheme for urban poor 42 Kotwal


20 Forest workers 43 Plantation workers
21 Sericulture 44 Woman associated with SHG
22 Toddy tappers 45 Sheep breeders
23 Power loom workers

It provides life insurance protection to people who are below poverty line or marginally above poverty line.
Persons between aged 18 years and 59 years and who are the members of the identified 45 occupational
groups are eligible to be covered under the Scheme

Shiksha Sahayog Yojana (SSY) as add-on Benefit


The scheme was launched on 31st December, 2001. Scholarship as a free add-on benefit is provided under
both Janashree Bima Yojana and Aam Admi Bima Yojana to maximum of two children of the beneficiary
studying between 9th to 12th standard (including ITI courses) @ Rs. 100 per month for each child payable
half yearly on 1st July and 1st January, every year. The benefit is without any additional premium.

For meeting the expenditure on Scholarship benefit under Aam Admi Bima Yojana a separate Fund has been
set up by Govt. of India called "Aam Admi Bima Yojana Scholarship Fund. Fund is maintained by LIC of India.
For Jana Shree Bima Yojana scholarship expenditure is paid out of Social Security Fund.

Apart from above two schemes, the earlier old Social Security Schemes namely Social Security Group
Scheme, Integrated Rural Development Programme (IRDP), Swarnjayanti Gram Swarojar Yojana(SGSY) for
the existing lives continued to be administered by LIC. These schemes are closed for the new lives from the
year 2000 onwards.

Micro-Insurance Products
"Jeevan Madhur" a simple savings related life insurance plan for low income persons was launched in 2006.
On surviving to the date of maturity, sum assured is paid alongwith vested bonus if any. On death of the
policy holder, death benefit amount equal to the total premiums payable during the entire term of the policy
will be paid alongwith vested bonus if any.

"Jeevan Mangal", LIC's second Micro Insurance product, was launched in 2009. It is a term insurance plan
with return of premiums paid on maturity, provided the policy is in force. On death during the term of the
policy, the sum assured under the basic plan is payable, provided the policy is in force

Varishtha Pension Bima Yojana (VPBY)


VPBY meant for senior citizens aged 55 years and above was launched on 14.7.2003. Under the scheme the
pensioner gets an effective yield of 9% per annum on the investment. The difference between the effective

www.ibpsguide.com | estore.ibpsguide.com | www.sscexamguide.com


Insurance Awareness Capsule

yield of 9% paid to the pensioner and that earned by LIC is compensated as subsidy to LIC by the
Government of India.

Universal Health Insurance Scheme (UHIS):


The four public sector general insurance companies have been implementing Universal Health Insurance
Scheme for improving the access of health care to poor families. The scheme provides for reimbursement of
medical expenses upto Rs.30,000/- towards hospitalization floated amongst the entire family, death cover
due to an accident @ Rs.25,000/- to the earning head of the family and compensation due to loss of earning
of the earning member @ Rs.50/- per day upto maximum of 15 days. The Universal Health Insurance Scheme
(UHIS) has been redesigned targeting only the BPL families. The premium subsidy has been enhanced from
Rs.100 to Rs.200 for an individual, Rs.300 for a family of five and Rs.400 for a family of seven, without any
reduction in benefits.

National Agricultural Insurance Scheme (NAIS)


The Government of India introduced the scheme from Rabi 1999-2000 season to protect the farmers against
losses suffered by them due to crop failure on account of natural calamities. The scheme is currently
implemented by Agriculture Insurance Company of India (AICIL). The scheme is available to all the farmers,
loanee and non-loanee, irrespective of size of their holding. The scheme covers all food crops (cereals,
millets and pulses) and oil seeds and Annual commercial/ horticultural crops. At present, 10% subsidy on
premium is available to small & marginal farmers. NAIS is presently being implemented in 24 States and 2
Union Territories except in States of Punjab & Arunachal Pradesh. Nagaland has given consent to implement
the scheme and Rajasthan has decided to implement WBCIS in place of NAIS. Since the inception of the
scheme and until up to 31.03.11 about 176 million farmers have been insured, covering an area of 269 million
hectares for a sum insured value of Rs. 2,21,213 crore, against a premium of Rs. 6589 crore. Claims to the
tune of about Rs. 22190 crore have been reported so far benefiting nearly 47.6 million farmers representing a
claim ratio of 1:3.37.

Claims are automatically calculated based on shortfall in the current season yield obtained from crop cutting
experiments conducted by State Governments under General Crops Estimation Survey (GCES) as compared
to threshold yield and settled through the rural banking network. The Company is making efforts to bring the
remaining States/ UTs into the fold of NAIS.

Pilot Modified National Agricultural Insurance Scheme (MNAIS)


Pilot MNAIS was launched for implementation in 50 districts during Rabi 2010-11 season. Modified NAIS has
many improvements over NAIS like the insurance unit for major crops has been lowered down to village /
village Panchayat, minimum indemnity level has been raised to 70%, threshold yield is based on past seven
years' yield excluding a maximum of two calamity years, pre-sowing and post-harvest loss are covered.
Besides these, On-account payment of claims during the season and payment of claims for sowing failure

www.ibpsguide.com | estore.ibpsguide.com | www.sscexamguide.com


Insurance Awareness Capsule

have also been included. The benefit of individual assessment of claims due to localized calamities i.e.
hailstorm and landslide has been extended to all the notified areas.

The minimum number of CCEs required to be conducted at village / village Panchayat level, for all crops
except groundnut, has been reduced to four.

AIC implemented MNAIS during Rabi 2010-11 in 32 Districts across 12 States. More than 3.45 lac farmers
were covered for a sum insured of Rs. 69193 lac. The gross premium was Rs. 45.20 crore, of which premium
collected from farmers was Rs. 2293 lac. Claims of Rs. 22.45 lac due to localized calamity i.e. hailstorm have
been paid to 3842 farmers in Uttar Pradesh while losses reported in Andhra Pradesh are being assessed and
would be paid soon. The final claims due to yield loss, shall be paid after receipt of yield data from the State
Governments.

Assam, Gujarat, Jharkhand, Karnataka, Madhya Pradesh, Maharashtra, Uttar Pradesh and Mizoram States
have so far issued Notification for implementation of MNAIS during Kharif 2011 for 20 districts.

Other States expected to notify the pilot MNAIS during Kharif 2011 season are Andhra Pradesh (3 districts),
Bihar (3 districts), Tamilnadu (3 districts), Uttarakhand (2 districts) and Haryana (1 district).
Orissa (5 districts) & Chhatisgarh (1 district) who piloted during Rabi 2010-11 have decided not to implement
the pilot during Kharif 2011, mainly due to increased load of CCEs at Village Panchayat level.

Pilot Weather based Crop Insurance Scheme (WBCIS)


Weather Based Crop Insurance Scheme (WBCIS) aims to mitigate the hardship of the insured farmers
against the likelihood of loss on account of anticipated crop loss resulting from incidence of adverse
conditions of weather parameters like rainfall, temperature, frost, humidity etc.

While crop insurance specifically indemnifies the cultivator against shortfall in crop yield, WBCIS is built upon
the fact that weather conditions affect crop production even when a cultivator has taken all the care to
ensure good harvest. Historical correlation studies of crop yield with weather parameters helps in developing
weather thresholds (triggers) beyond which crop starts getting affected adversely. Payout structures are
developed to compensate cultivators to the extent of losses deemed to have been suffered by them using the
weather triggers. In other words, WBCIS uses weather parameters as 'proxy' for crop yield in compensating
the cultivators for deemed crop losses. Pursuant to the budget proposals, AICIL introduced a Pilot Weather
Based Crop Insurance Scheme (WBCIS) in Karnataka during Kharif 2007 season covering 70 Hoblis in
respect of eight rain-fed crops.

AICIL implemented the pilot scheme on Weather Based Crop Insurance Scheme (WBCIS) during 2007-08 in 5
States spread over 190 Tehsils / blocks of 30 districts and covered about 6.71 lakh farmers growing in about
10.35 lakh hectares of crops.

www.ibpsguide.com | estore.ibpsguide.com | www.sscexamguide.com


Insurance Awareness Capsule

Important Schemes and its Functions

• PRADHAN MANTRI FASAL BIMA YOJANA (PMFBY)


• Arogya Raksha Yojana
• NATIONAL AGRICULTURAL INSURANCE SCHEME (NAIS)
( A NOVEL CROP INSURANCE SCHEME) {Question-Answer method description}

PRADHAN MANTRI FASAL BIMA YOJANA (PMFBY)

OBJECTIVES:

• To provide insurance coverage and financial support to the farmers in the event of failure of any of the
notified crop as a result of natural calamities, pests & diseases.
• To stabilise the income of farmers to ensure their continuance in farming.
• To encourage farmers to adopt innovative and modern agricultural practices.
• To ensure flow of credit to the agriculture sector.

IMPLEMENTING AGENCY (IA):


The Scheme shall be implemented through a multi-agency framework by selected insurance companies
under the overall guidance & control of the Department of Agriculture, Cooperation & Farmers Welfare
(DAC&FW), Ministry of Agriculture & Farmers Welfare (MoA&FW), Government of India (GOI) and the
concerned State in co-ordination with various other agencies; viz Financial Institutions like Commercial
Banks, Co-operative Banks, Regional Rural Banks and their regulatory bodies, Government Departments viz.
Agriculture, Co-operation, Horticulture, Statistics, Revenue, Information/Science & Technology, Panchayati
Raj etc.

DAC&FW has designated/empanelled Agriculture Insurance Company of India(AIC) and some private
insurance companies presently to participate in the Government sponsored agriculture /crop insurance
schemes based on their financial strength, infrastructure, manpower and expertise etc. The empanelled
private insurance companies at present are 1) ICICI-Lombard General Insurance Company Ltd. 2) HDFC-
ERGO General Insurance Company Ltd. 3) IFFCO-Tokio General Insurance Company Ltd. 4) Cholamandalam
MS General Insurance

Company Ltd. 5) Bajaj Allianz General Insurance Company Ltd. 6) Reliance General Insurance Company Ltd.
7) Future Generali India Insurance Company Ltd. 8) Tata-AIG General Insurance Company Ltd. 9) SBI
General Insurance Company Ltd. 10) Universal Sompo General Insurance Company Ltd. The selection of
insurance company from amongst the empanelled insurance companies to act as IA shall be done by the
concerned State Government for implementation of the scheme in their State. Such selection of IA shall be

www.ibpsguide.com | estore.ibpsguide.com | www.sscexamguide.com


Insurance Awareness Capsule

done from amongst the designated / empanelled companies which shall be be initially pre-qualified , strictly
on the basis of, experience, existence of infrastructure in the area and quality of services like coverage of
farmers & area, pay-outs in terms of quantum & timely settlement thereof, willingness to do publicity &
awareness campaigns etc. The final selection of IA from amongst the pre-qualified insurance companies
shall be done based on the lowest weighted premium quoted by a pre-qualified company for all notified
crops within the cluster of districts

MANAGEMENT OF THE SCHEME:

The existing State Level Co-ordination Committee on Crop Insurance (SLCCCI), Sub-Committee to SLCCCI,
District Level Monitoring Committee (DLMC) already overseeing the implementation & monitoring of the
ongoing crop insurance schemes like National Agricultural Insurance Scheme (NAIS), Weather Based Crop
Insurance Scheme(WBCIS), Modified National Agricultural Insurance Scheme(MNAIS) and Coconut Palm
Insurance Scheme(CPIS) shall be responsible for proper management of the Scheme. IA shall be an active
member of SLCCCI and District Level Monitoring Committee (DLMC) of the scheme.

UNIT OF INSURANCE:

The Scheme shall be implemented on an ‘Area Approach basis’ i.e., Defined Areas for each notified crop for
widespread calamities with the assumption that all the insured farmers, in a Unit of Insurance, to be defined
as „Notified Area‟ for a crop, face similar risk exposures, incur to a large extent, identical cost of production
per
hectare, earn comparable farm income per hectare, and experience similar extent of crop loss due to the
operation of an insured peril, in the notified area.

Defined Area (i.e., unit area of insurance) is Village/Village Panchayat level by whatsoever name these areas
may be called for major crops and for other crops it may be a unit of size above the level of Village/Village
Panchayat.

In due course of time, the Unit of Insurance can be a Geo-Fenced/Geo-mapped region having homogenous
Risk Profile for the notified crop.

For Risks of Localised calamities and Post-Harvest losses on account of defined peril, the Unit of Insurance
for loss assessment shall be the affected insured field of the individual farmer.

CROPS AND NOTIFIED AREA:

CROPS: The Scheme can cover all the Crops for which past yield data is available and grown during the
notified season, in a Notified Area and for which yield estimation at the Notified Area level will be available

www.ibpsguide.com | estore.ibpsguide.com | www.sscexamguide.com


Insurance Awareness Capsule

based on requisite number of Crop Cutting Experiments (CCEs) being a part of the General Crop Estimation
Survey (GCES).

NOTIFIED AREA: Notified Area is the Unit of Insurance decided by the State Govt. for notifying a Crop
during a season. The size of the Unit of Insurance shall depend on the area under cultivation within the unit.
For major crops, the Unit of

Insurance shall ordinarily be Village/Village Panchayat level and for minor crops may be at a higher
level so that the requisite number of CCEs could be conducted during the notified crop season. States may
notify Village / Village Panchayat as insurance unit in case of minor crops too if they so desire.

FARMERS TO BE COVERED: All farmers growing notified crops in a notified area


during the season who have insurable interest in the crop are eligible.

COMPULSORY COVERAGE: The enrolment under the scheme, subject to possession of insurable interest
on the cultivation of the notified crop in the notified area, shall be compulsory for following categories of
farmers:

1) Farmers in the notified area who possess a Crop Loan account/KCC account (called as Loanee Farmers)
to whom credit limit is sanctioned/renewed for the notified crop during the crop season.
AND
2) Such other farmers whom the Government may decide to include from time to time.

VOLUNTARY COVERAGE: Voluntary coverage may be obtained by all farmers not covered in 7.1 above,
including Crop KCC/Crop Loan Account holders whose credit limit is not renewed.

RISKS TO BE COVERED & EXCLUSIONS:

RISKS: Following risks leading to crop loss are to be covered under the scheme :-

YIELD LOSSES (standing crops, on notified area basis): Comprehensive

risk insurance is provided to cover yield losses due to non-preventable risks, such as

(i) Natural Fire and Lightning


(ii) Storm, Hailstorm, Cyclone, Typhoon, Tempest, Hurricane, Tornado etc.
(iii) Flood, Inundation and Landslide
(iv) Drought, Dry spells
(v) Pests/ Diseases etc.

www.ibpsguide.com | estore.ibpsguide.com | www.sscexamguide.com


Insurance Awareness Capsule

PREVENTED SOWING (on notified area basis):- In cases where majority of the insured farmers of a
notified area, having intent to sow/plant and incurred expenditure for the purpose, are prevented from
sowing/planting the insured crop due to adverse weather conditions, shall be eligible for indemnity claims
upto a maximum of 25% of the sum-insured.

POST-HARVEST LOSSES (individual farm basis): Coverage is available upto a maximum period of 14 days
from harvesting for those crops which are kept in “cut & spread” condition to dry in the field after harvesting,
against specific perils of cyclone / cyclonic rains, unseasonal rains throughout the country.

LOCALISED CALAMITIES (individual farm basis): Loss / damage resulting from occurrence of identified
localized risks i.e. hailstorm, landslide, and Inundation affecting isolated farms in the notified area.

EXCLUSIONS: Risks and Losses arising out of following perils shall be excluded:-

War & kindred perils, nuclear risks, riots, malicious damage, theft, act of enmity, grazed and/or destroyed by
domestic and/or wild animals, In case of Post–Harvest losses the harvested crop bundled and heaped at a
place before threshing, other preventable risks.

SUM INSURED / LIMIT OF COVERAGE:


In case of Loanee farmers under Compulsory Component, the Sum Insured would be equal to Scale of
Finance for that crop as fixed by District Level Technical Committee (DLTC) which may extend up to the
value of the threshold yield of the insured crop at the option of insured farmer. Where value of the threshold
yield is lower than the Scale of Finance, higher amount shall be the Sum Insured. Multiplying the Notional
Threshold Yield with the Minimum Support Price (MSP) of the current year arrives at the value of sum
insured. Wherever Current year‟s MSP is not available,

MSP of previous year shall be adopted. The crops for which, MSP is not declared, farm
gate price established by the marketing department / board shall be adopted.

Further, in case of Loanee farmers, the Insurance Charges payable by the farmers shall be financed by loan
disbursing office of the Bank, and will be treated as additional component to the Scale of Finance for the
purpose of obtaining loan.
For farmers covered on voluntary basis the sum-insured is upto the value of Threshold yield i.e threshold
yield x (MSP or gate price) of the insured crop.

PREMIUM RATES:
i) The Actuarial Premium Rate (APR) would be charged under PMFBY by IA. DAC&FW/States will
monitor the premium rates considering the basis of Loss Cost (LC) i.e. Claims as % of Sum Insured (SI)

www.ibpsguide.com | estore.ibpsguide.com | www.sscexamguide.com


Insurance Awareness Capsule

observed in case of the notified crop(s) in notified unit area of insurance (whatsoever may be the level of
unit area) during the preceding 10 similar crop seasons (Kharif / Rabi) and loading for the expenses
towards management including capital cost and insurer‟s margin and taking into account non-parametric
risks and reduction in insurance unit size etc.. The rate of Insurance Charges payable by the farmer will be
as per the following table:

S.No Season Crops Maximum Insurance


charges payable by farmer
(% of Sum Insured)

1 Kharif Food & Oilseeds crops (all 2.0% of SI or Actuarial rate,


cereals, millets, & oilseeds, whichever is less
pulses)

2 Rabi Food & Oilseeds crops (all 1.5% of SI or Actuarial rate,


cereals, millets, & oilseeds, whichever is less
pulses)

3 Kharif & Annual Commercial / Annual 5% of SI or Actuarial rate,


Rabi Horticultural crops whichever is less

ii) The difference between premium rate and the rate of Insurance charges payable by farmers shall be
treated as Rate of Normal Premium Subsidy, which shall be shared equally by the Centre and State.
iii) AIC shall calculate LC premium rates (till an Independent Agency/TSU takes over) based on latest
available yield data in month of February for Kharif crops and August for Rabi crops as per requirement of
the States and shall provide to DAC&FW/Concerned States before invitation for premium bidding.
iv) State Govt. would invite all the empanelled insurance companies to quote their actuarial premium
rates for the notified crop(s) in the notified insurance unit area, Indemnity Level, Threshold Yields, Sum
Insured etc. as indicated by the State for the season.
v) For more effective implementation, selection of Implementing Agency (IA) may be made through
adopting the cluster approach under which bunch of about 15-20 good & bad districts / areas with
reference to risks will be bid out. This will facilitate the uniform distribution of the risks among the
participating insurance companies and will avoid selection of districts / areas according to company‟s
choice. In case of smaller States, the whole State shall be assigned to one IA. This is also expected to take
care of districts which have traditionally had high actuarial premiums for crops due to high risk. Selection of
IA may be made for at least 3 years.

www.ibpsguide.com | estore.ibpsguide.com | www.sscexamguide.com


Insurance Awareness Capsule

vi) The designated / empanelled companies participating in bidding have to bid the premium rates for all
the crops notified / to be notified by the State Govt. and non-compliance will lead to rejection of company‟s
bid.
vii) The insurance coverage in terms of number of farmers & hectare-age should be at least at the
previous season's level.

SHARING OF RISK:
Risk will be shared by IA and the Government as follows:

The liability of the Insurance companies in case of catastrophic losses computed at the National level
for an agricultural crop season, shall be upto 350% of total premium collected (farmer share plus
Govt. subsidy) or 35% of total Sum Insured (SI), of all the Insurance Companies combined,
whichever is higher. The losses at the National level in a crop season beyond this ceiling shall be
met by equal contribution (i.e. on 50:50 basis) from the Central Government and the concerned
State Governments.

ESTIMATION OF CROP YIELD:


The State/UT Govt. will plan and conduct the requisite number of Crop Cutting Experiments (CCEs)
for all notified crops in the notified insurance units in order to assess the crop yield. The State / UT
Govt. will maintain single series of Crop Cutting Experiments (CCEs) and resultant Yield estimates,
both for Crop Production estimates and Crop Insurance.

Crop Cutting Experiments (CCE) shall be undertaken per unit area /per crop, on a sliding scale, as
indicated below:

INSURANCE UNIT Minimum no. of CCEs required to be


S No.
done

1. District 24
2. Taluka / Tehsil / Block 16
3. Mandal/Hobli/ Phirka / Revenue 10
Circle
4. Village / GramPanchayat/Patwar- 4 for major crops, 8 for other crops
Mandal/Patwari-Halka

However, a Technical Advisory Committee (TAC) comprising representatives


from Indian Agricultural Statistical Research Institute (IASRI), National Sample Survey Organization
(NSSO), Ministry of Agriculture & Farmers Welfare (GoI) and implementing agencies shall dispose/decide

www.ibpsguide.com | estore.ibpsguide.com | www.sscexamguide.com


Insurance Awareness Capsule

the issues relating to CCEs and all other technical matters. Inputs from RST/satellite imagery would also
be utilized in optimizing the sample size of CCEs.

Use of Mobile Phone Technology to improve Yield-data Quality & Timeliness


It has been felt that process of CCEs currently being conducted for estimating yield is lacking in reliability
and speed which affects the claims settlement. There is a need to have good quality, timely and reliable
yield-data. For addressing this problem, video/image capture of crop growth at various stages and
transmission thereof with CCE data on a real time basis utilizing mobile communication technology with GPS
time stamping, can improve data quality, / timeliness and support timely claim processing and payments.
States and insurance companies shall utilise this technology for the purpose.

The cost of using technology etc. for conduct of CCEs etc will be shared between Central Government and
State/U.T. Governments on 50:50 basis, wherever necessary, subject to a cap on total funds to be made
available by Central Government for this purpose based on approximate cost of procuring hand held
devices/Smart phones and other related costs.

INDEMNITY LEVEL (IL) and THRESHOLD YIELD (TY) :


(i) Three levels of Indemnity, viz., 70%, 80% and 90% corresponding to crop Risk in the areas shall be
available for all crops.

(ii) The Threshold Yield (TY)shall be the benchmark yield level at which Insurance protection shall be
given to all the insured farmers in an Insurance Unit.

(iii) The Threshold Yield for a crop in an Insurance Unit shall be based on average yield of last seven
years excluding two years of declared calamity if any, multiplied by the level of indemnity of the
area.

CALENDER OF ACTIVITY:

The time-lines for coverage, submission of yield data, price data etc. shall be decided by the SLCCCI
strictly keeping in mind the onset of monsoon, sowing period, crop cycle etc.

The seasonality discipline shall be same for loanee and non-loanee farmers.

The cut-off date is uniform for both loanee and non-loanee cultivators. Keeping in view the prevailing
agro-climatic conditions, rainfall distribution/irrigation water availabilities, sowing pattern etc. the
SLCCCI, in consultation with the insurance company shall fix seasonality disciplines of the coverage
and other activities in such a way that it doesn‟t encourage adverse selection or moral hazards. Broad

www.ibpsguide.com | estore.ibpsguide.com | www.sscexamguide.com


Insurance Awareness Capsule

seasonality discipline is given in the chart below. State wise details of seasonality will be provided in
the Operational Guidelines to be issued by DAC&FW.

S.No.
Kharif Rabi
Activity

1 February August
Issuance of Administrative Instructions by
Government of India

Conduct of SLCCCI meeting to decide


for notification of Crops and Notified
2 March September
areas, limits of Sum Insured and
adoption of Level of Indemnity etc.

3 Loaning period (loan sanctioned) for April to July October to

Loanee farmers covered on December


Compulsory basis.

4 Cut-off date for receipt of Proposals of 31th July 31st December


farmers (loanee & non-loanee).

5 Cut-off date for receipt of Declarations Within 15 working Within 15 working


of Loanee farmers covered on days for loanee days for loanee
compulsory basis & non-loanee farmers and seven farmers and
farmers covered on Voluntary basis working days for seven working
from Bank branches to Respective non-loanee farmers days for non-
Nodal Offices. after cut-off date loanee farmers
after cut-off date

6 Cut-off date for receipt of Declarations Within two working Within two
of farmers covered on Voluntary basis days after receiving working days after
from designated Insurance Agent(s) to declaration/ receive
Insurance Companies premium. declaration/
premium.

www.ibpsguide.com | estore.ibpsguide.com | www.sscexamguide.com


Insurance Awareness Capsule

Cut-off date for receipt of Declarations


Within seven Working Within seven working
of Loanee farmers covered on
days from receipt of days from receipt of
compulsory basis & non-loanee
7 Declarations by the Declarations by the
farmers covered on Voluntary basis
Respective Nodal Offices Respective Nodal
from Respective Nodal Offices of
of Banks. Offices of Banks.
Banks to Insurance Company

8 Cut-off date for receipt of yield data Within a month Within a month
from final harvest from final harvest

PROCEDURE FOR ASSESSMENT, PROCESSING AND APPROVAL OF CLAIMS:


I. Yield losses at Notified Area level: Once the Yield Data is received from the State/UT Govt.
as per the prescribed cut-off dates, claims will be processed, approved and settled by IA.

i) If the „Actual Yield‟ (AY) per hectare of the insured crop for the defined area [on the basis of
requisite number of Crop Cutting Experiments (CCEs)] in the insured season, falls short of the specified
threshold yield(TY) Yield‟ (RY), all the insured farmers growing that crop in the defined area are deemed to
have suffered shortfall in their yield.

The Scheme seeks to provide protection against such contingency to all insured farmers of an Insurance
Unit.

Claim Pay-outs based on Yield losses shall be calculated as per the following formula:

Assessment of prevented Sowing: The adverse weather conditions shall be defined in the notification and
shall be captured by notified weather station/s in the District. The extent of claims payable will be decided
on the basis of weather data recorded at the notified weather station/s for the purpose. The crop-wise scale
of payment, upto a maximum of 25% of Sum Insured shall be worked out by IA based on a notified pay-out
structure on the occurrence of pre-declared events such as month-wise deficit in aggregate rainfall during a
specified period assessed through Reference Weather Stations tagged for the Notified / Group of Notified
Area. The insurance coverage shall cease to operate for the crop in the notified area. The cover is available
during Kharif season for recognised rain-fed areas and crops. The data provider will be notified by the
SLCCCI.

Localized Calamity Loss Assessment:

www.ibpsguide.com | estore.ibpsguide.com | www.sscexamguide.com


Insurance Awareness Capsule

Loss assessment and modified indemnity procedures in case of occurrence of localized perils, such as
hailstorm, landslide, flood, and inundation shall be for a cluster of affected farms or affected village and the
settlement of claims, if any, will be each insured farmer covered under assessment.

i) The District Administration will assist IA in assessing the extent of loss.

Post-Harvest Loss Assessment:


i) Loss assessment and indemnity procedures in case of occurrence of Post-Harvest Loss shall be for
a cluster of affected farms or affected village and the settlement of claims, if any, will be each insured farmer
covered under assessment.

ii) The District Administration will assist IA in assessing the extent of loss.

On-Account Payment of Claims due to Mid-Season Adversity:


i) In case of adverse seasonal conditions during crop season viz. floods, prolonged dry spells, severe
drought, unseasonal rains, IA in consultation with concerned State Government/UT based on agro
meteorological data/ satellite imagery or any other proxy indicator will decide about crops/ areas for which
on account payment will be made, not exceeding 25% of likely claims.

ii) Appraisal of mid-season adversity and quantum of on-account payment will be established jointly by
Government of India/concerned State Government/UT and IA.

iii) On account payment will be implemented only in those districts/notified areas where such proxy
indicators can be established and will be considered for payment and only if the expected yield during the
season is likely to be less than 50% of normal yield.

IA shall process the claims liability assessed as per above mentioned methodology and approve the claims.

PROCEDURE FOR SETTLEMENT OF CLAIMS:


For coverage through Banks:-The claim amount along with particulars will be released to the individual
Nodal Banks. The Banks at the grass-root level, in turn, shall credit the accounts of the individual farmers
and display the particulars of beneficiaries on their notice board. The Banks shall provide individual farmer
wise details claim credit details to IA and shall be incorporated in the centralised data repository.

For coverage through other insurance intermediaries: The claim amount will be released electronically to
the individual Insured Bank Account.

Acreage discrepancy

www.ibpsguide.com | estore.ibpsguide.com | www.sscexamguide.com


Insurance Awareness Capsule

Some areas in the past have reported excess insurance coverage vis-à-vis planted acreage, leading to „over‟
insurance. Ideally the discrepancy should be handled at farm level to protect the interest of farmers with
genuine insurance coverage. However, in the absence of digitized farm records on a GIS platform, it would be
cumbersome to physically verify each farm. For the time-being, it is to be addressed as follows:
- Wherever the „acreage discrepancy‟ is likely, the acreage insured at IU level shall be compared with
average planted acreage of past three years, and the difference will be treated as „excess‟ insurance
coverage.

- Sum insured may be scaled down in the ratio the average of three years‟ actual planted acreage bears to
the insured acreage for the given crop.

- Claims shall be calculated on the scaled down sum insured

- Premium shall be retained by the insurance company for the portion of sum insured scaled down.

Once the individual farms (plots / survey numbers) are digitized and available on a GIS platform, it is
possible to overlay the crop cover as derived using satellite imagery on the GIS platform to identify the crop
and estimate the cropped area on each farm. This should lead to identifying the acreage discrepancy at
individual farm level.

MANAGEMENT OF THE SCHEME AND REVIEW:


Government of India shall issue operational guidelines and modalities, which may be appended from
time to time, for implementation of the scheme provisions with detailed steps and processes involved,
terms and condition applicable, roles and responsibilities of various agencies involved in execution of the
scheme and roles and responsibilities of other related stakeholders.

i) These operational modalities shall be considered as part of the scheme.

ii) The scheme may be reviewed periodically and additions, deletions and modifications of the
provisions may be done as deemed necessary.

iii) During each crop season, the agricultural situation will be closely monitored in the implementing
States / Union Territories. The State / UT Department of Agriculture and district administration shall set up
a District Level Monitoring Committee (DLMC), who will provide fortnightly reports of Agricultural situation
with details of area sown, seasonal weather conditions, pest incidence, stage of crop failure (if any) etc.

iv) The operation of the Scheme will be reviewed annually, and modifications as may be required would
be introduced. Periodic Appraisal Reports on the Scheme would be prepared by Ministry of Agriculture, the
Government of India / Implementing Agency.

www.ibpsguide.com | estore.ibpsguide.com | www.sscexamguide.com


Insurance Awareness Capsule

PUBLICITY & AWARENESS:


i) Adequate publicity needs to be given in all the villages of the notified districts/ areas. All possible
means of electronic and print media, farmer‟s fair, exhibitions including SMS messages, short films, and
documentaries shall be utilized to create and disseminate awareness, benefits and limitations of the
Scheme among the cultivators and the agencies involved in implementing the Scheme.
Agriculture/Cooperation Department of the States in consultation with IA shall work out appropriate Plan
for adequate awareness and publicity three months prior to the start of coverage period.

ii) IA shall also assist the State Government/ UT in capacity building for effective implementation of
the scheme and organize training workshops/sensitization programme for various stakeholders.

SERVICE TAX:
i) PMFBY is a replacement scheme of NAIS / MNAIS, and hence exempted from Service Tax liability
of all the services involved in the implementation of the scheme.

USE OF INNOVATIVE TECHNOLOGY:


DAC&FW shall carry out pilots in select areas, in collaboration with various States/UTs, national and
international research organisations / institutes, IMD, insurance companies, reinsurers etc. to make use of
available technology in the fields of remote sensing, aerial imagery, satellites etc. that can help in acreage
estimation, crop health / loss estimation, quicker yield estimation etc. with reduced manpower &
infrastructure. With development of number of satellites with high resolution images orbiting the Earth, there
have been great improvements in satellite imagery products. It has been reasonably proven the satellite
imagery can help in demarcating the cropped areas into clusters on the basis of crop health. This feature can
be successfully used to target the CCEs within the Insurance Unit (IU). Thus satellite imagery can help in
‘smart sampling’ of CCEs. While an IU with heterogeneous crop health may need standard sample of CCEs,
for eg. 4 CCEs per Village / Village Panchayat for major crops, the more homogenous IU may need a lower
sample size, say 2 CCEs. This is expected to minimize the total CCEs needed by about 30-40%. States should
progressively adopt this technique in generating yield estimates.
After proven strong correlation between RST / Satellite Imageries results and yield estimates through CCEs,
States may use the technologies in estimating the crop yields at IU level, subject to the satisfaction of
Central and State Governments and insurance companies with the accuracy of the yield estimates to service
the claims.

(i) The integrity of CCEs will be verified by use of GPRS enabled Mobile phones with cameras/smart
phones. These phones will also help in addressing the problem of area discrepancy by capturing pictures of
standing crops and will also help in quicker, accurate estimation of yields.

www.ibpsguide.com | estore.ibpsguide.com | www.sscexamguide.com


Insurance Awareness Capsule

(ii) Such technologies, after due consideration of pilot results by the Government shall be included in
the Scheme.
(iii) All state government shall use technology initiatives in the conduct and supervision of CCEs to
provide the yield data with minimum delay to IA for quick processing of the claims. The state governments
shall also use technology initiatives in the reporting of loss reports for on-account claim settlement, Claim
intimations for Localized calamity and Post-Harvest losses.
(iv) A centralized repository shall be maintained. Appropriate application (web based, app based etc.)
would be developed by NIC. The State Government, IA, Banks, Insurance Intermediaries shall use this
applications for inputting various operational data like notification related data, individual farmer wise
insurance coverage and claims details, crop loss details etc.

REVIEW OF THE SCHEME:


State Governments will review the performance of the scheme after one year and point out corrections, if
any, required in any of the provisions of the scheme to Govt. of India.

Weather Based Crop Insurance Scheme (WBCIS):-

The structure of farmer’s premium under WBCIS will be at par with the proposed PMFBY. Also, the
Criteria of selection of Implementing Agency and area allocation will be same as PMFBY.

Existing Premium Subsidy Structure

Sr. Premium Slab Subsidy by Central and State Government on 50 : 50 basis


No. and premium payable by farmer
1. Upto 2% No subsidy.

2. >2 - 5% 25% subsidy subject to minimum net premium of 2%


payable by farmer.

3. >5 –8% 40% subsidy subject to minimum net premium of 3.75%


payable by farmer

4. > 8% 50% subsidy subject to minimum net premium of 4.8% and


maximum net premium of 6% payable by farmers.

Revised Premium Subsidy Structure


The rate of Insurance Charges payable by the farmer will be as per the following table:

www.ibpsguide.com | estore.ibpsguide.com | www.sscexamguide.com


Insurance Awareness Capsule

S.No Season Crops Maximum Insurance


charges payable by farmer
(% of Sum Insured)

1 Kharif Food & Oilseeds crops (all 2.0% of SI or Actuarial rate,


cereals, millets, & oilseeds, whichever is less
pulses)

2 Rabi Food & Oilseeds crops (all 1.5% of SI or Actuarial rate,


cereals, millets, & oilseeds, whichever is less
pulses)

3 Kharif & Annual Commercial / Annual 5% of SI or Actuarial rate,


Rabi Horticultural crops whichever is less

Unified Package Insurance Scheme (UPIS)

1. Nature of the Scheme:


1.1 Unified Package Insurance Scheme aims at providing financial protection to citizens associated in
agriculture sector, thereby ensuring food security, crop diversification and enhancing growth and
competitiveness of agriculture sector besides protecting farmers from financial risks.

The cover will be for one full year except for Crop Insurance (which will be bi-annual separately for Kharif
and Rabi seasons) renewable from year to year. The scheme would be offered / administered through AIC &
empanelled General Insurance Companies willing to offer the product on similar terms with necessary
approvals and allocation of area on pilot basis.

The Loanee farmers will be covered through Banks/Financial Institutions whereas non-loanee farmer shall
be covered through banks and/or insurance intermediaries.

Suitability:

a) This policy is designed to take care of the insurance needs of farmers associated with agriculture
activities. This policy provides yield based crop insurance to the farmer based on his ownership rights
of land and sown crop.

www.ibpsguide.com | estore.ibpsguide.com | www.sscexamguide.com


Insurance Awareness Capsule

b) It covers both the personal assets of the farmer like the dwelling & its contents (Fire), the other
assets which help him in earning his livelihood such as Agricultural Pump Sets, and Agriculture
Tractor owned by farmer.

c) The policy also provides protection to farmer and his/her family members in case of the Accidental
Death / Disablement, accidental insurance protection of farmer‟s school/college going children and
provisioning of education fee to the students in case of death of parent.

d) Life insurance protection to the farmer and his/her family members.

e) The policy will be issued for a period up to 1 year.

Salient Features and Benefits:


1) The farmers package policy will be underwritten by the General Insurance Companies empanelled by
DAC&FW under crop insurance programmes and/or designated by this Department or through GIC
Companies having tie-up with concerned F.I./Banks for non-crop sections of the policy.

2) The policy contains 7 Sections. Crop Insurance is mandatory. However, farmers have to choose at least
two other sections also to avail the applicable subsidy under crop insurance section.

3) In case of crop insurance, applicable Farmer‟s share of premium ranging between 1.5% to 5% based on
their insured crops is payable by farmer & in case Actuarial premium is more, the Government will provide
subsidy equivalent to the difference between Actuarial premium and premium paid by farmer. The crop
insurance is based on area approach whereas all other sections are on individual basis.

4) If the farmers already availed any insurance policy of similar nature and sum insured not less than as
mentioned in the policy than they would be exempted from taking such section(s). However details of
such policy would be provided in their proposal form.

5) The rates above are indicative & subject to the concurrence of the insurers.

6) Sum Insured and premium rates are provisionally taken and may change according to the risk(s).

7) The detailed section wise tentative sum insured and applicable premium for the farmer(s) are in the
proposal forms enclosed at Annexure-I

8) The above premium rates are without service tax which is likely to be exempted.

www.ibpsguide.com | estore.ibpsguide.com | www.sscexamguide.com


Insurance Awareness Capsule

The Section – wise details of the scheme are as under:

SECTION-1

CROP INSURANCE :- PMFBY/WBCIS/ – State can choose any of these two

SECTION – 2

BUILDING AND CONTENTS INSURANCE (FIRE AND ALLIED PERILS)

The indemnity under this section is based on declared sum insured basis (first loss basis)

The Company will indemnify the Insured in respect of loss of or damage to the Contents/Buildings whilst
contained in the insured premises by:

a) Fire, Lighting, Explosion of gas in domestic appliances,

b) Bursting and overflowing of water tanks, apparatus or pipes,

c) Aircraft or articles dropped therefrom,

d) Riot, Strike, or Malicious Act,

e) Earthquake, (Fire and / or Shock) Subsidence and Landslide (including Rockslide) damage

f) Flood, inundation, storm, tempest, typhoon, hurricane, tornado or cyclone.

g) Impact damage

SPECIAL EXCEPTIONS
The Company shall not be liable in respect of:

a) Loss or damage by burglary and / or housebreaking or theft where any member of the insured‟s family is
concerned as principal or accessory.

b) Loss of or damage to articles of consumable nature.

www.ibpsguide.com | estore.ibpsguide.com | www.sscexamguide.com


Insurance Awareness Capsule

c) Loss of or damage to money, securities, stamps, stamp collections, bullion, livestock, motor vehicles and
pedal cycles.

d) Loss of or damage to deeds, bonds, bills of exchange, promissory notes, shares and stock certificates,
business books, manuscripts, documents of any kind, unset precious stones and Jewelry and Valuable.

SECTION – 3

Personal Accident Insurance

(Coverage as per Pradhan Mantri Suraksha Bima Yojana)

DETAILS OF THE SCHEME:


The scheme will be a one year cover, renewable from year to year, Accident Insurance Scheme offering
accidental death and disability cover for death or disability on account of an accident. The scheme would be
offered / administered through Public Sector General Insurance Companies (PSGICs) and other General
Insurance Companies willing to offer the product on similar terms with necessary approvals and tie up with
Banks for this purpose. Participating banks will be free to engage any such insurance company for
implementing the scheme for their subscribers.

Scope of coverage: All savings bank account holders in the age 18 to 70 years in participating banks will be
entitled to join. In case of multiple saving bank accounts held by an individual in one or different banks, the
person would be eligible to join the scheme through one savings bank account only. Aadhar would be the
primary KYC for the bank account.

Enrollment Modality / Period: The cover shall be for the one year period stretching from 1st June to 31st May
for which option to join / pay by auto-debit from the designated savings bank account on the prescribed
forms will be required to be given by 31st May of every year, extendable up to 31st August. Joining
subsequently on payment of full annual premium may be possible on specified terms. However, applicants
may give an indefinite / longer option for enrollment / auto-debit, subject to continuation of the scheme with
terms as may be revised on the basis of past experience. Individuals who exit from the scheme at any point
may re-join the scheme in future years through this modality. New entrants into the eligible category from
year to year or currently eligible individuals who did not join earlier shall be able to join in future years while
the scheme is continuing.
Benefits: As per the following table:

Table of Benefits Sum Insured


a. Death Rs. 2 Lakh

www.ibpsguide.com | estore.ibpsguide.com | www.sscexamguide.com


Insurance Awareness Capsule

Total and irrecoverable loss of both eyes or loss of use of


b. Both Rs. 2 Lakh
hands or feet or loss of sight of one eye and loss of use of
hand or
Foot
Total and irrecoverable loss of sight of one eye or loss of
c. use of Rs. 1 Lakh
one hand or foot

Premium: Rs.12/- per annum per member. The premium will be deducted from the account holder‟s savings
bank account through „auto debit‟ facility in one installment on or before 1stJune of each annual coverage
period under the scheme. However, in cases where auto debit takes place after 1st June, the cover shall
commence from the first day of the month following the auto debit.

Eligibility Conditions: The savings bank account holders of the participating banks aged between 18 years
(completed) and 70 years (age nearer birthday) who give their consent to join / enable auto-debit, as per the
above modality, will be enrolled into the scheme. In case of Joint Account holders, both the account holders
are eligible to join on payment of premium for each account holders.

Master Policy Holder: Participating Bank will be the Master policy holder on behalf of the participating
subscribers. A simple and subscriber friendly administration & claim settlement process shall be finalized
by the respective general insurance company in consultation with the participating Banks.

Termination of cover: The accident cover for the member shall terminate on any of the following events and
no benefit will be payable there under:

1) On attaining age 70 years (age nearest birthday).


2) At the time of renewal, closure of account with the Bank or insufficiency of balance to keep the
insurance in force.
3) In case a member is covered through more than one account and premium is received by the
Insurance Company inadvertently, insurance cover will be restricted to one only and the premium shall be
liable to be forfeited.
4) If the insurance cover is ceased due to any technical reasons such as insufficient balance on due
date for renewal or due to any administrative issues, the same can be reinstated on receipt of full annual
premium, subject to conditions that may be laid down. During this period, the risk cover will be suspended
and reinstatement of risk cover will be at the sole discretion of Insurance Company.
5) Participating banks will deduct the premium amount in the same month when the auto debit option
is given, preferably in May of every year, and remit the amount due to the Insurance Company in that
month itself.

www.ibpsguide.com | estore.ibpsguide.com | www.sscexamguide.com


Insurance Awareness Capsule

Administration:
The scheme, subject to the above, will be administered as per the standard procedure stipulated by the
Insurance Company. The data flow process and data proforma will be provided separately.

It will be the responsibility of the participating bank to recover the appropriate annual premium from the
account holders within the prescribed period through ‘auto-debit’

process.

Enrollment form / Auto-debit authorization in the prescribed proforma shall be obtained and retained by the
participating bank. In case of claim, the Insurance Company may seek submission of the same. Insurance
Company reserves the right to call for these documents at any point of time.

The experience of the scheme will be monitored on yearly basis for re-calibration etc., as may be necessary.

Appropriation of Premium:
1) Insurance Premium to Insurance Company: Rs.10/- per annum per member

2) Reimbursement of Expenses to BC/Micro/Corporate/Agent : Rs.1/- per annum per member

3) Reimbursement of Administrative expenses to participating Bank: Rs.1/- per annum per member

The scheme is liable to be discontinued prior to commencement of a new future renewal date if
circumstances so require.

SECTION – 4

Agriculture Pumpset Insurance (Upto 10 Horse Power)

The Insurance covers the Centrifugal pump sets (electrical and diesel) upto 10 Horsepower capacity which
are used for agricultural purposes only.

SCOPE OF COVER:

a) Fire & lightning

b) /Burglary (due to violent forcible entry provided the pump set is kept in a locked enclosure).

www.ibpsguide.com | estore.ibpsguide.com | www.sscexamguide.com


Insurance Awareness Capsule

c) Mechanical / electrical

d) Riot, Strike, malicious damage

SPECIFIC EXCLUSION TO AGRICULTURAL PUMPSET INSURANCE


a) Normal wear & tear, gradual deterioration due to atmospheric condition or otherwise.
b) Wilful act or gross negligence of the Insured or his representatives.

c) Faults existing at the time of commencement of insurance and known to the Insured or his
representative.

d) Loss or damage for which the manufacturer or supplier of property is responsible either by law or under
contract.

e) Cost of dismantling, transport to workshop and back as also cost of re-erection.

SPECIAL CONDITIONS
In the event of any occurrence which might give rise to a claim under this Section, the Insured shall -

a) immediately notify the Policy issuing Office of the Company by telephone or telegram as well as in
writing giving an indication as to the nature and extent of loss or damage.

b) take reasonable steps within his power to minimise / restrain the loss or damage of liability.

c) preserve the damaged or defective parts and make them available for inspection by an Official or
Surveyor of the Company.

d) furnish all such information and documentary evidences as the Company may require.

e) the Company shall not be liable for any loss or damage on which notice and completed claim form have
not been received by the Company within 14 days of its occurrence.

The liability of the Company under this Section in respect of any item of property sustaining damage for
which indemnity is provided, shall cease if the same item is kept in operation without being repaired to the
satisfaction of the Company.

SECTION – 5

www.ibpsguide.com | estore.ibpsguide.com | www.sscexamguide.com


Insurance Awareness Capsule

AGRICULTURAL TRACTORS INSURANCE


As per the provisions, terms, exceptions, conditions and endorsements as per standard Motor Policy.

Covers the insured against loss or damage to the Agriculture Tractor by fire, explosion, self-ignition or
lightning, burglary, housebreaking, theft, riot and strike, earthquake, fire and shock, inundation, typhoon,
hurricane, storm, tempest, cyclone, hailstorm, frost, landslide/rockslides by accidental external means,
malicious act, terrorism activity while in transit by road, rail, inland waterway. Also provides coverage against
death or permanent disablement of the driver, due to an accident while driving the Tractor insured during any
one policy period.

Legal Liability to Third Parties: - Compensates for death/ bodily injury to third parties up to in the event of
tractor being involved in an accident as per M.V. Act, 1988.

SECTION – 6

STUDENT SAFETY INSURANCE


Schedule of Benefits (for Parent/ Student): SI per student

1. Accidental death: Rs. 50000 (parent/student)

2. Permanent total disablement: Rs. 50000 (student)

3. Loss of one limb/Eye: Rs. 25000 (student)

4. Accidental hospitalization: Rs. 5000 (student)

In case of death of Father or Mother, the Claim amount to be converted into FD in the name of student till
attainment of adulthood.

Part 1

A) If at any time during the currency of this policy the parent / guardian/ student named in the schedule
shall sustain any bodily injury resulting solely and directly from accident caused by external violent and
visible means and if such injury shall within six calendar months of the occurrence be the sole and direct
cause of death or total and irrecoverable loss of two limbs or two eyes or 100%

www.ibpsguide.com | estore.ibpsguide.com | www.sscexamguide.com


Insurance Awareness Capsule

Permanent Total Disablement (permanently totally and absolutely disable the parent /guardian from
engaging in any employment or occupation of any description whatsoever) then the company shall pay to the
insured Student or parent / guardian as the case may be the capital sum insured stated in the schedule.

B) In addition to the above benefit mentioned under (A) (if liability is admitted under clause A above) the
student shall be reimbursed for the unexpired period of study, the tuition fee, development expenses,
boarding and lodging charges and other insured expenses (excluding payments made prior to accident and /
or overdue payments as on date of accident) on actual basis subject to the maximum limits as stated in the
schedule of benefit.

Part 2&3
If at any time during the currency of this policy the insured Student shall sustain any bodily injury resulting
solely and directly from accident caused by external violent and visible means and if such injury shall within
six calendar months of the occurrence be the sole and direct cause of death or total and irrecoverable loss of
two limbs or two eyes or 100% Permanent Total Disablement (permanently totally and absolutely disable the
insured student from engaging in any employment or occupation of any description whatsoever) then the
company shall pay to the parent / guardian or insured Student as the case may be the capital sum insured
stated in the schedule of benefits. In case of death of both student and the parent / guardian named in the
schedule of the policy resulting solely and directly from same accident caused by outward, violent and visible
means, within six calendar months of its occurrence then the company shall pay the legal heir of the parent /
guardian sums stated in the schedule.

Part 4
Subject to the terms, conditions & exclusions the Company undertakes that if during the period stated in the
Policy any insured student contracts any disease or suffers from any illness or sustains any bodily injury
through accident, and takes treatment at any Nursing Home/Hospital in India as an inpatient, the Company
will pay to the Insured Person such expenses as are reasonably and necessarily incurred subject to the limits
prescribed but not exceeding the Sum Insured in any one period of insurance stated against that person in
the policy.

SECTION – 7

LIFE INSURANCE

(as per Pradhan Mantri Jeevan Jyoti Bima Yojna (PMJJBY)


Benefits:

1. Death Cover : Rs. 2,00,000 per member

www.ibpsguide.com | estore.ibpsguide.com | www.sscexamguide.com


Insurance Awareness Capsule

2. To be provided by Life Insurance Companies

RULES OF THE SCHEME


a) In these Rules, the following words and expressions shall unless repugnant to the context, have the
following meanings:-

b) “THE SCHEME” shall mean „PRADHAN MANTRI JEEVAN JYOTI BIMA YOJANA‟ for the Savings Bank
Account Holders of „BANK‟.

c) “THE RULES" shall mean the Rules of the Scheme as set out below and as amended from time to time.

d) “THE MEMBER” shall mean a Savings Bank Account Holder who has been admitted to benefits of the
Scheme and on whose life an assurance has been or is to be effected in accordance with these Rules.

e) “TERMINAL DATE” shall mean in respect of each Member the Annual Renewal

Date following the date on which member completes the age of 55 or the member closes his account with
the Bank or discontinuance of premium payment whichever is earlier.

f) “THE ASSURANCE” shall mean the particular Assurance to be effected on the life of the Member.

g) “THE BENEFICIARY” shall mean the person or persons who has/have been appointed by the Member as
Nominee and whose name or names have been entered in the Bank Records.

ELIGIBILITY:-
The savings bank account holder of the participating banks aged between 18 years (completed) and 50 years
(age nearer birthday) and who have given the consent to join the scheme during the „enrollment period‟ are
eligible to join the scheme.

ADMISSION OF AGE:
Age as recorded by the Bank as per the Age Proof submitted by the Savings Bank Account holder.

EVIDENCE OF HEALTH:
Satisfactory evidence of health as required by the insurance company shall be furnished by every eligible
member, at the time of his entry into the Scheme, after the „Enrollment Period‟, as incorporated in the
“Consent-cum- Declaration Form” for joining the scheme.

PREMIUM:

www.ibpsguide.com | estore.ibpsguide.com | www.sscexamguide.com


Insurance Awareness Capsule

Premium to be deducted from member‟s SB Account. The premium is Rs.330/- plus Service Tax (if payable)
irrespective of date of entry i.e. during enrollment period or after that date during the first year. Renewal
premium is chargeable as per the rate decided from time to time on Annual Renewal dates.
ASSURANCE:

An assurance of Rs.2,00,000/- on death of the insured member is payable to the Nominee.


BENEFITS ON DEATH PRIOR TO TERMINAL DATE :

Upon the death of the Member prior to Terminal Date, the sum assured under the Assurance shall be
payable to the nominated Beneficiary, provided the assurance is kept in force by payment of premium for
that member.

TERMINATION OF ASSURANCE:
The Assurance on the life of a Member shall terminate on an Annual Renewal Date upon happening of any of
the following events and no benefit will become payable thereunder:-On attaining age 55 years (age nearest
birthday) on annual renewal date.

Closure of account with the Bank or insufficiency of balance to keep the insurance in force.

SUSPENSION OF RISK:
If the insurance cover is ceased due to any technical reasons such as insufficient balance for payment of
premium on due date of renewal, the same can be reinstated after the grace period on receipt of premium
and a satisfactory statement of good health.

RESTRAINT ON ANTICIPATION OR ENCUMBRANCE :


The benefits assured under the Scheme are strictly personal and cannot be assigned, charged or alienated in
any way.

DISCONTINUANCE OR AMENDMENT OF THE SCHEME:


The “Bank” or Insurance Company reserves the right to discontinue the Scheme at any time or to amend the
Rules thereof on any Annual Renewal Date subject to giving one month‟s notice. Any amendment to the
Rules of the Scheme will be done based on mutual agreement between Insurance Company and “Bank”.

JURISDICTION:
All Assurances issued under the Scheme shall be Indian Contracts. They will be subject to Indian Laws
including the Insurance Act, 1938 as amended, the Income Tax Act, 1961 and to any legislation subsequently
introduced. All benefits under the Scheme arising out of death of any Member shall be payable in Indian
Rupees.

www.ibpsguide.com | estore.ibpsguide.com | www.sscexamguide.com


Insurance Awareness Capsule

GENERAL CONDITIONS AND DEFINITIONS FOR ALL SECTIONS

Notice:
Every notice and communication to the Company required by this Policy shall be in writing to the Policy
issuing Office of the Company.

Misdescription:
This Policy shall be void and all premium paid hereon shall be forfeited to the Company in the event of
misrepresentation, misdescription or non-disclosure of any material particular.

Reasonable Care:
The Insured shall take all reasonable steps to safeguard the property insured against any loss or damage.
The Insured shall exercise reasonable care that only competent employees are employed and shall take all
reasonable precautions to prevent all accidents and shall comply with all statutory or other regulations.

Cancellation: (Except Sections-1,3&8- PMFBY/WBCIS, PMSBY & PMJJBY)


The Policy may be terminated at the request of the Insured in which case the Insurance Company will retain
the premium for the period this Policy has been in force at the short period scales of rates, subject to the
retention of minimum premium of Rs. 100 by the Insurer. No refund of premium shall be made if a claim is
reported in the policy.

The details of the cancellation procedure are as per guidelines.

Claims Procedure:
i) The Insured shall upon the occurrence of any event giving rise or likely to give rise to a claim under this
Policy:

a) In the event of theft lodge forthwith a complaint with the Police and shall take all practicable steps to
apprehend the guilty person or persons and to recover the property lost.

b) Give immediate notice thereof to the Company and shall within Fourteen (14) days thereafter furnish
to the Company at his own expense detailed particulars of the amount of the loss or damage, together with
such explanation and evidence to substantiate the claim as the Company may reasonably require.

ii) If the Insured or member of the Insured‟s family comprising the Insured‟s spouse and children shall die,
notice of death shall be given by the legal representative(s) forthwith. All certificates information and
evidence whether from a Medical Attendant or otherwise required by the Company shall be furnished at the
expense of the Insured or his legal representatives and shall be in such form and of such nature as the
Company may prescribe..

www.ibpsguide.com | estore.ibpsguide.com | www.sscexamguide.com


Insurance Awareness Capsule

iii) The Insured shall upon the occurrence of any event giving rise or likely to give rise to a claim under the
Policy give immediate notice thereof to the Company and shall forward to the Company forthwith every
written notice or information of any verbal notice of claim and shall send to the Company any writ, summons
or other legal process issued or commenced against the Insured and shall give all necessary information and
assistance to enable the Company to settle or resist any claim or to institute proceedings. The Insured shall
not incur any expenses in making good any claim without the prior consent of the Company and shall not
negotiate, pay, settle, admit or repudiate any claim without such consent.

Contribution:
In the event of any loss damage liability or expenses covered by this Policy there shall be any other insurance
covering the same loss damage liability or expenses, whether effected by the Insured or not this Policy shall
pay only so much of the excess of such loss damage liability or expenses as is not recoverable under such
other insurance subject always to the limitations of this Policy.

Fraud:
If any claim under this Policy shall in any respect be fraudulent or if any fraudulent means or devices are
used by the Insured or any one acting on the Insured‟s behalf to obtain any benefit under this Policy, all
benefits under the Policy shall be forfeited.

Indemnity:
The Company may at its option reinstate/replace or repair the property or premises lost or damaged or any
part thereof instead of paying the amount of the loss or damage or may join with any other insurer in so
doing but the Company shall not be bound to reinstate exactly or completely but only as circumstances
permit and in reasonably sufficient manner and in no case shall the Company be bound to expend more in
reinstatement than it would have cost to reinstate such property as it was at the time of occurrence of such
loss or damage and not more than the Sum Insured by the Company thereon.

Average:
If the property hereby insured shall at the time of any loss or damage be collectively of greater value than the
Sum Insured thereon, then the Insured shall be considered as being his own insurer for the difference, and
shall bear a ratable proportion of the loss or damage accordingly. Every item insured, if more than one, of the
Policy, shall be separately subject to this condition.

Grievance Redressal Mechanism


To address any grievance/ complaint under the scheme, a committee consisting of District Agriculture
Officer (Nodal Officer), representatives of implementing insurance company and bank/ financial institution
shall be constituted. The working of the grievance committee will be monitored by District Level Monitoring
Committee (DLMC) under the chairmanship of District Magistrate.

www.ibpsguide.com | estore.ibpsguide.com | www.sscexamguide.com


Insurance Awareness Capsule

Observance of Terms and Conditions:

The due observance and fulfillment of the terms, conditions and endorsement of this Policy in so far as they
relate to anything to be done or complied with by the Insured shall be a condition precedent to any liability of
the Company to make any payment under this Policy.

GENERAL EXCEPTIONS

The Company shall not be liable in respect of:

1. Loss or damage, liability or expenses whether directly or indirectly, occasioned by happening through or
arising from any consequences of war, invasion, act of foreign enemy, hostilities (whether war be declared or
not) civil war, rebellion, revolution, insurrection, mutiny, military, or usurped power or civil commotion or loot
or pillage in connection herewith.

2. Loss or damage caused by depreciation or wear and tear

3. Consequential loss of any kind or description.

4. a) Loss or damage directly or indirectly caused by or arising from or in consequence of or contributed to


by nuclear weapons material.

b) This Insurance does not cover loss or damage directly or indirectly caused by or arising from or in
consequence of or contributed to by ionising radiation or contamination by radioactivity from any nuclear fuel
or from any nuclear waste from the combustion of nuclear fuel. For the purpose of Condition 4 (b) only
combustion shall include any self-sustaining process of nuclear fission.

Arogya Raksha Yojana


Plan
Arogya Raksha Yojana is a comprehensive health insurance plan that offers people of rural India affordable
access to high quality healthcare, provided by a network of renowned hospitals and clinics, supported by
leading doctors and surgeons. The plan aims to better the quality of life for rural India through a number of
healthcare initiatives thereby reducing the severity of poverty resulting from health related catastrophes.
Arogya Raksha Yojana believes in the collective power of the masses to provide for expensive healthcare for
themselves and thus in time aims to be a self-sustaining model that inspires confidence and can be
duplicated in other areas in the country. The plan aims to provide the member with quality medical facilities
at affordable costs (This facility can be availed for as little as Rs.502 to Rs. 594 ($11.00) per annum, per
individual).

www.ibpsguide.com | estore.ibpsguide.com | www.sscexamguide.com


Insurance Awareness Capsule

Success in the long-term battle against medical afflictions of developing nations depends less on targeted &
controlled distribution of medicine and more on education and efficient infrastructure. It is therefore critical
to create awareness on health & hygiene and provide educational resources by leveraging on information
technology. Prevention in these regions is often much more important and more sustainable than
undertaking the usually cumbersome life-long treatment of a disease that has already declared itself. It is
often the simplest essentials that are lacking in order to break out of the vicious circle of infection and
ignorance.
At Arogya Raksha Yojana, we have deliberately focused our efforts on the most needy groups in the least
developed areas of rural India where the necessity is greatest and where in most cases, access to
fundamentals like food, drinking water and the most basic form of medical care cannot be guaranteed.
Benefits
SURGICAL COVER
1. Over 850 surgeries covered – this is a cashless facility
2. Treatment to be done ONLY at the listed Network Hospitals (Annexure 1)
3. Treatment only in GENERAL WARD

SURGICAL COVER – DETAILS OF COVER


The surgical cover includes the following:
1. Admission Charges
2. Anesthesia Charges
3. Surgeon’s Fee
4. Nursing Charges
5. OT charges
6. Bed charges
7. Medicines & Consumables during surgery.

COMMON SURGERIES THAT ARE COVERED


1. Appendectomy
2. Hysterectomy
3. Normal delivery
4. LSCS (Caesarian Section )
5. Hernia
6. ENT surgeries
7. Coronary Angiogram
8. Bypass surgeries
9. Closed Heart Surgeries
10. Other cardiac procedures
11. Removal of Tumors (minor)
12. Gastrectomy

www.ibpsguide.com | estore.ibpsguide.com | www.sscexamguide.com


Insurance Awareness Capsule

13. Cholesystectomy
14. Arthrotomy
15. Fracture of long Bones
16. Uretroscopic stone removal
17. Nephrolithotomy
18. TURP
19. Fissurectomy
20. Thyroidectomy
21. Tonsillectomy
22. Mastoidectomy
23. Tympanoplasty Commando Surgery
24. Varicose veins
25. Cataracts
These are the most common surgeries. ARY covers more than 1600 surgical procedures.

MEDICAL COVER
1. Medical Hospitalization refers to hospitalization of a patient that DOES NOT LEAD TO SURGERY
2. Maximum of 3 days PER ADMISSION & PER CARD amounting to a maximum of Rs. 650.
3. Hospitalization MUST be for a minimum of 24 hours.
4. Treatment and investigations to be done ONLY in Network Hospitals.
5. Treatment ONLY in the General Ward.

MEDICAL COVER – DETAILS


1. Room charges
2. Professional Fees
3. Nursing Care

MEDICAL COVER – COMMON ILLNESSES


1. Viral Fever
2. Diarrhea and Dysentery
3. Dehydration
4. Typhoid

ADDITIONAL BENEFITS
ARY members can access good primary healthcare treatment at the ARY clinics:
1. Free medical consultation
2. Subsidized generic medicines
3. Subsidized diagnostic investigations

www.ibpsguide.com | estore.ibpsguide.com | www.sscexamguide.com


Insurance Awareness Capsule

4. Nine clinics – Huskur & Hennagara in Anekal Taluk; Austin Town, K.R. Puram & Kalkunte in Bangalore
City; Mandya, Chikkballapur & Bagalkot
Premium:
The ARY Micro Health Insurance Scheme is a critical component of our healthcare services – it protects
poorer people against the debilitating effects of catastrophic illnesses. People who enroll with the insurance
program can avail of a complete cashless service for 845+ surgeries and subsidized service for
hospitalization that does not lead to surgery.
Details of the scheme given below:
PREMIUM:
No. of
Rs. Per life/year Total amount
people/family
1 410 410
2 410 820
3 341 1025
4 275 1100
5 275 1375
6 275 1650
7 275 1925

NATIONAL AGRICULTURAL INSURANCE SCHEME (NAIS)


- A NOVEL CROP INSURANCE SCHEME

Q 1 : What is insurance ?
Ans: Insurance is a fiancial arrangement whereby losses suffered by a few are met from the funds
accumulated through small contributions made by many who are exposed to similar risks.

Q 2 : What is crop insurance ?


Ans: Crop insurance is an insurance arrangement aiming at mitigating the financial losses suffered by the
farmers due to damage and destruction of their crops as a result of various production risks.

Q 3 : Was there any crop insurance scheme in India prior to NAIS ?


Ans: Yes. From 1972-73 to 1978-79, crop insurance schemes for crops such as cotton, groundnut, potato etc,
was implemented in selected places on "individual approach" basis. During the period from 1979 to 1984-85,
a pilot crop insurance scheme was implemented for Food crops & Oilseeds on "Area approach" basis. Based
on the experience of the pilot scheme, a Comprehensive Crop Insurance Scheme (CCIS) was implemented
from Kharif 1985 till Kharif 1999. The present crop insurance scheme, i.e., National Agricultural Insurance
Scheme (NAIS), launched by the Hon'ble Prime Minister on 22nd June 1999 replaced the CCIS from Rabi
1999-2000 season.

www.ibpsguide.com | estore.ibpsguide.com | www.sscexamguide.com


Insurance Awareness Capsule

Q 4 : What are the objectives of National Agricultural Insurance Scheme ?


Ans: The objectives of the NAIS are as under:-

1. To provide insurance coverage and financial support to the farmers in the event of failure of any of the
notified crop as a result of natural calamities, pests & diseases so as to restore their credit worthiness
for ensuing season.

2. To encourage the fanners to adopt progressive farming practices, high value in-puts and higher
technology in Agriculture.

3. To help stabilize farm incomes, particularly in disaster years.

Q 5 : What are the various risks covered under the scheme?


Ans: The Scheme provides comprehensive risk insurance for yield losses due to :

(i) Natural Fire and Lightening, Storm, Hailstorm, Cyclone, Typhoon, Tempest, Hurricane, Tornado Flood,
Inundation and Landslide.

(ii) Drought, Dry spells.

(iii) Pests / Diseases etc.

Q 6 : How many states are participating in the scheme?


Ans.: At present this scheme is being implemented in 23 States and UTs (except Punjab, Manipur, Nagaland,
Mizoram and Arunachal Pradesh among the States and Chandigarh, Daman & Diu, Delhi, Dadra & Nagar
Haveli and Lakshadweep among the UTs).

Q 7: What are the various crops covered under the scheme?


Ans.: The Scheme covers following crops

• Food crops (Cereals, Millets & Pulses): Some of the crops covered in various States are Paddy, Wheat,
Jowar, Bajra, Maize, Ragi, Korra, Kodo-kutki, Green gram, Black gram, Red gram, Horse gram, Gram,
Moth etc.

• Oilseeds: Some of the crops covered in various States are Groundnut, Sunflower, Soya bean,
Safflower, Castor, Sesamum, Niger etc.

www.ibpsguide.com | estore.ibpsguide.com | www.sscexamguide.com


Insurance Awareness Capsule

• Annual Commercial/Annual Horticultural crops : Sugarcane, Cotton , Potato, Onion, Ginger, Turmeric,
Banana, Pineapple, Jute, Tapioca, Chilli, Cumin, Coriander, Isabgol, Methi etc.

The crops are selected for insurance if the past yield data for 10 years are available, and the State Govt
agrees to conduct requisite number of Crop Cutting Experiments (CCEs) during the proposed season.

At present there are 35 different crops during Kharif and 30 different Rabi season are being insured under
National Agricultural Insurance Scheme in the country.

Q 8 : Who is eligible to be covered under the scheme? .


Ans: All farmers growing insurable crops and availing Seasonal Agricultural Operations (SAO) loans from
Banks / PACS are compulsorily covered under the Scheme by the Banks/ PACS, whereas the non-borrowing
farmers growing insurable crops can also avail the benefit of the Scheme by submitting prescribed proposal
forms at the nearest Banks/ PACS.

Q 9 : How is this scheme administered'?


Ans.: The Scheme is being implemented by Agriculture Insurance Company of India Limited (AICL) on behalf
of the Ministry of Agriculture, through its Regional Offices located at 17 State capitals.

Q 10 : What is the Unit of Insurance?


Ans : The scheme operates on the basis of Area Approach i.e. defined areas for each notified crop for
widespread calamities and individual assessment is done on experimental basis for localised calamities, such
as, hailstorm, landslide, cyclone and flood in certain pre-notified areas. The size of unit area varies from
State to State and crop to crop. Presently, the defined area is Block/ Mandal/ Taluka / Patwari halka /
Nyayapanchayat/ Gram Panchayat/ Village, etc.

Q 11 : What' s amount of sum insured under NAIS?


Ans : a) For loanee farmers:

Compulsory coverage : The amount of crop loan availed for the notified crop is the minimum amount of sum
insured covered on compulsory basis.

Optional Coverage : If the loanee-farmer so wishes he may insured his crop for a higher Sum Insured i.e, upto
the value of Threshold Yield (i.e., guaranteed yield) which is called normal coverge even go for addtional
coverage upto 150% value of average yield in the notified area. However, for additional coverage, the farmer
has to pay premium at actuarial rate as notified by the State Government.

www.ibpsguide.com | estore.ibpsguide.com | www.sscexamguide.com


Insurance Awareness Capsule

The value of Sum Insured in such cases is arrived at by multiplying the threshold yield/ average yield with the
latest available Minimum Support Price (MSP) announced by the Govt. or the market price provided by the
State Govt. in case the MSP is not announced.

b. For Non-loanee farmers:

Coverage at normal rates of premium is available upto the value of Threshold Yield . Additional coverage upto
150% of the value of Actual Yield can be obtained by payment of premium at actuarial rates.

Note : It may however be noted that in case of annual commercial and horticultural crops, only actuarial rates
of premium are charged at all the slabs of sum insured.

Q 12 : How much premium a farmer has to pay to get the insurance cover ?
Ans :

• For Kharif crops Premium is 3.5 % of Sum insured for all Oilseed crops and Bajra and 2.5% for all
others foodcrops including pulses

• For Rabi crops Premium rates are 1.5% for wheat and 2% for all other foodcrops including pulses and
oilseeds.

• However, of the above flat rates and the actuarial rate whichever is lower shall apply.

• For Annual Cmmercial and Horticultural crops atuarial premium rates are charged.

• The premium for Small & Marginal farmers is subsidised to the extent of 10% which is shared by the
State Govt. and Govt. of India.

Q13 : Does a non-loanee farmer need to produce some documents for obtaining insurance cover ?
Ans : The non-loanee farmer has to produce a proof (copy of land pass book / 7/12 extract / land revenue
receipt etc.) to prove that he / she is the owner of the land. In case of sharecroppers and tenant farmers
(who are not availing crop loan), a proof showing crop sharing / tenancy arrangement need to be produced.

Q 14 : How can a farmer get insurance coverage ?


Ans : All loanee farmers automatically and get compulsorily covered under this Insurance Scheme through
the PACS/Bank branches, as and when crop loans for insured crops are disbursed to them. All non-loanee
farmers desirous of availing insurance coverage should contact nearest Bank Branches before the stipulated

www.ibpsguide.com | estore.ibpsguide.com | www.sscexamguide.com


Insurance Awareness Capsule

time frame with a proposal for insurance. Non-loanee farmer should have a Bank A/c with the bank and pay
the requisite premium to get insurance coverage.

Q 15 : Are crop loans disbursed through Kisan Credit Cards (KCCs) eligible for insurance coverage
under the NAIS ?
Ans : Crop loans disbursed/withdrawn through KCCs are also eligible for coverage as per the terms and
conditions applicable to the regular crop loans.

Q 16 : When and how, the claims are settled under NAIS ?


Ans : In case of widespread calamities leading to damage and loss of crops, claims are settled on area
approach basis. Any insured crop in a notified area recording lower actual yield than the guaranteed yield as
per the crop estimation surveys conducted by the state govt., shall automatically become eligible for
compensation/ claim. The shortfall in yield is determined for each crop and is the difference between the
guaranteed yield and the current season's actual yield. Shortfall % is determined by expressing the shortfall
as a proportion of guaranteed Yield. Claim is then computed by multiplying the sum insured with the shortfall
%. Therefore no claims would be admissible/payable in case the current season's actual yield is more than
the threshold yield. Applicable amounts of claims so arrived at are routed to the farmers through the banks in
case of those farmers who are covered through the banks.

Even in case of non-loanee farmers who approach AIC directly for insurance coverage, the claim amounts are
paid by way of cheques or through designated bank branches such that claim amounts get credited to their
accounts. The claim assessment and payment would be done after receipt of the requisite yield data from
the concerned agencies.

The methodology of claim assessment based on individual approach shall be intimated to all concerned upon
finalising the areas and modalities.

Q 17 : Whether annawari or any similar declaration/ certification by the revenue or agriculture


departments of the state govt. at village/block /district level has any bearing on claim settlement ?
Ans : No, claims under the scheme are settled strictly as per the provisions and guidelines described in the
foregoing paragraphs and not according to declaration of flood/drought/annavari by any other
agency/authority.

Q 18 : Whether insured farmers whose crops are damaged need to intimate crop losses to bank/AIC to
be eligible for claim ?
Ans : In case of "Area approach", the farmers need not intimate crop losses to bank/AIC. The crop losses if
any or reduction in yield as compared to Guaranteed yield shall be reflected in Crop Estimation surveys and
the shortfall in yield, if any shall be paid as claim through the Bank. In case of areas notified for

www.ibpsguide.com | estore.ibpsguide.com | www.sscexamguide.com


Insurance Awareness Capsule

experimentation of individual loss assessment, the farmer need to intimate the crop loss within 48 hours with
the local revenue/ agriculture dept.

Q 19 : What are the essential requirements before a farmer becomes eligible for claim under the
Scheme ?
Ans : The essential requirements are :

• The farmer should have availed a crop loan for the insured crop. In case of non-loanee farmer, he
should have submitted a proposal for insurance with requisite premium.

• The proposal/crop insurance Declaration with accurate and complete particulars should have been
sent to AIC by the Bank along with requisite premium.

• The State Govt. conducts requisite number of Crop Cutting Experiments for the insured crop in the
insurance unit and submits the yield data to AIC within the prescribed date.

• The yield data so submitted by the State Govt. shows short fall as compared to the Guaranteed yield.

Q 20 :What are the benefits expected from the Scheme:


Ans : This Scheme is expected to

• Be a critical instrument of development in the field of crop production, providing financial support to
the farmers in the event of crop failure.

• Encourage farmers to adopt progressive farming practices and higher technology in Agriculture.

• Help in maintaining flow of agricultural credit.

• Provides significant benefits not merely to the insured farmers, but, to the entire community directly
and indirectly through spillover and multiplier effects in terms of maintaining production &
employment.

• Streamline loss assessment procedures and help in building up huge and accurate statistical base for
crop production.

Functions of Ombudsman

Brief note on the functioning of the offices of the Insurance Ombudsman

www.ibpsguide.com | estore.ibpsguide.com | www.sscexamguide.com


Insurance Awareness Capsule

The institution of Insurance Ombudsman was created by a Government of India Notification dated 11th
November, 1998 with the purpose of quick disposal of the grievances of the insured customers and to
mitigate their problems involved in redressal of those grievances. This institution is of great importance and
relevance for the protection of interests of policy holders and also in building their confidence in the system.
The institution has helped to generate and sustain the faith and confidence amongst the consumers and
insurers.

Appointment of Insurance Ombudsman


The governing body of insurance council issues orders of appointment of the insurance Ombudsman on the
recommendations of the committee comprising of Chairman, IRDA, Chairman, LIC, Chairman, GIC and a
representative of the Central Government. Insurance council comprises of members of the Life Insurance
council and general insurance council formed under Section 40 C of the Insurance Act, 1938. The governing
body of insurance council consists of representatives of insurance companies.

Eligibility
Ombudsman are drawn from Insurance Industry, Civil Services and Judicial Services.
Terms of office
An insurance Ombudsman is appointed for a term of three years or till the incumbent attains the age of sixty
five years, whichever is earlier. Re-appointment is not permitted..
Territorial jurisdiction of Ombudsman
he governing body has appointed twelve Ombudsman across the country allotting them different
geographical areas as their areas of jurisdiction. The Ombudsman may hold sitting at various places within
their area of jurisdiction in order to expedite disposal of complaints. The offices of the twelve insurance
Ombudsmans are located at (1) Bhopal, (2) Bhubaneswar, (3) Cochin, (4) Guwahati, (5) Chandigarh, (6) New
Delhi, (7) Chennai, (8) Kolkata, (9) Ahmedabad, (10) Lucknow, (11) Mumbai, (12) Hyderabad. The areas of
jurisdiction of each Ombudsman has been mentioned in the list of Ombudsman.

Office Management
The Ombudsman has a secretarial staff provided to him by the insurance council to assist him in discharging
his duties. The total expenses on Ombudsman and his staff are incurred by the insurance companies who are
members of the insurance council in such proportion as may be decided by the governing body.
Removal from office
An Ombudsman may be removed from service for gross misconduct committed by him during his term of
office. The governing body may appoint such person as it thinks fit to conduct enquiry in relation to
misconduct of the Ombudsman. All enquiries on misconduct will be sent to Insurance Regulatory and
Development Authority which may take a decision as to the proposed action to be taken against the
Ombudsman. On recommendations of the IRDA, the Governing Body may terminate his services, in case he is
found guilty.

www.ibpsguide.com | estore.ibpsguide.com | www.sscexamguide.com


Insurance Awareness Capsule

Power of Ombudsman
Insurance Ombudsman has two types of functions to perform (1) conciliation, (2) Award making. The
insurance Ombudsman is empowered to receive and consider complaints in respect of personal lines of
insurance from any person who has any grievance against an insurer. The complaint may relate to any
grievance against the insurer i.e. (a) any partial or total repudiation of claims by the insurance companies, (b)
dispute with regard to premium paid or payable in terms of the policy, (c) dispute on the legal construction of
the policy wordings in case such dispute relates to claims; (d) delay in settlement of claims and (e) non-
issuance of any insurance document to customers after receipt of premium.
Ombudsman's powers are restricted to insurance contracts of value not exceeding Rs. 20 lakhs. The
insurance companies are required to honour the awards passed by an Insurance Ombudsman within three
months.

Manner of lodging complaint


The complaint by an aggrieved person has to be in writing, and addressed to the insurance Ombudsman of
the jurisdiction under which the office of the insurer falls. The complaint can also be lodged through the legal
heirs of the insured. Before lodging a complaint:
i) the complainant should have made a representation to the insurer named in the complaint and the insurer
either should have rejected the complaint or the complainant have not received any reply within a period of
one month after the concerned insurer has received his complaint or he is not satisfied with the reply of the
insurer.
ii) The complaint is not made later than one year after the insurer had replied.
iii) The same complaint on the subject should not be pending with before any court, consumer forum or
arbitrator.

Recommendations of the Ombudsman


When a complaint is settled through the mediation of the Ombudsman, he shall make the recommendations
which he thinks fair in the circumstances of the case. Such a recommendation shall be made not later than
one month and copies of the same sent to complainant and the insurance company concerned. If the
complainant accepts recommendations, he will send a communication in writing within 15 days of the date of
receipt accepting the settlement.

Award
The ombudsman shall pass an award within a period of three months from the receipt of the complaint. The
awards are binding upon the insurance companies.
If the policy holder is not satisfied with the award of the Ombudsman he can approach other venues like
Consumer Forums and Courts of law for redressal of his grievances.
As per the policy-holder's protection regulations, every insurer shall inform the policy holder along with the
policy document in respect of the insurance Ombudsman in whose jurisdiction his office falls for the purpose
of grievances redressal arising if any subsequently. Steady increase in number of complaints received by

www.ibpsguide.com | estore.ibpsguide.com | www.sscexamguide.com


Insurance Awareness Capsule

various Ombudsman shows that the policy-holders are reposing their confidence in the institution of
Insurance Ombudsman.

Important Points to Remember

• 1818 saw the advent of life insurance business in India with the establishment of the Oriental Life
Insurance Company in Calcutta. This Company however failed in 1834.
• An Ordinance was issued on 19 Jan, 1956 nationalising the Life Insurance sector and Life Insurance
Corporation came into existence in the same year. The LIC absorbed 245 Indian and foreign insurers
in all.
• General Insurance in India has its roots in the establishment of Triton Insurance Company Ltd in the
year 1850 in Calcutta by the British. Established in 1907, the Indian Mercantile Insurance was the first
company to transact all classes of general insurance Business
• Formed in 1957, the General Insurance Council framed a code of conduct for ensuring fair conduct
and sound business practices.
• The Export Credit Guarantee Corporation of India (ECGC) was established on 30 July 1957 with an
objective to provide insurance cover in respect of risks in export trade.
• With the passing of the General Insurance Business (Nationalisation) Act, general
• insurance business was nationalized with effect from 1 Jan, 1973, 107 insurers were
• amalgamated and grouped into four companies: National Insurance Company, the New India
Assurance Company, the Oriental Insurance Company and the United India Insurance Company.
• The General Insurance Corporation (GIC) of India was incorporated as a company in 1971 and it
commenced business on 1 Jan 1973.
• In 1933, the Government set up a committee under the chairmanship of RN Malhotra, former Governor
of RBI, to propose recommendations for reforms in the insurance sector, which, among other things,
recommended that the private sector be permitted to enter the insurance industry.
• Following the recommendations of the Malhotra Committee report, in 1999, the Insurance Regulatory
and Development Authority (IRDA) was constituted as an autonomous body to regulate and develop
the insurance industry.
• The IRDA was incorporated as a statutory body in April 2000.
• The IRDA opened up the market in August 2000 and foreign companies were allowed ownership of up
to 26 per cent.
• The Authority has the power to frame regulations under Sections 114A of the Insurance Act, 1938 and
has from 2000 onwards framed various regulations ranging from registration of companies for carrying
on insurance business to protection of policyholder’s interests.
• In December 2000, the subsidiaries of the General Insurance Corporation of India were restructured
as independent companies and at the same time GIC was converted into a national re-insurer.
Parliament passed a bill de-linking the four subsidiaries from GIC in July, 2002.

www.ibpsguide.com | estore.ibpsguide.com | www.sscexamguide.com


Insurance Awareness Capsule

• there are 28 general Insurance companies, including the ECGC and Agriculture Insurance Corporation
of India and 24 life insurance companies operating in the country.
• The insurance sector is growing at a speedy rate of 15-20 per cent.
• Together with banking services, insurance services add about 7 per cent to the country’s GDP.
• The total business of all four public sector non-life insurance companies put together, which was at
Rs. 42,000 cr in 2013-14 is expected to cross Rs. 1 lakh cr by 2020.

www.ibpsguide.com | estore.ibpsguide.com | www.sscexamguide.com


www.ibpsguide.com

BANKING AWARENESS

CONTENT PAGE NO.


History Of Banking In India 2
Reserve Bank Of India 5
Nationalization Of Banks In India 13
Indian Banking Structure 15
All India Financial Institutions 19
Security Exchange Board of India & Insurance Regulatory and 25
Development Authority
Types Of Account 28
Non Banking Financial Companies (NBFC) 33
Credit Rating Agencies 36
Foreign Exchange Reserve 39
Financial Market 40
Small Finance Bank And Payments Bank 49
Payment And Settlement Systems Act, 2007 51
Financial Inclusion In India 73
Different Codes Used In Banking 76
Inflation 78
Priority Sector Lending 80
Basel Norms 84
Deposit Insurance And Credit Guarantee Corporation 88
Banking Ombudsman In India 89
Know Your Customer 90
Prompt Corrective Action 92
Goods And Services Tax 94
Great SBI Merger 100
Foreign Direct Investment 102
Miscellaneous 106
Bank Headquarters And Tagline 116
Banking Related Acts 123
Committees 126
First Among All Banks 132
Important Banking Abbreviations 133

www.ibpsguise,in | mock.ibpsguide.com | www.sscexamguide.com


Copyright © IBPS Guide 1
www.ibpsguide.com

HISTORY OF BANKING IN INDIA

The brief history of Indian banking system given below

YEAR EVENT

1770 Bank of Hindustan was established by British Government

1806 Presidency Bank of Bengal was established

1840 Presidency Bank of Bombay was established

1843 Presidency Bank of Madras was established

1865 Allahabad Bank was established. It‘s Oldest Joint Stock bank in India.

Oudh Commercial Bank was established in 1881 in Faizabad. At the time, it was the first
1881 commercial bank in India having limited liability and an entirely Indian board of directors.
It failed in 1958
Punjab National Bank (PNB) was established in Lahore by Indian Merchants – 1st Indian
1894
effort to continue till present day.

1904 Concept of Co-op Banks introduced by Lord Curzon

Swadeshi Movement (A number of banks established then have survived to the present such
1906 - 1911 as Catholic Syrian Bank, The South Indian Bank, Bank of India, Corporation Bank, Indian
Bank, Bank of Baroda, Canara Bank and Central Bank of India)
Central Bank of India first Indian commercial bank which was wholly managed by Indians,
1911
was established. It was called first truly Swadeshi Bank.

1913 - 1917 Due to World War 1 and 2 atleast 94 Banks in India were failed.

All the three Presidency Banks merged by British Government and named it as Imperial
1921
Bank Of India.

1926 Hilton Young Committee‘s Report on Central Bank

1927 Hilton Young Committee‘s Report introduced in Legislative Assembly and rejected.

1933 Central Banking Investigation Committee‘s report passed

1934 Reserve Bank Of India Act, 1934 was introduced on the basis of the 1933‘s report.

1935 RBI established with Headquarter in Calcutta.

www.ibpsguise,in | mock.ibpsguide.com | www.sscexamguide.com


Copyright © IBPS Guide 2
www.ibpsguide.com

1937 HQ of RBI permanently moved to Bombay.

First time demonetisation happened and banknotes of 1000 and 10000 rupee were
1946
withdrawn.
RBI was nationalised on 1st January 1949 (Governor is C.D. Deshmukh)
1949
Also, Banking Regulation Act, 1949 was introduced.
A.D. Gorwala Committee recommends renaming Imperial Bank of India as State Bank Of
1952
India.

1955 Imperial Bank of India was renamed as ‗State Bank of India‘ on 1st July, 1955.

1964 IDBI formed as wholly owned subsidiary of RBI by Act of Parliament

1969 14 Banks were nationalised on 19 July, 1969 with authorised capital over Rs 50 crores.

1971 Deposit Insurance and Credit Guarantee Corporation (DICGC) was established.

Regional Rural Banks (RRB) was formed for financial inclusion and development of rural
1975
economy.
Second time Government of India announced demonetisation and banknotes of 1000, 5000
1978
and 10000 rupee were abolished.

1980 6 banks were nationalised with authorised capital more than Rs. 200 crores.

National Bank for Agriculture and Rural Development (NABARD) was established for
1982
credit to agriculture and cooperative sector.

1987 HSBC Bank introduced ATM in Mumbai.

1988 Securities and Exchange Board of India (SEBI) was formed.

1990 Small Industries Development Bank of India (SIDBI) was established.

1992 National Stock Exchange (NSE) of India Limited was established

1. New Bank of India and PNB merged on 4 September, 1993. Total of nationalised
banks become 19 excluding SBI and its associates.
1993
2. Banking Regulation Act was amended and gates for new Private Sector Banks were
opened.

1994 ICICI Bank was established

1995 Banking Ombudsman scheme was launched

www.ibpsguise,in | mock.ibpsguide.com | www.sscexamguide.com


Copyright © IBPS Guide 3
www.ibpsguide.com

1994-96 Core Banking System started making its way into banks and branches.

1998-99 Kisan Credit Card introduced by RBI with NABARD.

1. Insurance Regulatory and Development Authority (IRDA) was established.


1999
2. Basel 1 norm was introduced in Indian Banking System.
TransUnion CIBIL Limited (Credit Information Bureau (India) Limited) is India‘s first
2000
Credit Information Company (CIC) founded in August 2000.
SARFAESI Act (Securitisation and Reconstruction of Assets and Enforcement of Security
2002
Interest 2002 Act) was introduced.

2003 Pension Fund Regulatory and Development Authority (PFRDA) was formed

2004 IDBI enters banking sector.

2005 Banking Ombudsman formed

2006 IDBI merges with IDBI Bank and IDBI acquires United Western Bank.

CTS (Cheque Truncation System) first implemented in NCR, New Delhi – deadline was 31
2007
December 2013.
1. IDBI Nationalised. Now total nationalised banks is 20 excluding SBI and its
associates.
2. SBI merged with State Bank of Saurashtra (1st associate bank merged with SBI) on
2008
13 August, 2008.
3. National Payments Corporation of India (NPCI) was formed.

1. ICICI acquires Bank of Rajasthan.


2010 2. SBI merged with State Bank of Indore. Its 2nd associate bank merger happened on
26 August, 2010.

2011 ‗Swabhiman Campaign‘ under Financial Inclusion launched.

2012 ‗RuPay‘ launched by National Payments Corporation of India.

Bhartiya Mahila Bank formed on 19 November, 2013.


2013
Total nationalised banks is 21 + SBI and its 5 associates = 27.
1. ‗RuPay‘ dedicated to India and formally introduced, by President Pranab
2014 Mukherjee. It is India‘s 1st indigenous/domestic payment card scheme.
2. ‗Jan Dhan Yojana‘ for Financial Inclusion launched.

www.ibpsguise,in | mock.ibpsguide.com | www.sscexamguide.com


Copyright © IBPS Guide 4
www.ibpsguide.com
3. Two new banking license issued by RBI to Bandhan Financial Services and IDFC.
4. RBI announced recall of pre-2005 currency notes.
1. Small Finance Bank and Payment Banks were introduced
2016 2. Government of India announced the Demonetization of all 500 and 1000 rupee notes
and new 500 and 2000 rupee notes were introduced.

2017 SBI merged with its five Associate Banks and Bharitiya Mahila Bank.

2019 Proposed Basel III implementation

RESERVE BANK OF INDIA

The Reserve Bank of India (RBI) was established via RBI Act 1934 on recommendation of the Hilton-young
commission or the Royal commission on India currency and Finance on April 1, 1935 with headquarter in
Kolkata. But it was shifted to Mumbai in 1937. The RBI was nationalized on 1 January, 1949.

Organization and Management:

Organization Structure: Central Board of Directors

www.ibpsguise,in | mock.ibpsguide.com | www.sscexamguide.com


Copyright © IBPS Guide 5
www.ibpsguide.com
The bank is headed by the Governor and the post is currently held by economist Urjit Patel. There are 4 Deputy
Governors B P Kanungo, S S Mundra, N S Vishwanathan and Viral Acharya.

The Central Board of Directors is the main committee of the Central Bank. The Government of India appoints
the directors for a 4-year term. The Board consists of a Governor, and not more than 4 Deputy Governors, 4
Directors to represent the regional boards, 2 from the Ministry of Finance and 10 other directors from various
fields. RBI wants to create a post of Chief Operating Officer (COO) and re-allocate work between the five of
them(4 Deputy Governor and COO).

Subsidiaries of RBI:

 National Housing Bank (NHB)


 Bharatiya Reserve Bank Note Mudran Private Limited (BRBNMPL)
 National Bank for Agriculture and Rural Development (NABARD)
 Deposit Insurance and Credit Guarantee Corporation (DICGC)

Functions of Reserve Bank of India

The Reserve Bank of India performs various traditional central banking functions as well as undertakes different
promotional and developmental measures to meet the dynamic requirements of the country.

The broad objectives of the Reserve Bank are:

 Regulating the issue of currency in India;


 Keeping the foreign exchange reserves of the country;
 Establishing the monetary stability in the country; and
 Developing the financial structure of the country on sound lines consistent with the national socio-
economic objectives and policies.

Main functions of the Reserve Bank are described below:

1. Monopoly Power of Note Issue

The Reserve Bank has the monopoly of note issue in the country. It has the sole right to issue currency notes of
all denominations except one-rupee notes. One-rupee notes, coins and small coins are issued by the Ministry of
Finance of the Government of India. At present, notes of denominations of rupees two, five, ten, twenty, fifty,
one hundred, five hundred, and two thousand are issued by the RBI. It not only issues currency but also
exchanges or destroys currency and coins not fit for circulation.

www.ibpsguise,in | mock.ibpsguide.com | www.sscexamguide.com


Copyright © IBPS Guide 6
www.ibpsguide.com
Prior to 1956, the principle of note issue of the RBI was based on the ‗proportional reserve system‘. This system
was replaced by the ‗minimum reserve system‘ in 1956 under which the RBI was required to hold at least Rs.
115 crore worth of gold as backing against the currency issued. The rest (i.e., Rs. 85 crore) should be in foreign
securities, so that— together with gold and foreign exchange reserve the minimum value of these assets is kept at
Rs. 200 crore.

2. Banker to the Government

The RBI acts as the banker to the Government of India and State Governments (except Jammu and Kashmir). As
such, it transacts all merchant banking functions for these Governments.

The RBI accepts and pays money on behalf of the Government and carries out exchange remittances and other
banking operations.

As the Government‘s banker, the RBI pro­vides short-term credit to the Government of India. This short-term
credit is obtainable through the sale of the treasury bills. Not only this, the RBI also provides ways and means of
advances (repayable within 90 days) to State governments. It may be noted that the Central Government is
empowered to borrow any amount it likes from the RBI.

The RBI also acts as the agent of the Government in respect of membership of the IMF and the World Bank.

Furthermore, the RBI acts as the adviser of the Government not only on banking and financial matters but also
on a wide range of economic issues (like financing patterns, mobilization of resources, institutional arrangements
with regard to banking and credit matters, international finance), etc.

3. Bankers‘ Bank

As a regulator and supervisor of the country‘s financial system, the RBI prescribes the broad parameters of
banking operations within which the entire banking and financial system operates in the country. The basic
objective of this activity of the RBI is to (i) maintain public confidence in the country‘s banking system, (ii)
protect the interests of depositors, and (iii) provide cost- effective banking services to the public.

As a bankers‘ bank, the RBI holds a part of the cash reserves of commercial banks and lends them funds for
short periods. All banks are required to maintain a certain percentage (lying between 3 p.c. and 20 p.c.) of their
total liabilities. The main objective of changing this cash reserve ratio by the RBI is to control credit.

The RBI provides financial assistance to commercial banks and State cooperative banks through rediscounting of
bills of exchange. As the RBI meets the needs of the commercial banks and cooperative banks, the RBI functions
as the ‗lender of the last resort‘.

www.ibpsguise,in | mock.ibpsguide.com | www.sscexamguide.com


Copyright © IBPS Guide 7
www.ibpsguide.com
The RBI has been empowered by law to supervise, regulate and control the activities of commercial and
cooperative banks. The RBI periodically inspects banks and asks them for returns and necessary information.

4. Controller of Credit

As an apex bank of the country, the RBI has been empowered to formulate, implement and monitor its monetary
policy with the objective of maintaining price stability (both internal and external) and ensuring adequate flow of
credit to the productive sectors.

The RBI controls the total supply of money and bank credit to subserve the country‘s interest. The RBI controls
credit to ensure price and exchange rate stability. To achieve this, the RBI uses all types of credit control
instruments quantitative, qualitative, and selective. The most extensively used credit instrument of the RBI is the
bank rate and now repo rate, cash, reserve ratio, etc. The RBI also relies on the selective methods of credit
control. But, it has fallen into disuse during the reform era.

5. Exchange Management and Control

One of the essential central banking functions performed by the RBI is that of maintaining the external value of
rupee. The RBI has the authority to enter into foreign exchange transactions both on its own account and on
behalf of the Government. The official external reserve of the country consists of monetary gold and foreign
assets of the Reserve Bank, besides (Special Drawing Rights or) SDR holdings.

The Reserve Bank, as the custodian of the country‘s foreign exchange reserves, is vested with the duty of
managing the investment and utilization of the reserves in the most advantageous manner. Being a manager of
foreign exchange, it manages the Foreign Exchange Management Act, (FEMA) 1999. As a manager of foreign
exchange, the RBI helps in facilitating trade (external) and payment and aims at promoting orderly development
and maintenance of the foreign exchange market in India.

6. Miscellaneous Functions:

The RBI collects, collates and publishes all monetary and banking data regularly in its weekly statements, in the
RBI Bulletin (monthly), and in the Report on Currency and Finance (annually).

7. Promotional and Developmental Functions

www.ibpsguise,in | mock.ibpsguide.com | www.sscexamguide.com


Copyright © IBPS Guide 8
www.ibpsguide.com
Apart from these traditional functions, the RBI performs various activities of promotional and developmental
nature. It attempts to mobilize savings for productive purposes. This is done in various ways. For instance, the
RBI has helped a lot in building the huge financial infrastructure that we see now.

This consists of such institutions as the Deposit Insurance and Guarantee Corporation (DIGC) (to safeguard the
interests of depositors against bank failure), the Agriculture Refinance and Development Corporation (to meet
the needs of agriculturists), IFCI, SFCs, IDBI, UTI (to meet the long and medium term needs of industry), etc.
As for cooperative credit movement, the RBI‘s performance is really commendable. This has resulted in curbing
the activities of moneylenders in the rural economy.

MONETARY POLICY:

Monetary policy is the process by which monetary authority of a country i.e. RBI controls the supply of money
in the economy by its control over interest rates in order to maintain price stability and achieve high economic
growth. In India, the central monetary authority is the Reserve Bank of India (RBI) is so designed as to maintain
the price stability in the economy.

The main objectives of Monetary Policy are,

 To maintain price stability.


 To ensure adequate flow of credit to productive sectors so as to assist growth.
 Arrangement of full employment
 Expansion of credit facility
 Equality & Justice Stability in exchange rate
 Promotion of Fixed Deposit
 Equitable distribution of Credit
Instruments of Monetary Policy:
The instruments of credit control or instruments of monetary policy are of two types:

1. Quantitative control: to control amount of credit.


2. Qualitative control: to control the allocation to different sections of economy.

www.ibpsguise,in | mock.ibpsguide.com | www.sscexamguide.com


Copyright © IBPS Guide 9
www.ibpsguide.com

MSF

LAF

REVERSE REPO
REPO

Quantitative Tools:

The Quantitative Instruments are also known as the General Tools of monetary policy. These tools are related to
the Quantity or Volume of the money. The Quantitative Tools of credit control are also called as General Tools
for credit control. They are designed to regulate or control the total volume of bank credit in the economy. These
tools are indirect in nature and are employed for influencing the quantity of credit in the country. The general
tool of credit control comprises of following instruments.

1. Reserve ratio:

Banks have to set aside certain percentage of reserves as cash or RBI approved assets. They are two types of
Reserve Ratios.

Cash Reserve Ratio (CRR):

Banks in India are required to hold a certain proportion of their deposits in the form of cash. However Banks
don't hold these as cash with themselves, they deposit such cash(aka currency chests) with Reserve Bank of India

www.ibpsguise,in | mock.ibpsguide.com | www.sscexamguide.com


Copyright © IBPS Guide 10
www.ibpsguide.com
, which is considered as equivalent to holding cash with themselves. This minimum ratio (that is the part of the
total deposits to be held as cash) is stipulated by the RBI and is known as the CRR or Cash Reserve Ratio.

When a bank's deposits increase by Rs100, and if the cash reserve ratio is 9%, the banks will have to hold Rs. 9
with RBI and the bank will be able to use only Rs 91 for investments and lending, credit purpose. Therefore,
higher the ratio, the lower is the amount that banks will be able to use for lending and investment. This power of
Reserve bank of India to reduce the lendable amount by increasing the CRR, makes it an instrument in the hands
of a central bank through which it can control the amount that banks lend. Thus, it is a tool used by RBI to
control liquidity in the banking system.

Statutory Liquidity Ratio (SLR):

Every bank is required to maintain at the close of business every day, a minimum proportion of their Net
Demand and Time Liabilities as liquid assets in the form of cash, gold and un-encumbered approved securities.
The ratio of liquid assets to demand and time liabilities is known as Statutory Liquidity Ratio (SLR). RBI is
empowered to increase this ratio up to 40%. An increase in SLR also restricts the bank's leverage position to
pump more money into the economy.

Net Demand Liabilities - Bank accounts from which you can withdraw your money at any time like your savings
accounts and current account.

Time Liabilities - Bank accounts where you cannot immediately withdraw your money but have to wait for
certain period. E.g. Fixed deposit accounts.

2. Open Market Operations:

Open market operation refers to the purchase and sale of Government securities by the Central bank in open
market.

In order to correct the excess demand or inflation, the central bank sells securities to the commercial banks and
general public. When commercial banks buy securities, their cash reserves are reduced directly. When people
buy securities, they make large withdraw of cash from commercial banks. Here their cash reserves are
diminished indirectly. Consequently, commercial banks‘ capacity to create credit is curtailed. This leads to a
reduction in the volume of investment on the part of businessmen and entrepreneurs and a decline in national
income. As a result, the state of excess demand or inflation is checked.

On the contrary, central bank can correct the state of deficient demand or deflation by purchasing securities in
the open market.

www.ibpsguise,in | mock.ibpsguide.com | www.sscexamguide.com


Copyright © IBPS Guide 11
www.ibpsguide.com
3. Rates:

Bank Rate:

This is the long term rate(Repo rate is for short term) at which central bank (RBI) lends money to other banks or
financial institutions. Bank rate is not used by RBI for monetary management now. It is now same as the MSF
rate. Current bank rate is 6.5%.

Liquidity adjustment facility (LAF):

It is a monetary policy tool which allows banks to borrow money through repurchase agreements. LAF is used to
aid banks in adjusting the day to day mismatches in liquidity. LAF consists of repo and reverse repo operations.

i. Repo rate:

Repo rate also known as the benchmark interest rate is the rate at which the RBI lends money to the banks for a
short term. When the repo rate increases, borrowing from RBI becomes more expensive. If RBI wants to make it
more expensive for the banks to borrow money, it increases the repo rate similarly, if it wants to make it cheaper
for banks to borrow money it reduces the repo rate. Current repo rate is 6.25%

ii. Reverse Repo rate:

Reverse Repo is the short term borrowing rate at which RBI borrows money from banks. The Reserve bank uses
this tool when it feels there is too much money floating in the banking system. An increase in the reverse repo
rate means that the banks will get a higher rate of interest from RBI. As a result, banks prefer to lend their money
to RBI which is always safe instead of lending it others (people, companies etc) which is always risky. Repo
Rate signifies the rate at which liquidity is injected in the banking system by RBI, whereas Reverse Repo rate
signifies the rate at which the central bank absorbs liquidity from the banks.

Marginal Standing Facility (MSF):

It is a special window for banks to borrow from RBI against approved government securities in an emergency
situation like an acute cash shortage. MSF rate is higher than Repo rate. Current MSF Rate: 6.5%.

Qualitative Instruments or Selective Tools:

The Qualitative Instruments are also known as the Selective Tools of monetary policy. These tools are not
directed towards the quality of credit or the use of the credit. They are used for discriminating between different
uses of credit. It can be discrimination favoring export over import or essential over non-essential credit supply.

www.ibpsguise,in | mock.ibpsguide.com | www.sscexamguide.com


Copyright © IBPS Guide 12
www.ibpsguide.com
This method can have influence over the lender and borrower of the credit. The Selective Tools of credit control
comprises of following instruments.

1. Margin Requirements:

The margin refers to the "proportion of the loan amount which is not financed by the bank". Or in other words, it
is that part of a loan which a borrower has to raise in order to get finance for his purpose. A change in a margin
implies a change in the loan size. This method is used to encourage credit supply for the needy sector and
discourage it for other non-necessary sectors. This can be done by increasing margin for the non-necessary
sectors and by reducing it for other needy sectors. Example: If the RBI feels that more credit supply should be
allocated to agriculture sector, then it will reduce the margin and even 85-90 percent loan can be given.

2. Selective Credit Controls:

Using this tool RBI may direct banks not to give loan for particular purpose or sector. For example: RBI may ask
bank not to give loans to traders of certain commodities like sugar, oil, pulses etc. This prevents hoarding by
traders of these commodities. This would help in increasing supply in market and thus reducing inflation.

3. Moral Suasion:

Moral Suasions are suggestion and guidelines by the RBI to the commercial banks to take so and so action and
measures in so and so trend of the economy. RBI may request commercial banks not to give loans for
unproductive purpose which does not add to economic growth but increases inflation in the economy.

NATIONALIZATION OF BANKS IN INDIA

The history of nationalization of Indian banks dates back to the year 1955 when the Imperial Bank of India was
nationalized and re-christened as State Bank of India under the SBI Act, 1955. Later on July 19, 1960, the 7
subsidiaries of SBI were also nationalized with deposits more than 200 crores. Those subsidiaries are

1. State Bank of Hyderabad (SBH)


2. State Bank of Indore
3. State Bank of Saurashtra (SBS)
4. State Bank of Mysore (SBM)
5. State Bank of Bikaner and Jaipur (SBBJ)
6. State Bank of Patiala (SBP)
7. State Bank of Travancore (SBT)

www.ibpsguise,in | mock.ibpsguide.com | www.sscexamguide.com


Copyright © IBPS Guide 13
www.ibpsguide.com
The Government of India issued an ordinance bill ―Banking Companies (Acquisition and Transfer of
Undertakings) bill, 1969‖ and nationalized the 14 largest commercial banks with effect from the midnight of 19
July 1969. These banks contained 85 percent of bank deposits in the country. All of these commercial banks
have a deposit base over Rs.50 crores. Those 14 banks are

1. Allahabad Bank
2. Bank of Baroda
3. Bank of India
4. Bank of Maharashtra
5. Canara Bank
6. Central Bank of India
7. Dena Bank
8. Indian Bank
9. Indian Overseas Bank
10. Punjab National Bank
11. Syndicate Bank
12. UCO Bank
13. Union Bank of India
14. United Bank of India

On 1980, Government of India nationalized another 6 banks; all of these banks have a deposit base over Rs.200
crores. The stated reason for the nationalization was to give the government more control of credit delivery. With
this nationalization, the Government of India controlled around 91% of the banking business of India. Those 6
nationalized banks are

1. Andhra Bank
2. Corporation Bank
3. New Bank of India
4. Oriental Bank of Commerce
5. Punjab & Sindh Bank
6. Vijaya Bank

Later on, in the year 1993, the government merged New Bank of India with Punjab National Bank. It was the
only merger between nationalized banks and resulted in the reduction of the number of nationalized banks from
20 to 19.

www.ibpsguise,in | mock.ibpsguide.com | www.sscexamguide.com


Copyright © IBPS Guide 14
www.ibpsguide.com

INDIAN BANKING STRUCTURE

Scheduled & Non –scheduled Banks:

 A scheduled bank is a bank that is listed under the second schedule of the RBI Act, 1934.
 In order to be included under this schedule of the RBI Act, banks have to fulfill certain conditions such
as having a paid up capital and reserves of at least 5 lakh and satisfying the Reserve Bank that its affairs
are not being conducted in a manner prejudicial to the interests of its depositors.
 Scheduled banks are further classified into commercial and cooperative banks.
 Non- scheduled banks are those which are not included in the second schedule of the RBI Act, 1934.

Commercial Banks:

 Commercial banks may be defined as, any banking organization that deals with the deposits and loans
of business organizations.
 Commercial banks issue bank cheques and drafts, as well as accept money on term deposits.
 Commercial banks also act as moneylenders, by way of installment loans and overdrafts.
 It also allow for a variety of deposit accounts, such as checking, savings, and time deposit.

www.ibpsguise,in | mock.ibpsguide.com | www.sscexamguide.com


Copyright © IBPS Guide 15
www.ibpsguide.com
 These institutions are run to make a profit and owned by a group of individuals.

Scheduled Commercial Banks (SCBs):

 Scheduled commercial banks (SCBs) account for a major proportion of the business of the scheduled
banks.
 SCBs in India are categorized into the five groups based on their ownership and/or their nature of
operations. State Bank of India and its subsidiaries are recognized as a separate category of SCBs,
because of the distinct statutes (SBI Act, 1955 and SBI Subsidiary Banks Act, 1959) that govern them.
 Nationalized banks and SBI and associates together form the public sector banks group.
 IDBI ltd. has been included in the nationalized banks group since December 2004.
 Private sector banks include the old private sector banks and the new generation private sector banks
which were incorporated according to the revised guidelines issued by the RBI regarding the entry of
private sector banks in 1993.
 Foreign banks are present in the country either through complete branch/subsidiary route presence or
through their representative offices.

Types of Scheduled Commercial Banks:

Nationalized Banks:

Both Public sector banks and SBI are categorized under nationalized banks. In these banks the majority stake is
held by the Government of India. The shares of these banks are listed on stock exchanges.

Private Sector Banks:

These are banks majority of share capital of the bank is held by private individuals. These banks are registered as
companies with limited liability. There are two types of Private sector banks

1. Old Private Sector Banks


2. New Private Sector Banks

Old Private Sector Banks:

 The banks, which were not nationalized at the time of bank nationalization that took place during 1969
and 1980, are known to be the old private-sector banks.
 These were not nationalized, because of their small size and regional focus. Most of the old private-
sector banks are closely held by certain communities their operations are mostly restricted to the areas

www.ibpsguise,in | mock.ibpsguide.com | www.sscexamguide.com


Copyright © IBPS Guide 16
www.ibpsguide.com
in and around their place of origin. Their Board of directors mainly consists of locally prominent
personalities from trade and business circles.
 For example City Union Bank, Karur Vysya Bank, Karnataka Bank, South Indian Bank etc.

New Private Sector Banks:

 The banks, which came in operation after 1991, with the introduction of economic reforms and
financial sector reforms are called "new private-sector banks".
 Banking regulation act was then amended in 1993, which permitted the entry of new private-sector
banks in the Indian banking s sector.
 Axis Bank, Kotak Mahindra Bank, Yes Bank, ICICI Bank, HDFC Bank are few example for the
Private sector banks.

Foreign Banks:

These banks are registered and have their headquarters in a foreign country but operate their branches in our
country. Examples of foreign banks in India are: HSBC, Citibank, Standard Chartered Bank, etc.

Regional Rural Banks:

 Regional Rural Banks were established under the provisions of an Ordinance promulgated on the 26th
September 1975 and the RRB Act, 1976 with an objective to ensure sufficient institutional credit for
agriculture and other rural sectors.
 The area of operation of RRBs is limited to the area as notified by GOI covering one or more districts
in the State.
 RRBs are jointly owned by GOI, the concerned State Government and Sponsor Banks, the issued
capital of a RRB is shared by the owners in the proportion of 50%, 15% and 35% respectively.
 Prathama bank is the first Regional Rural Bank in India located in the city Moradabad in Uttar Pradesh.

Cooperative Banks:

 A co-operative bank is a financial entity which belongs to its members, who are at the same time the
owners and the customers of their bank.
 Co-operative banks are often created by persons belonging to the same local or professional
community or sharing a common interest.
 It generally provides their members with a wide range of banking and financial services (loans,
deposits, banking accounts, etc).

www.ibpsguise,in | mock.ibpsguide.com | www.sscexamguide.com


Copyright © IBPS Guide 17
www.ibpsguide.com
 They provide limited banking products and are specialists in agriculture-related products. Cooperative
banks are the primary financiers of agricultural activities, some small-scale industries and self-
employed workers.
 It functions on the basis of ―no-profit no-loss‖.
 Anyonya Co-operative Bank Limited (ACBL) is the first co-operative bank in India located in the city
of Vadodara in Gujarat.

Structure of Cooperative Banking in India:

The structure of cooperative network in India can be divided into 2 broad segments-

1. Urban Cooperative Banks


2. Rural Cooperatives

Urban Cooperatives:

 Urban Cooperatives can be further divided into scheduled and non-scheduled. Both the categories are
further divided into multi-state and single-state. Majority of these banks fall in the non-scheduled and
single-state category.
 Banking activities of Urban Cooperative Banks are monitored by RBI.
 Registration and Management activities are managed by Registrar of Cooperative Societies (RCS).
These RCS operate in single-state and Central RCS (CRCS) operate in multiple state.

Rural Cooperatives:

 The rural cooperatives are further divided into short-term and long-term structures.
 The short-term cooperative banks are three tiered operating in different states. Those are,
1. State Cooperative Banks- They operate at the apex level in states
2. District Central Cooperative Banks-They operate at the district levels
3. Primary Agricultural Credit Societies-They operate at the village or grass-root level.

Likewise, the long-term structures are further divided into –

State Cooperative Agriculture and Rural Development Banks (SCARDS) - These operate at state-level.

Primary Cooperative Agriculture and Rural Development Banks (PCARDBS) - They operate at district/block
level.

www.ibpsguise,in | mock.ibpsguide.com | www.sscexamguide.com


Copyright © IBPS Guide 18
www.ibpsguide.com
The rural banking cooperatives have a complex monitoring structure as they have a dual control which has led to
many problems. A Forum called State Level Task Force on Cooperative Urban Banks (TAFCUB) has been set-
up to look into issues related to duality in control.

All banking activities are regulated by a shared arrangement between RBI and NABARD. All management and
registration activities are managed by RCS.

ALL INDIA FINANCIAL INSTITUTIONS

 All India Financial Institutions (AIFI) is a group composed of Development Finance Institutions (DFI)
and Investment Institutions that play a pivotal role in the financial markets.
 The financial institutions assist in the proper allocation of resources, sourcing from businesses that have
a surplus and distributing to others who have deficits - this also assists with ensuring the continued
circulation of money in the economy.
 Possibly of greatest significance, the financial institutions act as an intermediary between borrowers
and final lenders, providing safety and liquidity. This process subsequently ensures earnings on the
investments and savings involved.
 The development banks are classified by following way,

www.ibpsguise,in | mock.ibpsguide.com | www.sscexamguide.com


Copyright © IBPS Guide 19
www.ibpsguide.com
Industrial Finance Corporation of India (IFCI):

 The IFCI was the 1st specialized financial institution setup in India to provide term finance to large
industries in India.
 It was established on 1st July, 1948 under The Industrial Finance Corporation Act of 1948.
 The Industrial Financial Corporation of India is authorized to grant loans to industrial companies
repayable with twenty five years grants, loans in foreign currency to certain industries, under write
bonds, shares and debentures etc. provided they are disposed of by the I.F.C.I. within seven years,
guarantee deferred payments by importers of capital goods of foreign manufacturers, accept deposit
from the local institution, guarantee loans from any bank of a foreign country, subscribe shares of
industrial companies.
 The corporation‘s role now extends to the entire industrial spectrum in the country. The facilities and
services being provided by IFCI can be deemed to fall broadly under
 Project finance
 Financial services
 Promotional services
 The Industrial Finance Corporation has played a vital role in our industrial economy. Since its
inception, the Corporation has provided financial assistance to the underdeveloped industrial concerns.
 The Corporation has the power to examine the financial aspects of the industrial companies and give
valuable advice to the management for improving their schemes.
 IFCI. has launched promotional schemes like
 Subsidy in interest for women entrepreneurs
 Schemes for modernization of small scale industrial units,
 Consultancy fee subsidy for providing marketing assistance,
 Pollution control schemes etc.
 It is also diversifying its activities in the field of merchant banking to render other financial services
like project counseling, sanction of loans etc. I.F.C.I. is also showing concern for the development of
backward districts of the country.
 Industrial Development Bank of India (IDBI):
 Industrial Development Bank of India (IDBI) established under Industrial Development Bank of India
Act, 1964 is the principal financial institution for providing credit and other facilities for developing
industries and assisting development institutions.
 Till 1976, IDBI was a subsidiary bank of RBI.

www.ibpsguise,in | mock.ibpsguide.com | www.sscexamguide.com


Copyright © IBPS Guide 20
www.ibpsguide.com
 A committee formed by RBI with chairman S.H.Khan recommended the transfer of IDBI from a
developmental bank to a commercial bank
 In 1976 it was separated from RBI and the ownership was transferred to Government of India.
 With the Industrial Developmental Bank (Transfer of Undertaking and Repeat) Act 2013 IDBI became
IDBI Ltd.
 In September 2004 RBI incorporated IDBI as ‗scheduled bank‘ under RBI Act,1934
 IDBI is the tenth largest bank in the world in terms of development.
 Some of the financial institutions built with the support of IDBI are as follows
 Securities and Exchange Board of India (SEBI)
 National Stock Exchange of India (NSE)
 National Securities Depository Limited (NSDL)
 Stock Holding Corporation of India Limited (SHCIL)
 Credit Analysis & Research Ltd.
 Exim Bank
 Small Industries Development Bank of India (SIDBI)
 Entrepreneurship Development Institute of India

Small Industries Development Bank of India (SIDBI):

 SIDBI was set up on April 2, 1990 under an Act of Indian Parliament.


 It has its head office in Lucknow, Uttar Pradesh.
 Kshatrapati Shivaji is the chairman and managing director of SIDBI.
 It was set up to promote, finance and develop the Micro, Small and Medium Enterprise (MSME) sector
and for coordinating the functions of the institutions engaged in similar activities.
 ―To facilitate and strengthen credit flow to MSMEs and address both financial and developmental gaps
in the MSME eco-system‖ is the mission of SIDBI.
 The four basic objectives of SIDBI are:
▪ Financing
▪ Promotion
▪ Development
▪ Co-ordination

for orderly growth of industry in the small scale sector.

Facts about SIDBI:

www.ibpsguise,in | mock.ibpsguide.com | www.sscexamguide.com


Copyright © IBPS Guide 21
www.ibpsguide.com
 The business domain of SIDBI consists of Micro, Small and Medium Enterprises (MSMEs).
 It was set up as a wholly owned subsidiary of Industrial Development Bank of India, but now is an
independent financial institution.
 Its branches are available in all major clusters of the country.
 SIDBI is in the top 30 Development Banks of the World according to the ranking of The Banker,
London.
 SIDBI also provides financial support to National Small Industrial Corporation (NSIC) for providing
leasing, hire-purchase, and marketing support to the industrial units in the small-sector.

National Bank for Agriculture And Rural Development (NABARD):

 National Bank for Agriculture and Rural Development (NABARD) is an apex development bank in
India.
 Headquarters – Mumbai.
 It was established by the Committee set up by RBI to Review Arrangements for Institutional Credit for
Agriculture and Rural Development (CRAFICARD) under the Chairman Shri. B. Sivaraman on 12th
July 1982.
 Its main aim is to uplift rural India by increasing the credit flow for evaluation of agricultural and rural
farm sector.
 Chairman – Dr. Harsh Kumar Bhanwala.
 NABARD is active member of Alliance for Financial Inclusion
 NABARD replaced the following organizations
▪ Agricultural Credit Department
▪ Rural Planning and Credit Cell
▪ Agricultural Refinance and Development Corporation
 The initial corpus of NABARD was Rs.100 crores. Consequent to the revision in the composition of
share capital between Government of India and RBI, the paid up capital as on 31 May 2017, stood at
Rs.30,000 crore with Government of India holding Rs.30,000 crore (100% share).
 NABARD takes measures towards institutions which help in improving absorptive capacity of the
credit delivery system including
▪ Monitoring
▪ Formulating rehabilitation schemes
▪ Restructuring of credit institutions
▪ Training of personnel
 It coordinates the rural financing activities of the
www.ibpsguise,in | mock.ibpsguide.com | www.sscexamguide.com
Copyright © IBPS Guide 22
www.ibpsguide.com
▪ Government of India
▪ State Governments
▪ Reserve Bank of India
▪ Other national institutions concerned with policy formulation
 NABARD refinances financial institutions which finance the rural sector. These refinances are availed
by the following organizations
▪ State Co-operative Agriculture and Rural Development Bank (SCARDB)
▪ State Co-operative Banks(SCB)
▪ Regional Rural Banks(RRBs)
▪ Commercial Banks(CB)
▪ Other financial institutions approved by RBI
 It has 336 District offices across the country including 1-sub office at Port Blair and one special cell at
Srinagar.
 It has 6 training establishments.
 NABARD is also known as Self Help Group (SHG) Bank Linkage Programme. About 22 lakh SGHs
were credited through this programme.
 NADARD has a portfolio of Natural Resource Management Programmes in the following fields
▪ Watershed development
▪ Tribal development
▪ Farm innovation
 The RBI and NABARD has laid out guidelines for commercial, Regional Rural and Cooperative banks
to provide data regarding loans given by banks to the microfinance institutions.

Export-Import Bank of India (EXIM Bank):

 Export–Import Bank of India is the premier export finance institution in India, established in 1982
under Export-Import Bank of India Act 1981.
 EXIM Bank of India has been both a catalyst and a key player in the promotion of cross border trade
and investment.
 The Export-Import Bank of India ranks high among the specialized financial institutions in India.
 It offers financial assistance to the exporters and importers and also by acting as a link between the
various financial institutions to ensure overall development of the Indian financial market.
 The bank offers financial assistance to the various sectors like agriculture, export, import, and film
industry.

www.ibpsguise,in | mock.ibpsguide.com | www.sscexamguide.com


Copyright © IBPS Guide 23
www.ibpsguide.com
 For the agricultural sector the bank has arranged for unique financial programs like posting shipment
credit, terming loans etc. The category of term loans are issued for modernization, purchase of
equipments, acquisitions etc.
 For the exporters the bank provides warehousing finance, export lines of credit facilities. The funded
capital scheme of the bank includes long-term working capital, cash flow financing, and the non funded
capital scheme include letter of credit limits, guarantee limits.
 For the film industry the bank has arranged for cash flow financing for film production, funds for
exhibition in overseas market.
 The bank is engaged in offering specialized services Human Resource Management, Research and
Planning, Internal Audit etc.
 The Export-Import Bank of India has set up offices throughout India and in foreign countries as well.
 The head office is located at Mumbai and Shri Yaduvendra Mathur is a CEO of the bank.

National Housing Bank (NHB):

 NHB was set up on July 9, 1988 under the National Housing Bank Act, 1987.
 The Head Office of NHB is at New Delhi.
 NHB is wholly owned by Reserve Bank of India, which contributed the entire paid-up capital.
 Managing Director & Chief Executive Officer- Shri Sriram Kalyanaraman.

The objectives of NHB:

 To act as an apex institution for housing finance companies (apex means the central institution like RBI
for all the banks and financial institutions).
 To promote a network of dedicated housing finance institutions to adequately serve various regions and
different income groups.
 To provide a cost effective housing finance system to all the sections of society.
 To upgrade housing stock in the country, provide building materials for housing and supply of
buildable land.
 To encourage public companies to provide serviced land for housing.

Micro Units Development and Refinance Agency Ltd (MUDRA) Bank:

 Micro Units Development and Refinance Agency Bank (MUDRA Bank) was set up as a public sector
financial institution on 8 April 2015 under Pradhan Mantri MUDRA Yojana (PMMY).

www.ibpsguise,in | mock.ibpsguide.com | www.sscexamguide.com


Copyright © IBPS Guide 24
www.ibpsguide.com
 It is set up to provide loans at low rates to Micro-Finance Institutions (MFIs) and Non-Banking
Financial Companies (NBFCs) which then provide credit to MSMEs.
 Established with an initial corpus of Rs.20,000 crores and a credit guarantee corpus of Rs 3,000 crores.
 Under the aegis of Pradhan Mantri MUDRA Yojana (PMMY), MUDRA has already created its initial
products / schemes. The financial limit for these schemes are,
 Shishu : covering loans upto 50,000/-
 Kishor :covering loans above 50,000/- and upto 5 lakh
 Tarun :covering loans above 5 lakh to 10 lakh
 MUDRA‘s delivery channel is conceived to be through the route of refinance primarily to
Banks/NBFCs/MFIs.

Some important facts:

 MUDRA bank is not a physical bank.


 Eligible entities can apply for MUDRA Bank loan under PMMY in NBFCs, MFIs, Rural Banks,
District Banks, Nationalize Banks, Private Banks, Primary Lending Institutions and other
intermediaries.
 Any person who is eligible and having the need of loan of up to Rs 10 lakhs can approach for loans
under PMMY.
 It will not refinance agriculture sector under PMMY but the traders of vegetables & fruits are covered
under MUDRA Bank Schemes.
 The bank is set up as a subsidiary of the Small Industries Development Bank of India (SIDBI), and
later it will be converted to a separate institution.
 The bank was decided to act as regulator of MFIs, but later the decision was withdrawn and it will be
done by RBI.
 There is no fix interest rate in MUDRA loan; it will vary from bank to bank.
 MUDRA will be extending refinance support to RRBs for enhancing their liquidity.

SECURITY EXCHANGE BOARD OF INDIA (SEBI)

SEBI was constituted by Govt. of India during 1988 and accorded statutory powers under SEBI Act, 1992,
with the objectives:

▪ To promote the development of security market


▪ To protect interest of investors
▪ To regulate the security market

www.ibpsguise,in | mock.ibpsguide.com | www.sscexamguide.com


Copyright © IBPS Guide 25
www.ibpsguide.com
The SEBI Act was amended on 25th January, 1995 in Mumbai to give additional powers for ensuring orderly
development of the capital market and to enhance SEBI ability to protect the interests of the investors. SEBI
can file complaints in courts and notify its regulations without the prior approval of Central Government.

Management:

SEBI is managed by its Chairman and 5 members and has departments such as –

▪ Issue Management Department


▪ Primary Market Department
▪ Secondary Market Department
▪ Institutional Investment Department
▪ It has 2 advisory committees, one each for primary and secondary market to provide advisory
guidance in framing policies and regulations.

Functions of SEBI:

SEBI Functions are divided into:

▪ Regulatory functions
▪ Developmental functions

Regulatory Function:

 Regulation of stock market and self regulatory organizations


 Registrations & regulation of Self Brokers, sub brokers, merchant bankers
 Registration and Regulation of Investment Schemes, including Mutual Funds
 Regulating acquisition of Shares and takeover of companies
 Prohibition of illegal and unfair trade practices relating to securities market
 Prohibition of insider trading

Developmental function:

 Promotion of fair practices


 Promotion of self regulatory organizations
 Conducting Research and publishing important information to all market participants
 Promoting Investor‘s education and training of intermediaries

www.ibpsguise,in | mock.ibpsguide.com | www.sscexamguide.com


Copyright © IBPS Guide 26
www.ibpsguide.com
For the discharge of its functions efficiently, SEBI is vested with the following powers:

 To approve by−laws of stock exchanges.


 To require the stock exchange to amend their by−laws.
 Inspect the books of accounts and call for periodical returns from recognized stock exchanges.
 Inspect the books of accounts of financial intermediaries.
 Compel certain companies to list their shares in one or more stock exchanges.
 Registration brokers.

INSURANCE REGULATORY AND DEVELOPMENT AUTHORITY (IRDA)

 IRDA was set up as autonomous body under the IRDA Act, 1999.
 IRDA Act was passed upon the recommendations of Malhotra Committee report (7 Jan,1994),
headed by Mr R.N. Malhotra.
 IRDA‘s Mission is to protect the interests of policyholders and to regulate and develop the insurance
industry.
 It has its headquarters at Hyderabad, Telangana where it shifted from Delhi in 2001.
 Insurance Regulatory and Development Authority (IRDA) has been renamed as ‗Insurance Regulatory
and Development Authority of India‘ after the promulgation of Insurance Laws (Amendment)
Ordinance, 2014, by the President of India on December 26, 2014.

Objectives of IRDA:

 To promote the interest and rights of policy holders.


 To promote and ensure the growth of Insurance Industry.
 To ensure speedy settlement of genuine claims and to prevent frauds and malpractices
 To bring transparency and orderly conduct of in financial markets dealing with insurance.

Composition of Authority of IRDA:

The Authority is a ten member team of

 A Chairman and every other whole-time member – 5 years (Maximum age is 60 years).
 Five whole-time members (Maximum age is 62 years).
 Four part-time members (not more than 5 years)
 The current Chairman of IRDA is Mr.T.S.Vijayan.

Functions and Duties of IRDAI:

www.ibpsguise,in | mock.ibpsguide.com | www.sscexamguide.com


Copyright © IBPS Guide 27
www.ibpsguide.com
Section 14 of the IRDA Act, 1999 lays down the following duties, powers and functions of IRDA.

 Registering and regulating insurance companies


 Protecting policyholders‘ interests
 Licensing and establishing norms for insurance intermediaries
 Promoting professional organisations in insurance
 Regulating and overseeing premium rates and terms of non-life insurance covers
 Specifying financial reporting norms of insurance companies
 Regulating investment of policyholders‘ funds by insurance companies
 Ensuring the maintenance of solvency margin by insurance companies
 Ensuring insurance coverage in rural areas and of vulnerable sections of society.

TYPES OF ACCOUNT

Traditionally banks in India have four types of deposit accounts, namely

1. Saving Account

2. Current Account

3. Fixed Deposit Account or Time Deposit Accounts

4. Recurring Deposit Account

However, in recent years, due to ever increasing competition, some banks have introduced new products, such as

1. Demat Account
2. Basic Savings Bank Deposit Account (BSBDA)
3. NRI Accounts

Savings Account:

 Any resident individual- single accounts, two or more individuals in joint accounts, Associations, clubs etc., are
eligible for this account.

 Modest credit option available to the depositor.

 Two free cheque books will be issued per year.

 Internet banking facility will be provided without any charge.

 Balance enquiry, NEFT, Bill payment, Mobile recharge etc., are provided through mobile phones.

www.ibpsguise,in | mock.ibpsguide.com | www.sscexamguide.com


Copyright © IBPS Guide 28
www.ibpsguide.com
 Students can open this account with zero balance by providing the required documents.

 In this account interest will be given in daily basis.

 There is no restriction on the number and amount of deposits. But withdrawals are subjected to certain
restrictions.

 Some banks recommend to maintain a minimum amount to keep it functioning.

Current Accounts:

 Current Accounts are basically meant for businessmen and are never used for the purpose of investment or
savings.

 These deposits are the most liquid deposits and there are no limits for number of transactions or the amount of
transactions in a day.

 Most of the current accounts are opened in the names of firm / company accounts.

 Cheque book facility is provided and the account holder can deposit all types of the cheques and drafts in their
name or endorsed in their favor by third parties.

 No interest is paid by banks on these accounts. On the other hand, banks charge certain service charges, on such
accounts.

 The main objective of Current Account holders in opening this account is to enable them (mostly businessmen)
to conduct their business transactions smoothly.

 The current accounts do not have any fixed maturity as these are on continuous basis accounts.

Fixed Deposit Account or Time Deposit Accounts:

 In Fixed Deposit Account (also known as FD Account), a particular sum of money is deposited in a bank for
specific period of time.

 Its one time deposit and one time take away (withdraw) account. The money deposited in this account cannot be
withdrawn before the expiry of period.

 However, in case of need, the depositor can ask for closing the fixed deposit prematurely by paying a penalty.
The penalty amount varies with banks.

 A high interest rate is paid on fixed deposits. The rate of interest paid for fixed deposit varies according to
amount, period and also from bank to bank.

www.ibpsguise,in | mock.ibpsguide.com | www.sscexamguide.com


Copyright © IBPS Guide 29
www.ibpsguide.com
Recurring Deposit Account:

 Recurring deposit account or RD account is opened by those who want to save certain amount of money
regularly for a certain period of time and earn a higher interest rate.

 In RD account a fixed amount is deposited every month for a specified period and the total amount is repaid with
interest at the end of the particular fixed period.

 The period of deposit is minimum six months and maximum ten years.

 The interest rates vary for different plans based on the amount one save and the period of time and also on banks.

 No withdrawals are allowed from the RD account. However, the bank may allow to close the account before the
maturity period.

 These accounts can be opened in single or joint names. Banks are also providing the Nomination facility to the
RD account holders.

Demat Account:

 Demat account is an account in which the shares and securities are held in dematerialized form i.e. electronically
without any physical papers held.

 For getting a Demat account, one needs to go to one of the Depository Participants or DPs. DPs could be banks,
brokers or financial institution that have been allowed to provide this service. The DPs act as intermediary
between central depository and the investor.

 To carry out transactions in the stock market, one should get open a demat account.

 To open a demat account, KYC procedure is also followed.

 Multiple demat accounts can be opened.

 Demat accounts are held by a single person i.e. no joint accounts can be operated.

 There is no need of any minimum balance in demat account.

Basic Savings Bank Deposit Account (BSBDA):

 A person having BSBDA in a bank cannot have a Savings Account in the same bank. But he can have other
accounts such as fixed deposit and recurring deposit accounts.

 A person having savings account can open a BSBDA in the same bank. But he will have to close the savings
account within 30 days from the date of opening of BSBDA.

www.ibpsguise,in | mock.ibpsguide.com | www.sscexamguide.com


Copyright © IBPS Guide 30
www.ibpsguide.com
 BSBDAs can be opened in any commercial banks and also in foreign banks.

 This account will be considered as normal banking service.

 For this account, maintenance of minimum balance is not required.

 ATM card/ ATM cum Debit card, Rupay card will be given for the account holders.

 There are going to be no limit on the number of deposits that can be made in a month but, account holders will
be allowed most of 4 withdrawals in a month, which includes ATM withdrawals also.

 The above facilities will be given without any charge. There will be no charge levied for non-operation/
activation of in-operative basic saving bank deposit account.

 For this account, overdraft facility will be provided up to Rs. 5000/-.

 Total credits in such accounts should not exceed one lakh rupees in a year.

 Maximum balance in the account should not exceed fifty thousand rupees at any time.

 In a month, the total of cash withdrawals and transfers cannot exceed Rs 10,000.

 Foreign remittances cannot be credited to Small Accounts without completing normal KYC formalities.

 Small accounts are valid for a period of 12 months initially which may be extended by another 12 months if the
person provides proof of having applied for an Officially Valid Document.

NRI Accounts:

Not only the Indians living in India can have their accounts in the banks of India, but the people who leave India and reside
in some other country or become NRIs (Non-Resident Indian) and PIOs (Person of Indian Origin) can also maintain their
accounts in India enjoying the various benefits of the accounts as applicable.

There are three types of NRI/PIO accounts

1. NRO Account

2. NRE Account

3. FCNR Account

Non Resident Ordinary (NRO) Account:

 NRO account is a Savings Account/ Current Account/ Fixed Deposit Account/ Recurring Deposit account
opened by NRIs and PIOs.

www.ibpsguise,in | mock.ibpsguide.com | www.sscexamguide.com


Copyright © IBPS Guide 31
www.ibpsguide.com
 It is a rupee denominated account i.e. the amount in the account is maintained in Indian Rupees.

 The NRIs and PIOs who transfer from India and have funds gathered in India like rent income, pension, etc. can
enjoy the benefits of NRO account.

 If the residents who leave India have account in India, then that account can be converted to NRO account with
the same account number.

 So the account is efficient for maintaining local rupee earnings in India while living abroad.

 Joint account facility and nomination facility is available with Indian residents.

 Cheque book and ATM card facility is available for joint accountee in India.

 The credit balances in NRO account are subject to respective income tax bracket.

Non Resident External (NRE) Account:

 NRE account is a rupee denominated account which can be Savings Account/ Current Account/ Fixed Deposit
Account/ Recurring Deposit account opened by NRIs and PIOs.

 The NRIs and PIOs who want to transfer their foreign earnings to India can enjoy the benefits of NRE account.

 So the account is efficient to use foreign earnings in India while living abroad.

 It is a safe and simple online money transfer service.

 Joint account facility and nomination facility is available with other NRIs or Indian Residents.

 Loan facility is also available against funds in NRE account.

 The principal amount and the interest paid on the amount are not taxable in India.

 The funds in account can be converted to any foreign currency.

Foreign Currency Non Resident (FCNR) Account:

 FCNR account is a term deposit account that can be maintained by NRIs and PIOs in foreign currency.
Authorized dealer banks in India can allow deposits in any of the permitted currency (currency freely
convertible).

 The NRIs and PIOs who want to keep their savings as fixed deposits in Indian banks can apply for FCNR
account. They can gain more rate of interest in India than abroad.

 The maturity period of term deposit ranges from 1 year to 5 years.

www.ibpsguise,in | mock.ibpsguide.com | www.sscexamguide.com


Copyright © IBPS Guide 32
www.ibpsguide.com
 The account is maintained in foreign currency so the money is not converted to Indian Rupees as in case of NRO
account.

 So there is no Foreign Exchange risk, for example: if you deposited $100 in the account with 1% rate of interest,
after 1 year on maturity you will get $ 101 irrespective of what the previous or current currency rates are.

 Funds in FCNR account can be used for making local payments in India.

 Joint account facility and nomination facility is available with other NRIs or Indian Residents.

 Loan facility is also available against funds in FCNR account.

 The principal amount and the interest paid on the amount are not taxable in India.

 The funds in account can be converted to any foreign currency.

NON BANKING FINANCIAL COMPANIES (NBFC)

 Non-bank financial companies (NBFCs) are financial institutions that provide banking services without
meeting the legal definition of a bank, i.e. one that does not hold a banking license.
 NBFCs are registered under the company act, 1956 of India. NBFCs operations are regulated by the
Reserve Bank of India, within the framework of Reserve Bank of India Act, 1934.
 NBFCs do offer all sorts of banking services, such as loans and credit facilities, retirement planning,
money markets, underwriting, and merger activities. Examples of NBFC in India
▪ Fusion Microfinance Pvt Ltd,
▪ Svatantra Micofin Pvt. Ltd.,
▪ S. V. Creditcare Network Pvt. Ltd,
▪ Saija Finance Pvt. Ltd,
▪ LIC,
▪ GIC,
▪ UTI.
 Systematically important NBFCs are Non-banking Financial companies whose asset size is more than
Rs.500 Cr as per their balance sheet.
 The activities carried out by systematically important NBFCs have a direct bearing on the financial
stabilities of the overall economy.
 Types of NBFCs:
▪ Asset Finance Company

www.ibpsguise,in | mock.ibpsguide.com | www.sscexamguide.com


Copyright © IBPS Guide 33
www.ibpsguide.com
▪ Investment Company
▪ Loan Company
▪ Infrastructure Finance Company
▪ Systematically Important Core Investment Company
▪ Infrastructure Debt Fund
▪ Micro-Finance Institution
▪ Factors
▪ Mortgage Guarantee Companies
▪ Non-Operative Financial Holding Company
Difference between banks & NBFCs:

NBFCs lend and make investments and hence their activities are akin to that of banks; however there are a few
differences as given below:

i. NBFC cannot accept demand deposits;


ii. NBFCs do not form part of the payment and settlement system and cannot issue cheques drawn on
itself;
iii. deposit insurance facility of Deposit Insurance and Credit Guarantee Corporation is not available to
depositors of NBFCs, unlike in case of bank.

Important facts related to NBFC:

 NBFCs are allowed to accept/renew public deposits for a minimum period of 12 months and maximum
period of 60 months.
 They cannot accept demand deposits.
 The interest rates charged to the borrowers by NBFCs are not regulated by RBI. It has the right to
choose its own interest rates but it cannot offer interest rates higher than the ceiling rate prescribed by
RBI from time to time. However, NBFC should provide complete transparency to its customer about
the rate of interest charged in the application form.
 NBFCs cannot offer gifts/incentives or any other additional benefit to the depositors.
 NBFCs (except certain AFCs) should have minimum investment grade credit rating.
 The deposits with NBFCs are not insured.
 The repayment of deposits by NBFCs is not guaranteed by RBI.
 Certain mandatory disclosures are to be made about the company in the Application Form issued by the
company soliciting deposits.

www.ibpsguise,in | mock.ibpsguide.com | www.sscexamguide.com


Copyright © IBPS Guide 34
www.ibpsguide.com
 All NBFCs are not entitled to accept public deposits. Only those NBFCs to which the Bank had given a
specific authorization can accept public deposits.
 The RBI does not guarantee repayment of deposits by NBFCs even though they may be authorized to
collect deposits.
 If an NBFC defaults in repayment of deposit, the depositor can approach Company Law Board or
Consumer Forum or file a civil suit in a court of law to recover the deposits.
 There is no Ombudsman for hearing complaints against NBFCs.
 However, in respect of credit card operations of an NBFC, which is a subsidiary of a bank, if a
complainant does not get satisfactory response from the NBFC within a maximum period of 30 days
from the date of lodging the complaint, the customer will have the option to approach the Office of the
concerned Banking Ombudsman for redressal of his grievance/s.

Supervision and Inspection of NBFCS by RBI:

 The RBI conducts on-site inspection and off-site surveillance of NBFCs.


 Off-site surveillance is undertaken by calling for periodical returns. These are generally fortnightly,
monthly or annual returns.
 The on-site inspection is mainly used to ensure that the interest of the depositors is well protected and
these funds are not in danger of vanishing through losses or otherwise.
 The RBI conducts the inspection under the system known as the alphabets of CAMELS. It stands for
C = Capital Adequacy requirements
A = Asset quality, like standard etc., assets
M = Management, the level and expertise and appraisal capacity of management.
E = Earning capacity of NBFC
L = Liquidity, the level of liquidity and the components of liquidity are verified.
S = Systems and control exist in the NBFC, its effectiveness etc.

Some important NBFCs operational in India:


 HDFC – established in 1977 provides mortgages, life Insurance, mutual funds and Micro Finance
 Power Finance Company – established in 1986 provides financial consulting, investment banking and
loan management
 Reliance Capital – established in 1986 – provides asset management, insurance, broking and
distribution, commercial finance and mutual funds
 Infrastructure Development Finance Company – established in 1997 – provides finance for
infrastructure projects, corporate finance, mutual funds and investment banking

www.ibpsguise,in | mock.ibpsguide.com | www.sscexamguide.com


Copyright © IBPS Guide 35
www.ibpsguide.com
 Rural Electricity Corp. – established in 1969 – provides investment and private banking and asset
management
 Shree Ram Transport Finance – established in 1974 – provides consumer vehicle finance, city union
finance and micro finance
 Bajaj Holdings – established in 2007 – Asset management, loans and micro finance
 M & M financial – established in 1991 – financial services, micro finance and asset management.
CREDIT RATING AGENCIES

 A Credit Rating Agency is simply an institution that allocates Credit Ratings to the borrower‘s.
 The basis of these ratings is the debt repayment capacity of the borrower which involves timely interest
payments and the likelihood of default by the customer.
 This credit score reports are considered highly important for getting the loan.
 Credit Rating Agencies in the country are regulated by SEBI (Securities and Exchange Board of India).
 The SEBI (Credit Rating Agencies) Regulations, 1999 govern the credit rating agencies and provide for
eligibility criteria for registration of credit rating agencies, monitoring and review of ratings,
requirements for a proper rating process, avoidance of conflict of interest and inspection of rating
agencies by SEBI, amongst other things.

Importance of Credit Rating Agencies:

 These agencies determine the risk that is associated by investing in the companies.
 This aids in making Informed Investment Decisions. Credit Ratings give a fair estimate of the ability
of the organizations to fulfill their financial commitment.
 A high credit rating indicates a high possibility of paying back the loan.
 The Credit Rating of organizations also helps the lending institutions in deciding the loan eligibility of
the borrower.
 The increasing levels of default resulting from easy availability of finance, is another factor that has led
to the growing importance of the Credit Rating.
 Credit Rating also plays a vital role in financial markets. They assess the credit risk of the corporate or
government borrowers by analyzing the relevant information available regarding the borrower and its
economic circumstances. This analysis is reflected in Credit Rating. This rating represents an opinion
about the likelihood of meeting the financial obligations by the borrower.
 Credit Rating Agencies rates a wide range of entities, including:
▪ Industrial companies
▪ Banks

www.ibpsguise,in | mock.ibpsguide.com | www.sscexamguide.com


Copyright © IBPS Guide 36
www.ibpsguide.com
▪ Non-banking financial companies (NBFCs)
▪ Infrastructure entities
▪ Microfinance institutions xv. Insurance companies
▪ Mutual funds
▪ State governments
▪ Urban local bodies

Let us have a look at the top most credit agencies in India

CRISIL (Credit Rating Information Services of India Limited):

 CRISIL was established in 1987.


 It is India's first and major Credit Rating Agency.
 CRISIL has its headquarters in Mumbai, Maharashtra.
 It is a subsidiary of Standard & Poor‘s.
 Ashu Suyash is the current CEO of the organization.

CARE (Credit Analysis and Research Limited):

 CARE was established in 1993.


 This credit rating agency has its headquarters in Mumbai.
 CARE is promoted by Unit Trust of India or UTI, Canara Bank, IDBI and many other reputed banks
and companies dealing with financial services.
 Ratings conducted by CARE, are authenticated by Government of India.
 The Reserve Bank of India, SEBI, along with other regulatory authorities also recognizes the ratings of
CARE.
 Mr. Rajesh Mokashi is the current MD and CEO of the organization.

ICRA (Investment Information and Credit Rating Agency):

 It is an associate of Moody‘s Investors Service


 ICRA was founded in 1991.
 Mr. Naresh Takkar is the CEO of this agency.
 It has its headquarters in Gurugram.

ONICRA (Onida Individual Credit Rating Agency):

www.ibpsguise,in | mock.ibpsguide.com | www.sscexamguide.com


Copyright © IBPS Guide 37
www.ibpsguide.com
 ONICRA was founded in 1993
 The organization has its headquarters in Gurugram.
 Mr. Varun Mirchandani serves as the Executive Director of the organization.

Brickwork Ratings India Private Limited:

 Brickwork Ratings India Private Ltd was established in 2007.


 This credit rating agency has its headquarters in Bangalore, Karnataka.
 Mr. Vivek Kulkarni is the chairman of Brickwork India.

SMERA (Small and Medium Enterprises Rating Agency of India Limited):

 This credit rating agency was set up in 2005 exclusively for Micro, small and medium enterprises.
 It has its headquarters in Mumbai.
 Mr. Sankar Chakraborti is currently the CEO of the organization.

CIBIL:

 TransUnion CIBIL Limited (Formerly: Credit Information Bureau (India) Limited) is India‘s first
Credit Information Company (CIC) founded in August 2000.
 CIBIL collects and maintains records of an individual‘s payments pertaining to loans and credit cards.
These records are submitted to CIBIL by member banks and credit institutions, on a monthly basis.
 This information is then used to create Credit Information Reports (CIR) and credit scores which are
provided to credit institutions in order to help evaluate and approve loan applications.
 CIBIL was created to play a critical role in India‘s financial system, helping loan providers manage
their business.
 MD and CEO of CIBIL is Satish Pillai
 Headquarter of CIBIL is in Mumbai.

Major Credit Rating Agencies of the World

Moody's Investors Service:

 Moody‘s is founded by John Moody in 1909 to produce manual published basic statistic and general
information about stocks and bonds.
 It provides international financial research on bonds issued by commercial and government entities
 It ranks the creditworthiness of borrowers using a standardized rating scale

www.ibpsguise,in | mock.ibpsguide.com | www.sscexamguide.com


Copyright © IBPS Guide 38
www.ibpsguide.com
 According to Moody‘s rating system, rates from Aaa to C are assigned. Aaa for highest quality and C s
for lowest quality.
 The organization has its headquarters in New York, USA.
 Mr. Raymond W. McDaniel Jr serves as the CEO of the organization.

Standard & Poor's Financial Services LLC:

 Standard & Poor‘s is the world‘s leading Index provider and the foremost source of independent credit
ratings. It provides financial market intelligence to decision makers.
 It is founded by Henry Varnum Poor in 1860
 The organization has its headquarters in New York, USA.
 John L. Berisford serves as the acting President of this organization.
 It is a subsidiary of S&P Global.

Fitch Ratings:

 In 1914 Fitch was founded by John Knowles in New York City as Fitch Publishing Company.
 Later it merged with London based IBCA in 1997
 Fitch Ratings has its headquarters in New York and London.
 Paul Taylor is currently the CEO of the organization.
 Fitch Ratings is also the parent company of India Ratings & Research Private Limited that operates
from Mumbai.

FOREIGN EXCHANGE RESERVE

 Foreign exchange reserve can be defined as deposits of a foreign currency held by the central bank of a
country.
 Reserve Bank of India Act, 1934 and the Foreign Exchange Management Act, 1999 set the legal
provisions for governing the foreign exchange reserves.
 RBI is the sole authority to monitor Foreign exchange reserves.
 Reserve bank accumulates foreign currency reserves by purchasing from authorized dealers in open
market operations.
 Foreign exchange reserves of India act as a cushion against rupee volatility once global interest rates
start rising.

Need of Forex Reserve:

www.ibpsguise,in | mock.ibpsguide.com | www.sscexamguide.com


Copyright © IBPS Guide 39
www.ibpsguide.com
 Funding import like trade in different commodities in international market the trader may not accept the
local currency so to settle trade any accepted currency has to be given.
 Debt servicing to facilitate or settle borrowings in acceptable currency
 Stabilizing domestic currency ( it‘s known that exchange rate in free market is decide by foreign
currency comparison )
 Confidence building for markets
 Buffer for shocks (for emergency situation)

The Foreign exchange reserves of India consist of below four categories.

1. Foreign Currency Assets


2. Gold
3. SDR
4. Reserve Tranche Position in the IMF
 Foreign currency assets mean total foreign currency available with RBI.
 Gold explains total gold available with RBI.
 SDR is a basket of currency in which certain currencies are put in a particular ratio. Basket comprises
of Dollar, Pound, Yen, Euro, Renminbi It is an artificial unit of account created by IMF in 1969. It is
neither a claim nor a currency of IMF.
 Reserve Tranche Position is a particular percentage of SDR that a currency can use for its settlement.

FINANCIAL MARKET

A financial market is a broad term describing any marketplace where buyers and sellers participate in the trade
of assets such as equities, bonds, currencies and derivatives.

Financial markets are typically defined by having

 Transparent pricing
 Basic regulations on trading, costs and fees
 Market forces determining the prices of securities that trade.

Functions of financial Markets

1. Mobilization of saving & channelize them into more productive uses


2. Facilitate price discovery
3. Provide liquidity to financial assets
4. Reduces the cost of transaction & save time & efforts

www.ibpsguise,in | mock.ibpsguide.com | www.sscexamguide.com


Copyright © IBPS Guide 40
www.ibpsguide.com
A financial market consists of two major segments:

I. Money Market – the money market deals in short-term credit


II. Capital Market – the capital market handles the medium term and long-term credit.

MONEY MARKET

The money market is that part of a financial market which deals in the borrowing and lending of short term
loans generally for a period of less than or equal to 365 days. It meets the short term requirements of borrowers
and provides liquidity or cash to the lenders.

 It is a place where short term surplus investible funds at the disposal of financial institutions and
individuals are bid by borrowers, again comprising institutions and individuals and also by the
government.
 The Indian money market consists of Reserve Bank of India, Commercial banks, Co-operative banks,
and other specialized financial institutions. The Reserve Bank of India is the leader of the money
market in India.
 Money market does not refer to any specific market place. Rather it refers to the whole networks of
financial institutions dealing in short-term funds, which provides an outlet to lenders and a source of
supply for such funds to borrowers.
 It should be noted that money market does not deal in cash or money but simply provides a market for
credit instruments such as bills of exchange, promissory notes, commercial paper, treasury bills, etc.
These financial instruments are close substitute of money.
 Some Non-Banking Financial Companies (NBFCs) and financial institutions like LIC, GIC, UTI, etc.
also operate in the Indian money market.

Structure of Indian Money Market

www.ibpsguise,in | mock.ibpsguide.com | www.sscexamguide.com


Copyright © IBPS Guide 41
www.ibpsguide.com
Indian money market is characterized by two sectors

Organized sector: The organized sector is within the direct purview of RBI regulations.

Unorganized sector: The unorganized sector consists of indigenous bankers, money lenders, non-banking
financial institutions, etc.

Major functions of money market:

1. To maintain monetary equilibrium – It means to keep a balance between the demand for and supply of
money for short term monetary transactions.
2. To promote economic growth – Money market can do this by making funds available to various units
in the economy such as agriculture, small scale industries, etc.
3. To provide help to Trade and Industry – Money market provides adequate finance to trade and industry.
Similarly, it also provides facility of discounting bills of exchange for trade and industry.
4. To help in implementing Monetary Policy – It provides a mechanism for an effective implementation
of the monetary policy.
5. Money market provides non-inflationary sources of finance to government. It is possible by issuing
treasury bills in order to raise short loans.

Money Market Instrument:

www.ibpsguise,in | mock.ibpsguide.com | www.sscexamguide.com


Copyright © IBPS Guide 42
www.ibpsguide.com

Call Money:

 Call money is mainly used by the banks to meet their temporary requirement of cash. It is also known
as money at call and money at short notice.
 In this, market money is demanded for an extremely short period. The duration of such transactions is
from a few hours to 14 days.
 These transactions help stock brokers and dealers to fulfill their financial requirements.
 The rate at which money is made available is called as call rate.
 Rate is fixed by the market forces such as the demand for and supply of money.

Treasury Bill:

 It is a market for sale and purchase of short-term government securities.


 These securities are called as Treasury Bills, which are promissory notes or financial bills issued by the
RBI on behalf of the Government of India.
 There are two types of treasury bills:
o Ordinary or Regular Treasury Bills
o Ad Hoc Treasury Bills.
 Treasury bills are highly liquid instruments. At any time the holder of treasury bills can transfer or get
it discounted from RBI.
 The maturity period of these securities range from as low as 14 days to as high as 364 days.

www.ibpsguise,in | mock.ibpsguide.com | www.sscexamguide.com


Copyright © IBPS Guide 43
www.ibpsguide.com
 They have become very popular due to high level of safety involved in them.

Cash Management Bills:

 The Government of India, in consultation with the RBI, decided to issue a new short-term instrument,
known as Cash Management Bills (CMBs), to meet the temporary mismatches in the cash flow of the
Government.
 The CMBs have the generic character of T-bills but are issued for maturities less than 91.

Certificate of Deposits (CDs):

 The certificate of deposits is issued by the Commercial Banks


 They are worth the value of Rs. 25 lakh and in multiple of Rs. 25 lakh
 The minimum subscription of CDs should be worth Rs. 1 Crore.
 The maturity period of CD is as low as 3 months and as high as 1 year.
 These are the transferable investment instrument in a money market.
 The government initiated a market of CDs in order to widen the range of instruments in the money
market and to provide a higher flexibility to investors for investing their short term money.

Commercial Papers (CPs):

 Commercial paper (CP) is an investment instrument which can be issued by a listed company having
working capital more than or equal to Rs. 5 cr.
 The CPs can be issued in multiples of Rs. 25 Laths. However, the minimum subscription should at least
be Rs. 1 cr.
 The maturity period for the CP is a minimum of 3 months and maximum 6 months.
 Commercial paper (CP) is a popular instrument for financing working capital requirements of
companies.
 It can be issued for period ranging from 15 days to one year. Commercial papers are transferable by
endorsement and delivery.

Repurchase Agreements:

 A repurchase agreement, also known as a repo, is the sale of securities together with an agreement for
the seller to buy back the securities at a later date.
 The repurchase price should be greater than the original sale price, the difference effectively
representing interest, sometimes called the repo rate.

www.ibpsguise,in | mock.ibpsguide.com | www.sscexamguide.com


Copyright © IBPS Guide 44
www.ibpsguide.com
 The party that originally buys the securities effectively acts as a lender. The original seller is effectively
acting as a borrower, using their security as collateral for a secured cash loan at a fixed rate of interest.

CAPITAL MARKET:
 Capital Market is a market dealing in medium and long-term funds. It is an institutional arrangement
for borrowing medium and long-term funds and which provides facilities for marketing and trading of
securities
 So it constitutes all long-term borrowings from banks and financial institutions, borrowings from
foreign markets and raising capital by issuing various securities such as shares, debentures, bonds, etc.
 Both the government and private sector participate in this market for investment purposes.
 S.E.B.I. regulates the capital market in India.
 Capital market can be classified into
▪ Primary Market
▪ Secondary Market

Primary Market:

 The primary market is a market for new shares.


 The companies have to follow well defined procedures when they are auctioning their shares for the
first time. This is called Initial Public Offer.
 At this stage, the investment banks are involved in setting a price for the shares which the company is
issuing.
 The major players in the primary market are merchant bankers, mutual funds, financial institutions, and
individual investors.

Secondary markets:

 The secondary market is a market for trading of existing securities.


 The secondary market known as stock market or stock exchange plays an equally important role in
mobilizing long-term funds by providing the necessary liquidity to holdings in shares and debentures.
 It is an organized market where shares and debentures are traded regularly with high degree of
transparency and security.

Functions of the capital market:


1. Mobilization of Savings: Capital market is an important source for mobilizing idle savings from the
economy. It activates the ideal monetary resources and puts them in proper investments.
www.ibpsguise,in | mock.ibpsguide.com | www.sscexamguide.com
Copyright © IBPS Guide 45
www.ibpsguide.com
2. Capital Formation: Capital market helps in capital formation. Capital formation is net addition to the
existing stock of capital in the economy.
3. Speed up Economic Growth and Development: Capital market enhances production and productivity
in the national economy. As it makes funds available for a long period of time, the financial
requirements of business houses are met by the capital market.
4. Proper Regulation of Funds: Capital market also helps in proper allocation of these resources. It can
have regulation over the resources so that it can direct funds in a qualitative manner.
5. Continuous Availability of Funds: Capital market is place where the investment avenue is
continuously available for long-term investment. This is a liquid market as it makes funds available on
continues basis. Both, buyers and sellers can easily buy and sell securities.
Capital Market Instruments:

Government Securities (G-secs):

 Tradable instruments issued by the central government or state governments.


 They are both short term (T- bills) and long term (Dated bonds or Government bonds)
 Do not carry any risk of default and are hence called 'risk-free gilt-edged securities'.
 GOI also issues savings instruments (savings bonds, NSCs or special securities (power bonds, oil
bonds) but they are not fully tradable.
 Dated government securities are long-term and carry a fixed or floating coupon rate which is payable at
fixed time periods. The maturity of these securities can be up to 30 years.
 Dated security can be issued to any person, firm, corporate, state governments and trusts.
 Foreign companies owned by NRIs and FIIs registered with SEBI can also invest in them.
 Floating Rate Bonds carry variable interest rates with fixed percentage pegged to some benchmark rate.
 Capital Indexed Bonds carry a fixed interest rate over wholesale price index or consumer price index.
 Inflation Indexed Bonds carry interest rates linked to inflation rates (observed from WPI earlier, now
CPI) so that real return is positive from investments.
 Lately, RBI has allowed sovereign wealth funds, endowment funds, insurance and pension funds &
foreign central banks (Federal Reserve etc) to invest in G-secs if they are registered with SEBI.

Mutual Funds:

 Mutual funds raise money from the public, pool them and invest in stock market.
 They are regulated by SEBI.
 Structure of a mutual fund is as follows:

www.ibpsguise,in | mock.ibpsguide.com | www.sscexamguide.com


Copyright © IBPS Guide 46
www.ibpsguide.com
▪ Sponsor - the person who alone or in association with another organisation establishes a mutual
fund.
▪ Trust - It is registered as a trust according to provisions of Indian Trust Act, 1882 (for private trusts)
▪ Trustee -a corporate body which safeguards the interests of unit holders
▪ Custodian - A bank or a financial institution registered with SEBI which holds and safeguards the
securities owned within a mutual fund. E.g. SBI is the custodian of SBI mutual fund.

Hedge Funds:

 A pooled investment which is professionally administered by a firm.


 They invest in diverse markets and use various investment styles.
 They are comparatively more risk prone, aim at faster returns and generally avoid regulatory oversight.
 Available only to certain investors and can't be sold to general public.
 They are a form of Alternate Investment Funds (AIFs).

Alternative Investment Funds (AIFs):

 Newly created investment vehicle where investments are pooled in from real estate, hedge funds and
private equity. Capital can be pooled from both Indian and foreign investors.
 Regulated by SEBI.
 It excludes mutual funds, employee stock option and family trusts.

Venture Capital:

 Money provided by financial institutions for investment in rapidly growing companies.


 They manage the new firms alongside investing, as per terms.
 It's an important source of raising capital for start-up companies.

Angel Investors:

 An individual who provides capital for a business starts up.


 Usually, the investment is in exchange for convertible debt or ownership equity.
 They invest their own money, unlike venture capitalist who invests public money.
 AIs can register themselves as AIFs as per SEBI.
 Minimum investment should be Rs 25 lakh.
 Angel Funds are allowed to invest in overseas venture capital undertakings upto 25% of their investible
corpus in line with other AIFs.

www.ibpsguise,in | mock.ibpsguide.com | www.sscexamguide.com


Copyright © IBPS Guide 47
www.ibpsguide.com
 The lock-in requirements of investment made by Angel Funds in the venture capital undertaking is one
year.
 Angel Funds will be allowed to invest in startups incorporated within five years.

Chit Funds:

 An arrangement that a group of people arrive at to contribute money in a manner at periodic intervals
into a kitty.
 A member can withdraw that money through a lucky draw, auction or other agreed ways.
 Usually popular in Rural India, Tier2 and Tier3 cities due to under penetration of banking services.
 Chit funds are established under Central Chit Funds Act, 1982.
 An office "Registrar of Chit funds' in every state monitors their operations.

Exchange Traded Funds (ETFs):

 These are index funds listed and traded on stock exchanges.


 They comprise of a basket of stocks that has a composite index and the value of that depends on
underlying stocks.
 The major benefit is that you can invest in a diverse portfolio with the simplicity of trading a single
stock.
 ETFs are increasingly becoming a popular investment option throughout the globe.

Difference Between Money and Capital Market:

Money Market Capital Market

Money market is the place where short term Capital Market, where long term securities are
marketable securities are traded. traded is known as Capital Market.

Treasury bills, commercial papers, certificate of


Share, debentures, bonds, retained earnings are
deposits, trade credit are the instrument of money
the instrument of capital market.
market.

The money market instruments are rich in The instruments of the capital market are not that
liquidity. much liquid.

www.ibpsguise,in | mock.ibpsguide.com | www.sscexamguide.com


Copyright © IBPS Guide 48
www.ibpsguide.com

Money Market instruments gives lower returns as Capital market instruments gives higher returns
compared to capital market instruments. as compared to money market instruments.

The instrument traded in money market carry low


The capital market instruments carry high risk.
risk.

Money Market is unsystematic market and so the


trading is done off exchange, i.e. Over The It is broadly divided into two major categories:
Counter (OTC) between two parties by Primary Market and Secondary Market.
usingphones, email, fax, online, etc.

Redemption of money market instruments is Capital market instruments have a life of more
done with in a year. than a year.

Money Market in India is regulated by Reserve Capital Market in India is regulated by Securities
Bank of India. Exchange Board of India

SMALL FINANCE BANK AND PAYMENTS BANK

Small Finance Bank:

Small Finance banks are physical banks whose aim is to provide basic banking products such as deposits and
supply of credit but in a limited area of operation. Their work is to supply credit to small farmers, micro and
small industries, and other unorganized sector entities through high technology and also low cost operations.

Important Points about Small Finance Bank:

 Small Finance Banks are a type of niche banks.


 Usha Thorat is the head of Small Finance bank committee.
 NBFC, Micro- Finance Institutions and Local Area Banks who has 10 years of experience in banking
can apply for Small Finance Bank license.
 Small Finance Banks are licensed under section 22 of Banking Regulation Act 1949 and registered as
Public Limited Company under the Companies Act 2013.

www.ibpsguise,in | mock.ibpsguide.com | www.sscexamguide.com


Copyright © IBPS Guide 49
www.ibpsguide.com
 The bank shall primarily undertake basic banking activities of accepting deposits and lending to small
farmers, small businesses, micro and small industries, and unorganized sector entities
 Their primary targets are Small Business and MSMEs. They are not allowed to lend the deposited
money to big business or industries.

Condition for setting up Small Finance Bank:

 The minimum paid-up equity capital for small finance banks shall be Rs. 100 crore.
 Every small finance bank will be required to use the words ―Small Finance Bank‖ in its name in order
to differentiate it from other banks.
 Maximum loan size and investment limit exposure to single person is 10% and for group would be
restricted to 15% of capital funds.
 Annual branch expansion plans should be in compliance with the requirement of opening at least 25%
of its branches in unbanked rural centers.
 75% of its Adjusted Net Bank Credit (ANBC) should be advanced to the priority sector as categorized
by RBI.
 At least 50% of its loan portfolio should constitute loans and advances of up to Rs.25 lakh to micro
finance business.
 For the first three years, 25% of branches should be in unbanked rural areas.
 Minimum paid-up equity capital is 40% and it will be gradually brought down to 26% within 12 years
from date of commencement.
 Small banks can undertake financial services like distribution of Mutual Fund units, Insurance
products, Pension products and so on but not without prior approval from RBI.
 It cannot set up subsidiaries to undertake non-banking financial services activities.

Payment Banks:

Payment Banks are banks which will reach their customers through mobile phones rather than traditional bank
branches. They can be thought of as mobile wallets; however they can also have physical branches.

The objectives of payments banks will be to further financial inclusion by providing small savings accounts
and payments or remittance services to migrant labor workforce, low income households, small businesses,
other unorganized sector entities and other users.

Important Points about Payments Banks:

www.ibpsguise,in | mock.ibpsguide.com | www.sscexamguide.com


Copyright © IBPS Guide 50
www.ibpsguide.com
 A Payments bank is a type of non-full service niche bank in India.
 RBI formed a Payments Bank Committee on Comprehensive Financial Services for Small Businesses
and Low Income Households under the chairmanship of Nachiket Mor.
 Payments Banks are licensed under section 22 of Banking Regulation Act 1949 and registered as Public
Limited Company under the Companies Act 2013.
 Till now Airtel Payments Bank, Paytm Payments Bank and India Post Payment Bank started their
business.

Conditions for setting up Payments Banks:

 The minimum capital requirement to establish a payments bank is Rs. 100 cr


 They can offer only savings and current account in which deposit only up to Rs 1 lakh per customer is
permitted.
 They cannot lend money to people and also cannot issue credit cards. Though ATM or debit card can
be issued.
 A payments bank will need to invest 75% of its funds in government securities or treasury bills with
maturity up to one year.
 It can hold maximum 25% in current and fixed deposits with other scheduled commercial banks for
operations and liquidity management.
 The minimum initial contribution to the paid-up equity capital should be 40% for the first five years
from the commencement of business.
 The Payments Banks would be required to use the word ‗Payments‘ in its name to differentiate it from
other banks.
 They cannot accept NRI deposit. The revenue of these banks would come mainly from the transaction
fees.

PAYMENT AND SETTLEMENT SYSTEMS ACT, 2007

The Payment and Settlement Systems Act, 2007 (―PSS Act‖) empowers the Reserve Bank of India to
regulate and oversee all payment and settlement systems in the country and also to provide settlement finality
and a sound legal basis for the same.

The Act came into effect on 12th August 2008 vide a notification to that effect.

www.ibpsguise,in | mock.ibpsguide.com | www.sscexamguide.com


Copyright © IBPS Guide 51
www.ibpsguide.com
The PSS Act specifies that no person, other than the RBI, can operate a payment system except with due
authorization issued by the RBI (unless specifically exempted by the terms of the PSS Act itself.).

The Act provides for netting and settlement finality and gives formal oversight powers over all payment and
settlement systems with the RBI.

In the brief, the Act:

 Anoints the RBI as the authority that regulates payment and settlement systems;
 Makes it compulsory to obtain RBI authorization to operate a payment system;
 Warrants the RBI to regulate and supervise payment systems by determining standards and calling for
information, regular reports, documents etc;
 Warrants the RBI to audit and conduct on- and off-site inspections of the payment systems;
 Warrants the RBI to issue directives; and
 Provides for netting and settlement to be final and irrevocable.

Under this act two regulations have been made by RBI, One is, Board for Regulation and Supervision of
Payment and Settlement Systems (BPSS) 2008. This committee is formed by the central board of directors
of RBI. It deals with exercising its powers, the constitution of subcommittees and advisory committees for
payment and settlement related matters.

Another one is Payment and Settlement Systems Regulations, 2008. It deals the issues like the form of
application for authorization for commencing on a payment system and grant of authorization, payment
systems, furnishing of returns, documents, the furnishing of accounts and balance sheets by systems provider
etc.
In India payment system can be classified into three types

I. Paper Based Payment System


II. Electronic Payment System
III. Other Payment System

I. Paper Based Payment System:

Paper Based Payment System comprises of cheque and Demand draft.

www.ibpsguise,in | mock.ibpsguide.com | www.sscexamguide.com


Copyright © IBPS Guide 52
www.ibpsguide.com
Cheque:

A cheque is a document that orders a bank to pay a specific amount of money to the person in whose name the
cheque has been issued. Cheque is used to make safe and convenient payment. Cheque is a financial
instrument which can be transferred to another party by simply endorsing it.

After opening an account in a bank, with cheque book facility, the bank you will provide you with a cheque
book. However, there are various kinds of cheque book you receive and it would depend on the type of account
you have. The number of cheques in a cheque book also differs depending on the account. Charges also vary
depending on the type of account, like current account, where more cheque leaves are provided as compared to
individual savings account.

Parts of a Cheque:

Drawer: Maker or writer of a bill of exchange (cheque)

Drawee: The party that has been directed by the depositor to pay certain sum of money to the person

Payee: A person to whom money is paid or is to be paid

History of Cheques In India:

 The Cheque was introduced in India by the Bank of Hindustan. In 1881, the Negotiable Instruments
Act was enacted in India.

www.ibpsguise,in | mock.ibpsguide.com | www.sscexamguide.com


Copyright © IBPS Guide 53
www.ibpsguide.com
 The NI Act provided a legal framework for non-cash paper payment instruments in India.
 In 1938, the Calcutta Clearing Banks‘ Association, which was the largest bankers‘ association at that
time, adopted clearing house.
 Until 1 April 2012, cheques in India were valid for a period of six months from the date of their issue,
before the Reserve Bank of India issued a notification reducing their validity to three months from the
date of issue.

Different Types of Cheque:

1) Open Cheque Or Bearer Cheque:

 When the words "or bearer" printed on the cheque is not cancelled, the cheque is called a bearer
cheque.
 A bearer cheque is made payable to the bearer i.e. it is payable to the person who presents it to the bank
for encashment.
 However, such cheques are risky, this is because if such cheques are lost, the finder of the cheque can
collect payment from the bank.
 Bearer cheque can be transferred by mere delivery; they need no endorsement. In simple words a
cheque which is payable to any person who presents it for payment at the bank counter is called ‗Bearer
cheque‘.

2) Order Cheque:

 When the word "or bearer" printed on the cheque is cancelled and the word ‗order‘ may be written on
the cheque, the cheque is called an order cheque.
 An order cheque is one which is payable to a particular person.
 The payee can transfer an order cheque to someone else by signing his or her name on the back of it.

www.ibpsguise,in | mock.ibpsguide.com | www.sscexamguide.com


Copyright © IBPS Guide 54
www.ibpsguide.com

3) Crossed Cheque:

 Crossed cheque means drawing two parallel lines on the left corner of the cheque with or without
additional words like ―A/c Payee only‖ or ―&co‖ or ―Not Negotiable‖.
 A crossed cheque cannot be encashed at the cash counter of a bank but it can only b credited to the
payee‘s account.
 This is a safer way of transferring money then an Uncrossed or open cheque because we can find to
which account the money has been transferred.

www.ibpsguise,in | mock.ibpsguide.com | www.sscexamguide.com


Copyright © IBPS Guide 55
www.ibpsguide.com

4) Post Dated Cheque (PDCs):

 Post Dated Cheque‘s are cheques issued with future date on it.
 The cheque issued today will be vailed for three months from the date of issue.
 It is used for business purposes or making of payment in future date.
 For example; on 11/6/2017 you are issuing it dated 15/9/2017 than it will called post dated cheque and
will be vailed for three months from 15/9/2017.

5) Anti Dated Cheque:

 If a cheque bears a date earlier than the date on which it is presented to the bank, it is called as ―anti-
dated cheque‖.
 Such a cheque is valid upto three months from the date of the cheque.
 For example on 11/6/2017 you are issuing a cheque dated 1/6/2017 than it will called anti dated cheque
and will be vailed for three months form 1/1/2015.

6) Stale Cheque:

If a cheque is presented for payment after three months from the date of the cheque, it is called stale cheque. A
stale cheque is not honoured by the bank.

7) Gift Cheque:

 Gift cheque, it is a cheque forirted in decorative form issued for a small extra charge by the banks for
use by customers who wish to give presents of money on special occasions.
 Gift cheques may be purchased in unlimited numbers from every branch of the ‗X‘ Bank.

8) Traveller‘s Cheque:

 It is an instrument issued by a bank for remittance of money from one place to another.
 Travelers Cheques are accepted almost everywhere and are available in many denominations. There is
no-expiration in this type.

www.ibpsguise,in | mock.ibpsguide.com | www.sscexamguide.com


Copyright © IBPS Guide 56
www.ibpsguide.com
 Generally used by people on vacation in foreign countries instead of cash, as many businesses used to
accept traveler‘s cheques as currency.

9) Multilated Cheque:

 If a cheque is torn into two or more pieces such cheque is Mutilated Cheque.
 If it presented for payment, such a cheque the bank will not make payment against such a cheque
without getting confirmation of the drawer.
 In case, if a cheque is torn at the corners and no material fact is erased or cancelled, the bank may make
payment against such a cheque.

10) Blank Cheque:

 A cheque on which the drawer puts his signature and leaves all other columns blank is called a blank
cheque.
 A check that is signed by the payer but with no specific amount indicated, leaving this determination up
to the drawee.
 More generally, a term used for any situation in which an usually high level of trust is afforded by one
party to another.

11) Self Cheque:

 Self cheque means the cheque payable to the drawer himself.


 Normally, when the drawer is willing to get cash from the bank, they used to write the cheque favoring
―Self‖ and after tendering the cheque to the teller they will get payment for the cheque.
 Suppose you are having two different accounts in the same branch, you can issue a cheque from one
account favoring ―Self‖ and deposit the cheque in another account in your name and it is transfer of
funds from one account to another account by means of cheque.
 Suppose you are having one account with ABC bank and another account with XYZ bank, you can
issue a cheque drawn on ABC bank favoring ―Self‖(means your name) and can deposit the cheque in
the account with XYZ bank.
 Normally, bearer cheques will not create any issue; however, in the case of order cheques, you have to
establish your identity to XYZ bank.
 For example,

www.ibpsguise,in | mock.ibpsguide.com | www.sscexamguide.com


Copyright © IBPS Guide 57
www.ibpsguide.com

Demand Draft (DD):

 It is kind of a pre-paid negotiable instrument that is used to direct payments from one bank to another
bank or one of its own branches to pay a certain sum to the specified party.
 Demand drafts can only be made payable to a specified party, also known as pay to order.
 When a bank gets request for the issue of a DD by any individual or party, it either deducts the money
from the bank account (if the individual/party has bank account in that bank) or individual/party has to
give the amount in cash not exceeding Rs 50,000. In case of amount exceeding Rs 50,000, the payment
is to be made by cheque along with giving the PAN No.
 Like a cheque, DD also contains DD number and MICR No. at the bottom. DDs can also be used for
making payments abroad by issuing a draft in foreign currency.

www.ibpsguise,in | mock.ibpsguide.com | www.sscexamguide.com


Copyright © IBPS Guide 58
www.ibpsguide.com
II. Electronic Payment System:

In 1990 RBI took initiative for the electronic clearing service in order to enhance better payment and
settlement system in India.

Electronic Payment System in India can be classified into two types such as Gross Settlement System and Net
Settlement System.

RTGS (Real Time Gross Settlement System):

 Reserve Bank of India introduced the RTGS System in March 2004 with four bank branches on a pilot
basis, only for inter-bank transactions.
 RTGS is a real time and gross settlement system. Real Time means settlement of the transaction is start
at the time it received.
 Gross Settlement means transactions are individually processed. No other transaction can bunch with
other.
 The transaction is recorded in the books of RBI so it is final and irrevocable transaction.
 In RTGS, there is minimum limit for the transfer of money is Rs 200,000 (Rs 2 lakh) and there is no
maximum limit for the transfer of money.
 Customers can avail facility of RTGS between 9:00 am to 4:30 pm on weekdays and on Saturdays from
9:00 am to 2:00 pm
 In order to enter into RTGS transactions the bank must have CBS (Core Banking Solution) which is
assigned to enable bank and branches.

NEFT (National Electronic Fund Transfer):

 NEFT is introduced by RBI in 2005, it enables individuals, firms and corporate to transfer funds from
any bank branch to any individual, firm or corporate.
 In NEFT, transfer is done on the half - hourly basis. There are twenty three settlements from 8 am to 7
pm on weekdays and eleven settlements from 8 am to 1 pm on Saturdays.
 There is no maximum and minimum limit in NEFT for the transfer of funds in India.
 In order to enter into the RTGS the bank must be enable for NEFT.

National Payments Corporation Of India:

 National Payments Corporation of India has the important role to improve electronic payment system in
India. It introduced lot of products to develop digital payment in India.
 NPCI was founded in 2008 under section 8 of Companies Act 2013.

www.ibpsguise,in | mock.ibpsguide.com | www.sscexamguide.com


Copyright © IBPS Guide 59
www.ibpsguide.com
 NPCI is a not-for-profit organization owned by a consortium of major banks and has been promoted by
the country‘s central bank, the Reserve Bank of India.
 NPCI is a kind of platform that facilitates all electronic retail transactions involving money in India
(currently) and provides solutions to the economic and monetary problems through Direct Cash
Transfer, Financial Inclusion, Subsidy Allocation, One Access Point Mechanism etc.
 Just to take an example of Visa. Now it works as the most popular way to carry and transfer money
around the globe, which can be trusted by nearly all financial institutes.
 Or take for instance payment linking mechanisms of the world like- China Union Pay in China,
Vocalink in Uk, Bankserv in South Africa. NPCI seeks to operate as a service that can provide similar
benefits to Indian people.

The products of National Payments Corporation of India are,

1. National Financial Switch (NFS) - network of shared automated teller machines in India.
2. Immediate Payment Service (IMPS) - Real time payment with mobile number.
3. Unified Payment Interface (UPI) - Single mobile application for accessing different bank accounts
4. BHIM App - Smartphone app built using UPI interface.
5. *99# - mobile banking using USSD
6. National Automated Clearing House (NACH)- a web based solution to facilitate interbank, high
volume, electronic transactions.
7. Cheque Truncation System - online image-based cheque clearing system
8. Aadhaar Payments Bridge System (APBS) - Aadhar based payment solution
9. RuPay - card scheme
10. Bharat Bill Payment System (BBPS) - integrated bill payment system.

1. National Financial Switch (NFS):

 National Financial Switch was initiated by Institute of Development and Research in Banking
Technology and handed over to NPCI in 2009.
 NFS consists of a national switch to facilitate connectivity between bank‘s switches and their ATMs,
and inter-bank payment gateway for authentication and routing the payment details of various e-
commerce transactions.
 NFS is India‘s largest ATM connecting network.
 Any bank having a core banking solution system can be a part of NFS.

www.ibpsguise,in | mock.ibpsguide.com | www.sscexamguide.com


Copyright © IBPS Guide 60
www.ibpsguide.com
 Customers of member banks can withdraw money from any of these banks without incurring any extra
cost.
 Main purpose of NFS initially was to include rural and cooperative banks under its umbrella, but now
any bank can join this network for maximizing its reach.

2. Immediate Payment Service (IMPS):

IMPS is a tool through which one can transfer money instantly within banks across India through mobile,
internet and ATMs which is not only safe but also economical both in financial and non-financial perspectives.

This facility is available 24x7x365 and it is launched by National Payments Corporation of India on 22nd
November 2010.

Before IMPS system, the transactions could be done either by NEFT or by RTGS. But it creates lot of
inconvenience to the customer because of its working hours.

So NPCI along with some banks like SBI, BOI, UBI and ICICI in 2010 launched IMPS in Mumbai.

The participants for IMPS are:

▪ Remitter (Sender)
▪ Beneficiary (Receiver)
▪ Banks
▪ National Financial Switch by NPCI

Objectives of IMPS

 It is customer friendly so that customers do not have to wait for tomorrow to make remittances.
 Can make the payment simpler just with the use of mobile number.
 Can achieve digitization in doing retail payments.
 Can build the foundation for a full range of mobile based Banking services.

Some important facts:

 The bank should have an approval from RBI for Mobile Banking Service to be eligible to participate in
IMPS.
 Customer should do Mobile Banking Registration if he wants to transact through mobile.
 The customer gets a unique Mobile Money Identifier (MMID) which is one of the inputs to start the
transaction. It is a 7 digit number issued by banks.
 Every mobile phone be it a basic phone or Smartphone is eligible for IMPS.
www.ibpsguise,in | mock.ibpsguide.com | www.sscexamguide.com
Copyright © IBPS Guide 61
www.ibpsguide.com
 There is no need of bank account to avail IMPS.
 More than one account can be linked to single mobile number.
 The recipient is not required to register for IMPS.
 Individual banks can also charge money for IMPS as per bank policy.

3. Unified Payment Interface (UPI):

Unified Payments Interface (UPI) is a payment system launched by National Payments Corporation of India
and regulated by the Reserve Bank of India which facilitates the instant fund transfer between two bank
accounts on the mobile platform.

UPI is built over Immediate Payment Service (IMPS) for transferring funds using

1. Virtual Payment Address (a unique ID provided by the bank),


2. Account Number with IFS Code,
3. Mobile Number with MMID (Mobile Money Identifier),
4. Aadhaar Number, or a one-time use Virtual ID.

An MPIN (Mobile banking Personal Identification number) is required to confirm each payment which will be
a standardized 4- or 6-digit number similar to an ATM PIN.

Some important facts about UPI:

 UPI works on single click 2 factor authentication.


 Two-factor authentication is a security mechanism that requires two types of credentials for
authentication and is designed to provide an additional layer of validation, minimizing security
breaches. Two-factor authentication is also known as strong authentication.
 Before UPI launched, the customer who has more than on account need to download several banking
apps. For example if he has account in Indian, Axis and ICICI bank he need to download three different
app for his purpose.
 But in UPI interface the customer needs to download UPI app alone and its network connect all the
banks together and make it possible for the customer to make his/her transactions using a single app.
So, through the use of one app, one can perform all payment transactions.
 The limit on transaction through UPI system is Rs 1 lakh.
 Charges can be applied depending on the amount of money transferred.
 The service is available 24×7×365.
 Merchant will not be able to track even the account number.

www.ibpsguise,in | mock.ibpsguide.com | www.sscexamguide.com


Copyright © IBPS Guide 62
www.ibpsguide.com
 UPI system will not at all include the wallets.

4. BHIM App:

 BHIM (Bharat Interface for Money) is a Mobile App developed by National Payments Corporation of
India (NPCI).
 It was launched by Narendra Modi, the Prime Minister of India, at a Digi Dhan mela at Talkatora
Stadium in New Delhi on 30 December 2016.
 It has been named after Dr. Bhimrao R. Ambedkar.
 The app is built over the Immediate Payment Service infrastructure and allows the user to instantly
transfer money between the bank accounts of any two parties.
 It can be used on all mobile devices.
 The app is based on the Unified Payments Interface (UPI).
 It is an open platform and not a mobile wallet. It means unlike mobile wallets (Paytm, MobiKwik,
mPesa, Airtel Money etc) which hold money, the BHIM app is only a transfer mechanism, which
transfers money between different bank accounts.
 We can send or receive money to different bank accounts with ZERO Transaction charges by using
a) UPI payment addresses by dialing *99#
b) Account number with IFSC (Indian Financial System Code) code or MMID (Mobile Money
Identifier) Code
c) QR (Quick Response) code for a fixed amount of money
d) ‗Pay to Aadhaar Number‘ – can transfer money to the Aadhaar number linked with
beneficiaries‘ bank account.
 BHIM allow users to send or receive money to or from UPI payment addresses, or to non-UPI based
accounts (by scanning a QR code with account number and IFSC code or MMID Code)
 Transactions on BHIM are nearly instantaneous and can be done 24/7 including weekends and bank
holidays.
 BHIM also allows users to check the current balance in their bank accounts and to choose which
account to use for conducting transactions, although only one can be active at any time.
 Users can create their own QR code for a fixed amount of money, which is helpful in merchant —
seller — buyer transactions. Users can also have more than one payment address.
 If the 12-digit Aadhaar number is listed as a payment ID, the BHIM app will not require any biometric
authentication or prior registration with the bank or Unified Payment Interface (UPI).
 Transaction limit per transaction would not exceed Rs.10,000 and per day limit is Rs. 20,000.

www.ibpsguise,in | mock.ibpsguide.com | www.sscexamguide.com


Copyright © IBPS Guide 63
www.ibpsguide.com
5. *99#:

One of the innovative payment service launched by NPCI includes *99# service, which works on Unstructured
Supplementary Service Data (USSD) channel.

This service was launched envisioning the potential of Mobile Banking and the need for immediate low value
remittances which will help in financial deepening and inclusion of under banked society in the mainstream
banking services.

*99# service was dedicated to the nation by the Honorable Prime Minister of India Shri Narendra Modi on
28th August 2014 as part of Pradhan Mantri Jan Dhan Yojana (PMJDY).

Features of *99# Service:

▪ Works without Internet


▪ Round the clock availability (works even on holidays)
▪ Accessible through a common code *99# across all TSPs
▪ Works across all GSM service providers and mobile handsets
▪ Additional channel for banking and key catalyst for financial inclusion
▪ Service also offered through BC Micro ATMs to serve the rural populace

6. National Automated Clearing House (NACH):

Started by the National Payments Corporation of India (NPCI), NACH aims to create a better option for
facilitating clearing services than the existing Electronic Clearing Service (ECS) system.

NACH is a centralized, web-based clearing service that can ease the work of banks, financial institutions, the
government and the corporates by consolidating all regional ECS systems into one national payment system,
thereby removing any geographical barriers in efficient banking.

The service is now active in all Indian banks with core banking facility. It comes in two variants

i. ECS Credit - which involves distribution of salary, pensions, dividends, interest, etc to
the relevant stakeholders at set frequency and periods
ii. ECS Debit – which makes regular fixed payments towards insurance premiums, loan
repayments, recurring deposits, etc.

The process of activation of ECS mandates had a longer turnaround time (30 days) than what it is expected to
be in NACH (10 days). Also, the Aadhar-based benefit transfers have been simplified.

www.ibpsguise,in | mock.ibpsguide.com | www.sscexamguide.com


Copyright © IBPS Guide 64
www.ibpsguide.com
There are three types of electronic clearing services:

▪ Local ECS
▪ Regional ECS
▪ ECS

Some facts about NACH:

 From 1st May, 2016 NACH replaced ECS.


 Those who are using ECS already can use ECS till their validity expires. After this, one will have to fill
NACH forms instead of ECS forms.
 NACH forms can be availed from AMC (Asset Management Company) offices or their websites.
 The structure of NACH allows member banks to design their own products to ensure the needs of their
retain customers, corporate clients and Government.

Some of the benefits of NACH scheme:

 NACH Credit and Debit schemes can be initiated by institutions/individuals who apply for it.
 There is no need to go to bank branches/ collection centers after all formalities are completed while
applying.
 Freedom from paper handling such as cheques as all credits and debits are done electronically.
 So it avoids the loss and fraudulent of papers in transit.
 NACH is very cost effective.
 This system eliminates the local barriers and facilitates same day transactions anywhere in India.
 Customers need not keep track of due date for payments.

7. Cheque Truncation System (CTS):

Cheque truncation is a system between clearing and settlement of cheques based on electronic images. This
form of clearing does not involve any physical exchange of cheques.

In order to ensure speedy clearance of cheques ―Cheque Truncation System‖ was introduced in the year 2008
and it is defined under the section 6 of the Negotiable Instruments Act 1881.

Though MICR technology helped improve efficiency in cheque handling, clearing is not very speedy as
cheques have to be physically transported from the collecting branch of a bank to the drawee bank branch.

www.ibpsguise,in | mock.ibpsguide.com | www.sscexamguide.com


Copyright © IBPS Guide 65
www.ibpsguide.com
Under CTS, instead of physical movement of the cheque, an electronic image of the cheque is transmitted to
the drawee branch / bank. Along with the electronic image, certain key relevant information is also transmitted,
such as date of presentation, presenting bank details, data on the MICR band.

Features of cheques issued under CTS:

(a) Details of the bank and its logo: The bank details and its logo are printed on the face of the cheque.
However, it is printed in invisible ink.
(b) VOID pantograph: This is a wavelike design, which is visible to the naked eye and seen below the area
where the account number is printed.
(c) Cheque printer details: This is printed on the extreme left hand side of the cheque. The printer details
along with the words ‗CTS-2010‘ is mentioned along the area where you tear off the leaf from the
cheque book.
(d) Rupee symbol: The new symbol of the Indian rupee is printed beside the area where the amount in
figures needs to be written.
(e) Signature space indicator: The words ‗please sign above‘ are mentioned indicating the space where you
will need to sign the cheque.

Benefits of Cheque Truncation System:

 The CTS is more advanced and more secure.


 It provides faster clearance of cheques as local cheques are cleared on the same day as the cheque is
presented to the clearing house, while intercity cheques is cleared the next day.
 It reduces reconciliation and clearing frauds.

www.ibpsguise,in | mock.ibpsguide.com | www.sscexamguide.com


Copyright © IBPS Guide 66
www.ibpsguide.com
 It provides no geographical restrictions.
 It reduces operational risk and risks associated with paper clearing.
 There are no extra charges levied for collection of cheques drawn on a bank located within the grid.

8. AEPS (Adhaar Enabled Payment System):

The Adhaar-enabled payment systems (AEPS) developed by NPCL is a bank-led model that facilitates
financial inclusion by allowing transactions at Point of Sale or PoS (the micro ATM) through the business
correspondent (BC) using the Aadhaar authentication number.

The four Aadhaar-enabled basic types of banking transactions are

 balance enquiry
 cash withdrawal
 cash deposit
 Aadhaar to Aadhaar funds transfer

Now all that a customer needs for availing of the AEPS services are an individual identification number
(identifying the bank to which the customer is associated), an Aadhaar number and fingerprints captured
during his/her enrolment.

The objective of AEPS is,

 To empower a bank customer to use Aadhaar to access his/her Aadhaar-enabled bank account and
perform basic banking transactions that are intra-bank or interbank in nature through a business
correspondent.
 It serves another important goal of RBI in electronification of retail payments.
 It would enable banks to route the Aadhaar-initiated interbank transactions through a central
switching and clearing agency.
 It would facilitate disbursements of government entitlements like NREGA, social security pension,
handicapped old age pension, etc, of any central or state government bodies, using Aadhaar and
authentication thereof as supported by UIDAI.
 Another important goal AEPS serves is to facilitate inter-operability across banks in a safe and
secured manner.

9. RuPay Cards:

www.ibpsguise,in | mock.ibpsguide.com | www.sscexamguide.com


Copyright © IBPS Guide 67
www.ibpsguide.com
RuPay is the Indian domestic card payment network being set up by National Payments Corporation of India
(NPCI) at the behest of banks in India.

This project had been conceived by Indian Banks Association and has the approval of Reserve Bank of India.
The objectives to be fulfilled are:

 Reduce overall transaction cost for the banks in India by introducing competition to international card
schemes.
 Develop products appropriate for the country particularly for financial inclusion.
 Provide card payment service option to many banks that are currently not eligible for card issuance
under the eligibility criteria of international card schemes.
 Build environment whereby payment information of the country remains within the country
 Shift Personal Consumption Expenditure (PCE) from cash to electronic payments in a growing
economy with a population of 1.2 billion

The need for a domestic payment card the ―RuPay‖ card is on account of two factors:

(a) The high cost borne by the Indian banks for affiliation with international card associations in the
absence of a domestic price setter and

(b) The connection with international card associations resulting in the need for routing even domestic
transactions, which account for more than 90% of the total, through a switch located outside the
country.

NPCI has since been granted approval to launch the ―RuPay‖ affiliated cards for use at ATMs and Micro
ATMs.

NPCI has been advised to ensure that the use of these cards under the Aadhaar Enabled Payment System
(AEPS) is in strict compliance with the DBOD (Department of Banking Operations & Development)
guidelines on business correspondents (BCs).

RuPay will compete with Visa and MasterCard in terms of cost and quality of service, and it is only natural to
expect the incumbents also to reduce their rates.

It is imperative for India to create a low-cost electronic payment system if the ongoing Endeavour to overhaul
the existing system of welfare payments and subsidies is to succeed.

The unique identity project will deliver a unique numerical tag for every Indian resident, which can then be
used to create unique electronic bank accounts.

www.ibpsguise,in | mock.ibpsguide.com | www.sscexamguide.com


Copyright © IBPS Guide 68
www.ibpsguide.com
Transferring welfare and other payments from government treasuries directly to beneficiary accounts would be
possible. Thereafter, if the beneficiary can use electronic cards to spend the money in his account, it would
remove one more layer of administrative mediation with its potential for corruption and leakage.

10. Bharat Bill Payment System (BBPS):

Bharat Bill Payment System is an integrated bill payment system in India offering interoperable and accessible
bill payment service to customers through a network of agents, enabling multiple payment modes, and
providing instant confirmation of payment.

Bharat Bill Payment System (BBPS) was proposed by G. Padmanabhan committee in 2013.

National Payment Corporation of India (NPCI) had been identified to act as Bharat Bill Payment Central Unit
(BBPCU) which will be a single authorized entity for operating the BBPS.

The BBPCU will set necessary operational, technical and business standards for the entire system and its
participants, and also undertake clearing and settlement activities. It will work only as a medium to connect
multiple billers and agents through various Operating Units.

Banks and non-bank entities presently engaged in any of the bill payment activities falling under the scope of
BBPS can apply for approval from RBI under the Payment and Settlement Systems (PSS) Act 2007.

Some important facts about BBPS:

 The biggest advantage is that the bill can be paid anywhere and anytime.
 The system will provide multiple payment modes and instant confirmation of payment.
 Payments may be made through the BBPS using cash, transfer cheques, and electronic modes.
 Retail points will be set up for bill payments across the country who would be able to accept all kinds
of bills payments made through credit cards, debit cards, mobile wallets, net banking (IMPS, NEFT).
 The BBPS outlets would include banks, ATMs, business correspondents, kiosks etc.
 Payments would be made securely through the NPCI network with instant receipts getting generated.
 PayU India is the first non-bank entity which got RBI approval to set up and operate Bharat Bill
Payment System (BBPS).

Iii. Other Payment Systems:

Automated Teller Machine – ATM:

An electronic banking outlet, which allows customers to complete basic transactions, withdrawal of money,
depositing money and checking of one‘s own balances etc. without the aid of a branch representative or teller.
www.ibpsguise,in | mock.ibpsguide.com | www.sscexamguide.com
Copyright © IBPS Guide 69
www.ibpsguide.com
There are two primary types of ATMs

1. The basic units allow the customer to only withdraw cash and receive a report of the account's
balance.

2. The more complex machines will accept deposits, facilitate credit card payments and report account
information.

ATM does most of the functions of cashier in the bank. ATM is operated by plastic card issued by the bank
which is called as ATM Card, with its special features.

Advantages of ATM:

1. Round the Clock Services: ATM provides banking services to its customers round the clock, 24 hours a day,
7 days a week and 365 days a year.

2. Access to bank from any part of the world: Essential banking services like deposits, withdrawals transfer of
funds, etc can be accessed by customers from any part of the world.

3. Expansion of Services to any corner of the world: Of the Banks can expand their services to any corner of
the world by providing electronic access to its customers.

4. Reduction in cost of operation: This reduces human intervention and thereby reduces the cost of operations
and increases profitability of banks.

5. For shopping Purpose: Now days almost every shopping mall, restaurant and other organizations are
accepting credit card payments.

Types of ATM:

White Label ATM:

White Label ATMs are those ATMs which set up, owned and operated by non-bank entities. To aid financial
inclusion and drive ATM penetration in the country the Reserve Bank of India has permitted the launch of
White Labeled ATMs (WLAs) i.e. private non-bank companies to set up, own and operate its own brand of
ATMs in the country. These white label ATMs will not display logo of any particular bank. TATA launched
the first white label ATM in India under the brand name of Indicash.

Brown Label ATM:

www.ibpsguise,in | mock.ibpsguide.com | www.sscexamguide.com


Copyright © IBPS Guide 70
www.ibpsguide.com
These ATMs are owned and maintained by service provider whereas a sponsor bank whose brand is used on
ATM takes care of cash management and network connectivity.

Onsite ATM:

These are ATM machines that are set up in the premises where there is a bank branch so that both the physical
branch and the ATM can be used. This is known as being on site and this can be used for several purposes.
Many people can use this to avoid the lines that are present in the branch and hence save on the time required
to complete their transactions.

Offsite ATMs:

These are the machines that are set up on a standalone basis. This means that the bank has a place where there
is only an ATM machine then this becomes an offsite ATM. This is done to ensure that the bank reaches out to
more geographical areas and that people are able to use its services even when there is no bank branch in the
area.

Cash Dispenser: Allows only cash withdrawals, balance enquiry and mini statement requests, cash dispenser
(CD).

Worksite ATM: It is located within the premises of an organization and is generally meant only for the
employees of the organization.

Mobile ATM: It refers to an ATM that moves in various areas for the customers. Few priavte banks have
introduced ATM on wheels.

Green Label ATM: Provided for Agricultural Transaction.

Orange Label ATM: Provided for Share Transactions.

Yellow Label ATM: Provided for E-commerce.

Pink label ATM: Women banking.

Point of Sale Terminal:

www.ibpsguise,in | mock.ibpsguide.com | www.sscexamguide.com


Copyright © IBPS Guide 71
www.ibpsguide.com

A point of sale terminal (POS terminal) is an electronic device used to process card payments at retail
locations. A POS terminal generally does the following:

 Reads the information off a customer‘s credit or debit card


 Checks whether the funds in a customer‘s bank account are sufficient
 Transfers the funds from the customer‘s account to the seller‘s account (or at least, accounts for the
transfer with the credit card network)
 Records the transaction and prints a receipt

Mobile Wallet:

 It‘s a mobile-based virtual wallet, where you preload a certain amount in your account created with the
mobile wallet service provider, and spend it at online and offline merchants listed with the mobile
wallet service provider.
 For example, if you go to a coffee shop A, which is listed with XYZ mobile wallet, you can pay for
your coffee through the phone. Depending on the service provider, you can also pay through app, text
message, social media account or website.
 There are four types of mobile wallets in India
▪ open
▪ semi-open
▪ closed
▪ semi-closed
 Open wallets are the ones that allow you to buy goods and services, withdraw cash at ATMs or banks
and transfer funds. These kinds of wallets only issued by banks. M-Pesa by Vodafone is one such
example. Apart from the usual merchant payments, it also allows you to send money to any mobile
number bank account.
 Airtel Money is a semi-open wallet, which allows you to transact with merchants that have a contract
with Airtel. You can't withdraw cash or get it back. You'll have to spend what you load.

www.ibpsguise,in | mock.ibpsguide.com | www.sscexamguide.com


Copyright © IBPS Guide 72
www.ibpsguide.com
 Closed wallets are issued by a company to a consumer for buying goods and services exclusively from
that company, which are quite popular with e-commerce companies.
 Lastly, semi-closed wallets like Paytm, which do not permit cash withdrawal or redemption, but allow
you to buy goods and services at listed merchants and perform financial services at listed locations.

FINANCIAL INCLUSION IN INDIA

Financial inclusion is the initiative of the government of India, RBI and banks to provide basic banking
services to all section of society in urban areas or rural areas at affordable cost. The Government of India and
the Reserve Bank of India have been making concerted efforts to promote financial inclusion as one of the
important national objectives of the country.

www.ibpsguise,in | mock.ibpsguide.com | www.sscexamguide.com


Copyright © IBPS Guide 73
www.ibpsguide.com
The GOI and RBI with the help of banks has initiated financial inclusion program to provide banking services
to deprived and low income groups of our society at affordable cost.

Steps taken by RBI to support financial inclusion:

Opening of No-Frills Accounts:

Basic banking no-frills account is with nil or very low minimum balance as well as charges that make such
accounts accessible to vast sections of the population. Banks have been advised to provide small overdrafts in
such accounts.

Relaxation on Know-Your‘s-Customer (KYC) Norms:

 KYC requirements for opening bank accounts were relaxed for small accounts in August 2005; thereby
simplifying procedures by stipulating that introduction by an account holder who has been subjected to
the full KYC drill would suffice for opening such accounts.
 The banks were also permitted to take any evidence as to the identity and address of the customer to
their satisfaction.
 It has now been further relaxed to include the letters issued by the Unique Identification Authority of
India containing details of name, address and Aadhaar number.

Engaging Business Correspondents (BCs):

 In January 2006, RBI permitted banks to engage business facilitators (BFs) and BCs as intermediaries
for providing financial and banking services.
 The BC model allows banks to provide doorstep delivery of services, especially cash in-cash out
transactions, thus addressing the last-mile problem.

Use of Technology:

Recognizing that technology has the potential to address the issues of outreach and credit delivery in rural and
remote areas in a viable manner, banks have been advised to make effective use of information and
communications technology (ICT), to provide doorstep banking services through the BC model where the
accounts can be operated by even illiterate customers by using biometrics, thus ensuring the security of
transactions and enhancing confidence in the banking system

Adoption of EBT:

www.ibpsguise,in | mock.ibpsguide.com | www.sscexamguide.com


Copyright © IBPS Guide 74
www.ibpsguide.com
Banks have been advised to implement EBT by leveraging ICT-based banking through BCs to transfer social
benefits electronically to the bank account of the beneficiary and deliver government benefits to the doorstep
of the beneficiary, thus reducing dependence on cash and lowering transaction costs.

General Credit Card (GCC):

With a view to helping the poor and the disadvantaged with access to easy credit, banks have been asked to
consider introduction of a general purpose credit card facility up to 25,000 at their rural and semi-urban
branches.

The objective of the scheme is to provide hassle-free credit to banks‘ customers based on the assessment of
cash flow without insistence on security, purpose or end use of the credit. This is in the nature of revolving
credit entitling the holder to withdraw up to the limit sanctioned.

Simplified Branch Authorization:

To address the issue of uneven spread of bank branches, in December 2009, domestic scheduled commercial
banks were permitted to freely open branches in tier III to tier VI centers with a population of less than 50,000
under general permission, subject to reporting.

In the north-eastern states and Sikkim, domestic scheduled commercial banks can now open branches in rural,
semi-urban and urban centers without the need to take permission from RBI in each case, subject to reporting.

Opening of Branches in Unbanked Rural Centers:

To further step up the opening of branches in rural areas so as to improve banking penetration and financial
inclusion rapidly, the need for the opening of more bricks and mortar branches, besides the use of BCs, was
felt. Accordingly, banks have been mandated in the April monetary policy statement to allocate at least 25% of
the total number of branches to be opened during a year to unbanked rural centers.

Swabhimaan:

Swabhimaan is financial inclusion scheme launched by GOI to provide banking facilities in habitation with a
population in excess of 2000 by March 2012. This nationwide programme on financial inclusion was launched
in February, 2011 with its focus on bringing the deprived sections of the society in the banking network.

www.ibpsguise,in | mock.ibpsguide.com | www.sscexamguide.com


Copyright © IBPS Guide 75
www.ibpsguide.com
Pradhan Mantri Jan Dhan Yojana (PMJDY):

 Pradhan Mantri Jan Dhan Yojana (PMJDY) is a national mission to bring comprehensive financial
inclusion of all the households in the country.
 Under the PMJDY, any individual above the age of 10 years and does not have a bank account can
open a bank account without depositing any money.
 The scheme was to ensure the access to financial services such as banking / savings & deposit
Accounts, remittance, credit, debit cards, insurance and pension in affordable manner.
 The scheme was mostly targeted to the people belonging to the Below Poverty Line but is beneficial to
everyone who does not have a bank account.
 Jan Dhan Yojana has seen a great success, about 21 Crore accounts have been opened in just about one
and half year under the scheme. Out of the total 12.87 crore in rural area and 8.13 Crore accounts have
been opened in urban areas.
 Despite of zero minimum balance, there is 33074.89 crore rupees balance in these accounts with
28.88% accounts opened with zero balance.

DIFFERENT CODES USED IN BANKING

IFSC Code:

 IFSC Code stands for Indian Financial System Code.


 This code is an alpha-numeric code that is used to uniquely identify a bank branch participating in the
electronic payment systems in India like Real Time Gross Settlement (RTGS) and the National
Electronic Fund Transfer (NEFT).
 It is an 11 digit code.
 The components of IFSC code are:
▪ First 4 letters represent bank code.
▪ The 5th letter is a ‗0‘(Zero).
▪ Last six letters represent bank branch.

For example: IFS Code of a branch of Punjab National Bank in Delhi is PUNB0614800.

MICR:

 MICR stands for Magnetic Ink Character Recognition.

www.ibpsguise,in | mock.ibpsguide.com | www.sscexamguide.com


Copyright © IBPS Guide 76
www.ibpsguide.com
 The magnetic ink character recognition line is printed using special ink, which is sensitive to magnetic
fields that allow computers to read the printed information.
 This code is present at the bottom of cheques and other vouchers.
 MICR code is 9 digits numeric code used to identify a bank participating in electronic clearing scheme.
 The components of MICR code are:
▪ First 3 digits represents city code
▪ The second 3 digits represents Bank code
▪ The last 3 digits denotes bank branch.

For example: MICR-no of one of the Punjab National Bank in Delhi is 110024490.

SWIFT/BIC Code:

 SWIFT stands for Society for Worldwide Interbank Financial Telecommunication.


 It is a unique identification code for both financial and non-financial institutions approved by the
International Organization for Standardization (ISO).
 SWIFT codes are used mainly for international money transfer between banks.
 It is a standard format of Bank identifier Codes (BIC)
 It is an 8 or 11 digit code.
 The components of SWIFT code are:
▪ First 4 characters represent bank code.
▪ Next 2 characters represent ISO 3166-1 alpha-2 country code.
▪ Next 2 characters represent location code.
▪ The last 3 characters represent branch code.

www.ibpsguise,in | mock.ibpsguide.com | www.sscexamguide.com


Copyright © IBPS Guide 77
www.ibpsguide.com

For example: The SWIFT code of one of the branches of Indian Bank in Chennai is IDIBINBBASA.

BSR (Basic Statistical Returns) Code:

 BSR code is used by the Income Tax department in order to identify a bank branch for submission of
returns to the RBI.
 It is allotted to banks by Reserve Bank of India. While filling TDS/TCS (tax deducted at source/ tax
collected at source) returns, BSR code is used in details related to challan and deductee.
 It is a 7-digit code.

Example: BSR Code of a branch of Punjab National Bank in Delhi is 305066.

INFLATION

 Inflation is nothing but the more prices we pay for goods. It is the persistent rise of all goods and
services over a period of time. There are several factors that influence inflation in India.
 The major factors to be taken into account are the population, unbalanced economic growth, demand
for more money and increased taxes. On the flip side it has adverse effects on consumers.
 The day to day goods are sold considerably at a higher rate which makes difficult for the consumers to
afford their basic needs. Hence the need for money increases which is one of the major cause.
 There is huge money gap which could be the potential factor for increased price and inflation in India.
Increase in enormous expenditure can cause inflammatory gap at current prices.
 The rate of inflation is measured on the basis of price indices which are of two kinds
 Wholesale Price Index (WPI)

www.ibpsguise,in | mock.ibpsguide.com | www.sscexamguide.com


Copyright © IBPS Guide 78
www.ibpsguide.com
 Consumer Price Index (CPI)

Types of Inflation:

Demand – Pull Inflation:

When there is a mis-match between demand and supply, it will eventually pull up the prices. Here we have two
cases.

In first case, the demand increases over the same level of supply. In second case, the supply decreases with the
same level of demand. In both cases the situation of Demand-pull inflation arises.

Cost – Push Inflation:

An increase in factor input costs pushes up prices. In general the factors that could contribute to Cost-Push
inflation are increases in corporate taxes, rising wages, and rising raw materials cost.

Low Inflation:

Low inflation takes place in a longer period and the range of increase is usually in ‗single digit‘. Such inflation
has also been called as ‗creeping inflation‘.

Deflation:

 Deflation is the exact opposite of inflation.


 The persistent fall in the prices of all goods and services over a period of time is called deflation.
 When deflation occurs it is possible to buy more amounts of goods with the same amount of money.
 Deflation has often had the side effect of increasing unemployment in an economy, since the process
often leads to a lower level of demand in the economy.

Stagflation:

Stagflation is a situation in an economy where inflation and unemployment both are at higher levels.
Stagflation occurs when the economy isn‘t growing but prices are going up. Stagflation is basically a
combination of high inflation and low growth.

Galloping Inflation:

This is a ―very high inflation‖ running in the range of double-digit or triple digit (i.e. 20%, 100% or 200% a
year). The Russian economy showed such inflation after the disintegration of the ex-USSR in the late 1980s.

Hyperinflation:

www.ibpsguise,in | mock.ibpsguide.com | www.sscexamguide.com


Copyright © IBPS Guide 79
www.ibpsguide.com
This form of inflation is ‗large and accelerating‘ which might have the annual rates in million or even trillion.
In such inflation not only range of increase is very large but the increase takes place in a very short span of
time, prices shoot up overnight. This hasn‘t happened in the U.S. since the Civil War, occurred in Germany
before World War II, and in Zimbabwe in the 2000s. Such inflation quickly leads to a complete loss of
confidence in the domestic currency and people start opting for other forms of money.

Skewflation:

It is an un-usual inflation, where there is inflation in one particular sector for a particular period of time, while
the other sector is experiencing no changes at all or facing deflation.

Impacts of Inflation:

 It slows down the economic growth rate.


 Inflation redistributes wealth from creditors to debtors i.e. lenders suffer and borrowers benefit out of
inflation.
 Prices goes up, that mean you pay more money for the same product which you got it lesser earlier.
 With the rise in inflation, lending institutions feel the pressure of higher lending.
 Investment in the economy is boosted by the inflation (in the short-run).
 The standard of living declines.
 The export segment of the economy benefits due to inflation.
 Inflation gives an economy the advantage of lower imports and import-substitution as foreign goods
become costlier.

PRIORITY SECTOR LENDING

Priority Sector refers to those sectors of the economy which may not get timely and adequate credit in
the absence of this special dispensation.

The overall objective of priority sector lending is to ensure that adequate institutional credit flows into
some of the vulnerable sectors of the economy, which may not be attractive for the banks from the point of
view of profitability.

Priority sector came into much attention in 1972, following the National Credit Council‘s plea that
more emphasis should be given by commercial banks to the priority sector. Therefore, initially, in 1974, the
commercial banks were given a target of 33.33 % of their total credit should go to the priority sector.

www.ibpsguise,in | mock.ibpsguide.com | www.sscexamguide.com


Copyright © IBPS Guide 80
www.ibpsguide.com
Following the recommendations of Dr. K S Krishanaswamy Committee, this target was revised to 40%
of the total credit given by the banks.

The latest revision in PSL targets has been made by the M.V. Nair Committee in 2012. The categories
under priority sector lending are as follows

 Agriculture (Direct and Indirect finance)


 Micro and Small Enterprises
 Education
 Housing
 Export Credit
 Social Infrastructure
 Renewable Energy
 Others

The targets and sub-targets under priority sector lending are linked to Adjusted Net Bank Credit (ANBC).

Under PSL the domestic and foreign banks operating in India are furnished below:

Categories Domestic Scheduled Commercial Foreign banks with less than


Banks and Foreign banks with 20 20 branches
branches and above (It can be either
ANBC or Credit Equivalent Amount of
Off-Balance Sheet Exposure,
whichever is higher.)

Total Priority 34% for 2016-17 and 36% for


Sector 40%
2017-18

Agriculture 18% Not applicable

Micro
7.5% Not Applicable
Enterprises

Advances to Not Applicable


10%
Weaker

www.ibpsguise,in | mock.ibpsguide.com | www.sscexamguide.com


Copyright © IBPS Guide 81
www.ibpsguide.com
Sections

Agriculture:

Direct Agricultural advances denote advances given by banks directly to farmers for agricultural
purposes. Direct finance to agriculture shall include short, medium and long-term loans given for agriculture
and allied activities directly to individual farmer, Self-Help Groups (SHGs) or Joint Liability Groups (JLGs) of
individual farmers.

It also includes loans to small and marginal farmers for purchase of land for agricultural purposes,
Purchase of agricultural implements and machinery, Development of irrigation potential, Reclamation and
Land Development Schemes, Construction of farm buildings and structures, etc.

Indirect finance may include loan for construction and running of storage facilities to store agricultural
products. Indirect finance denotes to finance provided by banks to farmers indirectly, i.e., through other
agencies. Priority sector lending by commercial banks is monitored by Reserve Bank of India through
periodical Returns received from them.

Micro Small and Medium Enterprises:

The limits for investments as notified by Ministry of Micro Small and Medium Enterprise are:

www.ibpsguise,in | mock.ibpsguide.com | www.sscexamguide.com


Copyright © IBPS Guide 82
www.ibpsguide.com

Further, the following loans are also counted as priority sector loans under MSME:

 Loans to Khadi and Village Industries Sector (KVI)


 Loans to entities which provide inputs to artisans / village / cottage industries and their cooperatives
 Loans to Micro-finance Institutions, which in turn use this loan to disburse to MSME
 Loans under various schemes related to MSME scheme
 Overdraft under Pradhan Mantri Jan-dhan Yojana up to Rs. 5000.
 Outstanding deposits with SIDBI and MUDRA Ltd. on account of priority sector shortfall.

Education:

Education loans include loans and advances granted to only individuals for educational purposes up to
Rs. 10 lakh for studies in India and Rs. 20 lakh for studies abroad, and do not include those granted to
institutions.

Housing:

 For housing loans to individuals, limit to be counted as priority sector loans is Rs. 28 Lakh in Metros
and Rs. 20 Lakh in other cities, towns and villages.
 For repairing of house, limit is Rs. 5 Lakh in metros and Rs. 2 lakh in others.
 Loans to any government agency for construction of houses subject to ceiling of Rs. 10 Lakh per house
/ dwelling unit for weaker sections or slum clearing.

www.ibpsguise,in | mock.ibpsguide.com | www.sscexamguide.com


Copyright © IBPS Guide 83
www.ibpsguide.com
Export Credit:

Incremental export credit up to 2% for domestic banks and foreign banks with 20 branches and above.

Social Infrastructure:

 This includes loans up to Rs 5 crore per borrower for building social infrastructure for activities viz.
schools, health care facilities, drinking water facilities and sanitation facilities including construction/
refurbishment of household toilets and household level water improvements in Tier II to Tier VI
centers.
 It also includes loan to Micro-finance Institutions (MFIs) for on-lending to SHGs and JLGs for water
and sanitation facilities.

Renewable Energy:

 This includes loan up to Rs. 15 crore to borrowers for purposes like solar based power generators,
biomass based power generators, wind mills, micro-hydel plants and for non-conventional energy based
public utilities Viz. Street lighting systems, and remote village electrification.
 For individual households, the loan limit will be Rs 10 lakh per borrower.

Others:

 Loans not exceeding Rs 50,000/- per borrower provided directly by banks to individuals and their
SHG/JLG, provided the individual borrower‘s household annual income in rural areas does not exceed
Rs 100,000/- and for non-rural areas it does not exceed Rs 1,60,000/-.
 Loans to distressed persons not exceeding Rs 100,000/- per borrower to prepay their debt to non-
institutional lenders.
 Overdrafts extended by banks upto Rs 5,000/- under Pradhan Mantri Jan-DhanYojana (PMJDY)
accounts provided the borrowers household annual income does not exceed Rs 100,000/- for rural areas
and Rs 1,60,000/- for non-rural areas.
 Loans sanctioned to State Sponsored Organizations for Scheduled Castes/ Scheduled Tribes for the
specific purpose of purchase and supply of inputs and/or the marketing of the outputs of the
beneficiaries of these organizations.

BASEL NORMS

 Basel is a city in Switzerland which is also the headquarters of Bureau of International Settlement
(BIS).

www.ibpsguise,in | mock.ibpsguide.com | www.sscexamguide.com


Copyright © IBPS Guide 84
www.ibpsguide.com
 Basel is a set of international banking regulations put forth by the Basel Committee on Bank
Supervision (BCBS) that sets out the minimum capital requirements of financial institutions with the
goal of minimizing credit risk.
 The set of agreement by the BCBS, which mainly focuses on risks to banks and the financial system are
called Basel accord. The purpose of the accord is to ensure that financial institutions have enough
capital on account to meet obligations and absorb unexpected losses.

The following terms are necessary to understand Basel accord,

CAR - Capital Adequacy Ratio

CRAR - Capital to Risk Weighted Asset Ratio

IRB – Internal Ratings Based

RWA- Risk Weighted Assets

⇒Formulae for CAR = Total Capital / RWA*100

⇒ Total Capital= Tier1 capital + Tier2 capital

Tier 1 - The Tier- I Capital is the core capital

Paid up Capital, Statutory Reserves, Other disclosed free reserves, Capital Reserves which represent
surplus arising out of the sale proceeds of the assets, other intangible assets belong from the category of
Tier1 capital.

Tier 2 - Tier-II capital can be said to be subordinate capitals

Undisclosed reserves, Revaluation Reserves, General Provisions and loss reserves, Hybrid debt capital
instruments such as bonds, Long term unsecured loans and Debt Capital Instruments etc belong from the
category of Tier 2 capital.

Risk Weighted Assets

RWA means assets with different risk profiles; it means that we all know that is much larger risk in
personal loans in comparison to the housing loan, so with different types of loans the risk percentage on
these loans also varies.

BASEL 1:

www.ibpsguise,in | mock.ibpsguide.com | www.sscexamguide.com


Copyright © IBPS Guide 85
www.ibpsguide.com
 The BCBS was founded in 1974 as an international forum where members could cooperate on
banking supervision matters.
 The BCBS aims to enhance "financial stability by improving supervisory know-how and the quality
of banking supervision worldwide."
 In 1988, BCBS introduced capital measurement system called Basel capital accord, also called as
Basel 1.
 It focused almost entirely on credit risk. It defined capital and structure of risk weights for banks.
 The Basel I classification system groups a bank's assets into five risk categories, classified as
percentages: 0%, 10%, 20%, 50% and 100%. A bank's assets are placed into a category based on
the nature of the debtor.
 The minimum capital requirement was fixed at 8% of risk weighted assets (RWA). RWA means
assets with different risk profiles.
 India adopted Basel 1 guidelines in 1999.

BASEL 2:

 In 2004, Basel II guidelines were published by BCBS, which were considered to be the refined and
reformed versions of Basel I accord.

www.ibpsguise,in | mock.ibpsguide.com | www.sscexamguide.com


Copyright © IBPS Guide 86
www.ibpsguide.com
 The second Basel Accord, called Revised Capital Framework but better known as Basel II.
 It focuses on three main areas, those are called three pillars of Basel 2 accord
1. Minimum capital requirements
2. Supervisory review of an institution's capital adequacy
3. Internal assessment process and effective use of disclosure as a lever to strengthen market
discipline and encourage sound banking practices including supervisory review.

BASEL 3:

 In the wake of the Lehman Brothers collapse of 2008 and the ensuing financial crisis, the BCBS
decided to update and strengthen the Accords.
 In 2010, Basel III guidelines were released. Basel 3 accord is an enhancement of Basel 2 accord.

 Basel 3 measures aim to:


 Improve the banking sector's ability to absorb shocks arising from financial and
economic stress, whatever the source.
 Improve risk management and governance.
 Strengthen banks transparency and disclosures.
 Basel -3 recommended that the Capital Adequacy Ratio (CAR) be increased to 8% internationally,
while in India it is 9%.
 Out of the 9% capital adequacy requirement, 7 % has to be met by Tier 1 capital while the remaining
2% by Tier 2 capital.

www.ibpsguise,in | mock.ibpsguide.com | www.sscexamguide.com


Copyright © IBPS Guide 87
www.ibpsguide.com
 Presently Indian banking system follows Basel II norms.
 The Reserve Bank of India has extended the timeline for full implementation of the Basel III capital
regulations by a year to March 31, 2019.
 Nearly 2.4 lakh crore rupees are required to implement Basel 3 in India.

DEPOSIT INSURANCE AND CREDIT GUARANTEE CORPORATION (DICGC)

 DICGC is one of the wholly owned subsidiaries of the Reserve bank of India (RBI).
 It was established on 15 July 1978 under Deposit Insurance and Credit Guarantee Corporation Act,
1961 for the purpose of providing insurance of deposits and guaranteeing of credit facilities to the
customers of banks.
 This means that the money of customers who deposit money in the banks is insured by DICGC. And
the customers who take loans from banks are guaranteed of money by DICGC.
 Deposit Insurance Corporation (DIC) and Credit Guarantee Corporation of India Ltd. (CGCI) which
were merged to form DICGC with a view to integrate the functions of both DIC and CGCI.

Bank covered Insurance under DICGC:

 All scheduled commercial Banks & Cooperative Banks


 It also include foreign banks which is running in India also be covered under DICGC.
 It also include Indian Banks which is functioning outside India will also be covered under this Act.
 All Regional Rural Bank which is functioning in India also be covered under DICGC.
 There is a few exceptions which is listed below –
▪ Primary Agricultural Credit Society.
▪ Cooperative banks from Meghalaya , Chandigarh , Lakshadweep & Dadra & Nagar Haveli.

Types of Deposit covered under DICGC:

DICGC insures all bank deposits, such as saving, fixed, current, and recurring, etc. except the following types
of deposits.

 Deposits of foreign Governments;


 Deposits of Central/State Governments;
 Inter-bank deposits;

www.ibpsguise,in | mock.ibpsguide.com | www.sscexamguide.com


Copyright © IBPS Guide 88
www.ibpsguide.com
 Deposits of the State Land Development Banks with the State co-operative banks;
 Any amount due on account of and deposit received outside India;
 Any amount which has been specifically exempted by the corporation with the previous approval of the
RBI.

Features of DICGC:

 A maximum of Rs 1,00,000 is insured for each user for both principal and interest amount.
 If the customer has accounts in different banks, they all account are insured to a maximum of Rs
1,00,000. However, if there are more accounts in same bank, they all are treated as a single account.
 The insurance premium is paid by the insured banks itself. This means that the benefit of deposit
insurance protection is made available to the depositors or customers of banks free of cost.
 The Corporation has the power to cancel the registration of an insured bank if it fails to pay the
premium for three consecutive half-year periods.
 The Corporation may restore the registration of the bank, which has been de-registered for non-
payment of premium, if the concerned bank makes a request in this behalf and pays all the amounts due
by way of premium from the date of default together with interest.

BANKING OMBUDSMAN IN INDIA

Banking Ombudsman is a quasi judicial authority functioning under the Banking Ombudsman Scheme, 2006.
The authority was created to enable resolution of complaints of customers of banks relating to services
rendered by the lenders.

Banking Ombudsman Scheme is a mechanism created by the RBI to address the complaints raised by bank
customers. It is run by the RBI directly to ensure customer protection in the banking industry. According to the
RBI, ―The Scheme enables an expeditious and inexpensive forum to bank customers for resolution of
complaints relating to certain services rendered by banks.‖

The Banking Ombudsman Scheme was introduced under Section 35 A of the Banking Regulation Act, 1949 by
RBI with effect from 1995. The present Ombudsman scheme was introduced in 2006.

The Banking Ombudsman is a senior official appointed by the Reserve Bank of India. He has the responsibility
to redress customer complaints against deficiency in certain banking services. At present twenty Ombudsmen
were appointed by the RBI to settle complaints and they are appointed in state capitals.

www.ibpsguise,in | mock.ibpsguide.com | www.sscexamguide.com


Copyright © IBPS Guide 89
www.ibpsguide.com
All Scheduled Commercial Banks, Regional Rural Banks and Scheduled Primary Co-operative Banks are
covered under the Scheme.

The Banking Ombudsman can receive and consider any complaint relating to a number of deficiencies related
to banking operations including internet banking. RBI has mentioned a large number of service deficiencies by
banks to customers where the customers can approach the Ombudsman through a complaint.

Following are some of the instances:

 Non-payment or inordinate delay in the payment or collection of cheques, drafts, bills etc.
 Non-acceptance, without sufficient cause, of small denomination notes tendered for any purpose, and
for charging of commission in respect thereof
 Non-acceptance, without sufficient cause, of coins tendered and for charging of commission in respect
thereof
 Non-payment or delay in payment of inward remittances
 Failure to issue or delay in issue of drafts, pay orders or bankers‘ cheques
 Non-adherence to prescribed working hours

A customer can file a complaint before the Banking Ombudsman if the bank doesn‘t gives a reply to the
customer within a period of one month or the bank rejects the complaint, or if the complainant is not satisfied
with the reply by the bank.

KNOW YOUR CUSTOMER

 Know Your Customer is a process by which banks obtain information about the identity and address of
the customers. This process helps to ensure that banks‘ services are not misused.
 The KYC procedure is to be completed by the banks while opening accounts and also periodically
update the same.
 KYC guidelines were introduced in year 2002 by RBI and all banks were asked to make all accounts
KYC compliant by 31 December 2005.
 These guidelines are issued under Section 35 A of the Banking Regulation Act, 1949.

www.ibpsguise,in | mock.ibpsguide.com | www.sscexamguide.com


Copyright © IBPS Guide 90
www.ibpsguide.com
 KYC guidelines have been revisited time to time in the context of the Recommendations made by the
Financial Action Task Force (FATF) on Anti Money Laundering (AML) standards and on Combating
Financing of Terrorism (CFT).
 In January 2012, the Capital markets regulator SEBI‘s Chairman, Mr. U.K. Sinha, launched India’s
first Know Your Customer Registration Agency – KRA at Bombay Stock Exchange. The system
avoids duplication of customer details and is interoperable, which means that other market participants
can share the data and bring in more uniformity.

‗Officially Valid Documents‘ (OVDs):

 Banks need two types of document one for identity another for address along with a recent photograph.
 The Government of India has notified six documents as ‗Officially Valid Documents‘ (OVDs) for the
purpose of producing proof of identity. These are
1. Passport,
2. Driving License,
3. Voters Identity Card,
4. PAN Card,
5. Aadhaar Card
6. NREGA (National Rural Employment Guarantee Act) Job Card.
 If the document submitted by you for proof of identity does not contain address details, then you will
have to submit another officially valid document which contains address details.
E-KYC:

 e-KYC refers to electronic KYC.


 e-KYC is possible only for those who have Aadhaar numbers.
 While using e-KYC service, you have to authorize the Unique Identification Authority of India
(UIDAI), by explicit consent, to release your identity/address through biometric authentication to the
bank branches/business correspondent (BC).
 The UIDAI then transfers your data comprising your name, age, gender, and photograph electronically
to the bank.
 Information thus provided through e-KYC process is permitted to be treated as an ‗Officially Valid
Document‘ under PML (Prevention of Money Laundering) Rules and is a valid process for KYC
verification.

www.ibpsguise,in | mock.ibpsguide.com | www.sscexamguide.com


Copyright © IBPS Guide 91
www.ibpsguide.com

PROMPT CORRECTIVE ACTION

 RBI introduces Prompt Corrective Action when the Bank‘s financial conditions worsen below certain
limits (trigger points).
 The limit set is in the form of three conventional financial indicators which are called trigger points–
CRAR, Net NPA and Return on Assets.
 Trigger points implies the RBI imposes corrective action in accordance with the level of trigger points.

Prompt Corrective Action Framework:

 RBI has issued a policy action guideline (first in May 2014 and revised effective from April 1, 2017) in
the form of Prompt Corrective Action (PCA) Framework if a commercial bank‘s financial condition
worsens below a mark.
 The PCA framework is applicable only to commercial banks and not extended to co-operative banks,
non-banking financial companies (NBFCs) and FMIs.
 The PCA framework specifies the trigger points or the level in which the RBI will intervene with
corrective action. This trigger points are expressed in terms of parameters for the banks.
 The triggering of PCA means there will be several restrictions imposed on the banks from lending to
the distribution of dividends etc.

The parameters that invite corrective action from the central bank are:

1. Capital to Risk weighted Asset Ratio (CRAR)


2. Net Non-Performing Assets (NPA) and
3. Return on Assets (RoA)
4. Leverage ratio

When these parameters reach the set trigger points for a bank, the RBI will initiate certain structured and
discretionary actions for the bank.

As per the revised framework by the RBI, in April 2017, capital, asset quality and profitability continue to be
the key areas for monitoring. Along with this, leverage of banks also will be monitored.

The trigger points along with structured and discretionary actions that could be taken by the Reserve Bank are
described below:

1. CRAR

(i) CRAR less than 9%, but equal or more than 6% -

www.ibpsguise,in | mock.ibpsguide.com | www.sscexamguide.com


Copyright © IBPS Guide 92
www.ibpsguide.com
 Bank need to submit capital restoration plan
 Restrictions on RWA expansion, entering into new lines of business, accessing/renewing costly
deposits and CDs, and making dividend payments
 Order recapitalization
 Restrictions on borrowing from inter-bank market,
 Reduction of stake in subsidiaries,
 Reducing its exposure to sensitive sectors like capital market, real estate or investment in non-SLR
securities.

(ii) CRAR less than 6%, but equal or more than 3%:

 In addition to actions in hitting the first trigger point, RBI could take steps to bring in new
Management/ Board, appoint consultants for business/ organizational restructuring, take steps to
change ownership, and also take steps to merge the bank if it fails to submit recapitalization plan.

(iii) CRAR less than 3% :

 In addition to actions in hitting the first and second trigger points, more close monitoring; steps to
merge/amalgamate/liquidate the bank or impose moratorium on the bank if its CRAR does not improve
beyond 3% within one year or within such extended period as agreed to.

2. Net NPAs

(i) Net NPAs over 10% but less than 15% -

 Special drive to reduce NPAs and contain generation of fresh NPAs


 Review loan policy and take steps to strengthen credit appraisal skills, follow-up of advances and suit-
filed/decreed debts, put in place proper credit-risk management policies
 Reduce loan concentration
 Restrictions in entering new lines of business, making dividend payments and increasing its stake in
subsidiaries.

(ii) Net NPAs 15% and above:

 In addition to actions on hitting the above trigger point, bank‘s Board is called for discussion on
corrective plan of action.

3. ROA less than 0.25%:

www.ibpsguise,in | mock.ibpsguide.com | www.sscexamguide.com


Copyright © IBPS Guide 93
www.ibpsguide.com
 Restrictions on accessing/renewing costly deposits and CDs, entering into new lines of business,
bank‘s borrowings from inter-bank market, making dividend payments and expanding its staff
 Special drive to reduce NPAs and contain generation of fresh NPAs
 Restrictions on incurring any capital expenditure other than for technological upgradation and for some
emergency situations.

Corrective action that can be imposed on banks includes special audit, restructuring operations and activation
of recovery plan.

Promoters of banks can be asked to bring in new management, or even can supersede the bank‘s board, as a
part of corrective action.

GOODS AND SERVICES TAX

Goods and Services Tax (GST) is a comprehensive indirect tax on manufacture, sale, and consumption of
goods and services throughout India. GST would replace respective taxes levied by the central and state
governments.

What is GST?

 GST is a destination-based taxation system.


 It has been established by the 101st Constitutional Amendment Act and of 122 Amendment Bill.
 It is an indirect tax for the whole country on the lines of ―One Nation One Tax‖ to make India a unified
market.
 It is a single tax on supply of Goods and Services in its entire product cycle or life cycle i.e. from
manufacturer to the consumer.
 It is calculated only in the ―Value addition‖ at any stage of a goods or services.
 The final consumer will pay only his part of the tax and not the entire supply chain which was the case
earlier.
 There is a provision of GST Council to decide upon any matter related to GST whose chairman in the
finance minister of India.

What taxes at center and state level are incorporated into the GST?

At the State Level

 State Value Added Tax/Sales Tax


 Entertainment Tax (Other than the tax levied by the local bodies)

www.ibpsguise,in | mock.ibpsguide.com | www.sscexamguide.com


Copyright © IBPS Guide 94
www.ibpsguide.com
 Octroi and Entry Tax
 Purchase Tax
 Luxury Tax
 Taxes on lottery, betting, and gambling

At the Central level

 Central Excise Duty


 Additional Excise Duty
 Service Tax
 Additional Customs Duty (Countervailing Duty)
 Special Additional Duty of Customs

Benefits of GST:

For Central and State Governments:

Simple and Easy to administer: Because multiple indirect taxes at the central and state levels are being replaced
by a single tax ―GST‖. Moreover, backed with a robust end to end IT system, it would be easier to administer.

Better control on leakage: Because of better tax compliance, reduction of rent seeking, transparency in taxation
due to IT use, an inbuilt mechanism in the design of GST that would incentivize tax compliance by traders.

Higher revenue efficiency: Since the cost of collection will decrease along with an increase in the ease of
compliance, it will lead to higher tax revenue.

For the Consumer:

 The single and transparent tax will provide a lowering of inflation.


 Relief in overall tax burden.
 Tax democracy that is luxury items will be taxed more and basic goods will be tax-free.

For the Business Class:

 Ease of doing business will increase due to easy tax compliance.


 Uniformity of tax rate and structure, therefore, better future business decision making and investments
by the corporates.
 Removal of cascading effects of taxes.

www.ibpsguise,in | mock.ibpsguide.com | www.sscexamguide.com


Copyright © IBPS Guide 95
www.ibpsguide.com
 Reduction in transactional cost will lead to improved competitiveness.
 Gain to the manufacturer and exporters.
 It is expected to raise the country GDP by 2% points.

Regulatory Framework of GST

A new set up by Government of India named as ‗GST Council‘.

 The GST Council consists of


a) The Union Finance Minister (as Chairman),
b) The Union Minister of State in charge of Revenue or Finance, and
c) The Minister in charge of Finance or Taxation or any other Minister, nominated by each state
government.
 It is the 1st Federal Institution of India, as per the Finance minister.
 It will approve all decision related to taxation in the country.
 It consists of Centre, 29 states, Delhi and Puducherry.
 Centre has 1/3rd voting rights and states have 2/3rd voting rights.
 Decisions are taken after a majority in the council.

Supporting Laws to implement GST:

For the implementation of GST, apart from the Constitution Amendment Act, some other statutes are also
necessary. Recently 5 supporting laws to the GST were recommended by the council. 4 for the bills should be
passed by the parliament, while the 5th one should be passed by respective state legislatures. The details are
given below.

 The Central Goods and Services Tax Bill 2017 (The CGST Bill).
 The Integrated Goods and Services Tax Bill 2017 (The IGST Bill).
 The Union Territory Goods and Services Tax Bill 2017 (The UTGST Bill).
 The Goods and Services Tax (Compensation to the States) Bill 2017 (The Compensation Bill).

And a state GST will be passed by the respective state legislative assemblies.

Tax slabs are decided as 0%, 5%, 12%, 18%, 28% along with categories of exempted and zero rated goods for
different types of goods and services.

www.ibpsguise,in | mock.ibpsguide.com | www.sscexamguide.com


Copyright © IBPS Guide 96
www.ibpsguide.com
Further, a cess would be levied on certain goods such as luxury cars, aerated drinks, pan masala and tobacco
products, over and above the rate of 28% for payment of compensation to the States. Highest tax slab is pegged
at 40%.

The Principle of GST:

 The Centre will levy and collect the Central GST.


 States will levy and collect the State GST on the supply of goods and services within a state.
 The Centre will levy the Integrated GST (IGST) on the interstate supply of goods and services, and
apportion the state‘s share of tax to the state where the good or service is consumed.

The 2016 Act requires Parliament to compensate states for any revenue loss owing to the implementation of
GST.

Goods and Service Tax Network (GSTN):

 GSTN is registered as a not-for-profit company under the companies Act.


 It has been formed to set up and operate the information technology backbone of the GST.
 While the Central (24.5%) and the state (24.5%) governments hold a combined stake of 49%, the
remaining 51% stake is divided among five financial institutions—LIC Housing Finance with 11%
stake and ICICI Bank, HDFC, HDFC Bank and NSE Strategic Investment Corporation Ltd with 10%
stake each.
 GSTN had awarded Infosys Ltd the contract to develop the hardware and software for GST.

www.ibpsguise,in | mock.ibpsguide.com | www.sscexamguide.com


Copyright © IBPS Guide 97
www.ibpsguide.com
 The idea behind GSTN was to set up an entity that is equidistant from both the Central government and
the state governments, as it will advise both the Centre and the states on the information technology
network.

The rates will be applicable at under mentioned levels-

 Nil
 5%
 12%
 18%
 28%

Nil (No tax Slab):

There will be no tax applicable on certain goods like, fresh milk, meat, eggs, Butter Milk, Curd, Natural
Honey, Fresh vegetables, roots and tubers, fruits (other than those in frozen or preserved state), Coffee
beans, All cereal goods (other than those put up in unit container and bearing a registered brand name),
Flour, Soya beans, groundnuts, Cane jaggery, Puffed rice, bread, Prasad, Common Salt, Bindi, Sindur,
Plastic bangles, Wood charcoal, Judicial, Non-judicial stamps, Newspaper, Handloom etc.

5% Slab:

This tax slab would involve articles like, frozen fish, fish fillets, Ultra high temperature milk, Milk cream,
Cream yogurt, Whey, Ivory, Herb , bark, dry plant, frozen or preserved fruits and vegetables, Peel of citrus
fruits and melons, Coffee, tea, natural gums, resins, Vegetable fats and oils, beet, sugar, cane sugar, cocoa
beans, Mixes and doughs for the preparation of bread, Tobacco leaves, Unroasted iron pyrites, sulphur, All
ores and concentrates, Kerosene, Sulphonated castor oil, fish oil, Handmade safety matches, Building
bricks, Kerosene, Lifeboats etc.

12% Slab:

This includes items such as, Frozen meat, Butter and other fats, Cheese, Dry fruits, starches, Animal fats
and oils, Sausages and similar products, of meat, meat offal or blood, Fruit and vegetable juices, Roasted
chicory, Soya milk drinks, Beverages containing milk, Marble, Granite blocks, Bio-gas, Medicinal grade
Hydrogen peroxide, Fertilizers, Fountain/ Ball pen ink, Tooth powder, Agarbatti, Candles, Photographic

www.ibpsguise,in | mock.ibpsguide.com | www.sscexamguide.com


Copyright © IBPS Guide 98
www.ibpsguide.com
plates and films, Children‘s picture/ drawing/ colouring books, Umbrellas, Sand lime bricks, Sewing
Machines, Cell Phones etc.

18% Slab:

Items included in this tax slab are, Condensed Milk, Malt, Vegetable saps and extracts, Indian katha,
Glycerol, Vegetable waxes, Refined sugar, Pasta, Cornflakes, Waffles, Pastries and Cakes, Jams, Jellies,
Marmalades, Sauces, Soups, Ice cream, Food mixes, Diabetic foods, Petroleum jelly, Paraffin wax,
Fluorine, Chlorine, Bromine, Iodine, Colouring matter, Printing ink, Essential Oils, Artificial waxes, Safety
fuses, Insecticides, Wood tar, Kitchenware, Tableware, Safety Headgears (Helmets), Refractory bricks,
Camera, Speakers and Monitors etc.

28% Slab:

Following are the products included in this tax slab - Chewing gum, Cocoa butter, Extracts, essences and
concentrates of coffee, Non-alcoholic beverages, Aerated Water, Portland cement, Paints and Varnishes,
Artist/ Student or signboard painter‘s colours, Perfumes, Toothpaste, Fireworks, Sinks, Wash basins, Wall
paper/ coverings, Lamps, Lighting fittings, Pianos, Revolvers, Washing machines, Vacuum cleaners,
Motorcycles, Aircrafts for personal use, Yachts etc.

Important Points for Exams:

 Amendment Bill- 122


 Amendment Act – 101
 GST council formation based on Article – 279A
 GST Council Headquarters – New Delhi
 Chairman – Arun Jaitley
 GST Finance Ministers Panel Chairman – Amit Mishra (West Bengal Finance Minister)
 1st State to ratify GST Bill – Assam
 1st State passed GST bill – Telangana
 GST threshold limit for normal states – 20 lakh
 GST threshold limit for north eastern states – 10 lakh

www.ibpsguise,in | mock.ibpsguide.com | www.sscexamguide.com


Copyright © IBPS Guide 99
www.ibpsguide.com

GREAT SBI MERGER

State Bank of India (SBI), India‘s largest bank, became even larger with the merger of its five commercial
banking subsidiaries and Bharatiya Mahila Bank on April 1, 2017.

The five associated banks are:

1. State Bank of Bikaner & Jaipur (SBBJ),


2. State Bank of Mysore (SBM),
3. State Bank of Travancore (SBT),
4. State Bank of Patiala (SBP) and
5. State Bank of Hyderabad (SBH)

Bharatiya Mahila Bank Ltd is the first of its kind in the Banking Industry in India formed with a vision of
economic empowerment for women. Ex Indian Prime Minister Manmohan Singh inaugurated the system in
2013.

India is the 3rd country in the world to have a bank especially for women, after Pakistan and Tanzania.

Major reason for merger:

 Before merger SBI was the biggest bank of India in terms of assets. But still it was nowhere in top 50
banks of world. After the merger of the banks SBI‘s balance sheet rouses to the size of 41 trillion

www.ibpsguise,in | mock.ibpsguide.com | www.sscexamguide.com


Copyright © IBPS Guide 100
www.ibpsguide.com
rupees. Because of it, SBI placed into the top 50 banks of the world list. The ICICI bank, the 2nd rank
holder, is having balance sheet of 4 trillion rupees only.
 NPA (Non Performing Assets) is one of the biggest problems faced by banking sector. High NPA, in
simple terms means the repayment of loans back to banks is poor. The solution to NPA is capital
infusion. But in present scenario the capital is scattered into many regional banks. The merger of banks
will cause a large bank with large capital. This will solve the NPA problem.
 Better reach and network.
 India is a developing country and it is growing at high speed. The fast growing economy involves many
high budget projects. High budget project means huge amount of loans from bank. So we need a bank
which is capable of financing high budget project. But in present scenario, our banking system is
scattered into many small regional banks. So the consolidation of banks will solve this problem and
will help India to be in race of economic development.
 India is home to many international MNCs. Consolidation of banks will solve the financing issues of
these large MNCs. This will also attract other MNCs to open offices in India, which is good for a
developing country.
 International experience is also favorable towards banks consolidation. Banks in Japan between 1990
and 2004 gained a lot as a result of large scale merger and acquisition process.
 Larger bank may be less risky than a smaller one as the former will have a more diversified portfolio,
resulting in less vacillation in its earnings and consequently, higher credit ratings.

Highlights of Merger:

 SBI has rebranded its corporate website as ―bank.sbi‖ from the earlier sbi.co.in.
 The background to the SBI signboard has been changed from white to ―inky blue‖ and SBI will be
written in a new font called Effra.
 The designing and rebranding of SBI logo has been done by a company called Design Stack.
 The SBI logo symbolizes its role of a custodian that will keep customers‘ money safe.
 After the merger the tagline of SBI has remained same i.e. ―Banker to every Indian‖.
 With the merger, State Bank of India has entered the league of ‗Top 50 Global Banks‘. SBI is placed at
44th position making it in the top 50 leading banks globally.
 With the merger, SBI now has a balance sheet size of Rs 41 lakh crore, 2.77 lakh employees, 50 crore
customers, more than 25000 branches and nearly 59,000 ATMs.
 The market share of SBI has increased to nearly 22 per cent from 17 percent.
 Customers can access SBI across the globe at over 190 foreign offices in 36 countries.

www.ibpsguise,in | mock.ibpsguide.com | www.sscexamguide.com


Copyright © IBPS Guide 101
www.ibpsguide.com

FOREIGN DIRECT INVESTMENT (FDI)

India has already marked its presence as one of the fastest growing economies of the world. It has been
ranked among the top 10 attractive destinations for inbound investments. Since 1991, the regulatory
environment in terms of foreign investment has been consistently eased to make it investor-friendly.

The measures taken by the Government are directed to open new sectors for foreign direct investment,
increase the sectoral limit of existing sectors and simplifying other conditions of the FDI policy. FDI policy
reforms are meant to provide ease of doing business and accelerate the pace of foreign investment in the
country.

Automatic Route:

Under this route no Central Government permission is required.

Government Route:

Prior approval by government is needed via this route.

Sector Limit Entry Route

Banking- Public Sector 20% Government

Print Media
[Publishing of newspaper and periodicals
dealing with news and current
26% Government
affairs ][Publication of Indian editions of
foreign magazines dealing with news and
current affairs ]
Petroleum & Natural Gas
(Petroleum refining by the Public Sector 49% Automatic
Undertakings (PSU))

Broadcasting Content Services 49% Government

www.ibpsguise,in | mock.ibpsguide.com | www.sscexamguide.com


Copyright © IBPS Guide 102
www.ibpsguide.com

Infrastructure Company in the Securities


49%
Market[in compliance with SEBI Automatic
Regulations ]

Insurance 49% Automatic

Pension Sector 49% Automatic

Power Exchanges 49% Automatic

Multi Brand Retail Trading 51% Government

Private Security Agencies 74% Automatic

Automatic up to 49%
Government route
74%
Banking- Private Sector beyond 49% and up to
74%.

Agriculture & Animal Husbandry 100% Automatic

Plantation Sector
(Tea,Coffee,Rubber,Cardamom,Palm oil, 100% Automatic
Olive oil)

Mining 100% Automatic

Petroleum & Natural Gas(All other


100% Automatic
activity)

www.ibpsguise,in | mock.ibpsguide.com | www.sscexamguide.com


Copyright © IBPS Guide 103
www.ibpsguide.com

Automatic upto 49%


Above 49% under
Defense 100%
Government route on
case to case basis

Automatic upto 49%


Broadcasting Carriage Services 100% Government route
beyond 49%
Print Media
[Publishing/printing of scientific and
technical magazines/specialty journals/
periodical ] 100% Government

[Publication of facsimile edition of foreign


newspapers ]

Civil Aviation 100% Automatic

Airports[Greenfield projects ] 100% Automatic

Automatic up to 74%
Government route
Airports[Existing projects ] 100%
beyond 74%

Construction Development 100% Automatic

Industrial Parks 100% Automatic

Satellites- establishment and operation Automatic

www.ibpsguise,in | mock.ibpsguide.com | www.sscexamguide.com


Copyright © IBPS Guide 104
www.ibpsguide.com
100%

Automatic up to 49%
Government route
Telecom Services 100%
beyond 49%

Trading
[Cash & Carry Wholesale
100% Automatic
Trading/Wholesale Trading (including
sourcing from MSEs) ]

E-commerce activities 100% Automatic

Automatic up to 49%
Government route
Single Brand product retail trading 100%
beyond 49%

Processed Food Products 100% Automatic

Duty Free Shops 100% Automatic

Railway Infrastructure 100% Automatic

Asset Reconstruction Companies 100% Automatic

Credit Information Companies (CIC) 100% Automatic


White Label ATM Operations Automatic

www.ibpsguise,in | mock.ibpsguide.com | www.sscexamguide.com


Copyright © IBPS Guide 105
www.ibpsguide.com
100%

Non-Banking Finance Companies (NBFC) 100% Automatic

Pharmaceuticals[Greenfield] 100% Automatic

Pharmaceuticals[Brownfield]
100% Government

Railway Infrastructure 100% Automatic

MISCELLANEOUS

Non Performing Assets (NPA):

NPA means an asset or account of borrower, which has been classified by a bank or financial institution as sub
standard, doubtful or loss asset in accordance with the guidelines of RBI. Simply, NPA is a loan not recovered.
If a loan has been overdue for more than 90 days from its due date of payment, it will be considered as NPA of
the bank.

ATM (Automatic Teller Machines):

They are machines that dispense cash, receive cash, accept cheques, and give balance details and mini
statements to the customers through Computer network.

Bancassurance:

Bancassurance as the term suggests is Bank + Insurance. Bancassurance means selling insurance product
through banks. It is one of the para banking activity which the RBI has allowed the banks to take up. For
selling the insurance product, bank and insurance company come up in a partnership where the bank sells the
insurance company‘s insurance products to its clients.

www.ibpsguise,in | mock.ibpsguide.com | www.sscexamguide.com


Copyright © IBPS Guide 106
www.ibpsguide.com
Bouncing of a Cheque:

When an account has insufficient funds the cheque is not payable and is returned by the bank with a reason
―Exceeds arrangement‖ or ―funds insufficient‖.

Bank Rate:

It is the rate of interest charged by a central bank to commercial banks on the advances and the loans it
extends.

Basis Point:

One-hundredth of 1% point normally used for indicating cost of finance

Bullion Market:

A market where the trading of precious metals held like: Gold, Silver, Diamond, Platinum and Crystal.

Bull:

Bull is an investor who thinks the market a specific security or an industry will raise. Bulls are the optimistic
investors presently predicting good things of the market and bullish is a habit to purchase that share which is in
profit they are responsible to Rose in stock exchanges.

Bear:

It is an investor who believes that a particular security or market is headed downward. Bears attempt to profit
from a decline in prices. A Bear is generally pessimistic about the state of the given market.

Bridge Loan:

It is also known as swing loan, which is basically a real estate loan or a home loan, where the current
residence/real estate is pledged by the borrower as collateral in order to purchase a new residence.

Call Money:

It is a loan that is made for a very short period of a few days only with a low rate of interest.

Clearing House:

www.ibpsguise,in | mock.ibpsguide.com | www.sscexamguide.com


Copyright © IBPS Guide 107
www.ibpsguide.com
A clearing house is a place for exchange of cheques by banks; it facilitates transfer of funds from one bank to
another, which represents the proceeds of cheques. It is as a central meeting place for bankers to exchange the
cheques drawn on one another and claim funds for the same. Such operations are called as clearing operations.

Core Banking:

It is a general term used to describe the services provided by a group of networked bank branches.

Core Banking Solutions (CBS):

In this all the branches of the bank are connected together and the customer can access his/her funds or
transactions from any other branch.

CRR (Cash Reverse Ratio):

The amount of funds that a bank keep with the RBI. If the percentage of CRR increases then the amount with
the bank comes down.

Currency chest:

 To facilitate the distribution of banknotes and rupee coins, the Reserve Bank has authorised select
branches of scheduled banks to establish Currency Chests.
 These are actually storehouses where banknotes and rupee coins are stocked on behalf of the Reserve
Bank. As on June 30, 2006, there were 4428 Currency Chests and 4102 Small Coin Depots.
 The currency chest branches are expected to distribute banknotes and rupee coins to other bank
branches in their area of operation.

Currency swap:

It is a foreign-exchange agreement between two parties to exchange aspects (namely the principal and/or
interest payments) of a loan in one currency for equivalent aspects of an equal in net present value loan in
another currency. Currency swap is an instrument to manage cash flows in different currency.

Debit Card:

It is a card issued by the bank so the customers can withdraw their money from their account electronically.

Demat Account:

The way in which a bank keeps money in a deposit account in the same way the Depository Company converts
share certificates into electronic form and keep them in a Demat account.

www.ibpsguise,in | mock.ibpsguide.com | www.sscexamguide.com


Copyright © IBPS Guide 108
www.ibpsguide.com
Deflation:

Deflation is the general decline in the prices of goods or assets. In deflation, there is a tremendous lack of
liquidity in the market and the purchasing power of consumers is reduced.

Dishonour of Cheque:

Non-payment of a cheque by the paying banker with a return memo giving reasons for the non-payment.

Disinvestment:

The Selling of the government stake in public sector undertakings.

E-Banking:

It is a type of banking in which we can conduct financial transactions electronically. RTGS, Credit cards, Debit
cards etc come under this category.

EFT – (Electronic Fund Transfer):

In this we use Automatic teller machine, wire transfer and computers to move funds between different accounts
in different or same bank.

Exchange Rate:

Rate at which the domestic currency can be converted into foreign currency and vice versa.

Fiscal Deficit:

It is the amount of Funds borrowed by the government to meet the expenditures.

Fiat Money:

Fiat money is a legal tender for settling debts. It is a paper money that is not convertible and is declared by
government to be legal tender for the settlement of all debts.

Floating Rate:

An interest rate that is referenced to a market rate and is revised as per the change in the interest rates in the
economy. When interest rates in the economy rise, floating rates rise and vice versa.

Foreign Institutional Investors (FII):

www.ibpsguise,in | mock.ibpsguide.com | www.sscexamguide.com


Copyright © IBPS Guide 109
www.ibpsguide.com
They invest in the Indian capital market. These flows are large in magnitude and have a great impact on capital
market and exchange rate. FIIs are also permitted to use their investment in corporate bonds and government
securities as collateral to meet their margin requirements.

GDP:

The Gross Domestic Product or GDP is a measure of all of the services and goods produced in a country over a
specific period; classically a year.

GNP:

Gross National Product is measured as GDP plus income of residents from investments made abroad minus
income earned by foreigners in domestic market.

Grace Period:

It is an interest free period that is to be given by a creditor to a debtor after the period of the loan gets over,
before initiating the process of loss recovery. The grace period depends on the amount of the loan and also the
credit score of the borrower.

Hot money:

Money that is held in one currency but is liable to switch to another currency at a moment‘s notice in search of
highest available returns.

Junk Bonds:

Junk bonds are issued generally by smaller or relatively less well- known firms to finance their operations, or
by large and well-known firms to fund leveraged buyouts. These bonds are frequently unsecured or partially
secured, and they pay higher interest rates: 3 to 4 percentage points higher than the interest rate on blue chip
corporate bonds of comparable maturity period.

Inflation:

Inflation is defined as a sustained increase in the general level of prices for goods and services. It is measured
as an annual percentage increase. As inflation rises, every rupee you own buys a smaller percentage of a good
or service. Consequently, Inflation affects as a reduction in the purchasing power per unit of money- a loss of
real value in the medium of exchange and unit of account within the economy.

Initial Public Offering (IPO):

www.ibpsguise,in | mock.ibpsguide.com | www.sscexamguide.com


Copyright © IBPS Guide 110
www.ibpsguide.com
IPO is Initial Public Offering. This is the first offering of shares to the general public from a company wishes
to list on the stock exchanges.

Islamic Banking:

Islamic banking refers to a system of banking or banking activity that is consistent with Islamic law (SHARIA)
principles and guided by Islamic economics. Particular Islamic law Prohibits usury, the Collection and
payment of interest, also commonly called RIBA in Islamic discourse in addition, Islamic law prohibits
investing in businesses‘ that are considered unlawful, or HARAAM (such as Businesses that sell alcohol or
pork, or businesses that produce media such as gossip columns or pornography, which are contrary to Islamic
values). In the late 20th century, a number of Islamic Banks were created, to cater to this particular banking
market.

Kiosk Banking:

Doing banking from a cubicle from which food, newspapers, tickets, etc are also sold.

Leverage Ratio:

It is a financial ratio which gives us an idea or a measure of a company‘s ability to meet its financial losses.

Liquidity:

It is the ability of converting an investment quickly into cash with no loss in value.

Liquidity adjustments facility (LAF):

It consist of daily infusion or absorption of liquidity on a repurchase basis, through repo (liquidity injection)
and reserve repo (liquidity absorption) auction operations, using governments securities as collateral.

LIBOR:

London Inter Bank Offered Rate. An interest rate at which banks can borrow funds, in marketable size, from
other banks in the London interbank market.

Market Capitalization:

The product of the share price and number of the company‘s outstanding ordinary shares.

Market Stabilization scheme (MSS):

This Instrument for monetary management was introduced in 2004. Liquidity of a more enduring nature arising
from large capital flows is absorbed through sale of short –dated government securities and treasury bills.
www.ibpsguise,in | mock.ibpsguide.com | www.sscexamguide.com
Copyright © IBPS Guide 111
www.ibpsguide.com
Mortgage:

A Mortgage is a method of using property as a security for the performance of an obligation, usually the
payment of a debt. The term Mortgage refers to a legal device used for this purpose and it is also commonly
used to refer to a debt secure by the Mortgage.

Money Laundering:

Money laundering is the practice in a specific financial transaction to conceal the identity, source and
destination of money and is the main operation of underground economy. India has prevention of money
laundering act 2002 which was at latest amended in 2009.

Mutual Fund:

These are investment schemes. It pools money from various investors in order to purchase securities.

Monetary Policy:

It refers to the Central Government policy with respect to the quantity of money in the economy, the rate of
interest and the exchange rate.

National Income:

National Income is the money value of all goods and services produced in a Country during the year.

Nostro Account:

It is the overseas account which is held by the domestic bank in the foreign bank or with the own foreign
branch of the bank. For example the account held by state bank of India with bank of America in New York is
a Nostro account of the state bank of India.

Open Market Operations (OMO):

Outright sales/purchases of government securities, in addition to LAF, as a tool to determine the level of
liquidity over the medium term.

Participatory notes or P-Notes:

These are financial instruments used by the investors or hedge funds that are not registered with the SEBI to
invest in Indian securities.

Permanent Account Number (PAN):

www.ibpsguise,in | mock.ibpsguide.com | www.sscexamguide.com


Copyright © IBPS Guide 112
www.ibpsguide.com
PAN is a number issued by the Income Tax Department to their tax payers.

Plastic Money:

Plastic money is a name given to Credit cards, Debit cards, ATM cards anf International Cards issued by
banks.

Point of Sale (PoS):

PoS refers to a location at which a payment of a card transaction occurs

Prime Lending Rate (PLR):

Rate of interest at which a bank gives loan to its most reliable customer i.e., customer with ‗zero risk‘

Pass Book:

It is a book where all the bank transactions are recorded. They are mainly issued to Current or Savings Bank
account holders.

Repo Rate:

Commercial banks borrow funds by the RBI if there is any shortage in the form of rupees. If this rate increases
it becomes expensive to borrow money from RBI and vice versa.

Reverse Repo Rate:

It is the exact opposite of repo rate. It is the rate at which RBI borrows money from banks when it feels there is
too much money floating in the banking system.

Revenue deficit:

It defines that, where the net amount received (by taxes & other forms) fails to meet the predicted net amount
to be received by the government.

SARFAESI Act, 2002:

The full form of SARFAESI Act as we know is Securitization and Reconstruction of Financial Assets and
Enforcement of Security Interest Act, 2002. Banks utilize this act as an effective tool for bad loans (NPA)
recovery. It is possible where non-performing assets are backed by securities charged to the Bank by way of
hypothecation or mortgage or assignment.

www.ibpsguise,in | mock.ibpsguide.com | www.sscexamguide.com


Copyright © IBPS Guide 113
www.ibpsguide.com
 Upon loan default, banks can seize the securities (except agricultural land) without intervention of the
court.
 SARFAESI is effective only for secured loans where bank can enforce the underlying security e.g.
hypothecation, pledge and mortgages. In such cases, court intervention is not necessary, unless the
security is invalid or fraudulent. However, if the asset in question is an unsecured asset, the bank would
have to move the court to file civil case against the defaulters.

Self Help Group:

Self help group is a small volunteer association of poor people preferably from the same socio Economic
background. They come together for the purpose of solving their common problems through self help and
mutual help. The self help group promotes small saving among the members. The savings are kept with the
Bank. This common fund is in the name of SHG. Usually the number of members in one SHG doesn‘t exceed
20. The NABARD and NGOs the promoters of this group.

SLR (Statutory Liquidity Ratio):

It is amount that a commercial bank should have before giving credits to its customers which should be either
in the form of gold, money or bonds.

Special Economic Zone:

SEZ implies Special Economic Zone is one of the pieces of government‘s strategies in India. An extraordinary
Economic Zone is a land districts that financial law which is more liberal than the typical monetary laws in the
nation.

The essential aphorism behind this is to increment the investment from foreign, improvement of a foundation,
openings for work and increment the income level of the general population.

Special Drawing Rights (SDR):

It is a reserve asset (Paper Gold) created within the framework of the International Monetary Fund in an
attempt to increase international liquidity

Stock Market:

A stock market is a private or public market for trading of company, stock and derivatives of company stock at
an agreed price. Both of these are securities listed on stock exchange as well as those only traded privately.

Stag:

www.ibpsguise,in | mock.ibpsguide.com | www.sscexamguide.com


Copyright © IBPS Guide 114
www.ibpsguide.com
A Stag is an investor or speculator who subscribes to a new issue with the intention of selling them soon after
allotment to realize for quick profit.

Teller:

He/she is a staff member of the bank who cashes cheques, accepts deposits and perform different banking
services for the general mass.

Teaser Loan:

If a bank offers a slightly lower rate in the initial years and higher rate in later years, it is called a teaser loan.

Universal Banking:

When financial institutions and banks undertake activities related to banking like investment, issue of debit and
credit card etc then it is known as universal banking.

Virtual Banking:

Internet banking is sometimes known as virtual banking. It is called so because it has no bricks and boundaries.
It is controlled by the World Wide Web.

Vostro Account :

It is the account which is held by a foreign bank with a local bank, so if bank of America maintains an account
with state bank of India it will be a vostro account for state bank of India.

Wholesale Banking:

It is similar to retail banking with a slight difference that it mainly focuses on the financial needs of the
institutional clients and the industry.

Zero Coupon Bond:

It is a bond that is sold at good discount as it has no coupon.

www.ibpsguise,in | mock.ibpsguide.com | www.sscexamguide.com


Copyright © IBPS Guide 115
www.ibpsguide.com

BANK HEADQUARTERS AND TAGLINE


Headquarters of Public Sector Banks:

Head
No Bank Name Chairman Tagline
Quarters

1 Allahabad Bank Kolkata Mr. Rakesh Sethi A tradition of trust

2 Andhra Bank Hyderabad Mr. Suresh N Patel Where India Banks

India‘s International
3 Bank of Baroda Vadodara Mr. P. S. Jayakumar
Bank

Relationships beyond
4 Bank of India Mumbai Mr. Melwyn O Rego
Banking

Mr. Ravindra Prabhakar


5 Bank of Maharashtra Pune One Family One Bank
Marathe

Together we Can, It‘s


6 Canara Bank Bangalore Mr. Rakesh Sharma easy to change for
those who you love

Central to you since


7 Central Bank of India Mumbai Mr. Rajeev Rishi 1911, Build A Better
Life Around Us

A Premier Public
8 Corporation Bank Mangalore Mr. Jai Kumar Garg Sector Bank,
Prosperity for all

9 Dena Bank Mumbai Mr. Ashwani Kumar Trusted Family Bank

Your Tech-friendly
10 Indian Bank Chennai Mr. Kishor Kharat
bank

Mr. Good people to grow


11 Indian Overseas Bank Chennai
R.Subramaniakumar with

Oriental Bank of Where every individual


12 Gurugram Mr. Animesh Chauhan
Commerce is committed

www.ibpsguise,in | mock.ibpsguide.com | www.sscexamguide.com


Copyright © IBPS Guide 116
www.ibpsguide.com
The Name you can
13 Punjab National Bank New Delhi Sunil Mehta
Bank Upon

Where series is a way


14 Punjab & Sind Bank New Delhi Mr. Jatinder Bir Singh
of life

15 Syndicate Bank Manipal Mr. Arun Shrivastava Faithful. Friendly

Mr. Ravi Krishan


16 UCO Bank Kolkata Honors Your Trust
Takkar

Good people to bank


17 Union Bank of India Mumbai Mr. Arun Tiwari
with

The Bank that begins


18 United Bank of India Kolkata Mr. Pawan Kumar Bajaj
with ―U‖

Mr. Kishore Kumar A friend You can Bank


19 Vijaya Bank Bangalore
Sansi Upon

Banking For All, ―Aao


20 IDBI Bank Ltd Mumbai Mr. Mahesh Kumar Jain
Sochein Bada‖

Headquarters of Private Sector Banks:

No Bank Name Chairman Head Quarter Tagline

Everything is the same except


1 AXIS Bank Shika Sharma Mumbai
the name

2 Citi Union Bank Dr. N. Kamakodi Tamilnadu Trust and Excellene since 1904

3 Dhanlaxmi Bank G. Sreeram Tamilnadu Tann. Mann. Dhan

4 ICICI Bank Ltd Chanda Kochhar Mumbai Hum Hain na!!, Khayaal Apka

5 IndusInd Bank Romesh Sobti Mumbai We make money simple

www.ibpsguise,in | mock.ibpsguide.com | www.sscexamguide.com


Copyright © IBPS Guide 117
www.ibpsguide.com

6 Karnataka Bank P Jayaram Bhat Mangalore Your family bank across India

Kotak Mahindra
7 Uday Kotak Mumbai Let‘s make money simple
Bank

Catholic Syrian Anand


8 Thrissur Support all the way.
Bank Krishnamurthy

9 Federal Bank Shyam Srinivasan Kerala Your perfect banking partner

10 HDFC Bank Aditya puri Mumbai We understand your world.

11 Karur Vysya Bank K. Venkataraman Tamilnadu Smart way to bank

Lakshmi Vilas Parthasarathi The Changing Face of


12 Tamilnadu
Bank Mukherjee prosperity

13 Yes Bank Rana Kapoor Mumbai Experience our expertise

Experience next generation


14 South Indian Bank V.G Mathew Thrissur
banking

Tamilnadu H. S.
15 Tuticorin Be a step ahead of life
Mercantile Bank UpendraKamath

State Bank Group:

The Nation banks


on us; Pure
State Bank of Mrs. Arundhati
1 Mumbai Banking Nothing
India Bhattacharya
Else; With you all
the way

State Bank of Mrs. Arundhati The Bank with a


2 Rajasthan
Bikaner & Jaipur Bhattacharya vision.

State Bank of Mrs. Arundhati You can always bank


3 Hyderabad
Hyderabad Bhattacharya on us

4 State Bank of Bangalore Mrs. Arundhati Working for a better

www.ibpsguise,in | mock.ibpsguide.com | www.sscexamguide.com


Copyright © IBPS Guide 118
www.ibpsguide.com
Mysore Bhattacharya tomorrow

State Bank of Mrs. Arundhati Blending Modernity


5 Punjab
Patiala Bhattacharya with Tradition

State Bank of Mrs. Arundhati A Long Tradition of


6 Thiruvananthapuram
Travancore Bhattacharya Trus

Foreign Banks in India:

No Bank Country Tagline

1 AB Bank Ltd. Bangladesh

2 ABN AMRO Bank Netherland Making More Possible

Abu Dhabi Commercial


3 UAE Long Life Ambition
Bank

American Express
4 USA Do more
Banking Corporation

Australia and New


Australia
5 Zealand Banking Group We Live in Your World
Ltd.

6 Bank of America USA Bank of Opportunity

Bank of Bahrain &


7 Bahrain
Kuwait BSC

8 Bank of Ceylon Sri Lanka The Bank you can trust

9 Bank of Nova Scotia Canada

Bank International
10 Indonesia
Indonesia

www.ibpsguise,in | mock.ibpsguide.com | www.sscexamguide.com


Copyright © IBPS Guide 119
www.ibpsguide.com

11 Berclays Bank UK Fluent In Finance

The Bank For A Changing


12 BNP Paribas Bank France
World

China Trust Commercial


13 Taiwan We are Family
Bank

14 Citi Bank USA The Citi never sleeps.

Commonwealth Bank of
15 Australia
Australia

Credit Agricole
16 Corporate & Investment France Common Sense has a future
Bank

It‘s Time for an Expert;


17 Credit Suisse A.G Switzerland Whatever makes you happy;
Credit Suisse 360 Finance

18 CTBC Bank Co. Ltd. Taiwan

19 DBS Bank Singapore Living, Breathing Asia

20 Deustsche Bank Germany A Passion to Perform

21 Doha Bank Qatar

22 FirstRand Bank Ltd South Africa

23 HSBC Ltd Hong Kong The World's Local Bank

Industrial &
24 Commercial Bank of China Moving Forward
China Ltd.

25 Industrial Bank of Korea South Korea

www.ibpsguise,in | mock.ibpsguide.com | www.sscexamguide.com


Copyright © IBPS Guide 120
www.ibpsguide.com

J.P. Morgan Chase Bank The Right Relationship is


26 USA
N.A Everything

27 JSC VTB Bank Russia World Without Barriers

28 KBC Bank NV Belgium

29 Korea Exchange Bank South Korea

Krung Thai Bank Public


30 Thailand
Co. Ltd.

We Do Banking Services with


31 Mashreq Bank PSC UAE
High Focus on Innovation

32 Mizuho Bank Ltd. Japan Channel to Discovery

A Little Word for a big life;


33 National Australia Bank Australia
More give, less take

National Bank of Abu


34 UAE
Dhabi PJSC

35 Rabobank International Netherlands A Bank with Ideas

36 Royal Bank of Scotland UK Make It Happen

37 Sberbank Russia By Your Side; Always Nearby

38 Scotia Bank Canada You‘re Richer Than You Think

39 Shinhan Bank South Korea The Bank Of Bride

40 Societe Gednerale France Building Team Sprit Together

Your Trusted Partner in


41 Sonali Bank Ltd. Bangladesh
Innovative Banking

www.ibpsguise,in | mock.ibpsguide.com | www.sscexamguide.com


Copyright © IBPS Guide 121
www.ibpsguide.com

Standard Chartered
42 UK Your Right Partner
Bank

43 State Bank of Mauritius Mauritius

Sumitomo Mitsui
44 Japan One Bank, One SMBC
Banking Corporation

The Bank of Tokyo-


45 Japan
Mitsubishi UFJ, Ltd

The Royal Bank of


46 Netherlands
Scotland N.V.

47 UBS AG Switzerland We Will Not Rest

United Overseas Bank


48 Singapore United for Growth
Ltd

Westpac Banking
49 Australia Help is What we do
Corporation

50 Woori Bank South Korea Home to the Intelligent Investors

Small Finance Bank Headquarters:

No Bank Headquarters

1 AU Small Finance Bank Jaipur

2 Capital Local Area Bank Jalandhar

3 Fincare Small Finance Bank Ahmadabad

4 Equitas Small Finance Bank Chennai

5 ESAF Small Finance Bank Thrissur

6 Jana Small Finance Bank Bangalore

www.ibpsguise,in | mock.ibpsguide.com | www.sscexamguide.com


Copyright © IBPS Guide 122
www.ibpsguide.com

7 North East Small Finance Bank Guwahati

8 Suryoday Small Finance Bank Navi Mumbai

9 Ujjivan Small Finance Bank Bangalore

10 Utkarsh Small Finance Bank Varanasi

Payments Bank Headquarters:

Bank
No Head Quarters

1 Airtel Payments Bank Rajasthan

2 Paytm Payments Bank Noida

3 India Post Payments Bank New Delhi

BANKING RELATED ACTS

1. Negotiable Instrument Act–1881


2. The Bankers Books Evidence Act–1891
3. The Reserve Bank of India Act–1934
4. The Industrial Finance Corporation of India Act–1948
5. The Banking Companies (Legal Practitioner Clients‘ Accounts) Act–1949
6. The Industrial Disputes (Banking and Insurance Companies) Act–1949
7. The Banking Regulation(Companies) Rules–1949
8. The Banking Regulation Act–1949
9. The State Financial Corporations Act–1951
10. The Reserve Bank of India (Amendment and Misc. Provisions) Act–1953
11. The Industrial Disputes (Banking Companies) Decision Act–1955
12. The State Bank of India Act–1955
13. The State Bank of India (Subsidiary Banks) Act-1959

www.ibpsguise,in | mock.ibpsguide.com | www.sscexamguide.com


Copyright © IBPS Guide 123
www.ibpsguide.com
14. The Subsidiary Banks General Regulation–1959
15. The Deposit Insurance and Credit Guarantee Corporation Act–1961(DICGC)
16. The Banking Companies (Acquisition and Transfer of Undertakings) Act–1970
17. The Regional Rural Banks Act–1976
18. The Banking Companies (Acquisition and Transfer of Undertakings) Act–1980
19. The Export-Import Bank of India Act–1981
20. The National Bank for Agriculture and Rural Development Act–1981
21. Chit Fund Act–1982
22. Sick Industrial Companies (Special Provisions)Act–1985
23. The National Housing Bank Act–1987
24. SIDBI Act–1989
25. The Special Court (trial of Offences relating to Transactions in Securities) Act–1992
26. The Industrial Finance Corporation (Transfer of Undertakings and Repeal) Act–1993
27. Industrial Reconstruction Bank (Transfer of Undertaking & Appeal) Act–1997
28. The Securitization and Reconstruction of Financial Assets and Enforcement of Security Interest Act–
(SARFASI-2002)
29. Industrial Development Bank (Transfer of Undertaking & Repeal) Act–2003
30. Credit Information Companies (Rules & Regulation) Act–2005
31. The Industrial Finance Corporation of India Act–1948
32. The Banking Companies (Legal Practitioner Clients‘ Accounts) Act–1949
33. The Industrial Disputes (Banking and Insurance Companies) Act–1949
34. The State Financial Corporations Act–1951
35. The Reserve Bank of India (Amendment and Misc. Provisions) Act–1953
36. The Industrial Disputes (Banking Companies) Decision Act–1955
37. The State Bank of India Act–1955
38. The State Bank of India (Subsidiary Banks) Act-1959
39. The Subsidiary Banks General Regulation–1959
40. The Deposit Insurance and Credit Guarantee Corporation Act–1961
41. The National Bank for Agriculture and Rural Development Act–1981
42. Chit Fund Act–1982
43. Shipping Development Fund Committee (Abolition)Act–1985
44. Sick Industrial Companies (Special Provisions)Act–1985
45. The National Housing Bank Act–1987
46. The Special Court (trial of Offences relating to Transactions in Securities) Act–1992

www.ibpsguise,in | mock.ibpsguide.com | www.sscexamguide.com


Copyright © IBPS Guide 124
www.ibpsguide.com
47. The Industrial Finance Corporation (Transfer of Undertakings and Repeal) Act–1993
48. Industrial Reconstruction Bank (Transfer of Undertaking & Appeal) Act–1997
49. SIDBI General Regulations, 1990
50. Banking Regulation (Companies) Rules 1949
51. The Nationalized Banks (Management and Misc. Provisions)Scheme,1970
52. NABARD General Regulations 1982
53. Banking Companies (Period of Preservation of Records) Rules, 1985
54. Banking Companies (Regulation)Rules,1985
55. NABARD Bonds Regulations — 1988
56. The Banking Ombudsman Scheme, 2006
57. Factoring Act Rules, 2011
58. SARFAESI (Central registry) Rules,2011
59. Banker‘s Books Evidence Act, 1891
60. Banking Regulation Act, 1949
61. Banking Companies (Legal Practitioners‘ Clients‘ Account) Act, 1949
62. Banking Regulation (Companies) Rules, 1949
63. Banking Companies (Acquisition and Transfer of Undertaking) Act, 1969
64. Debts Recovery Appellate Tribunal (Procedure) Rules,1994
65. Foreign Contribution (Regulation) Act, 1976
66. Foreign Exchange Management Act, 1999
67. Indian Partnership Act, 1932
68. Indian Stamp Act, 1899
69. Indian Trusts Act, 1882
70. Limitation Act, 1963
71. Recovery of Debts due to Banks and Financial Institutions Act,1993
72. Reserve Bank of India Act — 1934
73. Negotiable Instruments Act, 1881
 Section 4 – Promissory note
 Section 5 – Bill of exchange
 Section 6 – Cheque
 Section 13 – Negotiable Instruments
 Section 123 – Cheque Crossed Generally
 Section 124, 126 – Cheque crossed specially
 Section 130 – Cheque bearing Not Negotiable

www.ibpsguise,in | mock.ibpsguide.com | www.sscexamguide.com


Copyright © IBPS Guide 125
www.ibpsguide.com
 Section 118 – Presumptions as to Negotiable Instruments
74. Reserve Bank of India Act, 1934
 Section 17 – Defines Business of RBI
 Section 18 – Deals with Emergency loans to Banks
 Section 22 – only RBI has the exclusive rights to issue currency notes in India.
 Section 24 – maximum denomination a note can be Rs. 10,000.
 Section 26 – Describes the legal tender character of Indian bank notes.
 Section 28 – Allows the RBI to form rules regarding the exchange of damaged and
imperfect notes.
 Section 31 – In India only the RBI or the central government can issue and accept
promissory notes that are payable on demand.
 Section 42(1) – Every scheduled bank must have an average daily balance with the RBI.

COMMITTEES

No Committee Name Focus Area

1 A. C. Shah Committee NBFC

2 Abid Hussain Committee Development of Capital markets

3 A Ghosh Committee Final Accounts


Modalities Of Implementation Of New 20 Point
4 A Ghosh Committee
Programme
5 A Ghosh Committee: Frauds & Malpractices In Banks
Coordination Between Term Lending Institutions
6 AK Bhuchar Committee
And Commercial Banks
7 Adhyarjuna Committee Changes in NI Act and Stamp Act

www.ibpsguise,in | mock.ibpsguide.com | www.sscexamguide.com


Copyright © IBPS Guide 126
www.ibpsguide.com

8 B. Eradi Committee Insolvency and Wind up laws


Institutional Credit For Agricultural & Rural
9 B Sivaraman Committee
Development
10 B. Venkatappaiah Committee All India Rural Credit Review

11 B.D. Shah Committee Stock Lending Scheme

12 BD Thakar Committee Job Criteria in bank loans (Approach)

13 Bhagwati Committee Unemployment

14 Bhagwati Committee Public Welfare

15 Bhave Committee Share Transfer Reforms


Coordination between Commercial Banks and
16 Bhide Committee
SFC‘s
17 Bhootlingam Committee Wage, Income and Prices

18 C. Rao Committee Agricultural Policy

19 C.E. Kamath Committee Multi Agency approach in Agricultural Finance

20 Chatalier Committee Finance to Small Scale Industry

21 Chesi Committee Direct Taxes

22 Cook Committee Capital Adequacy of Banks


Review Progress And Recommend Improvement
23 D R Mehta Committee
Measures Of IRDP
24 Damle Committee MICR

25 Dandekar Committee Regional Imbalances

26 Dantwala Committee Estimation of Employments

27 Dave Committee Mutual Funds (Functioning)

28 Dharia Committee Public Distribution System

29 D.R. Gadgil Committee Agricultural Finance

30 Dutta Committee Industrial Licensing

www.ibpsguise,in | mock.ibpsguide.com | www.sscexamguide.com


Copyright © IBPS Guide 127
www.ibpsguide.com

31 G. Sundaram Committee Export Credit

32 Gadgil Committee (1969) Lead Banking System

33 Godwala Committee Rural Finance

34 Goiporia Committee Customer Service in Banks

35 G.S. Dahotre Committee Credit Requirements of Leasing Industry

36 G.S. Patel Committee Carry Forward System on Stock Exchange

37 Hathi Committee Soiled Banknotes

38 Hazari Committee (1967) Industrial Policy

39 I.T. Vaz Committee Working Capital Finance in Banks

40 J. Reddy Committee Reforms in Insurance Sector

41 James Raj Committee Functioning of Public Sector Banks


Securities Transactions Of Banks & Financial
42 Jankiramanan Committee
Institutions
43 J.V. Shetty Committee Consortium Advances

44 K. Madhav Das Committee Urban Cooperative Banks

45 Kalyansundaram Committee Introduction of Factoring Services in India

46 Kamath Committee Education Loan Scheme

47 Karve Committee Small Scale Industry

48 K.B. Chore Committee To review the Symbol of Cash Credit Q


Role Of Banks In Priority Sector And 20 Point
49 KS Krishnaswamy Committee
Economic Programme
50 Khanna Committee Non Performing Assets

51 Khusrau Committee Agricultural Credit

52 L.K. Jha Committee Indirect Taxes

53 L.C. Gupta Committee Financial Derivatives

www.ibpsguise,in | mock.ibpsguide.com | www.sscexamguide.com


Copyright © IBPS Guide 128
www.ibpsguide.com

54 Mahadevan Committee Single Window System

55 Mahalanobis Committee Income Distribution

56 Marathe Committee Licensing of New Banks

57 M.L. Dantwala Committee Regional Rural Banks

58 Mrs. K,S, Shere Committee Electronic Fund Transfer

59 Nariman Committee Branch Expansion Programme


Improved Procedures For Transactions In PSU
60 Nadkarni Committee
Bonds And Units
Committee on comprehensive financial services for
61 Nachiket Mor
small businesses and low-income households
62 Narsimhan Committee Financial System

63 Omkar Goswami Committee Industrial Sickness and Corporate Restructuring

64 P.R. Nayak Committee Institutional Credit to SSI Sector

65 P. Selvam Committee Non Performing Assets of Banks


Productivity, Operational Efficiency and
66 P.C. Luther Committee
Profitability of Banks
67 P.D. Ojha Committee Service Area Approach

68 Pillai Committee Pay Scales of Bank Officers

69 P.L. Tandon Committee Export Strategy


Develop appropriate Supervisory Framework for
70 P.R. Khanna Committee
NBFC
71 Purshottam Das Committee Agricultural Finance and Cooperative Societies

72 R. Jilani Banks Inspection System of Banks

73 R.S. Saria Committee Agricultural Finance and Cooperative Societies

74 Raghavan Committee Competition Law

75 Raja Chelliah Committee Tax Reforms

76 Rajamannar Committee Centre-State Fiscal Relationships

www.ibpsguise,in | mock.ibpsguide.com | www.sscexamguide.com


Copyright © IBPS Guide 129
www.ibpsguide.com

Changes in Banking Laws, Bouncing of Cheques


77 Rajamannar Committee
etc.
78 Rakesh Mohan Committee Petro Chemical Sector
Ram Niwas Mirdha Committee
79 Securities Scam
(JPC)
80 Rangrajan Committee Computerization of Banking Industry

81 Rangrajan Committee Public Sector Disinvestment

82 Rashid Jilani Committee Cash Credit System

83 Ray Committee Industrial Sickness

84 R.G. Saraiya Committee (1972) Banking Commission

85 R.H. Khan Committee Harmonization of Banks and Ssis

86 R.K. Hajara Committee Differential Interest Rates Scheme


Enactment Having A Bearing On Agro Landings By
87 R.K. Talwar Committee
Commercial Banks
88 R.K. Talwar Committee Customer Service

89 R.N. Malhotra Committee Reforms in Insurance Sector

90 R.N. Mirdha Committee Cooperative Societies

91 R.V. Gupta Committee Agricultural Credit Delivery

92 S. Padmanabhan Committee Onsite supervision Function of Banks

93 S. Padmanabhan Committee Inspection of Banks (By RBI)

94 Samal Committee Rural Credit

95 S.C. Choksi Committee Direct Tax Law

96 Shankar Lal Gauri Committee Agricultural Marketing

97 S.K. Kalia Committee Role of NGO and SHG in Credit

98 S.L. Kapoor Committee Institutional Credit to SSI


Foreign Exchange Markets in NRI investment in
99 Sodhani Committee
India

www.ibpsguise,in | mock.ibpsguide.com | www.sscexamguide.com


Copyright © IBPS Guide 130
www.ibpsguide.com

100 S.S. Nadkarni Committee Trading in Public Sector Banks

101 SS Kohli Committee Willful Defaulters

102 SS Kohli Committee Rehabilitation Of Sick Industrial Units

103 SS Kohli Committee Rationalization Of Staff Strength In Banks

104 S.S. Tarapore Committee Capital Account Convertibility


Sukhmoy Chakravarty
105 To review the working of Monetary System
Committee
106 Tambe Committee Term Loans to SSI

107 Tandon Committee Follow up of Bank Credit

108 Tandon Committee Industrial Sickness

109 Thakkar Committee Credit Schemes to Self employed

110 Thingalaya Committee Restructuring of RRB

111 Tiwari Committee Rehabilitation of sick Industrial undertakings

112 U.K. Sharma Committee Lead Bank Scheme (Review)

113 Usha Thorat Panel Financial Inclusion

114 Vaghul Committee Mutul Fund Scheme

115 Varshney Committee Revised methods for Loans (> 2 lakhs)

116 Venketaiya Committee Review of Rural Financing System

117 Vipin Malik Committee Consolidated Accounting by Banks

118 Vyas Committee Rural Credit

119 Wanchoo Committee Direct Taxes

120 W.S. Saraf Committee Technology Issues in Banking Industry

121 Y.H. Malegam Committee Disclosure norms for Public Issues

122 Y.V. Reddy Committee Reforms in Small Savings

www.ibpsguise,in | mock.ibpsguide.com | www.sscexamguide.com


Copyright © IBPS Guide 131
www.ibpsguide.com

FIRST AMONG ALL BANKS

NO IMPORTANT FACTS BANK

1 First bank established in India Bank of Hindustan

2 First bank to introduce savings account in India Presidency Bank

3 First bank to introduce cheque system in India Bengal Bank

4 First bank to introduce internet banking ICICI Bank

5 First bank to introduce mutual fund State Bank of India

6 First bank to introduce credit card in India Central Bank of India

7 First bank to introduce ATM in India HSBC

8 First bank to provide Mobile ATM ICICI Bank

9 First bank to open branch outside India: Bank of India

10 Largest commercial bank in India State Bank of India

11 India‘s first financial Archive set up at Kolkata

12 Largest public sector bank in India SBI

13 Largest private sector bank in India ICICI Bank

14 Largest foreign bank in India Standard Chartered Bank

15 Bank having maximum overseas branches Bank of Baroda

16 First Governor of RBI Sir Osborne Smith

17 First Indian Governor of RBI C.D Deshmukh

18 First Joint Bank of India Allahabad Bank


First Indian Bank Started with Indian capital
19 Punjab National Bank
indigenous Bank
20 First Regional Rural Bank Prathama Grameen Bank

21 First Universal Bank in India ICICI Bank

www.ibpsguise,in | mock.ibpsguide.com | www.sscexamguide.com


Copyright © IBPS Guide 132
www.ibpsguide.com

First Bank to India listed in New York Stock


22 ICICI Bank
Exchange
23 First Bank in India launch talking ATM Union Bank of India
First Bank in India to launch its own Payment
24 State Bank of India
Aggregators
25 India‘s first all woman Bank Bhartiya Mahila Bank
First private bank to open dedicated branch for
26 RBL
startups
27 First Aadhaar based ATM launched by DCB Bank

28 First Bank to sell Indian gold coins Indian Overseas Bank

29 First Payment Bank Rajasthan m commerce

30 First Small Finance Bank Capital Small Finance Bank Limited


Contactless credit cards for small and midsized
31 ICICI
enterprises
First certified Green Bond at London Stock
32 Axis Bank
Exchange
33 First small and medium enterprises Bank HDFC

34 First Transaction on Block chain is executed by ICICI

35 First Issuer of Masala Bonds HDFC

IMPORTANT BANKING ABBREVIATIONS

NO ACRONYMS ABBREVIATIONS

1 ADB Asian Development Bank

2 ADF Automated Data Flow

3 ADR American Depository Receipt

4 AEPS Aadhaar Enabled Payment System

www.ibpsguise,in | mock.ibpsguide.com | www.sscexamguide.com


Copyright © IBPS Guide 133
www.ibpsguide.com

5 AIF Alternative Investment Fund

6 AIFI All India Financial Institutions

7 AIIB Asian Infrastructure Investment Bank

8 ALCO Asset/ Liability Committee

9 ALM Asset Liability Management

10 AMFI Association of Mutual Funds in India

11 AML Anti-Money Laundering

12 ANBC Adjusted Net Bank Credit

13 APBS Aadhaar Payment Bridge System

14 ARC Asset Reconstruction Companies

15 ARCIL Asset Reconstruction Companies India Limited

16 ARM Adjustable-Rate Mortgage

17 ASBA Applications Supported by Blocked Amount

18 ASSOCHAM Associated Chambers of Commerce and Industry of India

19 ATM Automated Teller Machine

20 BBB Bank Board Bureau

21 BBPS Bharat Bill Payment System

22 BCBS Basel Committee on Banking Supervision

23 BCSBI Banking Codes and Standards Board of India

24 BHIM App Bharat Interface for Money Application

25 BIRD Bankers Institute of Rural Development

26 BIS Bank for International Settlements

27 BOP Balance of Payments

www.ibpsguise,in | mock.ibpsguide.com | www.sscexamguide.com


Copyright © IBPS Guide 134
www.ibpsguide.com

28 BPLR Benchmark Prime Lending Rate

29 BRBNMPL Bharatiya Reserve Bank Note Mudran Private Limited

30 BSBDA Basic Savings Bank Deposit Account

31 BSE Bombay Stock Exchange

32 BSR Basic Statistical Returns

33 CAG Controller and Auditor General

34 CAR Capital Adequacy Ratio

35 CARE Credit Analysis and Research Limited

36 CASA Current and Savings Accounts

37 CBDT Central Board of Direct Taxes

38 CBLO Collateralized Borrowing and Lending Obligation

39 CBS CORE Banking Solution

40 CCEA Cabinet Committee on Economic Affairs

41 CCIL Clearing Corporation of India Limited

42 CD Certificate of Deposit

43 CDR Corporate Debt Restructuring

44 CDS Credit Default Swap

45 CECA Comprehensive Economic Cooperation Agreement

46 CEPA Comprehensive Economic Partnership Agreement

47 CIBIL Credit Information Bureau of India Limited

48 CII Confederation of Indian Industry

49 CMS Cash Management Services

50 CORE Centralized Online Real-time Exchange

www.ibpsguise,in | mock.ibpsguide.com | www.sscexamguide.com


Copyright © IBPS Guide 135
www.ibpsguide.com

51 CP Commercial Paper

52 CPFF Commercial Paper Funding Facility

53 CPI Consumer Price Index

54 CRAR Capital To Risk Weighted Asset Ratio

55 CRISIL Credit Rating Information Services Of India

56 CRM Customer Relationship Management

57 CRR Cash Reserve Ratio

58 CSP Customer Service Point

59 CSR corporate social responsibility

60 CTS Cheque Truncation System

61 CUB City Union Bank

62 CVA Credit valuation adjustment

63 DICGC Deposit Insurance and Credit Guarantee Corporation

64 DIPP Department of Industrial Policy and Promotion

65 DPG Deferred Payment Guarantee

66 DPN Demand Promissory Note

67 DRAT Debt Recovery Appellate Tribunal

68 DRI Differential Rate Of Interest

69 DRT Debts Recovery Tribunal

70 DSCR Debt Service Coverage Ratio

71 D-SIBs Domestic Systemically Important Banks

72 DTAA Double Taxation Avoidance Agreement

73 ECB External Commercial Borrowings

www.ibpsguise,in | mock.ibpsguide.com | www.sscexamguide.com


Copyright © IBPS Guide 136
www.ibpsguide.com

74 ECGC Export Credit Guarantee Corporation of India

75 ECS Electronic Clearing System

76 EDF Export Development Fund

77 EDI Electronic Data Interchange

78 EEFC Exchange Earners Foreign Currency Account

79 EFSF European Financial Stability Facility

80 EFTPOS Electronic Funds Transfer At Point Of Sale

81 EIB European Investment Bank

82 ELSS Equity Linked Saving Scheme

83 EMI Equated Monthly Installment

84 EPFO Employees‘ Provident Fund Organisation

85 EPOS Electronic Point Of Sale

86 ETF Exchange-traded fund

87 EXIM Export Import Bank of India

88 FATF Financial Action Task Force

89 FCA Financial Conduct Authority

90 FCCB Foreign Currency Convertible Bond

91 FCNR Foreign Currency Non-Repatriable account deposits

92 FDI Foreign Direct Investment

93 FEDAI Foreign Exchange Dealers Association Of India

94 FERA Foreign Exchange Regulation Act

95 FICCI Federation of Indian Chambers of Commerce and Industry

96 FII Foreign Institutional Investor

www.ibpsguise,in | mock.ibpsguide.com | www.sscexamguide.com


Copyright © IBPS Guide 137
www.ibpsguide.com

97 FIMMDA Fixed Income Money Markets and Derivatives Association of India

98 FINO Financial Inclusion Network Operations

99 FIPB Foreign Investment Promotion Board

100 FPI Foreign Portfolio Investment

101 FPO follow-on public offer

102 FRBMA Fiscal Responsibility and Budget Management Act

103 FSLRC Financial Sector Legislative Reforms Commission

104 FSRASC Financial Sector Regulatory Appointment Search Committee

105 FTA Free Trade Agreement

106 GAAR General Anti Avoidance Rule

107 GDP Gross Domestic Product

108 GDR Global Depository Receipt

109 GIRO Goverment Internal Revenue Order

110 GMS Gold Monetisation Scheme

111 G-SIBs Global Systemically Important Banks

112 GST Goods and Services Tax

113 GSTN Goods and Services Network

114 GVA Gross value added

115 HDFC Housing Development Finance Corporation

116 HNI High net worth individual

117 HUF Hindu Undivided Family

118 IADB Inter-American Development Bank

119 IAF India Aspiration Fund

www.ibpsguise,in | mock.ibpsguide.com | www.sscexamguide.com


Copyright © IBPS Guide 138
www.ibpsguide.com

120 IBA Indian Banks' Association

121 IBBI Insolvency and Bankruptcy Board of India

122 IBPS Institute of Banking Personnel Selection

123 IBRD International Bank for Reconstruction and Development

124 IBU IFSC Banking Unit

125 ICICI Industrial Credit and Investment Corporation of India

126 ICRA Investment Information and Credit Rating Agency of India Limited

127 IDB Islamic Development Bank

128 IDBI Industrial Development Bank of India

129 IDF Infrastructure Development Fund

130 IDR Indian Depository Receipts

131 IDRBT Institute for Development & Research in Banking Technology

132 IDS Income Declaration Scheme

133 IEPF Investors Education and Protection Fund

134 IFC International Finance Corporation

135 IGIDR Indira Gandhi Institute of Development Research

136 IGST Integrated Goods and Services Tax

137 IIB International Investment Bank

138 IIBF Indian Institute of Banking and Finance

139 IIBI Industrial Investment Bank of India

140 IMF International Monetary Fund

141 IMPS Immediate Payment Service

142 IMT Instant Money Transfer

www.ibpsguise,in | mock.ibpsguide.com | www.sscexamguide.com


Copyright © IBPS Guide 139
www.ibpsguide.com

143 INFINET Indian Financial Network

144 InvITs Infrastructure Investment Trusts

145 IPO Initial Public Offering

146 IPPB India Post Payments Bank

147 IRDA Insurance Regulatory and Development Authority of India

148 IRR Internal Rate Of Return

149 ISA Individual Savings Account

150 ITF in trust for

151 JLF Joint Lenders' Forum

152 KCC Kisan Credit Card

153 KGC Kisan Gold Card

154 KMB Kotak Mahindra Bank

155 KVB Karur Vysya Bank

156 KVIC Khadi and Village Industries Commission

157 KVP Kisan Vikas Patra

158 KYC Know Your Customer

159 LAB Local Area Bank

160 LAF Liquidity Adjustment Facility

161 LDB Land Development Bank

162 LERMS Liberalized Exchange Rate Management System

163 LIBOR London Inter-Bank Offered Rate

164 LIC Life Insurance Corporation Of India

165 MCLR Marginal Cost of Funds based Lending Rate

www.ibpsguise,in | mock.ibpsguide.com | www.sscexamguide.com


Copyright © IBPS Guide 140
www.ibpsguide.com

166 MDB Multilateral Development Banks

167 MDR Merchant Discount Rate

168 MFDF Micro Finance Development Fund

169 MFI Micro Finance Institutions

170 MIBID Mumbai Interbank Bid Rate

171 MIBOR Mumbai Inter Bank Offered Rate

172 MICR Magnetic Ink Character Recognition

173 MIS Management Information System

174 MMID Mobile Money Identifier

175 MMTC Metals and Minerals Trading Corporation of India

176 MOD Multi Option Deposit Account

177 MPC Monetary Policy Committee

178 MSF Marginal Standing Facility

179 MSMED Micro, Small and Medium Enterprises Development

180 MSS Market Stabilisation Scheme

181 MTN Medium Term Note

182 MUDRA Micro Units Development And Refinance Agency

183 NABARD National Bank For Agricultural And Rural Development

184 NACH National Automated Clearing House

185 NASDAQ National Association for Securities Dealers Automated Quotations

186 NAV Net Asset Value

187 NBFC Non-Banking Financial Company

188 NDS Negotiated Dealing System

www.ibpsguise,in | mock.ibpsguide.com | www.sscexamguide.com


Copyright © IBPS Guide 141
www.ibpsguide.com

189 NDTL Net Demand and Time Liabilities

190 NECS National Electronic Clearing Service

191 NEFT National Electronic Funds Transfer

192 NFC Near-field communication

193 NFS National Financial Switch

194 NHB National Housing Bank

195 NIBM National Institute of Bank Management

196 NPA Non-Performing Assets

197 NPCI National Payments Corporation Of India

198 NPS National Pension System

199 NPV Net Present Value

200 NRE Non Resident External Account

201 NRI Non Resident Indian

202 NRO Non-Resident Ordinary savings account

203 NSE National Stock Exchange

204 NSFDC National Scheduled Castes Finance And Development Corporation

205 NUUP National Unified USSD Platform

206 OECD Organisation For Economic Cooperation And Development

207 OLTAS Online Tax Accounting System

208 OMO Open Market Operations

209 ORM Operational Risk management

210 OTCEI Over the Counter Exchange Of India

211 OTP One-Time Password

www.ibpsguise,in | mock.ibpsguide.com | www.sscexamguide.com


Copyright © IBPS Guide 142
www.ibpsguide.com

212 OVD Officially Valid Documents

213 PACS Primary Agricultural Credit Society

214 PAN Permanent Account Number

215 PDO Public Debt Office

216 PFRDA Pension Fund Regulatory and Development Authority

217 PIN Personal Identification Number

218 PIS Portfolio Investment Scheme

219 P-Notes Participatory Notes

220 POA Power of Attorney

221 PoS Point of Sale

222 PPF Public Provident Fund

223 PPI Payment protection insurance

224 PPP Public Private Partnership/ Purchasing Power Parity

225 PRSF Partial Risk Sharing Facility

226 PSBs Public Sector Banks

227 PSL Priority Sector Lending

228 PSS Act Payment and Settlement Systems Act

229 QFI Qualified Foriegn Investors

230 QIB Qualified Institutional Buyer

231 QIP Qualified Institutional Placement

232 RBI Reserve Bank of India

233 RBS Royal Bank of Scotland

234 RDBMS Relational Database Management System

www.ibpsguise,in | mock.ibpsguide.com | www.sscexamguide.com


Copyright © IBPS Guide 143
www.ibpsguide.com

235 RDDBFI Recovery of Debts Due to Banks and Financial Institutions

236 REC Rural Electrification Corporation

237 REITs Real Estate Investment Trusts

238 RFC Resident Foreign Currency

239 RFID Radio-Frequency Identification

240 RIDF Rural Infrastructure Development Fund

241 RoA Return On Assets

242 RoE Return On Equity

243 RRB Regional Rural Bank

244 RTGS Real Time Gross Settlement

245 RWA Risk Weighted Assets

246 S4A Scheme for Sustainable Structuring of Stressed Assets

Securitization and Reconstruction of Financial Assets and Enforcement of


247 SARFAESI
Security Interest

248 SBI State Bank of India

249 SBNs Specified Bank Notes

250 SCB Scheduled Commercial Bank

251 SCC Selective Credit Control

252 SDR Special Drawing Rights

253 SEBI Securities and Exchange Board of India

254 SENSEX Sensitive Index

255 SEPA Single Euro Payments Area

256 SEZ Special Economic Zone

257 SFMS Structured Financial Messaging System

www.ibpsguise,in | mock.ibpsguide.com | www.sscexamguide.com


Copyright © IBPS Guide 144
www.ibpsguide.com

258 SGB Sovereign Gold Bond

259 SHG Self Help Group

260 SIDBI Small Industries Development Bank Of India

261 SIFI Systematically Important Financial Institution

262 SIP Systematic Investment Plan

263 SIPS Systemically Important Payment Systems

264 SLR Statutory Reserve Ratio

265 SMERA SME Rating Agency of India Limited

266 SMILE SIDBI Make in India Loan for Enterprises

267 SPMCIL Security Printing and Minting Corporation of India Limited

268 SPNS Shared Payment Network System

269 SSSBE Small-scale Service and Business Enterprises

270 STP Straight-through Processing

271 SWIFT Society For World Wide Inter Bank Financial Telecommunication

272 TARC Tax Administration Reform Commission

273 TCS Tax Collected at Source

274 TDR Term Deposit Receipt

275 TDS Tax Deducted at Source

276 TIEA Tax Information Exchange Agreement

277 TIN Taxpayer Identification Number

278 UEBA Universal Electronic Bank Account

279 UIDAI Unique Identification Authority of India

280 UPI Unified Payment Interface

www.ibpsguise,in | mock.ibpsguide.com | www.sscexamguide.com


Copyright © IBPS Guide 145
www.ibpsguide.com

281 USB Ultra Small Branch

282 USSD Unstructured Supplementary Service Data

283 UTI Unit Trust of India

284 VDBS Vertically Differentiated Banking System

285 VPA Virtual Payment Address

286 WBCIS Weather Based Crop Insurance Scheme

287 WCTL Working Capital Term Loan

288 WLAs White Label ATMs

289 WMA Ways and Means Advances

290 WPI Wholesale Price Index

291 YTM Yield to Maturity

www.ibpsguise,in | mock.ibpsguide.com | www.sscexamguide.com


Copyright © IBPS Guide 146

You might also like